Location via proxy:   [ UP ]  
[Report a bug]   [Manage cookies]                

BAJA MCQs

Download as pdf or txt
Download as pdf or txt
You are on page 1of 1201

CHAPTER 1

A SHORT HISTORY OF SURGERY


1. The oldest “book” in the world is:

A. The Old Testament


B. Plato’s Republic
C. Edwin Smith’s papyrus
D. Mishnah
E. Sushrata Sumhita
2. The Edwin Smith Papyrus:

A. Was composed about 1600 B.C.


B. Was an incomplete copy of a scroll written about 3000 B.C.
C. Described the management of wounds, fractures and dislocations
D. Was written in Babylon
E. Was found in Greece
3. The Greek Physicians during the Hippocratic era:

A. Were the first to base medical practice on experience of previous


cases, clinical history and physical examination
B. Had the proctoscope
C. Could do anal fistulectomy
D. Could do tonsillectomy
E. Could extract cataract
4. The first medical school was established in:

A. Salerno
B. Athens
C. Bologna
D. Alexandria
E. Montpellier
5. Herophilus was the:

A. First to dissect the human body and give the duodenum its name
around the 4th century B.C.
B. Founder of the science of Physiology
C. First to count the pulse and analyse its rate and rhythm
D. First to describe the blood pressure
E. First to count the respiratory rate and show its significance


 
6. The four cardinal signs of inflammation were first described by:

A. Galen
B. Hippocrates
C. Hunter
D. Erasistratus
E. Celsius
7. In ancient India (first century):

A. Surgery was practiced by the high caste


B. Plastic surgery was well-developed
C. Alya (surgery) was considered the most important branch of the art of healing
D. Steel surgical instruments were used
E. Antiseptics were first introduced
8. Concerning Galen:

A. He was born in Pergamon in Asia Minor and lived from 199 to 129 B.C
B. He studied in Alexandria
C. Most of his views on physiology were correct
D. His concepts formed the theoretical basis of medicine and surgery for fifteen
centuries
E. Formulated the basis of the germ theory
9. In the early Christian era:

A. Dissection of the body was encouraged


B. Disease was considered a visitation of evil and malevolent spirits
C. Disease was curable only by prayer
D. St. Agatha was the saint for diseases of the breast
E. Priests became physicians and were forbidden to practice surgery
10. The term “Doctor” for medical practitioners was:

A. Introduced in the 12th century


B. Introduced in the medical school in salerno
C. Used by those who successfully completed a 5-year course in medicine
D. Used by those who successfully completed a 3-year course in surgery
E. Used by those who successfully completed a year course in obstetrics and
gynaecology


 
11. “No man can be a good physician who knows no surgery and conversely
on one can be a good surgeon without knowledge of medicine”. This was
said by:

A. Hugh of Lucca
B. Lanfranc of Milan
C. Theodoric of Cervia
D. Pope Innocent VI
E. Henri de Mondeville
12. Andreas Vesalius:

A. Was born in Paris


B. Was Professor at Padua
C. Wrote “de Humani Corpori Fabrica” in 1543 at the age of 29
D. Was the father of modern anatomy
E. Challenged the views of Galen
13. “de Humani Corpori Fabrica”

A. Was a book on Physiology


B. Was written by Galen
C. Was beautifully illustrated
D. Contributed to the progress of surgery
E. Was first published in Greek
14. The Royal College of Surgeons of England was formed in:

A. 1725
B. 1801
C. 1845
D. 1885
E. 1902
15. The Oldest Surgical College is probably:

A. The Royal College of Surgeons of Edinburgh


B. The Faculty of Physicians and Surgeons of Glasgow
C. The Royal College of Surgeons of England
D. The Royal College of Surgeons of Ireland
E. The German Surgical College


 
16. Surgery was made a science and not just an art by:

A. William Cheselden
B. John Abernethy
C. Theodor Billroth
D. John Hunter
E. Lord Lister
17. John Hunter:

A. Lived from 1728 to 1793


B. Studied and worked in St. Thomas’ Hospital
C. Built a large museum of 13,000 collections of various animals
D. Studied inflammation, shock, wounds, aneurysm, and venereal
diseases
E. Made the greatest advancement of surgery ever made by an individual
18. Methods used to produce analgesia before the discovery of anaesthesia
included:

A. Compression of the carotid arteries


B. Mandrogora
C. Compression of nerves to limbs
D. Opium
E. Hypnotism
19. John Hunter:

A. Was the father of scientific surgery


B. Studied comparative anatomy
C. Was English by birth
D. Did appendicectomy
E. Performed the first repair of femoral aneurysm in Hunter’s canal
20. Hypnotism was first used in surgery in:
A. Calcutta
B. MGH, Boston
C. France
D. U.C.H., London
E. Royal Infirmary, Edinburgh


 
21. The first to demonstrate the analgesic effect of nitrous oxide was:
A. Horace Wells
B. Sir Humphrey Davy
C. Colton
D. Bigelow
E. C.T. Jackson
22. Anaesthesia was first successfully demonstrated in public:

A. By William Green Morton in 1846


B. At Peter Bent Brigham Hospital, Boston
C. With Dr. J.C. Warren as the surgeon
D. For removal of a lump of the neck
E. The patient was a child
23. Anaesthesia in childbirth was:

A. Introduced by James Young Simpson


B. Achieved at first with ether
C. Opposed by the church
D. Opposed by Queen Victoria
E. Entails high morbidity for mother and child
24. Endotracheal anaesthesia was introduced by:

A. Halstead
B. William Macewen
C. August Bier
D. Karl Kohler
E. John Liston
25. The germ theory of disease was established experimentally by:

A. Louis Pasteur
B. Joseph Lister
C. Jacob Henle
D. Koch
E. Summerlweiss


 
26. Joseph Litser:

A. Introduced asepsis
B. Used carbolic acid to dress wounds, sterilize instruments and spray
the air in the operating area
C. Was Professor of Surgery in University College Hospital, London
D. Influenced Volkmann, Lagenbeck and Thiersh
E. Is accredited with the germ theory of disease
27. Steam sterilization of surgical dressings, gowns and towels was
introduced by:

A. Volkmann
B. Ernst von Bergmann
C. Neuber
D. Lister
E. Lagenbeck
28. Theodor Billroth:

A. Did the first total laryngectomy for carcinoma


B. Perfected a new operation on dogs before trying it in humans
C. Introduced routine taking of temperature in surgical patients
D. Was a life-long-friend of Schubert
E. Was Professor of Surgery in Vienna
29. Appendicectomy:

A. Was a common operation at the beginning of the nineteenth century


B. Became more popular in Great Britain when Sir Frederic Treves performed
it on Edward VII
C. Was performed for the first successfully diagnosed acute appendicitis by
T.G. Morton of Philadelphia
D. Was described by Hippocrates
E. Was widely performed by the ancient Egyptians
30. Theodor Kocher:

               A. Developed the operation of thyroidectomy


B. Studied the physiology of parathyroid
C. Won the Nobel Prize in 1909
D. Was the greatest German Surgeon of all time
E. Lived from 1841 top 1917


 
31. The first successful salpingectomy for ruptured ectopic pregnancy
was done by:

A. Spencer Wells
B. Lawson Tait
C. Anton Wolfler
D. Thomas Morse
E. Summelweiss
32. Early gynaecological surgeons included:

A. Spencer Wells
B. Marion Sims
C. Sir Frederic Treves
D. Lawson Tait
E. Sir John Eric Erichson
33. Sir William Macewan:

A. Worked in London
B. Was the father of neurosurgery
C. Was the first to diagnose a cerebral abscess and successfully operate for
extradural haemorrhage
D. Introduced the endotracheal tube
E. Introduced use of bone wax to control haemorrhage in neurosurgery
34. Early neurosurgeons included:

A. Victor Horsley
B. E.C. Cuther
C. Harvey Cushing
D. Halstead
E. Robert Gross
35. Cardiac surgery was first pioneered in:

A. New York
B. Boston
C. Edinburgh
D. Vienna
E. Paris
36. Early cardiac surgeons included:

A. Robert Gross
B. Clarence Craaford
C. Henry Souttar

 
D. John Strieder
E. Alfred Blalock

37. The first successful cardiac operation was:

A. Ligation of patent ductus arteriosus


B. Blind mitral valvotomy for mitral stenosis
C. Repair of co-arctation of the aorta
D. Repair of Fallot’s tetralogy
E. Blind aortic valvotomy for aortic stenosis
38. Orthopaedic Surgery was:

A. The first sub-specialty to branch off from general surgery


B. Pioneered by Hugh Owen Thomas who practised in a London Hospital
C. Propagated by Sir Robert Jones considered the father of modern
orthopaedic surgery
D. Given a big boost during and after the first world war
E. Has always been exclusively dominated by trauma
39. Transplantation of:

A. Kidney was first done successfully by Lawler in Chicago in 1950


B. Kidney between identical twins was first done by Joseph Murray in Mayo
clinic in December 1954
C. Kidney has over 2-year survival in more than 60 per cent of patients
D. Heart was first done by C. Bernard on December 3rd, 1967 in Boston
E. Liver is now routine
40. The following was (were) involved in the development of blood
transfusion:

A. Jean Baptise Denis


B. James Blundell
C. Bernard Fantus
D. Karl Lansteinner
E. M. Hustin
41. Blood bank was first organized in:

A. Guy’s Hospital, London


B. John’s Hopkins, Baltimore
C. Cook County Hospital, Chicago
D. St. Mary’s London
E. State Hospital, Moscow


 
42. Discoverers of the blood groups included:

A. Arthus of Fribourg University


B. Karl Landsteinner
C. Mollison
D. DeCastello
E. Starli
43. The first laparoscopic procedure was:

A. Splenectomy
B. Done by Muhe
C. In 1985
D. Done in France
E. Done without anaesthesia
44. Laparoscopic surgery compared to open surgery generally:

A. Causes minimal tissue injury


B. Results in attenuated immuno suppression
C. Has low morbidity and mortality
D. May cause more severe pain
E. Is preferable in most procedures
45. Laparoscopic surgery can be used for:

A. Reflux oesophagitis
B. Adrenalectomy
C. Oesophagectomy
D. Partial gastrectomy
E. Pancreatectomy
46. In the future, it may be possible to do appendicectomy via:

A. The mouth
B. Vagina
C. The anus
D. A port opening in the anterior abdominal wall
E. Perineum
47. In telesurgery:

A. The surgeon does not have to be in the theatre


B. The operation is performed through a robot
C. Complex movements similar to those of the wrist and fingers are possible
but with tremors
D. The instruments are jointed to allow freedom of movement
E. The rates of conversion, complications and mortality are lower than in
laparoscopic surgery


 
CHAPTER 1
SHORT HISTORY OF SURGERY
ANSWER

1. C 34. A, C
2. A, B, C 35. B
3. A, B, C, D, E 36. A, B, C, D, E
4. A 37. B
5. A, C 38. A, C, D
6. E 39. A, C
7. B, C, D 40. A, B, C, D, E
8. B, D 41. C
9. B, C, D, E 42. B, D, E
10. A, B, C 43. B, C
11. B 44. A, B, C
12. B, C, D, E 45. A, B, C, D
13. C, D 46. A, B, C
14. C 47. A, B, D
15. A
16. D
17. A, C, D, E
18. A, B, C, D, E
19. A, B
20. D
21. B
22. A, C, D
23. A, C
24. B
25. A
26. B, D
27. B
28. A, B, C, E
29. B, C
30. A, C, E
31. B
32. A, B, D
10 
 
33. B, C, D

CHAPTER 1
A SHORT HISTORY OF SURGERY
 The oldest “book” in the world is:

 The Old Testament

 Plato’s Republic

 Edwin Smith’s papyrus

 Mishnah

 Sushrata Sumhita

 The Edwin Smith Papyrus:

 Was composed about 1600 B.C.

 Was an incomplete copy of a scroll written about 3000 B.C.

 Described the management of wounds, fractures and dislocations

 Was written in Babylon

 Was found in Greece

 The Greek Physicians during the Hippocratic era:

 Were the first to base medical practice on experience of previous

cases, clinical history and physical examination

 Had the proctoscope

 Could do anal fistulectomy

 Could do tonsillectomy

 Could extract cataract

11 
 
 The first medical school was established in:

 Salerno

 Athens

 Bologna

 Alexandria

 Montpellier

 Herophilus was the:

 First to dissect the human body and give the duodenum its name

around the 4th century B.C.

 Founder of the science of Physiology

 First to count the pulse and analyse its rate and rhythm

 First to describe the blood pressure

 First to count the respiratory rate and show its significance

 The four cardinal signs of inflammation were first described by:

 Galen

 Hippocrates

 Hunter

 Erasistratus

 Celsius

12 
 
 In ancient India (first century):

 Surgery was practiced by the high caste

 Plastic surgery was well-developed

 Alya (surgery) was considered the most important branch of the art of healing

 Steel surgical instruments were used

 Antiseptics were first introduced

 Concerning Galen:

 He was born in Pergamon in Asia Minor and lived from 199 to 129 B.C

 He studied in Alexandria

 Most of his views on physiology were correct

 His concepts formed the theoretical basis of medicine and surgery for fifteen
centuries

 Formulated the basis of the germ theory

 In the early Christian era:

 Dissection of the body was encouraged

 Disease was considered a visitation of evil and malevolent spirits

 Disease was curable only by prayer

 St. Agatha was the saint for diseases of the breast

 Priests became physicians and were forbidden to practice surgery

 The term “Doctor” for medical practitioners was:

 Introduced in the 12th century

 Introduced in the medical school in salerno

 Used by those who successfully completed a 5-year course in medicine

 Used by those who successfully completed a 3-year course in surgery


13 
 
 Used by those who successfully completed a year course in obstetrics and

gynaecology

 “No man can be a good physician who knows no surgery and conversely
on one can be a good surgeon without knowledge of medicine”. This was
said by:

 Hugh of Lucca

 Lanfranc of Milan

 Theodoric of Cervia

 Pope Innocent VI

 Henri de Mondeville

 Andreas Vesalius:

 Was born in Paris

 Was Professor at Padua

 Wrote “de Humani Corpori Fabrica” in 1543 at the age of 29

 Was the father of modern anatomy

 Challenged the views of Galen

 “de Humani Corpori Fabrica”

 Was a book on Physiology

 Was written by Galen

 Was beautifully illustrated

14 
 
 Contributed to the progress of surgery

 Was first published in Greek

 The Royal College of Surgeons of England was formed in:

 1725

 1801

 1845

 1885

 1902

 The Oldest Surgical College is probably:

 The Royal College of Surgeons of Edinburgh

 The Faculty of Physicians and Surgeons of Glasgow

 The Royal College of Surgeons of England

 The Royal College of Surgeons of Ireland

 The German Surgical College

 Surgery was made a science and not just an art by:

 William Cheselden

 John Abernethy

 Theodor Billroth

 John Hunter

15 
 
 Lord Lister

 John Hunter:

 Lived from 1728 to 1793

 Studied and worked in St. Thomas’ Hospital

 Built a large museum of 13,000 collections of various animals

 Studied inflammation, shock, wounds, aneurysm, and venereal

diseases

 Made the greatest advancement of surgery ever made by an individual

 Methods used to produce analgesia before the discovery of anaesthesia


included:

 Compression of the carotid arteries

 Mandrogora

 Compression of nerves to limbs

 Opium

 Hypnotism

 John Hunter:

 Was the father of scientific surgery

 Studied comparative anatomy

 Was English by birth

 Did appendicectomy

 Performed the first repair of femoral aneurysm in Hunter’s canal

 Hypnotism was first used in surgery in:

 Calcutta

16 
 
 MGH, Boston

 France

 U.C.H., London

 Royal Infirmary, Edinburgh

 The first to demonstrate the analgesic effect of nitrous oxide was:

 Horace Wells

 Sir Humphrey Davy

 Colton

 Bigelow

 C.T. Jackson

 Anaesthesia was first successfully demonstrated in public:

 By William Green Morton in 1846

 At Peter Bent Brigham Hospital, Boston

 With Dr. J.C. Warren as the surgeon

 For removal of a lump of the neck

 The patient was a child

 Anaesthesia in childbirth was:

 Introduced by James Young Simpson

 Achieved at first with ether

17 
 
 Opposed by the church

 Opposed by Queen Victoria

 Entails high morbidity for mother and child

 Endotracheal anaesthesia was introduced by:

 Halstead

 William Macewen

 August Bier

 Karl Kohler

 John Liston

 The germ theory of disease was established experimentally by:

 Louis Pasteur

 Joseph Lister

 Jacob Henle

 Koch

 Summerlweiss

 Joseph Litser:

 Introduced asepsis

 Used carbolic acid to dress wounds, sterilize instruments and spray

18 
 
the air in the operating area

 Was Professor of Surgery in University College Hospital, London

 Influenced Volkmann, Lagenbeck and Thiersh

 Is accredited with the germ theory of disease

 Steam sterilization of surgical dressings, gowns and towels was

introduced by:

 Volkmann

 Ernst von Bergmann

 Neuber

 Lister

 Lagenbeck

 Theodor Billroth:

 Did the first total laryngectomy for carcinoma

 Perfected a new operation on dogs before trying it in humans

 Introduced routine taking of temperature in surgical patients

 Was a life-long-friend of Schubert

 Was Professor of Surgery in Vienna

 Appendicectomy:

A. Was a common operation at the beginning of the nineteenth century


B. Became more popular in Great Britain when Sir Frederic Treves performed
it on Edward VII
C. Was performed for the first successfully diagnosed acute appendicitis by
T.G. Morton of Philadelphia
D. Was described by Hippocrates
E. Was widely performed by the ancient Egyptians
30. Theodor Kocher:

               A. Developed the operation of thyroidectomy


B. Studied the physiology of parathyroid
C. Won the Nobel Prize in 1909

19 
 
D. Was the greatest German Surgeon of all time
E. Lived from 1841 top 1917

31. The first successful salpingectomy for ruptured ectopic pregnancy


was done by:

 Spencer Wells
 Lawson Tait
 Anton Wolfler
 Thomas Morse
 Summelweiss
32. Early gynaecological surgeons included:

 Spencer Wells
 Marion Sims
 Sir Frederic Treves
 Lawson Tait
 Sir John Eric Erichson
33. Sir William Macewan:

 Worked in London
 Was the father of neurosurgery
 Was the first to diagnose a cerebral abscess and successfully operate for
extradural haemorrhage
 Introduced the endotracheal tube
 Introduced use of bone wax to control haemorrhage in neurosurgery
34. Early neurosurgeons included:

 Victor Horsley
 E.C. Cuther
 Harvey Cushing
 Halstead
 Robert Gross
35. Cardiac surgery was first pioneered in:

 New York
 Boston
 Edinburgh
20 
 
 Vienna
 Paris
36. Early cardiac surgeons included:

 Robert Gross
 Clarence Craaford
 Henry Souttar
 John Strieder
 Alfred Blalock

37. The first successful cardiac operation was:

 Ligation of patent ductus arteriosus


 Blind mitral valvotomy for mitral stenosis
 Repair of co-arctation of the aorta
 Repair of Fallot’s tetralogy
 Blind aortic valvotomy for aortic stenosis
38. Orthopaedic Surgery was:

 The first sub-specialty to branch off from general surgery


 Pioneered by Hugh Owen Thomas who practised in a London Hospital
 Propagated by Sir Robert Jones considered the father of modern
orthopaedic surgery
 Given a big boost during and after the first world war
 Has always been exclusively dominated by trauma
39. Transplantation of:

 Kidney was first done successfully by Lawler in Chicago in 1950


 Kidney between identical twins was first done by Joseph Murray in Mayo
clinic in December 1954
 Kidney has over 2-year survival in more than 60 per cent of patients
 Heart was first done by C. Bernard on December 3rd, 1967 in Boston
 Liver is now routine
40. The following was (were) involved in the development of blood
transfusion:

 Jean Baptise Denis


 James Blundell
 Bernard Fantus
 Karl Lansteinner
 M. Hustin
41. Blood bank was first organized in:

 Guy’s Hospital, London

21 
 
 John’s Hopkins, Baltimore
 Cook County Hospital, Chicago
 St. Mary’s London
 State Hospital, Moscow

42. Discoverers of the blood groups included:

 Arthus of Fribourg University


 Karl Landsteinner
 Mollison
 DeCastello
 Starli
43. The first laparoscopic procedure was:

 Splenectomy
 Done by Muhe
 In 1985
 Done in France
 Done without anaesthesia
44. Laparoscopic surgery compared to open surgery generally:

 Causes minimal tissue injury


 Results in attenuated immuno suppression
 Has low morbidity and mortality
 May cause more severe pain
 Is preferable in most procedures
45. Laparoscopic surgery can be used for:

 Reflux oesophagitis
 Adrenalectomy
 Oesophagectomy
 Partial gastrectomy
 Pancreatectomy
46. In the future, it may be possible to do appendicectomy via:

 The mouth
 Vagina
 The anus
 A port opening in the anterior abdominal wall
22 
 
 Perineum
47. In telesurgery:

 The surgeon does not have to be in the theatre


 The operation is performed through a robot
 Complex movements similar to those of the wrist and fingers are possible
but with tremors
 The instruments are jointed to allow freedom of movement
 The rates of conversion, complications and mortality are lower than in
laparoscopic surgery

CHAPTER 1
SHORT HISTORY OF SURGERY
ANSWER

 C 34. A, C
 A, B, C 35. B
 A, B, C, D, E 36. A, B, C, D, E
 A 37. B
 A, C 38. A, C, D
 E 39. A, C
 B, C, D 40. A, B, C, D, E
 B, D 41. C
 B, C, D, E 42. B, D, E
 A, B, C 43. B, C
 B 44. A, B, C
 B, C, D, E 45. A, B, C, D
 C, D 46. A, B, C
 C 47. A, B, D
 A
 D
 A, C, D, E
23 
 
 A, B, C, D, E
 A, B
 D
 B
 A, C, D
 A, C
 B
 A
 B, D
 B
 A, B, C, E
 B, C
 A, C, E
 B
 A, B, D
 B, C, D

24 
 
CHAPTER 2

MICROBIAL INFECTIONS

 Historically the early seemingly insurmountable major


obstacles to safe surgery are considered to have been:

 Haemostasis
 Sepsis
 Acute Pain Relief
 Access to internal organs
 Development of suture materials
 Match the surgical historical events with the credited pioneer:

 The genesis of aseptic surgery


 Development of antiseptic surgery
 The principle of antibiosis
 The concept of germ theory of sepsis
 Human Immune Deficiency Syndrome
 Semmelveis of Vienna 2. Jesiph Lister 3. Alexander
Fleming
4. Louis Pasteur 5. Montagne
3. Currently (21st Century) the most crucial obstacle to safe
surgery is
considered to be:
A. Haemostasis
B. Sepsis
C. Pain Relief
D. Access to internal organs
E. Development of suture materials
4. Which of these factors are considered important in the
establishment of
pyogenic surgical infections:

1
A. The Dose of the organism
B. Tissue environmental pH
C. Oxygen tension of tissues
D. Microbial synergism

E. Particular cytokines (TNF, Interleukins)


5. The most important factor influencing the establishment of a
pyogenic
infection is:
A. Infectivity of the organism
B. Dose of the inoculum
C. Anaerobic environment in the tissue
D. Sutures contained in the tissue
E. Attendant tissue necrosis
6. The minimum quantum of most pyogenic organisms required to

establish

an infection is:

A. 109 g. of tissue or per ml of exudate


B. 107 g. of tissue or per ml of exudate
C. 105 g. of tissue or per ml of exudate
D. 103 g. of tissue or per ml of exudate
E. 102 g. of tissue or per ml of exudates
7. The resistance of a patient is crucial to the establishment of a

pyogenic

infection and the most important determinant of this is:

 Systemic co-morbidity (Diabetes, malignancy)


 Anaemia

2
 Malnutrition
 Diseases of the immune system
 Associated jaundice

8. Enhancement of host defence mechanism is crucial to the


prevention of
establishment of pyogenic infections. The most important
contribution
comes from:
 Neutrophil granulocyte function

 Tissue histiocyte function

 Function of Fixed Histiocytes

 Non-thymic dependent lymphocytes

 Thymic dependent lymphocytes.

9. Surgical infections:

A. Are often determined in their extent by anatomical barriers


B. Invariably require surgical intervention to ensure a cure
C. May often settle on conservative measures when presenting early.
D. In the elderly are often complicated by tissue necrosis
E. May be complicated by the systemic
Inflammatory response syndrome (SIRS)

3
10. Staphylococcus pyogenes:

A. Causes pustules, furuncles and carbuncles


B. Produces spreading infections of the skin
C. Causes necrotising fascitis
D. Are often found in the nasoharynx
E. Is responsible for a very severe form of pneumonia with lung
abscess formation.

11. Staphylococcus pyogenes produces localizing infections on


account of:
 The ά-toxin that is associated with it
 The erythrogenic genic toxin it produces
 The agent coagulase it produces
 Local necrotizing activity
 Enhancement of haemostasis
12. Streptococcus pyogenes produces the following toxins:

A. Leucocidin
B. Haemolysin
C. Erythrogenic toxin
D. Hyaluronidase
E. Deoxyribonuclease
13. β-haemolytic streptococcus pyogenes:

A. Is found in the pharynx of 5-10% of the population


B. Spreads by close contact and droplets
C. In tissues spreads along the tissue planes
D. Is responsible for tonsillitis, otitis media and puerperal sepsis

4
E. 90% of them beleng to the Lancefield Group A
14. Streptococcus viridans:

A. Is a β-haemolytic streptococcus
B. Is a normal comensal of the Duodenum
C. Characteristically causes intra-oral infections – tonsilitis
D. May seed on to cardiac valves producing subacute bacterial
endocarditis
E. Frequently contaminates chronic leg ulcers and burns

15. Among the Gut organisms:

A. 97% are Esch. coli.


B. 1% are Bacteroides Fragilis
C. The non lactose fermenters are pathogenic
D. The lactose fermenters however predominate
E. Are agents that readily induce endobacteraemic shock
16. Haemophilus influenzae is:

A. A Small Gram-ve bacillus


B. Often found in the upper respiratory tract
C. Causes epiglottitis, meningitis, and bronchitis
D. Usually sensitive to penicillin
E. A frequent culprit in the systemic inflammatory response
syndrome (SIRS)
F. Responsible for endotoxic shock in respiratory infections.
17. Kelbsiella Organisms are:

A. Gram-ve rods
B. Responsible for pneumonic lesions

5
C. Occur in hepatic abscesses
D. Noted to contribute to wound infections after transplantation
procedures
E. A frequent cause for endotoxic shock
18. Pneumococci are:

A. Gram +ve diplococci


B. Encountered as commensals in the anterior nares
C. May be isolated as cause for Primary Peritonitis
D. A notable cause for the Systemic Inflammatory Response
Syndrome.
E. When producing wound infection often precipitates endotoxic
shock.

19. A man of 60 with a week’s history of swelling of the left leg, with
fever,
malaise and anorexia and pitting oedema of the ankle is most
likely to be
suffering from:
A. Deep Venus Thrombosis of (Lt.) leg.
B. Congestive cardiac failure
C. Cellulitis of (Lt.) leg
D. Lymphaedema of the leg
E. Pyomyositis of the Lt. leg
20. The following are likely complications of Q19:

A. Phlegmasia Caeraleus Dolorens


B. Lymphangiitis and lymph adenitis
C. Necrosis of significant portions of the overlying skin

6
D. Lymphoedema
E. Pulmonary embolism
21. Treatment strategy of patient in Q.19:

A. Bed Rest
B. Elevation of affected limb
C. Immobilization of the affected limb
D. Crystaline penicillin 2 million, units 8 hourly 1/M
E. Debridement of dead and dying skin and soft tissues.
22. Cellulitis of the Scalp:

A. Is produced by staphylococcus pyogenes


B. Occurs in the Subaponeurotic layer of the scalp
C. Is coterminous with the attachment of the epicranial aponeurosis
D. May result in ischaemic necrosis of the cranium
E. May be complicated by thrombophlebitis of the superior sagittal
sinus

23. Orbital Cellulitis:

A. Results from infection of the paranasal air sinuses


B. Is accompanied by ptosis and paresis of ocular movements
C. May produce panophthalmitis
D. Is complicated by meningitis
E. May result in Cavernous Sinus thrombosis
24. Ludwig’s Angina is:

A. Characterised by pronounced intra oral oedema accompanying


Inflammations of the submandibular space
B. Caused by Streptococcus pyogenes traceable to the 1st and 2nd

7
lower
molar teeth
C. Hemmed in by attachment of deep fascia to the hyoid bone
D. Associated with spread of inflammatory fluid along the sheath of the
stylohyoid muscle.
E. An emergency because it may precipitate asphyxiation.
25. In Ludwig’s Angina:

A. There is associated upward and forward displacement of the tongue


B. Oedema of the glottis is an ever present danger
C. Management is initially with large doses of parenteral penicillin and
metronidazole
D. There is need for timely surgical decompressing incision
E. The outcome of timely management is satisfactory.
F. The decompression is preferably done under local anaesthesia
G. The Incision is parallel to the lower berder of the mandible
H. The Incision divides the deep fascia and mylo hyoid muscles
I. The Incision should be at least 2.5cm below the angle of the
mandible

26. In infection of the pharyngo-maxillary space:

A. The lesion is in a pyramidal space.


B. The Base of the space is against the base of the skull
C. The Apex is at the Greater Cornu of the hyoid
D. The Medial wall is formed by the superior pharyngeal constrictor
muscle.
E. The responsible Streptococcal organisms reach the space from the
tonsils

8
F. The Immediate antecedent may be tonsellectomy or peritonsillar
abscess.
G. Trismus and a tender mass over the parotid are the hall mark
features.
27. A Phlegmon:

A. Is a unilocular abscess
B. Is an inflammation which is spreading and diffuse
C. Usually incorporates small multiple tiny pockets of pus
D. Is usually the result of infection with mixed organisms – streps and
staphs.
E. Is often occasioned by extensive suppuration and tissue necrosis
28. An Abscess is:

A. The result of ischaemia of the involved tissue


B. Produced by liquefaction of dead tissue by proteolytic enzymes in
polymorpho nuclear leucocytes.
C. Delimited from the contiguous normal tissues by a pyogenic
membrane
D. Usually fluctuant when ripe
E. Ripe when there is point tenderness
F. Not usually fluctuant when ripe, if it is deeply seated

29. A Carbuncle is:

9
A. The result of infection of several hair follicles
B. Produced by Streptococcus pyogenes infection
C. Seen in tissues well endowed with thick columns of fat
D. Characterised by radial spread of infection
between the skin and deep facia
E. Associated with ischaemic necrosis
30. Carbuncles:

A. Usually at presentation do not exhibit pyogenic membrane


B. Seldom generate profound systemic symptoms
C. Tend particularly to occur in poorly controlled diabetics
D. Should be treated energetically with lincomycin
E. Have the potential to produce spreading thrombophlebitis.
31. Bacteraemia:

A. Implies the transient invasion of the blood or lymphatics by bacteria


B. May follow disturbances of trivial cutaneous, mucosal foci of infection
C. Is usually asymptomatic
D. May engender lifelong disability from enodocarditis and
pyelonephritis
E. Should be antecipated and managed by prophylactic exhibition of
antibiotics.
32. Pyaemia:

A. Implies the presence of septic emboli in the circulation.


B. Results from cardiac vegetations and clumps of included organisms.
C. Usually leads to seeding of abscesses in the location of red marrow
in the
in the body.
D. Of the systemic circulation leads to lung abscesses
E. Of the Hepatic Portal system produces pylephlebitis

10
33. The Systemic Inflammatory Response Syndrome (SIRS):

A. Is characterised by Fever, Chills, Malaise, Anorexia, and Vomiting


B. May persist after apparent elimination of the invading organism
C. May follow non infective conditions – e.g. trauma, pancreatitis
D. Frequently manifests clinically as multiple organ dysfunction
syndrome (MODS)
E. Represents uncontrolled activation of the inflammatory pathways.
34. The Clinical manifestations of septicaemia (Blood Stream

Infection) include:

A. Malaise, apathy and anorexia


B. Subnormal temperature
C. Listlessness, confusion, delirium
D. Nausea and vomiting in children
E. Profuse sweating
35. The Scalded Skin Syndrome:

A. Is a form of infective gangrene


B. Is caused by a virulent strain of staph pyogenes
C. Is marked by crepitus in the surrounding skin
D. Is often accompanied by Gram-ve septicaemia
E. Rapidly responds to hyerbaric oxygen
36. Which of the following factors is/are of importance in the
establishment
of infection?

 The dose of the organism


 The virulence of the organism
 Microbial synergism
 Associated diseases
 Age of the patient

11
37. Currently the organism responsible for most wound infections on
surgical
wards is:
 Streptococci
 Staphylococci
 Pneumococci
 Haemophilus influenza
 Proteus organism
38. This organism is to be found in:
A. Pharynx of 5-10 per cent of the population
B. The anterior nares of about 50 per cent of the population
C. The upper respiratory tract of normal people
D. The oral cavity of normal people
E. Lower gastro-intestinal tract of patients
39. Which of the following clinical conditions may be regarded as
common
sequelae of cellulitis of the limb seen in the tropical environment?

 Localised abscess formation


 Skin necrosis with ulceration
 Pyomyositis
 Septic arthritis
 Pyoderma gangrenosum
40. In which of the following sites does cellulitis carry a particular

risk?

12
A. Neck
B. Scalp
C. Orbit
D. Philtrum of upper lip
E. Anterior nares

41. Which of the following lesions represents infection of hair

follicles?

A. Cellulitis
B. Carbuncle
C. Vesicle
D. Pyoderma gangrenosum
E. A macule
42. The danger from bacteraemia lies particularly in:

A. Seeding organisms on delicate kidney tubules


B. Lodgement of bacteria on tissue implants
C. Implantation of organisms on vulnerable cardiac valves
D. Development of fulminating pyomyositis
E. Initiation of disseminated intravascular clotting
43. Which of the following is/are regarded as evidence of
unfavourable
prognosis in the patient with severe pyogenic infection?
 Absence of significant pyrexia or subnormal temperature
 Total white cell count and absolute eosinophil count within normal
limits
 Cerebral manifestation involving changes of adequate hydration

13
 Rapid diminution of urinary output in face of adequate hydration
 Normal red cell count.
44. Match the following forms of synergistic Gangrene/Cellutitis with
their
causative organisms:
 Melaney’s Gangrene
 Ulcerative Gingivitis
 Cancrum Oris
 Necrotising Fasciitis
 Fournier’s Gangrene

 Aerobic staph-aureus and anaerobic pepto streptococcus


 Fusi formis bacillus and spirochete
 Aerobic Gram positive cocci and Bacteroides melaninogenicus
 Coli forms, Staphylococci and Bacteroides
 Staphylococci, Ecoli and Fuso bacterium and Clostridium welchi
45. In Synergistic Gangrene:

A. The anaerobic partners are usually the primary pathogens


B. The aerobic partners contribute the destructive enzymes that
establish the aggressive lesion
C. Eliminating one partner is however sufficient to bring the lesion
under control.
D. It is the aerobic component that threatens life on invasion of the
blood stream
E. The principle of management is primarily massive antibiotic dosing
I/V only.
46. The conditions under which Tetanus wound infection is likely to

14
occur
include the following:
 Road Traffic Accidents
 Injuries in agricultural fields
 Deep wounds with muscle damage
 Foreign body containing wounds
 Some physiological wounds – birth wounds
47. In tetanus wound Infection:

A. The average incubation period is 6 to 10 days:


B. The incubation period gauged from wounding to first symptom is
fallacious
C. The Period of Onset (1st symptoms to 1st reflex spasm) is a
dependable
prognostic index
D. The first symptom is trismus
E. The first symptom is pain the jaw
F. The first symptom is reappearance of pain at the wound site.
G. Hesitancy of micturition is common.

48. In clinical tetanus:

A. Initial tonic muscle spasm spares the limbs


B. The visceral muscles, skeletal or smooth, are not involved.
C. Reflex muscle contractions (clonic spasms) are a measure of the
severity
of the illness.
D. Pulmonary ventilation is diminished but cyanosis does not occur
E. The patient shows no mental disturbance even in severe cases
49. Tetanus prophylaxis:

A. Has no bearing on the outcome of management of a particular case


B. Has had no influence on the prevalence of neonatal tetanus
C. Is equally effective in both active and passive forms
D. Must be applied in all exposed patients

15
E. In the active form traditionally requires a dose of toxoid at intervals
of
 weeks and 6 months.
F. Can be rapidly induced with 3 doses on day 1, 4 and 7
50. In Tetanus prophylaxis:

A. An injured previously immunized patient needs only a booster dose


of 0.5mls
of APT (alum precipitated toxoid)
B. Combination of toxoid and immune serum enhances the immune
response
C. Where the Human immune serum (ATS) is available a
subcutaneous test
dose is superfluous.
D. A non-immunised patient can be given 250 units of Human ATS, but
must later
undergo active immunization.
E. A prophylactic dose of equine serum is useful only once in a
patient’s lifetime.

51. The objectives in the management of clinical Tetanus includes:

A. Limiting production of toxin within the wound


B. Neutralising circulating toxins
C. Eliminating source of infection
D. Controlling convulsions or tonic rigidity
E. Preventing complications
F. Nutritional supportive therapy

52. The modern management of clinical tetanus requires availability

of:

A. High Dependency Unit


B. Intensive Care Unit

16
C. Specialised Intensive Care Unit
D. General Ward Services
E. Dependable Ambulatory Service Unit
F. A Directing Team: Physician, Surgeon, Anaesthetist
53. Mild Cases of Clinical Tetanus:

A. Have tonic rigidity but no clonic convulsions


B. Show both tonic rigidity and clonic spasms
C. Do not have swallowing difficulties
D. Have respiratory embarrassment
E. Maybe controlled by sedation with Promethazine without muscle
relaxants.
F. Require the passing of a naso-gastric tube for feeding purposes
G. Usually need intubation and ventilation by intermittent positive

pressure mechanism

54. The Seriously ill patient with clinical Tetanus:

A. Has tonic rigidity, occasional reflex spasms and dysphagia


B. Has dysphagia only without tonic rigidity or reflex spasm
C. Requires a naso-gastric tube for feeding
D. Should be managed initially without tracheostomy or intubation
E. Usually requires doses of muscle relaxants
55. The Dangerously ill patient with Clinical Tetanus:

A. Experiences major cyanotic convulsive attacks


B. Needs maintenance of a clear air way by intubation and control of
muscle spasms
C. Is threatened more by the systemic effects of the toxin than the
action
of latter on the CNS
D. Can be managed with I/V magnesium sulphate in centres without
ventilation facilities
E. Benefit more from antibiotic prophylaxis using metronidazole
than penicillin.

17
56. In the Dangerously ill patient with clinical Tetanus Significant
prognostic
signs include:
 Pronounced tachycardia
 Severe Diaphoresis (Sweating)
 Wide fluctuations of the blood pressure
 Intense peripheral vaso constriction
 Increased Urinary excretion of catecholamines
(> 1000ugm/day)
57. In patients dangerously ill with Tetanus:

A. The mortality in general units does not exceed 10%


B. Protection of the eyes from conjunctivitis is essential
C. Hyperbaric oxygen currently shows no proven benefit
D. The centrally acting sympatholytic agent CLONIDINE improves the
outcome of management
E. Adequate nutrition by gavage should be started early.
58. In Gas Gangrene:

A. The causative organisms Cl. welchii, Cl. perfringens, Cl. novy, Cl.
oedematiens
Cl. septicum are sacharolytic
B. The causative organisms Cl. sporogenes, and Cl. haemolyticus are
proteolytic
C. The most important toxin produced is the α-toxin
D. The anaerobic organisms are introduced more often by faecally
contaminated cloathing
E. Excessive haemorrhage and use of tourniquet are predisposing
causes.
59. In Gas Gangrene:

A. Gross muscle damage is seldom a predisposing factor.


B. Streptococcal and staphylococcal organisms promote establishment
Of the infection

18
C. Retained foreign bodies do not affect the outcome of management
D. The causative organism may be autogenous (gastric and biliary
mucosa)
E. The lower limbs are more often affected than the upper limbs.

60. Following a road traffic accident a man of 50 presents with a


rapidly
developing swelling of his right thigh of 24 hours duration with
local pain and
oedema. The skin soon darkens but there are no rashes or
blisters and in
particular no crepitations can be elicited. The patient feels unwell
anorexic
and on examination has a temperature of 39°C, pulse 110/min ; he
is pale and
shows a tinge of jaundice.

The most likely diagnosis is:

 Haematoma of thigh muscles


 Compound fracture of shaft of femur
 Pyomyositis of right thigh
 Claustidial myonecrosis of right thigh
 Cellutitis of the right thigh
 Deep venous thrombosis affecting common femoral vein
61. Indicated Therapy in question 52 is:

A. Bed Rest with elevation of the leg


B. Therapeutic doses of anticoagulants
C. Exhibition of Cloxacillin 1500mg 6 hourly for 7 days
D. Exhibition of Penicillin 2mega units 6 hourly I/M
E. Exhibition of Penicillin, Metrondazole and extensive Debridement
62. Following an above knee amputation in a diabetic woman of 45

19
there is rapid
painful swelling of the stump within 24 hours, with oedema, and
discharge
of blood stained brownish exudates from a drain placed in the
wound. The
patient feels unwell but she is apyrexial, not anaemic and has a
pulse of
100min.

The most likely diagnosis is:

 Haematoma of the stump


 Operative Site Infection
 Retained foreign body
 Gas gangrene
 Deep venous thrombosis affecting common femoral vein
63. Prophylactic Measures in Clostridium welchii infection include

(s):

A. Early debridement of the wound


B. Lavage of the wound with saline and H202
C. Injection of an ampoule of polyvalent antitoxin
D. Prophylactic crystalline Penicillin one mega unit
E. The use of hyperbaric oxygen
64. In the treatment of an establised case of clostridial myonecrosis:

A. Action needs to be prompt


B. Excision of all suspicious muscle is the sheet anchor of

20
management
C. Irrigation of the wound and packing with gauze wrung in H202 is of
no
value
D. Crystalline Penicillin 10 mega units a day – with metronidazole and
clindamycin together is better than penicillin alone
E. Polyvalent antitoxin 15,000 unit/Kg body wt. is essential
F. Hyperbaric oxygen is of little practical value
65. The distinction between Clostridial myonecrosis and necrosting

fasciitis

can be made on basis of which one of these criteria:

 Local tissue necrosis


 Skin Erythema
 Detection of gas in the tissues radiologically
 The physical characteristics of the exudates

E. Systemic symptoms and signs


66. Erysepelas:

A. Arises from cutaneous infection with strains of streptococcus


pyogenes:
B. Has a predilection for the face
C. Is characterized by a sharply defined border of its lesion
D. Frequently ends in suppuration
E. Often impairs lymphatic drainage of part affected
F. Has a predilection for the eyelids
G. Is unusual in patients with normal immunological resistance
67. In the treatment of Erysepelas:

21
A. The antibiotic of choice is ciprofloxacin
B. The antibiotics must be given for a minimum period of 5days
C. The beneficial effects of antibiotics are evident within 72 hours
D. Care must be taken to ensure adequate nutrition
E. Monitoring to ensure against subsequent functional damage to
lymphatics
68. Anthrax:

A. Is caused by a Gram –ve anaerobic spore bearing organism


B. Shows a predilection for the central nervous system and the
urogenital
tract
C. May be acquired from handling animal hides
D. Is encountered in two systems of the body only: the respiratory
and digestive
systems
E. Occurs in sporadic/epidemic form
F. May take the form “a malignant pustule”
G. Is transmitted by inhalation and the oro-faecal route

69. The Pulmonary form of anthrax:

A. Is the commonest manifestation of the disease


B. Follows the most severe course
C. Takes the form of a suppurative pneumonia with lung abscesses
D. Is usually rapidly fatal, within 36 hours
F. Responds better to Erythromycin than Crystalline Penicillin

22
70. The Intestinal form of Anthrax is:

A. More contagious than the pulmonary form


B. Marked by profuse diarrhoea and shock
C. Usually painless
D. Frequently apyrexial
E. Best treated by the combination therapy: ciprofloxacin and
metroridazole
F. Attended by multiple intestinal perforations.

71. The Cutaneous form of Anthrax:

A. Takes the form of a furuncle


B. Shows a dark necrotic centre
C. Follows a mild course
D. Should not be incised
E. Is the most contagious form of the disease
F. Responds to local application of antiseptic agents: povidone iodine
72. Rabies:

A. May be contracted through the bite of any mammalian animal


B. Shows through its symptoms and signs within 48h of the bite
C. In most cases affects the jaw muscles first
D. Is associated with drooping of the mandible
E. Is marked by profuse salivation
F. Causes progressive muscle paralysis

G. Causes death within 2-5 days from onset


H. Is confirmed by Negri Bodies in the brain
I. Is no longer a significant public health problem in many tropical
countries.
73. In Rabies:

23
A. A 10-14 days period of observation of the suspects is mandatory
B. The incubation period is shorter for facial and trunk bites
compared
with bites on the limbs
C. Pain at the site of the bite is the most important early symptom
D. The hydrophobia is produced by laryngeal muscle spasm
E. The Diagnosis is confirmed by the finding of Negri bodies in the
brain
74. In the management of Rabies:
A. Prophylactic human diploid vaccine (HDCV) is the most effective
measure
B. The vaccine should be given immediately after exposure
C. Pre-exposure prophylaxis is essential for animal handlers
D. Where risk from rabies is serious combined rabies antiserum and
vaccination gives best protection
E. Thorough irrigation of the wound with soap reduces the number
developing
clinical rabies
F. Thorough wound irrigation lowers the mortality from 90% to 5%
G. It is safer to leave the wound unsutured
H. Tetanus toxoid and antibiotic should also be given

75. Ten hours following a mid-thigh amputation for atherosclerotic

24
disease,
a 70-year old man develops fever, tachycardia, weakness with
intense
wound pain and rapid swelling and oedema of the stump. The
patient
rapidly becomes anaemic and 24 hours later becomes mildly
icteric; there
is evidence of impaired peripheral circulation and his 24-hour
urinary output
is about 600ml.

The most likely diagnosis is:

A. Post-operative falciparum malarial infection


B. Post-operative haematoma and haemorrhagic shock
C. Transfusion reaction
D. Clostridial myonecrosis
E. None of the above
76. You will advise:

A. Extensive wound debridement to the point of higher amputation


B. Exhibition of penicillin in doses of 10 million units a day
C. Administration of gas gangrene serum in doses of 15,000 units per
kg
D. Packing the wound with gauze wrung in hydrogen peroxide
E. None of the above
77. Which of the following animals is/are known to carry and

transmit rabies?

A. Bat
B. Squirrel
C. Horse
D. Cattle
E. Cat

25
78. Currently which of the following do you consider the most
effective first line
regime against the mycobacterium tuberculosis?

 Isoniazid 300 mg daily + thiacetazone 150mg daily for seven


months
 Streptomycin 1G once a day 30 days, Isoniazid 300 mg +
Rifampicin
600 mg daily for 8 months
 Streptomycin 1G once a day 30 days, PAS + Isoniazid for 1 year
 INAH 300 mg daily + PAS for 1 year
 Rifampicin 600 mg + Isoniazid 300 mg daily for 18 months
79. Which of the following is/are known to be manifestation(s) of

yaws?

A. Pyoderma gangrenosum
B. Pachyderma keratitis
C. Scaling dermatitis of the palm
D. Erythema nodosum
E. Lymphoedema of the affected foot
80. A young man of 25 presents with a painless, firm round ulcer on
the glans of
the penis which develops two weeks after sexual exposure. He
has no
systemic symptoms.

26
He is most likely to be suffering from:

 Chancroid
 Syphilis
 Genital herpes
 Aphthous ulcer
 Lymphogranuloma venereum

81. Noted extragenital sites for primary syphilitic lesion include:

A. Fingers
B. Anal canal
C. Lips
D. Inner thigh
E. Tongue
82. Which of the following chemotherapeutic agents exert their
influence by
dissolution of the cell wall of the organisms?
 Penicillin
 Cephalosporins
 Lincomycin derivatives
 Cloxacillin derivatives
 Nystatin
83. Which of the following chemotherapeutic agents act by
competition with the
organisms for substrate by influencing the ribosomes inhibiting
protein

27
synthesis?
 Sulphadimidine
 Tetracycline
 Chloramphenicol
 Erythromycine
 Gentamicin
84. Prophylactic antibiotic therapy is justified in:

A. Open wounds of the abdominal, thoracic and pericardial cavities


B. Compound fractures
C. Burns
D. Pre-operative bowel preparation
E. Appendicectomy for uncomplicated acute appendicitis

85. Prophylactic antibiotic therapy is justified in:

A. After routine inguinal herniotomy


B. Elective cardiac operations in patients with rheumatic heart
disease
C. Patients with strangulating intestinal obstruction
D. Where vascular prostheses are employed
E. Patients with severe acute pancreatitis
86. Which of the following factors is (are) considered as HIGHLY
SIGNIFICANTLY affecting the development of post-operative
wound
infection?
 Wound bacterial load at end of operation
 Potentially dirty operation
 Patient’s environment – ward accommodation

28
 Age of the patient
 Haematoma formation
87. Which of the following factors is/are considered as significantly
affecting the
development of post-operative wound infection?
 Glove punctures
 Nasal carriage-patient and staff
 Emergency operation
 Skin towels
 Skin carriage of organisms
88. Which of the following may be considered as effective means of
preventing
hospital infection?
 Removal of the sources and reservoir of infection

 Blocking the routes of transfer to the patient

 Improving the patient’s resistance

 Establishing plenium ventilation in the theatres

 Extensive use of prophylactic antibiotics

89. The temperature in the operating theatre should ideally be kept

at:

A. 10-15˚C
B. 15-20˚C
C. 20-25˚C
D. 25-30˚C
E. 30-35˚C

29
90. The humidity in the operating theatre should ideally be:

A. 35-45 per cent


B. 45-55 per cent
C. 55-65 per cent
D. 65-75 per cent
E. None of the above
91. The ventilation system of the theatre should have ideally which
of the
following changes of air?

 About 5 per hour


 About 10 per hour
 About 15 per hour
 About 20 per hour
 About 25 per hour
92. Which of the following forms of ventilation may be considered
ideal for
the operating theatre?

 The plenium system

 The turbulent method


 The unit air conditioning system
 Through ventilation across the room
 A fan extraction system

93. Staphylococcus pyogenes:

A. Is carried in the anterior nares of 50 per cent of the population


B. Is no longer important as a cause for would infection in most
tropical

30
countries
C. Is responsible for boils, carbuncles, breast abscesses and
osteomyelitis
D. Produces the exotoxin--toxin
E. Is usually sensitive to cloxacillin, lincomy cin and cephalothin
94. Streptococcus pyogenes:

A. Is carried in the pharynx of 5 to 10 per cent of the population


B. Is less important now as cause for wound infection compared
with
the coliform organisms
C. Is responsible for tonsillitis, otitis media, puerperal sepsis and
erysipelas
D. Produces the exotoxins erythrogenic toxin, hyaluronidase
E. Usually responds to crystalline penicillin which is the antibiotic
of choice
95. Esch. Coli:

 Are normal inhabitants of the large bowel


 Frequently infect cervical and facial wounds
 May, if of a particular strain, produce infantile diarrhoea
 Elaborate a powerful endotoxin producing shock
 Aminoglycosides, e.g. kanamycin, gentamicin are the agents
of choice
96. Pseudomonas pyocyanea:

A. Usually occurs as a normal commensal in the bowel


B. Is an organism of low pathogenicity and is encountered as a
secondary invader in superficial wounds
C. Produces pus of a characteristic odour and bluish green
colour
D. Elaborates an endotoxin responsible for septic shock
E. Is sensitive to pyopen

31
97. Cellulitis of the limb:

A. Is commonly produced by streptococcus pyogenes


B. Most frequently follows inoculation of the organism through a
blister or abrasion
C. May in severe cases cause brawny oedema of the limb
D. Is often complicated by localizing abscesses
E. Should be treated by early incision and decompression of
the
of the turgid subcutaneous tissues
98. Cellulitis of the scalp:

A. Occurs from infection of the sub-aponeurotic layer of


areolar tissue
B. May lift the periosteum from the skull when pus forms
C. May be complicated by cerebral vein thrombosis
D. Should be treated initially by large doses of crystalline
penicillin
E. Nearly always has fatal outcome whatever the mode of
management
99. Cellulitis of the orbit:

A. Results from spread of infection from the paranasal sinuses


B. Usually produces proptosis and paresis of ocular
movements
as a notable feature
C. Carries a risk of spread of infection to the meninges and
cavernous sinus
D. May produce panophthalmitis which may spread to the
other eye
E. Should be treated initially with large doses of penicillin until

32
culture
results are known

100. Cellulitis of the pharyngo-maxillary space:

A. Results from tonsillitis, peritonsillar abscess or after a


tonsillectomy
B. Often accompanies an attack of Ludwig’s angina
C. Produces painful trismus, odynophagia and tender
swelling over the
lower part of the parotid gland as typical clinical features
D. May be complicated by erosion of the carotid vessels and

their main

braches
E. Should be treated with appropriate antibiotics and early
surgical
decompression of the space
101. Cellulitis of the pharyngo-maxillary space:

A. Is often a sequel to glossitis


B. Is rather insidious in onset and diagnosis is nearly always
late
with fatal consequences
C. May be complicated by thrombophlebitis of the jugular
vein
D. Seldom requires surgical decompression as an essential

33
feature
of treatment
E. Shows a strong tendency to recurrence
102. A carbuncle:

A. Results from infection of a group of hair follicles by staph


pyogenes
B. Is usually restricted to skin in specialized areas of the
body, e.g.
neck, hands, scalp
C. Is liable to occur particularly in diabetics
D. Appearing on the lips or on scalp may produce
thrombophlebitis
of cerebral sinuses
E. Should be treated initially by excision of the lesion

103. Bacteraemia:

A. Implies transient invasion of the circulation, blood and


lymphatics
by pathogenic organisms
B. Is usually asymptomatic and harmless
C. Invariably leads to abscess formation in the well
vascularized organ,
e.g. the spleen
D. May follow tooth extraction or urethral catheterization
E. Produces micro aneurysms of the peripheral arterioles
104. Septicaemia:

34
A. Implies invasion of the circulation by actively multiplying
organisms
B. Arises from intravascular septic foci, e.g. abscesses,
infected
thrombosis
C. Is announced by chills and rigors and profound prostration
D. Usually produces significant leucocytosis with a shift to the
left
and a disappearance of eosinophils
E. Is associated with a severe hypercatabolic state with
marked
weight loss
105. Tetanus:

A. Produces a number of exotoxins of which tetanospamin is


the most
important
B. Is more likely to follow puncture wounds in which necrotic
tissue
and pyogenic organisms are present
C. Toxin reaches the central nervous system via the blood
stream
D. Toxin interferes with the acetyl-choline/cholinesterase
balance at
the peripheral motor end plates
E. Infection with a period of onset of less than 48h has a poor
prognosis

35
106. Modern management of tetanus:

A. Emphasizes a team approach


B. Emphasizes drainage of pus and careful wound toilet
C. Requires administration of human immune serum in doses
of
500 units daily (maximum 6000 units)
D. Advocates antibiotic prophylaxis against the organisms of
respiratory
and urinary tract infection
E. Is closely related to the stage of development of the
disease
107. Modern management of clinical tetanus:

A. Is best dealt with initially in the intensive care setting


B. Advocates early wound excision or amputation
C. Requires administration of equine serum (200,000 units)
D. Eschews prophylactic use of antibiotics
E. Advocates early tracheostomy in patients having
swallowing
difficulty
108. The severely ill patient with clinical tetanus:

A. Has tonic rigidity, occasional reflex spasm and difficulty


with
swallowing
B. Has opisthotonus as his main significant feature
C. Is best fed by using a nasogastric tube
D. May require tracheostomy using a cuffed tube
E. Requires sedation with promethazine (100-150 mg) or
amylobarbitone
(200 mg) 6 hourly

36
109. The severely ill patient with clinical tetanus:

A. Usually has difficulty in swallowing but not cyanotic attacks


B. Has tonic but no clonic muscle spasm
C. Always needs intravenous alimentation
D. Invariably needs a tracheostomy and intermittent positive
pressure ventilation
E. Requires regular sedation with diazepam
110. The dangerously ill patient with clinical tetanus:

A. Experiences major cyanotic convulsive attacks


B. May need a tracheostomy with a cuffed tube
C. Needs control of spasms by regular doses of curare or
robaxine
and intermittent positive pressure ventilation
D. Usually has better chances of recovery if he survives
seven days
or more
E. Has a poorer prognosis if his urinary excretion of
catecholamines
is well above the upper limit of 49 ugm per day
111. The dangerously ill patient with clinical tetanus:

A. Has difficulty in swallowing as his presenting feature


B. Should have an endotracheal tube rather than
tracheostomy for the
control of the airway

37
C. Usually has his spasms best controlled by succinyl
choline
D. Has a mortality of at least 10-15 per cent
E. Does better if a sympatholytic agent such as bethanidine
is added
to the treatment regime

112. Gas gangrene:

A. Results from the action of saccharolytic (e.g. Clostridium


perfringens)
and proteolytic (e.g. Cl. sporogenes) clostridia
B. Is mainly the effect of the alpha-toxin of clostridium
welchii, a
lecithinase
C. Readily establishes in deep wounds of the thigh
D. Develops within 10-12 hours of injury
E. Is clinically marked by profuse clear, brownish, offensive
exudates
113. Gas gangrene:

A. Is due to the action of proteolytic clostridia


(Cl.sporogenes and
histolyticum) only
B. Is mainly the effect of collagenase and hyaluronidase,
exotoxins
produced by clostridium welchii

38
C. Readily establishes in deep wounds contaminated by
clostridia
and aerobic organisms
D. Develops 48 hours after injury
E. Invariably gives rise to tissue clostridial
114. Gas gangrene:

A. Is primarily a clostridial myonecrosis


B. Is noted to show a predilection for high thigh amputations
C. May present a picture of shock, anaemia and jaundice
D. Is best treated by extensive wound debridement, anti-gas
gangrene
serum (systemic and local) and heavy doses of penicillin
E. Is not objectively benefited by hyperbaric oxyegen

115. Erysipelas:

A. Is a cutaneous infection due to streptococcus pyogenes


B. As a lesion has a well-defined border
C. Frequently affects the skin of the face
D. Seldom suppurates
E. Usually produces no systemic symptoms
116. Erysipelas:

A. Is a subcutaneous infection due to streptococcus


pyogenes
B. Shows a predilection for the lymphatics
C. Attacks individuals whose health status is below par
D. Nearly always produces sloughing of the skin and

39
subcutaneous
tissues
E. Responds briskly to crystalline penicillin in doses of 1 to 2
mega l/M
units daily
117. Erysipelas:

A. Is a cutaneous infection due to staphylococcus


B. Shows a predilection for cutaneous appendages
C. Resolves or ends in some superficial desquamation of
the skin
D. Produces fibrosis of lymphatics thus impairing drainage
of affected
area
E. Responds particularly briskly to cefoxitin
118. Anthrax:

A. In Africa occurs in sporadic epidemics in pastoral


communities
B. Shows a predilection for the skin, the respiratory tract
and the
intestine
C. Is most contagious in its pulmonary form
D. Is mildest in its cutaneous form
E. Responds briskly to penicillin with judicious replacement
of fluid and
Electrolytes

119. Anthrax:

A. Is endemic in most pastoral communities in Africa

40
B. Shows a predilection for the lymphoreticular system
C. Is most contagious in its intestinal form
D. May lead to pulmonary oedema which is rapidly fatal
E. May be treated with tetracycline in penicillin sensitive
individuals
although less effectively
120. Clinical Rabies:

A. May be carried and transmitted by any mammalian


animal
that bites
B. Is evident in the transmitting animal within 48 hours of a
bite
C. Should cause the death of the transmitting animal within
10-14 days of a bite
D. Has an extremely variable incubation period averaging
50-60days
E. Successively produces restlessness, paraesthesiae,
hyperaesthesia,
painful muscle spasms, convulsions and generalized
paralysis
121. Clinical Rabies:

A. In Africa is carried mostly by dogs and cats


B. Is evident in the transmitting animal within 24 hours of
the bite
C. Is confirmed by the demonstration of Negri bodies in the
brain
of the suspected animal
D. Has a shorter incubation period for facial compared with
trunk bites
E. Usually kills the patient through asphyxia

41
122. Clinical Rabies:

A. Cannot be suspected from symptoms and physical signs


alone
B. Is liable to supervene in only 50 per cent of individuals
exposed
to infection owing to natural immunity
C. Is best treated by prophylactic administration of the
attenuated virus
D. Has its mortality reduced from 90 per cent to 5 per cent
by thorough
irrigation of the infected wound with soap solution
E. Is not affected in its outcome by the age and nutritional
status of
the patient
123. Primary tuberculosis:

A. Enters the body through the portals of the lung, tonsils


and
Peyers patches
B. In the lung produces a Ghon’s focus – subpleural lesion
and hilar
lymphadenitis
C. May occasionally produce military tuberculosis
D. Usually yields to the patient’s immune mechanism and
settles
spontaneously
E. Evokes an anergic response in the patient

42
124. Primary tuberculosis:

A. Usually enters the body through the skin


B. In the tonsils produces tuberculous lymphadenitis leading
to glandular
calcifications
C. Nearly always heals with calcification of the affected
tissue
D. Confers on the patient a lifelong state of hypersensitivity
to the
tubercle bacillus
E. Is no longer commonly encountered in African
populations

125. Tuberculous cervical lymphadenitis:

A. Is the result of entry of tubercle bacillus through the tonsillar


route
B. In most tropical countries is produced by the human strain of
mycobacteria tuberculosis
C. In many tropical countries presents as collar stud abscesses
D. In the early stages takes the form of firm discrete swellings
E. Should be treated in the early stages by specific chemotherapy
and supportive measures
126. Tuberculous cervical lymphadenitis:

A. Is occasionally acquired through entry of the organism from


carious teeth
B. In most tropical countries is produced by the bovine strain of

43
the organism
C. Is usually associated with active pulmonary lesions
D. In the later stages the glands are matted together through
periadenitis
E. Should be treated in later stages (sinuses) by excisional surgery
of gland and sinuses and skin grafting
127. Regarding Tuberculosis:

A. It afflicts one-third of the world’s population


B. There are Ten million new cases worldwide per annum
C. There are 3 million deaths per annum
D. Ninety five percent of cases and 98% of deaths occur in the
developing world.
E. In the developing world 75% of cases are in the economically
productive
age group (15-50)

128. The most common route of primary infection with


Mycobacterium
Tuberculosis is:
 Directly to the lung establishing a Primary Complex
(Gohn’ focus)
 Through the Tonsils to the lymph nodes of the neck
 Via the Payers patches of the lower ileum to the mesenteric
lymph nodes
of the ileo-caecal angle
 The genitourinary mucosa – urethra and bladder
 The peritoneum via the terminal ileal mucosa

44
 Through the joints and long bones in early septiaemia
129. Primary Pulmonary Tuberculosis:

A. Usually yields to the patient’s immune mechanism


B. Frequently proceeds to systemic dissemination resulting in Miliary
Tuberculosis
C. In most cases leaves the patient with a positive reaction to
extracts
of the mycobacterium
D. Is often complicated by a lung abscess
E. Is more frequently encountered in patients with S.C Disease
130. Tuberculous Cervical Lymphadenitis:

A. Is the result of entry of Mycobacterium Tuberculosis


through the upper oesophageal mucosa
B. Is confirmed by demonstrating the lesion in the tonsillar mucosa
C. May arise from consumption of unpasteurized milk
D. Currently more often arises from the human strain of
Mycobacterium
Tuberculosis
E. Follows a rapidly progressive course of illness.

131. The outcome of Tuberculous Cervical Lymphadenitis may be:

A. Resolution
B. Calcification
C. Fibrosis with matting together
D. Haemorrhagic infarction
E. Caseation and formation of a Collar-stud abscess

45
132. A Ghanaian male of 30 presents with a painless cervical
swelling below
the level of the angle of the mandible of six weeks duration. He
admits
to a period of anorexia, malaise and weight loss but denies
coughs and
night sweats. The swelling is firm, discrete, and non tender.
There are
no other cervical swellings:

The most likely diagnosis is:


 Metastatic carcinoma of the pharynx
 Hodgkin’s lymphoma
 Pyogenic subacute lymphadenitis

 Tuberculous Cervical lymphadenitis


 Mixed Parotid Tumour
133. A man of 60 presents with a progressive painless cervical
swelling of
4 months duration. He has had a non productive cough for a
month but
has not experienced pyrexia or night sweats. The swelling is
soft, fluctuant
and non tender. There has been no response to a week’s
exhibition of
ciprofloxacin and metronidazole.

The most likely diagnosis is:

A. Bronchial cyst
B. Degeneration of lymphatic metastases
C. Suppurative lymphadenitis

46
D. Collar stud Abscess (Cold Abscess)
E. Cervical Dermoid
F. Lipoma
G. Cystic hygroma
134. A man of 70 presents with a chronically discharging cervical
sinus at the
level of the angle of the mandible, of two months duration. He
admits to
recent coughs with rusty spectrum but denies pyrexia and night
sweats.

The most likely diagnosis is:


 Actinomycosis
B. Malignant lymphoma
C. Acquired branchial fistula
D. Tuberculous sinuses from Tuberculous lymphadenitis
E. Degenerating chemodectoma
135. Notwithstanding advances in diagnosis and chemotherapy of
Tuberculosis, the World’s TB problem is more daunting than
ever.
The root cause for this state of affairs is now recognized as:

 Lack of effective treatment agents


 Lack of control schedules in the context of National Tuberculosis
Programmes
 Profusion of antituberculous agents
 Inappropriate surgical intervention
 Undue emphasis on nutrition
136. The management of a patient with a diagnosis of tuberculosis
must be
preceded by a standardized case definition which is determined

47
by:
 Site of TB lesion: pulmonary or extra pulmonary
 Result of sputum smear: positive or negative
 Previous anti TB treatment
 Severity of TB infection
 Appearance of Resistant Strains

137. The aims of antituberculous chemotherapy are:

A. Cure the patients quickly with minimum interference with their


lives
B. Prevent death from active disease or its late complications
C. Prevent relapse
D. Prevent emergence of acquired resistance
E. Protect the community from transmission of infections
138. The most important innovation in the management of
Tuberculosis
in recent years is:
 Emphasis on Nutrition
 Emphasis an Ambiance – Hygiene and Sunlight
 Introduction of Rifampicin
 Introduction of Policy of Directly Observed Therapy
 Close Supervision policy
139. In the use of anti-tuberculous drugs the following

observations are true:

A. Most of the agents are eliminated by metabolic degradation


through the liver
B. The safest drugs in patients with renal failure are Isoniazid and

48
and Rifampicin
C. The most effective Drugs are Isoniazid and Rifampicin
D. Isoniazid, Rifampicin Pyrazinamide and Ethambutol are
recognized as hepatoxic
E. Streptomycin is potentially Ototoxic and causes deafness in
babies - best
avoided.
 Cross-reaction of HIV therapy with Rifampicin may be life
threatening

140. Actinomyces Israeli:

A. Are Gram +ve aerobic organisms


B. Have Gram +ve filamentous hyphae
C. Occur as commensals in the normal mouth
D. Are often associated with carious teeth
E. Have a predilection to bones, joints, and skeletal muscle
141. Actimonycosis:

A. Is produced by the anaerobic Gram positive fungus,


Actinomyces Israeli
B. Is characterized by chronicity and production of nodular
granulomata
C. May be confirmed by identification of “sulphur granules” in the
in the exudate
D. Most commonly occurs in the cervicofacial region
E. Readily responds to crystalline penicillin in doses of 6-10 million

49
units daily for 6 weeks

142. Actinomycosis:

A. Is produced by the microaerophilic Gram negative fungus,


Actinomyces
Israeli
B. Seldom spreads along lymphatics to the lymph nodes
C. Is readily confirmed in any lesion by culture of the organism
D. Shows a predilection for the cervicofacial, thoracic, hepatic and
ileo-caccal sites
E. May in recalcitrant cases respond to tincture of iodine in doses
of 0.3 ml t.d.s.

143. The typical actinomycotic lesion:

A. Is a granuloma
B. Is firm and nodular
C. Has indefinite edges
D. Usually spreads along fascial planes
E. Seldom spreads along lymphatics to lymph nodes
144. Actinomycotic infections:

A. Are characterized by chronicity


B. Show few systemic manifestations
C. Are often marked by anorexia and weight loss
D. Can be confirmed by identification of the organism in the
in the discharge

50
E. Show a predilection for the face, chest and the liver
145. Cervico-facial actinomycosis:

A. Is the commonest manifestation of the disease


B. Most frequently arises from the buccal mucosa
C. Causes induration of the skin over the mandible but sinuses are
rare
D. Seldom shows evidence of secondary infection
E. Should not be diagnosed without the characteristic solphur
granules.
146. Thoracic Actinomycosis:

A. Occurs mostly through the respiratory tract


B. May result by direct invation from the neck
C. Has resulted from transdiaphrgmatic migration of hepatic lesions
D. Usually spares the lungs and pleura
E. Is frequently complicated by empyema
F. Seldom shows on chest x-rays
G. In the early stages cannot be distinguished from other chronic
chest infections

147. In Hepatic Actinomycosis:

A. The organism reaches the liver via the portal vein, hepatic artery
or from contiguous pulmonary lesions through the diaphrgram
B. There is rapid liver necrosis
C. The liver is riddled with numerous abscesses
D. There is progressive cachexia
E. The diagnosis is usually made at laparotomy
F. Jaundice is invariable at presentation.

51
148. In Ileocaecal actinomycosis:

A. The organism gains access to deeper layers of the intestine


through
mucosal breaches
B. There is granulomatous infiltration of the abdominal wall muscles
C. The mesenteric lymph nodes are usually not involved
D. The presentation may be indistinguishable from acute appendicitis
E. Distinction from ileocaecal tuberculosis or caecal carcinoma is
usually
Impracticable
F. The diagnosis is often made at laparotomy
149. Yaws:

A. Is produced by an organism which morphologically and


antigenetically is indistinguishable from Treponema pallidum
B. Is showing indications of resurgence in many developing
countries
C. In its primary form is usually extragenital
D. In its secondary form is heralded by rashes as constitutional
symptoms of the primary lesion subside
E. Is best treated by a course of Penicillin aluminium monostearate

150. Yaws:

A. Is produced by Treponema pallidum


B. Is now effectively under control in most developing countries
C. In its primary form may occur anywhere in the body
D. In its secondary form is characterized by successive crops of

52
pleomorphic rashes
E. Responds equally to neosalvan, bisttmuth and P.A.M.
151. Yaws:

A. In its primary form shows a predilection for the leg


B. In its primary form has an incubation period of 3-4 weeks
C. Is usually productive of a positive Wassermann reaction within
3-4 weeks of the primary lesion
D. In its secondary form is characterized by rashes symmetrically
distributed in the body
E. Responds equally to procaine-penicillin, P.A.M. (WHO),
chloramphenicol and tetracycline
152. Tertiary yaws:

A. Is notable for its stereotyped manifestations


B. May show as scaly dermatitis of the palms
C. Is a well-known cause for a fibrosing splitting lesionof the heel
of the foot in the tropics
D. Is responsible for that disfiguring deformity of the face called
Leontiasis ossea
E. Is recognized cause for pathological fractures of the long bones

153. Tertiary yaws:

A. Is notable for its varied manifestations


B. May show as dactylitis of the phalanges, paronychia and nail
atrophy

53
C. Is responsible for destructive lesion of the hand and soft palate
and the nasal cartilages and bones
D. Is responsible for that deformity of the face called Goundou
E. Usually spares the long bones of the body
154. In Yaws (Framboesea):

A. The causative organisms is the Spirochete Treponema pertenue


B. The organism is morphologically and antigenetically
indistinguishable
from Treporema pallidum.
C. The organism is innoculated on to an abrasion or ulcer.
D. The primary lesion is extra genital and usually on the leg
E. There is evidence of cross immunity with syphilis
155. In Primary Yaws:

A. The lesion is extra genital, usually on the leg


B. The initial lesion is painless
C. The enlarging lesion is covered by a scab-mama pain
D. Constitutionally there is little or no disturbance
E. There is no lymphadenopathy

156. In Secondary Yaws:

A. Rashes appear as the constitutional symptoms of the primary


lesion subside
B. A succession of Rashes starts about 3 months from the primary

54
lesion
C. The rashes differ from syphilis in being asymmetrical in distribution
D. By auto innoculation older lesions are surrounded by new
generation
of daughter papules
E. The typical yaw when fully developed is an escresence with
yellowish
crust raised 0.5-2cm above the skin
F. Healing of the lesions leaves pigmented spots especially on the
palms and hands
157. The Lesions of Tertiary Yaws include:

A. Gross scarring and disfigurement of the leg


B. Dactylitis and swelling of the phalanges of the hands
C. Dense fibrosis of the sole with painful fissures producing a
crab-like gait
D. Gummatons involvement of nasal processes of the maxilla with
paranasal swellings
E. Periostitis of the long bones – Tibia, periosteal nodes
F. Deformity of the long bones – sabre-shaped deformity
158. In the treatment of Yaws:

A. Procaine Penicillin is the drug of choice


B. Given in doses of 1.2 mega unit stat, then 600,000 units daily
for six days
C. Penicillin aluminum monostearate is longer acting and is favoured
by WHO
D. Erythromycin has been used in penicillin sensitive subjects
E. Community monitoring is essential to avoid recrudescence

55
159. Gonorrhoea:

A. Is the most commonly encountered venereal disease in most


tropical countries
B. Shows important differences in its manifestation in the two sexes
C. Usually produces only mild or minimal systemic symptoms
D. Is more likely to establish in the anterior rather than posterior
urethra
E. Affecting the prostate is liable to result in attacks of acute sharp
pain
160. Gonorrhoea:

A. Is apparently decreasing in incidence in most developing


countries
B. Shows no sex differences in its manifestations
C. In the male shows equal tendency to establish in anterior or
posterior urethra
D. Affecting the seminal vesicles may lead to painful nocturnal
emissions
E. Is a common cause for genital warts
161. Gonorrhoea:

A. In the female is completely asymptomatic in 50 per cent of


individuals
B. In the female shows a predilection for the cervix uteri, urethra
and
Bartholin’s glands
C. As a cause for urethral stricture may affect any part of the
male urethra
D. May be treated with a single dose Kanamycin 2G I.M
E. Treated with large doses of penicillin tends to mask associated
syphilis

56
162. Non-gonococcal urethritis:

A. Is now almost as serious a problem as gonorrhoea


B. Is increasing in incidence because of indiscriminate use and
inadequate dosage of antibiotics
C. Is known to have a longer incubation period than gonococcal
urethritis
D. Is diagnosed by exclusion of gonorrhea by repeated cultures
E. Is best treated with tetracycline 500 mg q.d.s. for 5 days
163. Non-gonococcal urethritis:

A. Is seldom encountered in urban tropical populations


B. Is often presumed to arise from TRIC viruses, and chlamydia
trichomatis
C. Has an incubation period of 1-4 weeks
D. Is free of the cicatrizing complications of the urethra
E. May in refractory cases be treated by furacin urethral supp
ositories
164. Gonorrhea in the male:

A. Has an incubation period of 14 days


B. Shows first as irritation and congestion of the urethral meatus
C. Is marked by severe systemic symptoms in most patients
D. Often shows spontaneous relief of symptoms within a fortnight (14
days)
E. In most cases shows haziness in both glasses in the two glass
test
165. Posterior urethritis in gonorrhea affecting the male:

A. Is usually late in onset


B. May be precipitated by ill-advised instrumentation during treatment
C. Is marked by urgency and terminal haematuria

57
D. Is associated with painful nocturnal erections
E. Is confirmed by the finding of haziness in both glasses in the
two glass test
F. May show as an exquisitely tender and fluctuant prostate

166. In Gonococcal urethritis in the male:

A. Chronicity is a tendency seen in posteriorly placed lesions


B. Persistent discharge is common usually from prostatitis and
vesiculitis
C. The site of predilection for stricture formation is the peno-bulbar
junction.
D. The site of predilection for stricture formation is the anterior
urethra
E. A sign of chronicity is warts on glans and prepuce
F. Systemic complications may take the form of arthritis,
conjunctivitis
and iridocyclitis
G. Systemic` complications may take the form of endocarditis and
pyaemic
absceses
167. In the treatment of Gonococcal infection:

A. Penicillin and Streptomycin combination is effective


B. Ceftriaxone 250mg 1/M once is the drug of choice but follow up is
essential
C. Patients undergoing treatment should abstain from sexual activity
for a minimum of one week
D. There need be no serious restriction on use of alcohol
E. Serological tests to detect masked syphilic infection is no longer
necessary
F. All treated patients should be followed up for a minimum period of

58
one month
168. Gonococcal infection in the female:

A. Is completely asymptomatic in about 50% of patients


B. May only show as vaginal discharge
C. Is frequently encountered as a Bartholin abscess
D. Is seen most frequently on the cervix uteri

E. Is more likely to be diagnosed from culture-than examination of


stained smears
F. Is most commonly complicated by pelvic inflammatory disease
169. Non-gonococcal Urethritis:

A. Is mostly produced by Chlamydia trachomatis


B. Is a problem produced largely from indiscriminate use of antibiotics
C. Has a much longer incubation period than gonococcal urethritis
D. May occasionally be attributable to viruses and mycoplasma
E. Treatment is difficult to assess because the disease is often self
limiting
F. Ofloxacin has been shown to be effective
170. Syphilis:

 In its primary form occurs most commonly in the genital region


as a firm rounded ulcer
 Presenting as a primary chancre lacks the hardness and friability
of
a malignant ulcer
 In its primary form evokes little or no systemic reaction
 In its secondary form produces rashes with are typically
symmetrical
and free from itching

59
 In its primary or secondary form is best treated with procaine
penicillin in doses of 600,000 units daily for 15 days

171. Syphilis:

 In its primary form occurs most commonly as an extragenital


indurated ulcer
 Presenting as a primary chancre has the hardness and friability
of
a malignant ulcer
 In its primary form evokes marked regional lymph node
enlargement
which is firm, non-tender and discrete
 In its secondary form produces pustular irregularly distributed
rashes
which are often itchy
 May in penicillin sensitive individuals be treated with erythromycin
in doses of 500 mg 6 hourly for 20 days
172. Tertiary syphilis:

A. Is characterized by the basic lesion known as the gumma


B. Leads to progressive destruction of the elastic tissue of the aorta

60
C. May present as isolated cranial nerve palsies
D. Is best treated by a course of penicillin aluminium monostereate
1.2 mega units twice weekly for seven weeks
E. Is not attended by reversion of serological tests to normal
after treatment
173. Tertiary syphilis:

A. May produce lesions which if not treated may become malignant


B. Produces intimal proliferations at the ostia of arteries arising
from the ascending aorta
C. Always need to be confirmed serologically prior to treatment
D. Is best treated by a course of crystalline penicillin 1 mega unit
daily for seven weeks
E. Is recognized cause for pathological fractures of long bones of
the
lower limbs

174. In congenital syphilis:

A. The more recent the infection in the mother the greater the
probability and severity of the disease in the infant
B. Diagnosis of the early lesion needs to be confirmed by isolation
of
Treponema pallidum from the mucosal and skin lesions
C. Radiological changes in the skull and long bones are usually
present
D. The neurological and cardiovascular changes occur about the
time
of puberty
E. Treatment should primarily be through prophylactic treatment of
the

61
mother
175. In congenital syphilis:

A. Serological tests are positive in overt cases


B. Positive serological tests in cases showing no clinical evidence of
the disease may be due to maternal reagins
C. Positive serological tests after the age of 6 weeks must be
accepted
as valid evidence of disease
D. Infants should be treated with doses of 150,000 units of procaine
penicillin daily for 15 days
E. Subsequent development is not affected by any mode of
treatment
176. Treponema Pallidum:

A. Is serologically currently indistinguishable from Treponema


Pertenue
B. Does not survive outside the body even when moist
C. Can now be readily cultured in specific media
D. Can be identified by dark ground illumination
E. Enters the body through the skin, mucous membranes and the
foetal placenta

177. In Primary Syphilis:

A. The Primary chancre is located on the frenal region under the


prepuce in the male most commonly
B. The Primary chancre is located most commonly on the mons pubis
in the female
C. The Primary chancre is commonly located on the labium minora,
clitoris, or cervix in the female

62
D. The Primary lesion does not show contact bleeding
E. There is marked regional lymphadenopathy
F. There is little or no systemic reaction to the infection
178. In Primary Syphilis:

A. Extra genital lesions are commonly encountered on the lips, oral


cavity, fingers and perineal areas
B. The Diagnosis is dependent on demonstrating the organism by
dark
field microscopy of a wet specimen
C. The VDRL Test is important but needs to be repeated fortnightly
over
a 3 month period
D. The fluorescence Treponmal Antibody Absorption Test
(FTA-ABS) becomes positive before the VDRL
E. Malignant Syphilis is encountered in HIV infected patients
F. The CD4 count may be as low as 200

179. In Secondary Syphilis:

A. The particular features show mostly in the skin and mucous


membranes
B. The cutaneous lesions are macular, papular and maculo-papular
C. Lesions are characteristically symmetrically distributed in the body

63
D. Itching is intense
E. Serological tests are usually positive and prove useful in differential
diagnosis
F. The mucosal lesions invade deeply the cheek and bucal mucosa
with risks of secondary haemorrhage.
G. Raised flat topped condyloma appear at mucocutaneous junctions

anal, vula, mouth
H. Generalised lymphadenopathy is common
I. Lymphadenopathy is symmetrical
180. In the Management of Primary and Secondary syphilis:

A. The antibiotic of choice is Ofloxacin


B. The antibiotic of choice is Penicillin delivered in Depot preparations
for 15 days
C. Erythromycin 500mg 6hourly is given in penicillin sensitive patients
for 20 days
D. Serological monitoring is essential for at least 2 years.
E. Persistent positive reaction calls for repetition of antibiotic regime.
F. The CSF Should be normal before discharge from surveillance

181. In Tertiary Syphilis:

A. The basic lesions are evident within a year of onset of the disease
B. The basic lesion is an extensive necrosis bordered by epitheliod

64
cells
C. The basic lesion is the gumma, a necrotic granuloma
D. The necrotising lesions leave ulcers that are typically under mined
E. The necrotizing lesions produce punched out ulcers
F. The resulting ulcers heal leaving tissue paper thin scars
G. The cutaneous lesions show a predilection for the bony areas of
the leg and forearm
H. The tongue may show superficial atrophic glossitis
I. The tongue may show interstitial glossitis with leukaplakia
J. Perforation of the palate and nasal septum may occur
182. In Tertiary Syphilis:

A. Intimal proliferation may lead to narrowing of the ostia of the


coronary
arteries
B. Atherosclerosis is enhanced leading to aortic aneurysm formation
C. Progressive destruction of elastic tissue leads to dilatation of the
aortic ring
D. The characteristic neurological disturbance is a dissociated sensory
loss
E. Assessment of progress is based on clinical rather than serological
findings
F. Serological tests revert to normal within a year of completion of
treatment regime

183. Congenital Syphilis may take the form of:

65
A. Abortion in the first trimester
B. Still birth at term
C. Premature labour with maceration of the foetus
D. Normal child developing signs within first few weeks at home
E. Normal child developing signs in later childhood
F. Child without clinical signs or serological changes
184. Early Congenital Syphilis:

A. Is usually diagnosed within 5 days of birth


B. Follows the pattern of primary syphilis
C. Is evidenced by patchy excoriations and condylomata affecting the
mucoperiostium of the nose
D. Typically spares the skin in its manifestations
E. Has no visceral abdominal manifestations
F. Can be confirmed using serological tests done within two weeks
of birth

185. Late Congenital Syphilis:

A. Develops from the age of puberty or after 15 years


B. In the eye shows as vitreous haemorrhages and retinal exudates
C. In the ear presents with Tinnitus and Vertigo
D. May present as a painless arthropathy of the larger joints
(Clutton’s joints)
E. May produce changes in the cusp patterns in the lower molar teeth

186. Genital herpes:

66
A. Is caused by the common herpes virus
B. Is encountered in both sexes
C. Is characterized by itching in the glans as the first symptom
D. Usually presents as an area of vesicular rashes and excoriations
of
the glans and prepuce
E. Is treated with sulphadimidine 1G qds for 5 days and topical
1 per cent hydrocortisone cream
187. Genital Herpes:

A. The Causative agent is the Human Papilloma Virus HPV Type l


B. Is marked by itching as the first symptom
C. Shows as gross oedema of the penis and scrotum
D. Is distinguished from other genital lesions by the attendant severe
pain and tenderness
E. In the mother carries no significant risks for the neonate

188. Lymphogranuloma venereum:

A. Is caused by Chlamydia trachomatis types L1-L3


B. In its primary form produces a rather transitory herpetiform
lesion
C. In its secondary form presents 2-6 weeks after exposure
D. May in the male produce gross tender lymphadenopathy prone
to
breakdown in the formation of discharging sinus
E. Is in the female liable to produce vulval lymphoedema with
infection

67
189. Lymphogranuloma venereum:

A. May present with a history of the primary sore in a third


of all cases
B. May in the male produce sufficient lymphatic obstruction to result
lymphoedema of the scrotum
C. Is in the female liable to produce rectal stricture
D. Is more reliably confirmed by the lymphogranuloma venereum
complement fixation test
E. Is best treated with tetracycline 500 mg q.d.s. for 7 days
190. Lymphogranuloma Venereum:

A. Is a venereal disease produced by Chlamydia trachomatis


serotype
L1-L3
B. Presents in Primary and Secondary phases
C. In the Primary form is rather transitory
D. In the Primary phase may take the form of an urticurial rash.
E. In the Primary phase may take the form of herpetiform vesicles
F. May take the form of a superficial genital sore
191. Lymphogranuloma Venereum in the Male:

A. Is due to infection with Chlamydia trichomatis


B. In its secondary phase shows only slight systemic disturbance
C. Is evidenced by gross tender inguinal lymphadenitis
D. Is marked by severe rectal symptoms
E. Is the cause for some cases of lymphoedema of the scrotum
F. May result in colliquative necrosis of the inguinal lymph nodes with
discharging sinuses

68
192. In Lymphogranuloma Venereum in the Female:

A. The organism is Chlamydia trachematis sero type L1-L3


B. The Primary lesion is usually located high in the vagina
C. The Primary lesion is usually located on the cervix
D. The Primary lesion spreads to the para rectal tissue
E. The earliest symptoms are due to associated proctitis
F. The patient passes blood, pus and mucus in the stools
G. There is associated perianal pruritus and tenesmus
H. Constipation with passage of pencil stools is common
I. A Tubular rectral fibrous stricture is a common complication
J. Vulval lymphoedema, with dermatosis and ulceration may occur
K. Lymphogranuloma venereum complement fixation test is more
sensitive than the Frei test
193. In the Frei Test for lymphogranuloma venereum:

A. The test is read at the end of 24h.


B. A papular reaction at least 4mm indicates a positive result
C. A control injection in the opposite forearm of physiological saline
is essential
D. The test is usually specific
E. All Chlamydial infections give a positive reaction
F. A positive reaction takes a long time to develop after infection
G. A positive reaction persists throughout life.
H. Lymph node biopsy provides confirmatory evidence

69
194. In the treatment of lymphogranuloma venereum the preferred

option is:

A. Sulphadimidine 1G qds for 10 days


B. Penicillin (aluminuim monostearate) 1.2 mu twice weekly for
seven weeks
C. Clindamycin 600mg I/M daily for one week
D. Ofloxacin 200mg I/M for 24 hours only.
E. Tetracycline 500mg qds for 10 days
195. In the management of lymphogranuloma venereum:

A. The place of surgery is limited


B. Fomentation of bubos should be done to induce fluctuation
C. Aspiration through the area of pointing in the skin is preferable
D. Incision through the point of maximum tenderness is the gold
standard
E. Attendant rectal stricture should be treated by colostomy
F. Attendant rectal stricture is best treated by dilatation
196. The Human Immunodeficiency virus:

A. Was discovered in 1986


B. Has a predilection for skin and subcutaneous tissue
C. Shows low concentration in CSF and synovial fluid
D. Is more likely to be found in pooled blood products (Factor VIII)
concentrate)
E. Has its lowest occurrence rate in single-donation (Packed RBCS)
F. Internationally contamination of donated blood has been reduced
by
80 percent through screening.

70
197. In HIV AIDS infection:

A. The asymptomatic antibody negative person may still transmit the


infection
B. Sero conversion takes about 3 weeks
C. The virus targets polymorpho-nuclear neutrophils
D. The virus targets neutrophils and lymphocytes equally
E. The virus targets T4 helper lymphocytes
F. Persistent Generalized Lymphadenopathy (PGL) is usually
asymmetrical
G. The mean duration of persistent Generalised lymphadenopathy is
3 months
I. For lymphadenopthy to be significant it must be noted in at least 3
anatomical sites
198. The AIDS Related Complex (ARC) in HIV/AIDS infection is
diagnosed in
which of these situations?

A. Severe malaise and lethargy

B. Weight loss of more than 10% of body weight


C. Unexplained diarrhoea for > one month
D. Leukoplakia
E. Skin rashes
 Oral thrush
 Night sweats for more than a month
 Unexplained fever for more than a month

71
 Persistent Generalised lymphadenopathy

199. The neoplastic diseases linked to the HIV/AIDS status are:

A. Carcinoma of the colon


B. Primary liver cell cancer
C. Carcinoma of the bronchus
D. Kaposi sarcoma
E. Osteogenic sarcoma
F. Cervical cancer
G. Extranodal high grade B cell lymphoma
H. Melanoma
I. Medullary Thyroid carcinoma
200. The commonest neoplastic disorder seen in HIV infection is:

A. Carcinoma of the bronchus


B. Carcinoma of the large bowel
C. Leukaemia
D. Kaposi sarcoma
E. Primary liver cell cancer
201. HIV Associated Kaposi sarcoma differs from endemic Kaposi

sarcoma

in which of these features?

 The distribution in the HIV related lesion is centripetal

72
 There are symmetrical facial plaques
 There is often leg oedema and pulmonary infiltrates
 The lesion is mainly visceral in HIV in patient
 The Human Herpes Virus 8 (HHV8) is encountered only in the HIV
associated kaposi sarcoma.

202. Regarding HIV transmission in surgical procedures:

A. Routine testing of all patients cuts down the risks to the surgeon
B. It is pragmatic to assume that the risk in all procedures is significant
C. Wearing masks and goggles when using endoscopes and bone
and
dental drills has no protective value
D. Skin Contact with body fluids carries little risk
E. Wound irrigation entails no risk to the health attendant
203. The major risk factors of occupationally acquired HIV infection
include
the following except:
 Puncture Wound
 Visible blood on the injuring device
 Placement of hollow needle in a vessel
 Working on patients with late stage HIV
 Contact with the intact skin.
204. The Recommendation for post-exposure prophylaxis (PEP) for
health care
workers include:

73
 Action should be instituted within 2- 4h of injury
 Washing skin with soap and water
 The contaminating source is tested for HIV
 In HIV +ve Class 1 source exposure the 2 drug post exposure
prophylaxis
is advised (zidovudine + lamivudine)
 In HIV +ve Class 2 source exposure the 3 drug post exposure
prophylaxis
is advised (zidovudine, lamivudine and indinavir)
 In mucous membrane exposure large volume exposure requires a
two
drug post exposure prophylaxis
 HIV Tests of the worker should be done at 6 weeks, 3, 6 months
and
1 year

205. To diagnose Clinical Aids the minimum requirements are:

A. 1 feature of ARC and 1 demonstration of opportunistic infection


B. 2 features of ARC and 1 demonstration of opportunistic infection
C. 3 features of ARC and 2 demonstrations of opportunistic infection
D. 2 features of ARC and 2 demonstrations of opportunistic infection
E. 3 feature of ARC and 1 demonstration of opportunistic infection
206. Antibiotics:

A. Must not be given in acute surgical conditions until the diagnosis


has been established
B. Used therapeutically need to be given over a minimum period,
usually about 4 days
C. Are in general more effective in controlling established infections

74
than they are in preventing them
D. Which are bactericidal generally act by dissolution of the cell
walls
of the organisms
E. Should always be given with vitamin supplements in all patients
207. In the general use of antibiotics in surgery:

A. The agent must be with held until a reasonably firm diagnosis


has been made
B. Antimicrobial agents are more effective in the cellulitic phases of
a
pyogenic infection
C. The most effective combination of agents must be given in
adequate
doses for at least 5 days
D. The Bacteriocidal agents act by destruction of the bacterial cell
wall
E. The Bacteriostatic agents act by competing with the organism for
substrate

208. In which of the following surgical procedures is prophylactic use


of the
antibiotics justified:
A. Laparotomy for perforated duodenal ulcer
B. The operation of Truncal Vagotomy and Pyloropasty
C. Surgical treatment of strangulated inguinal hernia
D. Hydrocelectomy for a lesion of several years standing
E. Total mastectomy with axillary clearance for Stage II breast cancer

75
 Appendicectomy for acute appendicitis
 Appendicectomy for perforated appendicitis
 Appendicectomy for gangrenous appendicitis
 Splenectomy for trauma in a 40 year old man
 Repair of tendons severed by a Kitchen knife
209. In the prophylactic exhibition of antibiotics:

A. A broad spectrum bacteriocidal agent is desirable


B. The ideal route is the oral route
C. Graduated doses are desirable
D. The agent should ideally be delivered at induction of anaesthesia
E. The antibiotic exposure should be for at least 36h
210. The ideal theatre design:

A. Is currently the modular pattern


B. Ensures that the suite is independent and in a cul-de-sac
C. Puts emphasis on floor to ceiling tiling
D. Puts a premium on the plenium type ventilation
E. Does not include sterilization equipment in the suite

211. The ideal theatre design:

A. Ensures that ease of access is of overriding importance


B. Makes for a clear demarcation of sterile from non-sterile areas
of the operating room
C. Avoids wall shelves

76
D. Has a preference for the unit air conditioner
E. Makes provision for sterilization facility within the suite
212. Which of these organisms is noted to be a particularly
dangerous cause
for epidemic nosocomial infection:

A. Β-haemolytic streptococcus pyogenes Lancefield Group A


B. Klebsiella
C. Haemophilus influenzae
D. Staphylococcus pyogenes phage type 80-81
E. Pseudomonas aeroginosa
F. Streptococcus pneumoniae
G. Proteus vulgaris
H. Bacteroides frogilis
I. Esch. coli.
213. Regarding the epidemiology of nosocormial infections:

A. Primary incisional surgical site infections are the commonest –


45%
B. Urinary tract infections are responsible for 40% of cases
C. Respiratory tract infections account for 10-15%
D. Blood stream infections are increasingly associated with parenteral
nutrition.
E. Surgical intensive care units have a higher incidence of pneumonia

214. The most important factor determining the genesis of Surgical

Site

77
infection (SSI) is:

 Age of the patient


 Duration of the operation
 Ward setting of the patient
 Skin preparation for surgery
 Bacterial content of the wound at end of operation
215. Which of these factors have been shown to significantly affect
the
development of Surgical Site infection:

A. Choice of skin disinfectant used


B. Glove puncture during surgery
C. Whether suture material is absorbable or non absorbable
D. Pre-operative showering for patient
E. Duration of the operative procedure
216. The features that discourage nosocomial infections in the

design of operating theatres are:

 Location in a cul-de-sae
 Separation of sterilization from theatre suite
 Elimination of shelves
 Tiled theatre walls
 Plenum ventilation with air changes of 20 per hour
 Temperature range of 18-25°C
 Humidity of 55-65%
 Physical barriers protecting the clean areas of the theatre

78
217. Antibiotics:
A. Should be recognized as adjuncts and not substitutes for
indicated
surgical therapy
B. May considerably mask the signs of infective processes
C. Are more effective in the early cellulitic and hyperaemic stages
of infection
D. May be given prophylactically at the time of drainage of
abscesses
E. Used therapeutically must be assessed continuously with respect
to development of side effects

CHAPTER 2

79
MICROBIAL INFECTIONS
ANSWERS
1. A, B, C 38. B 75. D
2. A1, B2, C3, D4, E5 39. B 76. A, B, C, D
3. B 40. A, B, C, D, E 77. A, B, C, D,
E
4. A, B, C, D, E 41. B 78. B
5. A 42. A, B, C 79. C
6 C 43. A, B, C, D 80. B

7. C 44. A1, B2, C3, D4, E5 81. A, B, C


8. A 45. A, B, D 82. A, B, C, D,
E
9. A, C, D, E 46. A, B, C, D, E 83. A, B, C, D,
E
10. A, E 47. A, B, C, D, G 84. A, B, E, D,
E
11. C 48. A, C, E 85. B, C, D, E

12. A, B, C, D, E 49. E, F 86. A, B, C, D,


E
13. A, B, C, D, E 50. A, D, E 87. A, B, C, D

14. C, D, E 51. A, B, C, D, E, F 88. A, B, C, D

15. C, D, E 52. A, B, C, D, E, F 89. C


16. A, B, C 53. A, C, E 90. C

17. A, B, C, D 54. A, C, D 91. D

18. A, C 55. A, B, C, D, E 92. A

19. C 56. A, B, C, D, E 93. A, C, D, E

20. B, C 57. B, C, D, E 94. A, B, C, D,


E
21. A, B, C, D, E 58. A, B, C, D, E 95. A, C, D, E

22. C, D, E 59. B, D, E 96. B, C, D, E


23. A, B, C, D, E 60. D 97. A, B, C

80
24. A, B, C, D, E 61. E 98. A, C, D
25. A, B, C, D, E, F, G, H, I 62. D 99. A, B, C, D,
E
26. A, B, C, D, E, F, G 63. A, B, C, D 100. A, C, D, E

27. B, C, D, E 64. A, B, C, D, E, F 101. C


28. B, C, D, E, F 65. E 102. A, B, C, D,
29. A, C, D, E 66. A, B, C, E, F, G 103. A, B, D
30. A, C, D, E 67. A, B, D, E 104. A, B, C, D,
E
31. A, B, C, D, E 68. C, E, F, G 105. A, B, C, D,
E
32. A, B, C, D, E 69. B 106. A, B, C, D,
E
33. A, B, C, D, E 70. B 107. A, B
34. A, B, C, D, E 71. A, B, C, D 108. A, C, D, E
35. A, B 72. A, B, C, D, E, F, G, H 109. A, E
36. A, B, C, D, E 73. A, B, C, D, E 110. A, B, C, D,
E
37. B 74. A, B, C, D, E, F, G, H 111. D, E

112. A, B, C, D, E 150. C, D 188. A, B, C, D, E


113. C 151. A, B, C, D 189. A, B, C, D, E
114. A, B, C, D 152. B, C, E 190. A, B, C, E, F
115. A, B, C, D 153. A, B, C, D 191. A, B, C, E, F
116. B, C, E 154. A, B, C, D, E 192. A, B, C, D, E,
F, G, H, I, J, K
117. C, D 155.A, B, C, D 193. C, F, G, H
118. A, B, C, D, E 156. A, B, D, E, F 194. A
119. D, E 157. A, B, C, D, E, F 195. A, F
120. A, B, C, D, E 158. A, B, C, E 196. A, D, E
121. A, C, D 159. A, B, C, D, E 197. A, E
122. B, C, D 160. D, E 198. A. B, C, D, E, F,
G, H, I
123. A, B, C, D 161. A, B, C, D, E 199. D, F, G
124. B, C, D 162. A, B, C, D, E 200. D
125. A, B, C, D, E 163. B, E 201. A, B, C
126. A, D, E 164. B, D 202. B
127. A, B, C, D, E 165. A, B, C, D, E, F 203. E

81
128. A 166. A, B, C, D, E, F, G 204. A, B, C, D, E,
F, G
129. A, C 167. B 205. B
130. C, D 168. A, B, C, D, E, F 206. A, B, C, D
131. A, B, C, E 169 A, B, C, D, E 207. A, B, C, D, E
132. D 170. A, B, C, D, E 208. C, F
133. D 171. C, E 209. A
134. D 172. A, B, C, D, E 210. A, B, C, D, E
135. B 173. A, B, C, E 211. B, C
136. A, B, C, D, E 174. A, B, C, D, E 212. D
137. A, B, C, D, E 175. A, B, C, D 213. A, B, C, D, E
138. D 176. A, D, E 214. E
139. A, B, C, D, E, F 177. A, C, D, E, F 215. E
140. A, B, C, D 178. A, B, C, D, E, F 216. A, B, C, D, E,
F, G, H
141. A, B, C, D, E 179. A, B, C, E, H, I 217. A, B, C, D, E
142. B, D, E 180. B, C, D, E, F
143. A, B, C, D, E 181. C, E, F, G, H, I, J
144. A, B, C, D, E 182. A, C, E
145. A 183. A, B, C, D, E, F
146. A, B, C, E, G 184. C
147. A, C, D, E 185. D
148. A, B, C, D, E, F 186. A, B, C, D, E
149. A, B, C, D, E 187. B, D

82
83
84
CHAPTER 4

INFECTIONS BY NEMATODES AND TREMATODES


1. Lymphatic dwelling filarial worms include:
A. Onchocerca volvulus, Brugia malayi
B. Wuchereria bancrofti, Brugia malayi
C. Brugia timori, Onchocerca volvulus
D. Wuchereria bankrofti, Brugia timori
E. Brugia timori, brugia malayi
2. Vectors involved in the transmission of wuchereria bankrofti include the
following species of vectors:
A. Culex
B. Aedes
C. Anopheles
D. Simulium
E. Trypanosome
3. Brugia timori is found commonly in the following places:
A. West Africa
B. Central Africa
C. South America
D. Indonesia
E. East Africa
4. Reservoir of infection for Brugia malayi are:
A. Man
B. Cat
C. Dog
D. Monkey
E. Vulture
5. Which of the following nematodes does Anopheles barbirostris transmit?
A. Wuchereria bankrofti
B. Brugia malayi
C. Brugia timori
D. Dracunculus medinensis
E. Onchocerca volvulus
6. The foci of onchocerciasis are predominantly the following areas:
A. West Africa
B. North America
C. Central and East Africa
D. Yemen
E. Central and South America
7. The number of people affected by oncocerciasis is between:
A. 10 to 15 million
B. 20 to 40 million
C 45 to 60 million
D. 65 to 80 million
E. Over 100 million
8. Africa’s contribution to the total burden of onchocerciasis is about:
A. 20%
B. 45%
C. 70%
D. 80%
E. 95%
9. The pathological lesions in filariasis are produced predominantly by
A. The adult worms
B. The microfilariae
C. Vectors transmitting the disease
D. Bacterial infection of the lymph vessels
E. Viral infection of the lymph vessels

Use the following background information to answer questions 10 to 12.

A 40 year old man reports at your consulting room with a history of “Prolonged
period of ill health” for which he has consulted several prayer camps and
traditional healers without improvement. No other relevant information is available.
The highlights of your examination shows the following:

1. Lymphoedema of the scrotum, penis, and lower limbs


2. Hydrocele with lymphangiitis of the scrotum
3. Lymphocele
4. Adenolymphocele and chyluria.
5. Fairly good conjunctivae, with no appreciable weight loss
10. The likely diagnoses include:
A. Filariasis
B. Onchocerciasis
C. Dracuntiasis
D. Schostosomiasis
E. Malignant obstructive tumour
11. Which of the following investigations would you carry out to help clinch a
definitive diagnosis?
A. Blood film prepared from a sample taken between 12 noon and 8 pm
B. Blood film prepared from a sample taken between 8pm and 2am
C. Doing a CT scan of the affected areas
D. Doing a plain X-ray of the affected areas
E. Taking urine for culture and sensitivity
12. Treatment of the above condition will include:
A. Oral administration of ivermectin
B. Oral administration of albendazole
C. Antibiotics for secondary bacterial infection
D. Surgery for drainage of abscess and correction of the effects of obstruction
E. Radiotherapy
13. In heavy onchocercal infections, an infected adult may have as many as about
50 to 200 million microfillariae in the skin at any one time. In such heavy
infestations microfilariae may be found in the:
A. Blood
B. CSF
C. Urine
D. Lens
E. Sputum
14. Clinical features of onchocerciasis include:
A. Deep pigmentation of the skin
B. Overhanging skin esp. in the groin
C. Chronic conjunctivitis
D. Hydrocoele
E. Epididymo-orchitis
15. Onchocerciasis is commonly found around river banks and basins in endemic
countries because:
A. The high humidity in the area favours the survival of the vectors that transmit the
disease
B. The vectors feed on aquatic plants in the water bodies
C. The vectors have ready access to the water bodies as source of drinking water
D. The vectors breed in the water bodies
E. The all-year round blooming trees and thicket at the banks provide shade and hiding
place for the vectors
16. Early detection of re-infection in onchocerciasis controlled areas can be done
through:
A. Slit lamp examination of the anterior chamber of the eye
B. Skin snip
C. Immunodiagnostic test
D. Darkfield microscopy
E. Identification of a nodule in the skin
17. The current drug for the treatment of onchocerciasis is ivermectin:
A. It is administered in a dosage of 150micrograms/kg body wt intradermally
B. It is administered in a dosage of 150micrograms/kg body wt orally
C. It is macrofilaricidal in action
D. It is microfilaricidal in action
E. It is microstatic in action
18. The major side effects of ivermectin are:
A. Pruritis
B. Dyspnoea
C. Arthralgia
D. Conjunctivitis
E. Dizziness
19. The minor side effects are:
A. Fever
B. Dizziness
C. Headache
D. Arthralgia
E. Painful glandular swelling and tachycardia
20. Ivermectin is contraindicated in which of the following groups of people?
A. Pregnant women
B. Lactating mothers in the first week of lactation
C. Children under 15 years of age
D. Extremes of age i.e. under 1 year and above 70 years
E. Seriously ill patients
21. A patient with onchocerciasis is declared “cured” when:
A. The skin snip is no longer positive for onchocerciasais
B. When microfilariae are not seen in the anterior chamber of the eye
C. When the patient has received ivermectin yearly for 5 years
D. When the patient has received a course of diethyl carbamazine
E. When onchocercomata in the skin are non-tender
22. A single oral administration of ivermectin depletes skin microfilariae for a few
months and microfilariae re-appear at a certain percentage of pre-treatment levels
within one year. This percentage is about:
A. 5
B. 8
C. 12
D. 15
E. 20
23. The Onchocerciasis Control Program in West Africa achieved the following:
A. Prevented some 600,000 people from going blind
B. Protected some 40 million people from the disease
C. Permitted about 25 million hectares of fertile land to be reclaimed for
food production
D. Established settlement for about 1 million people
E. Eradicated onchocerciasis from West Africa
24. The Onchocerciasis Control Program was an International Control Effort
aimed at comfronting the challenges posed by the disease in West Africa:
A. The program started in 1965 along the Volta River Basin
B. The program was started by 7 West African countries
C. The head quarters was in Ouagadougou, Burkina Faso
D. The Executing Agency was UNICEF
E. The control program was extended westward in 1978 to prevent invasion of
infected flies from the western side into the controlled areas
25. The countries that participated in this program include:
A. Ghana, Nigeria, Burkina Faso
B. Togo, Benin, Ghana
C. La Cote d’Ivoire, Mali, Nigeria
D. Ghana, Togo, Burkina Faso
E. Benin, Mali, Guinea
26. The Western Extension added which of the following countries?
A. Guinea
B. Senegal
C. Sierra Leone
D. Gambia
E. Guinea Bissau
27. African Programme of Onchocerciasis Control (APOC) is another International
disease control effort. This program is currently running in which of the
following countries?
A. Nigeria
B. Mali
C. Uganda
D. Niger
E. Togo
28. Guinea worm is prevalent in which of the following areas?
A. West Indies
B. Sudan
C. Middle East
D. Southern parts of North America
E. West and Central Africa
29. The disease agent dracunculus medinensis:
A. Is transmitted through drinking water containing an infected intermediate host, the
cyclops
B. The incubation period of the disease in man is about 9 – 12 months
C. The male adult worm is about 2 -3 cm shorter than the female worm
D. The female worm is about 1 meter long
E. About 40% of guinea worm infections is located on the legs
30. Dracunculiasis can be treated with:
A. Suramin
B. Niridazole
C. Diethylcarbamazine
D. Metronidazole`
E. Albendazole
31. Transmission of Guinea worm infection can be interrupted through the following
measures:
A. Provision of pipe-borne water
B. Filtering drinking water with nylon or linen filter
C. Provision of deep wells
D. Preventing cattle from drinking from dams
E. Mass distribution of diethylcarbamazine just before the usual onset
of outbreaks
32. Guinea worm is currently earmarked for eradication. Eradication measures are in
place in the following areas
A. Ghana
B. Nigeria
C. Sudan
D. India
E. North America
33. Financiers of the guinea worm eradication program include:
A. United States Agency for International Development
B. World Bank
C. Bill and Belinda Gates Foundation
D. Governments of Canada and USA
E. Opec Fund for International Development
34. Possible reactions of the human host to guinea worm infection include:
A. Allergy
B. Cyst formation
C. Abscess formation
D. Calcification
E. Disintegration and absorption
35. Important trematodes of public health importance in man are:
A. Dracunculus medinensis
B. Schistosoma haematobium
C. Schistosoma mansoni
D. Schistosoma japonicum
E. Wuchereria bankrofti
36. Schistosoma haematobium:
A. Has been known for over 3,000 years in Egypt
B. Grows to maturity in the lungs
C. Adult worm resides in the vesical plexus of man
D. Female adult worm carries the male worm in its gynaecophoric canal
E. Can be found in other hosts like the cat, dog, pig and cow
37. Shistosoma japonicum is found in the following areas:
A. Middle East
B. West, Central and East Africa
C. China, the Philippines and Japan
D. Caribbean and Central America
E. Burma and Thailand
38. Which of the following statements are true?
A. The intermediate host for Schistosoma haematobium is bulinus
B. The intermediate host for schistosoma mansoni is oncomelania
C. The intermediate host for schistosoma japonicum is biomphalaria
D. These blood flukes mature in the lungs
E. The infective stage in man is the miracidium
39. In schistosomiasis
A. The pathological reactions are provoked by the live eggs that pass through the wall
of the viscera
B. The viscera commonly involved in this include the bladder, ureters, seminal
vesicles, vagina, cervix and fallopian tubes
C. Eggs that are held up in the wall of the viscera die after 3 weeks
D. Miracidia liberated from eggs must enter the appropriate intermediate host within
48 hours or die
E. Eggs that are excreted with the urine or faeces can live outside fresh water for 48
hours
40. The pathological changes in schistosomiasis depend on
A. Intensity of infection
B. Frequency of infection
C. Duration of exposure
D. Socio-economic status of patient
E. Sex of patient
41. Katayama syndrome
A. Results from allergic reaction to schistosomules in the liver
B. Occurs in s. mansoni and s. japonicum infections
C. Occurs in s. haematobium and s. japonicum infections
D. Is a complex symptom of pyrexia, malaise, painful tender enlarged liver,
spleenomegaly and jaundice
E. Is a complex symptom of pyrexia, hyperaemia and sessile or pedunculated polyps
in the mucosa of the large bowel
42. The following may be affected in s. haematobium infection
A. Ureters
B. Large bowel
C. Seminal vesicles
D. Lungs
E. Liver
43. In light urinary schistosomiasis, urine specimen for examination should be
collected:
A. In the early morning
B. Between 10am and 2pm
C. Between 2pm and 6pm
D. Between 6pm and 10pm
E. At midnight
44. In s. haematobium infection
A. Ova with lateral spines in the urine is diagnostic
B. Ova with terminal spines may be found in the faeces
C. An x-ray of the pelvis and abdomen may reveal calcification of the bladder wall and
lower end of the ureters even in early infections
D. Ova may be seen in snips of rectal mucosa
E. The circumoval precipitin test is a serological test that can be used as a follow up
for success in treatment
45. In intestinal schistosomiasis
A. Examination of the faeces reveals the eggs
B. The eggs of the intestinal schistomes have terminal spines
C. Liver biopsy is contraindicated in hepatomegaly
D. Barium swallow is indicated if oesophageal varices are suspected
E. X-ray of the abdomen helps in diagnosis
46. A 30 year old patient reports at your consulting room with a complaint of
terminal haematuria. After doing the necessary things including laboratory
investigations, you have come to a firm conclusion that the patient is suffering
from s. haematobium infection. Which of the following drugs will be appropriate
for treatment of the condition?
A. Praziquantel
B. Metrifonate
C. Oxamniquine
D. Niridazole
E. Stibophen

47. Which of the following drugs would you use to treat a diagnosed case of
s.mansoni in a 40 year old lady?
A. Praziquantel
B. Metrifonate
C. Stibophen
D. Oxamniquine
E. Niridazole
48. Certain nematodes pass through the lungs during their developmental stages in
man. They include:
A. Dracunculus medinensis
B. S.mansoni
C. Wuchereria bancrofti
D. Onchocerca volvulus
E. S.japonicum
49. Mass suppressive treatment with appropriate drugs is one of the methods used
to control schistosomiasis. Strategies under this method are:
A. Multiphasic chemotherapy
B. Selective population chemotherapy
C. Mass chemotherapy
D. Targeted chemotherapy
E. Oppurtunistic chemotherapy
50. Other methods of control of schistosomiasis are environmental measures such
as:
A. Provision of toilet facilities in the vicinity of recreational swimming places
B. Construction of foot bridges at water crossing points
C. Construction of irrigation schemes that encourage stagnation of the irrigation water
D. Wearing of rubber boots or other protective clothing for people who by necessity
need to wade through water bodies
E. Vigorously toweling the skin surfaces 24 – 48 hours after contact with suspected
water with application of 40% alcohol to the skin
CHAPTER 4
INFECTIONS BY NEMATODES AND TREMATODES
ANSWERS

1. B, D, E 38. A
2. A, B, C 39. B, C, D
3. D 40. A, B, C
4. A, B, C, D 41. A, B, D
5. C 42. A, B, C, D, E
6. A, C, D, E 43. B
7. B 44. B, D, E
8. E 45. A, B, D
9. A 46. A, B, D
10. A 47. A, D, E
11. B 48. B, E
12. A, B, C, D 49. B, C, D
13. A, B, C, E 50. A, B, D
14. B, C
15. D
16. C
17. B, D
18. B, E
19. A, C, D, E
20. A, B, E
21. -
22. E
23. A, B, C
24. B, C
25. B, D, E
26. A, B, C, E
27. A, C
28. A, B, C, E
29. A, B, D
30. B, D, E
31. A, B, C
32. A, B, C, D
33. B, C, D, E
34. A, B, C, D, E
35. B, C, D
36. A, C
37. C, E
CHAPTER 6
CUTANEOUS ULCERS, SINUSES, FISTULAE

 Concerning an ulcer, the :


 Edge is where the healthy skin begins
 Floor is what is palpated
 Base is what is seen around the ulcer
 Size is helpful in diagnosis
 The margin is synonymous with the edge
 A specific ulcer may be caused by:
 Mycobacterium ulcerans
 Deep venous thrombosis
 Treponaema pertenue
 Esch. Coli
 Sickle cell disease
 A tropical ulcer:
 Occurs only in the tropics
 Is caused by Borrelia vincenti
 Is predisposed to by malnutrition and walking bare-footed
 May be initiated in a minor skin wound
 Starts as a painless ulcer
 A tropical ulcer:
 Starts as a painful septic blister
 Has a ragged, foul-smelling black, or grey slough of the skin when
the blister ruptures
 In the acute stage, has an unhealthy floor with offensive, yellowish
discharge
 In the acute stage, has a sloping edge
 70% of cases found in lower limbs
 A tropical ulcer of 8 weeks duration:
Becomes a chronic ulcer
Has a sloping edge
Has a grey or pink granulation in the floor
Has a soft base
Contains the causative organisms which are identifiable by culture
of a swab
 Complications of a tropical ulcer include:
 Pyaemia

1
 Chronic osteomyelitis of the underlying bone
 Tetanus
 Lymphoedema of the foot
 Malignant change in the ulcer

 The usual treatment of a tropical ulcer in the acute stage include:


 A 5-day course of streptomycin or sulphonamide
 Bed rest
 Daily eusol dressing
 Excision of the ulcer
 Application of a Plaster-of-Paris
 A tuberculous ulcer:
 Has a regular outline
 Has thin, blue, undermined edges
 Has a floor covered with pink granulation and thin watery discharge
 Has an indurated base
 May be found just above the medial malleolus
 Buruli ulcer:
 May start as a painless subcutaneous nodule on the trunk
 May heal proximally while spreading distally
 Progresses slowly and has sloping edges
 May have bridges of skin across it
 Is usually successfully treated by drugs alone.
 Which of the following drugs can be used in the treatment of
Buruli ulcer:

 Rifampicin
 Clofazimine
 Streptomycin
 Co-trimoxazole
 INAH
 An ulcer of yaws:
 May start as a granulomatous papule
 Has sloping edges
 Has dirty yellowish sloughing floor
 May heal spontaneously, the skin over it becoming depigmented

2
 Does not cause enlargement of regional lymph nodes
 An ulcer:

 With a nodular floor is likely to be non-specific


 With a serpiginous outline is likely to be tuberculous
 In the sole is most probably neuropathic or malignant melanoma
 With a punched out edge is most probably syphilis or yaws
 With an indurated base is malignant

 A non-specific ulcer:
 In the acute phase is painful and has a sloughing floor covered with
purulent discharge and a sharp edge with oedematous tender
surrounding skin
 In the transition phase has a sloping edge and grayish-yellow floor.
 In the reparative or healing phase is painless and pink granulation
fills the floor
 In the chronic phase has grayish floor with a thin odourless
discharge
 The fifth phase is malignant change
 In an indolent or callous ulcer, the:
 Base is indurated
 Floor has pink granulation tissue
 Edges are rigid and hard
 Surrounding skin is warm and oedematous
 Epthelium at the edge grows inwards
 Complications of non-specific ulcers of the lower leg include:
 Chronic osteomyelitis
 Amyloidosis
 Below knee lymphoedema
 Deformities of the foot or ankle
 Venous thrombosis
 A painful circular ulcer with sloping edges on the dorsum of the
foot near the big toe in a 60-year old man could be due to:

 Leprosy
 Varicose veins

3
 Arterial disease
 Malignant melanoma
 An abscess
 The prime cause of a venous ulcer following deep venous
thrombosis is:
 Varicosity of the long saphenous vein
 Venous stasis
 Back pressure in the veins
 Destruction of the valves of the perforating and deep veins
 High venous pressure in the legs
 A venous ulcer usually:
 Is situated just above the medial malleolus
 Is irregularly circular
 Has a deep floor with pink granulation
 Has punched out edges
 Has indurated base

 Essentials in the treatment of a venous ulcer include:


 Application of an antibiotic dressing to the ulcer daily
 Below knee supportive occlusive elastic, elastoplasts or viscopaste
bandage from the toes to the tibial tubercle
 Administration of antibiotics
 Early stripping of varicose veins
 Physiotherapy
 Decubitus ulcer:

Results from inadequate nursing care


Shows first as a darkened area of the skin
May expose muscles and tendons but not bone
Can occur in any part of the body
Can be prevented by 3-hourly turning of the patient and adequate
skin care
 Neurotropic(Trophic) ulcer:
 May be caused by leprosy or diatetes mellitus
 Is usually on the dorsum of the foot
 Is usually shallow
 Heals quickly
 May be painful

4
 Haemoglobinopathic ulcer:
 Is most often found in the lower limb close to the lateral malleolus
 Is associated with atrophic skin and hyperpigmentation
 Heals quickly
 Often breaks down again after healing
 Bed nest worsens it
 Haemoglobinopathic ulcer is difficult to treat because of:
 Lowered oxygen tension
 Recurrent haemolytic crisis
 Fibrosis around the ulcer
 Low haemoglobin level
 Lack of appropriate personnel
 A 50-year old man has a 0.5cm ulcer on the nose which has a
raised and rolled up edge. It is of 2/12 duration.

The most likely clinical diagnosis is:

 Squamous cell carcinoma


 Malignant melanoma
 Syphilitic ulcer
 Non-specific ulcer
 Basal cell carcinoma

 A sinus:
 Connects two epithelial surfaces
 May be congenital
 Has a discharge which is always present
 Is treated with antibiotics
 Unresolved infection may cause acquired sinus.
 Causes of a sinus include:
 Chronic ulcer
 Achnonycosis
 Amyloidosis
 Buruli ulcer
 Foreign body e.g. Sutures
 In the treatment of a sinus:
 Excision of sinus is indicated in congenital type
 Dressing alone adequate for acquired type

5
 Tract of the sinus may be laid open and dressed
 In tuberculous, sinus treatment of TB lesion leads to healing
 Penicillin exhibition contributes to healing
 Causes of acquired fistula include:
 Abscess that ruptures into a cavity and skin
 Burns
 Granulomatous lesions
 Benign tumours
 Schistosoma haematobium
 The following are examples of neoplastic ulcers:
 Squamous cell carcinoma
 Renal cell carcinoma
 Kaposi sarcoma
 Malignant melanoma
 Dermato fibrosarcoma Protuberans
 Diabetic Ulcer:
 Ulcers in a patient with Diabetes Mellitus
 May be caused by infection
 May be caused by trauma
 May be caused by pressure necrosis
 Basis in hypoaesthesia from blood loss

 The following are examples of specific ulcers:


 Pyogenic ulcers
 Decubitus ulcers
 Syringomyelia
 Tropical ulcers
 Syphilitic ulcers
 Tuberculous ulcer:
 Seen usually in the groin and on the neck
 May be associated with normal sized lymph nodes
 Culture of discharge may show AFBS
 Could be associated with night sweats

6
 May be associated with tuberculous bone infection
 In Buruli ulcer differential diagnosis of the pre ulcerative form
include:
 Onchocercoma nodule
 Lipoma
 Sebacous cyst
 Naevus
 Phyconycosis
 Complications of Buruli ulcer include:
 Anaemia
 Contracture
 Proximal lymphoedena
 Marjolin’s ulcer
 Osteitis
 In the treatment of Buruli ulcers the following practitioners are
involved:
 Physiotherapist
 General Surgeon
 Orthopedic Surgeon
 Opthalmologist
 Maxilloficial Surgeon

CHAPTER 6
CUTANEOUS ULCERS, SINUSES, FISTULAE
ANSWER

 A
 A, C

7
 C, D
 A, B, C
 A, B, C
 B, C, D, E
 B
 B
 A, B, D
 A, C
 C, D
 C, D
 A, C
 A, C
 C, D
 C, E
 D
 A, B
 B
 A, B, E
 A
 B, D
 A, B, C, D
 E
 B
 B, E
 A, C, D
 A, C
 A, B, C, D
 A, B, C, D
 D, E
 A, C, E
 A, C, E
 A, B, D
 A, C

8
CHAPTER 8
SHOCK

 In all forms of shock:

 There is inadequate tissue perfusion

 The blood volume is reduced

 There is cellular hypoxia

 Cardiac function is impaired

 There is vasoconstriction in the microcirculation

 The microcirculation consists of:

 The capillaries

 The arterioles and capillaries

 The venules and capillaries

 An anteriole, venule and the interposed capillaries

 The venules

 Which of the following is/are true of the circulation?

 Each heart beat propels 70-90ml of blood at a pressure of about


120mmHg

 About 30% of the blood is contained in the micro-circulation

 About 20% of the blood is in the main arteries

 Only about one-fifth of the capillaries are normally open at any one

1
time

 The average adult has 75-80ml of blood per kg/body weight

 The following are vasodilators:

 Histamine

 Bradykinin

 Kallidin

 Beta-receptors

 Local products of metabolism

 As a result of cellular hypoxia:

 Anaerobic glycolysis of a mol of glucose produces 3 ATP units

 Sodium with chloride and water enters the cell

 Acid phosphatase leaves the cell

 The lipoprotein membranes of the lysosomes break down

 Hydrogen released from glucose combines with oxygen to form


water

 Adrenaline causes:

 Constriction of veins

 Constriction of arterioles in muscles

2
 Vasodilatation of coronary arterioles

 Stimulation of respiration and increase in dead space

 Dilatation of cutaneous arterioles

 Angiotensin:

 Is a powerful vasopressor

 Stimulates release of ADH

 Is elaborated essentially in the kidneys

 Stimulates release of cortisol

 Is responsible for thirst in shock

 Acidosis in shock results from:

 Diminished G.F.R.

 Anaerobic glycolysis

 Vasoconstriction of the arterioles

 Depression of respiration in the early stages

 Breakdown of the gut mucosal barrier

 In haemorrhagic shock:

 The serum catecholamines are elevated

 Renal excretion of sodium is usually increased

 There may be glycosuria

 Metabolic activity is increased

 Immunity is depressed

 During the stage of decompensation in haemorrrhagic shock:

3
A. Venous return is reduced

B. Interstitial fluid is increased

C. Vasoconstriction is marked

D. The coronary and cerebral arterioles become constricted and so

reduce the cardiac and cerebral perfusion respectively

E. The capillary endothelium is damaged by products of tissue


metabolism

 Replenishment of blood volume after bleeding:

 Is most rapid in the first 6h

 Takes 72h to be restored to normal

 Is aided by a rise in venous pressure

 Is aided by aldosterone and ADH

 Haemopoiesis is maximal in the 72first hours

 Which of the following has stabilizing effect on lysosomal


membranes?

 ACTH

4
 Glucagon

 Cortisol

 Histamine

 Nor-adrenaline

 The pulse in Class III hypovolaemic shock is WEAK because:

 The sino-atrial node is stimulated by adrenaline

 Of poor filling of the vessels with blood

 The pulse is fast

 The blood pressure is low

 The arterioles are constricted

 Poor tissue perfusion is suggested by:

 Cold extremities

 Restlessness

 Oliguria

 Anxiety

 Hypertension

 Which of the following signs of hypovolaemic shock is/are due to


vasoconstriction?

 Cold skin

 Pale conjunctivae

 Restlessness

 Collapsed peripheral veins

5
 Rapid deep respiration

 Physiological reactions in shock involve the:

 Aortic bodies

 Cardio-inhibitory centre

 Adrenal cortex

 Subfornical organ

 Pancreas

 In the management of haemorrhagic shock crystalloids:

 Rapidly restore the volume of the blood

 Lower the viscosity of the blood and thereby improve


microcirculation

 May make blood transfusion unnecessary

 Improve oxygen uptake by the tissues

 Do not affect blood product requirements

 Dextran 70:

 Causes hypervolaemia

 Inhibits factor VIII

6
 Prevents adhesiveness of platelets

 Causes rouleaux formation of red cells

 Does not cause anaphylactic shock

 Dextran 70 should be used with care in the treatment of


haemorrhagic shock because it may cause:

 Extravascular dehydration

 Bleeding especially in patients with wounds

 Anaphylactic shock

 Interference with subsequent blood grouping

 Predispose to the systemic inflammatory syndrome (SIRS)

 Which of the following drugs may be used in the treatment of


haemorrhagic shock?

 Isoproterenol

 Nor-adrenaline

 Dopamine

 Phenoxybenzamine

 Chlorpromazine

 Perhaps the most effective drug to use in haemorrhagic shock, if


tissue perfusion remains inadequate in spite of adequate fluid
therapy, is:

 Isopreterenol

 Dopamine

 Phenoxybenzamine

7
 Chlorpromazine

 Dobutamine

 Dopamine:

 Stimulates vasodilatation of renal and splanchnic vessels

 Increases GFR and renal sodium excretion

 Is inotropic and has a half-life of 2 mins

 Is a precursor of nor-adenaline and so acts only on alpha-


adrenergic receptors

 Causes visceral vasconstriction

 In haemorrhagic shock:

 Mannitol should be given to promote diuresis

 It is advisable to use alkalis to correct the acidosis

 Hydrocortisone is of proven value

 Raising the leg will increase venous return by about 2L

 Acidosis with hypokalaemia may be expected to supervene

 The most useful single observation during treatment of


hypovolaemic shock is:

 Pulse and blood pressure every 15 minutes

8
 Central venous pressure

 Colour of conjunctivae

 Hourly urine output

 Filling of cutaneous veins and warmth of the skin

 The central venous pressure is a function of:

 Venous blood flow

 The tone of the main veins

 Distensibility and contractility of the right atrium and ventricle

 The intrathoracic pressure

 Contractility of the left ventricle

 “Irreversible” shock maybe due to:

 Inadequate fluid replacement

 Continued blood loss

 Metabolic alkalosis

 Undetected organ injury

 Multiple use of antibiotics

9
 In shock lung adult Respiratory Distress Syndrome (ARDS):

 The weight of the lung is markedly increased: 1000-2000 g

 The lesions include atelectasis, oedema of interstitial tissue,


basement

and alveolar membranes and intra alveolar haemorrhage

 The lesions are the direct effect of shock on the lung

 Rapid and difficult respiration, perhaps with a dry cough, often


starts 24h

after the injury

 Large doses of hydrocortisone favourably influence the prognosis

 Causes of shock lung (ARDS) include:

 Direct effect of shock on the lung

 Direct injury of the lung

 Excessive fluid administration

 Oxygen toxicity

 Fat embolism and thrombo-embolism

 Endotoxin:

 Is a complex lipopolysaccharide bonded to protein

 Is found in the cell wall of gram-negative bacteria only

 Is released by living bacteria

 Has toxic effects on white blood cells, platelets and


reticuloendothelial cells

 Is found as a component of the cell wall of all organisms

10
 The following are released in septic shock:

 Nor-adrenaline and adrenaline

 Histamine

 Thromboxane A2

 Serotonin

 Kinins

 In septic shock:

 Teichoic acid is produced in gram-negative bacteraemia

 Increased capillary permeability is due in part to the toxic effect of


the cytotoxic products of complement-induced PMN aggregation

 Fluid is lost from the vascular to the interstitial space

 Release of endorphin is an important factor

 Spontaneous bleeding does not occur

 Septic shock:

 May follow hypovolaemia

 May occur during treatment of septicaemia

 Is always characterized by high output failure, low peripheral


resistance and normal or high central venous pressure

11
 Is not caused by skin infections

 Is likely cause of hypotension two hours after onset of severe acute


pancreatitis

 In most cases of septic shock:

 The skin is warm and dry initially

 The pulse is initially rapid and feeble

 Shivering may precede the attack

 Apprehension and confusion may be the initial manifestation

 There is hypoventilation

 In the treatment of septic shock:

 Blood is needed urgently

 Corticosteroids, given in massive doses, have been shown to inhibit


the production of arachidonic acid derivatives by complement-
activated PMN

 Antibiotics are not particularly beneficial as the organisms are


already dead

 Digitalis may be required

 Fluid therapy is of secondary importance , and may be dispensed


with

 Which of the following is TYPICAL of neurogenic shock


(vasovagal syndrome)?

 Sweating

 Hypotension

 Pallor

12
 Bradycardia

 Loss of consciousness

 Cardiogenic shock:

 Maybe caused by pneumothorax or pulmonary embolism

 Leads to venous congestion, severe hypertension and dyspnoea

 Leads to cold and clammy skin

 Raises the C.V.P.

 Is attended by severe pain across the chest

 In anaphylactic shock:

 Reticulo-endothelial cells are involved in the antigen-antibody


reaction

 Adrenaline histamine, plasma kinins are among the substances


released

 There is generalized vaso-constriction of peripheral vessels

 The bronchioles are constricted

 Penicillin injection may be the cause

 The following are used in the treatment of anaphylactic shock:

 Adrenaline

13
 Antihistamine

 Hydrocortisone

 Aminophylline

 Intravenous fluid

Two days after resection of a segment of gangrenous ileum, a


patient becomes apprehensive and confused. The skin is warm
and dry, the temperature 39˚C, the blood pressure 90/60 and the
pulse 120/min and bounding. The abdomen is slightly tender and
breathing is rapid

 The most likely clinical diagnosis is:

 Hypovolaemic shock

 Septic shock

 Malaria

 Anaphylactic shock

 Peritonitis

 The most important measure to adopt is:

 Intravenous antibiotics

 Rapid infusion of Ringer’s lactate or normal saline

 Intravenous hydrocortisone

 Rapid infusion of blood

 Intramuscular chloroquine

 B-endorphin:

 Is a peptide hormone produced mainly by the hypothalamus

14
 Is an opiate antagonist

 Raises the blood pressure

 May cause severe respiratory depression

 Is released with ACTH

 Which of the following does/do not cause vasodilatation?

 Kinins

 Prostacyclin

 Thromboxaine A2

 C5a, C5, C3 components

 Serotonin

 In systemic inflammatory response syndrome (SIRS), there must


be two or more of the following:

 Temperature > 38˚C or < 36˚C

 Heart rate > 100 beats/min

 Respiratory rate > 25/min or PaCo2 < 32mmHg

 WBC > 12,000 mm3 or < 3,000 mm3

 In septic shock, the heart is affected by:

 TNF – a, IL – 8

 Nitric oxide

15
 Coronary microvascular damage

 Elevation of 2,3DPG levels

 After recovery from moderate blood loss:

 The haematocrit is raised

 The extravascular fluid volume is reduced

 Production of red blood cells is maximal in 7-10 days

 Erythropoietin production is stimulated by IL-6

 The serum protein level is increased

 In septic shock, the plasma level of:

 Platelet activator factor is raised

 Thrombomodulin is elevated

 Protein C is raised

 Protein S is raised

 AT III is raised

 Persistence of the following in the serum predicts Multiple Organ


Dysfunction Syndrome

 TNF

 IL -1

 IL -4

 IL -6

 IL - 10

16
 Activation of neutrophils in septic shock leads to:

 Their adherence to and damage of vascular endothelium

 Tissue infiltration

 Increased phagocytic activity against all bacteria

 Production of anti-oxidants

 Production of nitric oxide

 Multiple Organ Dysfunction Syndrome is probably due to:

 Activation of Coagulation pathways

 Metabolic alkalosis

 Depletion of anti-oxidants

 DIC

 Hyperkalaemia

 In septic shock:

 The WBC shows leucopaenia after initial leucocytosis

 There is thrombocytopaenia

 Blood culture is invariably positive for bacteria

 The count of the RBC is low or normal

 In neurogenic shock:

17
 The arterioles and venules of the muscles are constricted and those
of the skin dilated

 The cause may be pain or fright

 Feeling of heat is experienced

 The pulse is rapid and the blood pressure low

 The patient is laid flat with the head high

CHAPTER 8
SHOCK
ANSWERS

 A, C 30. A, B, C, D, E

 D 31. B, C, D

 A, D, E 32. A, B

 A, B, C, D, E 33. A, C, D

 A, B, D 34. B, D

 A, C, D 35. D

 A, B, E 36. A, C, D

 A, B, C 37. A, B, D, E

18
 A, C 38. A, B, C, D, E

 A, B 39. B

 A, D 40. A, B, C

 C 41. D, E

 B, 42. C

 A, B, C, D 43. A

 A, B, D 44. B, C

 A, B, C, D, E 45. B, C

 A, B, C, D 46. A

 A, B, C, D 47. A, D

 A, B, C, D 48. A, B, E

 A, C, D, E 49. A, C, D

 B 50. B, D

 A, B, C 51. B, C

 D

 D

 A, B, C, D

 A, B, D, E

 A, B, D

 B, C, D, E

 A, D

19
CHAPTER 10
PRINCIPLES OF FLUID AND ELECTROLYTE THERAPY AND
ACID-BASE DISTURBANCES

 Total body water expressed as percentage of body weight is:


 60 in the adult male and 40 in the female
 75 in the neonate
 Higher in the obese
 Lower in the aged
 Lower in the muscular
 In the distribution of water in the adult male body (expressed as
percentage of body weight):
 40 per cent is intracellular
 12 per cent is interstitial
 2 per cent is transcellular
 6 per cent is intravascular
 1.5 percent peritoneal
 Concerning intracellular ions:
 Potassium is the most important cation, about 140 mmol/l
 The sodium concentration is about 20 mmol/l
 Phosphates are the most important anions, about 26 mmol/l
 The protein concentration is about 9 mmol/l
 Bicarbonate ion is negligible, 10mmol/L
 Which of the following is/are true of extracellular ions?
 Sodium is the most important cation, about 140 mmol/l in the
intravascular fluid and 143 mmol in the interstitial fluid
 Calcium concentration is about 4 mmol/l
 Chloride is the most important anion, about 100 mmol/l in both
intravascular and interstitial fluids
 Protein concentration is about 2 mmol/l in both the intravascular
and interstitial fluids
 Bicarbonate concentration is about 22-29 mmol/l

 Concerning intravenous drips:


 A continuous infusion should as far as possible be set up in the long
saphenous vein

1
 For a cut-down, a mask and gloves should be worn
 With the standard drip set, the number of drops per minute is
determined by multiplying the number of litres to be given in 24h by
10
 The drip set, needle and vein should be changed every 24h
 The giving set should be sterilized with ethylene oxide

 Fluid loss in the adult from:


 The lungs and skin in the tropics is about 1700 ml
 The lungs and skin in the temperate region is about 500 ml
 Urine in the tropics is about 1500 ml
 Urine in the temperate region is about 2000 ml
 Faeces in the tropics or temperate region is about 300 ml
 For an adult patient on intravenous fluid therapy in West Africa,
the daily requirement of:

 Water is 3500 ml
 Sodium is 130 mmol
 Potassium is 70 mmol
 Glucose is 50 g
 Vitamin C is 100 mg
 A litre of Ringer’s lactate contains in mmols:

 Sodium 130
 Potassium 2
 Calcium 6
 Chloride III
 Bicarbonate 23
 Three litres of the following fluids, if given throughout the 24-h
period to an adult, can provide the daily water and sodium
requirements?

 Normal saline
 Badoe’s maintenance solution
 Ringer’s lactate
 Fluid 5:4:1
 5 per cent dextrose in normal saline
 A litre of Darrow’s solution contains in mmols:

2
 Potassium 26
 Sodium 124
 Chloride 104
 Bicarbonate 56
 Calcium 4
 Potassium in adequate amounts occurs in:

 Citrus fruits
 Tomato juice
 Coconut milk
 Tea
 Meat soup

 Hyperkalaemia may occur in:

 Shock
 Typhoid perforation
 Burns
 Severe metabolic alkalosis
 During management of diabetes mellitus with soluble insulin
 In the treatment of hyperkalaemia the following may be used:

 Magnesium sulphate
 Insulin
 Glucose
 Calcium chloride
 Normal saline
 Clinical features of hypokalaemia include:

 Lethargy
 Vomiting
 Hiccough
 Hyporeflexia
 Paraesthesia
 A litre of gastro-intestinal replacement solution contains:

 Sodium 100 mmol


 Potassium 10 mmol
 Ammonium 12 mmol
 Chloride 110 mmol

3
 Glucose 50 g
 In the early phase of dehydration due to peritonitis, the serum
concentration of:

 Serum sodium is low


 Serum potassium is normal
 Serum chloride is low
 Serum bicarbonate is low
 Blood urea is normal
 In the urine in dehydration due to acute intestinal obstruction:

 Sodium is normal
 Potassium is increased
 Chloride is increased
 Urea is low
 Osmolality is increased

 Internal fluid shifts occur in:

 Vomiting
 Peritonitis
 Diarrhoea
 Burns
 Enterocutaneous fistula
 The ideal solution to use in rehydrating a dehydrated patient
without acid-base disturbance is:

 Normal saline
 5 per cent dextrose in normal saline
 Ringer’s lactate
 Badoe’s maintenance solution
 Darrow’s solution
 The reliable parameter during rehydration of a dehydrated patient
is:

 Skin turgor and moistness of tongue


 Filling of subcutaneous veins

4
 Central venous pressure
 Hourly urine output
 Half-hourly pulse and blood pressure
 A litre of diarrhoeal stools contains on an average about:

 120 mmol of sodium


 25 mmol of potassium
 90 mmol of chloride
 45 mmol of bicarbonate
 In cholera, the preferred fluid is:

 Ringer’s lactate
 Darrow’s solution
 Fluid5:4:1
 Normal saline + potassium
 Dextrose/saline + potassium
 Clinical features of hypokalaemia include:
 Convulsions
 Paralytic ileus
 Hypotension
 Nausea
 Drowsiness

 E.C.G. changes(s) in hypokalaemia may be:


 Flat T wave
 Inverted T wave
 Prolonged QRS complex
 Flat P wave
 Depressed ST segment
 Which of the following is/are true of potassium in a 70 kg man?

 The total amount is about 3150 mmol


 About 4 per cent of it is in the extracellular fluid
 The serum potassium is a reliable reflection of intracellular
potassium concentration
 About 10 mmol is lost in sweat daily
 There is usually no potassium in the urine immediately after surgery

5
The serum electrolytes (mmol/l) of a patient are Na: 130, K:3.2, Cl:
90, HCO3: 30. The blood urea is 9 mmol/l and the pH 7.46.

 The most likely clinical diagnosis is:

 Diarrhoea
 Peritonitis
 Gastric outlet obstruction
 Renal failure
 Entero-cutaneous fistula
 The best treatment will be:

 Ringer’s lactate + KCl later


 Normal saline + KCl later
 5 per cent dextrose/normal saline + KCl later
 Darrow’s solution
 Gastro-intestinal replacement solution
The serum electrolytes (mmol/l) of a patient are Na: 128, K:3.0, Cl:
90, HCO3: 21. The blood pH is 7.30 and the urea 10 mmol/l/.

 The most likely clinical diagnosis is:

 High intestinal obstruction


 Gastric outlet obstruction
 Renal failure
 Diarrhoea
 Diarrhoea and vomiting

 The initial treatment will be:

 Ringer’s lactate + KCl later


 Normal saline + KCl later
 5% dextrose/normal saline + KCl later
 Darrow’s solution
 Gastro-intestinal replacement solution

 The serum electrolytes (mmol/l) of an ill patient are Na: 135, K: 3.8,
Cl: 95, HCO3: 24. The blood urea is 7.5mmol/l and the pH 7.4. The
diagnosis may be:

6
 Peritonitis
 Acute intestinal obstruction
 Diarrhoea
 Acute renal failure
 Entero-cutaneous fistula
 The serum electrolytes (mmol/l) of a patient are Na: 125, K: 6.1, Cl:
106, HCO3, 20. The blood urea is 10 mmol/l/ and the pH 7.2. The
diagnosis is:

 Severe diarrhoea
 Entero-cutaneous fistula
 Renal failure
 Acute intestinal obstruction
 Diarrhoea and vomiting
 Conditions that may lead to hypomagnesaemia include:

 Acute intestinal obstruction


 Acute pancreatitis
 Peritonitis
 Malabsorption syndrome
 Cirrhosis of the liver
 Features of hypomagnesaemia include:

 Confusion
 Hyporeflexia
 Muscular twitches
 Vomiting
 Rapid deep respiration
 Which of the following has the same electrolyte composition as
plasma?
 Gastric juice
 Duodenal secretion
 Bile
 Pancreatic secretion
 Ileal secretion

 A 20-years old patient with acute peritonitis and dry tongue has

7
the following serum electrolytes (mmol/l): Na:140, K: 3.8, Cl:95,
HCO3, 25, blood urea 7.5. The interpretation is:

 There is no sodium deficiency


 There is no potassium deficiency
 There is water deficiency
 There is hypo-chloraemia
 There is no metabolic acid-base disturbance
 Gastro-intestinal replacement solution is used for the replacement
of:

 Naso-gastric aspiration
 Losses from entero-cutaneous fistula
 Losses from diarrhoea
 Losses from vomiting not due to gastric outlet obstruction
 Losses from peritonitis
 Water requirement of infants is different from that of adults
because:

 Respiratory rate is higher


 Loss per unit surface area of the body is much smaller
 Kidneys concentrate urine more effectively
 Solute load of the urine is greater on account of a higher metabolic
rate
 Infants have relatively more fat compared with protein stores
 Dehydration in infants:

 Develops more quickly than in adults because the daily water


losses constitute a greater proportion of the T.B.W. and E.C.F.
 If mild results in dry mouth and inelastic skin
 If severe results in hypotonia, inability of the infant to raise his head
and cold extremities
 Is commonly corrected with normal saline
 Is readily corrected with Ringer’s lactate
 For a 9 Kg baby:

 The daily water requirement is about 1080 ml


 The daily sodium requirement is about 18 mmol
 The daily potassium requirement is about 5 mmol
 In moderate dehydration the additional water and sodium
requirements

8
are about 900 ml and 72 mmol respectively
 The daily energy requirement for homeostasis is equivalent to
150G.
of glucose

 Acidosis may cause:

 Muscle cramps
 Hypotension
 Convulsion
 Rapid shallow breathing
 Hyperkalaemia
 About buffers:

 There is much more buffering in the extracellular fluid than in the


cells
 The principal buffers in the cells are the phosphate and protein
systems
 The most important buffer in the blood is the carbonic acid-
bicarbonate system
 Buffers in the red cells are the haemoglobin and carbonic acid-
bicarbonate
 The pH of the urine is USUALLY:

3.5-4.5
4.5-5.5
5.5-6.5
6.5-7.5
7.5-8.5
 Causes of respiratory acidosis include:

 Hyaline membrane disease


 Hysteria
 Salicylate poisoning
 Morphine
 Hypoxia

 Laboratory findings in respiratory acidosis include:

 Decreased blood pH
 PCO2 above 45 mmHg

9
 Initial elevation of HCO3
 Elevated total buffer base
 Depressed serum potassium
 Metabolic acidosis may be caused by:

 Congestive cardiac failure


 Pneumonia
 Enterocutaneous fistula
 Intestinal obstruction
 Starvation

 In metabolic acidosis:

HCO3 is not higher than 21 mmol/1


The pH is not higher than 7.38
The kidneys manufacture ammonia from phenylalamine
The buffer base is decreased
Serum potassium level may be depressed
 Metabolic alkalosis may be caused by:

 Peritonitis
 Gastric outlet obstruction
 Shock
 Paralytic ileus
 Biliary fistula
 At the end of the second post operative day after partial
gastrectomy in a 40-year old man, his Intake-Output chart is as
follows:
Intake: IL Ringer’s lactate + 2L 5% Dextrose + Vits B and C
Output: Urine 750 ml, Nasogastric aspiration 1000 ml
For the next 24h, you may prescribe in addition to Vits B and C

 2L Ringer’s lactate, 2L 5% Dextrose, 40 mmol KCL


 IL Ringer’s lactate, IL Normal saline, 2L 5% Dextrose, 40 mmol KCL
 3L Badoe’s Maintenance solution, IL Gastro-intestinal replacement
solution
 3L Badoe’s Maintenance solution, IL Gastro-intestinal replacement
solution, + 40 mmol KCl

10
 2L Normal saline, 2L 5% Dextrose, 40 mmol KCL

 Oral rehydration solution contains per litre:


Sodium : 90 mmol

Potassium 20 mmol

Chloride 80 mmol

Bicarbonate 30 mmol

Glucose 20G
 Fluid 5:4:1 contains per litre:


Sodium bicarbonate 5g

Sodium chloride 4g

Potassium chloride 1g

Glucose 50g
 The Ghanaian male of 60kg who:

 Is moderately dehydrated has lost about 2.4L of fluid


 Is in shock because of loss of ECF has lost about 4.5L of fluid
 Is in shock because of bleeding has lost at least a litre of blood
 Requires IV potassium because of hypokalaema must be given the
potassium at 30 mmol/h
 Has severe diarrhoea needs at least 10 mmols of potassium per
day
CHAPTER 10
PRINCIPLES OF FLUID AND ELECTROLYTE THERAPY AND
ACID-BASE DISTURBANCES

ANSWERS

 B, D 28. D
 A 29.A
 A, C, D 30.A, B, C, E
 A 31. C
 B, C 32. B, D, E
 A, C 33. A, C
 B, E 34. C
 A, D 35. C, E

11
 B, D 36. A, D
 B, C, D 37. A, D
 A, B, C, D, E 38. A, C
 A, C 39. A, B, D
 B, C, D 40. B, E
 A, D 41. B, D
 A, E 42. B
 B 43. A, D, E
 B, E 44. A, B
 B, D 45. A, C, D, E
 C 46. B, D
 D 47. B
 A, B, C, D 48. B, C
 C 49. A, B, C,D, E
 B, C 50. C
 A, B, E 51. A, C
 A
 C
 C

12
CHAPTER 11
BLOOD TRANSFUSION, HAEMOSTASIS AND ABNORMAL
HAEMOSTASIS
1. A blood donor should:

         A. Be between 18-65 years and over 46 kg in weight


B. Have a haemoglobin of over 14.5g/dl
C. Not have had malaria in the past three months
D. Be free from hypertension, massive splenomegaly or hepatomegaly
E. Not have had an operation within the last 12 months
2. Cryorecipitate is rich in:
A. Factor III
B. Factor VI
C. Factor VII
D. Factor VIII
E. Factor IX
3. In blood stored for 14 days:
A. About 86 per cent of the red cells survive 24 h. after transfusion
B. The plasma concentration of potassium is about 14 mmol/l
C. The ionized calcium is about 1 mmol/l
D. There is little or no activity of Factor V or Factor VIII
E. The pH is 7.0
4. Sensation of heat and pain along the vein being used for blood
transfusion
soon after transfusion has begun is suggestive of:
 Haemolytic reaction
 Allergic reaction
 Thrombophlebitis
 Simple febrile reaction
 Bacterial contamination

1
5. A patient who is having blood transfusion for the first time
complains of
fullness in the head, dyspnoea, constricting pain in the chest and
fever.

The most likely clinical diagnosis is:


 Simple febrile reaction
 Allergic reaction
 Haemolytic reaction
 Septicaemia
 Septic shock
6. You will:
A. Slow down the transufions
B. Take blood from the patient for culture
C. Stop the transfusion, take blood from the patient and the remainder
of
the donor blood for further grouping and cross-matching
D. Give antihistamine, steroids and antibiotics
E. Give intravenous sodium bicarbonate, 50 ml of 25 per cent mannitol
and normal saline
7. About 4 h. after the starting of blood transfusion a patient
complains of pain
in the joints, muscles and fever. There is no headache.

The most likely clinical diagnosis is:


 Haemolyic reaction
 Malaria
 Allergic reaction
 Septicaemia
 Enteric fever

2
8. You will give:
A. Chloroquine injection
B. An antihistamine
C. An antibiotic
D. A diuretic, sodium bicarbonate and normal saline
E. Nothing
9. Mild jaundice developing about 5 days after uneventful blood
transfusion
may be due to:
 Haemolysis of old donor red cells
 Production of antibodies which were too low to be detected at the
time
of cross-marching
 Early viral hepatitis
 ABO antigen-antibody incompatibility
 Sepaticaemia
10. Which of the following cannot be transmitted by blood stored for
more
than 48h?
 Viral hepatitis
 Syphilis
 Malaria
 Trypanosomiasis
 Infectious mononucleosis
11. Bleeding following mismatched blood transfusion is due to the
release of:

3
A. Vasoactive substances
B. Adenosine Diphosphate
C. Free haemoglobin
D. Platelet factor 3
E. 5, hydroxytryptamine

12. Cardiac arrest following massive blood transfusion may be due


to:
A. Acidosis
B. Hyperkalaemia
C. Hypocalcaemia
D. Hypothermia
E. Hypernatraemia
13. Bleeding diasthesis following massive transfusion of banked
blood is
due to:
 Fibrinolysis
 Thrombocytopaenia
 Factor VIII deficiency
 Swelling of the red cells
 Deficiency of factor II
14. Bleeding following mismatched blood transfusion is due directly
or
indirectly to:
 Thrombocytopaenia
 Intravascular defibrination
 Fibrinolysis
 Depressed factor XII

4
 Hypocalcaemia
15. Red blood cells in banked blood:
A. Swell by about 20 per cent
B. Have a normal concentration of ATP
C. Have a diminished concentration of 2, diphosphoglycerate
D. Have a normal life-span

16. The effects of acidosis on the heart include:


A. Relaxation
B. Increased contractility
C. Increased irritability
D. Predisposition to ventricular fibrillation
E. Cardiac arrest
17. Dextran 70 satisfies which of the following criteria for a
replacement
fluid?
 The colloidal osmotic pressure should be equivalent to that of
normal plasmas
 It should be of low viscosity
 The half life should be at least 6 and preferably 12 h.
 It should not interfere with blood groping
 It should not interfere with haemostasis or coagulation
18. Aggregation of platelets in haemostasis is aided by:
A. Exposed subendothelial collagen fibrin
B. Adenosine triphosphate released from damaged tissues and red
cells
C. Thromboxane A2 formed from arachidonic acid on the platelet

5
membrane
D. Prostacyclin synthesized in the block vessel wall from arachidonic
acid
19. Which of the following is/are involved in the conversion of
soluble fibrin
monomer (Factor 1a to insoluble fibrin polymer (Factor 1b)?
 Thrombin
 Factor V
 Calcium ions
 Factor VII
 Factor XIII

20 Which of the following is/are involved in the conversion of


prothrombin
to thrombin?
 Calcium ions
 Kallikrein
 Factor X
 Factor V
 Factor XIII
21. Which of the following may be involved in the activation of factor
X?
A. Factor VIII
B. Factor IX
C. Calcium ions
D. Tissue thromboplastin
E. Factor VII
22. Which of the following cause(s) disaggregation of platelets?

6
A. Fibrinogen degradation products
B. Prostacyclin
C. Antithrombin III
D. First component of complement
E. Adenosine diphosphate
23. Fibrinogen degradation products:
A. Result from lysis of fibrin
B. Stimulate thrombin
C. Form non-clotting complexes with fibrin monomers
D. Stimulate fibrin polymerization
24. Which of the following opposes the action of thromboxane A2?
A. Plasma
B. Thrombin
C. Prostacyclin
D. Antiplasmin
E.Thromboplastin

25. In thrombocytopaenia, spontaneous bleeding may occur when the


platelet
Count per mm3 is less than:
A. 90,000
B. 80,000
C. 70,000
D. 60,000
E. 50,000
26. Thrombocytopaenia may be caused by:
A. Sulphonamides
B. Phenylbutazone
C. Indomethacin
D.Chloroquine
E. Oxytetracycline

7
27. Platelet aggregation is impaired by:
A. Cyclophosphamide
B. Aspirin
C. Quinidine
D. Phenacetin
E. Chloramphenicol
28. Bleeding in uraemia is caused by:
A. Deficient number of platelets
B. Defective release of platelet factor 3 (pf3)
C. Diminish content of PF3
D. Inhibition of platelet aggregation
E. Decreased activity of factor VIII

29. In haemophilia:
A. Factor VII activity is less than 30 per cent of normal
B. The disease is transmitted by a sex-lined dominant gene from a
normal male carrier to male off springs
C. The partial thromboplastin time is prolonged
D. The bleeding and prothrombin times are prolonged
30. Obstructive jaundice leads to depression of:
A. Factor II
B. Factor V
C. Factor VII
D. Factor IX
E. Factor X
31. Disseminated intravascular coagulation may occur in:
A. Amniotic fluid embolism

8
B. Retained dead foetus
C. Mismatched blood transfusion
D. Hypovolaemic shock
E. Viperine venom
32. In disseminated intravascular coagulation:
A. Fibrinogen level is usually below 1.0 g/ litre
B. The bleeding time is normal
C. The clotting time is prolonged
D. Platelet count is depressed
E Factor V is normal but Factor VIII is depressed

33. In the management of disseminated intravascular coagulation the


following
the following may be used:
 Fresh blood
 Fibrinogen
 Heparin
 Fresh frozen plasma
 Dextran
34. Excessive bleeding:
A. In prostatic surgery may be due to activation of plasminogen by
urokinase in urine
B. In liver transplantation is due to depression of factor II
C. In extra-corporeal circulation may be due to destruction of red cells
D. During operation is treated with low doses of heparin if disseminated
intravascular coagulation is the cause

9
35. Diseases that may be transmitted by blood transfusion include:
A. Cytomegalic inclusion disease
B. Toxoplasmosis
C. Brucellosis
D. Acquired Immune Deficiency Syndrome
E. Amoebiasis
36. Use of whole blood for transfusion in Ghana is justified in:
A. Sudden loss of 30% or more blood volume
B. Chronic anaemia
C. Patients undergoing exchange transfusion
D. Twin delivery
E. Patients with pancytopenia

37. The following are indications for FFP use:


A. Improvement in nutritional status
B. Disseminated intravascular
C. Idiopathic thrombocytopenic purpura
D. Emergency treatment of Warfarin overdose
E. Deficiencies of coagulation factors when specific factors are not
available
38. Features of febrile non-haemolytic transfusion reaction (FNHTR)
include:
A. Rigors
B. Temperature of 38.2C
C. Nausea
D. Severe headache
E. Hyper-salivation
39. In managing FNHTR, the following must be done:
A. Discard the unit of blood immediately

10
B. Stop the transfusion temporarily
C. Give antipyretics
D. Inform the blood bank
E. Give Hydrocortisone with subsequent transfusions
40. The following are immediate transfusion reactions:
A. ABO incompatibility
B. Post-transfusion thrombocytopenic purpura
C. Circulatory overload
D. Infectious mononucleosis
E. Air embolism
41. Concerning ABO incompatibility:
A. The mortality rate can be as high as 50%
B. It usually results from wrong identification of blood and recipient
C. It involves the haemolysis of recipient cells by donor antibodies
D. The shock may lead to oliguria
E. The recipient usually gets jaundiced within an hour

42. Post-transfusion thrombocytopenic purpura is:


A. Due to cross reactivity of white cell antibodies
B. Common in multiparous women
C. Managed with Prednisolone
D. Common in adolescents
E. Usually associated with petechiae
43. The following agents have been implicated in post transfusion
hepatitis:
A. Hepatitis A
B. Hepatitis B
C. Hepatitis C
D. Hepatitis D
E. Hepatitis Non-A, non-B, non-C
44. Should a patient’s total blood volume be replaced by transfused

11
blood
within 24 hours, the following measures should be taken:
 The blood should be warmed before transfusing
 Intravenous potassium should be give with every second unit
 With every litre of blood given, 10mls of 10% calcium gluconate
should be given
 The patients Hb level should be checked every 4 hours
 The patients acid-base and electrolytes should be checked
frequently
45. Methods of autologous transfusion include:
A. Optimal dosing with erythropoietin and iron
B. Preoperative blood donation
C. Hidden paid donation
D. Acute isovolaemic haemodilution
E. Intraoperative blood salvage

46. The following are plasma expanders:


A. Dextran
B. Cryoprecipitate
C. Albumin
D. Hetastarch
E. Normal Saline
47. For adequate haemostasis the following should occur:
A. Contraction of the injured vessel
B. Increased pressure within the injured vessel
C. Endothelial cell secretion of factor VIII
D. Platelet plug formation
E. Triggering of coagulation at site of injury

12
48. Activated Protein C inhibits activated factors:
A. VIII
B. II
C. X
D. V
E. IX
49. In DIC there is:
A. Thrombocytopenia
B. Lymphocytosis
C. Dysfibrinogenaemia
D. Shortened APTT
E. Increased D-dimer levels

50. In the initial pre-operative assessment of a patient’s


haemostasis, the
following tests must be done:
 FBC
 D-dimer level
 APTT
 PT
 Bleeding time

13
CHAPTER 11
BLOOD TRANSFUSION, HAEMOSTASIS AND ABNORMAL
HAEMOSTASIS

1. A, D, E 38. A, B, C
2. D 39. B, C, D

3. A, D 40. A, C, E

4. A 41. A, B, D

5. C 42. B, C, E

6. C, E 43. A, B, C, D, E

7. C 44. A, C, E

8. B 45. B, D, E

9. A, B 46. A, C, D

14
10. B 47. A, D, E

11. B, D 48. A, D

12. A, B, C, D 49. A, E

13. B, C 50. A, C, D

14. A, B, C

15. A, C
16. A, C, D, E
17. C
18. A, C
19. A. C, E
20. A, C, D, E
21. A, B, C, D, E
22. B
23. A, C
24. C
25. E
26. A, B, C
27. B
28. B, C, D
29. A, C
30. A, C, D, E
31. A, B, C, D, E
32. A, C, D
33. A, B, C, D
34. A, C
35. A, B, C, D
36. A, C
37. B, D, E

15
CHAPTER 12
NUTRITION IN SURGERY

 The following nutritional element(s) is are INCORPORATED in the


synthesis of tissues:

 Proteins

 Carbohydrates

 Electrolytes

 Water

 Vitamins

 The following can be provided by the rectal route:

 Water

 Sodium

 Glucose

 Amino acids

 Potassium

 The following can be provided by subcutaneous infusion:

 Water

 Sodium

 Dextrose

 Potassium

1
 Amino acids

 Glucogenic amino acids include:

 Glycine

 Leucine

 Valine

 Phenylalamine

 Alanine

 The following may cause thrombophlebitis if infused in a


peripheral vein:

 30 per cent sorbitol

 10 per cent intralipid

 10 per cent dextrose

 10 per cent fructose

 5 per cent alcohol

 Carbohydrates (C) fat (F) and protein (P) provide energy in the
following

percentages:

C F P

2
 50 30 20

 55 35 10

 45 40 15

 50 35 15

 50 40 10

 Fructose:

 Is metabolized in all cells in the body

 Does not require insulin for the initial stages of its metabolism

 Metabolism increases lactic acid production and so may cause


acidosis in dehydrated patient

 Metabolism does not require ATP

 Is not treated differently from glucose metabolically by the body

 Sorbitol:

 Is a sugar alcohol

 Is converted to glucose

 Is poorly absorbed by the renal tubules

 May cause oliguria

 Does not affect the acid-base balance of the body

 Xylitol:

 Is a pentavalent sugar alcohol

 Is converted to fructose-6-phosphate

 May cause liver damage

3
 May produce metabolic alkalosis

 Alcohol:

 Can be given intravenously as 3 or 5 per cent solution

 Is quickly metabolized

 May produce ultrastructural changes in liver

 Amount given must be limited to 2 g/kg body weight/day

 Is entirely of no nutritional value

 Tissues that require glucose for energy in starvation include:

 Brain

 Liver

 Red blood cells

 Polymorphs

 Skeletal muscle

 Fat for intravenous feeding:

 Has high energy density

 Is hyperosmolar

 Can be given in a peripheral vein

 Is usually supplied as 30% emulsion of soya bean or sunflower oil

 How much muscle tissue (g) is broken down to produce 1 g

of nitrogen?

4
 20

 25

 30

 35

 40

 Which of the following amino acids can be used in the D-form?

 Methionine

 Glycine

 Alanine

 Phenylalanine

 Proline

 Utilization of amino acids essential for synthesizing tissues


depends on:

 The balance between non-essential and essential amino acids

 The presence of amino acids in D-form

 A minimum Energy: Nitrogen ration of about 840j: 1g

 D minimum potassium requirement of 5mmol/g nitrogen

 In intravenous nutrition:

 Vitamin, electrolyte and trace elements can be added to


carbohydrate or

amino acid solution or fat emulsion

 Complications of the use of hypertonic glucose solutions include

hypokalaemia, mild dehydration and bacterial and fungal sepsis

5
 Central vein catheterization is mandatory for the administration of
isotonic

solution such as fat and low concentration of amino acids

 If peripheral vein cannulation is used, the drip site must be changed

every 72h

 Eczema, pustules in the face or groin, alopecia and diarrhoea


during TPN

suggest:

              A. Essential fatty acid deficiency


B. Zinc deficiency
C. Hypophosphataemia
D. Avitaminosis B & C
E. Hypoproteinaemia
 Observations to be made during intravenous feeding includes:
A. Strict daily intake-output and weight chart

 Serum electrolytes, blood pH and urea every 48h

 Estimation of blood glucose every 48h

 Full blood count every72h

 Testing of urine for glycosuria twice a day

 Intravenous nutrition should be seriously considered in:

 Severe burns

 Sigmoid colon fistula

 Severe amoebiasis

 Massive resection of small intestine

 Partial gastrectomy

 Problems of TPN include:

6
 Metabolic alkalosis

 Pneumothorax

 Anaemia

 Hyponatraemia

 Inaccuracy of some laboratory investigations

 Complications of TPN include:

 Septicaemia

 Over-hydration

 Haemothorax

 Rebound hypoglycaemia

 Thrombosis

 Concerning nutritional status, body mass index (Kg/m2) of:

 18.5-24.9 is appropriate weight

 25-29.9 is obesity

 < 16 is severe malnutrition

 16-17 is mild malnutrition

 30-35 is overweight

 The following nutrients enhance postoperative immunity:

 Glutamine

 Glucose

 Arginine

 Nucleotides

7
 Omega-3 fatty acids

 Precursors of gluconeogenesis include:

 Lactate

 Fatty acid

 Glycerol

 Arginine

 Ketones

 In the early stages of starvation, weight loss is due to loss of:

 Fat

 Water

 Sodium

 Proteins

 Glucose

 During parenteral feeding the following tests must be done


weekly:

 Liver function tests

 Clotting studies

 Full blood count

 Serum calcium

 Weight

 The grossly malnourished surgical patient is at risk of:

 Pneumonia

8
 Wound dehiscence

 Deep venous thrombosis

 Cerebrovascular accident

 Urinary tract infection

 Complications of enteral tube feeding include:

 Fracture of the tube

 Reflux

 Aspiration pneumonia

 Displacement of the tube into trachea

 Perforation of the bowel

 The body in the average adult male has the following


compositions (%): water (w), protein (p), fat (f), minerals (m):

W P F M

 65 15 15 5

 60 15 20 8

 55 15 25 5

 60 18 15 7

 50 25 15 10

9
 A standard TPN regimen for an adult contains:

 Glucose 250g, lipid (20% intralipid) 500 ml

 Amino acid (nitrogen) 14g

 Sodium 100 mmol

 Potassium 100 mmol

 Vitamins A, B, C, D, K

 Signs of malnutrition include:

 Cheilosis

 BMI 18.0kg/m2

 Serum albumin about 3.5g/dl

 Lymphocyte count < 1500/m3

 Hb of 11.0g in a woman aged 45

10
CHAPTER 12
NUTRITION IN SURGERY
ANSWERS

 A, B, C, D, E 29. D

 A, B 30. A, B, C, D, E

 A, B, C, D, E 31. A, B, D

 A, C, E

 C, E

 D

 B, C

 A, C

 A, B, C

 A, B, C

 A, C, D

 A, C

 C

 A, D

11
 A, C, D

 B

 B

 A, B, E

 A, C, D

 B, C, E

 A, B, C, D, E

 A, C

 A, C, D, E

 A, C, D

 B, C

 A, C, D

 A, B, E

 A, B, C, D, E

12
CHAPTER 13
MANAGEMENT OF THE INJURED PATIENT

 Which of the following measures is/are suitable for control of


external bleeding from a limb at the site of an accident?

 Elevation of the part


 Local pressure on the main artery at a convenient point
 Application of a tourniquet in severe injury
 Application of a clean bandage
 Splinting the Lamb

 Which of the following statements is/are true of the use of a


tourniquet to control external bleeding at the scene of an
accident?

 Should be reserved for rare occasions of severe injury to the


extremities
 Once applied should be clearly labeled
 Should not be covered by clothes
 Should be loosened at 20-minutes intervals for periods of 2 minutes
during transportation
 The person applying it should see to its removal.

 Which of these conditions is/are recognized complications of rib


fractures?
 Haemothorax
 Pneumothorax
 Surgical emphysema
 Rupture of the spleen
 Laceration of aortic arch

 A rib fracture is best treated by:


 Adequate doses of systemic analgesic
 Strapping of the corresponding side of the chest
 Blockage of the appropriate intercoastal nerve and active
physiotherapy
 Open external fixation
 External fixation

 Which of the following developments suggest(s) cardiac


tamponade?
 Raised venous pressure with systolic descent

1
 Reduction in cardiac output
 Pulsus paradoxus
 Pulsus alternans
 Collapsing pulse

 Which of the following developments suggest(s) cardiac


tamponade?
 Fluctuating but generally falling blood pressure
 Weak hart sounds and increased cardiac dullness
 Radiologically enlarged cardiac shadow
 Decreased pulse pressure
 Pericardial friction rub

 Which of the following organs is most likely to be affected in a


penetrating injury of the abdomen?

 Small intestine
 Spleen
 Liver
 Colon
 Pancreas
 In which of these injuries would the serum amylase level be raised
above normal?

 Ruptured bladder
 Fractured pelvis
 Duodenal perforation
 Rectus muscle haematoma
 Ruptured spleen
 In which of these injuries would the serum amylase level be raised
above normal?

 Duodenal rupture
 Gastric perforation
 Pancreatic injury
 Intestinal perforation
 Liver injury
 Which of the following radiological features is/are often seen in
the patient with ruptured spleen?

 Loss of the splenic outline on coned views


 Loss of t he left psoas shadow
 Elevation of the left cupola of the diaphragm
 Indentation of the gastric air bubble

2
 Pneumoperitoneum
A man of 40 who sustained a crushing injury when his V.W car
was in collision with a minibus presented in Casualty with severe
upper abdominal pain and dyspnoea. On examination he was
anxious, not anaemic with a pulse of 96/min and a BP of
110/75mmHg. His chest was clear, he was tender in the right
hypochondrium and his liver was palpable and tender. The bowel
sounds were normal and rectal examination was unremarkable.

 The most likely clinical diagnosis is:


 Ruptured right kidney
 Intestinal perforation
 Subcapsular haematoma of the liver
 Haemothorax (right)
 Ruptured liver

 You would advise:


 Serial collection and examination of urine
 Urgent laparotomy
 Careful observation of the patient for systemic and local signs
 Thoracostomy and under-water seal drainage
 None of the above.
A child who fell from a tree presents in the casualty Department
after the incident with abdominal pain and increasing distension.
On examination he is slightly pale, pulse 100/min, BP 110/70 Hg
and his peripheral perfusion is judged normal. His abdomen is
mildly tender but there is no guarding. A patch of ecchymesis is
developing in the left upper quadrant and left lower chest; no
chest signs are detectable.

 The most likely clinical diagnosis is:


 Ruptured kidney
 Ruptured liver
 Intestinal perforation
 Rectus muscle haematoma
 None of the above
 You would advise:
 Immediate laparotomy
 Serial collection of urine for examination
 Thoracostomy

3
 Four quadrant tap or peritoneal lavage
 Plain X-ray of the abdomen
 Which of the following may be encountered as complications of
management of hepatic injuries?

 Atelectasis
 Empyema
 Purulent intraperitoneal collection
 Secondary haemorrhage
 Thrombocytopaenic purpura

 Which of the following may be recognized at laparotomy for


trauma as signs of duodenal injury?

 Glossy retroperitoneal oedema


 Crepitations in the retroperitoneal area
 Haematoma or petechiae over the ascending and transverse colon
 Bile staining of the periduodenal tissues
 Contusion of right kidney

 Which of the following could be regarded as indications for


exploration of retroperitoneal haematomas?

 Lesions adjacent to the duodenum


 Lesions adjacent to ascending colon
 Lesions adjacent to descending colon
 Associated ileal injury
 Associated pelvic fractures
 Which of the following may be associated with renal injuries?
 Chest injuries
 Bladder rupture
 Urethral rupture
 Rupture of the lumbar muscles
 Rupture of the spleen

 Which of the following may be encountered as complications of


renal injuries?
 Secondary bleeding
 Perirenal infection
 Hydronephrosis
 Thrombosis of renal artery
 Renovascular hypertension

A man of 35 who was involved in a R.T.A was seen in the Casualty

4
Department 24h complaining of severe lower abdominal pain. He
had passed urine with some difficulty initially but denied
haematuria. He was pale and anxious and an enlarged tender
bladder was felt in the lower abdomen; a small blood clot was
seen at the tip of the penis and no urinary extravasation was seen,
the perineum was normal. No pelvic fractures were detected.

 Which of the following injuries has he sustained?


 Intraperitoneal rupture of the bladder
 Extraperitoneal rupture of the bladder
 Intrapelvic rupture of the urethra
 Rupture of bulb of the urethra
 Fracture of the shaft of the penis
 You would advise:
 Urethral catheterization under aseptic conditions and
urethrocystogram
 Urgent laparotomy after resuscitation
 Urgent exploration of the bladder
 Suprapubic cystostomy and rail roading of a catheter through
urethra and bladder
 None of the above

A carpenter who fell across a beam is brought to the Emergency


Room 48h after the incident with lower abdominal pain, fever and
vomiting. On examination he was ill, pale and showed
extravasation of urine over the scrotum, penis and lower
abdomen. There was a tender perineal haematoma.

 The most likely diagnosis is:


 Rupture of the membranous urethra
 Rupture of the bulbous urethra
 Rupture of the penile urethra
 Extraperitoneal rupture of the bladder
 Intra-peritoneal rupture of the bladder
 You will advise:
 Suprapublic cystostomy and appropriate antibiotics
 Conservative treatment and appropriate antibiotics
 Exploration and primary suture
 Exploration and rail roading of a catheter through urethra and
bladder
 Urethrostomy
 Which of these positions may scalp haematoma occupy?

5
 Subaponeurotic
 Subcutaneous
 Subperiosteal
 Intracutaneous
 Extradural
 Scalp haematoma should be treated:
 By needle aspiration
 By incision and evacuation
 Conservatively and should settle in 1-10 weeks
 By exhibition of broad spectrum antibiotics
 By weekly injection of hyaluronidase
 Which of these features may be regarded as frequent modes of
presentation of chronic subdural haematoma?
 Headache
 Vomiting
 Fluctuating level of consciousness
 Psychotic changes
 Pseudo bulbar palsy

A man of 45 involved in a R.T.A. suffered a transient loss of


consciousness. He drove home, but later in the evening he
complained of headaches and on his way to hospital became
rather drowsy. The casualty officer noted a right parietal
tenderness with some oedema and his right pupil which was
marginally larger reacted poorly to light.

 The most likely clinical diagnosis is:


 Acute subdural haematoma
 Subarachnoid haemorrhage
 Extradural haemorrhage
 Cerebral oedema
 Cerebral laceration
 You will advise:
 Cerebral decongestants
 Lumbar puncture
 Exploratory burrholes
 A parietal osteoplasticflap
 Ventricular tap
 In the unconscious patient a fixed dilated pupil is due to which of
the following developments?

 Ipsilateral injury to the occulomotor nerve

6
 Optic nerve damage
 Pontine injury
 Extradural haematoma
 Damage to the optic chiasmma
 Indication(s) for elevation of a closed, depressed fracture include:
 Radiographic evidence of indriven bones spicules
 Evidence of cerebral compression
 Depression causing disfigurement, especially in the forehead
 Depression more than 4mm in functionally important areas of the
cortex
 Depression in a child without evidence of compression.
 Noted complications of head injuries include:
 Subarachnoid haemorrhage
 Meningitis
 Cerebral abscess
 Fat embolism
 Hypernatraemia and hypernatraemia
 Chronic subdural haematoma:
 Is commonly caused by trauma
 Commonly presents with mental changes
 Is treated by craniotomy and evacuation
 Haematoma enlarges because of defective haemostasis
 May be confused with cerebrovascular accident

 Raised intracranial pressure complicating had injury:


 May cause brain herniation
 Is usually caused by the accumulation of vasogenic oedema fluid
 May be treated by hyperventilation
 May be treated by the use of a ventriculostomy
 Causes neuronal damage via ischaemia
 The severity of head injury may be indicated by:
 The duration of retrograde amnesia
 The presence of a skull fracture
 The Glasgow coma score
 The duration of anterograde amnesia
 The obliterationof CSF spaces on head CT scan
 Systemic causes of secondary brain injury include:
 Pypexia
 Hypotension
 Hypoglycaemia

7
 An extradural haematoma
 Hypocarbia
 The most common cause of cardiorespiratory failure in a spinal
cord injury patient is:

 Severe systemic sepsis


 Distended abdomen
 Fat embolism
 Hypovolaemic shock
 Air embolism
 The hallmarks of neurogenic shock are:
 Tachycardia
 Hypofension
 Hyperpyrexia
 Bradycardia
 Warm extremities
 In the investigations of acute head injury:
 MRI scan is the investigation of choice
 Plain x-rays are very useful
 The presence of skill fracture on x-rays increases the incidence of
a haematoma 6x.
 X-rays are mandatory in penetrating injuries
 LP is hardly indicated

 Which of the following may be regarded as complication/s of


arterial injuries?
 Arteriovenous fistula
 False aneurysm
 Volkmann’s contracture
 Paralysis
 Pulmonary embolism
 At the scene of an accident first aid measures:
 Should include a check on the adequacy of the airway
 Must ascertain adequacy of the circulation
 Should ensure institution of protective measures in the movement
of victim
 Must assume the possibility of spinal injuries until proved otherwise.
 Should assign the highest priority to blood loss
 At the road side practicable respiratory resuscitative measures
should include:
 Positioning the patient in the supine position with the neck

8
extended
 Early endotrachael intubation of the severaly injured
 Scooping out vomited gastric contents with gauze or handkerchief
 Strapping an unstable segment of chest wall with a pad
 Inserting a needle in the second intercostal space of the patient with
tension pneumothorax
 At the road side practicable respiratory resuscitative measures
should include:
 Grasping the tongue between thumb and index finger and drawing
it forwards in the unconscious patient.
 Maintaining patency of the airway by working the angles of the
mandible with the thumb
 Fixing a clean cloth over any open chest wound
 Lying the patient on the paradoxically moving segment of chest wall
during transportation
 Aspiration of the chest of the patient mildly dyspnoeic with
haemothorax
 In the transportation of the injured person to hospital:
 The ideal means is a ground or air ambulance
 Speed of transit often determines the prognosis
 The position of choice in transit is supine with the head to the side
 The unconscious patient should have endotracheal intubation
 All resuscitatory effort should continue unabated in transit
 Emergency or crashroom care of the injured person:
 Aims at consolidating what has been achieved at the road side
 Emphasizes control of life threatening asphyxia
 Ensures control of continuing external bleeding by means of
haemostats and ligatures
 Should ensure that the patient is made fit for any exploratory
procedure necessary
 Includes careful monitoring of the patient’s vital signs, and his
peripheral perfusion

 In the emergency or crashroom, respiratory resuscitative


measures should include:
 Suctioning of the trachea of the unconscious patient
 Intubation with a cuffed endotracheal tube if suctioning fails to
improve the airway
 Nasotracheal intubation where the oral cavity is the main site of
trauma

9
 Due consideration for tracheostomy in patients with respiratory
difficulties early
 Transfer of the patient with paradoxically moving chest to
management by intermittent positive pressure ventilation
 In the emergency or cashroom, respiratory resuscitative
measures should include:
 Suctioning of the trachea of all severely ill patients
 Endotracheal intubation of all unconscious patients
 Nasotracheal as opposed to endotracheal intubation of all patients
with obstructed airway
 Control of the paradoxically moving chest by strapping
 Relief of tension pneumothorax by passage of intercostal tube drain
through second intercostal space.
 Emergency or crashroom management of the injured patient
should ensure:
 Administration of enough blood and fluid to prevent shock
 Accurate monitoring of the patient’s vital signs
 Tetanus prophylaxis through toxoid administration in open wounds
or burns
 Administration of a bolus dose of a broad spectrum bacteriocidal
antibiotic in all cases of abdominal trauma
 Thorough investigation for other causes of unconsciousness
besides head injury
 Emergency or crashroom management of the injured patient
should ensure:
 Rapid infusion of blood and fluids to bring the blood pressure within
normal limits
 Careful monitoring of the central venous pressure and hourly urine
output as best indices of tissue perfusion
 Tetanus prophylaxis through antitetanus serum administration in
open wounds and burns
 Administration of bolus dose of a broad spectrum bacteriocidal
antibiotics in all cases of thoracic trauma
 Administration of bolus dose of a broad spectrum bacteriocidal
antibiotics in all cases of significant head injury
 In the definitive care of the patient with multiple injuries:
 A team work approach is rewarding
 In general, priority is determined by the rapidity with which injuries
affect cardio-respiratory function
 Judicious use of abdominal paracentesis and lavage expedites the
diagnosis of latent injuries
 Orthopaedic injuries are usually kept splinted and treated on a
semi-emergency basis.
 Hand injuries are deserving of early attention taking precedence

10
over other closed fractures.
 Judicious use of FAST enhances early diagnosis
 In the definitive care of the patient with multiple injuries:
 The work of the team of experts should be co-ordinated by an
experienced general surgeon
 Tension pneumothorax or open sucking wounds should take
precedence over head injuries
 Use of abdominal paracentesis and lavage makes no difference to
timing of diagnosis
 Orthopaedic injuries should always be managed under the same
anaesthetic with abdominal injuries
 Contaminated wounds or compound fractures should be dealt with
within the shortest possible time.
 Thoracic trauma:
 Account for 25 per cent of deaths following automobile accidents
 Produces derangement in respiratory movements with
hypoventilation and respiratory acidosis
 Should be managed by urgent attention to the airway
 May demand attention to open sucking wounds, pneumothorax and
haemothorax
 Cannot be adequately managed without chest X-ray.
 A rib fracture:
 Occurs at point of impact or at its weakest point which is the angle
 If uncomplicated is not a serious problem except in the elderly in
whom respiratory acidosis could supervene
 May be complicated by pneumothorax and surgical emphysema
 May in the elderly lead to lung collapse, consolidation, and
respiratory failure
 Should be treated by intercostals nerve block and active
physiotherapy
 Multiple rib fractures:
 May produce a flail chest
 May produce a “stove” in chest
 May through paradoxical chest movements produce a mediastinal
flutter
 Usually increases intrapulmonary shunting and thereby produce
dyspnoea, tac-typnoea and even cyanosis
 If causing significant paradoxical movements need stabilization by
intermittent positive pressure ventilation
 Ruptured diaphragm:
 Usually results from blunt trauma or crush injuries
 Usually occurs on the left side
 Is often indicated by appearance of intra-abdominal structures in

11
the left chest
 Presents with shoulder tip pain as a frequent feature
 Is often attended by high incidence of strangulation of herniated
bowel

 Ruptured diaphragm:
 Usually occurs from vibration injuries
 Occurs with equal frequency on both sides
 Is often attended by severe shock
 Nearly always presents with shoulder tip pain
 Is best seen on a lateral chest X-ray
 Haemothorax:
 Results from bleeding originating from the systemic as well as
pulmonary circulation
 Causes collapse of underlying lung
 Eventually leads to a reduction of cardiac output
 Can be reliably diagnosed on clinical grounds
 Should be so treated as to achieve complete and sustained re-
expansion of the lung
 Rupture of a major airway:
 May result from blunt trauma
 Usually causes dyspnoea associated with haemoptysis
 Is an important cause for tension pneumothorax
 Should be treated by early thoracotomy and repair of the tear in the
passages
 May lead to broncho-stenosis, infection and lung tissue destruction
 Bullet wounds of the abdomen:
 Usually call for exploratory laparotomy
 Are often associated with chest injuries
 Usually show a small entry point but considerable exit damage
 From a shot gun at close range is regarded as particularly
devastating
 Having a better prognosis compared to stab wounds
 Diagnosis of blunt abdominal injuries:
 Is frequently made difficult because of clouding of consciousness
from associated head injuries or alcoholism
 Is much helped by information on mechanism of injury
 Is facilitated by a reliable record of the vital signs during
observation of the patient
 Should be entertained in event of a rising white cell count,
particularly levels above 15 x 10-3 per dl
 Affecting the pancreas, stomach, duodenum, intestine should be

12
suspected if serum amylase levels are elevated and sustained.
 Diagnostic abdominal paracentesis in abdominal trauma:
 Is most useful in patients who are comatose from associated head
injuries or alcoholic intoxication
 Has a diagnostic accuracy of 85 per cent
 Is usually productive of blood that does not clot on standing
 Has the practical value of shortening the period of observation for
patients with occult injuries
 Should avoid areas where bowel could be attached to the parieties.
 Peritoneal lavage in abdominal trauma:
 Represents an improvement on the accuracy of diagnostic
paracentesis
 Is performed through a low midline incision under local anaesthesia
 Is best evaluated by a microscopic and chemical analysis of the
perfusate
 Suffers from the disadvantage of enhanced sensitivity
 May be positive in many cases of retro-peritoneal haematoma not
requiring surgical exploration
 Delayed rupture of the spleen:
 More often represents only a delayed recognition of splenic injury
 May result from digestion of initial clot by enzyme released by
concomitant damage to tail of the pancreas
 May be diagnosed earlier by means of selective celiac angiography
in suspicious cases
 Should be treated by splenectomy
 Presents a more formidable operative problem than primary rupture
 Ruptures of the liver:
 Are usually the result of crush injuries to the lower chest and upper
abdomen
 May present as subcapsular haematoma
 Should be explored on suspicion
 If treated by resection should have T-tube drainage of the
extrahepatic biliary tree
 Are associated with thoracic injuries in nearly 50 per cent of cases.
 Ruptures of the liver:
 May follow vibratory forces to the abdomen
 If significant frequently perish at the road side
 Should be treated by simple drainage alone if simple and not
actively bleeding at operation
 If survived leave no stigmata of hepatic insufficiency
 Are associated with head injuries in nearly 50 per cent of cases.
 Pancreatic injury:

13
 Is more often inflicted by penetrating trauma
 Is often associated with stomach, liver and duodenal lesions
 Is diagnosed when high serum amylase levels are accompanied by
peritoneal signs
 Need exploration on suspicion
 If explored should be adequately drained
 The duodenum:
 Is most frequently injured in its second part
 Is more frequently injured by penetrating trauma
 The site of injury is often detected late because signs are frequently
in abeyance
 If suspected to be injured should be urgently explored
 The site of injury is liable to fistula formation

 Colonic injuries:
 Usually result from blunt injuries following road traffic accidents,
falls and impalement
 May be masked in delayed injuries resulting from confusions or
ischaemia
 Should be explored on suspicion by laparotomy
 If simple and linear may be treated by suture in two layers
 If extraperitoneal should be treated by exteriorization
 Colonic injuries:
 May be produced by foreign bodies which have passed through the
alimentary canal
 May be detected by careful physical, rectal and sigmoidoscopic
examination
 Should have a pre-operative antibiotic cover, e.g. ampicillin, 3rd
generation cephalosporins
 If extensive should be treated by resection and primary
anastomosis
 Having an invariably poor prognosis
 Retroperitoneal haematoma:
 Most commonly results from blunt abdominal trauma
 Presents considerable diagnostic difficulty
 Is most commonly associated with pelvic fractures
 Gives rise to a positive peritoneal tap in 50 per cent of cases
 Gives rise to a positive peritoneal tap in 10 per cent of cases
 Retroperitoneal haematoma:
 Most commonly results from penetrating abdominal trauma
 Should be suspected in patients showing hypotension after
abdominal trauma but no overt abdominal signs or external

14
haemorrhage
 When expanding indicates associated injury of the great vessels
 Is suggested by tender mass in the flanks
 Should be explored when adjacent to pelvic fractures
 Renal injuries:
Occur most commonly in road traffic accidents (35 per cent of such
cases)
 May be associated with injuries to the lumbar muscles and fractures
of the spine
 Should be suspected where loin pain follows abdominal trauma
 May present with a renal mass representing extravasation of urine
 In 80 per cent of cases respond to conservative measures
 Ureteric injuries:
 Commoly accompany renal injuries
 If extraperitoneal may produce pelvic or loin swelling
 May present as pain in the flanks or lower abdomen following
crushing trauma
 Show on isotopic renogram as increased counts on the side of the
ligated or damaged ureter
 Presenting as anuria should be urgently explored and treated by
bilateral ureter-onoecystostomy

 Urteric injuries:
 May follow trauma producing hyperextension of the spine
 May produce urinary vaginal fistula
 May show urographically as ureteric dilatation, hydronephrosis or
extravasation of contrast
 May how urographically as non-functioning kidney of affected side
 Presenting late should need management for uraemia and infection
prior to surgical repair
 Intraperitoneal rupture of the bladder:
 May be caused by road traffic accidents and contact sport
 Is usually follow trauma to lower abdomen in a patient with a full
bladder
 Is suggested by a history of anuria, after trauma with few drops of
blood stained urine and ascites and peritonitis
 Can be confirmed by retrograde cystogram
 Should be treated by early repair of the vesical tear and bladder
drainage by urethral or susprapublic catheter.
 Extraperctoneal rupture of the bladder:
 May complicate pelvic fractures
 Usually follow lower abdominal injury in a patient with empty
bladder

15
 Is suggested by a tender suprapubic mass with extravasation of
urine after lower abdominal trauma
 Peritonitis is usual clinical presentation.
 Should be treated by preliminary supra-pubic cystostomy and
delayed accurate repair of bladder tear
 Injuries to the membranous urethra:
 May be “complete or “partial”
 Usually occur just beyond the apex of the prostate gland
 Usually lead to extravasation of urine on to the perineum
 Should be suspected in the patient who after abdominal trauma has
urethral bleeding and difficulty in micturating
 Should be treated in the early lesions by urethral catheterization
under aseptic conditions.
 Injuries to the membranous urethra:
 May co-exist with vesical injuries
 Is seen in 10 per cent of pelvic fractures
 Seldom lead to extravasation of urine on to the perineum or the
groin
 Should be suspected in a patient who develops tender palpable
bladder with blood at the tip of the penis after abdominal trauma
 If overlooked for more than 48 h should be treated by suprapubic
cystotomy only in the first instance.

 Closed injury to the bulbous urethra:


 Is likely to be produced by straddle injury
 Leads to extravasation of urine spreading to penis and scrotum
 Is liable to lead to infection if not treated early
 May be treated conservatively if the patient passes urine freely and
there is minimal perineal swelling
 If even late may be treated by exploration and primary suture
 Closed injury to the penile (pendulous) urethra:
 May result from injudicious instrumentation
 May cause urethral bleeding and penile swelling
 Should be investigated by means of urethral catheterization under
aseptic conditions
 May be treated conservatively in the absence of urination problems
 May be complicated by urethra-cutaneous fistula formation
 Skull fracture:
 Is more often compound than simple
 Is not a reliable guide to the severity of a head injury
 If basal may be difficult to see on an X-ray
 If linear and of the vault requires no special treatment besides

16
observation
 May contribute to the decision to admit a head injury for observation
 Extradural haematoma:
 Is a haematoma between the skull and the dura
 Is usually found in the temporal region
 Usually occurs in children without a skull fracture
 Is attended by early pupillary constriction on the side of the lesion
followed by dilatation
 Should be treated by urgent evacuation of haematoma and arrest of
bleeding through exploratory burr holes
 Extradural haematoma:
 Is a haematoma between the bony skull and its periosteum
 May occasionally be seen in the parietal or frontal region
 Is not always attended by loss of consciousness
 May be marked by a slowing pulse and a rising blood pressure
 May be complicated by epilepsy and a rising blood pressure.
 May be complicated by epilepsy in a third of survivors
 Subdural haemorrhage:
 Is acute if it occurs within 48h and usually complicates severe head
injuries
 In the acute form may be difficult to distinguish from an extradural
haemorrhage
 May occasionally result from rupture of an aneurysm or angioma
 Often presents with fluctuation in level of consciousness
 Shows diagnostic changes on carotid angiography (postero-anterior
views)

 Subdural haemorrhage:
 Is subacute when it occurs between 2-14 days after injury
 Increases in size by repeated haemorrhages or osmostic absorption
of fluid.
 In the chronic form may be associated with a history of head trauma
in about half the patients
 Not uncommoly presents as a psychosis leading to admission to a
mental hospital
 Should be treated by exploratory burr holes in the temporal, parietal
and frontal positions.
 The patient that is unconscious with a head injury:
 Should be positioned on his side with a slight head down tilt
 Should be so positioned to ensure a clear airway which is checked
by regular suctioning
 Should have adequate skin care to forestall bed sores

17
 Should receive judicious infusion of intravenous fluids in the first
48h.
 May benefit from a course of dexamethzone 10mg I/V, then 5mg
i/m 6 hourly for 4 days.
 Fractures of the base of the skull:
 Classically present as C.S.F. leakage from the nose or ear
 Should have any fluid leakage from nose or ear tested for sugar
 Should be treated with bed rest and systemic antibiotics
 If not responding to conservative measures should be treated with
repair of the dural tear by a fascial patch
 May be responsible for intracranial aeroceles
 Cerebreal oedema:
 Is common complication of head trauma
 Clinically mimics intracranial haematoma
 Is much aggravated by hypoxia
 May be treated by infusion of 20 per cent mannitol (250-500ml)
administered over about 20 minutes
 Is attended by a high incidence of late post-traumatic epilepsy.
 Cerebral oedema:
 Is a constant threat to the life of the victim of head injury
 Is confirmed at exploratory burr holes when the brain is soft,
swollen and contused
 Is often aggravated by associated chest disease or injury
 Should be treated by a regime including restriction of fluid to 2L and
sodium 150mmol in 24h
 May be complicated in the first few days by epilept form fits

 Post-traumatic epilepsy:
 Is recognized as early if it occurs within one week of injury
 If early takes the form of a focal motor attack
 Of the late type has an incidence of 5 per cent but tends to be
persistent
 Should be treated prophylactically, identifying all patients with a
predisposing type of injury
 If early may be treated with intramuscular diazepam 10mg
 Post-traumatic epilepsy:
 Is recognized as late if it occurs after one week after injury
 Of the early variety has an incidence of 5 per cent of non-missile
injuries
 Of the late type has an incidence of 15 per cent and tends to persist

18
 Should be treated with phenobarbitone 30 mg t.i.d. in adults and
children over 12 years
 Has a worse prognosis than idiopathic epilepsy
 Arterial injuries:
 I f of the contusion type are more likely to follow fractures and
dislocation of joints
 Should be suspected when paralysis, pain, paraesthesiae and
pallor are noted in a limb after injury.
 Should be treated by early exploration after seeing to fractures and
dislocations
 Affecting vessels less than 5mm in diameter are treated by ligation
rather than repair
 Should after exploration be managed by 100 elevation of the
affected limb
 Features to be observed in the Glasgow Coma Scale include:
 Eye opening
 Corneal reflex
 Pupillary reaction to light
 Motor response and patellar reflex
 Verbal response
 In the Glasgow Coma Scale a patient with eye opening to pain,
incomprehensible verbal response, and forearm flexion to pain
has a score of:
 10
 9
 8
 7
 6

 A patient with minor head injury and no loss of consciousness


who is sent home and not hospitalized should be instructed to:

 Walk around his room frequently in the first 48h

19
 Take a light diet
 Take aspirin or paracetamol if he has headache
 Take antacids to prevent acute gastric erosions
 Return immediately to hospital if a member of the family is anxious
about him
 If a patient with minor head injury without loss of consciousness
is sent home and not hospitalized, his relations should be
instructed to look out for:

 Vomiting
 Frequency of urine
 Abnormal behavior or restlessness
 Weakness of an arm or leg
 Increasing sleepiness
 Primary or impact brain damage produces:
 Intradural haematoma
 Cerebral confusion
 Brain swelling
 Diffuse axonal injury in the white matter
 Subarachnoid haemorrhage
 “Minor” head injuries not requiring hospitalization include:
 Small scalp laceration without loss of consciousness
 Simple fracture of the cranial vault without loss of consciousness
 Concussion
 Minor contusion of scalp with post traumatic amnesia of less than 5
minutes.
 Fracture base of skull without C.S.F. otorrhoea.

20
CHAPTER 13
MANAGEMENT OF THE INJURED PATIENT
ANSWERS

 A, B, C, D 39. A, B, C 77. A, D,
E
 A, B, C, D, E 40. A, B, C, D 78. A, B,
C, D
 A, B, C, D 41. A, C, D, E 79. A, B,
D, E
 C 42. A, B, C, D 80. B, C,
D, E
 A, B, C 43. A, B, C, E 81. A, B,
C, D, E
 A, B, C, D 44. A, B, C, D, E 82. B, C,
D, E
 A 45. A, B, C, D, E 83. A, B,
C, D, E
 C 46. E 84. A, B,
C, D, E
 A, B, C, D 47. A, B, C, D, E 85. A, B,
C, D, E
 B 48. B, D 86. A, B,
C, D, E
 C 49. A, B, C, D, E, F 87. A, B,
C, D
 A, C 50. A, B, E 88. A, C,
D, E
 E 51. A, B, C, D, E 89. A, B,
C, D, E
 A 52. A, B, C, D, E 90. A, B,
D
 A, B, C, D 53. A, B, C, D, E 91. A,B,
C, D, E
 A, B, C, D 54. A, B, C, D, E 92. A, E
 A, B, C 55. C, D, E 93. D
 A, B, C, D, E 56. A, B, C, D, E 94. B, E
 A, B, C, D, E 57. A, B, C, D, E 95. A, C,
D, E
 C 58. A, B, C, D 96. B, D

21
 A 59. A, B, C, D, E 97. A
 B 60. A, B, C, D, E
 A 61. A, B, C, D, E
 A, B, C, D 62. A, B, C, D, E
 C 63. A, B, C, E
 A, B, C, D 64. A, B, C, D
 C 65. A, B, C, D, E
 C 66. A, B, C, D, E
 A, B, C, D 67. A, B, C, D
 A, B, C, D, E 68. A, B, C, D
 A, B, C, D, E 69. A, B, C, D
 A, B, D, E 70. B, C, D
 A, C, D, E 71. A, B, C, D, E
 C, D, E 72. B, C, D, E
 A, B, C, D 73. A, B, C, D, E
 A 74. A, B, C, D, E
 B, D 75. A, B, C
 D, E 76. A, B, D

22
CHAPTER 14
BURNS

 A Burn:
 Is coagulative destruction of skin which may be caused by hot water
 Occurs most commonly in children
 Is pathologically different from scald
 Injury depends on the temperature of the heat source but not on the
duration of exposure of the tissues to such temperature
 In the adult may be due to RTA
 In deep partial thickness burns destruction of the following occur:
 Lymph vessels
 The hair follicles and sebaceous glands
 Sweat glands
 Red cells
 Capillaries
 Mediators released after burns include:
 Histamine
 Cyclo-oxygenase pathway products
 Serotonin
 Prostaglandins
 Thyroxine
 Cold water immersion after thermal injury:
 Increases the risk of infection
 Reduces pain after partial thickness burns
 Can result in hypothermia if applied to large burns
 Has been reported to stabilize mast cells
 May decrease oedema if applied immediately
 The protein content of oedema fluid in burns may be as much as
(g/1):
 60
 20
 30
 40
 50
 The electrolyte composition of oedema fluid in burns:
 Has a higher sodium concentration than plasma
 Has a lower potassium concentration than plasma
 Has a higher bicarbonate concentration than plasma

1
 Has a lower chloride concentration than plasma
 Is the same as that of plasma

 In burns:
 The maximum oedema fluid obtainable is about 10% of body
weight or 50% of total E.C.F
 There is initial vasodilatation followed by vasoconstriction
 The rate of fluid exudation from the capillaries is highest in the first
6h. and falls in 48h.
 Increased capillary permeability may occur even in unburnt areas
 Immediate charring of the skin lessens external and subcutaneous
fluid loss
 Anaemia in burns may be caused by:
 Thrombosis
 Increased red cell fragility
 Changes of dressings
 Bone marrow depression
 Skin grafting
 In burns:
 There is depression of cellular and immunological mechanisms
 Leucocytes cannot phagocytose bacteria normally
 There is depression of lysosomes
 IgG is not affected
 Transfer of leucocytes and antibodies to sites of bacterial infection
is rapid and not depressed
 In burns:
 Renal failure is commonly caused by toxic substances produced in
the burnt skin
 The urine may occasionally be port-wine
 Uraemia is always associated with oliguria
 There may be necrosis of the distal convoluted tubules
 The renal vessels are dilated
 Organisms that may infect a burn in the first 24h include:
 Proteus
 Streptococcus
 Pseudomonas

2
 Esch. Coli
 Staphylococcus
 Predominant organisms in a scald wound by seven days include:
 Staphylococcus
 Pseudomonas
 Pneumococcus
 Clostridium tetani
 Streptococcus

 In the exposure method of treating burn wound:


 Antibiotic spray, povidone or mercurochrome may be used
 The eschar inhibits bacterial growth
 Any environment is suitable
 It is not necessary for attendants or visitors to be gowned or
masked
 The method should end as soon as the integrity of the eschar is
broken
 Silver sulphadiazine cream:
 Is highly antibacterial and prevents pseudomonas and other
infections
 May cause severe hyponatraemia and hypokalaemia
 May cause argyria
 Is non-toxic but painful
 Is applied daily or on alternate days with or without dressing
 Silver nitrate dressing:
 Is effective in the prevention or treatment of pseudomonas infection
 Is applied every 24h and is time consuming
 Stains beddings and the floor
 Leaches sodium, potassium and chloride from the burn wound
 Is widely used.
 Lypphilized heterograft of pig skin:
 Limits fluid and electrolyte loss from the burn wound
 Is vascularized
 Does not harbour infection
 Should be changed weekly until an autograft is available
 Is a xenograft
 Exposure method is suitable for burns of the:

3
 Face
 Neck
 Entire trunk
 Perineum
 Axilla
 Curling’s ulcer:
 Occurs only in the stomach and duodenum
 Occurs only in severe burns
 Is often complicated by bleeding
 May be prevented by the administration of antacids or cimetidine to
severely burnt patients in the first three weeks
 Is best treated by partial gastrectomy or vagotomy and drainage

 The commonest cause of death in burns is:


 Shock
 Renal failure
 Burn wound sepsis
 Septicaemia
 Pneumonia
 After severe burns:
 There is positive nitrogen balance
 The resting metabolic expenditure may be 150-200% of normal
 High calorie high protein diet is essential
 Enteral nutrition can always provide all the energy and protein
 The protein requirement can be estimated from the urinary nitrogen
 Prognosis in burns depends on:
 Age of patient
 Extent of burn
 Depth of burn
 Respiratory tract injury
 Pre-existing disease
A 20-year old West African male in Lagos weighing 60kg sustained

4
burns
of the face, right upper limb and anterior part of the trunk
following a fire
in is bedroom at 12 midnight. He is brought to the ‘Casuality’ at
2.00 a.m.
 Your first concern will be to:
 Check his pulse and blood pressure and administer morphine.
 Remove his clothes and determine the extent and depth of his burn
 Check the rate of his breathing
 Find out if the airway is clear
 See if there is singeing of his nasal hair or soot in the nostril
 Examination of the burnt area shows that there are blisters, areas
of erythema and other areas of skin which are all sensitive to pin
prick.
The burn is:
 Superficial partial thickness
 Deep partial thickness
 Full thickness and superficial partial thickness
 Partial thickness (superficial and deep)
 Full thickness

 The burn is estimated at 30% of his body surface. The total fluid
requirement (in litres) in the first 24h is about:
 5.5
 6
 6.5
 7.2
 7.5
 The fluids will consist of:
 2L 5 per cent dextrose, IL blood and the rest as Ringer’s lactate
 Ringer’s lactate

5
 2L blood and the rest as ringer’s lactate
 2L 5 per cent dextrose and the rest as Ringer’s lactate
 IL 5 per cent dextrose; 2L dextran 70 and the rest as Ringer’s
lactate
 Half the fluids should be given by:
 6a.m
 8a.m
 10a.m
 12noon
 2p.m
 Initial investigations will include:
 Haematocrit
 Blood group
 Serum electrolytes and blood urea
 Culture of the burnt skin
 White cell count and differential
 The antibiotic to administer at this stage is:
 Chloramphenicol
 Cloxacillin or penicillin
 Gentamicin
 Oxytetracycline
 Cephaloridine
 The most important parameter to monitor is:
 Quarter-hourly pulse and blood pressure
 Central venous pressure
 Filling of peripheral veins
 Hourly urinary output
 Skin temperature and orientation

 After cleaning the burn wounds with one per cent centrimide you
will apply:
 Terracortril spray daily
 0.5% silver nitrate dressing and add more silver nitrate every 3h

6
 Silver sulphadiazine cream on alternate days without dressing
 Mafenide creame twice daily
 Povidone-iodine daily
 The fluid requirement (in litres) in the second day is
approximately:
 4
 4.5
 5
 5.5
 6
 On the second day the urine is found to be port-wine in colour.
You will after examining the urine:
 Restrict fluid intake
 Order an urgent I.V.P.
 Give blood
 Give 10% mannitol
 Give sodium bicarbonate
 On the 8th day he complains of some pain in the wound, anorexia
and generally being unwell. His temperature is 38.50C and the skin
around the discharging wound looks reddened.
The clinical diagnosis is:
 Cellulitis
 Lymphangitis
 Septicaemia
 Burn wound sepsis
 Necrosis of the burn
 Effects of electrical injuries include:
 Cardiac or respiratory arrest
 Ventricular fibrillation
 Auricular fibrillation
 Rupture of blood vessels
 Thrombosis of vessels
 Late complications of electrical burn injuries include:
 Impotence
 Cataract
 Epilepsy
 Intractable headache
 Abnormal gait

7
 In electrical injuries:

 Direct current is more dangerous than alternating current


 The action potentials of muscles and nerves are altered
 The contact burns are small and deep whether the tension is high
or low
 Fluid loss is massive and cannot be estimated from the body
surface area burnt
 Amputation may be necessary
 A patient with flash burns of the left side of the head, neck, left
arm and lower extremity (leg) has suffered what percentage body
surface burn?
 9
 18
 5
 30
 27
 The advocates of the open treatment of burns consider it superior
to the closed method because:
 It tends to reduce the incidence of burn contractures
 It tends to reduce the oedema in the burned area.
 It is more economical of nursing care
 It keeps the skin surface cool and dry and thereby deprives micro-
organisms of an environment favoruable for multiplication
 It saves on antibiotics
 Impairment of immunity in trauma or burn patients is associated
with:
 Abnormalities of neutrophil function
 Immunosuppressive factors in the serum
 Polycythaemia
 Decreased lymphocyte mitogenic response
 Circulating suppressor leucocytes
 Smoke inhalation is in the early post-burn phase associated with:
 Hypoxaemia
 Increased platelet adhesiveness
 Hypercapnia
 Abnormal chest X-ray
 Elevated carboxyhaemoglobin
 Concentration of organisms per gram of tissue indicative of ‘burn
wound sepsis’ is:

8
 102
 103
 104
 105
 106

 In the Burn tissue:


 It is invaded by Gram-ve organisms within 24hrs
 It is invaded by Gram+ve organism with 7 days
 Bacteria invade the eschar and also by direct spread
 Sepsis in full thickness is poorly controlled because of reduced
blood supply
 Local invasive infection present as septicemia
 In the burn patient the following complications may occur:
 Cushing ulcers
 Splenic eulargement
 Gastric dilatation
 Pyelonephritis
 Flash backs may occur
 In the burn patient with cardiopulmonary complication:
 Pulmonary oedema may follow inhalation injury
 Inflammation not important here
 Aspiration of gastric contents plays a role
 Tracheitis of intubation may occur
 Heat from inhaled gases plays a part
 For prevention of post burns contracture:
 Physiotherapy should start from day 1
 Splinting is very important
 Night splints may be used up to 6 months
 Pressure garments are mandatory
 Early surgery is useful
 In burn prevention:
 Education should be limited to the home environment
 Education on cooking methods is very important with regards to
scalds in children
 Appropriate design of materials for household appliances should be
considered

9
 Inflammable liquids should not be stored in the home
 Epileptics should be banned from getting near fires
 In electrical burn:
 Extent of injury may be difficult to assess
 In the presence of myoglobinuria IV lasix in indicated
 Infusion of IV fluids must be at the rate to increase urine output to
above
50 mls/hr
 Excision of necrotic muscles is indicated 48hrs after diagnosis
 Silver sulphadiazine is contraindicated in this type of burn.

 Chemical burns may be caused by:


 Cement
 Strong acids
 Strong alkali
 Petroleum products
 Concentrated brine
 Principles of management in burns:
 Restore
 Reconstruct
 Repair
 Revive
 Rehabilitate
 In burns the following investigations on blood taken from the
patient is important:
 Cortisol levels
 Adrenaline levels
 Haematocrit
 Blood area
 Creatinine

10
CHAPTER 14
BURNS
ANSWER

 B, E 35. A, B, C, D, E
 A, B 36. B, D, E
 A, B, C, D 37. B
 B, C, D, E 38. D
 E 39. A, B, D, E
 E 40. A, C, E
 A, D 41. D
 A, B, C, D, E 42. C, D
 A, C 43. C, D, E
 B, D 44. A, C, D, E
 B, E 45. B, C, E
 B 46. B, C, E
 A, B, E 47. A, C
 A, E 48. A, B, C, D
 A, C 49. A, C, D, E
 A, E 50. C, D

11
 A, B, D
 C, D
 C
 B, C, E
 A, B, C, D, E
 D
 D
 C
 D
 B
 A, B, C
 B
 D
 C
 E
 D, E
 D
 A, B, D, E

12
CHAPTER 15

PRE-PERI- AND POST-OPERATIVE CARE AND POST-OPERATIVE


COMPLICATIONS

 Investigations that MUST be done as part of the pre-operative assessment


of fitness before an emergency appendicectomy in a 30 year old man
include:

 Chest X-ray

 Urinalysis for albumin, casts and sugar

 Stool R/E

 Sickling and haemoglobin

 White cell count

A 40-year woman is to have partial gastrectomy for a simple gastric ulcer.

 In the pre-anaesthetic assessment, the points to be noted in the history


include:

 Previous operation and anaesthetic experience

 Drug therapy

 Previous dysentery and urinary difficulty

 Social habit regarding alcohol and tobacco

 Concurrent disease



 The operation should be postponed if she has:

 Cold

 Diabetes for which she has been receiving treatment

 A history of asthma

 A history of deep vein thrombosis

 A haemoglobin of 10g/dl

1


 Clinical examination does not reveal any abnormality. The investigations


that MUST be done include:

 Chest X-ray

 Blood urea and electrolytes

 E. C. G.

 Liver function tests

 Pro-thrombin, bleeding and clotting times



 Obesity predisposes to:

 Airway obstruction

 Respiratory infection

 Wound infection

 Urinary tract infection

 Deep venous thrombosis



 A diabetic for operation must:

 If possible be placed in the middle of the operation list

 Have his blood sugar done before and after the operation

 Not be given insulin before operation

 Have his urine tested 4-hourly after operation

 Be put on routine antibiotics to prevent infection



 In a patient with typhoid perforation, which one of the following may


cause cardiac arrest or affect reversal of muscle relaxants after operation

2
if it is not corrected before operation?

 Hyponatraemia

 Hypochloraemia

 Hypokalaemia

 Dehydration

 Anaemia



 According to the classification of the American Society of


Anaesthesiologists, the grade of a patient:

 With gastric outlet obstruction is 4

 With a haemoglobin of about 10g/dl but with no symptoms of


breathlessness in 3

 With respiratory obstruction from a foreign body is 2

 With SS or SC disease is 3

 Who is moribund and is not expected to live 24h with or without surgery is
6



 In the peri-operative management of a patient:

 Anaesthesia can be started with one cylinder of oxygen on the anaesthetic


machine

 It is advisable for the person conducting the anaesthesia to prepare the


intravenous agents himself

 There is no need for an assistant when anaesthesia is being induced

 A vein at the back of the hand is always preferable to one at the forearm
for setting up a drip

 The rate of infusion can be safely increased by squeezing the plastic


container.

3


 In the anaesthesized patient in the supine position:

 The legs may be crossed if necessary

 If the arm is not used for infusion the hand may be placed under the
buttock

 If it is necessary to use an arm-board for infusion, the arm should not be


higher than the shoulder to prevent traction on the brachial plexus

 There is no need for any measures if the patient is longer than the table

 The kidney bridge can be used





 In the lateral position:

 The tidal volume is not affected

 There is increased ventilation and increased perfusion of the upper lobes

 There is decreased ventilation and increased perfusion of the lower lobes

 Retention of carbon dioxide may occur if the patient breathes


spontaneously

 The kidney bridge does not affect breathing



 Concerning the position of the anaesthetized patient:

 The tidal volume is decreased by the prone position but not by the steep
Trendelenburg position

 Venous bleeding is less and the intracranial pressure low in the sitting
position

 There is a tendency to hypertension in the sitting position

 The steep Trendelenburg position may cause cerebral oedema

 In the prone position a firm pillow must be placed under the shoulders and

4
another under the abdomen



 In the monitoring of the anaesthetized patient:

 The cause of a rise or fall of more than 15mmHg from the resting blood
pressure should be found

 Cardiac failure is the likely cause of a high C.V.P. and hypotension

 A collapsed and immobile reservoir bag while the patient is breathing


suggests air-way obstruction

 The colorimetric method of estimating blood loss can be done during the
operation

 The respiratory rate should be between 12 and 30



 In the anaesthetized SS or SC patient:

 Oxgen given should be 50 per cent if the haemoglobin is less than 8g/dl

 The transfusion trigger can be 8.0g/dl

 An intravenous infusion should always be set up

 It is not essential to replace blood as it is lost

 Transfused blood should be as fresh as possible



 Hypotension in the peri-operative period may be caused by:

 Early carbon dioxide retention

 Vagal inhibition

 Mismatched blood transfusion

 Reaction to pain or fear

 Anaesthetic agents

5


 Vagal inhibition (viscera-cardiac reflex):

 Results in cessation of respiration and a fast heart if the heart beat can be
heard

 May be caused by dilatation of the anus or cervix

 Does not lead to cardiac arrest

 May be treated by deepening the anaesthesia

 May be prevented or treated by intravenous injection of atropine



 Cardiac arrest during operation may be caused by:

 Increased release of catecholamines and hypoxia from asphyxia

 Excessive blood loss

 Hyponatraemia

 Myocardial infarction

 Hyperventilation



 Inflation and deflation of an anaesthetized patient with an endotrachael


tube are difficult. Possible causes include:

 Extended head

 Blood in the endotrachael tube

 Pressure on the trachea from without

 Hypercapnia

 Prior regurgitation or vomiting

An anaesthetized patient vomits and develops dyspnoea, wheezing,


cyanosis and tachycardia.

6
 The most likely clinical diagnosis is:

 Bronchopneumonia

 Collapse of a lung

 Asthma

 Exudative pneumonitis

 Acute heart failure



 Management will include:

 Head-up position and pharyngeal suction.

 Intubation and suction of the trachea and bronchi and irrigation with small
amounts of saline

 Administration of oxygen and halothane

 Administration of adrenaline, steroids and antibiotics

 Administration of digoxin



 Which of the following observations is/are MANDATORY after an


abdominal operation such as cholecystectomy?

 Rate and volume of respiration

 Pulse and blood pressure chart

 Central venous pressure

 Four-hourly temperature

 Intake-output chart





 In the care of the airway in an anaesthetized patient recovering from

7
unconsciousness:

 The lateral position is more effective than the semi-prone in draining


secretions from the month

 Insertion of an airway in the mouth always ensures a patent airway and so


need not be checked

 The chin should be supported with the fingers and the head extended

 An endotrachael tube should not be left in situ until the patient recovers as
it is most uncomfortable

 Suction should be available at all times



 Post-operative hypoxaemia:

 Increases with age and does not occur after operations which do not
interfere with respiratory movements

 Occurs even when the duration of anaesthesia is less than 20 minutes

 Is worse after spontaneous breathing than after artificial ventilation

 May be caused by a cold environment as in an air-conditioned theatre

 Is in the majority of patients treated by ensuring that the inspired oxygen is


at least 20 per cent of the air breathed.



 Oxygen administration:

 If done by a mask may give an inspired oxygen concentration of 40-50


per cent

 Of greater than 50% is needed in most patients.

 If prolonged may cause pulmonary oedema in certain patients

 In sicklers should be for only 24 hours

 May lead to apnoea in patients with chronic respiratory diseases.



8
 The incidence of post-operative vomiting is influenced by:

 The choice of the volatile anaesthetic agent used

 The site of operation

 Duration of operation

 The choice of premedication

 Dilatation of the cervix



 In the post-operative care of a 45-year old patient after splenectomy:

 Analgesics should be given 6-hourly for 48 h

 The wound should be inspected and dressed every other day

 Heparin should be given to prevent deep vein thrombosis

 The serum electrolytes, PO2 , PCO2 and pH should be checked daily

 Breathing exercises, coughing and expectoration should be encouraged.





 About 30 minutes after partial gastrectomy, a patient who has not


regained consciousness is cyanosed and very dyspnoie. The first
condition to consider is:

 Aspiration of vomitus

 Obstruction of the airway

 Massive pulmonary collapse

 Residual paralysis following the use of a long-acting muscle relaxant

 Myopathy



 Measures taken in the treatment of massive pulmonary collapse include:

9
 Administration of morphine

 Chest percussion, postural drainage, coughing and expectoration

 Bronchoscopy

 Administration of aminophylline and ampicillin

 Oxygen theraphy



 Haemorrhage 24 hours postoperatively may be caused by

 Dislogement of a blood clot

 Oozing from a raw area

 Fragments of malignant prostrate, lung or brain in the circulation

 Incompatible blood transfusion

 Infection causing necrosis of a blood vessel

Fifteen minutes after vagotomy and gastro-jejunostomy, a 35-year old


man who at the end of the operation had a blood pressure of 120/80
mmHg and a regular pulse of 90/min and of good volume, is found by the
sister-in-charge of the recovery ward not to be breathing. His blood
pressure is unrecordable and there is no demonstrable heart beat.

 The most likely clinical diagnosis is:

 Death

 Cardiac arrest

 Myocardial infarction

 Respiratory failure

 Cerebrovascular accident



 His condition could have been caused by:

 Obstruction of the airway

10
 Shock

 Excessive administration of oxygen

 Transfusion at 2 litres of 10-day old blood of room temperature

 Aspiration of vomitus



 Your initial step(s) will be to:

 Examine the patient yourself

 Request an urgent ECG

 Hyperextend the bead, open the mouth, pull out the tongue and apply
suction if necessary

 Start external cardiac massage and artificial ventilation by month to mouth


breathing

 Increase the rate of intravenous infusion



 Your next step will be to:

 Administer adrenaline intravenously

 Administer digoxin intravenously

 Apply a face mark with the head still extended and give 100% oxygen by
manual compression of a breathing bag such as ambu bag

 Administer nikethamide I. V

 Take blood for urgent Hb, serum electrolytes, PO2 , PCO2 and pH



 Fifteen minutes after the diagnosis the patient is still unconscious, you
will:

 Continue to give oxygen

 Stop any further activity

11
 Continue external cardiac massage

 Administer procaine amide, more adrenaline and nikethamide I.V.

 Request an urgent X-ray of the chest and pulmonary angiogram



 Which of the following drugs may be used in the management of cardiac


arrest?

 Digoxin

 Lignocaine hydrochloride

 Magnesium sulphate

 Sodium bicarbonate

 Adrenaline



 Which of the following drugs may be used during the management of


cardiac arrest?

 Naloxone

 Isoprenaline

 Calcium chloride

 Propranolol

 Amiodarone



 In external cardiac massage:

 The rate of compression should be 80/minute

 The heels of the hand are used

 The heels of the hand may be placed over the sternum or sternum and
adjacent ribs

12
 The patient should be supine and on an ordinary bed

 30% of the normal blood flow may be provided





 Complications of external cardiac massage include:

 Fractured ribs

 Laceration of the liver

 Perforation of the stomach

 Laceration of the spleen

 Laceration of the diaphragm



 Important parameters to be monitored during cardiopulmonary


resuscitation include:

 Blood pressure

 Carotid or radial pulse

 Size of the pupil

 Level of consciousness

 Spontaneous respiratory effort



 Postoperative convulsions could be due to

 Pyrexia

 Epilepsy

 Hypocalcaemia

 Hyponatraemia

13
 Water overload



 Malignant hyperpyrexia after anaesthesia:

 Is rare and is caused by muscle-membrane disorder

 Is consistently associated only with suxamethonium and halothane

 Presents with rigidity, tachypnoea, sweating and rapid rise in temperature


after the agent is given

 Occurs only on the first exposure to the anaesthetic agent

 Is treated by cooling the patient and administering lignocaine or


procainamide for arrhythmias



 Sclerema:

 Occurs in older children

 Is caused by hypothermia following excessive heat loss during operation

 Is characterized by stiff immobile joints and hardness of the skin

 May end in respiratory insufficiency because of pneumonia

 Is prevented by operating on the patient in warmed up towel and switching


off air-conditioners in the theatre.



 Atelecasis:

 Is a common post-operative complication especially after thoracic or


upper abdominal operations

 Usually occurs 5-7 days after operation

 Is commonly caused by hypoventilation and impaired coughing

 Can be prevented by prophylactic antibiotic therapy

 Predisposes to bronchopneumonia

14


 Two days after sigmoid colectomy, a 50-year old man develops pyrexia,
rapid shallow breathing, cyanosis, cough and pain in the left side of his
chest. On examination, respiratory movements are diminished and
percussion is dull in the left lower chest. Breath sounds are diminished on
that side.



 The most likely diagnoses includes

 Pulmonary embolus

 Bronchio pneumonia

 Pulmonary atelectasis

 Tracheo-bronchitis

 Aspiration pneumonia



 An X-ray of the chest may show:

 Diffuse patchiness of a lobe of the left lung

 Consolidation of a lobe of the left lung

 Raised diaphragm

 Mediastinal shift towards the left

 Fluid in the left pleura



 Pulmonary embolism:

 Is in 90% of patients preceded by clinical evidence of thrombosis in the


leg

 In the majority of patients is caused by emboli originating in the calf veins

 In most cases occurs within the first 14 days after operation

15
 May mimic pneumonia or myocardial infarction

 Does not show any abnormal ECG changes



 Which of the following drugs may be used in pulmonary embolism?

 Morphine

 Atropine

 Sodium bicarbonate

 Urokinase

 Streptokinase



 Indications for embolectomy in pulmonary embolism include:

 Impending death

 Persistent hypotension

 Persistent hypoxia

 Low cardiac output

 Right ventricular failure



 Post-operative hiccough may be caused by:

 Irritation of the trachea by anaesthetic gases

 Irritation of the mucosa of the oesophagus or stomach by anaesthetic gases

 Retention of urine

 Distended stomach

 Peritonitis



16
 Failure to pass urine 12 hours postoperatively in spite of adequate fluid
therapy.

 Is usually caused by heightened parasympathetic activity

 Maybe caused by pain

 Is the most commonly caused by urethral obstruction

 May not give rise to a palpable distended bladder

 Is best treated by catheterization

Twenty-four hours after partial gastrectomy, a 45-year old man has


passed only 200ml of urine, Catheterization yields a further 100 ml of
urine.

 Your provisional diagnosis will be:

 Acute renal failure

 Acute tubular necrosis

 Pre-renal renal failure

 Post-renal renal failure

 Dehydration

It is found that the blood pressure dropped to 80/60 mmHg for about 30
minutes during the night. The intravenous fluid administered during the
previous 24 h was 3 litres and the nasogastric aspiration was 1.5 litres.
Clinically the blood pressure is now 120/80Hg and the skin is dry and
warm.

 Your tentative diagnosis will be:

 Acute renal failure

 Acute tubular necrosis

 Pre-renal renal failure

 Post-renal renal failure

 Acute glomerulonephritis

Examination of the urine gives the following results: osmolality 300

17
mOsm/L, red cells, casts and albumen present, urea 140 mmol/L, sodium
70mmol/L The blood urea is 10 mmol/L.

 The diagnosis is:

 Chronic renal failure

 Acute tubular necrosis

 Pre-renal renal failure

 Post-renal renal failure

 Acute glomerulo-nephritis



 You will give in the next 24h:

 300ml of mannitol 20 per cent in 20 minutes followed by 4500ml of


crystalloids

 4500ml of crystalloids

 About 2000ml of crystalloids

 1L of crystalloids

 Frusemide I. M. + 4500ml of crystalloids



 The most likely clinical diagnosis is:

 Paralytic ileus

 Subacute intestinal obstruction

 Gastric outlet obstruction

 Gastric dilatation

 Uraemia



 Treatment will include:

18
 Enema saponis

 Passage of nasogastric tube and continous aspiration

 Administration of intravenous fluid

 Urgent laparotomy

 Administration of antibiotics



 Enterocolitis:

 Is a lethal diarrhoeal disease caused by saph albus

 Follows prolonged oral intake of antibiotics or abdominal surgery

 Causes hypotension, pyrexia and distended, painful and tender abdomen

 Is confirmed by microscopy and culture of the stools

 Is treated with aggressive fluid therapy and administration of crystalline


penicillin, cloxaclillin, erythromycin or lincomycin



 Wound infection:

 Is when a wound is painful, tender, swollen and reddened

 Is most commonly caused by Esh, coli

 Is related to the duration of operation or poor surgical technique

 Usually occurs 2-3 days after operation

 Is treated with antibiotics until it heals





 Abdominal wound dehiscence (burst abdomen):

 Occurs in vertical but not transverse incisions

19
 Occurs most commonly on the 7th post-operative day

 Has a high mortality

 Is in 90% of patients associated with post-operative cough, vomiting and


abdominal distension

 Is preceded by serous discharge from the wound



 Factors associated with wound dehiscence include:

 Malignancy

 Uraemia

 Excessive Vit E administration.

 Liver disease

 Obesity



 Oestrogens and oral contraceptives:

 Increase fibrinogen and prothrombin

 Have no effect on factors VII and VIII

 Accelerate aggregation of platelets

 Have no effect on antithrombin III

 Increase factors IX and X



 The most important factor in post-operative thrombo-embolism is


probably:

 Obesity

 Venous stasis

 Age

20
 Post-operative changes in the coagulating factors

 Roughening of the endothelium



 Clinical features of deep vein thrombosis:

 Is more common in the right limb

 Occurs commonly in the produnda femoris veins

 Is symptoms-free in about two-thirds of patients

 May have pyrexia as the first symptom

 In most cases given a positive Homan’s sign



 Heparin:

 Is antithrombin

 Is antiprothrombin

 Is antithromboplastin

 Depresses manufacture of prothrombin

 Depresses manufacture of factors VII, IX an X

CHAPTER 15

 B, D

 A, B, D, E

 A

 B

 A, B, C, E

 B, D

 C

21
 D

 B, E

 C

 C, D

 B, D

 B

 B, C, E

 B, C, E

 B, E

 A, B, D

 B, C, E

 D

 B

 A, B, D, E

 C, E

 A, D

 A, C, E

 A, B, C, D, E

 E

 B

 A, B, C, E

 A, B, C, D

 B

 A, B, E

22
 C, D

 C

 A, C

 B, D, E

 B, C, E

 B, E

 A, B

 C, D, E

 B, C, E

 A, C, E

 B, C, E

 A, C, E

 B, C

 A, C, D

 C, D

 A, B, C, D, E

 B, C, D, E

 B, D, E

 B,D

 A

 C

 B

 C

 D

 B, C

23
 C, D

 C

 D

 A, B, D, E

 A, C, E

 D

 C, D

 A, B, C



24
CHAPTER 16

SUPERFICIAL SOFT TISSUE SWELLINGS

 Keloid:

 Is characterized by deposition of an unduly large amount of


collagen which is raised above the skin level but does not
extend beyond the margin of the wound.

 Occurs in the darkly pigmented but not white races

 Results from the undue prolongation of the phase of collagen


deposition and maturation

 Is associated with a predisposition which is inherited as a


dominant character

 Appears after four months of the wound and continues to


grow beyond a year.

 Keloid:

 Is found at all ages

 May occur in any part of the body

 If quiescent, may start growing again during pregnancy

 May become malignant

 Is the same as hypertrophic scar

 SATISFACTORY methods of treating keloids include:

 Excision

 Excision followed by radiotherapy

 Injection of triamcinolone acetonide into them

 Excision followed by injection of a solution made of nitrogen


mustard, hyalase and triamcinolone into the wound

1
 None of the above

 A sebaceous cyst:

 Contains desquamated keratin, sebum, fat and cholesterol


crystals

 Can occur in any part of the body although the commonest


sites are the scalp and face

 May rarely become malignant

 Is spherical, firm, attached to skin and has a definite edge

 Is lined by columnar epithelium

 Dermoid cyst:

 Results from transplantation of a piece of epithelium into the


underlying soft tissue during an injury.

 Often contains hair

 If external angular in position may have an intracranial


extension

 May be the cause of a pre-auricular sinus and a sinus of the


suprasternal space

 Is often seen on the fingers of tailors

 A ganglion:

 Is mucinous degeneration of connective tissue arising from a


tendon sheath or the synovial membrane or capsule of a joint

 Does not occur in the palm

 Is usually unilocular

 Is tense or soft, fluctuant, circumscribed and attached to skin

2
 Is best treated by aspiration of its contents and injection of
hydrocortisone sclerosant

 Varruca vulgaris:

 Is usually seen in the elderly

 Is often multiple and occurs commonly in the hands and


knees

 Is an epidermal overgrowth caused by a virus

 Is spherical and usually about 0.5cm or more in diameter

 Disappears with the surface application of salicylic acid

A 25-year old man has two swellings over the right greater trochanter.
The swellings are firm, mobile, circumscribed, non-tender,
subcutaneous and attached to the deep fascia.

 The most likely clinical diagnosis is:

 Fibromata

 Lipomata

 Implanatation dermoids

 Calcified guinea worms

 Onchocercomata

 Naevi(moles)

 Are present in every human being

 Get more numerous from birth till adulthood

 Are always flat and hairless

 Are collections of melanocytes always in the basal layer of


the epidermis

3
 Should be excised as they may become malignant.

 Signs of malignant change in a naevus include:

 Bleeding

 Ulceration

 Hairiness

 Increase in size

 Increase in pigmentation

 Desmoid tumour:

 Is a rare encapsulated fibrosarcoma

 Arises from the muscular aponeurosis of the anterior


abdominal wall usually above the umbilicus

 May originate at the site of operation scar or injury

 Occurs mostly in parous women in the third or fourth decade

 Is treated by wide excision followed by irradiation

 Haemangiomas:

 Are congenital tumours involving blood vessels

 If capillary are intradermal and not raised above the skin


surface

 If cavernous are elevated above the skin surface

 If capillary may involve muscle, nervous tissue or bone

 If cavernous may be associated with dyschondroplasia

4
 The treatment of cavernous haemangioma at the age of five is:

 Leave alone

 Excision

 Injectionof a sclerosant

 Application of carbon dioxide snow

 Cryosurgical using liquid nitrogen

 Neurofibroma:

 Arises from the nerve cell

 May be associated with scoliosis

 Is inherited as a Mendelian dominant

 May become malignant

 Is excised in most patient s

 Lipoma:

 Is a swelling of abnormal fat enclosed in a capsule of


connective tissue

 Does not occur beyond t he subcutaneous region

 Is not found in the palm or sole

 Is soft but not fluctuant

 May be multiple, small and painful

 Squamous cell carcinoma

 Is commoner in Africans than in Caucasains

 Is commonest in the lower limbs in Africans and in the head

5
and neck region in Caucasians

 Spreads early to the lymphatics but vascular spread is


unusual

 Is microscopically well-differentiated in the majority

 May arise from sinuses following chronic would infection.

 In the treatment of squamous cell carcinoma:

 If the underlying bone is not involved wide excision or radio


therapy is effective.

 If the underlying bone is involved, excision followed by the


radio therapy is the treatment of choice

 Block dissection should be done routinely whether or not the


lymph nodes are palpable.

 Recurrence after irradiation responds well to further irradiation

 Cytotoxic chemotherapy has been found effective in about


50% of patients.

 Cytotoxic drugs currently useful in squamous cell carcinoma


especially of the head and neck include:

 5-Fluouracil

 Methotrexate

 Cyclophosphamide

 Cisplatin

 Bleomycin

 Basal cell carcinoma:

 Is as common in Caucasians as it is in Africans

6
 Occurs in the face in 90% of patients

 Has a raised everted edge

 Spreads by direct infiltration and lymphatics but not by the


blood s stream

 Is best treated by local excision followed by adjuvant


chemotherapy.

 Malignant melanoma:

 Is twice as common in women as in men

 In Caucasians is found most commonly in the lower leg

 Of the nodular type is the commonest variety

 Of the lentigo maligna type is very slow-growing, occurs


mainly in older age group

 Of the eye is equally common in Africans and causasians

 In the treatment of malignant melanoma:

 A subungual lesion is treated by amputation of the affected


finger

 The lesion is widely excised and followed by radiotherapy

 Block dissection is usually done only if or when the regional


nodes are enlarged and mobile

 Fixed regional nodes are irradiated

 Lesions may rarely disappear spontaneously

 Cytotoxic drugs of benefit in malignant melanoma include:

 Vincristine

7
 Melphalan

 Adriamycin

 Actionmycin D

 Dacarbazine

 In the prognosis of malignant melanoma:

 Lesions on the extremities have a better prognosis than those


on the trunk

 Lesions confined to the primary site have a 5-year survival of


about 50% and those with regional lymph nodes involvement
15%.

 Nodular lesions have the best prognosis

 Men have a better prognosis than women

 The 5-year survival is better than for most other forms of


cancer if it is diagnosed early and treated adequately.

 Dermatofirosarcoma:

 Is thrice as common in females

 Occurs mostly in the abdominal wall

 Is usually nodular and may resemble a neurofibroma

 Is of low-grade malignancy, locally invasive and tends to


recur repeatedly after excision

 Is treated by wide excision or radiotherapy

 Kaposi’s sarcoma:

 Is a multifocal sarcoma of blood vessels

8
 Is most common in Jews, Italians and Africans and the highest
incidence in Africa is in Uganda

 Has recently been observed in a highly malignant form


among Americans, especially homosexuals, suffering from
acquired immune-deficiency disease.

 Occurs in all age groups and is twice as common in males in


adults and children.

 Histological is of two types – monomorphic and mixed.

 Nodules of Kaposi’s sarcoma are found in:

 Bones

 Intestines

 Brain

 Lungs

 Liver

 The skin nodules in Kaposi’s sarcoma:

 In the white-skinned are brown

 Do not normally ulcerate

 Are often preceded by oedema which in the lower limb


affects the whole limb

 May spontaneously become flattened and hypopigmented

 May follow a chronic, indolent benign course

 Lymphadenopathic Kaposi’s sarcoma:

 Is usually seen after 15 years

9
 May be localized or generalized

 Can easily be differentiated from Hodgkin’s or tuberculosis

 Has a good prognosis

 X-ray of bones in Kaposi’s sarcoma may show:

 Pathological fractures

 Sclerosis

 Osteoporosis

 Bone destruction but with thin cortical margin

 Cystic changes

 Kaposi’s sarcoma can be treated effectively by:

 Surgery

 Surgery and radiotherapy

 Radiotherapy

 Radiotherapy and chemotherapy

 Chemotherapy

 Cytotoxic drugs that may be used in the treatment of Kaposi’s


sarcoma include:

 Paclitaxel

 Chlorambucil

 Doxorubicin

 Vincristine

 Etoposide

10
 Fibrosarcoma:

 Is seen frequently in young adults

 May arise in a scar

 Is circumscribed, firm or hard and often warm

 Spreads by he lymphatics and blood stream

 Is best treated by wide excision and adjuvant chemotherapy

 Rhabdomyosarcoma:

 Is a rare malignant tumour of non-striated muscle

 Occurs in all age groups

 Spreads by lymphatics and blood stream

 Of the embryonal type is seen in childhood and in the trunk


and periphery

 Has a poor prognosis as early recurrence is the rule

 A hypertrophic scar

 During growth is raised above the skin and is itchy

 After growth is flat and wide

 Continues to grow after one year

 Pathologically consists of collagen nodules

 Is treated by excision

 A keloid differs from healthy skin by

 A rich vasculature

11
 Low mesenchymal density

 Thickened epidermal cell layer

 Presence of broad bundles of collagen

 Predominance of sweat glands

 Malignant melanoma

 Accounts for 4% of skin cancers

 Accounts for 70% of skin cancer related deaths

 Is increasing worldwide at a faster rate than any other cancer


– 10% yearly

 Has highest incidence in Auckland – 56.2 per 100,000

 Worldwide, the incidence has doubled over the last 10 years

 Risk factors in developing malignant melanoma include

 Pre-existing benign naevus

 Freckles

 High naevus count

 Increased CFC in the atmosphere

 Sunburn from age 20years

 Concerning melanoma

 Worldwide, the commonest is superficial spreading


melanoma

 Superficial spreading melanoma arises from a pre-existing


mole

 Nodular melanoma is seen most commonly on the legs and

12
trunk and grows slowly

 Acral lentiginous melanoma is the commonest form in blacks


and occurs in the soles, palms and nail beds

 Lentigo maligna melanoma is usually seen in the elderly and is


fast-growing

 WHO recommends that the excision margin should be 2cm if the


thickness of the melanoma is

 <1mm

 1-1.5mm

 1.5-2mm

 2.1-4mm

 >4mm

 Prognosis in malignant melanoma is better

 In those over 60 years

 If it arises in a pre-existing naevus

 In non-whites

 Genital sites

 In stable diabetics

 Rhabdomyosarcoma

 May be found in child aged 6-10yrs

 Is very fleshy and lobulated

 May be found in the bladder and vagina

13
 Hepatic metastases occur very early

 Wide excision of the lesion together with involved muscle is


the treatment of choice

 Rhabdomyosarcoma

 Is unlikely to respond to irradiation treatment

 Cyclophosphamide can be used in the treatment of this


tumour cyclically.

 Infants as young children are worse off progrostically


compared to ulcer patients

 Intracavitary rhabdomyoscra has the resemblance of bunch


of grapes

 Vinblashire can be used in the treatment within tumour

 Liposcraoma

 May simulate a benign lipoma

 May be seen commonly on the flanks

 Is usually very large in size

 Is treated by wide excision

 May be found in the abdominal cavity

 In Aids-related kaposi’s Sarcoma

 This form of the disease in slow growing

 Lesions here are centrifugal

 May occur in the mouth and throat of the patient

 Prognosis is poor

14
 May present a jigger

 In AIDS – related Kaposi’s Sarcoma

 Stool occult blood test is positive in the patient with skin


lesions

 FNAC may be required for diagnosis

 Lesions may be found in the stomach at oesophagus

 Sheletal survey may show bone lesions

 A differential diagnosis of this condition is pyogenic


granuloma

 The following are forms of Kaposi sacroma

 Nodular

 Fungating

 Fleshy

 Plaques

 Visceral

 Soft tissue sarcomas

 Arise from mesodermal tissue

 Account for 5% of tumours

 Arise in the lower limb in 80% of cases

 Have incidence of 2:100,000

 Are commoner in 5th and 6th decades

15
 In soft tissue sarcomas

 Macroscopically this tumour may consist of firm homogenous


or mucoid tissue

 Exposure to tanners can lead to this condition

 The tumour usually have a well defined capsule

 Histological type of this tumour is anglosarcoma

 Spread through lymphatics is rare

 The following are varieties of melanomas

 Nodular

 Acral lentiginous melanoma

 Subcutaneous

 Lentigo maligma melanoma

 Superficial necrotizing melanoma

 In subungual melanoma

 10% of cases occur in blacks

 The lesion may appear as a longitudinal pigmentation under


the nail

 Most are nodular melanomas

 The thumb may be involved

 One sees the commonest cause of in transit metastasis

16
 The following are important in the assessment of melanoma

 Tumour thickness

 Presence of lymph nodes

 Presence of ulceration

 Host response

 Lymphatic vessels invasion

 In investigations for melanoma

 Excisional biopsy is indicated

 Chest Xray

 Abdomal ultrasound of abdomen and pelvis

 Serum LDH

 Uric acid levels

 Bowan’s Disease

 Is extra-epithetial squamous carcinoma of low malignancy

 Is found only in sun exposed areas

 Occurs in both sun exposed and non sun exposed areas

 Is slightly palpable and crusted

 Response to topical 5% 5 FU cream application.

 Squancus cell carcinoma

17
 Is 2-3 times more common in woman

 It spreads by infiltrating surrounding and subjacent tissues and


bone

 Usually shows early lymphatic spread

 Seldom shows vascular metastases.

 May have a nodular floor.

 Marjolins ulcer is a squanous cell carcinoma caused by chronic


irritation with repeated breakdown and repair due to

 Burno

 Varicose vein

 Pressure sore

 Lonizing radiation

 Chronic ulcers

 Neurofibromatosis

 Is an autosomal recessive disorder

 Is composed of Schwann cells, fibroblasts, mast cells and


vascular components

 May be only composed of skin

 Exhibits a plexiform type known as type 2 neurofibromatosis

 May become malignant in 10% of deep seated lesions

 Lymphangioma

 It is a congenital malformation of the lymphatic channels

 In its superficial form consisting of lymph cyst is called cystic

18
hygroma

 May have associated elements of haemangioma

 May produce macroglossia

 Presenting as Cystic hygroma is usually compressible

Answers

 C

 C, D

 E

 A, C

 B, C, D

 A

 B, C

 E

 A, B

 A, B, D, E

 C, D

 B, C, E

 A

 B, D

 E

 B, D

 A, E

 B, D, E

19
 B

 A, D

 C, E

 B, E

 A, B, E

 B, C,D

 A, C

 A, B, C, D, E

 B, E

 B

 C, D, E

 D, E

 A, C, D, E

 A, B

 C, E

 A, B, D

 A, C, D

 A, B, D, E

 A, B, C, D

 A, B, D

 C, D, E

 B

 B, C, E

 A, B, D

20
 A, C, D

 C, D, E

 B, C, D

 A, D, E

 A, D, E

 A, C, D, E

 A, B, D

 B, C, D

 A, C, D, E

 A, B, C, D

 C, D, E

 B, D, E

 A. B, E

 B, E

 A, C, D

21
CHAPTER 44
ACUTE RENAL FAILURE, HAEMATURIA, HAEMATOSPERMIA
 Acute renal failure:

 In oliguria 24hr urine output is <400mls

 Extra – renal uraemia is due to irreversible renal hypoperfusion

 Improperly treated pre-renal uraemia may lead to acute tubular


necrosis

 Acute obstructive renal failure ultrasonography shows dilated


ureters and calyces

 The end products of metabolism that may accumulate include


phosphates and creatinine

 Extra-renal failure:

 Reduction in GFR is due to decrease in renal blood flow

 In cases where the fluid and electrolyte imbalance worsens further


decrease in renal blood flow results from vasoconstriction

 Rehydration and frusemide induce diuresis.

 There is decrease of sodium concentration in urine

 Urine urea concentration is markedly reduced

 Intrinsic renal damage:

 The causes may be due to aminoglycosides use

 The nephrotoxins tend to cause patchy tubular necrosis

 In situations where acute tubular necrosis is likely to occur ,


administration of mannitol or frusemide may be protective

 Herbal preparations may cause it

 There is high concentration of urine creatinine

1
 Acute obstructive uropathy:

 Alternating periods of oliguria and polyuria is suggestive

 Anuria following a major pelvic operation is frequently due to


ureteric injury or ligation

 Urine urea concentration is usually low

 Vesical schistosomiasis may present as anuria

 Pelvic tumours are common cause

 In the aetiology of acute renal failure, pre-renal failure may be


caused by:

 Entero-cutaneous fistula

 Ruptured ectopic pregnancy

 Myocardial infarction

 Cirrhosis of the liver

 Radiation

 Non-obstructive causes of anuria include:

 Cortical necrosis

 Severe acute tubular necrosis

 Ischaemia of transplanted kidney

 Pelvic neoplasia

 Bilateral renal artery occlusion

 In the investigation of a patient with acute renal failure:

 24hr urine output is usually less than 400mls

 Glycosuria, proteinuria and haematuria may point to ATN

 In renal hypoperfusion, the urine/plasma ratios of urea and


osmolarity are decreased

2
 Radiographic contrast media may cause renal damage

 Urine sodium over 30mmol/l may point to pre-renal uraemia

 In the management of pre-renal failure:

 Acute blood loss is best replaced with normal saline

 Adequacy of volume replacement is best determined using CVP


measurements

 In the use of frusemide to induce diuresis, the state of hydration is


not important

 The maximum dose of frusemide is 80mg in a 70kg adult

 Mannitol may precipitate pulmonary oedema

 In oliguric phase of ATN:

 There is metabolic alkalosis

 There is a rise in serum bicarbonate and potassium levels

 There are falls in plasma free calcium and blood platelets

 Dietary sodium intake is encouraged due to a fall in plasma sodium


levels

 The changes in body fluids can be determined by daily weighing of


the patient

 In the emergency treatment of hyperkalaemia

 Under ECG monitoring, 40mls of 10% calcium gluconate can be


given slowly

 50% dextrose (100mls) and 20 units of insulin as bolus or


continuous infusion is temporary measure

 Calcium resonium resin is used in patients with hypertension

 Serum potassium of 6mmol is an indication for dialysis

 ECG changes seen in hyperkaelaemia include tenting of T waves


and flattening of the P waves.

3
 Acidosis in renal failure:

 It ameliorates the effects of hyperkalaemia

 It is best treated using hypertonic sodium bicarbonate

 Treatment by dialysis is effective

 If treated with hypertonic sodium bicarbonate may be complicated


by pulmonary oedema

 It is a form of respiratory acidosis

 The indications for dialysis includes:

 Blood urea >30mmol/l despite conservative measures or daily rise


of urea above 5.5mmol/l

 Rapid rise of potassium to levels > 6mmol

 Fall of serum bicarbonate about 2mmol daily or below 10mmol

 Severe uraemia with gastro intestinal bleeding and dehydration

 Pulmonary oedema

 In peritoneal dialysis

 The peritoneum acts as a semipermeable membrane

 Crystalloids diffuse from the circulation to the dialysis fluid in the


peritoneal cavity if they are absent in the fluid

 Compared to haemodialysis, peritoneal dialysis requires less


equipment

 Open insertion of peritoneal catheter is preferred in suspected


adhesions from previous surgery

 The dialysis fluid may be hypertonic 6% glucose and 1000mls is


adequate

 Contra-indication to peritoneal dialysis include:

 Wound dehiscence

 Extensive infections of the anterior abdominal wall

4
 Polycystic kidneys

 Recent laparotomy

 Extensive burns of anterior abdominal wall

 In peritoneal dialysis:

 There may be protein loss into the dialysate

 Bladder may perforate leading to peritonitis

 Closed method of catheter insertion is preferred in an unconscious


patient

 The peritoneal catheter need not be secured as it needs to be


changed frequently

 Radio-opaque peritoneal catheter line is essential for radiological


demonstration

 Haemodialysis:

 Venous canulation can be inserted into the internal jugular vein

 Arteriovenous shunt may involve the use of the radial artery

 Heparin use is contraindicated in all situations as it may cause


excessive bleeding

 The patient’s blood is passed on the other side of cellophane


membrane in the same directions

 In emergency, haemofiltration using venous canulation can be used

 Disequilibrium syndrome:

 It follows rapid dialysis

 It is due to lagging of urea and bicarbonate in the brain behind


those in the blood

 Clinical presentation includes headaches, vomiting, fits and


unconsciousness

 It is treated by the administration of barbiturates orally

5
 Prevention is by slow but short periods of dialysis

 In the diuretic phase of acute renal failure:

 The urine output increases to over 2000mls/24hrs

 There is loss of sodium and gain in potassium

 May be observed as the first presentation by a patient

 Vigorous rehydration is essential

 Frequent measurement of body weight, blood urea, and creatinine


levels are needed for management

 The survival results after acute renal failure from tubular necrosis
is worse in:

 Surgical patients

 Non-surgical patients

 ICU-patients

 Patients less than 60yrs

 Cases due to vascular surgery

 Acute obstructive uropathy:

 The definition of the cause may require ureteropyelography

 Percutaneous nephrostomy is required in prostatic obstruction

 Emergency corrective surgery is mandatory

 In the presence of inoperable malignant lesions, ureterostomy is


contra-indicated

 Diuretic phase is not a feature

 Anaesthesia in renal failure:

 Acute renal failure can be prevented by maintenance of adequate


circulating blood volume

 Mannitol and frusemide to induce diuresis is required in short

6
procedures

 In established acute renal failure, digoxin, pethidine and morphine


are best avoided

 Sudden cardiac arrest is due to hypokalaemia and acidosis

 Fresh blood is required for transfusion ,if large quantities are


required

 Chronic renal failure:

 Over 90% of renal tissue is usually lost for nitrogenous waste


products to be retained

 Progress of renal failure may be accelerated by recurrent urinary


tract infection and hypertension

 Condition may be symptomless in early stages

 It can lead to epileptic attacks

 A low protein diet of 0.5g/kg body weight may be required.

 Haematuria:

 It is the abnormal presence of >3 RBCs in spun urine

 The presence of proteinuria as well suggests glomerular disease

 Red blood cells are absent in the centrifuged urine in chemical


haematuria

 It is usually due schistosomiasis in tropical countries

 Haematuria in sickle cell disease is commonly as a result of


papillary necrosis

 Terminal haematuria of recent onset in a 56 year old man is most


likely

due to:

 Vesical schistosomiasis

 Bladder calculi

7
 Renal cell Carcimona

 Prostatic calculi

 Urinary tract infection

 In the investigations of haematuria:

 Sterile pyuria may suggest genito-urinary tuberculosis

 Diverticulitis of bladder diverticulum may cause painful haematuria

 Haematuria from BPH is due to prostatic varices

 Haematuria from bladder diverticulum can be due to neoplasia

 Cystoscopy is of no value in a suspected upper urinary tract lesion

 The causes of haematospermia include:

 Prostatitis

 Schistosomiasis

 TRUS biopsy of prostate gland

 Congested urethra from sexual excess

 Urethritis

 In the management of haematospermia:

 Transrectal ultrasound helps detect lesion of the seminal vesicles

 VDRL is be indicated

 Urethrocystoscopy is required

 A five day course of antibiotics is usually adequate in the presence


of urethritis

 Urethral dilatation is contraindicated if the cause is a urethral


stricture

8
CHAPTER 44
ACUTE RENAL FAILURE, HAEMATURIA, HAEMATOSPERMIA
ANSWERS

9
 A,C,D,E 25. A, B, C, D

 A,B,C,D 26. A, B, C, D, E

 A,B,C,D 27. A, B, C

 A,B,D,E

 A,B,C,D

 A,B,C,E

 A,B,D

 B,D,E

 C,E

 A,B,C,D,E

 C,D

 A,B,C,D,E

 A,B,C,D,E

 A,B,D,E

 A,B,E

 A,B,E

 A,B,C,E

 C,E

 A,C,E

 A

10
 A,C,E

 B,C,D,E

 A,B,C,E

 B,E

11
CHAPTER 55
CANCER CHEMOTHERAPY

 Cytotoxic drugs can be used to CURE some cases of:

 Hodgkin’s lymphoma

 Thyroid carcinoma

 Burkitt’s lymphoma

 Choriocarcinoma

 Malignant melanoma

 In the cell cycle:

 DNA synthesis takes place in the S- (synthetic) phase

 The cell prepares for division in the G1 (pre-synthetic) phase

 Proliferating cells after mitosis enter the G2 (post-synthetic) phase

 Cells not in cycle but potentially able to do so are in GO (resting)


phase

 The duration of S- and G2- phases are 6-8h and 2h respectively

 Tumours cells susceptible to cancer chemotherapy are those:

 With long generation time

 High growth fraction

 That are not proliferating

 In the resting phase

 In the synthetic phase

1
 Non-proliferating cells comprise:

 Cells in the post-synthetic phase

 Nutritionally deficient cells

 Differentiated cells

 Dying cells

 Resting cells

 Cytotoxic anti-metabolites include:

 Methotrexate

 Thiotepa

 5 Fluoro-uracil

 Cytosin arabinoside

 Adriamycin

 Anti-metabolites:

 Inhibit nucleic acid biosynthesis

 Exert their maximum effect in the G1 (pre-synthetic) phase of the


cell

 Are structurally similar to normal metabolites of the cell

 Act as substrates for enzymes of metabolism they resemble

2
 Bind strongly and irreversibly to cellular enzymes thereby blocking

their activities

 Methotrexate is used in the treatment of:

 Nephroblastoma

 Choriocarcinoma

 Osteosarcoma

 Gastric carcinoma

 Head and neck squamous cell carcinoma

 Methotrexate:

 Inhibits dihydrofolate reductase

 Reduces the intracellular pool of reduced folates

 Activity is enhanced by folinic acid (cirtrovorum factor)

 Is usually administered orally or parenterally over a 3-5-day period

 Administered intrathecally is useful in meningeal leukemia and


Burkitt’s lymphoma

 Toxic effects of methotrexate include:

 Haemorrhagic cystitis

 Haemorrhagic enteritis

 Stomatitis

 Conjunctivitis

 Myelo-suppression

 Contra-indications to the use of methotrexate include:

3
 Renal disease

 Hypertension

 Liver disease

 Peptic ulcer

 Diabetes mellitus

 5-Fluoro-uracil:

 Is a potnt inhilbitor of thymidylate synthetase

 Inhibits purine synthesis

 Is eliminated through hepatic degradation and renal and gastro-


intestinal secretions

 May be administered intra-thecally for Burkitt’s lymphoma or


medulloblastoma

 Is usually administered intravenously weekly

 5-Fluoro-uracil is useful in the treatment of:

 Breast cancer

 Ovarian cancer

 Testicular carcinoma

 Carcinoma of the large bowel

 Lung cancer

 Toxic effects of 5-Fluoro-uracil include:

4
 Liver damage

 Bone marrow depression

 Gastro-enteritis

 Nephrotoxicity

 Alopecia

 6-mercapto-purine:

 Is a purine agonist

 Is used in the treatment of acute leukaemia in children

 Is given intravenously

 May cause oral ulceration and bone marrow depression

 Is nephrotoxic

 Alkylating agents:

 Are purine antagonists

 Activate and release chemical linkages that bind or cross-bind to

DNA strands

 Are the oldest group of drugs for cancer therapy

 Alkylate nucleic acid

 Can alkylate RNA and enzymes

5
 Alkylating agents include:

 Cyclophosphamide

 Thioguanine

 L-phenylalanine mustared

 Bleomycin

 Nitrogen mustard

 Cyclophosphamide is used in the treatment of:

 Osteosarcoma

 Burkitt’s and other lymphomas

 Bladder cancer

 Breast cancer

 Leukaemia

 Toxic effects of Cyclophosphamide include:

 Haemorhagic cystitis

 Alopecia

 Diarrhoea

 Bone marrow supression

 Exfoliative dermatitis

 Nitrogen mustard:

 Is the oldest alkylating agent

 Acts slowly

6
 Is administered or intravenously

 May cause vomiting

 Can be used to treat all lymphomas and neoplastic effusions

 Cytotoxic antibiotics include:

 Vinblastine

 Actinomycin D

 Endoxan

 Cisplatin

 Daunorubicin

 Actinomycin D:

 Is produced by a streptomyces species

 Binds with DNA and inhibits DNA directed RNA synthesis

 Is administered orally or intravenously

 Is usually given daily for 5 days every 6 weeks

 May cause diarrhoea, stomatitis or alopecia but not marrow


supression

 Actinomycin D is used singly or in combination for the treatent


of:

7
 Nephroblastoma

 Squamous cell carcinoma of the skin

 Malignant melanoma

 Non-Hodgkin’s lymphoma

 Choriocarcinoma

 Toxic effects of dactinomycin include:

 Atrophic dermatitis

 Alopecia

 Cardiomyopathy

 Gastro-enteritis

 Stomatitis

 Doxorubicin (adriamycin) may be used in the treatment of:

 Soft tissue sarcoma

 Osteosarcoma

 Breast cancer

 Childhood lymphoma

 Cancer of the cervix

 Bleomycin may be used to treat:

 Astrocytoma

 Cancer of thecervix

 Cancer of the penis and scrotum

 Oat cell carcinoma of the lung

8
 Transitional cell cancer of the bladder

 Vincristine:

 Is vinca alkaloid

 Interferes with DNA synthesis

 Is usually adminstered intravenously

 Is usually given daily

 Is less useful than vinblastine

 Vincristine is used in combination with other cytotoxics in the

treatment of:

 Leukaemia

 Lymphoma

 Nephroblastoma

 Rhabdomyosarcoma

 Osteosarcoma

 Toxic effects of vincristine or vinblastine include:

 Peripheral neuropathy

 Diarrhoea

9
 Convulsion

 Psychosis

 Hepatic failure

 The objectives of adjuvant chemotherapy include destruction or

control of:

 The primary tumour

 Micrometastases

 Surviving malignant cells at the primary site after treatment of the


primary

 Overt distant metastases

 Nodal metastases

 In current adjuvant chemotherapy:

 A combination of drugs is usually given in cycles

 The combined drugs should act in the same phase of the cell cycle
to ensure maximum “kill”

 The toxic effects of the drugs should not overlap

 Therapy is usually continued even if metastases appear as these


may be destroyed

 Mortality and local recurrence rate at 5 years are reduced in some


malignant disease

 Parameters to be checked before and after each cycle in adjuvant


chemotherapy include:

 Haemoglobin, white cell count and differential and platelets

10
 Alkaline phosphatase

 Serum calcium

 Serum creatinine

 Plasma proteins

 Drugs used in the cyclical combination adjuvant therapy in breast

cancer include:

 Daunorubicin

 Cyclophosphamide

 5-Fluoro-uracil

 Vinblastine

 L-phenylalanine mustard

 Drugs used in the cyclical combination adjuvant chemotherapy of


nephroblastoma include:

 Dactinomycin

 CCNU

 Doxorubicin

 Thiotepa

 Cisplatin

 Drugs used in the cyclical combination adjuvant chemotherapy of


osteosarcoma include:

 Methotrexate

 Mitomycin

 Doxorubicin

11
 Chlorambucil

 Streptoxocin

 Chemotherapeutic agents may be administered by the following


routes:

 Subcutaneously

 Sublingually

 Suppository

 Topically

 Orally

 Modalities of cancer treatment include:

 Surgery

 Radiation

 Immune modulation

 Temperature modulation

 Hydrotherapy

 In the calculation of body surface area for the dosing of drugs


you need the following:

 Sex of the patient

 Weight of the patient in kg

 Height of the patient in cm

12
 Body mass index of the patient

 Fluid volume of the patient

 In cell cycle kinetics, the S phase is the phase where:

 RNA synthesis takes place

 DNA synthesis takes place

 Protein synthesis takes place

 Anti metabolite anticancer drugs act

 The greater variation in length of time is

 In the pre-assessment of patients for the chemotherapy, the


following

are necessary:

 Clinical grade of the tumour

 Haemoglobin level

 Surface area

 Visual acuity

 Kidney function tests

 A Karnofsky scale of 50% implies:

 Requires considerable assistance

 Requires occasional assistance

 Normal activity with effort

 Unable to carry on normal activity

 Requires frequent medical care

13
 The following measures are taken to prevent tumour lysis
syndrome:

 Adequate fluid intake

 Allopurinol

 Lowering urinary pH

 Blood transfusion

 Reduction in tumour mas

 The following anti –cancer agents are anti-metabolites:

 6-Mercaptopurine

 5-Flurouracil

 Actinomycin D

 Cytosine Arabinoside

 Cyclophosphamide

 Side effects of chemotherapy include:

 Halitosis

 Loss of hair

 Vomiting

 Gall stone

 Thrombocytopenia

 Cytotoxic drugs kill cancer cells through the following


mechanism(s):

14
 Apoptosis

 Dehydration

 Neovascularization

 Oxidation

 Ion deprivation

CHAPTER 55 ANSWERS

 A,C,D

 A,D,E

 B,E

 B,C,D,E

 A,C

 A,C,D

 B,C,E

15
 A,B,D

 B,C,E

 A,C,D

 A

 A,D

 B,C

 B,D

 B,C,D,E

 A,C,E

 B,D,E

 A,B,D

 A,D,D

 B,E

 A,B,D

 A,E

 A,D,E

 A,B,C,D

 B,C

 A,C

 A,B,C,D

 A,C

 B,C

 A,C,E

 A,B,D

16
 B,C

 A,C

 AC

 A,D,E

 A,B,C,D

 B,C

 B,D

 A,B,C,E

 A,E

 A, B,E

 A,B,D

 BCE

 A

17
CHAPTER 56
ORGAN TRANSPLANTATION
 Rejection:

 Is hyperacute if it occurs immediately the organ is transplanted

 If hyperacute, is caused by preformed antibodies against the donor


material in the serum of the recipient

 May be caused by the generation, stimulated by the donor material,


of large numbers of polymorphs with the ability to destroy the
transplanted material

 From implantation to destruction takes 10-20 days to complete

 Of hyperacute type can occur even when there are only minor
antigenic differences between the donor and the recipient

 Privileged sites include the:

 Upper limb

 Bladder

 Cornea

 Skin

 Pancreas

 In matching recipients and donors, the most important typing is


the:

 HL-A

 DR Locus

 Sex and race

1
 ABO blood group

 PMN blood group

 In the same family, a donor should have the following identical


with the recipient:

 DR. Locus

 HL-A

 ABO blood group

 PMN blood group

 Rh factor

 In unrelated individuals the chance of success of a donor graft


depends on identical:

 Sex and race

 ABO blood group

 HLA

 DR. Locus

 PMN blood group

 The current methods of preventing transplanted organ rejection


include:

 Whole body irradiation

 Azathioprine plus prednisone

 Methotrexate

 Prednisone

2
 Cyclosporin A

 Cyclosporin A:

 Is a fungal peptide

 Is quite toxic to the bone marrow

 Has selective action against T-cells especially those responsible for


allograft rejection

 Is the most powerful immunosuppressive agent

 Requires additional steroids routinely

 The commonest cause of end-stage renal failure is:

 Congenital polycystic kidney

 Chronic pyelonephritis

 Malignant hypertensive kidney disease

 Acute glomerulo-nephritis

 Chronic glomerulo-nephritis

 Before a patient receives a kidney transplant:

 Chronic pyelonephritis, if present, is usually treated aggressively


with appropriate antibiotics

 Carious teeth are extracted

 Infection must be searched for and eradicated

 Barium meal is mandatory and a peptic ulcer, if found, is treated


vigorously with cimetidine

 If the kidneys are causing hypertension they should be removed

 Patients with cerebral damage as a result of the following should


be

3
excluded as organ donors:

               A.  Hypothermia
B. Cerebral haemorrhage
C. Head injury
D. Drug overdosage
E. Cerebral thrombosis
11. Criteria for establishing brain-death include:
               A. Absence of respiratory or spontaneous movement when the patient
is removed from the ventilator for one minute
B. Absence of response to deep pain
C. Absence of corneal reflex
D. Constricted fixed pupil
E. Absence of ocular deviation response when ice-cold water is run
into the
external auditory meatus

12. The donor kidney:


A. Is preferably removed from the donor while the circulation is still
intact
B. Should as soon as it is removed be cooled by perfusing cooling fluid
through the renal artery
C. Is stored in a plastic bag under sterile conditions at 10˚C
D. Can be stored in a plastic bag at 4˚C for 48h before use
E. May be stored for 72h if it connected to a machine providing
continuous
pulsatile flow of cold preservative solution through the renal artery
13. In renal transplantation, the:

A. New kidney is positioned in an iliac fossa


B. Renal artery is anastomosed to the aorta
C. Renal veins is anastomosed to the inferior vene cava
D. Ureter is re-implanted in the bladder
E. Kidney usually starts excreting urine on the operating table
14. Rejection of a transplanted kidney is characterized by:

A. Subnormal temperature
B. Deteriorating renal function
C. Pain and tenderness in the region of the transplanted kidney
D. Leucopaenia
E. Swelling of the transplanted kidney
15. In acute rejection of a transplanted kidney:

4
 The blood pressure falls
 The weight of the patient falls and there may be slight oedema of
the ankle
 Biopsy of the kidney shows massive infiltration of the parenchyma
by lymphocytes
 Steroid dosage is increased
 Further episodes are not likely to occur after the initial attack
16. The commonest cause of death after renal transplantation is:

A. Hypertension
B. Acute pancreatitis
C. Myocardial infarction
D. Secondary haemorrhage from the vascular anastomoses
E. Infection
17. The Complications of renal transplantation and its treatment
include:

 Peptic ulceration
 Tuberculosis
 Neoplasm of the reticulo-endothelial system
 Avascular necrosis of bone
 Diabetes mellitus

18. In the after-care of a patient with a transplanted organ:

 Follow-up must be regular


 Watch should be kept for infection, but minor infection can be
ignored
 Immunosuppression should be stopped when there is infection
 Cushing’s syndrome is an indication for stopping steroids
 Fungal infection may be difficult to eradicate
19. After renal transplantation, graft survival of:

A. Cadaver (non-related) kidney at 1 year is > 90%


B. Cadaver (non-related) kidney at 5 years is > 80%
C. Sibling kidney at 1 year is > 90%
D. Sibling kidney at 2 years is > 85%
E. Sibling kidney at 5 years is > 80%
20. In heart transplantation:

 An indication is cardiomyopathy which does not respond to medical


treatment

5
 It is not absolutely essential that the donor heart is re moved while
still beating
 The donor heart must be plunged in ice-cold preservative fluid as
soon as it is removed and re-implantation may wait for 12h
 An initial feature of rejection is an increase in the E.C.G. voltage
 The average survival at 1 year is about 90% and at 5 years about
70%
21. Indications for liver transplantation include:

 Liver metastases
 Primary liver cell cancer without metastases
 Cirrhosis of the liver without portal hypertension
 Viral hepatitis with liver failure
 Biliary atresia
22. Complications of liver transplantation include:

 Stricture of the common bile duct


 Acute cholecystitis
 Biliary sludge
 Biliary calculi
 Liver cell carcinoma
23. In liver transplantation:

 Speed of removal and cooling of the donor is very essential as the


liver is very sensitive to warm ischaemia
 The liver does not suffer rejection to the same extent as the kidney
 The first successful transplant was done in 1976
 The 1 year survival is about 30%
 The 1-year survival is about 90%

24. Corneal transplant:


A. Is used extensively throughout the world
B. Is indicated for corneal opacification
C. Is stored at household refrigerator temperature
D. Can be stored for 72h before use
E. Requires immunosuppression
25. Currently, the most effective immunosuppressant in clinical use
is:

A. Cyclosporin A
B. Radiation

6
C. Prednisone
D. Azathioprine + corticosteroids
E. Monoclonal antibodies to T-cell
26. Tacrolimus:

 Is 5 times as potent as cyclosporin A


 Inhibits calcineurin thereby impairing expression of several critical
T-cell activation genes
 Is routinely used without steroids
 Is useful in rescue regimes for acute rejection
 May cause renal dysfunction, diabetes and pancreatitis
27. In heart-lung transplantation:

 Indications include cystic fibrosis of the lungs and bronchiectasis


 The donor lungs need not be on ventilation during harvesting
 The phrenic, vagus and recurrent laryngeal nerves of the recipient
must be preserved
 Complications include sqamous metaplasia of the bronchioles and
myo-intimal hyperplasia of the cardiac vessels
 Survival rates at 1 year and 5 years are 65% and 40%
respectively
28. The small bowel is the most hazardous abdominal organ to
transplant
primarily because of:

 The severe post-operative sepsis


 The development of lymphoproliferative disorders
 Rejection of the graft even after a year
 The abundance of lymphocytes in it
 Its permeability
29. Concerning pancreatic transplantation:

 It is needed equally in patients with Type I and Type II diabetes


mellitus
 Nephropathy with renal dysfunction or need for dialysis is an
important indication
 The commonest type is simultaneous pancreas-kidney procedure
 Urinary drainage via the duodenum is better than the enteral
drainage
 Advanced retinopathy does not improve but may stabilize

7
30. Conditions that may lead to small bowel failure include:

A. Nercotising enterocolitis
B. Mesenteric infarction
C. Crohis disease
D. Exomphalos
E. Volvulus neonatorum
31. Complications of total parenteral nutritional (TPN) include:

A. Liver fibrosis
B. Recurrent severe dehydration
C. Recurrent shock
D. Sepsis (SIRS)
E. Multiple organ dysfuncion

8
CHAPTER 56
ORGAN TRANSPLANTATION
ANSWERS

 A, B 26. B, D, E
 C 27. C, D, E
 D 28. D
 B, C 29. B, C, E
 B, C, D, E 30. A, B, C, E
 B, D, E 31. A, B, D, E
 A, C, D
 E
 B, C, E
 A, D
 B, C
 A, B
 A, D, E
 B, C
 C, D
 E
 A, B, C, D, E
 A, B
 A, B, C, D, E
 A, E
 B, E
 A, C, D
 A, B, E
 A, B, C
 E

9
CHAPTER 57

ANAESTHESIA

 Drugs used in the pre-anaesthetic period for their effect on the central
nervous system include:

 Pethidine

 Hyoscine

 Phenoxybenzamine

 Diazepam

 Propranolol



 Papaveratum (Omnopon):

 Inhibits vomiting

 Has no effect on bowel activity

 Depresses respiration

 May cause a slight rise in blood pressure

 Causes sedation



 Pethidine:

 Is a synthetic drug and 50mg is equivalent to 10mg of morphine

 Causes spasm of smooth muscle

 Causes less addiction than morphia

 Has less sedative effect than morphia

 Depresses respiration less than morphia



 Pentazocine (Sosegon):

1
 Is derived from nalorphine, a morphine agonist

 Has a powerful analgesic action of short duration

 Does not depress respiration

 Does not stimulate nausea or vomiting

 Tends to depress the blood pressure



 Tranquillizers:

 Depress the central nervous system

 Reduce excitement

 Induce sleep

 Produce calmness

 Produce analgesia





 Phenothiazines:

 Have anti-histaminic and anti-emetic effects

 Lower the body temperature

 Have beta-adrenergic blocking effect

 Suppress bronchial secretions and relax bronchial muscles

 Have analgesic action



 Neuroleptics include:

 Methadone(physeptone)

 Promethazine

2
 Haloperidol(serenase)

 Chlorpromazine

 Nitrazepam(mogadon)



 Butyrophones:

 Are good anti-emetics in clinical use

 Have a beta-blocking action

 Have a powerful central sedating action without producing sleeping

 Reduce psychomotor agitation

 Examples are droperidol and chordiazepoxide(Librium)



 Diazepam:

 Is psycho-sedative and allays apprehension particularly under stress

 Has no effect on respiration or the cardiovascular system

 Has a muscle relaxant effect

 Has an anti-histaminic effect

 Is a phenothiazine



 Hyoscine:

 Like atropine stimulates the CNS before depressing it

 Depresses secretion of sweat, tears and saliva

 Produces tachycardia and bronchodilation

 Stimulates emptying of the bladder

 Causes constriction of the pupils

3


 Drugs used in the management of cardiac arrhythmias include:

 Phenoxybenzamine

 Propranolol

 Alprenolol

 Atropine

 Lorazepam



 Drugs to avoid in asthmatics include:

 Propranolol

 Pethidine

 Oxprenolol

 Morphine

 Chlorpromazine



 The effect(s) of suxamethonium on a digitalized patient with a heart rate


of 60 or less may be:

 Tachycardia

 Severe bradycardia

 Cardiac arrest

 Arrhythmia

 Infarction



 In a patient on a diuretic such as frusemide, neostigmine may:

4
 Stimulate respiration because of acidosis

 Depress respiration because of alkalosis

 Be ineffective in reversing neuromuscular blockade because of


Hypokalaemia and result in respiratory paralysis

 Be ineffective in reversing neuromuscular blockade because of


Hyponatraemia and result in respiratory paralysis

 Reverse neuro-muscular blockade and stimulate respiration because of


hypocalcaemia



 In a patient on mono-amine oxidase inhibitor(MAOI):

 It takes about 7 days for mono-amine concentration to become normal


after cessation of MAOI.

 A normal dose of pethidine may result in coma

 Phenobarbitone in normal doses is not effective

 Ephedrine may cause severe hypotension

 MAOI should be stopped at least 3 weeks before anaesthesia



 Which of the following antibiotics causes(s) a non-depolarizing block


potentiated by non-depolarizing muscle relaxants when given intra-perit
oneally or sprinkled as a powder into a raw area?

 Penicillin

 Streptomycin

 Kanamycin

 Chloramphenicol

 Neomycin



 In chronic alcoholics :

5
 Large doses of anaesthetic agents are required

 Death may result under anaesthesia because of liver failure

 Death may result under anaesthesia because of cardiac arrest

 Death may result under anaesthesia because of hypoxia



 A patient is off steroid therapy. During operation steroid cover is not


needed if he has been off steroids for:

 Less than one month

 Less than two months

 More than two months

 More than three months

 More than six months



 The uptake and rate of uptake of inhalational agents by tissues depend


on:

 The partial pressure of the agent in the alveolar air, blood and tissues

 Rate of diffusion of the agent across the alveolar membrane

 The rate of respiration

 Solubility of the agent in the blood and tissue

 The cardiac output



 Concerning the solubility of anaesthetic agents in blood:

 The more soluble the agent, the quicker its cerebral uptake

 The more insoluble the agent the quicker the induction

 Diethyle ether and methoxyfluorane are very soluble in blood

6
 Nitrous oxide and cyclopropane are relatively insoluble in blood



 Induction occurs quickly in the:

 Thyrotoxic patient

 Aged

 Shocked patient

 Frightened patient

 Dehydrated patient



 The following occur in the 2nd stage of ether anaesthesia:

 Struggling

 Breath-holding

 Poor muscle tone in all muscles except the diaphragm

 Dilated pupils

 Vomiting



 In the 3rd stage of ether anaesthesia:

 There is regular, automatic breathing

 The eyeball is central

 The pupils may not react to light

 There may be apnoea

 There may be coughing



 The following agent(s) does not sensitize the myocardium to infiltrated


adrenaline and does not precipitate ventricular fibrillation and cardiac

7
arrest:

 Enflurane

 Ether

 Halothane

 Nitrous oxide

 Isoflurane



 Liver dysfunction has been proven to occur after:

 Nitrous oxide

 Choloroform

 Cyclopropane

 Ether

 Ethyl chloride



 Nitrous oxide:

 Is the oldest anaesthetic agent

 Is sweet smelling and irritant

 Is a good analgesic but weak anaesthetic

 Gives rapid induction and recovery because it is very soluble in blood

 Has no muscle relaxant properties



 Etomidate

 Is an ethyl imidazole carboxylate derivative

 Causes loss of consciousness in 90 seconds

8
 Soluble in intralipid, ethyl glycol and ethanol

 Dysrrhythmias may occur in children during induction

 Causes little nausea and vomiting





 Midazolam:

 Is a water soluble benzodiazepine

 It has a shorter onset of action than diazepam with a shorter duration

 It is a recommended anticonvulsant

 It is used for sedation in IV doses of 2.5-7.5mg

 The dose for induction is 0.07mg/kg



 Inflammable and explosive agents include:

 Halohane

 Diethyl ether

 Cyclopropane

 Nitrous oxide

 Methoxyfluorane



 Halothane:

 Is a good analgesic and a potent depressant of the respiratory and


vasomotor centres

 May cause hypotension and ventricular extrasystoles

 Gives good muscle relaxation

 Gives rapid and smooth induction and delayed recovery with short

9
anaesthesia

 Increases salivary and bronchial secretions



 Non-depolarising muscle relaxants:

 Pancuronium is an amino-steroid with some hormonal activity

 Mivacurium is metabolized by liver esterases

 Cis-atracurium releases more histamine than atracurium

 Rocuronium is extensively metabolized by the liver to form glucoronides

 Vecuronium is a pancuronium derivative



 Factors that may affect the action of muscle relaxants include:

 Metronidazole

 Carbon dioxide tension

 Sex

 Serum clotting factors

 Age



 Adrenaline infiltration of tissues may precipitate dangerous arrhythmias


if the anaesthetic agent is:

 Desflurane

 Nitrous oxide

 Enflurane

 Halothane

 Diethylether



10
 Anaesthetic agents that cause a fall in the blood pressure include:

 Nitrous oxide

 Halothane

 Ether

 Sevoflurane

 Isoflurane



 Agents that raise the blood pressure include:

 Thiopentone

 Propofol

 Etomidate

 Ketamine

 Methohexitone



 Thiopentone:

 Is a thiobarbitone belonging to the short acting group of barbiturates

 2.5% solution has a pH of 7.5

 Induction dose for a fit adult is 3.5-6.0 mg/kg

 Produces sleep within 15 seconds of injection

 Crosses the placental barrier and there is maximum concentration in the


foetus within one minutes of injection



 Thiopentone:

 Total dose at any one time should not exceed 500mg to prevent liver
dysfunction

11
 Patients can handle machinery or drive 6h after recovery from it.

 Is not potentiated by alcohol

 As a rule is safe to use in patients with a systolic pressure of less than


100mmHg

 In small or large doses increases sensitivity to somatic pain.



 Thiopentone:

 Unlike other anaesthetic agents depresses the sensitivity of the respiratory


centre to carbon dioxide

 Is a good relaxant to muscles but shivering is common in the immediate


post-operative period

 May cause laryngospasm or respiratory arrest

 May be used even in the presence of trismus or severe oro-pharyngeal


infection

 Is a satisfactory and safe anaesthetic as the sole agent for short procedures
such as diagnostic D & C



 Intra-arterial injection of thiopentone:

 Causes severe spasm of the injured vessel with disappearance of the radial
pulse

 Causes a dull ache of the forearm

 May lead to gangrene of the arm

 Is treated by leaving the needle in situ and injecting papeverine,


hyaluronidase and procaine or tolazoline

 May be further managed by a brachial plexus or stellate ganglion block



 Neostigmine:

12
 Is available as the bromide or the methylsulphate

 It acts by stimulating the neuromuscular junction directly

 During the administration of neostigmine hypoxia may cause a cardiac


arrest

 The most important side-effect of neostigmine use is salivation

 It may also cause miosis



 Ketamine:

 May be given intravenously or intramuscularly

 Intravenously induces anaesthesia in a minute and intramuscularly in 3-


5minutes; a single dose lasts 30 minutes

 Acts by “dissociative anaesthesia” which is characterized by analgesia


combined with deep sleep

 Maintains pharyngeal and laryngeal muscle tone and reflexes and so the
airway

 Causes a fall in blood pressure accompanied by tachycardia



 Ketamine:

 In the absence of a sedative pre-medication may cause delirium

 Commonly causes hallucinations, dreams and visual disturbances which


may last for days

 Does not affect respiration

 Can be used to induce poor risk patients and for short procedures

 May be used in the absence of resuscitating equipment.



 The most suitable anaesthetic agent to use for eclampsia or status


epilepticus is:

13
 Thiopentone

 Propofol

 Eomidate

 Methohexitone

 Ketamine



 Muscle relaxants:

 Were introduced into clinical practice in 1942

 Produce paresis or paralysis of all striated muscles

 Produce relaxation which is not as great as with deep anaesthesia

 Destroy acetylcholine released at the motor end plate

 Make it possible to use just sufficient anaesthesia to produce


unconsciousness



 Concerning neuromuscular transmission:

 The motor end-plate is formed by fine myelinated terminal nerve branch


lying in a gutter of a specialized part of the muscle fibre

 The gutter of the motor end-plate is extensively lined by anti-


cholinesterase

 Acetyl choline is formed from acetic acid and choline in the presence of
coenzyme A and stored in vesicles at the nerve terminals.

 On arrival of motor nerve impulse acetyl choline is released

 The released acetyl choline is hydrolysed by the anit-cholinesterase



 Non-depolarizers include:

 Vecuronium

14
 Suxamethonium

 Cisatracurium

 Pancuronium

 Decamethonium



 Desensitizing block:

 Occurs when there is an alteration in the threshold of the motor end-plate

 Is found in conditions producing hyperkalaemia in the cells

 Occurs when a depolarizing drug is used after a non-depolarizer

 Occurs when a depolarizer is used in the presence of streptomycin

 Is reversed by the administration of calcium



 Suxamethonium:

 Causes muscle paralysis of more rapid onset and shorter duration than
suxemethonium

 Is slowly hydrolysed by serum cholinestarases and can be identified in the


blood after 14 minutes

 Causes tachycardia and salivation which can be prevented with atropine

 May cause cardiac arrest in burnt or severely traumatized patients

 Crosses the placental barrier and so is unsafe to use in Caesarean section



 Tubocurarine:

 Causes muscular paralysis which appears within 3 minutes and last 40


minutes

 May in the dehydrated cause re-curarization during recovery from


anaesthesia after reversal

15
 Crosses the placental barrier but not in sigficant amounts to have any
action on the foetus

 Is unsafe in severe asthmatics as it may cause bronchospasm following


release of histamine

 Causes transient hypotension



 Secretion drying doses of anti-muscarinic agents:

 Atropine 0.1mg in children

 Atropine 0.3mg in adults

 Glycopyrrolate 0.2mg in children

 Glycopyrrolate 0.8mg in adults

 Hyoscine 0.4mg in adults



 Pancuronium bromide (Palvulon):

 Is a steroid depolarizing muscle relaxant

 Is more potent than rocuronium

 Is all excreted unchanged

 Does not cause hypotension because its ganglion blocking effect is very
weak

 Crosses the placental barrier



 Neostigmine:

 Is a synthetic anticholinesterase

 Forms a complex with cholinesterase thereby preventing destructionof


acetylocholine

 Inhibits the neuromuscular junction

16
 Causes tachycardia, salivation and stimulation of the bladder and gut

 Should be preceded by atropine injection



 Gases which are all gaseous in the cylinder at room temperature


include:

 Nitrous oxide

 Air

 Oxygen

 Carbon dioxide

 Ethylene oxide



 The high pressure of gases in cylinders is reduced in the new all-steel


anaesthetic machines to:

 K Pa-8.98

 K. Pa-98

 K Pa-5.98

 K Pa-2.33

 K Pa-1.33



 The best aid against fitting a cylinder to the wrong outlet of the
anaesthetic machine is:

 The specific colour of the cylinder

 The embossed name of the gas

 The embossed chemical name of the gas

 The pin-index system

17


 An anesthetic cylinder with:

 Black body and white colour contains Carbon dioxide

 Blue colour all over contains nitrous oxide

 Grey colour all over contains oxygen

 Blue colour all over with white shoulder contains entonox.



 The reservoir bag of the anaesthetic machine:

 Collects gases during inspiration

 Converts the continuous flow of gases from the machine to an intermittent


flow to the patient

 With large bore tube and expiratory valve forms Macintosh circuit

 Is usually used for spontaneous or controlled breathing



 To avoid hypoxia, the proportion of oxygen during anaesthesia should be


at least:

 21%

 25%

 30%

 35%

 40%



 The Arye’s T-piece system:

 Has no valves

 Is used for all ages

18
 To prevent retention of carbon dioxide fresh gas flow of at least twice the
patient’s minute volume is required.

 Has a low resistance in the system

 Can be used for spontaneous or controlled breathing without detriment to


the patient.



 For a fit adult breathing spontaneously the gas flow per minute should
be at least:

 7 litres containing 3 litres of oxygen and 4 litres of nitrous oxide

 8 litres containing 3 litres of oxygen and 5 litres of nitrous oxide

 8 litres containing 4 litres of oxygen and 4 litres of nitrous oxide

 9 litres containing 4 litres of oxygen and 5 litres of nitrous oxide

 9 litres containing 3 litres of oxygen and 6 litres of nitrous oxide.



 For 24h after a general anaesthesia in the out-patient, a patient should


refrain from:

 Driving a motor vehicle

 Cooking

 Eating solid food

 Taking alcoholic drinks

 Taking an analgesic



 A patient who has had general anaesthesia in the outpatient should not
be discharged if:

 Surgery was prolonged

 There was excessive peri-operative bleeding

19
 He is an asthmatic

 There was prolonged drowsiness post-operatively

 He has no escort



 Local analgesic agents produce loss of sensation of:

 Pain

 Touch

 Temperature

 Propioception

 Pressure



 The first nerve fibres to be blocked in local analgesia are:

 Alpha fibres

 Beta fibres

 Delta fibres

 B fibres

 C fibres



 Adrenaline injected together with a local analgesic:

 Reduces the rate of absorption of the analgesic

 Causes prolongation of action of the analgesic

 May increase the toxicity of the analgesic

 Prevents arrhythmia

 May precipitate fainting

20


 Adrenaline added to local analgesic:

 Should not exceed 0.5mg in one sitting

 Is used in concentration of 1:1,000

 Is contraindicated in sickle cell disease

 May be used in the finger or penis

 May be used in spinal analgesia

 Acute cardiovascular failure following local analgesia may be due to:

 Vasoconstriction

 Myocardial depression

 Stimulation of the vasomotor centre

 Interference with the conduction system of the heart

 Disturbance of the parasympathetic innervation of the heart.



 Treatment of cardiovascular reactions following local analgesic may


require

 Lowering the legs of the patient

 Intravenous fluids

 Injection of atropine

 Injection of morphine

 Administration of oxygen



 Cerebral features of toxicity of local analgesics include:

 Convulsions, restlessness

21
 Crying and hyperventilation

 Slurred speech

 Coma

 Hemiplegia



 Treatment of toxic cerebral effects of local analgesics may require:

 Thiopentone

 Diazepam

 Oxygen

 Artificial ventilation

 Cortisol



 Methaemoglobinaemia:

 Follows the use of 600mg of prilocaine in an adult

 Does not occur with lignocaine

 Presents as cyanosis and anaemia

 Is treated with oral methylene blue

 Is readily detected by pulse eximetry



 Cocaine

 Does not decompose on boiling

 Is used as a surface analgesic for corneal analgesia and as a spray for the
nose, throat and larynx

 Is used as local analgesic

22
 Has a maximum dose of 100mg in an adult

 Is a potent vasodilator



 Procaine:

 Is used as a standard reference drug for measuring the potency and


toxicity of other local analgesic agents

 Is decomposed by boiling

 In 1-2% solution acts within 2-5 minutes of infiltration and is effective for
1h

 Is more toxic than lignocaine and toxic symptoms include convulsions and
dyspnoea

 Has a wide margin of safety



 Mepivacaine

 Is more potent but less toxic than procaine

 Withstands repeaed boiling

 Is used in spinal analgesia where its action lasts 6hrs

 Is also used for peripheral nerve block and local infiltration

 Is hydrolysed by plasma cholinesterase



 Lignocaine:

 Is stable to boiling and autoclaving

 Is used for local infiltration, peripheral nerve block and epidural block

 Duration of action is ½ hr without and 2 h with adrenaline

 Has a good surface analgesic effect and is used as an ointments, cream,


jelly or spray

23
 Recommended concentration for local infiltration is 2-4% and for
peripheral nerve block 1-2% and the maximum safe dose without
adrenaline is 400mg





 Bupivacaine:

 Is more potent and more toxic than mepivacaine

 With epidural, caudal or nerve block is effective for 3-4h or more

 Is the agent of choice for sympathetic block

 Undergoes slow elimination from the systemic circulation

 For epidural block the dose of the 0.5% solution is 10-20ml



 Which of the following should be available before a local analgesic agent


is given?

 Cardiopulmonary resuscitation equipment and drugs

 Means of ventilating the lungs

 Thiopentone, diazepam and suxamethonium

 Oxgyen and machine for delivering it

 Nitrous oxide



 In a brachial plexus block:

 The supraclavicular approach is through a point 1cm above the


midclavicle, the needle being directed downwards inwards and dorsally
until it strikes the first rib

 The axillary approach is through a point 1 cm below and 3cm lateral to the
mid clavicle, the needle being directed posteriorly for 2cm

 Both approaches may cause haemothorax and troublesome bleeding

24
 The supraclavicular and not the axillary approach may cause paralysis of
the phrenic nerve

 The supraclavicular approach is suitable for procedures on the whole


upper arm and the axillary approach only for procedures on the forearm
and hand



 In intravenous regional analgesia:

 The compressing blood pressure cuff is inflated to a pressure between the


diastolic and systolic pressures before the administration of the analgesic
agent

 The calculated dose of local analgesic agent is given rapidly intravenously


as a bolus

 The onset of action is 10-15 minutes and is accompanied by tingling or


warm sensation in the limb

 The compressing blood pressure cuff should be left on for at least 30


minutes and can be left for 1½ h

 At the end of operation the blood pressure cuff is removed all at once.



 In intravenous regional analgesia, if the compressing blood pressure cuff


is released within 20 minutes of the administration of the analgesic agent
the following may occur:

 Drowsiness and coma

 Tachycardia

 Excitement

 Convulsion

 Giddiness



 In Spinal analgesia:

25
 The lumbar puncture can be performed between L1 and S2

 A line connecting the highest points of the iliac crests passes through L3
or inter-space L3-L4

 The spread of the block depends on the site of injection and volume of
agent injected

 The spread of the block is not affected by the rate of injection of the agent

 Hyperbaric solution when injected with the patient in upright position


spreads more cephalad than caudually.



 In spinal analgesia:

 Testing analgesia by pinching the skin with artery forceps causes less
discomfort to the skin

 Hypotension is always caused by vasodilation

 Hypotension is best treated with a vasopressor

 Atropine prevents bradycardia from possible blockade of sympathetic


fibres to the heart

 The minimum safe systolic artierial pressure is 60-70mmHg in a healthy


adult of 35 years



 In mid-spinal block the:

 Table is brought to a 5-10 degrees head down position

 Head-down position is maintained for 5-10minutes

 Patient then lies supine or on the same lateral side of the planned
procedure

 Height of analgesia may reach T10-T6



 Headache after spinal analgesia:

26
 Characteristically sets in within 24-72h after the spinal

 Gets worse in the sitting or upright position and diminishes in the lying
position

 Is usually located in the occipital region and radiates to the neck

 May be prevented by nursing the patient in the first 12h on a bed with the
foot end slightly raised and advising the patient not to read or smoke
during this period

 Subsides after a few days



 In headache due to low subdural pressure:

 Resting in the patient on a bed with the foot end raised relieves it

 Intravenous infusion of 200ml of glucose 50% in normal saline is


beneficial

 Use of a tightly applied abdominal binder is beneficial

 Antibiotics may be administered



 Neurological complications of spinal analgesia include:

 Paralysis of the olfactory nerve

 Paralysis of the abducens nerve

 Cauda equina syndrome

 Ascending myelitis

 Meningo-encephalitis



 Advantages of spinal analgesia include:

 Simplicity and low cost

 Reduction of blood loss during operation

27
 Reduced incidence of post-operative bleeding

 Reduced post-operative observation

 Provision of good muscle relaxation and profound analgesia



 The advantages of epidural block include:

 Headache is rare

 The technique is simple to learn

 Neurological complications are less frequent than after spinal analgesia

 Muscle relaxation is as good as in spinal analgesia

 Hypotension does not occur



 The concentrations of lignocaine for the different types of analgesia are:

 0.25-0.5% for infiltration and intravenous analgesia

 0.5-1% for epidural and caudal block

 1-2% for peripheral nerve block

 3-4% and 10% for surface analgesic

 5% for spinal analgesia



 The recommended concentration and dosage of bupivacine for epidural


block are:

 0.25-0.5% : 12-20ml (30-100mg)

 0.25-0.5% : 25ml (45-120mg)

 1% : 20ml (200mg)

 1% : 15ml (150mg)

28
 2% : 10ml (200mg)



 Beta-adrenergic blocking agents:

 The reference agent is atenolol

 Propranolol has a local anaesthesia effect

 Esmolol is metabolished by plasma esterases

 Because of its long duration of action esmolol is used for the resection of
pheochromocytoma

 Oxyprenolol is the drug of choice for attenuating the sympatho-adrenal


response to laryngoscopy and intubation.



 Propofol:

 Is an aqueous emulsion

 Is equipotent with methohexitone

 It has a slower distribution than thiopentone

 It is both pro and anti-convulsant

 Anphylactic reactions frequently occur



 The Magills circuit

 Consists of a corrugated tubing which has a capacity of about 800mls

 Has a reservoir bag of about 3 litres

 The minimum fresh gas flow is for an adult is 8 litres

 It is used mainly in adults for spontaneous ventilation

 The expiratory valve which is near the anaesthetic end of the circuit must
be fully open during use.

29


 For laryngoscopy

 The patient may be put in the lateral position

 The neck is extended and the head flexed

 Overextension may obstruct the view

 The landmark for the intubation is the epiglottis which is posterior to the
larynx

 It is easy to use the lower incisors as a fulcrum to make intubation easier



 Intravenous anaesthetics used in day care anaesthesia include

 Methohexitone

 Thiopentone

 Ketamine

 Etomidate

 Propofol



 In conscious sedation

 The patient is in an arousable state and is painfree

 Loss of consciousness must be brief

 Propofol and morphine combinations improve postoperative pain

 Sulfentanyl is too potent to be used

 Ketamine is contraindicated because of its pharmacokinetics



 Caudal epidural block

 Is performed through the sacral hiatus formed by failure of the 3-5

30
vertebrae to fuse

 The hiatus may not be present in about 40% of individuals

 Is usually used for operations on the perineum

 Identification of the space is by a sudden give as the needle passes through


the sacral ligament

 It is not advisable to use local anaesthetics agents with adrenaline.

CHAPTER 57

 A, D

31
 C, E

 D

 B

 B,D

 A, B, D

 B, C, D

 A, C, D

 A. C

 B, C

 B, C

 A, C,D

 B, D

 C

 A, B

 B, C, E

 A, C

 C, D, E

 A, B, D, E

 B, C, D

 B. C. E

 A, B, E

 A, B, C

 A, B, D, E

 B, C

32
 A, C, E

 A, C, D

 A, D

 B, C

 B

 E

 B, E

 D

 B, D, E

 D

 C, D

 A, E

 C

 A, C

 C

 A

 A, B, C, D

 A

 A, E

 C, D

 A, C, D

 A, C

 A, D

 B, C, D

33
 E

 B, D

 A, B, E

 B, C

 B

 D

 B, D

 B, D

 C

 A, C, D, E

 B,

 A, B, D

 A, B, D, E

 A, C

 E

 A, B, E

 A

 B, D

 B, C, E

 A, B, C, D

 A, B, C, D

 A, C

 B, D

 A, C, E

34
 A, B, D

 A, B, C, D

 A, B, C, E

 A, B, C, D

 A, D

 C, D

 A, C, D

 C

 D

 B, C, D

 A, B, D

 A, C

 B, C, D, E

 A, B, D,E

 A, C

 A, C, D, E

 A

 B, C

 A, B, C, D, E

 D

 C

 A, B, C, E

 A

 C

35
36
CHAPTER 58

PRINCIPLES OF RADIOTHERAPY

 Ionising radiation:

 Expels an electron from an atom

 Expels a proton from an atom

 Forces an additional electron into an atom

 May raise the energy level of an electron within the atom making
the atom more reactive – excitation

 Causes absorption of energy by cells

 Concerning the types of radiation:

 X-rays are emitted spontaneously during the decay of a radioactive


material

 Gamma rays are artificially produced by a suitable electrical


generator

 X-rays and gamma rays are electromagnetic radiations

 Neutrons, which are negatively charged, are emitted from some


radioisotopes such as Californium 252

 Beta-particles are particulate radiations emitted during radioactive


decay

 LET (linear energy transfer):

 Is the energy transferred to the absorbing material along the tract of

1
the radiation

 Is high with alpha-particles

 Is low with X-rays

 Is high with gamma rays

 If high implies that the radiation dissipates its energy within a very
short distance, no more than the thickness of a tissue paper

 Effects of radiation on mammalian cells include:

 Delay in cellular division


 Loss of normal cellular functions
 Loss of ability to reproduce indefinitely
 Damage to chromosomes seen at first division after radiation
 Decrease in the number of chromosomes
 The interaction of the ions formed along the radiation tract in the
water surrounding the cells produces:
 H
 O
 OH
 HO2
 H2O2

 Radiation damages cells through its direct action on:


 RNA

 DNA

 Genes

 Cytoplasm

 Cell membrane

 Radio sensitivity of a tumour:

 Is high if it is rapidly growing

2
 Is high if the partial pressure at the time of irradiation is low

 May be increased by the administration of cysteine.

 Is increased by the effect of previous irradiation

 Such as breast cancer is depressed by oestrogens

 Radio-curability of a tumour depends on:

 Its radio sensitivity

 Its size, the bigger one being more difficult to eradicate

 Its site

 Its clinical stage

 The age of the patient

 Which of the following will cause the least damage to normal


tissue and give the best cosmetic result?

 1000cGy weekly

 500cGy twice weekly

 333cGy three times weekly

 200cGy daily except Saturday and Sunday

 Concerning the clinical effects of radiation:

 The smaller the volume or surface irradiated, the smaller the


tolerated dose

 A smaller number of fractions allows for a better tissue repair

 High energy radiations produce maximum damaging effects below


the skin and spare the skin.

 The biological effect does not generally decrease as the dose rate
decreases

3
 100cGy to the whole body will cause aplastic anaemia

 Radiation to the eye results in:

 Early conjunctivitis which frequently recovers after cessation of


radiotherapy.

 Corneal ulceration infrequently as the cornea being avascular is


radio-resistant

 Cataract if the dose is >100cGy

 Glaucoma very commonly

 Irdocyclitis and xerophthalmia commonly

 The effect of radiation on the skin is severe if the:

 Dose per fraction is large

 Surface area is large

 Overall Time is prolonged

 Patient is young

 Skin is on the trunk

 The late effects of moderate to severe therapeutic doses of


radiation on the skin include:

4
 Pigmentation

 Thickening

 Dryness

 Scattered areas of dilatation of dermal capillaries

 Epilation

 The early effects of radiation on the skin include:

 Transient erythema

 Oedema

 Depigmentation

 Dry desquamation

 Excoriation/moist desquamation

 The late effects of irradiation of the buccal mucosa and salivary


glands includes:

 Enlarged salivary glands with copious thick secretion

 Loss of taste

 Dysphagia

 Thickening of the mucosa

 Dental atrophy

 The early bladder reaction to radiation include:

5
 Frequency of micturition

 Fibrosis and shrinkage

 Haematuria

 Dysuria

 Retention of urine

 Irradiation has the following late effects on the intestines:

 Strictures

 Granulomata formation

 Malabsorption

 Ulceration of the muscosa

 Perforation

 To avoid renal failure both kidneys are not irradiated beyond a

dose of (cGy):

 500

 1000

 1800

 3000

 4000

 Features of radiation sickness include:

 Nausea with or without vomiting

 Headache

6
 Anorexia

 Diarrhoea

 Malaise

 The severity of radiation sickness:

 Does not depend on the dose of radiation

 Does not depend on the volume of tissue irradiated

 Depends on the anatomical site of tissue irradiated

 Occurs early when the chest in irradiated

 Is not related in any way to the psychological factors

 Whole body radiation exposure to 400-600 cGy will result in:

 Nausea and vomiting

 weight loss

 Epistaxis

 Ulceration of mucosal surfaces

 Death in all the victims

7
 All living things are exposed to background radiation from:

 Granites

 Sea

 Soil

 Sun

 Air

 Cancer of the skin is common in:

 Indigenous people of tropical Africa

 Australian whites

 Albinos

 Caucasians in Northern Europe

 Indigenous people of South Africa

 Precancerous effects of cosmic rays on the delicate and sensitive


skin include:

 Depigmentation

 Hyperkeratosis

 Cracks

 Fissures

 Ulcers

 Previous Radiation exposure is associated with:

 Skin cancer

8
 Dermatitis

 Leukaemia

 Osteosarcoma

 Malformations

 Pre-operative radiotherapy:

 May make an inoperable tumour operable

 Prevents dissemination of tumour cells during operation

 Makes an incurable tumour curable

 Does not delay wound healing

 Should be followed by surgery within 3-5 weeks

 The following skin tumours are radiosensitive:

A. Malignant melanoma

B. Squamous cell carcinoma

C. Kaposi’s sarcoma

D. Basal cell carcinoma

E. Mycosis Fungiodes

 For most head and neck cancers:

 Radiotherapy is equivalent to surgery in early lesions

 Moderately advanced tumours do better with pre-operative


radiotherapy and surgery

 Lymph node metastases respond better to radiotherapy than to

9
surgery

 Those involving the salivary glands are treated by surgery or


radiotherapy

 Advanced tumours are treated by pre-operative radiotherapy and


surgery

 In malignant tumours of the thyroid, radioiodine is used for:

 Papillary carcinoma

 Medullary carcinoma

 Anaplastic carcinoma

 Follicular carcinoma

 Lymphoma

 In carcinoma of the breast, mono therapy with radiotherapy is


indicated in:

 Local recurrence

 Inoperable tumour

 Bone metastases

 Stage 1 tumour after extended simple mastectomy

 Stage 1 tumour after partial mastectomy

 In the gastro-intestinal tract, radiotherapy is useful as adjuvant


therapy:

 Gastric carcinoma

 Gastric lymphoma

 Carcinoma of the colon

 Adeno carcinoma of the small intestine

10
 Squamous carcinoma of the anus

 In the male genitor-urinary system, radiotherapy may be indicated


for:

 Advanced carcinoma of the prostate

 Epithelioma of the penis

 Adeno-carcinoma of the kidney

 Transitional cell carcinoma of the bladder

 Carcinoma of the urethra

 In the female genital tract, radiotherapy is the primary treatment


of:

 Carcinoma of the overy

 Adeno-carcinoma of the uterus

 Carcinoma of the cervix

 Carcinoma of the vagina

 Carcinoma of the vulva

11
CHAPTER 58
PRINCIPLES OF RADIOTHERAPY
ANSWERS

 A, D

 C, D

 A, B, C

 A, B, C, D, E

 A, B, C, E

 B

 A, B

 A, B, C, D

 D

 C,

12
 A, B, C, D, E

 A, B

 B, C, D

 A, D, E

 B, D, E

 A, C, D

 A, C, D, E

 C

 A, B, C, D, E

 C, D, E

 A

 C, D

 B, C, D

 B, C, D

 A, B, C, D, E

 A, C, E

 C

 A, B

 A, D

 A, B, C, E

 A, B, E

 A, D,E

 C, D,E

13
14
CHAPTER 60

AN OUTLINE OF THE MANAGEMENT OF SOME


CONGENITAL ANOMALIES

 Congenital anomalies that may present as ACUTE Emergencies in


the new born include:
 Diaphragmatic hernia
 Hypertrophic pyloric stenosis
 Micrognathia
 Aganglionic megacolon
 Meningocele
 Acute respiratory obstruction immediately after birth may be due
to:
 Patent ductus arteiosus
 Bilateral posterior choanal atresia
 Oesophageal atresia
 Goitre
 Mediastinal tumour
 A 2-day old neonate with respiratory disrress and cyanosis hasa
scaphold abdomen. The apex beat is in the 5th interspace on the
right of the sternum and no murmurs are heard.
You will suspect:
 Dextrocardia
 Congenital lobar emphysema
 Diaphragmatic hernia
 Atresia of the jejunum
 Co-arctation of the oarta
 Urgent measures will include:
 Administration of oxygen with a face mask
 Intubation and administration of oxygen
 Plain X-ray of the chest and abdomen
 Barium swallow
 Barium swallow administration of antibiotics
 In micrognatha:
 Respiratory obstruction is usually most severe when the baby is
sleeping

1
 The baby should be nursed on the lateral side
 Feeding is done with the baby lying prone over the mother who is in
the supine position
 Tube feeding is occasionally necessary
 The crisis is usually over by 3 months after birth

 Respiratory distress with mediastinal shift in a neonate may be


due to:
 Congenital lobar emphysema
 Tetralogy of Fallot
 Pneumothorax
 Pneumonia
 Lung cysts
 Meningomyelocele:
 Is less common than meningocele
 May cause paraplegia and urinary incontinence
 May be associated with hydrocephalus
 Requires urgent surgical treatment
 Is satisfactorily treated even when there is neurological deficit
 Tumours that may be present at birth include:
 Nephroblatoma
 Sacro-coccygeal teratoma
 Retinoblastoma
 Sternomatoid tumour
 Burkitt’s lymphoma
 Congenital anomalies that should as far as possible be corrected
in the first year of life include:
 Undescended testis
 Cleft lip
 Cleft palate
 Epispadias
 Inguinal Hernia
 Vomiting starting 3-4 weeks after birth may be due to:
 Duodenal atresia
 Meconium ileus
 Hypertropnic pyloric stenosis
 Malrotation of the gut

2
 Biliary atresia
 Persistent copious bile-stained vomiting within 48h of birth may
be
due to:

 Duodenal or jejunal atresia


 Meconium ileus
 High anorectal anomaly
 Cerebral injury
 Congenital hydronephrosis

 Congenital anomalies that do not pose immediate threat to the life


of a neonate but are potentially lethal include:

 Ventricular or atrial septal defect


 Exomphalos
 Epispadias
 Spina bifida with or without hydrocephalus
 The following are examples of major congenital malformations:

 Isolated cleft of the palate


 Bilateral posterior choanal atresia
 Cystic hygroma of the Lt. submandibular region
 Vascular signs of the trachea
 Idiopathic tension pneumothorax
 Successful management of congenital malformations depends
on:

 Accurate history
 Promptness of differential diagnosis and examination
 The quality of nursing care
 The ambience of the theatre setting
 The competence of the surgeon
 The following congenital malformations can be treated
conservatively:

 Micrognathia

3
 Pneumothorax
 Thymoma
 Tracheal collapse
 Naevus of the zygomatic areas
 Developmental abnormalities:

 Can present in adulthood


 Are usually present at birth
 Are classed as major depending on their size
 Are also classed as major if surgical intervention is required
 Have no known causation nor actiology

 Developmental abnormalities:

 Are confined to specific parts of the body


 Occur only in specific geographic locations
 Usually occur in combination with other malformations
 Affect survival depending on the size
 Affect survival also depending on the organ or system affected

4
CHAPTER 60

AN OUTLINE OF THE MANAGEMENT OF SOME


CONGENITAL ANOMALIES
ANSWERS

 A, C, D
 B, D, E
 B, C
 B, C
 C, D, E
 A. C, E
 B, C
 B, D
 B, E
 C
 A
 A, B, E
 B, C
 D, E

5
 C, E
 A, B
 C, E

6
CHAPTER 61
ULTRASOUND SCANNING COMPUTERIZED TOMOGRAPHY
MAGNETIC RESONANCE IMAGING AND RADIOISOTOPE
SCANNING
 The following statements are true about ultrasound imaging:

 It utilizes frequencies in the range of 100 KHz to 3 MHz.

 2MHz transducers are for deep structures or fat patients.

 The most commonly used display mode in clinical settings is the A-


mode.

 Doppler imaging is useful in differentiating a cyst from a mass.

 Ultrasound is useful for the detection of hydrocephalus in elderly


patients.

 Concerning endosonography, which statements are true?

 It is useful for evaluating the oesophagus.

 It is a better way of evaluating the ovaries.

 It is the best radiological modality for evaluating duodenal ulcers.

 It is commonly used for guiding prostatic biopsies.

 It is used intra-operatively to detect occult liver metastases.

 The following are characteristics of solids and cysts on


ultrasound:

 Simple cysts have internal echoes.

 Very hyperechoic masses often cast acoustic shadows.

1
 Cysts usually are associated with back-wall enhancement.

 Simple cysts have thin walls.

 Calculi often cast acoustic shadows.

 Which of the following are not indications for duplex and colour
doppler sonography:

 The determination of the cause of paralytic ileus.

 The determination of deep vein thrombosis.

 To determine tissue circulation prior to an intervention.

 To determine the nature of a cerebro-vascular accident in adults.

 To determine the arterial perfusion of transplanted organs.

 The following statements about CT scan imaging are true:

 The greater the density of the matter in the path of the x-ray beam,
the darker the image.

 Air has a density of 20 Hounsfeld units.

 Fat has a density between 0 and -100.

 CT scans are better than ultrasound at imagine gas.

 Oral contrast is often used to opacify the stomach and bowel.

 The following statements about MRI scan imaging are true:

 Blood flow can be detected without any contrast injection.

2
 It utilizes ionizing radiation

 Intravascular thrombi are seen by their strong signals.

 MRI imaging depends on the re-emission of radio waves from


protons in the tissues.

 Bone has low proton density and hence gives off high signal
intensity.

 Which of the following statements are true:

 CT is fast replacing MRI in medical imaging.

 MRI is better than CT for soft-tissue imaging.

 MRI is better than CT for detecting calcifications.

 Non-ionic iodinated contrast media are very useful in modern CT


imaging.

 Non-ionic iodinated contrast media are very useful in modern MRI


imaging.

 Which of the following statements are true:

 The first imaging modality to be requested in a suspected case of a


renal mass is an ultrasound scan.

 The first imaging modality to be requested in a suspected case of


blunt abdominal trauma is an ultrasound scan.

 The first imaging modality to be requested in a suspected case of


jaundice is an ultrasound scan.

 The first imaging modality to be requested in a suspected case of a


renal cyst is a plain x-ray.

 CT scans are better than ultrasound for intracranial imaging.

 Regarding radioisotope imaging:

3
 Xenon or krypton gas may be used for perfusion studies.

 Xenon or krypton gas may be used for ventilation studies

 Technetium imino diacetic acid derivatives are excreted by


hepatocytes in jaundiced patients.

 Technetium imino diacetic acid derivatives are not excreted by


hepatocytes in jaundiced patients.

 Adverse reactions and radiation doses for radioisotope scanning


are more than those of conventional x-ray.

 Which of the following statements are true:

 A barium meal study is important in the evaluation of rectal


bleeding.

 A barium swallow study is important in the evaluation of rectal


bleeding.

 A barium enema is more important than a barium meal in the


evaluation of rectal bleeding.

 Gastric ulcers are best detected with ultrasound.

 Gastric ulcers can be reliably detected with barium meal evaluation.

4
CHAPTER
IMAGING

 B, D

 A, B, D, E

5
 B, C, D, E

 A, D

 C, D, E

 A, C, E

 B, D

 A, C, E

 B, C

 A, B, E

6
CHAPTER 62

DAY SURGERY

 Which of these statements best defines day surgery?


 Quality surgical care in 24 hours, in as near normal environment
as can possibly be obtained
 Safe cost effective hospital care within 24 hours
 Hospital care with sacrosanct limit on 24 hours admission
 Foreshortened hospital care without complications
 24 hours hospital care without re-admission
 The benefits of day surgery include:
 Shortening of waiting lists for routine surgery
 Enhanced quality of care, as this ensures high level skillful
performance in surgical and anaesthetic techniques
 Recovery is quick, post operative pain readily controlled
 Patients convalesce in the security of their own homes
 Increasing availability of part time staff ensures coverage by
competent staff
 In tropical practice the ideal form of day surgery to implement
is:
 Day care surgery unit-self contained
 Day care ward-utilising the services in the main theatres
 Day care ward with separate recovery facilities
 Day surgery in general ward-using minor lists
 Day surgery in general ward-using day surgery lists
 Which factors determine the selection of patients for day
surgery in general surgery?
 Nature of pathology – Hernias, hydrocoeles, biopsies, lumps,

1
varicose veins, abscesses
 Severity of pathology
 Duration of disease
 Patient’s living distance from hospital
 Demonstrable willingness to abide by care instructions

 Candidates for day surgery:


 Should be A.S.A. Class I without exceptions
 Should be up A.S.A II providing underlying disability is controlled
 Should have a working telephone
 Should have a functioning household sewerage system including
internal lavatory.
 Should have own transport available
 Control of post operative pain is important in day surgery.
Which of these procedures is acceptable:
 All skin incisions are infiltrated with local anesthetic before
incision to reduce anaesthetic required.
 Sublingually or rectally delivered agents extend the routine
administration
of analgesics
 An oral dose of analgesic an hour before surgery is
recommended
 Anti-emetics are also indicated
 Appropriate posturing of the patient in the first 24 hour - is
essential to avoid postural hypotension
 Wound care is particularly important in day surgery. Which of
these procedures is desirable:

2
 The wounds should be closed with absorbable subcuticular
suture if possible
 The wound should be covered with transparent dressing
 There should be no interference with the dressing until the
wound is healed
 The wound needs to be inspected before discharge from centre
to ensure all bleeding has ceased
 The state of the wound is recorded before discharge

 Family benefits of day care surgery in children include:


 Parent may be available at home to take care of other siblings
 Extended family members may help take care of the child
 Siblings are left on their own to take care of themselves
 There is dislocation of the family
 There is reduction in cost to the patient
 Benefits of day care surgery to the child include:
 Child recovers in an environment he is used to resulting in
minimum
psychological disturbance
 Child is at increased risk of acquiring infections
 Child can relate with his siblings and thus has a better recovery
 Wound healing is much faster
 There is less need for the child to depend on the parents for care
10. Benefits of day care surgery to the hospital include:
A. Less use of bed space

3
B. Increased use of hospital personnel
C. Increased availability of financial resources
D. Increased visitation of doctors
E. More efficient use of time
11. Contra-indications to day care surgery in children include:
A. Long distance from the home
B. Preterm delivery of a child
C. Lack of education of parent
D. Absence of community health nurses
E. Low socio-economic status of parent

12. Co-morbid conditions that may make day care surgery unsafe
include:
A. Sickle cell disease
B. Cardiac failure
C. Extra-digit
D. Pre-auricular skin tag
E. Renal anomaly
13. Indications for day care surgery in children include:
A. Inguinal hernias
B. Hydrocoles
C. Umbilical hernias
D. Rectal Biopsies
E. Circumcisions

4
14. Those who must be involved in preparing a child for day care
surgery
include:
 The parents mainly
 The surgeon mainly
 The Anaesthetist mainly
 The nurse mainly
 All of the above
15. It is not possible to do day care surgery without the following:
A. A community health nurse
B. Internet facilities
C. Good Communication system
D. Good transportation system
E. A ward dedicated solely to day care surgery

CHAPTER 62
DAY SURGERY
ANSWERS

 A
 A, B, C, D, E
 A
 A, B, C, D, E
 B
 A, B, C, D, E
 A, B, C, D, E
 A, B, E

5
 A, C
 A, E
 A, B
 A, B, E
 A, B, C, D, E
 A, B, C, D, E
 C

6
CHAPTER 63
MINIMALLY INVASIVE SURGERY
 The following lesions can be diagnosed using the colonoscope:

 Intussusception

 Polyps

 Ulcerative Colitis

 Diverticular disease

 Volvulus

 The following is true of pneumoperitoneum during laparoscopy:

 Compartment syndrome occurs when the pressure is above10mm


Hg

 Oxygen is the best gas to use because it prevents hypoxia

 Omental emphysema is a known complication

 Metabolic acidosis may occur with the most commonly used gas

 The presence of pneumoperitoneum minimizes injury

 Laparoscopic surgery demands special training because of


difficulties with:

 Hand-eye co-ordination

 2D vision

 Lack of tactile feedback

 Complications of pneumoperitoneum

 Paradoxical hand movement

1
 In laparoscopic surgery:

 Shoulder tip pain is not an uncommon complication

 The two methods for gaining access are with the Verres needles
and the Hasson technique

 Advantages include a lower incidence of bile duct injuries

 Incisional hernias are extremely rare

 Trendelenburg positioning can aid cholecystectomy

 Complications of laparoscopy include the following:

 Cardiac arrhythmias

 Superior vena cava syndrome

 Air embolism

 Diffusion hypoxia

 Penetrating injury to urinary bladder

 Concerning Upper gastrointestinal endoscopy:

 The patient is sedated using a benzodiazepine

 The sedated patient is allowed to drive only after 6 hours

 Injection of botulinum toxin may give relief in achalasia

 Moniliasis of the oesophagus may be a pointer to HIV infection

 Dilatation of malignant strictures is contraindicated because of risk


of tumour dissemination

 The following procedures may be carried out at endoscopy:

2
 Confirmation of duodenal perforation

 Laser ablation

 Placement of gastrostomy tube

 Band ligation

 Evaluation of depth of penetration of a tumour

 The following are important for image production in laparoscopy:

 Automatic pneumoperitoneum insufflators

 Hopkins rod lens telescope

 Video camera

 Light source

 Verres needle

 Port site metastases may be caused by

 Turbulence of gas

 Mechanical inoculation

 Leakage around the trocar

 Not using enzymatic solution following surgery on a patient with


known cancer

 Tissue trauma

 Concerning thoracoscopic surgery:

 Patients with hyperhidrosis may benefit

 The pressure of CO2 used is 5mmHg less than in laparoscopy

 Lung resections are best not done thoracoscopically

3
 Complicaions include formation of bullae

 It is indicated for biopsy of mediastinal masses

CHAPTER 63
MINIMALLY INVASIVE SURGERY
ANSWERS

 A, B, C, D, E

 C, D, E

 A, B, C, E

 A, B

 A, C, E

 A, C, D

 B, C, D, E

 B, C, D

 A, B, C

 A, E

4
5
CHAPTER 22
THE THYROID AND PARATHYROID GLAND

1. In the anatomy of the thyroid gland the:

         A.  Isthmus lies in front of the first, second and third tracheal rings
B. Weight of the gland is 7-25 g in the African
C. “Strap muscles” consist of the sternothyroid, sterno-hyoid and the
inferior belly of the omo-hyoid
D. Medial relations of the lateral lobes are the hyoid, thyroid and cricoid
cartilages
upper six tracheal rings, oesophagus and recurrent laryngeal nerve
E. Superior and middle thyroid veins drain into the internal jugular vein
and the
Inferior thyroid veins into the innominate vein

2. In the physiology of the thyroid:

 T3 is four times more active than T4 by weight


 The iodine in the blood is concentrated 10-25 times by the gland
 T3 and T4 are bound in the blood only to thyroxine binding globulin
 Only about 0.05 per cent of T4 and 0.5 per cent T3 in the blood are
unbound, free and physiologically active
 T.S.H. stimulates the trapping of iodine and synthesis of thyroid
hormones
but not the storage and release of the hormones

3. Probably the most reliable current laboratory test in suspected


hyperthyroidism is:

 Serum T3 concentration
 T3 resin uptake
 Serum T4
 Serum free thyroxine
 Free T4 index

4. The most sensitive test of hypothyroidism is:

 Free T4 index
 Serum thyroid stimulating hormone
 Test of hypothalamic-pituitary axis

1
 Protein bound iodine
 Serum T3 concentration

5. In thyroid scan:

 If radioactive technetium is used, the scan is done at 30 minutes


 If radioactive iodine is used the scan is done at 12 h
 A cold (non-functioning) nodule may be an adenoma, a cyst or a
malignant tumour
 A nodule with most of the isotope concentrated in it is toxic or
autonomous
 A nodular goiter has the isotope concentrated in the nodules

6. Thyroglossal cyst:

 Is always in the midline


 Characteristically rises on protrusion of the tongue but does not move
on swallowing
 Is soft and fluctuant and attached to the skin
 May become infected
 May require multiple skin incisions for its removal and that of its tract

7. Goitre:

 Is an enlarged thyroid due to any cause except malignancy


 Is only palpable when the gland is at least 50 g
 May be euthyroid or hyperthyroid but not hypothyroid
 Afflicts at least 200 million people in the world according to WHO
estimates
 Occurs invariably in mountainous terrain

8. A simple goitre may be caused by:

 Low calcium content of drinking water


 Phenylbutazone
 Tolbutamide
 Some proprietory cough or asthma mixtures
 Paraminosalicylic acid

9. Endemic simple goitre:

 Exists when more than 10 per cent of any community have goitre
 Does not occur in West Africa

2
 Is caused principally by low iodine content of the local soil
 Is caused by cassava containing cyanogenic glucosides in some parts
of Africa
 Affects adolescent boys and girls in roughly the same ratio

10. Sporadic goitre:

 Is pathologically different from endemic goitre


 May be caused by pubertial growth, pregnancy, lactation or drugs
 Is not caused by iodine deficiency in the food
 Is clinically indistinguishable from endemic goitre
 May be caused by deficiency of enzymes necessary for oxidizing
iodine

11. In the pathology of simple goitre:

 There are initially localized areas of hyperplasia and hypertrophy


of the thyroid cells
 During the resting phase the follicles are distended with colloid
 Spurs of epithelium may project into the follicles
 Hyperplasia is followed by atrophy
 Nodules result from cycles of hyperplasia and atrophy

12. A patient with simple nodular goitre may complain of:

 Hoarseness of voice
 Snoring during sleep
 Difficulty in swallowing
 Oedema of face and conjunctivae
 Engorged manubrial subcutaneous veins

13. Simple nodular goitre:

 May occur at any age from the neonate to the elderly but the peak
incidence is from 21 to 30
 Affects women five times more than men
 Is symptomless in the vast majority of patients(apart from the
swelling)
 Is multinodular involving both lobes in about 50 per cent of patients
 Feels soft or firm and has indefinite edges

14. Diffuse hyperplastic goitre:

3
A. Is seen more commonly in endemic areas
B. May affect children
C. Is seen most commonly in girls from around puberty to 20
D. Is a uniformly enlarged and soft thyroid gland
E. Does not become big enough to compress the trachea

15. Complications of nodular goitre include:

 Tetany
 Infection
 Sudden haemorrhage
 Malignancy
 Myxoedema

16. Investigation(s) essential in simple nodular goitre is/are:

 X-ray of neck
 Free thyroxine index
 Serum T3 concentration
 Direct laryngoscopy
 Isotope uptake test

17. A nodular goitre apparently confined to one lobe is treated by:

A. Administration of thyroxine
B. Excision
C. Lobectomy of the affected lobe
D. Administration of Lugol’s iodine
E. Subtotal thyroidectomy

18. Diffuse hyperplastic goitre is usually treated by:

 Administration of thyroxine
 Subtotal thyroidectomy
 Administration of Lugol’s iodine
 Lobectomy
 Radio-iodine

19. Thyrotoxicosis:

 Is a clinical condition resulting always from the stimulating effects of


thyroxine

4
 If primary, is associated with a pre-existing diffuse hyperplastic
goiter
 If secondary, may be associated with malignant thyroid
 May be precipitated by ingestion of large doses of iodine in a patient
with long-standing goitre
 Is uncommon in Africans

20. In thyrotoxicosis:

 There is peripheral polymorphonuclear leucocytosis


 The thyroid and retro-orbital tissues are infiltrated by lymphocytes
and
follicles of lymphocytes may be seen in the thyroid
 Thyroid stimulating antibodies are found in the sera of all patients
 Immunoglogulins and complement are deposited in the stroma of the
thyroid
 There is peripheral lymphocytosis

21. In primary thyrotoxicosis:

 The acini are always small in size


 The acini may be increased in number
 The epithelial cells are always enlarged
 There may be infolding of the tall columnar epithelium into the lumen
of the acini
 The colloid of the acini is scanty

22. The following is/are found in the orbit and extraocular muscles in
exophthalmos

 Erythrocytes
 Plasma cells
 Giant cells
 Polymorphs
 Reticulocytes

23. Symptoms of thyrotoxicosis include:

 Constipation
 Insomnia

5
 Gynaecomastia
 Oligomenorrhoea
 Tiredness

24. SIGNIFICANT symptoms in thryotoxicosis include:

 Preference for cold weather


 Dyspnoea on effort
 Excessive sweating
 Occasional irritability
 Increased appetite and normal weight

25. The IMPORTANT signs in thyrotoxicosis include:

 Moist hands
 Auricular fibrillation
 Lid lag
 Palpable thyroid
 Hyperkinetic movements

26. Exophthalmos:

 Is protrusion or prominence of the eyeball


 Is always bilateral
 Occurs in most patients with primary thyrotoxicosis
 If malignant maybe associated with a euthyroid state
 Is usually painful

27. Feature(s) of exophthalmos is/are:

 Conjunctival oedema
 Diplopia
 Corneal ulceration
 Diminishing visual acuity
 Directly related to the severity of hyperthyroidism

28. Pertibial myxoedema:

 Involves only the pretibial skin


 May not be bilateral
 Is characterized by irregular thickening of the skin with coarse hair
 Is caused by infiltration of the skin with mucin-like substance
 Is associated with myxoedema

6
29. The thyroid gland in thyrotoxicosis:

 Is enlarged in all patients


 If primary is uniformly enlarged and rather soft or firm
 If secondary, is firm and multinodular in all patients
 In over 95 per cent has a thrill and bruit over it
 Shrinks to normal size during treatment with antithyroid drugs

30. In a district hospital in West Africa, the investigations(s) needed to


aid the
diagnosis of thyrotoxicosis in most patients is/are:

 Serum T4, T2 resin uptake, FT41


 Wayne’s clinical diagnostic index
 Isotope uptake test
 Sleeping pulse chart
 Achilles tendon reflex duration

31. Primary thyrotoxicosis and anxiety neurosis can be differentiated


by:

 Weight loss
 Blood pressure
 Sleeping pulse rate
 Restlessness
 Appetite

32. The differential diagnoses of unilateral exophthalmos include:

 Primary thyrotoxicosis
 Meningioma
 Carotid-cavernous fistula
 Optic nerve glioma
 Orbital tumour

33. In the drug treatment of thyrotoxicosis:

 Propranolol reduces anxiety


 Methly thiouracil prevents trapping of iodine and binding of iodine

7
to tyrosine
 Potassium perchlorate prevents trapping of iodine and coupling of
MIT and DIT to T2 and T4
 Carbimazole prevents binding of iodine to tyrosine and coupling of
MIT and DIT to T2 and T4
 Phenobarbitone relieves palpitation and excessive sweating

34. In the drug treatment of thyrotoxicosis:

 The patient usually becomes euthyroid in about 4 weeks


 Treatment is continued at maintenance dose for 6 months after the
patient becomes euthyroid
 50% of patients relapse in 1-6 months after cessation of treatment
 Digoxin may be indicated
 It is inadvisable to give thyroxine as it delays remission

35. In the treatment of thyrotoxicosis the adult dose of:

 Carbimazole is 10-15 mg 8 hourly


 Sodium perchlorate is 10-20 mg 8 hourly
 Propyl-thiouracil is 200-400 mg 8 hourly
 Carbimazole at maintenance level is 1-2 mg 8 hourly
 Methylthiouracil is 50-100 mg 8 hourly

36. Toxic reactions of anti-thyroid drugs include:

 Diarrhoea
 Arthralgia
 Skin rashes
 Depression
 Anaemia

37. Drug therapy in thyrotoxicosis is the treatment of choice in:

 Patients with large goitres


 Men
 Adolescents
 Patients with exophthalmos
 Secondary thyrotoxicosis

8
38. Radio-active iodine:

 Depresses the activity of most of the thyroid cells but does not
destroy them
 Is fully effective in 4 weeks
 May induce carcinomaof thethyroid in the elderly
 Is only given to patients over 55 years
 Leads to hypothyroidism in 40-50% of the patients in 5 years

39. Indications for subtotal thyroidectomy in thyrotoxicosis include:

 Relapse after previous drug therapy


 Recurrence after previous operation
 Drug goitre
 Thyrocardiac
 Multinodular goitre

40. Subtotal thyroidectomy in thyrotoxicosis:

 Gives rapid and permanent cure


 Involves removal of three-quarters of the thyroid
 Is the elective treatment in all patients in the developing countries
because patients may not take their drugs
 Is contra-indicated if the gland is impalpable
 Should be preceded by laryngoscopy as about 15% of patients may
have cord paralysis without clinical manifestation

41. Propranolol:

 Is an alpha-blocking agent
 Controls the symptoms and signs of thyrotoxicosis because it can
depress the production of thyroxine
 Is effective within 24 h of administration and the dose is80 mg orally
daily
 Is usually given for 4-7 days before and 10-14 days after subtotal
thyroidectomy
 Is currently preferred to the usual anti-thyroid drugs for making toxic
patients euthyroid

42. Soon after the removal of the endotracheal tube after subtotal

9
thyroidectomy
for secondary thyrotoxicosis, a patient is found to be having
severe difficulty
in breathing.

The most likely cause is:

 Collapse of the trachea


 Haemorrhage
 Oedema of the larynx
 Damage to both recurrent laryngeal nerves
 Massive collapse of a lung

43. You will do the following:

 Pass a bronchoscope and aspirate any mucus plugs


 Open the wound immediately
 Reinsert the endotracheal tube
 Give oxygen
 Do a tracheostomy

44. Respiratory obstruction due to post-operative haemorrhage after


subtotal tyroidectomy:

 Usually occurs within 24 h after operation


 Is due to collection of blood in the subcutaneous space
 Is prevented by a well-placed corrugated drain
 Occurs only if there is considerable collection of blood in the wound
 Is best treated by the team that performed the operation

45. Recurrent laryngeal nerve paralysis:

 Is permanent
 If unilateral causes hoarseness of voice and non-productive cough
 If bilateral causes severe hoarseness of voice and difficulty in
breathing
 If bilateral requires early repair of the nerves
 If bilateral and there is no recovery after one year, requires
arytenoidectomy with lateral fixation of one cord

10
46. Hypoparathyroidism after subtotal thyroidectomy:

 Usually occurs 10-14 days post-operatively


 Causes paraesthesia and numbness of the distal parts of the limbs
 Causes weakness of the limbs
 Is controlled with vitamin D
 Is usually temporary as all the parathyroids are removed only rarely

47. Thyrotoxic crisis after subtotal thyroidectomy for thyrotoxicosis:

 Is fairly common
 Is caused by excessive release of TSH in the post-operative period
 Is characterized by excitation, profuse sweating, hyperpyrexia and
and tachycardia
 Is treated with propranolol, Lugol’s iodine, carbimazole, diazepam and
intravenous fluids
 May require oxygen administration and cold sponging

48. In the late complications of subtotal thyroidectomy for


thyrotoxicosis:

 Hypothyroidism occurs in about 10% of patients in 2-5 years


 Recurrence of thryrotoxicosis occurs in about 2% of patients
 All the recurrence occur in the first year post-operatively
 Hypothyroidism is best treated with thyroid extract
 Tracheal compression from fibrosis around it may occur

49. In the treatment of severe, progressive exophthalmos:

 Ophthalmic solutions of guanethidine or bethanidine in methylcellulose


is instilled locally for adrenergic blockade
 Steroids are given in large doses to reduce oedema and for
Immunosupression
 Metronidazole is given for its anti-microbial effect
 Lateral tarsorrhhaphy may become necessary to protect the eye
 Transcranial orbital decompression must be done if the eyelids
cannot be apposed

50. Thyrotoxicosis in pregnancy may cause:

11
 Abortion
 Postmature labour
 Stillbirth
 Eclampsia
 Foetal goitre and hypothyroidism

51. In the treatment of thyrotoxicosis in pregnancy:

 Drug treatment is effective


 The drugs should be discontinued in the last 2 months of pregnancy
because of their possible effects on the foetus
 The baby’s T.S.H. and FT4 should be done after birth
 Subtotal thyroidectomy is effective
 Subtotal thyroidectomy should be done in the first trimester and
post-operative thyroxine given to prevent hypothyroidism in the foetus

52. Thyrotoxicosis in children:

 Is best treated by antithyroid drugs for 3 years


 May relapse in most of the patients after
 drug therapy, further treatment
being required
 After drug treatment, permanent remission occurs in 50-75% of
patients
 May present as chorea or emotional disturbance
 May be treated by operation if there is relapse

53. Subtotal thyroidectomy for thyrotoxicosis in children has a:

 Low incidence of hypothyroidism


 Low incidence of recurrence
 High incidence of tetany-about 17%
 High incidence of permanent hypoparathyroidism-about 10%
 High incidence of damage to the recurrent laryngeal nerves

54. Hypothyroidism:

 Is fairly common in Africa


 May be due to excessive ingestion of P.A.S., phenylbutazone

12
or aspirin
 If congenital is always associated with a goitre
 In the adult is not associated with a goitre
 If congenital causes mental and physical underdevelopment
sometimes
with deaf-mutism

55. Clinical features of hypothyroidism in the adult include:

 Paraesthesia with muscle pain


 Diminished sweating but tolerance to cold
 Hoarse voice
 Puffiness of supraclavicular fossa, wrist and periorbital area
and fine hair
 Pulse lower than 80/minute and normal ankle jerks

56. Suppurative thyroiditis:

 Usually occurs in goitrous nodules often after previous


haemorrhage
 Is most commonly caused by staphylococcus
 Presents with acute pain in a previously painless goitre with dysphagia
and often earache
 On examination gives a very tender warm goitre which does not move
on swallowing
 Is treated by antibiotics and needle aspiration of any abscess

57. In auto-immune thyroiditis (Hashimoto’s disease):

 Auto-antibodies to thyroxine-binding globulin are the cause


 The parenchyma of the thyroid is destroyed in some parts and there
are no areas of hyperplasia
 The interstitial tissue is infiltrated with plasma cells and lymphocytes
with active germinal centres
 There may be areas of fibrosis between the lobules
 The gland in all patients is atrophic and replaced by fibrous tissue and
lymphocytes

58. Auto-immune thyroiditis may present with:

13
A. Pain or discomfort in the thyroid area of the neck
B. Severe hyperthyroidism
C. Hypothyroidism
D. Nodular goitre
E. Solitary nodule

59. In the management of auto-immune thyroiditis:

A. The diagnosis can only be established by needle biopsy


B. Prednisone is given for about 10 days
C. Thyroxine is given for 1-3 months
D. Subtotal thyroidectomy may become necessary
E. A course of antibiotics may be benefical

60. Adenoma of the thyroid:

 Is more common in men


 Is firm, encapsulated and movable within the thyroid
 May be functional and “hot” or non-functional and “cold”
 Does not cause thyrotoxicosis
 Is treated by excision

61. Malignant neoplasm of the thyroid:

 Is fairly common
 Is seen from childhood to old age with a peak incidence in Africa
In the 4th decade
 Is 3 times more common in men
 May arise in a nodular goitre
 May present with hypothyroidism

62. In the aetiology of malignant neoplasms of the thyroid:

A. Children exposed to irradiation of the head and neck may develop


carcinoma 20 years later
B. About 38% of those who were exposed to the atom bomb in Hiroshima
and Nagasaki developed carcinoma of the thyroid
C. Sustained T.S.H. stimulation in endemic areas induces predominantly
papillary carcinoma
D. Sustained T.S.H. stimulation in non-endemic areas induces

14
predominantly
mixed papillary and follicular carcinoma
E. Heredity is a factor in some cases of medullary carcinoma

63. Papillary adenocarcinoma:

A. Is the commonest carcinoma of the thyroid in the world


B. Accounts for most cases in children
C Has the highest incidence in the 2nd decade
D. Is slow-growing and remains localized for a long time
E. Metastasizes usually to the cervical and upper mediastinal lymph
nodes and the blood stream

64. Follicular adenocarcinoma:

A. Is the commonest carcinoma of the thyroid in Nigeria


B. Has the highest incidence in the 6th decade
C. Is slow-growing and usually osteosclerotic when it metastasizes
to bone
D. Contains colloid and its metastases can concentrate radio-iodine
E. May present with a pathological fractures

65. Medullary carcinoma:

 May be associated with “café au lait” spots and neurofibromata


 Is often inherited as a recessive genetic trait
 May secrete histamine, prostaglandins, somatostatin, serotonin
and calcitonin
 Has the highest incidence in the 6th and 7th decades
 Spreads by the lymphatics and blood vessels

66. Anaplastic adenocarcinoma:

 May present with respiratory obstruction or dysphagia


 Consists entirely of spindle-shaped small cells
 Spreads early by direct infiltration and only later via the lymphatics
and blood vessels
 Occurs in all age groups especially the young
 Is radio-sensitive

15
67. A 35-year old woman presents with a solitary thyroid nodule which
has been
present for about 4-years, but has recently been enlarging. It is firm,
rather
circumscribed and fairly mobile. There are no enlarged cervical
lymph nodes.

You will consider:

A. A cyst
B. An adenoma
C. Nodular goitre
D. Papillary or follicular adenocarcinoma
E. Anaplastic carcinoma

68. If the scan shows it to be “hot” with some uptake in the rest of the
thyroid,
you will consider:

A. A cyst
B. An adenoma
C A papillary or follicular adenocarcinoma
D. Nodular goitre
E. An autonomous nodule
69. If the scan shows it to be “cold”, you will consider:

 A cyst
 An adenoma
 A papillary or follicular carcinoma
 Nodular goitre
 An autonomous nodule

70. Carcinoma of the thyroid may present as:

 Horner’s syndrome
 Cervical lymphadenopathy without thyroid swelling
 A pulsatile skull swelling
 Hyperthyroidism
 Diarrhoea

16
71. Measures taken in the treatment of operable papillary or follicular
carcinoma
of the thyroid include:

 Near-total thyroidectomy
 Irradiation
 Radio-active iodine
 Cytotoxic therapy with adriamycin or bleomycin
 Administration of thyroxine

72. Treatment measure in anaplastic carcinoma of the thyroid include:

 Near-total thyroidectomy
 Irradiation
 Radio-iodine
 Cytotoxic therapy with bleomycin and adriamycin
 Administration of thyroxine

73, In the prognosis of carcinoma of the thyroid:

 84% and 60% of those with papillary carcinoma are alive at 5 and 30
years
 60% and 40% of those with follicular carcinoma are alive at 5 and 30
years
 No patient with anaplastic carcinoma is alive at 3 years
 Those over 40 have a better prognosis than those under 40
 Women have a better prognosis than men

74. The most frequently encountered symptom (Hall mark) of


thyrotoxicosis is:

 Palpitations
 Profuse sweating
 Progressive weight loss inspire of polyphagia
 Consistent heat intolerance
 Diarrhoea

75. In clinical diagnosis the most dependable sign (Hall mark) of


thyrotoxicosis
is:

17
 Moist hands
 Atrial fibrillation
 A sleeping Pulse Rate > 120/min
 A sleeping Pulse Rate > 90/min
 Pretibital mxyoedema

76. In the Exophthalmos seen in the Hyperthyroidism the:

 Upper eyelid is tangential to the pupil


 Upper eyelid is at least tangential to the upper limbus
 Upper eyelid exposes the sclera in all cases
 Lower eyelid is tangential to the limbus
 Lower eyelid retracts to expose the sclera

77. In a cervical swelling which moves with swallowing, which


combination of
clinical signs supports a diagnosis of thyroid cancer (follicular
carcinoma):

A. Firm, Tender, ill defined


B. Soft non tender, clear edges
C. Firm non tender, clear edges
D. Soft tender well defined
E. Rubbery, non-tender well defined

78. Which of these investigations would you order:

 X-rays of the neck for the thoracic inlet


 Ultrasound scan of the neck swelling
 Radioiodine (T132) uptake of entire gland
 Open tissue biopsy
 Estimate serum level of calcitonin

79. A man of 45 presents with a painless right sided anterior neck


swelling which
moves with deglutition. On examination the lesion is a non tender,
well defined solitary nodule; there were no signs of toxicity, but
patient has hot flushes and diarrhoea

The most likely diagnosis is:

18
 Benign solitary nodule
 Hashinoto’s thyroiditis
 Lymphadenoid goitre
 Papillary carcinoma of the thyroid
 Medullary carcinoma of the thyroid

80. Your investigation of choice is:

 Plain x-ray of neck to show thoracic inlet


 Ultrasound scan of the gland
 FNAC under ultrasound guidance
 Radioactive lodine uptake test
 MRI study of the thyroid gland

81. For treatment you would recommend:

 Daily dosing with L. Thyroxine 0.1 mg od. for 3 months


 Radioiodine therapy
 External Radiation with linear accelerator
 Total Thyroidectomy
 Subtotal Thyroidectomy

82. In the anatomy of the parathyroid glands:

A. The colour is purple


B. They are situated on the medial borders of the thyroid lobes
C. They may be found in the superior, anterior or posterior mediastinum
D. They are usually on the surface of the thyroid but may be embedded in
it
E. There are four but the number may vary

83. Parathyroid hormone:


 Increases resorption of calcium from bone
 Decreases renal tubular reabsorption of calcium
 Increases proximal tubular reabsorption of phosphate
 Promotes absorption of calcium from the gut
 Secretion is controlled by the level of the plasma concentration
of ionic calcium

84. Clinical features of hypoparathyroidism include:

19
 Convulsions
 Stridor and dyspnoea
 Excessive sweating
 Pyrexia
 Restlessness
85. Late complications of hypoparathyroidism include:

 Chronic duodenal ulceration


 Cataract
 Dementia
 Chronic renal failure
 Mental sluggishness

86. Diagnostic laboratory findings in hypoparathyroidism include:

 Low serum phosphate


 High urine phosphate
 Low serum calcium
 Low or absent urine calcium
 High alkaline phosphatase

87. Hyperparathyroidism may be caused by:

 A simple adenoma of the parathyroid in a few patients


 Multiple adenomata in the majority of patients
 Hyperplasia of all the glands in some patients
 Carcinoma of the parathyroid in about 10% of patients
 Chronic renal failure, rickets, Vit. D into-xication or steatorrhoea
in a few patients

88. Hyperparathyroidism:

 Is an uncommon disease
 Is more common in men
 May be seen at any age
 May be symptomless
 May precipitate acute pancreatitis

89. Symptoms of hyperparathyroidism include:

20
 Anorexia
 Nausea and vomiting
 Muscle cramps
 Numbness around the lips, polyuria and polydipsia
 Restlessness

90. Bony features of hyperparathyroidism include:

 Thickening of the mandible or maxilla


 Shortening of the height
 Pathological fractures
 Bone pain and tenderness especially of the spine and shoulder
 Cystic bony swellings

91. Radiological features of hyperparathyroidism include:

 Sclerotic skull
 Subperiosteal resorption of bone seen particularly in the
middle phalanges of the index and middle fingers, upper part
of the tibia and neck of the femur
 Nephrocalcinosis
 Periosteal thickening of the radius and ulna
 Calcification of the spleen

92. In hyperparathyroidism:

 The plasma calcium is over 2.55 mmol/L and may be as high as


5mmol/L
 The plasma phosphate is low, less than 0.8 mmol/L in about 90% of
patients
 The urinary phosphates and calcium are elevated
 The plasma calcium remains elevated when cortisone 150mg is given
daily if primary hyperparathyroidism
 The serum chloride is less than 95 mmol/L and the serum bicarbonate
more
than 24 mmol/L

93. Hypercalcaemia may be caused by:

21
 Excessive intake of antacids
 Multiple myelomatosis
 Sarcoidosis
 Osteitis deformans
 Multiple bone metastases

94. Symptoms of hypercalcaemic crisis include:

 Severe weakness
 Abdominal cramps
 Copious vomiting
 Irritability
 Drowsiness

95. In the management of hypercalcaemia due to acute


hyperparathyroidism:

 3 L of normal saline is given in 9 h to restore extracellular fluid volume,


establish diuresis and form a complex with calcium which is excreted
by
the kidney
 I.V. phosphate 1 g or oral phosphate 3 g daily is given
 Prednisone 30-100 mg daily helps to reduce the serum calcium by
reducing mobilization of calcium from bone
 Haemodialysis may be required
 Exploration of the parathyroids is undertaken as soon as the patient’s
general condition improves

96. In the post-operative management after excision of an adenoma of


the
parathyroid:

 Chvostek’s and Trousseau’s signs are checked 4 hourly


 The serum calcium is checked daily
 The serum phosphate is checked daily
 The urinary excretion of calcium is estimated daily
 If the serum calcium becomes low calcium gluconate, Vit. D2 and
EDTA
are administered simultaneously.

22
CHAPTER 22
THE THYROID AND PARATHYROID GLAND

ANSWERS

 A, B, E

 A, B, D

 D

 B

 C

 D, E

 D, E

 B, C, D, E

 A, C, D, E

 B, D, E

 A, B, C, D, E

 A, B, C, D, E

 C

 A, B, C, D

 C, D

23
 A

 C

 A, B, C

 C, D

 B, E

 B, D, E

 B

 B, C, D, E

 A, C

 A, B, D, E

 A, D

 A, B, C, D

 B, C, D

 B

 B, D

 C, E

 A, B, C, D, E

 D

 A, C, D

 A

24
 B, C, E

 C

 E

 A, E

 A, C, D

 C

 D

 C, D, E

 A

 B, E

 B, E

 C, E

 A

 A, B, D

 A, C

 A, D

 A, C

 C, D

 B, E

 A, C, D

25
 A, C

 A, D

 A, C, D

 B, D

 B, C

 B, D

 A, E

 A, B, D

 A, D, E

 A, C, E

 A

 B

 B, E

 A, B, C, D

 A, B, D, E

 A, C

 B, D

 A, B, E

 D

 D

26
 B, E

 A

 B

 E

 C

 D

 C, D, E

 A, D, E

 A, B

 B, C, E

 C, D

 C, E

 A, C, D, E

 A, B, D, E

 E

 B, C

 A, C, D

 B, C, E

 A, C, D, E

 C, D, E

27
 B

28
SURGERY

CHAPTER 23

THE NECK

 In a cut throat, if the wound is above the level of the hyoid


cartilage the following structures may severed:

A. Epiglotis
B. Common carotid artery
C. External jugular vein
D. Internal jugular vein
E. Brachial plexus

 In the management of a cut throat, the most essential measure is


to:

A. Stop any bleeding


B. Treat for shock
C. Prevent infection
D. Ensure a clear airway
C. Prevent air embolism

 Complications of cut throat include:

A. Horner’s syndrome

 Aerial fistula
C. Dysphonia

1
D. Mediastinitis
E. Surgical emphysema

 Ludwig’s angina:

A. Is severe inflammation of the mouth


B. Is caused by staph aureus

 May cause respiratory obstruction

 Should always be drained

 Is often dental in origin

 A branchial cyst:

 Is persistence of the space formed by fusion of the second with the


fifth
branchial arch

 Contains a clear thin fluid

 Is usually lined by columnar epithelium

 Usually presents in childhood

 Is more common in females

 A branchial fistula:

A. Is usually acquired from rupture of a branchial cyst


B. Is in most cases a true fistula with the internal opening in the tonsilar

2
pit
C. Usually presents around puberty
D. Opens at the junction of the middle and lower thirds of the anterior
border of
the sternomastoid

 Passes between the internal and external carotids

 A cystic hygroma:

 Is a congenital carvenous lymphangioma consisting of a unilocular


cyst
containing clear fluid

 Occurs in the posterior triangle of the lower third of the neck, axilla
or cheek

 May cause respiratory obstruction

 Has indefinite edge

 May regress spontaneously if it becomes infected

 Chemodectoma:

A. Usually benign but 20% are malignant


B. A tumour of the internal carotid artery
C. Soft and transmits the carotid pulsation
D. Located at the level of the hyoid bone deep to the sternomastoid
E. Mobile in all directions

 A firm immobile swelling in the mid-lateral part of the neck of a


neonate is most likely a:

A. Lymph node
B. Lipoma
C. Sternomastoid tumour
D. Aberrant thyroid

3
E. Neurofibroma

 A soft fluctuant, circumscribed, midline swelling under the chin of


a one-year old boy is most probably a:

A. Thyroglossal cyst
B. Dermoid cyst
C. Brancial cyst
D. Sebaceous cyst
E. Ranula

A 25 year old man has a 3 month old, painless swelling below the
angle of the mandible. It is tense, fluctuant mobile, and most
deep to the sternomastoid

 The most likely diagnosis is:

A. Chronic abscess
B. Pharyngeal diverticulum
C. Lipoma
D. Branchial cyst
E. Laryngocele

 Painless, multiple, matted hard swellings with skin attachment in


the mid-lateral part of the neck in a 45-year old man may be due
to:

A. Metastases
B. Hodgkin’s lymphoma
C. Tuberculosis
D. Sarcoidosis

4
E. Multiple neurofibromatosis

 Suspected malignant right supra clavicular lymphadenopathy in


50 years
old man may be due to primary cancer of:

A. Larynx
B. Right lung
C. Stomach
D. Pancreas
E. Right breast

 A firm mobile swelling behind the ramus of the mandible in a 40-


year old man may be:

A. Chemodectoma
B. Lymph node
C. Parotid tumour
D. Submandibular salivary gland tumour
E. Lipoma

 Injuries in the posterior triangle of the neck:

A. Unless directed inferiorly seldom involve vital structures


B. Very frequently involves vital structures
C. Must be explored in all cases
D. Is often associated with dysphonia
E. CT scan is the investigation of choice

 In Ludwig’s angina:

A. Strept viridians and E. coli are the usual organisms


B. Advanced carcinoma of the floor of the mouth may be a causative
factor

5
C. Excessive salivation and trismus are presenting symptoms
D. Broad spectrum antibiotics such as cefuroxime and metronidazole
should be
Administered
E. The tongue is oedematous and is displaced upwards and forwards

 In tuberculous cervical lymphadenitis:

A. The upper deep cervical lymph nodes may be invaded by the


tubercle via the
tonsils in adults and the elderly
B. The lower cervical nodes may be secondarily infected from
pulmonary
tuberculosis via the mediastinal nodes in children and young adults
C. Occasionally all the cervical nodes are affected probably as a result
of blood-
borne invasion
D. HIV/AIDS may be a predisposing factor
E. Lymph nodes in the supraclavicular region or posterior triangle are
affected in
the elderly

 In the investigation of tuberculous lymphadenitis:

A. Lymphopaenia will be found if there is associated HIV

6
B. The ESR may be normal
C. Screening for HIV is not essential
D. The Mantoux test, unlike in pulmonary tuberculosis, is usually
unremarkable
E. Excision biopsy may be necessary to establish the diagnosis

 Branchial cyst:

A. Has abundant lymphoid tissue in the wall


B. Occurs mostly in patients aged 20-25 years
C. Is fluctuant and transilluminates light
D. Protrudes from the anterior border of the lower third of the
sternomastoid
muscle
E. Has to be differentiated from a cyst of the space of Burns

 Brancial fistula:

A. Arises from failure of the second branchial arch to fuse with the fifth
arch
B. Arises from failure of the first branchial arch to fuse with the fourth
arch
C. Has abundant lymphoid tissue in the wall
D. Should be excised through 2 or 3 small transverse incisions over the
course
of the tract
E. Is lined by columnar and ciliated epithelium in its inner portion and
stratified
squamous in its outer portion

 Cystic hygroma:

A. May be present at birth and may cause obstructed labour


B. Is brilliantly translucent

7
C. Is best treated by sclerotherapy as this has been found to be very
effective
D. Is completely compressible
E. Has a 90% chance of recurrence following excision

 Carotid body tumour:

A. Is a rare but rapidly growing tumour of the carotid body


B. Occurs in patients aged 30-50 years
C. Diagnosis is confirmed by Duplex Doppler Imaging
D. Cannot be excised because of its location
E. Is best treated by radiotherapy

8
9
CHAPTER 23
THE NECK
ANSWERS

 A, C, D

 E

 B, C, D, E

 C, E

 A, E

 C, D, E

 B, C, E

 A, D

 C

 B

 D

 A

 A, B, C

 B, C

 A

 A, B, C, D

 C, D, E

10
 A, E

 A, B, C

 A, C, D, E

 A, B

 B, C

11
CHAPTER 25

DIAPHRAGM

 The origins of the muscular partition which is the diaphragm


include the following except:

 Each of the lower six ribs close to the costochondrial junction

 Posterior aspect of the xiphoid process

 The body of the sternum

 The bodies of the upper two (left) or three (right) lumbar


vertebrae

 The lumbo-costal arches-the medial and lateral arcuate


ligaments

 The two domes of the diaphragm rise in full expiration to as high


as:

 The 4th intervertebral space on the right, 5th on the left

 The 5th intervertebral space on the right, 4th on the left

 The 4th intervertebral space on the right, 3rd on the left

 The 5th intervertebral space on the right 6th on the left

 The 6th intervertebral space on the right 5th on the left

 The oesophageal opening through the diaphragm occupies the:

1
 Level of T10 and 2.5 cm to the left of the mid-line

 Level of T10 and 2.5 cm to the right of the mid-line

 Level of T9 and 2.5 cm to the left of the mid-line

 Level of T9 and 2.5 cm to the right of the mid-line

 Level of T11 and 2.5 cm to the left of the mid-line

 Which of the following structures accompany the oesophagus


through the oesophageal opening of the diaphragm?

 The two vagal nerves and the azygos vein

 The two vagal nerves and the left gastric artery

 The two vagal nerves and the left phrenic nerve

 The posteror vagus and the two sympathetic trunks

 The two sympathetic trunks only

 Which of the following features are true of the caval opening


through the

diaphragm?

 Occurs at T7 level and transmits the right phrenic nerve as well

 Occurs at T8 level and is closely associated with the passage


opening

 for the right phrenic nerve

 Occurs at T9 level and transmits the right phrenic nerve as well

 Occurs at T10 level and 1cm away from the site of entry for the
right phrenic nerve

2
 Occurs at T11 level and transmits the right phrenic nerve

 Competence of the gastro-oesophageal junction is primarily


dependent on:

 The closely fitting left crus of the diaphragm

 The oblique angle of entry of the oesophagus into the cardia

 The mucosal rosette or fold at the cardia

 The functional lower oesophageal sphincter pressure

 An anatomical gastro-oesophageal sphincter

 Which one of these options comprehensively describes the


function of the diaphragm:

A. Powerfully active in movements of respiration


B. Exerts indirect effect on the circulation
C. Actively effects respiration and indirectly influences the circulation
 Active in quiet respiration, influences the circulation and
indirectly affects intra abdominal organs

 Active in respiration, and maintains circulation

 The largest component of the three peripheral muscular origins of


the diaphragm is:

A.  The Central tendon


B. The Lumbar
C. Costal
D. Sternal
E. Psoas major

 Concerning the topographical relations of the diaphragm:


A. The right dome in full expiration rises to the 4th costal cartilage
B. The left done in full expiration rises to the 5th costal cartilage

3
C. The central tendon is level with junction of the 6th costal cartilage
with the
sternum
D. The largest foramen is at level of the 12th dorsal vertebra
E. The oesophageal opening is at level of 10th dorsal vertebra
F. The oesophageal opening also transmits the two vagi and the
branches
of the left gastric artery
G. The inferior vena caval opening is at level of the 8th dorsal vertebra
H. The caval opening also transmits the right phrenic nerve

 The nerve supply of the diaphragm is derived from:


A.  Medial supra clavicular nerve (C3, 4, 5)
B. Intermediate supra clavicular nerve (C3, 4, 5)
C. Lateral supra clavicualr nerve (C3, 4, 5)
D. Phrenic nerve (C3, 4, 5) and lower six intercostals nerves
E. The vagus nerve

 An obese woman of 54 presents with retrosternal pain unrelated


to meals but aggravated when in bed at night and relieved by
antacids. She had only epigastric tenderness as the objective
sign on physical examination. Upper gastrointestinal series
were reported as non-contributory but the stools were positive
for occult blood.

The most likely clinical diagnosis is:

 Acute exacerbation of duodenal ulcer

 Hookworm infestation

 Chronic cholecystitis

 Reflux oesophagitis

 Chronic gastric ulcer

 This may be confirmed by:

4
 Oesophageal and gastric motility studies

 Upper gastro-intestinal pH monitoring

 Oesophago-gastroduodenoscopy

 Routine stool examination

 Oral cholecystogram

 You would advise:

 Bed rest, antacids and milk drip

 Oral bephenium hydrochlotide

 Course of cefoxitin and observation

 Weight reduction, avoidance of corsets, sucking antacids

 Billroth 1 partial gastrectomy

 Which of the following condition(s) would be considered


indications(s)

for operation in hiatal hernia?

 Predominating symptoms of oesophagitis

 Oesophageal stenosis

 Haemorrhage

 Water brash

 Nocturnal dyspepsia

5
 The most essential element in repair of hatus hernia is:

 Narrowing of the oesophageal hiatus

 Re-establishment of an infradiaphragmatic segment of

oesophagus

 Restoration of the angle between oesophagus and stomach

 Fixation of the gastric fundus to the anterior abdominal wall

 Crural butressing with non absorbable material

 Concerning hiatal hernia:


A. Accounts for 98% of diaphragmatic herniae
B. Are considered quite common among African Subjects
C. Are practically nearly always congenital in origin
D. Have no relation to the body habitus of the patient
E. Are usually para-oesophageal in type (85%)

 Sliding hiatus hernia:

A. Is the commonest variety (85 percent of hiatus herniae)


B. Is always associated with gastro oesophageal reflux
C. In most cases exist in patients with a competent cardia
D. Is frequently associated with a congenitally “short oesophagus”
E. Is most readily diagnosed from appearances at upper GI endoscopy
F. Is often so large as to produce cardiac compression erymptons

 The essential mechanism responsible for the competence of the


gastro oesophageal junction is:

A. The anatomical closely fitting left crus of the diaphragm


B. Positive intra-abdominal pressure acting on the infra-diaphragmatic
segment of the oesophagus
C. The acute angle between the lower oesophagus and stomach
D. The physiological lower oesophageal sphincter (L.O.S)
E. The plugging effect of the gastric “mucosal rosette”

6
 At the lower oesophageal sphincter (LOS):

 A critical pressure of < 6mmHg and sphincter length of < 1cm carry
a 90% chance of reflux
 A critical pressure of < 12mmHg and sphincter length of < 2cm
carry
a 90% chance of reflux
 A critical pressure of < 18mmHg and sphincter length of < 3cm
carry
a 90% chance of reflux
 A critical pressure of < 24mmHg and sphincter length of < 4cm
carry
a 90% chance of reflux
 A critical pressure of < 30mmHg and sphincter length of < 5cm
carry
a 90% chance of reflux

 In patients with sliding hiatal hernia:

A. The diagnosis is more often made from the associated symptoms of


reflux
oesophagitis
B. The condition is usually a symptomatic
C. The symptoms correlate poorly with degree of oesophagitis
D. The dyspepsia and fullness after meals differs from the
manifestations
of peptic ulcer disease
E. Stooping or lying down do not affect the severity of the pain
F. The epigastric discomfort shows no recognised radiation patterns
G. Heart burn is experienced but there is no regurgitation of fluid or
aspiration simulating asthmatic attacks

 A woman of 55, five years postmenopausal, presents with


recurrent retrosternal and epigastric pains not affected by
posture, of three year duration. In recent months she has had
frequent asthmatic attacks especially at night and the pain has
tended to radiate between the shoulder blades and along the
side of the neck and the arms. Dysphagia has set in the past

7
month.

The most likely diagnosis is:

A. Angina from Ischaemic heart disease

B. Gallstone colic

C. Subacute pancreatitis

D. Sliding hiatal hernia with reflux

E. Penetrating Duodenal ulcer

 Myocardial infarction

 Subacute appendicitis

 Oesophageal carcinoma

 The most useful diagnostic test in this patient would be:

A. Plain x-ray of the abdomen


B. Barium swallow study
C. Upper GI Endoscopy
D. Oesophageal manametric studies and 24h pH monitoring
E. Sonographic studies

 The treatment of choice for this patient is:

A. Bed rest and a cardiac regime


B. Tripple Therapy for helicobacter eradication
C. Tripple Therapy for PUD with weight loss and postural adjustment
D. Esomeperazole 40mg bd for 3 months
E. Urgent laparoscopic fundopliation procedure

 The indications for surgical intervention in patients with sliding

8
hiatal
hernia include:

 Persistent symptoms of oesophagitis unabated by conservative


measures

 Where oesophageal spasm, fibrosis, ulceration and


haemorrhage supervene

 Mucosal histological changes

 Persistent combined refluxer

 Pulmonary symptoms are unresponsive to treatment

 The contra indications to anti-reflux procedures are:

 Air swallowing (aerophagy) mimicking reflux

 Motility disorders–poor peristalsis, achalasia

 Iatrogenic reflux due to surgical interference with cardia


competence mechanism

 Non dilatable or non-mobilizable strictures

 Presence of associated peptic ulcer disease

 The three criteria that must obtain to warrant surgical anti reflux

procedure are:

 (i) Oesophageal acid exposure (ii) LOS defect (iii) adequate


oesophageal contractility

 (i) Oesophageal acid exposure (ii) low mucosal PD (iii)


adequate
oesophageal contractility

 (i) Oesophageal acid exposure (ii) oesophageal stricture (iii)


adequate
oesophageal contractility

 (i) Normal oesophageal pH (ii) high mucosal P.D (iii) Normal

9
oesophageal
contractility

 (i) Increased oesophageal pH (ii) low mucosal P.D (iii) Adequate


oesophageal contractility

 In para-oesophageal hernia:

A. The oesophago-gastric junction is normally sited and competent

 The oesophago-gastric junction is displaced above the


diaphragm and competent

 The oesophago-gastric junction is normally placed and


incompetent

 The oesophago-gastric junction is displaced above the


diaphragm and incompetent

 The oesophago-gastric junction is normally placed and


incompetent,
and the hernia is confined to the lesser curvature

 In a para-oesophageal hiatal hernia:

A. There is a true sac disposed anteriorly which may contain


intestine

B. The is a true sac disposed posteriorly containing intestine

C. There is a sliding sac disposed posteriorly

D. There is a sliding sac disposed anteriorly containing intestine

E. There is no demonstrable sac

10
 The mixed hiatal hernia:

A. Is a sliding hernia with a super imposed para-oesophageal lesion

 Is a true hernia with a competent cardia in the normal position

 Is a sliding henia with the cardia in the normal position

 Usually has a competent cardia

 In terms of space occupied in the mediastinum is the smallest of


the
three hiatal hernias

 The manifestations of reflux and oesophagitis are:

A. Exceptional in patients with sliding hiatal hernia

B. Uncommon in patients with para-oesophageal hiatal hernia

C. Seldom occur in patients with mixed hiatal hernia

D. Are combined with dysphagia in para-oesophageal hernia

E. Are combined with haematemesis in patients with para-


oesophageal

hiatal nernia

11
31. A woman of 60 presents with epigastric pains radiating to the
retrosternal

area, made worse by eating, of six months duration. Over the


past month she has noticed dysphagia with bouts of
coughing hiccups and increasing difficulty in breathing and
palpitations:

The most likely diagnosis is:

A. Sliding hiatal hernia

B. Peptic ulcer disease – gastric ulcer

C. Dissecting aortic aneurysm

D. Large para-oesophageal hiatal hernia

E. Recurrent spontaneous pneumothorax from tuberculosis

32. The most useful diagnostic investigation is:

 Radiological imaging-plain and contrast x-ray

 Upper G.I. endoscopy

 Ultrasonography

 Manometric oesophageal studies

 Motility and pH oesophageal studies

12
33. The procedure of choice in the management of this patient is:

 Fair trial of medical treatment with proton pomp inhibitors –


esomeperazole

 Tripple therapy for elimination of helicobacter pylori

 Course of anti tuberculous therapy observing the tenets of D.O.T

 Emergency diagnostic laparotomy

 Surgical exploration and repair of a para-oesophageal hiatal


hernia

34. The goals of surgery in the sound repair of hiatus hernia


include:

A. Normalisation of the LOS presence – 12-20mmHg

 Restoration and fixation of adequate length of abdominal


oesophagus 3-5cm

 Restoration of the acute angle between oesophagus and


stomach

 Re-establishing the gastro oesophageal flap-valve mechanism

 Narrowing the hiatus

 Improving oesophageal peristalsis and gastric emptying

35. The most frequently used anti reflux procedure currently is:

A. Nissen fundoplication at laparotomy

B. Nissen laparoscopic fundoplication procedure

13
C. The Hill gastropexy – fundus to accurate ligament

D. Belsy two-third wrap of fundus on oesphagus by the thoracic


route

E. Angel chick silicone prosthesis

36. Which of the following diaphragmatic herniae usually does not


have

a true sac as ascertained at operation?

A. Postero-lateral (bochdalek’s) diaphragmatic hernia

B. Paraoesophageal (rolling) diaphragmatic hernia

C. Retrosternal or parasternal (Morgagni’s) diaphragmatic hernia

D. Traumatic diaphragmatic hernia

E. Sliding hiatal

37. A neonate presents within 2 h of birth with progressive


respiratory

distress and cyanosis with intercostal and subcostal suction.


On

examination the left side of the chest is tympanitic and the

14
abdomen

is flat, soft and unimpressive. A radiolucent shadow is seen on


the

hemithorax on X-rays.

The most likely diagnosis is:

 Congenital cystic disease of the lung

 Spontaneous pneumothorax

 Congenital postero-lateral (Bochdalek’s) diaphragmatic hernia

 Sliding hiatal hernia

 Retrosternal (Morgagni’s) hernia


38. The most useful confirmatory investigation is:

A. Lateral projection X-ray of the chest


B. Diagnostic pleural tap
C. Upper gastrointestinal barium series
D. Urgent arterial blood gas analysis
E. Bronchoscopy

39. You would advise:

A. Intercostal tube drain or thoracostomy


B. Intermittent bronchoscopic suction and administration of
Of oxygen
C. Urgent operative intervention by laparotomy
D. Tracheostomy and adequate ventilation
E. Thoracotomy

15
40. A child of four presents with shortness of breath on exertion,
dyspnoea

after meals and failure to thrive. On examination, cyanosis is


evident

soon after exertion and the left side of the chest is tympanitic.
Plain

X-ray of the chest shows a cystic shadow in the left hemithorax.

The most likely diagnosis is:

A. Congenital postero-lateral diaphragmatic hernia (stabilized)


B. Congenital cystic lung disease
C. Ephysematous bullae+
D. Congenital cyanotic heart disease
E. Fibrocystic disease of the lung

41. You would advise:

A. Thoracostomy and under water seal drainage


B. Thoracostomy and exploration of the left hemithorax
C. Laparotomy, exploration for defects and repair
D. Tracheostomy and intermittent positive pressure ventilation

16
E. Thoraco-abdominal exploration

42. Postero-lateral Diaphragmatic hernia is:

A. An acquired diaphragmatic hernia, the result of chest trauma


B. An extension laterally of a hiatal hernia
C. A congenital anomaly resulting from failure of the pleuro-
peritoneal
membrane to join the septum transversum
D. A congenital defect in the septum transversum
E. The result of degeneration of the lateral plate mesoderm

43. A neonate presents two hours after birth with acute respiratory
distress, tachypnoea, grunting and cyanosis. On examination
there is a mediastinal shift to the right with typanitic note on
percussion on the left. The abdomen is unremarkable.

The most likely diagnosis is:

A. Dextrocardia with congenital heart disease


B. Spontaneous pneumothorax
C. Congenital cystic disease of the lung
D. Postero-lateral diaphragmatic hernia (Bochdalek’s)

E. Hyaline membrane disease

44. The most useful diagnostic investigation in question 43 is:

A. Plain x-ray of the abdomen, P. A and Lateral


B. Plain x-ray of the chest, P. A and Lateral
C. Abdominal ultrasound

17
D. Ultrasonic study of the chest
E. Xenon radio scan of the chest and abdomen

45. A young boy of six presents with shortness of breath on


exertion, dyspnoea after meals and failure to thrive of several
months duration. On examination there is a mediastinal shift
to the right, but the chest moves symmetrically and there is
increased resonance on the left with diminished breath sands.
There are no added breath sounds.

The most likely diagnosis is:

A. Dextrocardia with congenital heart disease


B. Spontaneous pneumothorax (Left)
          C. Congenital cystic disease of the lung
D. Posteral-lateral Diaphragmatic hernia (Bochdalek’s) (Lt.)
E. Diaphragmatic tumour (Lt.) with pleural effusion

46. The most useful diagnostic investigation in question 45 is:

18
A. Plain x-ray of chest P.A and Lateral

B. Plain x-ray of Abdomen P.A and Lateral

C. Barium meal study

D. Abdominal ultrasound

E. Xenon radio scan of the chest

47. The treatment for patients in Questions 43 and 45 is:

A. Extra corporal membrane oxygenation (ECMO)


B. Delayed surgical exploration and repair
C. Extra corporal membrane oxygenation prior to surgical repair
D. Intermittent positive pressure ventilation
E. Exploratory thoracotomy of the left thoracic cavity

48. A woman of 45 presents with a dragging discomfort in the left


hypochondrium of several years duration; this has on
occasions been accompanied by left shoulder tip pain. She has
also been experiencing bouts of constipation and flatulence;
there have been no colicky abdominal pains. Examination
yielded no signs but a chest x-ray showed a rounded discrete
shadow adjacent to the left side of the pericardium.

The most likely diagnosis is:

19
A. Pericardial tumour
B. Sliding hiatal hernia
C. Tumour of the diaphragm
D. A left parasternal hernia (Morgagni hernia)
E. Chronic pancreatitis

49. The most useful diagnostic investigation is:

A. Plain x-ray of the chest, P.A and Lateral


B. Plain x-rays of the abdomen, P.A. and Lateral
C. Abdominal ultrasound
D. A barium enema examination
E. Colonoscopy

50. A 40 year old professional driver presents with several months


history of post prandial retrosternal pain, bloating and
flatulence. Two years earlier he had been involved in a head
on collision accident necessitating admission to hospital for
three weeks, after which he had been discharged with a clean
bill of health. On examination his chest was clear and moved
normally. The only abnormal finding was variable dullness
alternating with tympanicity on percussion on the left side of
the chest. The abdomen was unimpressive.

The most likely diagnosis is:

A. Chronic gastric ulcer

B. Chronic cholecystitis

C. Myocardial infarction (posterior)

D. Dissecting aortic aneurysm (post-traumatic)

E. Acquired diaphragmatic hernia

20
51. The most useful diagnostic investigation is:

A. Plain x-ray of the chest, P.A and Lateral


B. Plain x-ray of the Abdomen P.A. and Lateral
C. Abdominal ultrasound
D. Upper GI Endoscopy
E. Barium meal study

52. The treatment of choice in patient in question 50 is:

A. Full implementation of a trial of medical treatment for gastric ulcer


B. Offer of cholecystectomy after verification by ultrasound
C. Laparotomy with a view to peptic ulcer surgery
D. Operative intervention by the trans thoracic route
E. Trial of coronary artery thrombosis regime

53. Which of the following may be encountered as complications of

traumatic diaphragmatic hernia?

A. Simple intestinal obstruction

B. Strangulating intestinal obstruction

C. Gastric ulcer

D. Duodenal ulcer

E. Volvulus of the stomach

54. Singultus (hiccup) may be caused by:

A. Meningitis

B. Encephalitis

21
C. Cerebral haemorrhage

D. Cerebral tumour

E. Pleurisy

55. Singultus (hiccup) may be caused by:

A. Pericarditis

B. Aortic aneurysm

C. Subphrenic abscess

D. Haemoperitoneum

E. Mediastinal tumours

56. Which of the following procedures may arrest an attack of

singultus (hiccups)?

 The vasalva manoeuvre

 Pressure on the sides of the neck

22
 Rebreathing in a paper bag

 Administration of CO2 5% O2 95% mixture

 Administration of N2 5% O2 95% mixture

57. The muscular partition called the diaphragm:

A. Arises from a peripheral muscular band distributed in three

groups-lumbar, costal and sternal

B. Has a centrally placed trifoliate tendon made up of interlacing


fibres

C. Has two domes which rise in full expiration as high as the 4th

dorsal intervertebral disc right and the 5th on the left

D. Has three major foramina transmitting the aorta, oesophagus

and the inferior vena cava

E. Is supplied with sensory fibres by the lower six intercostals nerves

58. The aortic foramen of the diaphragm:

A. Is actually a retrodiaphragmatic opening

B. Is at the level of the 12th thoracic vertebra

C. Is bounded on either side by the crura

D. Is the lowest of the three major openings through the partition

E. Has the oesophageal opening in front and to the left of it at

a higher level and about 2.5cm from the midline

23
59. Hiatus herniae:

A. By definition occur through the oesophageal opening of the

diaphragm only

B. Account for 98% of diaphragmatic herniae

C. Are reported by numerous publications emanating from the

African continent as rare among negroes

D. Do not at the moment present a major clinical problem in

developing countries

E. Are usually of the sliding type (85%) and rarely of the para-

oesophageal (10%) or mixed type (5%)

60. Sliding hiatus hernia:

A. Is usually due to upward displacement of the abdominal


oesophagus and upper stomach through a lax hiatus
B. Has a predilection for the obese and the elderly
C. Presenting with dyspepsia is clinically indistinguishable from
other dyspepsias, e.g. peptic ulceration
D. May be demonstrated on barium swallow in 95% of cases

24
E. If asymptomatic requires no treatment

61. Sliding hiatus hernia:

A. Is often associated with persistent or intermittent excessive

contraction of oesophageal longitudinal muscle

B. Usually presents in people over 50 years of age

C. May be associated with an oesophagus which is only

apparently foreshortened

D. Can be diagnosed by means of pressure measurements in 70%


cases

E. Should be actively treated in all cases when picked up to forestall

Complications

25
62. Sliding hiatus hernia:

A. Is frequently associated with cardial incompletence

B. Is slightly more common in women

C. May show on plain postero-anterior chest views as a gas filled

shadow superimposed on the cardiac shadow

D. If associated with reflux may be diagnosed in 97 per cent of cases

by pH studies

E. Should in all symptomatic cases be treated initially by


conservative means

63. In para-oesophageal hiatus hernia:

A. The oesophago-gastric junction is normally sited and competent

B. The fundus of stomach rolls up into the chest in front of the

oesophagus

C. A sliding hernia may later develop thus rendering the cardia

again incompetent

D. Dysphagia is a prominent symptom

E. Radiological and endoscopic examination provide useful


information

on the position, nature and extent of the lesion

64. In para-oesophageal hiatus hernia:

A. The oesophago-gastric junction is displaced above the diaphragm

and incompetent

26
B. The fundal bulge is usually into the left pleural cavity

C. Symptoms arise because of the large space occupying lesion

in the mediastinum

D. Anaemia is a notable feature

E. Surgical treatment is indicated in all symptomatic cases

65. Surgical procedures to repair hiatus hernia:

A. May be carried out through the chest or abdomen

B. Should ensure that the oesophageal hiatus is optimally narrowed

C. Should aim at re-establishment of an infra-diaphragmatic segment

of oesophagus as the most important feature

27
D. Should endeavour to restore the acute angle between oesophagus
and stomach

E. Should ensure that the stomach is hitched on the diaphragm

66. Postero-lateral (Bochdalek’s) diaphragmatic hernia:

A. Results from congenital defect through the pleura-peritoneal

canal in the posterior aspect of the diaphragm

B. Occurs more commonly on the left than right

C. Most commonly presents as the cause of acute respiratory distress

in the new born child

D. Should be treated surgically without undue delay even in the


apparently

stabilized infant

67. Retrosternal or parasternal (Morgagnl’s) hernia:

A. Occurs through the potential defect between the costal and sternal
fibres

of the diaphragm

B. Seldom gives rise to symptoms and rarely presents in childhood

C. May be noted in mass radiology surveys in adults

D. May occasionally present with intestinal obstruction

E. Is best treated operatively

68. Eventration of the diaphragm:

28
A. Results from partial or complete failure of the embryonal cervical

myotome contribution to the diaphragm

B. Makes for a higher position of the affected hemidiaphragm

C. Cannot usually be distinguished clinically and radiologically from a

diaphragmatic hernia

D. Is usually asymptomatic

E. Usually neither needs nor requires repair

69. Traumatic diaphragmatic herniae:

A. Are produced by direct or indirect violence

B. Usually occur in the left tendinous portion

C. Are often complicated by haemothorax which obscures herniated

viscera on physical or radiological examination

D. Usually have no true sac

E. Should be treated by operative repair to avoid complications

29
30
CHAPTER 25
DIAPHRAGM
ANSWERS

1. C 38. A
2. A 39. C
3. A 40. A
4. B 41. C
5. B 42. C
6. D 43. D

7. D 44. B
8. B 45. D

9. A, B, C, D, E, F, G, H 46. A

10. D 47. C
11. D 48. D
12. A, B, C 49. D
13. D 50. E
14. A, B, C 51. E
15. B 52. D
16. A 53. A, B, E

17. A 54. A, B, C, D, E

18. D 55. A, B, C, D, E

19. A 56. A, B, C, D

20. B, C 57. A, B, C, D, E

21. D 58. A, B, C, D, E

22. C 59. A, B, C, D, E
23. C 60. A, B, C, D, E

24. A, B, C, D, E 61. A, B, C, D

31
25. A, B, C, D, 62. A, B, C, D, E

26. A 63. A, B, C, D, E
27. A 64. B, C, E
28. A 65. A, B, C, D
29. A 66. A, B, C, D, E
30. B 67. A, B, D, D, E
31. D 68. A, B, C, D, E
32. A 69. A, B, C, D, E
33. E
34. A, B, C, D, E, F
35. B
36. A
37. C

32
CHAPTER 28
THE BREAST
 The average normal breast overlies the:
 2nd – 7th ribs
 Pertoralis major
 Serratus anterior
 Upper part of rectus sheath
 Intercostal muscles
 In the anatomy of the breast
 The structural unit is the lobule
 There are 20-25 lobes each formed by the coalescence of groups of
lobules
 Lactiferous ducts, each draining a lobe, open separately and
radially on the summit of the nipple
 The bulk of the areola and nipple is made up of contractile smooth
muscle fibres inserted directly to the skin
 The skin of the nipple and areola is thin, sensitive and contains
large sebaceous glands and hair follicles
 In the lymphatic drainage of the breast:
 The internal mammary chain of lymph nodes drains about 20% of
lymph mainly from the medial aspect
 The axillary group of lymph nodes drains about 50% of lymph
mainly from the lateral side
 Some lymph drains to the opposite breast and axilla and the liver
 Malignant cells necessarily metastasize to lymph nodes draining
the corresponding breast quadrant
 Retrograde tumour emboli are the rule when a forward system of
lymphatics is blocked.
 Increase in the size and number of the mammary lobules occurs
in:
 Puberty
 Menstruation
 Pregnancy
 Lactation
 Normal growth
 The normal breast depends on the following for its full
development and growth:

 Cortisone
 Prolactin
 Oxytocin

1
 Oestrogens and progesterone
 Growth hormone

 The breast depends on the following for milk production:


 Cortisol
 Follicle stimulating hormone
 Growth hormone
 Oestrogens and progesterone
 Prolactin
 The most important factor in the maintenance of lactation is:
 Oxytocin
 Infant suckling
 Progesterone
 Growth hormone
 Prolactin
 In anomalies of the breast:
 Polymazia is an accessory nipple
 Polymazia may co-exist with polythelia
 Polymazia and polythelia usually undergo the same physiological
changes as the normal breast
 Polymazia may be confused with lipoma
 Polythelia may be confused with melanoma
 Unilateral acute mastitis:
 Results from cracks on the nipple due to trauma of infant suckling
 Is most commonly caused by Esch. Coli
 Quickly resolves on appropriate antibiotic therapy
 If inadequately treated may lead to chronic breast abscess without
overt manifestation of its inflammatory origin
 Requires suppression of lactation
 Failure of the normal infant breast to develop may be due to:
 Congenital adrenal hyperplasia
 Basophil adenoma of the anterior pituitary
 Ovarian dysgenesis
 Turner’s syndrome
 Hyperthyroidism
 The commonest cause of precocious breast development is:
 Albright’s syndome
 Lutein cyst
 Granulosa cell tumour

2
 Adrenal tumour
 Constitution or genetic predisposition

 Massive breast hypertrophy in pregnancy:


 Usually commences in the 3rd trimester
 May have a neuro-endocrine aetiology
 Is often associated with a pre-existing breast dysplasia
 Normally regresses after parturition
 Usually requires surgical reduction of breast size
 Benign mammary dysplasia is:
 Characterized by local tissue mass with cysts containing clear or
bluish-green fluid
 Probably an abnormal ‘target’ response to F.S.H. stimulation
 Common between 20 and 45 with the highest incidence between
20-25
 More common in multiparous women
 Precancerous
 The microscopical characteristics of benign mammary dysplasia
include:
 Lymphocytic infiltration
 Increased stromal proliferation
 Ductal epithelial hyperplasia
 Glandular hyperplasia
 Vascular estasia
 A patient with benign breast dysplasia may present with:
 Persistent breast pain during and after menstruation
 Axillary lymphadenopathy
 Irregular menstruation
 Oligomenorrhoea
 Serous or greenish nipple discharge
 A breast lump due to benign breast dysplasia:
Is firm in consistency
Has a definite edge
Is partly attached to skin
Is not attached to underlying muscle
Has nodular surface which is more easily felt with the tips of the
fingers
 Measures that may be taken in the management of benign
mammary dysplasia include:

3
 Reassurance if the whole or both breast are affected
 Excision biopsy for definite localized mass
 Danol orally for 2 months
 Simple mastectomy in those over 40years
 Aspiration of cysts

 Duct ectasia:
 Is dilatation of the lactiferous ducts
 Occurs between 25 and 70 but especially between 25 and 35
 Is seen equally in all parts of the breast
 Results from obliteration from fibrosis and ageing of the periacinar
and perilobular lymphatics
 Leads to the accumulation of yellow or brown cheesy material in the
ducts which can be expressed like toothpaste
 Structures seen in the microscopic picture of duct ectasia
include:
 Plasma cells
 Lymphocytes
 Fibrous tissue
 Lipid material
 Giant cells
 Duct ectasia may present as:
 Retraction of the nipple
 Turbid yellow, green, brown or black nipple discharge
 Mammillary fistula
 Areolar swelling with attachment of skin, surrounding tissue and
axillary
lymphadenopathy
 Subareolar abscess
 Measures taken in the management of duct ectasia include:
 Reassurance
 Observation
 Excision biopsy
 Simple mastectomy
 Oestrogen and antibiotic therapy
 In traumatic fat necrosis:
 The patients are obese
 There is in most patients a history of trauma to the breast
 Saponification of fat by blood and tissue lipase causes a foreign
body inflammation

4
 There is fibrous tissue with spaces containing liquefied fat, fatty acid
crystals and chalk-like material in the centre
 Irritation of cells may precipitate cancer formation
 A lump in traumatic fat necrosis:
 Is soft to firm
 May be attached to skin and has irregular surface
 Has indefinite edges
 Is usually attached to the underlying muscle
 May cause retraction of the nipple

 A breast cyst:
 May contain a papillary carcinoma in its wall
 May be caused by benign mammary dysplasia or traumatic fat
necrosis
 May occasionally be caused by intraduct carcinoma
 Occurs in the lower outer quadrant in most patients
 Is best excised
 A cyst of the breast should be excised if after aspiration:
 The lump does not disappear completely
 The fluid obtained is greenish or brown
 The lump reappears within 2-4 weeks
 There is pain in the breast
 Other cysts appear
 Measures that may be taken in the management of galactocele
include:
 Cold compress
 Massaging of the lump
 Aspiration
 Stopping feeding on the affected breast
 Excision biopsy initially to exclude malignancy
 The clinical signs of a chronic breast abscess include:
 Hard consistency
 Tenderness
 Indefinite edge
 Peau d’orange
 Enlarged axillary lymph nodes
 Pericanalicular fibroadenoma
 Occurs in the age group 14-45 with a peak around 30

5
 Microscopically consists of an overgrowth of fibrous, acinar and
ductal epithelial structure
 Is usually 1-10cm in diameter
 Is clinically firm, not attached to skin or underlying muscle but the
edge may be indefinite
 May be multiple
 Intracanalicular fibroadenoma:
 Usually occurs in the age group 30 to 50
 Is commoner than the percanalicular type
 When cut across looks like cysts enclosing cauliflower-like masses
 Microscopically shows delicate proliferating connective tissue most
dense around the ducts
 Rarely becomes sarcomatous

 Intracanalicular fibroadenoma:
 May be up to about 10cm in diameter and occupy most of the
breast
 Is soft
 Has nodular surface
 Is mobile
 Is well encapsulated and not attached to skin, surrounding breast
tissue or underlying muscle
 Cystosarcoma phylloides:
 Is pathologically different from fibroadenoma
 Grows slowly but attains a large size, up to 30cm in some patients
 Is multicystic with papillary growths extending into the cysts
 Does not become malignant
 Is seen usually in women between 20 and 30
 In cystosarcoma phylloides:
 The subcutaneous veins are distended and the skin stretched over
it
 The skin may ulcerate
 The edges are definite and there is no attachment to underlying
muscle, surrounding tissues or skin
 The surface is lobulated and solid and cystic areas can be felt
 The treatment is simple excision
 Duct papilloma:
A Arises from the epithelium of an acinus
B. May become malignant
C. Presents commonly with recurrent bleeding from the nipple or a
swelling

6
D. Is usually seen in patients between 20 and 50
E. Is best treated by simple mastectomy
 Epidemiologically, the risk in a woman developing carcinoma of
the breast is increased by:

 Cancer of the breast in the mother, aunt or sister


 Menarche at 15 and menopause at 45
 First pregnancy at 18
 Nulliparity
 Cancer of the corpus uteri or ovary

 Epidemiologically, the risk in a woman developing carcinoma of


the breast is increased by:

 Benign mammary dysplasia


 Previous breast cancer
 Low fat intake
 Bilateral oophorectomy by 35 years
 Rauwolfia compounds in hypertensives over 50 years
 The commonest type of carcinoma of the breast is the:
 Medullary carcinoma
 Intraductal comedo-carcinoma
 Diffuse infiltrating (scirrhous) carcinoma
 Intraduct carcinoma
 Paget’s disease of the nipple
 Diffuse infiltrating (scirrhous) carcinoma of the breast:
Is usually large in size 7-10cm in diameter
Is hard with irregular edges
Is tough and gritty to cut
Has a uniformly grey surface when cut
Contains spheroidal or polygonal tumour cells and dense hyaline
acellular fibrous tissue stroma
 Medullary (encephaloid) carcinoma of the breast:
 Is large and soft and appears encapsulated
 When cut has a grey surface with area of haemorrhage and
necrosis

7
 Is very cellular with little stroma
 Is densely infiltrated with lymphocytes and plasma cells
 Has a poor prognosis
 In EARLY carcinoma of the breast, the lump:
 Is hard and small
 Has definite edges
 May be attached to the skin
 Is not attached to the underlying muscle
 Is associated with retracted nipple
 In Paget’s disease of the breast:
 There is eczema-like excoriation of the areola and nipple
 The nipple may be eroded
 There is usually a palpable lump in the breast
 The nipple is retracted
 Axillary lymph nodes are usually palpable

“A 50year old woman presented with a swelling, 2cm in diameter, in the upper
lateral quadrant of the right breast. There was no attachment to skin or
underlying muscle and the nipple was not retracted. A solitary axillary lymph
node was palpable. Extended simple mastectomy was done following a core
biopsy. The axillary nodes were histologically free of metastases. No distant
metastases were detected.

 The clinical stage is:


 TINOMO
 T2NIMO
 TINIMO
 State 2
 Stage 1
A tumour 3cm in diameter in the lower lateral quadrant of the right
breast with attachment to the skin less than the diameter of the
tumour but not to the underlying muscle, has associated 2 firm
mobile lymph nodes proven on histology to be affected . There
are no distant metastases.
 The clinical stage is:
 T2(b) NI (b) MO
 T2(a) NI (b) MO
 T2(b) NI (b) MO
 T4(b) NI (b) MO
 T4(a) NI (b) MO
 The Clinical stage (Manchester) is:

8
 Stage 1
 Stage 2
 Stage 3
 Stage 4
 Investigations to be done in clinically established early carcinoma
of the breast where all facilities exist include:

 X’ray of the chest


 Skeletal scintigraphy
 Mammography
 Plain X-ray of the skeleton
 Thermography
 Carcinoma of the breast may present as:
 Creamy or milky nipple discharge
 Axillary lymphadenopathy without breast swelling
 Backache
 Hemiplegia
 Splenomegaly

 Paget’s disease of the breast without a demonstrable breast lump,


enlarged axillary lymph nodes or distant metastasis is:
 TO NO MO
 TIS NO MO
 TI NO MO
 Stage 2
 Stage 1
 Problems that have made treatment of carcinoma of the breast
difficult and deductions from most clinical trials inaccurate
include:
 About 40% of tumours are biologically fast-growing with early nodal
and systemic spread
 In clinical stage 1 systemic spread would have taken place in about
25%
 In 30% of clinically impalpable axillary nodes, metastases are
present
 In 40% of clinically palpable axillary nodes there is no metastasis
 Staging of medially or centrally sited tumour is inaccurate because
they do not usually metastasize to axillary lymph nodes
 Structures removed in radical mastectomy include:
 Breast
 Pectoralis minor

9
 Sternal part of pectoralis major
 Clavicular part of pectoralis major
 Axillary lymph nodes and fat
 Structures removed in extended simple mastectomy include:
 Breast
 Pectoralis minor
 Sternal portion of pectoralis major
 Clavicular portion of pectoralis major
 Axillary lymph nodes and fat

 Ulcerating stage 4 carcinoma of the breast may in West Africa be


treated by:
 Hormonal therapy
 Cystotoxic chemotherapy
 Simple mastectomy
 Hormonal therapy plus simple mastectomy
 Cytotoxic chemotherapy plus simple mastectomy

 In the mechanism of action of the hormones in cancer of the


breast:

 Androgens directly inhibit oestogen and progesterone secretion by


the ovaries
 Oestrogens probably directly inhibit tumour growth
 Small doses of oestrogens may stimulate tumour growth
 Tamoxifen and nafoxidine bind the oestrogen receptors and there-
by
in hibit oestrogen uptake by tumour cells
 Corticoids bind the prolactin receptors and thereby inhibit prolactin
uptake by the tumour cells
 A premonopausal West African patient with Stage 4 carcinoma of
the breast responded well to bilateral oophorectomy for 4 years.
She now has a local recurrence. The following may be advised:
 Adrenalectomy
 Large doses of cortisone
 Androgens
 Tamoxifen or nafoxidine
 Pituitary ablation

10
 In the response of carcinoma of the breast to hormonal therapy:
 Response of patients with primary tumours with oestrogen or
progesterone receptors is 60%
 The shorter the interval between treatment of the primary disease
and the appearance of metastases the better the response.
 Visceral deposits are more likely to regress than osseous deposits
 Premenopausal patients respond best and menopausal patients
worst
 A patient who has not responded to the first line of hormone therapy
is more likely to response to the second line
 Radiotherapy in the treatment of carcinoma of the breast:
 Is of proven value in early cases and improves the overall survival
rates
 Reduces the incidence of local recurrence and has same effect on
subsequent development of distant metastases
 Alleviates pain in advanced cases or local recurrence or bony
deposits
 Is beneficial for treating fungating or ulcerating tumour
 Does not depress immunity
 Substances being tried or used for non-specific immunotherapy
in cancer include:
 B.C.G. vaccine
 Corynebacterium parvum
 Autograft of irradiated cancer cells
 Levamisole
 Inteferon

 The most important prognostic index in cancer of the breast is


the:
 Menstrual status
 Presence or absence of axillary lymph nodes
 Site of the tumour in the breast
 Hormone dependence
 Size of the tumour at the time of diagnosis
 Multiple, circumscribed, firm or hard lumps with rather smooth
surface in both breasts of a 20-year old woman may be:

 Multiple fibroadenomata
 Carcinomatous deposits
 Fibroadenosis
 Burkitt’s lymphoma

11
 Lymphoma
 A nipple discharge which is clear, creamy, green, brown or black
without red cells is associated with:

 Duct estasia
 Duct papilloma
 Benign mammary dysplasia
 Galactorrhoea
 Medullary carcinoma
 A bloody nipple discharge may be caused by:
 Duct estasia
 Intraductal comedo-carcinoma
 Breast abscess
 Scirrhous carcinoma
 Paget’s disease
 A purulent nipple discharge may be caused by:
 Benign mammary dysplasia
 Breast abscess
 Traumatic fat necrosis
 Duct ectasia
 Leiomyosarcoma

 In the management of a patient with a nipple discharge without an


associated lump:

 The discharge should always be examined for red and pus cells
and cancer cells and microorganisms
 If the discharge is blood-stained, the patients should be followed up
monthly and a biopsy done if it does not stop in 3 months
 If the discharge is not blood-stained and it does not stop
spontaneously, then a biopsy should be done.
 If the patient is less than 40 and the discharge is not blood-stained
but persists, a biopsy is not essential
 A course of antibiotics is beneficial
 A 4 cm non-tender soft, rather fluctuant, lobulated mobile swelling
with definite edges and no attachment to skin or muscle in a

12
normal breast in a 45-year old woman is mostly likely:

 Early carcinoma
 Chronic abscess
 Galactocele
 Cyst of benign mammary dysplasia
 Lipoma
 A firm non-tender discrete 3 cm lump with indefinite edges and no
attachment of skin or pectoralis muscle nor associated retraction
of the nipple in the ‘lumpy’ breast of a 45-year old woman may be:

 Benign mammary dysplasia


 Firbroadenoma
 Carcinoma
 Duct ectasia
 Chronic abscess
 Gynaecomastia:
 Is feminine enlargement of the male breast due to hypertrophy of
mammary tissue
 Causes dilated and tortuous ducts and there may be milky nipple
discharge
 In boys at puberty may be associated with enlargement of the penis
 In boys at puberty may be associated with impotence
 In adults does not require operative treatment

 Gynaecomastia may be caused:


 Choriocarcinoma
 Growth hormone
 Prepubertal orchitis
 Thyrotoxicosis
 Sickle cell disease
 Gynaecomastia may be caused by:
 Reserpine
 Marihuana
 Digitalis
 Diazepem

13
 Methyldopa
 The commonest cause of swelling in the adult male breast is:
 Gynaecomastia
 Firboadenosis
 Lipoma
 Sebaceous cyst
 Carcinoma
 Carcinoma of the male breast:
 Accounts for about 1% of all male cancers and about 2% of all
breast cancers
 May present with serous or serosanguinous nipple discharge
 Is best treated by mastectomy followed by irradiation if the axillary
lymph nodes are involved.
 If advanced always responds to orchidectomy or oestrogens
 Has a worse prognosis than in women.
 In Carcinoma of the breast in West Africa:
 Most patients are between the ages of 35 and 50
 The risk of a breast lump being malignant is higher in a 66 year old
than a
36 year old woman
 About 65% have advanced disease
 Tamoxifen will be effective adjuvant treatment in the majority of
patients
 The incidence is rare below age 20
 Cancer of the breast:
 Is estimated to affect about 1 million women worldwide annually
 Affects 1 in 10 women in Africa their lifetime
 Is second only to cervical cancer in incidence worldwide
 Accounts for about 15% of all cancers in Accra
 Has a high incidence in Japan

The histology report following right mastectomy in a 30 year old


woman showed the following: Breast specimen contained a mass
measuring 34 x 52mm in diameter in the upper inner quadrant of
the breast. Infiltrating ductal carcinoma with no insitu
component. Grade 1. No vascular or perineural invasion. Axillary
fat contained 14 lymph nodes, with tumour deposits in 3 but with
extracapsular spread. Estrogen receptors (20% positive).
Progesterone receptors (strongly positive). HER 2 (1+)

 In this patient:

14
 The pathological staging is T1N2Mx
 The tumour is poorly differentiated
 Chemotherapy will be useful
 A FISH test is necessary
 Hormonal therapy using Anastrozole is indicated
 In the treatment of this patient:

 Radiotherapy to the chest wall is indicated


 Radiotherapy to the supraclavicular nodes in indicated
 Radiotherapy to the internal mammary nodes is indicated
 Hormonal therapy using Anastrozole is indicated
 Herceptin is indicated
 A woman of 42 presents with a hard 2cm lump in the upper outer
quadrant of the left breast and mobile nodes in the axilla:

 A clinical diagnosis of Breast Cancer T1N1Mx is correct

 An excision biopsy is preferable for diagnosis

 A mammogram is indicated before a diagnostic biopsy


 Attachment of the mass to the pectoralis muscle makes it Clinically
T4a
 Tethering does not change the clinical staging
 Associated nipple retraction worsens the prognosis

 A woman of 42 presents with a hard 2cm lump in the upper outer


quadrant of the left breast and mobile nodes in the axilla. If the
lump is found to be malignant:

 Surgical options include mastectomy

 Surgical options include wide local excision

 Neoadjuvant chemotherapy is indicated

 Surgical options include Level III axillary clearance

 There is a greater risk of local recurrence if she is treated by wide


local excision and radiotherapy instead of mastectomy

15
 In the hormonal treatment of carcinoma of the breast:

A. Oophorectomy is an option in postmenopausal patients

 Tamoxifen may be given to premenopausal patients

 A patient who has had her ovaries removed will further benefit from
the Aromatase inhibitor anastrozol

 Tamoxifen treatment lasts for 60 months

 It is best given concurrently with Chemotherapy in the adjuvant


setting
 Concerning prognosis in Carcinoma of the breast:

 Most recurrences occur within the first year, but some occur as long
as 15-20
years after treatment:

 The number of axillary lymph nodes involved is directly related to


poor prognosis
 ER-, PR- and HER2- tumours have worse prognosis than ER-, PR-
and HER + tumours
 Young patients (<30) live longer than older patients (>50)

 Radiotherapy to involved internal mammary nodes does not


improve survival
 Concerning prognosis of Breast cancer:

 Basal tumours have worse prognosis than Luminal A tumours

 Patients with BRCA gene mutation have a better prognosis than


sporadic cancers
 Ki67 and vascular invasion signify poorer prognosis

 For the histological type, stage and age of patient, Caucasians and
Blacks have the same prognosis

 Immunohistochemistry results have a significant bearing on


prognosis
 In the diagnosis of breast cancer:

16
A. Mammography is important even when the diagnosis is ‘obvious’

 Skin Incision biopsy is indicated in Inflammatory cancer when there


is no definite lump
 At least 10 cores must be taken during core biopsy

 Digital mammography and MRI may be used for a woman of 30

 Biopsy is indicated even if clinical and radiological assessment are


normal

 In the examination of a breast lump:


A. Clear bilateral nipple discharge from many ducts is of serious
significance
 The findings of a linear cord-like structure in indicative of Mondor’s
disease
 Attachment of the lump to the chest wall can be demonstrated by
immobility of the lump when the hands are pressed firmly on the
waist
 A tender mass arising from the skin is likely to be benign

 Findings of a hard irregular mass with peau d’orange of overling


skin, ill-defined edges and axillary lymph nodes may be present in a
benign lesion.
 In a patient receiving Cyclophosphamide, Adriamycin and 5-
Fluorouracil chemotherapy:

 The drugs are typically administered every 4 weeks

 Cardiotoxicity from Cyclophosphamide may occur if the patient has


pre-existing cardiac disease

 When given as neoadjuvant therapy, the patient is reviewed after 2


- 3 cycles because the most serious complications are more likely

17
to occur at this time
 Fever with an absolute neutrophil count (ANC) of 0.9 is an
oncological emergency
 50% dose reduction for the subsequent cycles is indicated if the
patient has an ANC of 0.8 on the 10th day following chemotherapy.
 Features that suggest malignancy in a patient with a breast lump
are:

A. Multiplicity of lumps in other quadrants, on examination


B. Associated bilateral duct nipple discharge
C. A large lump measuring 10cm
D. Associated ipsilateral axillary nodes measuring 1.5cm
E. Significant weight loss in the absence of other systemic symptoms
and signs
 The following are true about the spread of breast cancer:

 Local spread to the skin is responsible for tethering

 Haematogenous spread may lead to pleural effusion

 Direct spread to the nipple causes nipple retraction

 Spread to the thoracic vertebra is an oncological emergency

 Spread to the lungs is an oncological emergency.

CHAPTER 28
THE BREAST

 B, C, D E 36. C 71. C

 C, D 37. B, C, E 72. A, B

 A, C, E 38. A, B, C, D 73. A, C,

18
E

 C 39. A, D 74. A, B

 B, D, E 40. A, B 75. B, C,
D

 A, C E 41. C, E 76. A, B,
C, E

 B 42. D 77. A, C,
E

 B, D, E 43. B 78. A, B,
D

 A, C, D 44. A, B, C 79. B, D,
E

 A, C, D 45. B, C, D 80. D

 E 46. B, E 81. D

 C, E 47. B, C, D 82. B, D

 A 48. A, B, C, E

 B, C, D 49. A, E

 E 50. D, E

 A, D, E 51. B, C, D

 A, B, C, E 52. B, D

 A, D, E 53. A

 A, B, C, D, E 54. C, D

 A, B, C, D, E 55. A, B, D, E

 C 56. B

 C, D 57. A, D, E

 B, C 58. A, C

19
 A, C, E 59. B

 A, C 60. B, D

 B, C 61. C

 A, B, C, D, E 62. E

 B, E 63. A, C, D, E

 A, C, E 64. B, C

 A, B, D, E 65. A, B, C, D

 B, C 66. A, B, C, D, E

 A, B, C, D 67. A

 B, C 68. B, C, E

 A, D, E 69. A, B, C, E

 B, E 70. A, D

20
CHAPTER 29

THE UMBILICUS AND ANTERIOR ABDOMINAL WALL


(EXCLUDING EXTERNAL HERNIAE)

 Infection of the umbilical cord may be caused by:


 Staph aureus
 Esch coli
 Strep. Haemolyticus
 Cl. Tetani
 Cl. Welchii

 Complications of neonatal omphalitis include:


 Cirrhosis of the liver
 Portal vein thrombosis and portal hypertension
 Umbilical sinus or adenoma
 Peritonitis
 Abscess of the anterior abdominal wall

 Umbilical calculus may contain:


 Uric acid crystals
 Dirt
 Calcium oxalate
 Calcium phosphate
 Desquamated epithelium

 A congenital umbilical fistula is usually between the umbilicus and the:


 Ileum
 Jejunum
 Bladder
 Transverse colon
 Stomach

 The urachal cyst:


 Is derived from the allantois
 Is formed when the umbilical end of the urachus is obliterated and the
vesical end remains patient
 Lies in the midline
 Is intraperitoneal
 May become infected and form recurrent abscess

 The following incision(s) can be extended without ill effects:


 Paramedian
 Pararectal (Battle’s)
 Gridiron (McBurney’s)

1
 Oblique
 Upper transverse

 The scar from the following incision(s) tends to be weak:


 Lanz
 Subcostal (Kocher’s)
 Lower median
 Paramedian
 Pararectal (Battle’s)

 Upper median(Midline) incision:


 Divides skin, rectus sheath, rectus muscle, and peritoneum
 Is almost bloodless
 Provides quick access to the stomach and duodenum
 Is readily infected
 Is liable to incisional herniation

 In a paramedian incision the:


 Incision is vertical and about 3cm from the umbilicus
 Anterior rectus sheath is incised and dissected from the rectus muscle in
the lateral direction
 Rectus muscle is displaced medially
 Posterior rectus sheath is incised with the attached peritoneum
 Nerves to the rectus are divided

 The pararectal (Battle’s) incision:


 Is medial to the semilunaris at right angle to the spino-umbilical line at the
junction between the lateral and middle thirds
 Requires the rectus muscle to be retracted medially
 Provides rapid access to the appendix
 Damages the inferior epigastric artery
 Is recommended

 The oblique incision:


 Is above the inguinal ligament and begins lateral to the linear semilunaris
to the tip of the 12th rib
 Divides the external and internal oblique but not the transversus
 May damage the lower thoracic and lumbar neves
 Provides good exposure on the right to the caecum, ascending colon and
ureter
 Gives a sound scar

 The gridiron (McBurney’s) incision:


 Affords rapid access to the appendix
 Affords access to the tubes and ovaries

2
 May damage the lilio-inguinal and illiohypogastric nerves
 Is at right angles to the spino-umbilical line two-thirds up from the anterior
superior iliac spine
 Divides the external oblique aponeurosis at right angles to the line of its
fibres

 In the Lanz incision:


 The skin incision is transverse in the line of the skin crease from the
anterior superior iliac spine
 The external oblique apneurosis is divided in the line of its fibres
 The internal oblique and tranversus muscle fibres are split in their
direction
 Good access is provided for the caecum, ascending colon and right ureter
 The scar may be almost invisible

 In the left subcostal incision the:


 Skin incision is about 5cm below the costal margin from the midline to
the linear semilunaris
 Anterior and posterior rectus sheaths and rectus muscle are divided in the
line of the incision
 7th, 8th and 9th thoracic nerves are not divided.
 Access to the stomach and spleen is adequate
 Scar is strong

 In the pfannestiel incision the:


 Skin incision is in the line of the skin crease about 5cm above the pubis
and inguinal ligaments
 Anterior rectus sheath and recti are also divided transversely
 Posterior rectus sheath with the peritoneum is divided vertically
 Llio-hypogastric and ilio-inguinal nerves may be damaged
 Scar is strong

 Rupture of the inferior epigastric artery:


 Occurs in young adults
 Occurs at the level of the arcuate line following severe blow to the
abdomen
 Causes a tender lump in the lower abdominal wall which disappears when
the abdominal muscles are tensed up
 May be confused with an appendix mass
 Is treated by evacuation of the clot and ligation of the artery

 The following are noted complications of neonatal omphalitis:


 Umbilical hernia
 Omentel bursa infection

3
 Splenic abscess
 Portal vein thrombosis with concomitant prehepatic portal hypertension
 Umbilical sinus formation

 Which of the following conditions may present as derivatives of the


neonatal vitello-intestinal duct
 The falciformligament
 Urachal fistula
 Meckel’s diverticulum
 Sister Mary Joseph’s nodule
 An uncolith(umbilical calculus)
 Ligamentum teres

 A tumour appearing in the umbilical scar is likely to be a secondary


neoplasm deriving most likely from the:
 Bronchus
 Kidneys
 Ovary
 Lymphatic tissue
 Skeletal muscle

 A fleshy swelling presenting in the umbilical region of a 40year old


nulliparous woman, and which bleeds intermittently is most likely to be a
:
 Sister Mary Jospeh’s nodule
 Oncolith(Umbilical calculus)
 Umbilical melanoma
 Endometrioma
 Urachal cyst

 A frail woman of 75 previously well, presents with sudden onset of pain in


the right lower abdomen. There is no fever, nausea or vomiting. On
examination she is not anaemic, her pulse is 96/min with BP of 195/115.
There is localized tenderness with guarding in the right iliac fossa; a
vague mass is discernable at the site.
The most likely diagnosis is:
 Appendix mass
 Twisted ovarian cyst
 Paracaecal obscess
 Diverticular abscess
 Repture of inferior Epigastric Artery

 A woman of 42 who presents at 32 weeks into her third pregnancy having


experienced a bout of the flu followed by upper respiratory tract infection
evidenced by headaches, unproductive cough and fever. On examination
the uterine mass is non tender and is consistent with her dates; the pulse

4
is 84/min, BP 140/90 and temperature 37.70C. There is an area of
localized tenderness in the ® lower quadrant with guarding and a vogue
mass is discernable. The most likely diagnosis is:
 Abruptio placenta
 Acute degeneration of uterine fibroids
 Acute appendicitis
 Rapture of inferior Epigatsric Artery
 Twisted ovarian cyst

CHAPTER 29

 A, C, D
 B, D, E
 B, E
 A, C
 A, C, E
 A, D, E
 C, E
 B, C
 A, D
 B, C
 A, D, E
 A, C
 A, B, C, E
 B, E
 A, C, E
 B, D, E
 A, D
 C
 C
 D
 E
 D

5
CHAPTER 30
HERNIA (EXCLUDING DIAPHRAGMATIC HERNIA)

 A hernia:
 Is the protrusion of a viscus or part of it from the cavity in which it is
enclosed.
 Is the commonest condition encountered in general surgical practice.
 Afflicts about 10% of a community
 Is the commonest cause of intestinal obstruction in Black Africa
 Accounts for about 40% of cases of intestinal obstruction in Ghana.

 Concerning hernia:
 An inguinal hernia accounts for 95-97%
 A femoral hernia is a hernia through the femoral sheath
 A spigelian hernia is a hernia through the internal oblique
 A paraumbilical hernia is a hernia through the blinded linear alba in the
umbilicus
 An epigastric hernia is a hernia through the linear alba midway between
the xiphisternum and the umbilicus.

 The commonest hernia in women is:


 Femoral
 Umbilical
 Inguinal
 Paraumbilical
 Epigastric

 Aetiological factors in hernia formation include:


 Urethral stricture
 Frequent pregnancies
 Obesity
 Ageing
 Sites of penetration of nerves

 Aetiological factors in hernia formation include:


 Sedentary work
 Anatomical defects
 Wound infection
 Diarrhoea
 Weight lifting

 Uncommon contents of a hernia sac include:


 Omentum
 Fallopian tubes
 Parts of the colon

1
 Bladder
 Appendix

 Complications of a hernia include:


 Fistula formation
 Infertility
 Rupture of the sac
 Retention of urine
 Impotence

 The inguinal canal:


 Contains the spermatic cord, testicular vessels and ilio-inguinal and
genitofemoral nerves in the male
 Is about 5cm long
 Embryologically allows the passage of the round ligament to the labium
major
 Has its anterior wall formed by the external oblique muscle
 Has its posterior wall formed by the transversalis fascia, transversus
abdominis and buttressed in the medial half by the conjoint tendon

 The internal inguinal ring:


 Is V-shaped
 Is an obliquely-placed evagination or opening in the transversus
abdominis.
 Lies 1.25cm above a point midway between the anterior superior iliac
spine and the public tubercle
 Is pulled upwards and outwards behind the transversus when the
abdominal muscles are put under strain by any increase in intra-abdominal
pressure.
 Is bounded above by the lower arching fibres of the internal oblique and
below and medially by the inferior epigastric vessels

 The external inguinal ring is:


A. An opening in the external oblique aponeurosis
B. Situated just above and medial to the pubic tubercle
C. U-Shaped
D. 3.0 x 2.0 cm
E. Covered directly by skin

 Hesselbach’s triangle is:


 Bounded laterally by the inferior epigastric artery
 Bounded medially by the medial border of the rectus sheath
 In 20% of people not reinforced by aponeurotic fibres from the transervus
abdominis
 Buttressed by the internal oblique muscle
 Often pushed forward to form a diffuse or wide-necked indirect inguinal

2
hernia

 The femoral triangle:


 Is bounded medially by the Sartorius
 Is bounded laterally by the gracilis
 Has its floor formed by the iliopsoas laterally and the pectineus medially
 Contains lymph nodes and in oder from the medial to the lateral end, the
femoral canal, femoral artery, femoral vein and femoral nerve
 Has the fascia lata, superficial fascia and skin forming its anterior wall

 The femoral canal:


 Is obliquely-placed
 Is up to 5cm long
 Extends from the femoral ring in the abdomen to the saphenous opening in
the thigh
 Contains several lymph nodes and fat
 Is covered above by peritoneum

 The femoral ring is:


 About 1cm wide and in the anterior abdominal wall
 Narrower when the conjoint tendon is inserted to the pubic tubercle or the
pubic tubercle and the medial part of the pectineal line rather than to the
entire pectineal line.
 Bounded anteriorly by the inguinal ligament
 Bounded posteriorly by the pubic bone and pectineal ligament
 Bounded medially by the lacunar ligament and fascia from the iliopubic
tract.

 The testis:
 Develops from the genital ridge in the posterior abdominal wall lateral to
the developing kidney
 Lies in the iliac fossa near the internal inguinal ring from the second to the
seventh month of intra-uterine life
 Has traversed the inguinal canal by the 8th month and is at the external
inguinal ring
 Descends into the scrotum in the 9th month
 Descends later on the right than on the left and in about 10% of infants
descent is not complete at term.

 The processus vaginalis is:


 Normally completely obliterated
 Patent proximally in 60% of neonates
 Patent proximally in 40% of infants at 2 years
 Patent proximally in 20% of infants at 3 years
 Patent proximally in 15-30% of adults without an inguinal hernia

3
 Failure of obliteration of the processus vaginalis may lead to:
 Indirect inguinal hernia in infacts but not in adults
 Encysted hydrocele of the cord
 Infantile or congenital hydrocele
 Vaginal hydrocele
 Cyst of the canal of Nuck

 Concerning inguinal hernia:


 The female/Male ratio varies from 1/20 to 1/55
 The direct type can normally pass through the external inguinal ring
 The direct type usually occurs in all age groups, the incidence rising with
age.
 The indirect type is more common on the left than on the right in both
sexes
 The direct type is more common on the left than on the right in both sexes

 A primary indirect inguinal hernia:


 Is acquired
 Is predisposed to by a weakened internal inguinal ring
 Comes out obliquely through the inguinal canal
 Can usually enter the labium majus
 Never appears on coughing when the internal inguinal ring is occluded
with a finger

 A direct inguinal hernia:


 Is acquired
 Is usually due to attenuation of the transversalis fascia or deficiency of re-
inforcing aponeurotic fibres from the transversus abdominis
 Is as liable to strangulation as the indirect type
 May not appear on coughing when the internal inguinal ring is occluded
 Can usually enter the scrotum

A 5-year old boy has a scrotal swelling extending from the right half of
the groin. It is soft, fluctuant and irreducible; there is no obvious cough
impulse and the testis is impalpable.

 The diagnosis may be:


 R.I.H. – complete and irreducible
 Vaginal hydrocele
 Encysted hydrocele of the cord
 Congenital hydrocele
 Infantile hydrocele

A 20-year old man has a tense fluctuant ovoid swelling without a cough
impulse in the left groin which moves with traction on the testis.

4
 The most likely clinical diagnosis is:
 Sebaceous cyst
 L.I.H. – irreducible, incomplete and probably containing omentum
 Encysted hydrocele of the cord
 Cyst of the epididymis
 Lipoma

 In the management of inguinal hernia, a truss is indicated in:


 Patients with a medical disease
 Elderly men with wide-necked direct hernia
 Elderly men with complete, reducible indirect hernia
 Elderly men with incomplete, reducible indirect hernia
 Patients with recurrent indirect inguinal hernia

 The essential requirements in the operative treatment of an indirect


inguinal hernia in an adult include:
 Excision of the hernia sac at the neck.
 Tightening of the internal inguinal ring
 Strengthening of the posterior wall of the inguinal canal
 Tightening of the external inguinal ring
 Strengthening of the anterior wall of the inguinal canal

 The essential requirement(s) in the operative treatment of an indirect


inguinal hernia in a child is (are):
 Excision of the hernia sac at the neck
 Tightening of the internal inguinal ring
 Strengthening of the posterior wall
 Strengthening of the anterior wall
 Tightening of the external inguinal ring

 The essential requirements in the operative treatment of a wide-necked


direct inguinal hernia in a middle-aged man include:
 Excision of the hernia sac at the neck
 Repair of the transversalis fascia
 Tightening of the internal inguinal rink
 Strengthening of the posterior wall
 Tightening of the external inguinal ring

 Tanner slide:
 Is separation of the attachment of the external oblique oponeurosis from
the rectus sheath
 Is an incision in the transversalis fascia in the triangle of Hesselbach
 Is an incision in the anterior rectus sheath close to the attachment of the
external oblique aponeurosis
 Is an incision in the conjoint tendon

5
 Relieves tension on Bassini repair

 The essential requirements in the operative treatment of an indirect


inguinal hernia in an adult female include:
 Excision of the hernia sac at the neck
 Narrowing of the internal inguinal ring
 Division of the round ligament
 Suturing of the conjoint tendon and internal oblique muscle to the inguinal
ligament
 Tanner slide

 Post-operative complications of inguinal herniorrhaphy include:


 Urinary tract infection
 Testicular atrophy
 Persistent pain in the wound
 Paralytic ileus
 Hydrocele

 Recurrence after inguinal herniorrhaphy is:


 7-14% overall
 5-10% in the indirect variety
 25-30% in the direct variety
 30-40% for recurrent inguinal hernia
 Commoner in men than in women because women usually have smaller
hernia

 Causes of recurrence of inguinal hernia after repair include:


 Age of the patient
 Duration and size of the hernia
 Wound infection
 Manual work
 Associated hydrocele

 Femoral hernia:
 Is as common in Black Africa as in Europe
 Is five times commoner in females
 Is more common in the multiparous than in the nulliparous
 Occurs in all age groups
 In Ghanaian males is commoner than umbilical hernia

 Aetiological factors in the causation of a femoral hernia include


 A congenital sac
 Failure of attachment of the conjoint tendon to the entire pectineal line
 Wide femoral ring
 Chronic constipation

6
 Weakening of the femoral sheath by the presence of the femoral vein and
artery

 A femoral hernia:
 Is usually below and lateral to the pubic tubercle
 May lie in front of or above the inguinal ligament
 Has a visible and palpable cough impulse in every patent unless it is
strangulated
 May not disappear when the patient lies down
 Commonly contains small intestine, bladder, colon or appendix and
occasionally omentum

 A 40-year old woman has a 2-year old lump in the right groin, below and
lateral to the pubic tubercle and with doubtful cough impulse. It
disappears as soon as the patient lies down and elevates her leg above the
trunk. The most likely clinical diagnosis is:
 Femoral hernia
 Encysted hydrocoele in a femoral hernial sac.
 Saphena varix
 Femoral aneurysm
 Psoas abscess

 Femoral hernia:
 Can be adequately controlled by a truss
 Is more liable to strangulation than any other hernia
 Is most commonly strangulated by the lateral edge of the lacunar ligament
 May be strangulated at the saphenous opening
 May be strangulated by the neck of the sac or by the defect in the
transversalis fascia

 Essential requirements in the operative treatment of femoral hernia


include:
 Excision of the sac
 Excision of femoral canal
 Closure of the femoral ring
 Strengthening of the transversalis fascia at the site where the hernia
orginates
 Narrowing of the saphenous opening

7
 Recurrence after femoral herniorrhaphy is:
 5-10%
 10-20%
 20-25%
 25-30%
 About 35%

 Exomphalos:
 Occurs in 1 in 6,000 births
 Is major if it is large
 Is minor if it is small
 Has associated congenital anomalies such as heart disease, cleft palate and
intestinal atresia in about 80% of patients
 Is associated with non-rotation of the gut in about 25% of patients who
frequently are premature

 In exomphalos major:
 The causes are failure of development of the abdominal cavity and non-
rotation of the bowel
 The hernia sac is translucent and jelly-like and consists of amnion,
Wharton’s jelly and peritoneum
 The umbilical cord is attached to the superior border of the sac
 The sac contains spleen, the transverse colon, and stomach and at times
small intestine as well.
 The defect in the abdominal wall is always large

 In exomphalos minor:
 The cause is incomplete return of the midgut from the umbilicus to the
abdominal cavity
 The hernia contains a loop of small intestine
 The defect is small
 The umbilical cord is attached to the midpoint
 Treatment is always operative

 Treatment of exomphalos major


 Is conservative in all patients
 Includes local application of an antiseptic such as povidone-iodine or
aqueous mercurochrome
 Usually requires intravenous fluid administration for a few days
 Does not require nasogastric aspiration
 May result in a small umbilical hernia which is repaired when the child is
12-18 months

 Infantile umbilical hernia:


 Is congenital

8
 May be predisposed to by umbilical sepsis
 Is commoner in premature infants
 Is commonly irreducible
 Is treated by Mayo’s overlapping operation in most patients

 Paraumbilical hernia:
 Is more common in multiparous obese women
 Usually has a narrow neck
 May rupture
 Can be satisfactorily controlled with an abdominal belt in most patients
 If treated by operation has a negligible recurrence rate.

 Epigastric hernia:
 Occurs commonly in all age groups but especially in those between 20
and 50
 Is equally common in the sexes
 May be multiple
 Is a protrusion of extra-peritoneal fat in a few but in the majority has a true
hernia sac.
 Usually has a small defect, about 1cm or less, through the linea alba.

 Epigastric hernia:
 Causes epigastric pain or discomfort in most patients
 Always has a cough impulse but it may not be reducible
 Rarely strangulates
 Should be left alone if symptomless
 Has a recurrence rate of 1% after operative repair

 Predisposing factors in incisional hernia include:


 Burst abdomen
 Lower midline or rectus muscle splitting incision
 Wound infection
 Placement of drainage tube in the wound
 Suturing of the anterior rectus sheath with nylon

 Incisional hernia:
 Has a wide and not a narrow neck
 May not have a sac
 Does not strangulate
 May rupture
 Should be repaired in every patient if possible

 Spigelian hernia:
 Usually occurs in the elderly
 Often causes localized pain, worse on coughing or straining and relieved

9
by lying down
 Usually appears medial to the lateral border of the rectus
 Spreads downwards and laterally or upwards and laterally
 May be strangulated by the cresentic rigid defect

 Obturator hernia:
 Is commoner in males and on the right
 Is usually seen in the middle-aged between 40-50
 Usually presents with acute intestinal obstruction
 Causes referred pain or paraesthesia along the posterior part of high or
knee
 Is always treated by operation.

 Lumbar hernia:
 Is commoner in males
 Is usually seen in people between 50 and 70
 May be confused with or cold abscess
 Is acquired but may very rarely be congenital
 Need not be repaired

 Sites of internal hernia include:


 Additus to the lesser sac
 Diaphragm
 Paracaecal fossa
 Openings in the mesentery and broad ligament
 Intersigmoid fossa

 In Linchtenstein repair

 After herniotomy a square shaped piece of polypropylene is cut and


sutured to the posterior wall.

 The lateral end is split to accommodate the spermatic cord.

 The mesh is placed over the inguinal floor between the inguinal ligament
and the conjoint tendon.

 The mesh is absorbed over a period of 6 months.

 When the mesh becomes incorporated, tissue collagen grows into it.

 In tension-free repair,

 The procedure reduces both early and post-operative pain

10
 There is early return to work

 Recurrence is reduced.

 Recurrence may be due to placement of the mesh too laterally

 Recurrence may be due to separation of the mesh from the inguinal


ligament.

 REPAIR OF INGUINAL HERNIA

 A truss is the best method of managing a hernia in the aged.

 Herniotomy involves excision of the sac at its neck

 In the repair there is the need to tighten the deep inguinal ring around the
spermatic cord.

 Repair of the posterior wall should be done in those from the age 6 years.

 The best suture to us is dyed vicryl 1 suture.

 REPAIR OF INGUINAL HERNA

 Bassini repair should only be done with Tanner slide.

 In Bassini repair the most medial stitch should pass through the
periosteum of the pubic tubercle.

 In nylon darn repair the gap between the inguinal ligament and the
conjoint tendon is re-enforced with 2 layers of monofilament nylon.

 In the above the use of polypropylene is an alternative.

 Shouldice repair is similar to the above but the transversalis fascia is


sutured in 2 layers.

 LAPAROSCOPIC REPAIR OF INGUINAL HERNIA

 The inguinal hernia may be repaired transperitoneally

 The inguinal hernia may be repaired extra peritoneally.

 In total extra-peritoneal repair the space is enlarged by a disposable


balloon dissector.

11
 Laparoscopic repair is not suitable for bilateral inguinal hernia.

 Laparoscopic repair is not suitable for direct inguinal hernia

 Extra abdominal herniae,


 Richter's hernia may cause strangulation without obstruction.
 In Maydl's hernia usually only a small segment of bowel is resected.

 The anterior wall of the inguinal canal is reinforced medially by fibres


of the internal oblique.
 Direct inguinal hernias are more common in the over-40 age group.
 Direct hernias have a smaller risk of strangulation than indirect
hernias.
 Considering epigastric herniae,

(a) Midline incisions are the preferred incisions


(b) The content is usually small bowel.
(c) Dyspepsia may be a presentation
(c) Repair is best done vertically.
(d) Divarication of the recti requires repair in all situations.
60. Considering groin hernias,
(a) Direct hernias have a smaller risk of strangulation than indirect hernias.
(b) Clinical distinction between direct and indirect hernia is usually
accurate.
(c) The superficial inguinal ring lies above and lateral to the pubic
tubercle.
(d) If the hernia is a sliding type, that is, the sac is closed with a purse-string
suture, rather than transfixion.
(e) Direct sacs do not usually need to be opened during repair.

12
CHAPTER 30

 A, B, D
 A
 C
 A, B, C, D
 B, C, E
 B, D
 A, C
 C, E
 D
 A
 A, C
 C
 C, E
 A, C, D, E
 C, D, E
 B, C, E
 B, C, E
 A
 C, D, E
 A, B
 D, E,
 C
 B, D
 A, B, C
 A
 B, D
 C, E
 A, D
 B, C, E
 A, B, C
 A, B, C
 C
 B, C
 A, B, D

13
 A, C
 B, C, E
 A, C
 B
 A, E
 A, B
 A, B, C, D, E
 B,C, E
 B, C
 A, C, E
 C, E
 C, D
 A, B, C, D
 B, D
 A, B, D, E
 C, E
 A, B, C, D
 A, B, C, D, E
 B, C, E
 A,B,C,D,E
 B, C
 B,C,D,E
 A,B,C
 A, D, E
 C
 A, D, E

14
CHAPTER 31
THE ACUTE ABDOMEN

 The acute abdomen:

 Is an abdominal condition of sudden onset

 Does not require an operation in most cases

 With pain of sudden onset is suggestive of a perforated viscus

 With pain of insidious onset which is worse on movement but


somewhat relieved on lying still is suggestive of bleeding

 With spasmodic pain associated with sweating or vomiting is


suggestive of colic

 In an acute abdomen, a history of frequent copious vomiting, the


vomitus being clear at first, then bile-stained and finally brown in
colour, is suggestive of:

A. Acute gastritis
B. Gastric outlet obstruction
C. Gastro-enteritis
D. Acute high intestinal obstruction
E. Acute pancreatitis

 In acute abdomen in a 2-year old infant, a history of frequent anal


passage of bloody slime without faeces is very suggestive of:

A. Bacillary dysentery
B. Intussusception
C. Amoebic dysentery
D. Cholera

1
E. Ankylostomiasis

 In acute abdomen in a West African adult, a history of frequent


passage of stools with blood and mucus is suggestive of:

A. Bacillary dysentery
B. Ulcerative colitis
C. Acute appendicitis
D. Amoebic dysentery
E. Cholera

 In acute abdomen in a 20-year old woman, a history of recent


vaginal discharge is suggestive of:

A. Cervicitis
B. Pelvic inflammatory disease
C. Gonorrhoea
D. Trichomonad vaginitis
E. Endometritis

 In acute abdomen, a history of preceding headache, fever and


malaise is suggestive of:

A. Malaria

 Acute appendicitis
C. Typhoid perforation
D. Amoebic perforation
E. Perforated duodenal ulcer

 In a 25-year old woman with acute abdomen, the finding of pale

2
conjunctivae and hypotension should immediately raise a
suspicion of:

A. Twisted ovarian cyst


B. Ruptured Graafian follicle
C. Degenerating myoma uteri
D. Missed abortion
E. Ruptured ectopic pregnancy

 If in a 30-year old man with acute abdomen, you observe that the
abdomen does not move on respiration, you will suspect:

A. Acute intestinal obstruction


B. Generalized peritonitis
C. Haemorrhage
D. A ruptured hollow viscus
E. Acute pancreatitis

 An acute abdomen in a 25-year old woman with absent bowel


sounds may be due to:

A. Acute mechanical intestinal obstruction


B. Ruptured tubal pregnancy
C. Typhoid perforation
D. Uncomplicated acute appendicitis
E. Gastro-enteritis

 An acute abdomen in a 40-year old man with increased high-


pitched bowel sounds may be due to:

A. Acute diverticulitis of the sigmoid colon


B. Gastritis

3
C. Rupture of abdominal aortic aneurysm
D. Mesenteric thrombosis
E. Strangulated internal hernia

 Investigations that must be done In every adult West African


patient with acute abdomen include:

A. Straight X-ray of the chest and abdomen


B. 4-quadrant peritoneal tap
C. Full blood count and sickling
D. Blood film for malaria parasites
E. Urinalysis

 In a 4-quadrant tap in an adult male with acute abdomen, if the


aspirate is:

A. Yellowish, the diagnosis is probably perforation of the ileum

 Blood-stained, the diagnosis is probably acute pancreatitis or


strangulation intestinal obstruction

 Blood in all the quadrants, the needle has probably punctured a


vessel and

4
not entered the peritoneum
D. Turbid, the diagnosis is probably peritonitis
E. Nil, there is probably nothing serious in the abdomen

 If there is sugar in the urine in a 40-year old woman with acute


abdomen the condition may be:

A. Pre-diabetic coma
B. Acute pancreatitis
C. Acute abdomen in a diabetic
D. Urinary tract infection
E. Viral hepatitis
A 12-year old West African boy known to suffer from sickle cell
disease has a 24-h history of persistent pain in the R.I.F. which
started around the umbilicus. There is nausea but no vomiting.
There are no bowel or urinary symptoms and no joint aches. On
examination, the conjunctivae are moderately pale, the tongue
slightly furred, the temperature 38oC and the pulse 100/min. There
is marked tenderness in the R.I.F. with guarding rebound
tenderness. Rectal examination is unremarkable.

 The most likely clinical diagnosis is:

A. Sickle-cell crisis
B. Acute appendicitis
C. Mesenteric adenitis
D. Right iliac adenitis
E. Malaria

5
 You will advise:

A. Observation
B. Paracetamol
C. Paracetamol and artesunate + amodiaquinel
D. Immediate appendicectomy
E. Antibiotics
A 15-year old Ghanaian boy has for the past 3 days been having
headache, malaise, evening pyrexia and pain especially in the
R.I.F. He has been vomiting on and off but has not been
constipated. There is some frequency but no dysuria. On
examination, the temperature 39oC and the pulse 108/min. The
conjunctivae are pink and the tongue clean. The abdomen is soft
with slight tenderness in the R.I.F without guarding. Rectal
examination is unremarkable.

 The most likely clinical diagnosis is:

A. Acute appendicitis
B. Enteric fever
C. Sickle cell disease
D. Urinary tract infection
E. Malaria

 The essential investigations will include:

A. White cell count and differential


B. Blood culture

6
C. Blood film
D. Urinalysis
E. Sickling

 Meanwhile you will advise:

A. Appendicectomy
B. Ciprofloxacin + metronidazole
C. Paracetamol
D. Artesunate + Amodiaquine + Paracetamol
E. Septrin

A 30-year old man has a 4-hour history of sudden onset of severe


epigastric pain which quickly spread to the whole abdomen. He
has vomited once and has not had any bowel action. He is found
on examination to be anxious and in pain. The conjunctivae are
pink, the tongue is coated, the temperature 37oC, pulse90/min and
of good volume and the BP 120/80. Respiration is shallow and the
non-distended abdomen does not move with it. The hernia orifices
are clear. The abdomen is generally tender with rebound

7
tenderness and rigidity especially in the epigastrium. Bowel
sounds are infrequent and rectal examination is tender.

 The most likely clinical diagnosis is:

A. Acute intestinal obstruction


B. Perforated duodenal ulcer
C. Generalized peritonitis from acute appendicitis
D. Acute pancreatitis
E. Perforated typhoid

 If there is uncertainty about the diagnosis, the most useful


diagnostic investigation will be:

A. Serum amylase
B. 4-quadrant paracentesis
C. Full blood count
D. Widal
E. Straight X-ray of the abdomen and chest in the erect position

 Important investigation(s) will be:

A. Serum electrolytes and blood urea


B. Haematocrit
C. Haemoglobin and sickling
D. Urine for sugar, proteins and cells
E. Blood film and blood culture

 Treatment will include:

A. Glucagon I.V.
B. Intravenous fluid therapy
C. Nasogastric decompression

8
D. Observation
E. Antibiotic therapy

 The cardinal clinical features of acute mechanical intestinal


obstruction include:

A. Colicky abdominal pain


B. Constipation
C. Vomiting and/or distension
D. Generalised abdominal rebound tenderness
E. Absent bowel sounds
A 20-year old male in Accra who has been having headache,
malaise, fever, and watery stools without blood for 5 days
suddenly experiences severe lower abdominal pain which
becomes generalized. Vomiting has been repeated and the
diarrhoea has stopped. He is ill with a temperature of 39oC, pulse
120/min and blood pressure of 90/60. The conjunctivae are rather
pale and the tongue dry and coated. The abdomen moves slightly
on respiration and is generally tender with rebound tenderness
and guarding especially in the lower part. The bowel sounds are
infrequent and rectal examination reveals tenderness in the recto-

9
vesical pouch and no blood or mucus on the finger.

 The most likely clinical diagnosis is:

A. Intussusception
B. Gastro-enteritis
C. Typhoid perforation
D. Peritonitis from amoebiasis
E. Acute appendicitis

 The management will include:

A. Intravenous fluid therapy


B. Potassium supplementation
C. Administration of metronidazole + phthalylsulphathiazole
D. Intravenous ciprofloxacin and metronidazole
E. Observation for 24-48h

 The following medical conditions may “cause” acute abdominal


pain.

A. Myocardial infarction
B. Glomerulonephritis
C. Malaria
D. Meningitis
E. Herpes zoster

10
 The clinical features of acute pancreatitis include:

A. Constant severe pain in the upper abdomen often with penetration


to the back
B. Absolute constipation
C. Profuse vomiting
D. Abdominal distension with tenderness and guarding mainly in the
lower
abdomen
E. Shock and a tinge of jaundice

 In biliary colic:

A. The pain originates in the epigastrium or right hypochonrium


B. The pain radiates to the lower end of the left scapula
C. There may be associated sweating, vomiting or jaundice
D. There is tenderness with rebound tenderness and guarding in the
right
hypochondrium
E. A straight X-ray of the abdomen usually reveals a calculus

 In ureteric colic:

A. The pain is persistent and severe


B. The pain starts in the loin and radiates towards the groin
C. There may be a desire to micturate
D. The urine is macroscopically bloody

11
E. The patient may be restless

 In myocardial infarction:

A. The pain is in the precordium and may radiate down the left arm
B. The upper abdomen is tender and guarded
C. The abdomen is silent
D. Cardiac signs are present
E. The diagnosis is usually established with E.C.G. in doubtful cases

 In salpingitis:

A. The pain is usually bilateral


B. There is usually a history of frequency, dysuria, haematuria and
vaginal
discharge
C. Tenderness is low down in both iliac fossae and above the pubis
D. Vaginal examination reveals tenderness in both fornices
E. A right-sided salpingitis is often difficult to differentiate from pelvic
appendicitis

 In gastroenteritis:

A. There is constant pain in the epigastrium or central abdomen


B. The pain persists even after the bowel is opened
C. There is frequent passage of watery stools
D. The vomitus is clear at first and later becomes brown or black
E. Abdominal tenderness is diffuse and deep

12
 In right basal pneumonia in children:

A. There may be referred pain in the right side of the abdomen


B. There is tenderness with some guarding in the R.I.F.
C. Dyspnoea and pyrexia are present
D. Breath sounds are not diminished
E. An x-ray of the chest should be done if there is any doubt

CHAPTER 31
THE ACUTE ABDOMEN
ANSWERS

 A, C, E

 D

 B

13
 A, D

 B, C

 A, C

 E

 B, D

 B, C

 E

 A, C,D, E

 A, B, D

 A, B, C

 B

 D

 E

 A, B, C, D, E

 D

 B

 E

 A, C, D

 B, C, E

 A, C

 C

 A, B, D

 A, C, E

 A, C, E

 A, C

 B, C, E

14
 A, D, E

 A, B, C, D, E

 C, E

 A, D, E

15
CHAPTER 32

PERITONEAL CAVITY

 The peritoneum:

 Has about the same surface area as the skin, approximately 1.8m2

 Is lubricated by about a litre of pale yellow fluid

 Consists of a surface mesothelium and an underlying loose muscular tissue

 Is semi-permeable

 Is all supplied by autonomic nerves

 In the anatomy of the subphrenic space:

 The space is divided into right and left compartments by the falciform
ligament

 The falciform ligament sweeps from the diaphragm and back of the linear
alba as far down as 2cm below the umbilicus

 The falciform ligament is attached to the anterior and superior surfaces of


the liver in the median place

 The coronary ligament suspends the liver superiorly from the diaphragm
and divides the subphrenic space into anterior and posterior parts

 The right subphrenic space communicates with the subhepatic space and
general peritoneal cavity

 The sub-hepatic space:

 Lies behind the posterior surface of the liver

 Is divided into right and left by the lesser omentum

 On the right is continuous posteriorly with the upper part of the right
paracolic gutter and communicates anteriorly with the right sub-phrenic
space and the general peritoneal cavity

 On the left communicates with the left paracolic gutter

 On the right is a more common site of abscess formation than the sub-
phrenic space

1
 The epiploic foramen to the lesser sac (foramen of Winslow) is:

 Bounded posteriorly by the inferior vena cava and aorta

 Bounded anteriorly by the common bile duct, hepatic artery and portal
vein

 Bounded superiorly by the caudate lobe of the liver

 Bounded inferiorly by the first part of the duodenum

 Occasionally the site of strangulated bowel

 Recesses of the peritoneum in which a loop of bowel may herniated and


become strangulated include:

 The superior ileo-caecal recess between the inferior surface of the


terminal ileum, the mesentery and the medial surface of the caecum

 The obturator foremen

 The inferior ileo-caecal recess between the inferior surface of the terminal
ileum, the meso-appendix and the triangular ileo-caecal fold

 A para-duodenal recess between duodeno-jejunal flexure and the superior


mesenteric vein

 A recess formed by the inverted attachment of the pelvic mesocolon

 The anterior layer of the lesser sac is formed by the:

 Caudate lobe

 Lesser omentum

 Stomach

 Transverse colon

 Transverse mesocolon

 Primary peritonitis:

 Is uncommon

2
 Is seen in boys and girls between 4 and 10

 Is caused by pneumococcus and is occasionally secondary to


pneumococcal chest infection

 Gives a greenish pus which is most profuse in the pelvis

 Can be safely differentiated from secondary peritonitis from clinical


examination.

 Acute peritonitis may be caused by:

 Hepatitis

 Septicaemia

 Amoebic colitis

 Enteric fever

 Leakage of pelvic abscess

 Concerning acute peritonitis:

 The commonest cause in males in West Africa is typhoid perforation

 The commonest cause in females in West Africa is acute salpingitis


followed by typhoid perforation

 Perforated peptic ulcer is common in West Africa

 Acute diverticulitis is fairly common in West Africa

 Acute appendicitis is the commonest cause in Europe

 Organisms usually isolated in acute generalized peritonitis include:

 Strep. Haemolyticus

 Pseudomonas

 Staph. Epidermidis

 Bacteroides

 Clostridia

3
 The pathological processes in acute generalized peritonitis include:

 Vascular constriction

 Increased capillary permeability

 Out-pouring of copious amount of sero-fibrinous fluid

 Rupture of capillaries

 Matting of viscera

 Fibrous adhesions following acute peritonitis are due to:

 Massive fibrin deposition

 Vit. C deficiency

 Deficiency of proteolytic enzymes

 Relative ischaemia in parts of the inflamed peritoneum leading to growth


of capillaries to the area

 Excessive growth of fibroblasts

 In acute generalized peritonitis:

 There is loss of large amounts of plasma, 4-6L, into the peritoneal cavity

 There is loss of E.C.F. into the loss connective tissues of the peritoneum

 There is increased pancreatic and biliary secretion into the gut

 There is increased absorption of water and electrolytes by the intestines

 Loss of sodium, potassium chloride and protein is considerable.

 The rapid absorption of toxins and bacteria from the peritoneum in acute
generalized peritonitis is due to:

 Increased vascularity of the peritoneum

 Paralytic ileus

4
 Semi-permeability of the peritoneum

 Extensiveness of the peritoneum

 Increased activity of the bacteria

 In generalized peritonitis there may be:

 Respiratory alkalosis

 Metabolic acidosis

 Hypoproteinaemia

 Hyperchloraemia

 Hypocalcaemia

 In generalized peritonitis, causes of death directly related to it include:

 Acute tubular necrosis

 Congestive cardiac failure

 Respiratory failure

 Cerebro-vascular accident

 Septicaemia

 The pain in acute generalized peritonitis is:

 Intermittent

 Severe

 Burning in character

 Made sores by movement

 Relieved somewhat by breathing

 In generalized peritonitis:

5
 Copious persistent vomiting is the rule

 Constipation is usual but there may be diarrhea

 Appetite is not usually affected much

 Pyrexia is invariable even in the early stage

 A careful history will often indicate the site of origin of the pain

 Breathing in acute generalized peritonitis is:

 Rapid

 Deep

 Grunting

 Stertorous at times

 Irregular at times

 The abdomen in acute generalized peritonitis:

 Is not distended

 Moves very little with respiration

 Is tender with rebound tenderness

 Has board-like rigidity in most patients

 On auscultation at times gives rise to only audible heart and breath sounds

 Investigations useful in establishing a diagnosis of acute generalized


peritonitis or its cause include:

 Four quadrant peritoneal tap

 Serum electrolytes, blood urea and pH

 White cell count and differential

 Straight X-ray of the abdomen and chest

 Vaginal and urethral smears in the female

6
 In acute generalized peritonitis, the primary condition is most probably:

 Acute hemorrhagic pancreatitis if the site of maximum tenderness and


rebound tenderness is in the epigastrium

 In West African typhoid perforation if the site of maximum tenderness and


rebound tenderness is in the lower abdomen especially the right half.

 Viral hepatitis if the site of maximum tenderness is the right


hypochondrium

 Acute appendicitis in situs inversus if the site of maximum tenderness in a


45-year old man is the L.I.F.

 Perforated peptic ulcer if the site of maximum tenderness is the


epigastrium

 Major conditions to be considered in the differential diagnosis of acute


generalized peritonitis include:

 Ruptured aortic aneurysm

 Malaria

 Strangulation obstruction

 Acute dysentery

 Urinary tract infection

 The PRINCIPLES of treatment in acute generalized peritonitis include:

 Blood transfusion

 Restoration of fluid and electrolyte deficits

 Nasogastric decompression of the bowel

 Administration of suitable antibiotics

 Relief of pain

 In acute peritonitis, operative intervention is usually urgent in:

 Perforation of a hollow viscus

7
 Acute diverticular disease

 Acute cholecystitis

 Amoebic colitis

 Puerperal sepsis

 Contra-indications to laparotomy in a patient with generalized peritonitis


on expectant treatment include:

 Moribund patient

 Localization of abscess

 Leakage of localized abscess

 Associated hypertension

 Continued improvement on expectant treatment

 In the prognosis of acute generalized peritonitis:

 The prognosis is better if treatment is instituted before 12 h than after 12 h

 Is as good in the aged as in the young adult provided there is no pre-


existing disease

 Perforation of the duodenum is more serious than one of the ileum


because of the acid-soiling in the former

 Paralytic ileus worsens the prognosis

 Development of a localized abscess gives a better prognosis

 Bile peritonitis:

 May be caused by gangrenous gall bladder of rupture of the liver

 Does not usually cause paralytic ileus

 Is usually associated with deep jaundice

 Is treated usually by cholecystectomy, choledochostomy and drainage of


the peritoneal cavity

 Has a low mortality

8
 Sites of intra-peritoneal abscess include the:

 Lesser sac

 Subhepatic space

 Bare area of the liver

 Paracolic spaces

 Paraduodenal recess

Ten days after closure of typhoid perforation, a 20-year old man develops
pain in the right upper abdomen, cough and a swinging temperature.
There is tenderness in the right upper abdomen over the liver between the
ribs. A few crepitations are heard over the right lower lobe.

 You will suspect:

 Rt. Subphrenic abscess

 Rt. Subhepatic abscess

 Rt. Basal preumonia

 Rt. Empyema

 Liver abscess

 The most useful investigation will be:

 White cell count and diffentential

 Liver biopsy

 Plain X-ray of the chest and abdomen

 Sputum culture

 Aspiration of the right pleura and culture

 Treatment will be a course of:

 Antibiotics

9
 Antibiotics and aspiration

 Metronidazole and antibiotics

 Antibiotics and extraperitoneal drainage

 Antibiotics and pleural drainage

 Complications of subphrenic abscess include:

 Empyema

 Rupture into the lung

 Liver abscess

 Peritonitis

 Septicaemia

 In subphrenic abscess a straight X-ray of the chest and abdomen may


show:

 Increased vascularity of the lungs

 Elevated diaphragm

 Little or no movement of the diaphragm with breathing

 Obliteration of the costo-phrenic angle

 Fluid with air below the diaphragm

 A right subhepatic abscess:

 Is less uncommon than subphrenic abscess

 May be a sequel to acute appendicitis perforated peptic ulcer or operation


on the stomach or biliary system

 May cause pain in the left shoulder and cough

 Gives rise to tenderness in the right hypochondrium, costal margin or over


the 12th rib

 Settles on antibiotics

10
 A pelvic abscess:

 Is the commonest intra-peritoneal abscess

 In the female is located mainly between the bladder and the uterus

 May present as a suprapubic mass or as a boggy mass bulging into the


posterior wall of the rectum

 Settle on expectant treatment in most patients

 In the female is best drained through the vagina

 Abscess in the lesser sac:

 May be a sequel to acute pancreatitis or perforation of a duodenal ulcer

 Presents as a tender, mobile, epigastric mass

 On barium meal shows anterior displacement of the stomach

 May case obstruction of the second part of the duodenum

 Settles on conservative treatment

 Surgical site infection

 Surgical site infection is defined as an infection related to a surgical procedure


that affects the surgical wound or deeper tissues handled during the procedure.

 Surgical site infection can be reliably identified from laboratory data alone

 Postoperative peritonitis is an organ-space surgical-site infection.

 Fifteen to 30 per cent of all intra-abdominal infections occur following an


operation.

 The diagnosis is usually delayed.

 Surgical site infection

 The most common cause is a technical error compromising the vascular


supply to an anastomosis.

 It results in necrosis and leakage of intestinal contents into the peritoneal


cavity.

11
 Iatrogenic perforation of a hollow viscus is another cause.

 An intra-abdominal hematoma may become secondarily infected, resulting in


an abscess.

 Selective use of CT scan in patients presenting with right lower quadrant pain
is helpful in reducing the incidence of non-therapeutic appendectomy.

 Abscesses in between loops of small bowel.

 Physical signs are usually diagnostic.

 Adhesions of small bowel may result.

 Lavage during surgery may prevent this.

 X’ray of the abdomen may be helpful in diagnosis.

 The abscess tends to be single.

12
CHAPTER 32

 A, D

 A

 B, C,E

 B, D, E

 C, E

 A, B, C

 A, C, D

 B, C, E

 E

 B, D, E

 B, C, E

 A, C, D

 B, E

 A, C, D

 B, C

 A, C, E

 B, C, D

 B, E

 A, C

 B, C, E

 A, D, E

 B, E

 A, C, D

 B, C, D, E

13
 A

 A, B, E

 A, D, E

 A, D

 A, B, D

 A, B

 C

 D

 A, B, D, E

 B, C, D E

 A, B, D

 A, D

 C, D

 A, D, E

 A,B,C,D,E

 B,C,D

14
CHAPTER 33
THE APPENDIX

 The appendix:

A. Arises from the medial aspect of the caecum about 5cm below the
ileocaecal
junction

 Is 7.5cm long on the average, but varies from 1.5cm to 30cm


C. Has a narrow lumen in the adult which can only admit a match stick
D. Has a narrow lumen in the infant and a usually obliterated lumen in
the aged
E. Is supplied by two branches of the ileocolic artery

 The appendix:

A. Has a meso-appendix attached to the lateral border from the base to


near the
tip

 Has aggregation of lymph follicles most profuse in the infant and at


15 years and absent in the aged
C. Is retrocaecal in 54% of patients
D. Is pelvic in 20% of patients
E. Is retro- or pre-ileal or paracaecal in 16%

 A mucocele of the appendix may:

A. Be a retention cyst following stenosis or carcinoid tumour of the


base
B. Result from tall and narrow epithelium which assumes malignant

1
propensities
C. May lead to pseudomyxoma peritonei
D. Present with pain in the R.I.F. or a palpable mass
E. Be calcified

 In pseudomyxoma peritonei:

A. Serous cystadenoma of the ovary may be the cause


B. The peritoneal cavity contains creamy gelatinous mucus
C. The abdomen is distended, tense and tender with rebound
tenderness
D. The peritoneum is washed with saline and thiotepa instilled
E. Thiotepa intravenously is beneficial

 Carcinoid tumour of the appendix:

A. Is usually small and golden yellow in colour

 Is usually found at laparotomy with metastases in the regional


lymph nodes and the liver
C. Usually causes attacks of vasomotor flushes, diarrhea and
bronchospasm
D. May present as acute appendicitis
E. Is treated by right hemicolectomy

2
 The incidence of acute appendicitis:

A. Is high in Europe, America and Asia and low in Africa


B. In Tropical Africa is the same in urban and rural communities
C. Increases with economic affluence and consumption of low fibre diet
D. In the U.S.A. is higher in Caucasians than in Blacks
E. Is higher in skilled than in unskilled workers

 The effects of cellulose-depleted diet include:

A. Low residue
B. Short transit time
C. Formation of firm and tenacious faeces
D. Development of low intra-luminal pressure
E. Formation of faecoliths

 The appendix may be obstructed by:

A. Ova of schistosoma
B. Lymphoid hyperplasia
C. Hypertrophy of the muscular layer
D. Fibrous adhesions in the lumen
E. Serosal adhesions and kinking

 Aetiological factors in acute appendicitis include:

A. Upper respiratory tract infection


B. Gastro-enteritis
C. Bacteria-Staph. aureus, Bacteroides, Strep faecalis and Esch coli

3
D. Luminal obstruction
E. Salpingitis in the female

 Acute appendicitis:

A. Is common under the age of 2 and in the aged


B. Is most common in the 2nd and 3rd decades with nearly 70% of
patients
C. In those over 40 accounts for 20% of patients
D. Before puberty is twice as common in male as in female
E. After puberty is equally common in the sexes

 In simple acute (catarrhal) appendicitis:

A. The mucosa and submucosa are inflamed and oedematous


B. The lumen is filled with pus and there is rapid rise in intraluminal
pressure
C. The serosa is red with increased vascularity
D. Peritonitis may result if it is not treated
E. Resolution usually occurs with formation of fibrous adhesions either
within the lumen or on the serosa

4
 Usual causes of perforation or gangrene of the appendix in acute
appendicitis include:

A. Foreign body in the appendix


B. Thrombosis of the appendicular artery
C. Venous stasis
D. Faecolith

 Pus in the lumen

 Acute appendicitis may lead to:

A. Subhepatic abscess
B. Pylephlebitis or liver abscess
C. Mesenteric arterial thrombosis
D. Cholangitis

 Acute intestinal obstruction

 In acute appendicitis, the pain:

A. Typically starts as central abdominal pain which is visceral pain


referred to the
centre of the eleventh thoracic segment

 May be colicky

 May settle in the right lumber region or hypogastrium

 May radiate from the right iliac fossa to the centre of the abdomen

 May improve for some hours only to become more severe later

 In acute appendicitis:

A. Nausea is a prominent symptom and vomiting is usually frequent


B. Anorexia does not usually occur

5
C. Diarrhea occurs in about 20% of patients especially in the young
D. Frequency and dysuria may occur
E. There is pyrexia with associated tachycardia from the beginning of
the attack

 The earliest and most important single sign in acute appendicitis


is:

A. Rebound tenderness

 Guarding
C. Typical “appendiceal foetor”
D. Pain on flexing the right hip
E. Localised tenderness over the appendix

 Children with acute appendicitis:

A. Are usually restless and unable to sleep

 Invariably have pyrexia and tachycardia


C. May or may not have anorexia or vomiting
D. Have rectal tenderness which is of diagnostic importance
E. Have a high incidence of complications due mainly to failure of the
under-
developed omentum to localize the infection

 Acute appendicitis in pregnancy:

A. Is more common in the third trimester than in the first two

6
B. In the first two trimesters may pose diagnostic problems
C. In the third trimester gives rise to pain in the right upper abdomen
and rather
laterally
D. In the third trimester causes marked muscle guarding
E. In the third trimester causes pain which may be confused with labour
pains

 In female children the usual differential diagnosis in acute


appendicitis include:

A. Acute cholecystitis
B. Left basal pneumonia
C. Malaria
D. Non-specific mesenteric adenitis
E. Ruptured Graafian follicle

 Generalized peritonitis in acute appendicitis:

A. Most commonly occurs from the spread of infection from the serosal
surface of
the appendix to the visceral and parietal peritoneum

 Is more common before 14 and after 40 than between 14 and 40


C. Is more common in female
D. Does not occur in those who seek medical aid within 24h of onset of
symptoms
E. Is seen in about 12% of patients in developing countries and about
6% of
patients in developed countries on admission

 An Appendix mass in the R.I.F.:

A. Consists of the inflamed appendix buried in the centre of the

7
caecum,
omentum, coils of terminal ileum and the sigmoid colon
B. Is seen in patients with average duration of symptoms of 7-8 days
C. Nearly always proceeds to suppuration
D. Is proportionately less common after 40 years
E. Is difficult to differentiate from a appendix abscess

 The appendix mass:

A. Moves with respiration

 Has indefinite edge


C. Is tender
D. Has no guarding over it
E. Is soft

 The differential diagnosis of appendix abscess in a man of 35


include:

A. Amoeboma

8
B. Ileo-caecal tuberculosis
C. Carcinoma of the caecum
D. Pyomyositis of the anterior abdominal wall
E. Right hydronephrosis

 Important investigations in a 25-year old man with a 7-day tender


mass in the R.I.F. suspected to be due acute appendicitis include:

A. White cell count and differential


B. Barium enema
C. Colonoscopy
D. Intravenous pyelogram
E. Straight X-ray of the abdomen

 The differential diagnosis in a girl of 10 with a tender mass in the


R.I.F. include:

A. Pedunculated myoma uteri


B. Burkitt’s lymphoma of the right ovary
C. Appendix mass
D. Right iliac adenitis
E. Retroperitoneal sarcoma

 The differential diagnosis of an appendix mass in a woman of 30


include:

A. Pedunculated myoma uteri


B. Carcinoma of the body of the uterus
C. Right ovarian cyst
D. Right hydronephrosis
E. Crohn’s disease
Seven days after appendicectomy for gangrenous appendix a boy
of 15 develops pyrexia, tachycardia, frequency and spurious

9
diarrhea.

 You will suspect most strongly:

A. Urinary tract infection


B. Pelvic abscess
C. Dysentery
D. Entero-colitis

 Pelvic peritonitis

 The most useful investigation will be:

A. Straight X-ray of the abdomen


B. Urine culture and routine examination
C. Stool examination and culture
D. White cell count and differential
E. Haemoglobin

 In pyeliphlebitis:

A. There is thrombosis of the hepatic vein


B. Rigors occur
C. Pain and tenderness over the liver occur
D. There is jaundice

10
 There may be subsequent liver abscesses
A tender mass in the R.I.F. in a 45-year old man is found in two
weeks to be half its size. There is no further reduction in size
during the next three weeks.

 The most likely clinical diagnosis is:

A. Resolving appendix mass


B. Resolving amoeboma of the caecum
C. Resolving ileo-caecal tuberculosis
D. Resolving actinomycosis
E. Carcinoma of the caecum with resolving paracaecal abscess

 The most useful investigation will be:

A. Full blood count and therapeutic trial with metronidazole


B. Mantoux test
C. Stool for amoebae and occult blood
D. Barium enema
E. Colonoscopy

 The treatment of uncomplicated acute appendicitis is:

A. Observation for 48h plus antibiotic therapy


B. Immediate appendicectomy with antibiotic prolylaxis
C. Antibiotics for 24h before appendicectomy
D. Conservative with the patient in Fowler’s position
E. Immediate appendicectomy plus full antibiotic and metronidazole
therapy

11
 The treatment of appendix mass /abscess is:

A. Immediate drainage of abscess and appendicectomy


B. Immediate drainage of abscess
C. Antibiotic therapy
D. Expectant, drainage of abscess being undertaken if there is
extension or lack
of progress
E. Expectant, drainage of abscess being done if the abscess or mass
does not
resolve in one week

 Contra-indications to immediate appendicectomy for acute


appendicitis include:

A. Presence of a mass/abscess
B. High temperature and tachycardia
C. Absence of guarding or rebound tenderness
D. Settling inflammation as shown by decreasing pain and tenderness
and near

12
normal pulse
E. Associated sickle cell disease

 In a patient with generalized peritonitis following acute


appendicitis the treatment will include:

A. Immediate laparotomy and appendicectomy


B. Aggressive intravenous fluid therapy and nasogastric
decompression
C. Administration of analgesics, antibiotics such as amikacin,
gentamicin or
kanamycin and ampicillin or cefotaxime
D. Laparotomy, appendicectomy and drainage of the peritoneum after
6-12h of
conservative management
E. Laparotomy and drainage of the peritoneum after 6-12h of
conservative
management

 Indications for drainage of an appendix abscess include:

A. Swinging temperature and/rising pulse rate


B. Slow regression of the abscess
C. A patient under 14 or over 40
D. Persistent or increasing or spreading pain with increasing or
spreading
tenderness, rebound tenderness and guarding

 Improvement in the general condition making the patient fit for


operation

 In the expectant management of an appendix mass/abscess:

A. Antibiotics are essential in all cases


B. An hourly pulse and 4-hourly temperature chart should be kept

13
C. An analgesic such as pethidine should be administered 6-8 hourly
D. The mass should, if possible, be examined twice daily and its size
and degree
and distribution of pain and tenderness observed
E. Intravenous fluid therapy is essential in all cases

 In the operation of appendicectomy, the appendix is found by:

A. Following the taenia to the lower end of the caecum


B. Bringing the caecum into the wound and examining its posterior
surface
C. Bringing a loop of ileum into the wound and following it to the
terminal part
D. Locating the ileo-caecal junction and following the medial border of
the
caecum downwards
E. Following the antero-lateral border of the caecum downwards

 Drainage of the peritoneal cavity is after appendicectomy for


acute appendicitis in the following situations:

A The appendix is very acutely inflamed

 The appendix is perforated or gangrenous

14
C. There is pus around the appendix
D. There is localized peritonitis without pus formation
E. The appendix is pelvic in position

 The commonest post-operative complication after


appendicectomy for acute appendicitis is:

A. Haemorrhage

 Paracaecal abscess
C. Paralytic ileus
D. Generalized peritonitis
E. Wound infection

 The mortality of acute appendicitis is:

A. Almost nil in non-perforated or non-gangrenous cases

 3-6% or about 12% when there is perforation or peritonitis


respectively
C. Negligible when there is a mass or abscess
D. Higher in children being 20-30% in infants under 2 years, and lower
in the
aged, being less than 2% in those over 40
E. About 1% and 1.5-2% overall in England and Ghana respectively

 Acute appendicitis:

A. The frequency is decreasing in Western countries

 The incidence is steadily increasing in developing countries


C. It now accounts for 10% of surgical emergencies in Accra
D. The incidence is higher in urban than in rural areas in developing
countries
E. It is as common in Nairobi as in Accra

15
 In obstructive acute appendicitis:

A. Fibrous adhesions or stricture of the lumen is the usual cause


B. Inflammatory oedema accounts for the rapid rise in intraluminal
pressure in
the obstructed segment
C. The part of the appendix proximal to the obstruction may or may not
be
inflamed
D. The increased tension causes venous stasis
E. The pain starts as a dull ache in the periumbilical region

 The of events in acute appendicitis include:

A. Spontaneous resolution

 Fatal termination from gangrene and/or perforation

 Formation of an appendix mass

 Development of an appendix abscess

16
 Recurrent acute attacks

 In acute appendicitis:

A. Anorexia is invariable
B. The pain may start in the R.I.F. and not the centre of the abdomen
C. Bowel function is usually normal
D. Boas’s sign may be positive
E. The psoas sign may be positive

 In acute appendicitis in the elderly:

A. The diagnosis can be difficult as the history may not be typical

 The pain is usually sharp but poorly localized


C. Tenderness is usually marked
D. The incidence of complications is high
E. The omentum is effective in walling of the infection

 In the investigations of acute appendicitis:

A. Leucocytosis is present in 90% of cases


B. High resolution ultrasonography improves the clinical diagnosis
C. The Alvarado scoring system may assist the diagnosis
D. Selective contrast-enhanced CT scanning may be cost-effective
especially in
the elderly
E. Urinalysis should be done to exclude diabetes mellitus and porphyria

 The differential diagnosis of acute appendicitis include:

A. Solitary acute diverticulitis of the caecum


B. Acute cholecystitis
C. Right acute epididymo-orchitis

17
D. Herpes Zoster
E. Sickle Cell crisis

 In appendix mass:

A. The general condition of the patient is generally good


B. Diarrheoa, from irritation of the terminal ileum, may be a feature
C. The temperature is swinging and the pulse elevated
D. The mass only be palpable under general anaesthesia in some
cases
E. The rest of the abdomen is soft and not tender

 In recurrent acute appendicitis:

A. The attacks are mild and recurrent

 The attacks are severe and recurrent


C. The appendix is usually fibrosed
D. The stool should be examined to exclude worm infestation
E. There is usually no tenderness in the R.I.F on deep palpation as the
caecum
is overlying the appendix

18
CHAPTER 33
THE APPENDIX
ANSWERS

 B, C 34.A, D

19
 D 35.B, C, D

 A, B, C, D, E 36. A, D

 D 37.B, D

 A, D 38. A, D

 C, E 39. B, C

 A, C, E 40. E

 A, B, D, E 41.A, B, C, E

 A,C, D 42.A, B, D, E

 B 43.C,D

 A, C 44. A, B, C, D, E

 B, D, E 45. A, B, E

 A, B, E 46. A, D

 B, C 47. A, B, C, D, E

 C, D 48. A, B, C, D, E

 E 49. A, D, E

 A, B 50. A, C, D

 C, E

 C, D

 B, E

 A, B, E

 B, C

 A, B, C, D, E

 A

 B, C, D, E

20
 A, C, D, E

 B

 D

 B, C, D, E

 E

 D

 B

 D

21
CHAPTER 34
ACUTE INTESTINAL OBSTRUCTION

 Acute intestinal obstruction is said to be:

A. High when the site of obstruction is relatively near to the pyloric


sphincter
B. Low when the site is relatively near to the ileo-caecal valve
C. A simple occlusion obstruction when only the lumen of the bowel is
occluded
D. A closed loop obstruction when nothing can escape from or enter
the loop proximally or distally

 A strangulated femoral hernia is:

A. A simple obstruction
B. A strangulation obstruction
C. A closed loop obstruction
D. The commonest cause of intestinal obstruction in women in West
Africa

 Gases in simple occlusion intestinal obstruction:

 Are derived mostly from putrefaction and fermentation of intestinal


contents by bacteria
B. Consist of nitrogen 70% carbon dioxide 6-9% oxygen 10% and other
gases10%
C. Are absorbed by the blood
D. Can pass through the obstruction distally

 Intraluminal pressure in acute intestinal obstruction:

1
A. Is raised by gases and intestinal secretions
B. May be 10mmHg in small bowel obstruction
C. May be 25mmHg in large bowel obstruction
D. Causes outpouring of fluid from the plasma into the lumen of the gut
and
peritoneum
E. Does not affect the blood supply

 In simple occlusion obstruction:

 Absorption of water, sodium and potassium by the obstructed bowel


ceases
by 12h

 The rate of secretion of water, sodium and potassium by the


obstructed bowel
increases significantly with time

 The fluid secreted by the obstructed bowel has a higher


concentration of
sodium than the normal bowel
D. The fluid secreted by the obstructed bowel has a normal
concentration of
Potassium
E. There is net absorption of water from the segment

2
 Death from simple occlusion intestinal obstruction is due
primarily to:

A. Peritonitis
B. Loss of water and electrolytes
C. Renal failure
D. Respiratory failure
E. Septicaemia

 Release of strangulation obstruction may lead to:

A. Severe septic shock


B. Septicaemia
C. Melaena
D. Hypokalaemia
E. Hypernatraemia

 In a strangulated hernia:

A. Blood is found in the hernia sac


B. Blood is not found in the wall of the obstructed gut
C. Blood is not found in the lumen of the gut
D. The mesenteric veins may be thrombosed

 The principle cause of death in strangulation obstruction is:

A. Peritonitis
B. Loss of water and electrolytes
C. Shock
D. Renal failure

3
E. Septicaemia

 In obstruction at the sigmoid due to stenosis, perforation is most


likely to occur in the:

A. Sigmoid proximal to the obstruction


B. Descending colon
C. Caecum
D. Terminal ileum
E. Transverse colon

 The most likely cause of death in closed loop obstruction of the


large bowel is:

A. Loss of water and electrolytes


B. Peritonitis
C. Renal failure
D. Shock from sequestration of blood
E. Acid-base disturbance

 The rapid distension of a closed loop obstruction of the colon


with a competent ileo-caecal valve is due to:

4
A. Ileal contents entering the caecum
B. Swallowed air
C. Fermentation of contents by bacteria
D. Diffusion of air from the plasma
E. Exudation of water and electrolytes from the plasma

 In acute mechanical intestinal obstruction:

A. Colicky pain in the hypogastrium suggests obstruction of the


terminal ileum
B. Faeculent vomitus is due to faeces
C. There need not be absolute constipation
D. Rebound tenderness and guarding suggest strangulation
obstruction
E. Bowel sounds are increased but of normal pitch

 In intestinal obstruction:

A. If a plain X-ray of the abdomen is requested it should be done


before a soap
and water enema
B. Concentric rings in the distended bowel in the supine film of the
abdomen
suggest jejunal or proximal ileal obstruction
C. The serum electrolytes may be normal
D. There may be metabolic alkalosis in low obstruction
E. The erect abdominal x-ray films always show fluid levels

 If all the following antibiotics were available, which of them would


you use in a patient with strangulation obstruction?

A. Kanamycin

B. Ciprofloxacin

5
C. Ampicillin
D. Metronidazole
E. Gentamicin

 Gastric aspiration in acute intestinal obstruction:

A. Removes regurgitating gastro-intestinal contents but not swallowed


air
B. Improves the tone of the obstructed bowel
C. Has no effect on the intestinal secretion of water and electrolytes in
the
obstructe bowel

 May cause alkalosis

 If at laparotomy for intestinal obstruction the caecum is found to


be distended, then the obstruction may be in the:

A. Terminal ileum
B. Transverse colon
C. Sigmoid colon
D. Jejunum
E. Rectum

 Concerning prognosis in acute intestinal obstruction:

6
A. The nature of obstruction is important
B. The cause of obstruction per se is not important
C. Low obstruction has a worse prognosis than high obstruction
D. Resection does not significantly raise the mortality
E. The child has a better chance than the young adult

 In black Africa:

A. Strangulated inguinal hernia is the commonest cause of intestinal


obstruction
B. Strangulated femoral hernia is common
C. Sigmoid volvulus is common in Kenya, Uganda and Nigeria
D. Carcinoma is an uncommon cause of obstruction
E. Adhesions causing obstruction are not common

 A strangulated indirect inguinal hernia:

A. Is most commonly constricted at the internal ring


B. May be constricted at the external ring and by the neck of the sac
C. Is equally common on the left and right
D. Is said to be of the Maydl’s type if only a part of the circumference is
strangulated

 Is unlikely to cause bowel gangrene

 Richter’s strangulation:

A. Is when in a big hernia, two loops of bowel in the sac as well as the
intervening loop in the abdomen are strangulated
B. Is more common in strangulated femoral hernia
C. May not be associated with constipation
D. May, if untreated, lead to entero-cutaneous fistula
E. May not be associated with vomiting

7
 In a strangulated hernia containing small bowel:

A. There is constant pain in the hernia


B. There is no associated colicky abdominal pain
C. The hernia is tense because of the contained bowel
D. The hernia is tender because of the congestion of the bowel
E. There is no cough impulse because the communication with the
abdomen at
the neck is occluded

 Expectant treatment of strangulated inguinal hernia:

A. Is permissible if the hernia is seen within 10h


B. Requires elevation of the foot of the bed
C. Requires administration of morphine 15mg
D. Requires taxis, i.e. manual reduction of the hernia when the pain
subsides
E. Has the advantage that the patient can be sent home as soon as the
hernia is
reduced

 A loop of bowel may be strangulated in:

A. Epiploic foramen

8
B. An acquired defect in the mesentery
C. Broad ligament
D. Base of sigmoid mesocolon
E. Retroperitoneal fossa at the duodeno-jejunal junction
A 60-year old woman has colicky central abdominal pain of 12h
associated with vomiting and absolute constipation. She also has
pain on the medial aspect of the left thigh.

 The most likely clinical diagnosis is:

A. Strangulated left femoral hernia


B. Inflammation of the left tube
C. Strangulated bowel in the broad ligament
D. Strangulated left obturator hernia
E. Twisted left ovarian cyst
A 40-year old woman who had a Caesarian section 2-years
previously, complains central colicky abdominal pain with
vomiting and absolute constipation. Examination reveals a dry
tongue, pulse 84/min, BP 110/80, moderately distended abdomen
without any area of tenderness, rebound tenderness or guarding.
There is a median sub-umbilical scar.

 You will advise:

A. Intravenous fluids and electrolytes


B. Nasogastric decompression
C. Intake-output chart
D. Administration of pethidine with reassessment of the patient before
the next
dose is administered
E. Laparotomy as soon as possible

9
 Indications for laparotomy during expectant management of
simple occlusion obstruction caused by post-operative adhesions
include:

A. Increasing abdominal distension


B. Absence of pain
C. Increasing nasogastric aspirations
D. Failure to pass flatus or faeces
E. Change of colour of nasogastric aspiration from brown to green

 Clinical features distinguishing strangulation obstruction from


simple occlusion include:

A. Persistence of pain between spasms of colicky abdominal pain


B. Profuse and early vomiting
C. Rapid pulse and low blood pressure
D. Localized abdominal tenderness, rebound tenderness or guarding

10
E. Sunken eyes, dry inelastic skin

 Volvulus of the sigmoid:

A. Is usually in a clockwise direction


B. Is associated with a long, dilated, redundant, often thick-walled
sigmoid
hanging on a long mesentery
C. Usually has a wide meso-colon attached to the posterior wall
D. Is primarily a strangulation obstruction
E. Distends rapidly with gas mainly from bacterial putrefaction of
faeces

 Volvulus of the sigmoid:

A. Is equally common in the sexes


B. Is characterized by intermittent crampy pain mainly in the lower
abdomen with
gross distension
C. Causes early vomiting and absolute constipation
D. In a straight X-ray, may be depicted by three curved lines meeting at
a diffuse
lower point

 Invariably presents with sigmoid strangulation

 Volvulus of the sigmoid may be relieved by:

A. Enema saponis
B. Sigmoidoscopy and passage of a rectal tube
C. Laparotomy and detorsion of the bowel of the bowel
D. Laparotomy and resection of the sigmoid
E. Sedation and use of I/V smooth muscle relaxants

11
 Intussusception:

A. Is composed of two layers of intestinal wall


B. In over 90% of patients occurs between 3 months and 2 years most
commonly
between 6 and 9 months
C. Among the Yorubas, is equally common among adults and children
and the
sexes
D. In adults, is nearly always secondary to a polyp or tumour more
often
malignant than simple
E. In children, is twice as common in males

 In intussusception:

A. The blood supply is compressed between the returning and


receiving layers
B. The lumen may be occluded by marked swelling of the neck
C. The process most commonly starts in the last 50cm of the ileum

12
D. The commonest type is the ileo-colic except among the Yorubas in
whom the
caeco-colic is the commonest
E. The intusscepting bowel does not reach the anus

 In intussusceptions:

A. The baby is usually malnourished


B. The diagnostic features consist of colicky abdominal pain with or
without
vomiting, rectal bleeding and a palpable lump
C. Gastroenteritis has to be differentiated
D. The definitive treatment is by operative reduction but reduction by
hydrostatic
pressure is to be preferred as it gives certain results
E. The pain is so severe but does to present with shock

 The commonest tumour of the small intestine causing intestinal


obstruction is:

A. Adenoma
B. Leiomyoma
C. Carcinoma
D. Lipoma
E. Intestinal polyposis

 Intestinal obstruction:

A. By carcinoma of the large bowel accounts for 8% of obstruction in


West Africa
B. If caused by carcinoma in the right colon is best treated by a right
hemicolectomy

 If caused by carcinoma of the sigmoid colon is best relieved by

13
sigmoid
colectomy

 If caused by carcinoma of the rectum is best relieved by a


transverse
colostomy

 The straight X-ray of the abdomen of a 10-year old boy shows air-
fluid levels mainly in the central part. There are also translucent
entangled lines in the right lower part.

The most likely diagnosis is:

A. Intussusception
B. Faecal impaction
C. Intestinal obstruction by foreign bodies
D. Intestinal obstruction by ascaris lumbricoides
E. Intestinal obstruction of unknown cause

A 65-year old man complains of sudden severe, persistent, cramp-


like central abdominal pain, vomiting and bloody dirrhoea. His
pulse is 120/min and irregular and the BP 80/50. The abdomen is
distended, tender and guarded and the bowel sounds are absent.

14
 The most likely clinical diagnosis is:

A. Intussusceptions
B. Myocardial infarction
C. Mesenteric vascular occlusion
D. Strangulation obstruction of unknown cause
E. Ruptured aortic aneurysm

 Paralytic ileus may be caused by:

A. Toxic paralysis of the mesenteric plexus by inflammation


B. Increased in plasma motilin pre and per-operatively
C. Retroperitoneal haemorrhage
D. Depression of neuromuscular excitation by hypokalaemia,
hypomagnesaemia
or hyponatraemia

 Uraemia

 Clinical features of paralytic ileus include:

A. Cramp-like central abdominal pain with profuse vomiting


B. Absolute constipation
C. Increasing tender abdominal distension
D. Low-pitched bowel sounds
E. Coffee ground vomiting

 Treatment of post-operative paralytic ileus includes:

A. Nasogastric decompression
B. Purgation
C. Soap and water enema
D. Intravenous fluids and electrolytes

15
E. Laparotomy

 Drugs that may be used in the treatment of paralytic ileus include:

A. Guanethidine
B. Neostigmine
C. Adrenaline
D. Propranolol
E. Chlorpromazine

 Common causes of neonatal intestinal obstruction include:

A. Intestinal atresia and stenosis


B. Anomalies of rotation
C. Annular pancreas
D. Strangulated inguinal hernia
E. Anorectal anomalies

 Intestinal atresia:

A. Is commonest in the duodenum and least common in the ileum


B. May be a complete or partial septum across the lumen
C. May result from in-utero aseptic necrosis and fibrosis caused by
ischaemia
D. May be due to developmental error in the first few weeks of

16
gestation
E. Is associated with other major malformations in most patients

 Meconium ileus:

A. Occurs in 50% of patients with mucoviscidosis


B. Is a simple occlusion obstruction
C. Is determined by a dominant gene
D. May lead to perforation

 The cardinal features of neonatal intestinal obstruction include:

A. Abdominal distension
B. Vomiting which is always bilious
C. Failure to pass meconium in 36 hours
D. Oedema of the abdominal wall

 In suspected neonatal intestinal obstruction, if an erect X-ray of


the abdomen shows:

A. “Double bubble” gas fluid shadow, the diagnosis is duodenal atresia


B. Multiple air-fluid levels, the diagnosis is jejunal or ileal atresia
C. Distended small bowel and ground glass appearance, the diagnosis
is
meconium ileus
D. Distended small bowel and soap bubble appearance, the diagnosis
is ileal
atresia

17
 In acute intestinal obstruction:

A. Closed loop obstruction occurs in its pure form in volvulus of the


sigmoid colon
B. The site of obstruction is unrelated to the ampula of Vater
C. The Friman-Dahl lines are a feature of sigmoid volvulus on supine
abdominal
X-ray
D. There is relatively early onset of distension in high intestinal
obstruction
E. Strangulation is always associated with occlusion of the bowel
lumen

 In the pathophysiology of acute intestinal obstruction:

 The secretion of water, sodium and potassium by the obstructed


bowel
increases steadly and significantly with time

 Rising intraluminal pressure may reach 25mmHg in the small bowel


and
25mmHg in the large bowel

 Dehydration is early in high intestinal obstruction but electrolyte


imbalance is
delayed because of the large absorptive area

18
 Release of the obstruction in strangulation obstruction may lead to
severe
endotoxic shock

 Hypoxia inhibits the growth of Clostridia and Bacteroides


A 50 year old man with atrial fibrillation presents with a history of
sudden onset of severe central abdominal pain. The pain is out of
proportion with the physical findings. He claims to have passed
altered blood per rectum. His F.B.C. reveals a profound neutrophil
leucocytosis.

 The most likely diagnosis is:

A. Carcinoma of the large bowel


B. Mesenteric vascular occlusion
C. Crohn’s disease of the colon
D. Amoebic colitis
E. Bacillary dysentery

CHAPTER 34
ACUTE INTESTINAL OBSTRUCTION
ANSWERS

19
 C, D 40. B,C

 A, B, C 41. A,D

 B 42. A, B, E

 A, B, C, D 43. A, B, E

 A, B, C 44. B, C, D

 B 45. D

 A, B, C 46. A, C

 A, D 47. A, B, C

 C 48. C

 C 49. A, B, D

 B 50. B

 C

 C, D

 A, B, C

 B, D

 B, D

 B, C, E

 A

 A, D

 A, B

 B, C, D

 A, D, E

 B, C

 A, B, C, D, E

20
 D

 A, B, C, D

 A, C, D

 A, C, D

 B, E

 B, D

 A, B, C, D

 B, C, E

 C, D

 B, C

 D

 B, C, D

 D

 C

 A, C, D, E

21
CH
APT
ER
35

ACUTE SALPINGITIS AND ECTOPIC


PREGNANCY

1. The uterine (Fallopian) tubes:


A. Are about 15 cm long

B. Are attached to the corpus uteri near the upper


part

C. Have a short interstitial portion embedded in the myometrium and communicating


with the uterine cavity by a wide opening

D. Have the uterine vessels as their main blood


supply

E. Have a mucous membrane consisting of ciliated columnar epithelium and thrown


into branching folds which almost obliterate the lumen.

2. Acute salpingitis may be


caused by:

A. N. gonorrhea

B. Esch. coli

C. Chlamydia trachomatis

D. Streptococcus

E. Staph. aureus

1
3. Acute salpingitis may follow:
A. Laparoscopy

B. Insertion of I.U.D.

C. Cauterization of the cervix

D. Hysterosalpingography
E. Normal delivery
4. In gonorrhea:

A. Infection of the urethra precedes that of the vulvo-vaginal glands and


cervix

B. Infection spreads from the cervix to the uterine tubes via the lymphatics
and blood vessesls

C. Pelvic periotonitis follows leakage of exudates from the tubes via the
lymphatics and blood vessles
C. The endometrium is rapidly infected directly from the cervix but the infection is
transient

E. Infecetion of the cervix takes a long


time to resolve

5. After abortion:

A. The placental site is always infected

B. Infection usually spreads directly from the placental site to the


endosalpinx

C. Infecetion spreads from the endometrium to the parametrial tissues, broad


ligament and
peritoneum via the lymphatics and blood vessels

D. Spread of infection to the adnexa is


usually slow

E. Infecetion resolves fairly rapidly

2
6. Acute salpingitis:
A. Is the commonest cause of peritonitis in the female
in West Africa

B. Accounts for about 6% of


gynaecological disease

C Is commonly secondary to peritonitis from


acute appendicitis

D. Occurs in all age groups

E. Usually affects one tube

7. In gonococcal salpingitis:

A. The mucosa may be almost normal

B. The inflammatory process is confined to the


muscular wall

C. The fimbriae of the Fallopian tubes may be


destroyed
D. The outer end of the tube may be blocked

E. Pyosalpinx may result

8. In post-abortal salpingitis:

A. There is intense inflammation and


ulceration of the mucosa

B. The wall of the tubes is thickened

C. Peritubal and peri-ovarian adhesions


are formed

D. Blockage of the outer end of the tube with resulting


hydrosalpinx is the rule

E. The pathological lesion is essentially


an endosalpingitis

3
9. Clinical features of acute salpingitis
include:

A. Delayed or precipitated early menstruation which may be heavy


and prolonged
B. Gradual onset of diffuse lower abdominal pain which settles in both iliac fossae or
just bove
the pubis
C. Malaise, pyrexia persistent vomiting and absolute constipation
D. Distended lower abdomen with tenderness but not rebound
tenderness or guarding

E. Tenderness in both fornices and pain on cervical excitation on


bimanual examination

10. The most useful investigation in acute


salpingitis is:

A. White cell count and differential

B. ES.R.

C. A cervical (or high vaginal) swab

D. Urethral swab

E. Blood culture

11. Conditions that may be confused with acute salpingitis


include:

4
A. Perforated duodenal ulcer

B. Cystitis

C. Rupture of corpus luteal cyst

D. Acute appendicitis

E. Torsion of a pedunculated fibroid

12. Complications in acute


salpingitis inclue:

A. Intestinal
obstruction

B
Pyelonephritis

C. Tubal
pregnancy

D. Renal
failutre

E. Ovarian

13. Measures taken in the definitive treatment of acute


salpingitis indluce:

A. Bed rest

B.
Administration of
analgesics

C.
Administration of
antibiotics

5
D. Vaginal
douches

E. Laparotomy

14. Ectopic
pregnancy:

A Is a condition in which a fertilized ovum is implanted in the uterine tube


and not in the body
of the utuerus or ovary

B. Occurs most commonly in the ampulla which is also the


least dangerous site

C. In the isthmus is dangerous although rupture and severe haemorrhage are


not inevitable
D. Is 40 times commoner in women who have had endosalpingitis than in the
normal population

E. Has an incidence of about 1 in 28 pregnancies in the West Indies and in 35


in West Africa

15. Aetiological factors in ectopic


pregnancy include:

A Previous
tubal reconstruction

B. Intra-uterine contraceptive device

C. Oligomenorrhoea

D. Leucorrhoea

E. Previous ectopic pregnancy

6
16. Aetiological factors in ectopic
pregnancy include:

A. Previous enteric fever with or


without complication

B. Previous
puerperal sepsis

C. Myomata
uteri

D. Ovarian cyst

E. Recurrent
urinary tract
infection

17. In ectopic
pregnancy:

A. There is decidual reaction in the


endosalpinx of the tube

B. The vessels of the tube are eroded with


extravasations of blood

C. The uterus does


not increase in size

D. There is a decidual reaction in


the endometrium

E. There is no Change in the


vasularity of the uterus

18. Following abortion of the embryo into the lumen of the tube in tubal
pregnancy, there
may be:

7
A. Incomplete
abortion

B. Complete
abortion
C. Absorption of
the ambryo

D. Tubal mole
formation

E. Peritubal or
pelvic haematocele

19. Rapidly fatal heavy and prolonged haemorrhage usually follows rupture
of:

A. Ovarian
pregnancy

B. Interstitial
pregnancy

C. Fimbrial pregnancy
D. Isthmial pregnancy

E. Ampullary pregnancy

20. The clinical features of the acute type of ruptured tubal


pregnancy include:

A. An invariable history of 6-8 weeks amenorrhoea and


morning sickness

B. Sudden severe camping pain which starts in the hypogastrium or an iliac fossa
and becomes
generalized

C. Vaginal bleeding which precedes the abdominal


pain

D. Distended ternder abdomen with rebound tenderness and rigidity especially


in the lower part

8
E. Exquisiste tenderness in the affected fornix on bi-
manual examination.

21. Clinical features in subacute-chronic type of ruptured tubal


pregnancy include:

A. Vague or colicky and recurrent lower abdominal pain which may be


confined to an iliac fossa
B. Vaginal bleeding, the blood being small in quantity and dark and
preceded by the pain

C. Pallor, rapid pulse and hypotension

D. Distended tender lower abdomen with rebound


tenderness and rigidity

E. Positive cervical excitation, exquisitely tender fornix with


usually a palpable mass

22. Clinical features of ruptured tubal pregnancy


may include:

A. Retention of urine

B. A normal uterus

C. Irregular vaginal bleeding

D. A large tender mass in the pouch or Douglas

E. A history of collapse

23. Useful aid(s) to a definite diagnosis in difficult cases of subacute or chronic


type of
rupturedtubal pregnancy is/are:
A. Biological test for pregnancy

9
B.
Culdocentesis

C. Pelvic examination
under anaesthesia

D. Laparoscop
y of the
pelvis
E. Curettage

24. Operative procedures in ruptured tubal


pregnancy include:

A.
Salpingostomy
B.
Salpingectomy
C. Partial
salpingectomy

D.
Salpingo-
oophorectomy

E.
Salpingo-
hysterectomy

25. Complications of ruptured tubal


pregnancy include:

A.
Retention of
urine

B.
Intestinal
Obstruction

10
C. Urinary
tract infection

D. Pelvic
abscess

E. Acute
renal failure

26. At what gestational age do tubal ectopic pregnancies, usually present


clinical
symptoms?
A. 2 weeks

8. 4 weeks

C. 6 weeks

D. 8 weeks
E. varies with location

27. What is the etiology for vaginal bleeding in cases of ectopic


pregnancy?
A. Coagulopathy
B. Sloughing of deciduas
C. Bleeding from the tube
D. Progesterone excess
E. Trauma

28. A culdocentesis that obtains 3ml of straw-coloured fluid would be


considered
A. Negative
B Positive
C. Nondiagnostic
D. Unsatisfactory
E Need for laparoscopy

11
29. Spielgelberg's criteria for ovarian pregnancy exclude:
A. An inteact fallopian tube
B. An ovary of normal position
C. Ovarian tissue in the wall of gestational sac
D. A fallopian tube connected to the uterus by the ovarian ligament
E. Normal Fimbrial ovarica

30. All of the following are required to establish a diagnosis of acute


salpingitis except
A. Cervical tenderness
B. Temperature greater than 360C
c. Adnexal tenderness
D. Direcet abdominal tenderness
E. Laparoscopic evidence of inflamed tubes

31. The initial treatment for presumed pelvic inflammatory


disease is based on

A.
Clinica
l
suspici
on

B.
Cervic
al
gram
stain

C.
Anaer
obic
Cultur
e

D. While cell
count less than
800

E.

12
Reboun
d
tendern
ess

32. A common sequel of infection by Clamydia


trachomatis is

A. A
recurrent
vulvar
growths

B. Cyclic
migratory
arthralgia

C. Involuntary infertility

D.
Vagini
smus

E.
Endoc

13
arditis

33. Antibodies to Clamydia are found in approximately what percentage of


sexually active
women?
A. 1-5%

B. 10-15%

C.
20-
40%

D.
60-
80%

E.
Greate
r than
90%

34. Following initial infection by Neiseria gonorrhea, symptoms


first appear in

A.

1-2
days

B. 3-5
d
a
y
s

C. 6-7
d
a
y

14
s

D. 8-
1
0

d
a
y
s

E.

More
than 6
weeks

35. The most frequent site of infecetion with gonorrhea in women is the
A. Bartholin's glands
B. Skene's glands
C Cervix
D. Urethra
E Rectal crypts

KEY
. 1. E

2. A,B,C,D,E

3. B,C,D,E

4. C,D

5. C

6. A

7. C,D,E

8. B,C

9. A,E

10. C

11. A,B,C,D,E

12. A,C

13. A,B,C 15
14. B,D,E

15. A,B,E

16. B,C,D
23. D

24. A,C,D

25. A,B, C,D,E

26. E
27. B

~
A
28.
29. D

30. B

31. A

32. C

33. C

34. B
35. C

16
CHAPTER 36
PERITONEUM, OMENTUN, MESENTERY AND
RETROPERITONEUM
 Ascites:

 Is clinically detectable in the adult when 1000ml of fluid is present in


the peritoneal cavity

 Gives rise to fluid thrill and localized dullness of the abdomen which
is persistent even when the patient changes his position

 In a child may be caused by Burkitt’s lymphoma

 On straight X-ray of the abdomen shows air-fluid levels

 Is nearly always associated with dyspnoea

 If ascetic fluid is an exudate it:

 Contains leucocytes

 Is protein-rich

 Has a low specific gravity

 Contains red cells

 Is frothy as aspirated

 Causes of an exudate in the peritoneal cavity include:

 Decreased colloid osmotic pressure of plasma

 Increased capillary permeability

 Increased capillary hydrostatic pressure

 Increased capillary filtration pressure

1
 Hyperaldosteronism

 Causes of ascites in carcinomatosis peritonei include:

 Mild chemical inflammation of the peritoneum

 Hypoproteinaemia

 Retention of sodium

 Obstruction of subserous lymphatics by tumour cells

 Obstruction of capillaries by tumour cells

 Blood-stained ascetic fluid is suggestive of:

 Nephrotic syndrome

 Cirrhosis of the liver

 Abdominal tuberculosis

 Fibroma of the ovary

 Disseminated carcinoma of the peritoneum

 Straw-coloured ascetic fluid may be due to:

 Congestive cardiac failure

 Cirrhosis of the liver

 Carcinoma of an abdominal viscus

 Abdominal tuberculosis

 Lymphoma

 Causes of ascites in cirrhosis of the liver include:

2
 Increased flow of lymph in the hepatic hilar lymphatics

 Increased capillary permeability

 Increased intrahepatic pressure

 Hyperaldosteronism

 Hypoglobinaemia

 Ascites without evidence of oedema in other parts of the body


may

be due to:

 Cirrhosis of the liver

 Chronic renal failure

 Carcinomatosis peritonei

 Congestive cardiac failure

 Abdominal tuberculosis

 Ascites and associated hydrothorax without evidence of


associated

oedema in other parts of the body in a 40-year old woman may


be

due to:

 Malnutrition

 Thecoma of the ovary

 Mesothelioma of the pleura

 Constrictive pericarditis

 Fibroma of the ovary

 In a 20 year old woman with ascites and amenorrhoea for 2


months, anaemia, relative lymphocytosis and a raised ESR

3
support a diagnosis of:

 Ovarian carcinoma

 Meig’s syndrome

 Tuberculous peritonitis

 E coli peritonitis

 Chronic salpingitis

 Fluid from tuberculous ascites:

 Abounds with lymphocytes-over 250/ml

 Abounds with polymorphonuclear leucocytes

 Has protein concentration of less than 35 g/l

 Has a low specific gravity

 Is not straw coloured

 Pseudomyxoma peritonel:

 Is a condition in which the peritoneal cavity is filled with bluish


gelatinous secretion of mucin

 Arises generally from adenoma or adenocarcinoma of the appendix

and occasionally from pseudo-mucinous cystadenoma of the ovary

 May cause intestinal obstruction

 Is usually diagnosed pre-operatively

 Tends not to recur after laparotomy and washing of the peritoneum

4
with saline followed by instillation of mycomycin C

 Acute non-specific mesenteric lymphadenitis:

 Is more common in childhood and rare after puberty

 Is probably caused by arbovirus

 Causes cellular hyperplasia of the mesenteric nodes which are


initially enlarged, pink and soft and later white and firm

 Causes injection of the small intestine, caecum and appendix

 Is treated by appendicectomy and excision of the affected lymph


nodes

 The features of non-specific mesenteric lymphadenitis include:

 Previous upper respiratory infection in most patients

 Centrally located abdominal pain and guarding

 Tenderness in the lower half of the abdomen

 Invariable leucocytosis of 15,000 or over on the first day

 Pyrexia of about 38˚C C usually

A 10-year old girl in Freetown complains of a painless abdominal


lump for 3 months. The lump is found to be in the centre of the
abdomen, about 10 cm in diameter, cystic, smooth-surfaced and
non-tender, and moves upwards and downwards but not
sidewards:

 The most likely clinical diagnosis is:

 Ovarian cyst

5
 Omental cyst

 Pancreatic cyst

 Enterogenous cyst

 Renal cyst

 Investigations to consider include:

 Gastro-intestinal barium series

 Cystoscopy

 Intravenous pyelogram

 Laparoscopy

 Aortograpy

 Complications of omental cyst include:

 Torsion

 Haemorrhage

 Rupture

 Infection

 Calcification
A 50-year old woman complains of a 4-month slightly painful
abdominal mass. The mass is about 5 cm wide and 10 cm long
and located on either side of the midline about the mid-abdomen.
It is hard and immobile and has indefinite edges and nodular
surface:

 It may be:

 Carcinoma of the pancreas

 Carcinoma of the transverse colon

 Retroperitoneal liposarcome

6
 Retroperitoneal lymphoma

 Retroperitoneal neurofibroma

 The diagnosis will be established by:

 G.I. Barium series

 Aortograpy

 Laparotomy and biopsy

 Laparotomy

 I.V.P.

 Torsion of the omentum:

 Is a fairly common emergency

 Usually occurs in non-obese patients between 30 and 50

 Is usually secondary to adhesion of omentum to the peritoneum

or hernia sac

 Is usually mistaken for acute appendicitis

 It treated by untwisting it

 Radical treatment of pseudomyxoma peritonei requires the


following:

 Greater and lesser omentectomy with splenectomy

 Cholecystectomy, antrectomy, right hemicolectomy

 Pelvic and diseased anterior parietal peritonectomy

7
 Left hemicolectomy, stripping of the diaphragm

 Intra-operative, intra peritoneal infusion of mitomycin

CHAPTER 36
PERITONEUM, OMENTUN, MESENTERY AND
RETROPERITONEUM
ANSWERS

 C

 A, B

8
 B

 A, B, D

 E

 A, B, C, D, E

 A, C, D

 A, C, E

 B, D, E

 C

 A

 A, C

 A, C

 B, D

 D

 A, C

 A, B, C, D, E

 C, D, E

 C

 C, D

 A, B, C, E

9
CHAPTER 36
PERITONEUM, OMENTUN, MESENTERY AND
RETROPERITONEUM
 Ascites:

 Is clinically detectable in the adult when 1000ml of fluid is present in


the peritoneal cavity

 Gives rise to fluid thrill and localized dullness of the abdomen which
is persistent even when the patient changes his position

 In a child may be caused by Burkitt’s lymphoma

 On straight X-ray of the abdomen shows air-fluid levels

 Is nearly always associated with dyspnoea

 If ascetic fluid is an exudate it:

 Contains leucocytes

 Is protein-rich

 Has a low specific gravity

 Contains red cells

 Is frothy as aspirated

 Causes of an exudate in the peritoneal cavity include:

 Decreased colloid osmotic pressure of plasma

 Increased capillary permeability

 Increased capillary hydrostatic pressure

 Increased capillary filtration pressure

1
 Hyperaldosteronism

 Causes of ascites in carcinomatosis peritonei include:

 Mild chemical inflammation of the peritoneum

 Hypoproteinaemia

 Retention of sodium

 Obstruction of subserous lymphatics by tumour cells

 Obstruction of capillaries by tumour cells

 Blood-stained ascetic fluid is suggestive of:

 Nephrotic syndrome

 Cirrhosis of the liver

 Abdominal tuberculosis

 Fibroma of the ovary

 Disseminated carcinoma of the peritoneum

 Straw-coloured ascetic fluid may be due to:

 Congestive cardiac failure

 Cirrhosis of the liver

 Carcinoma of an abdominal viscus

 Abdominal tuberculosis

 Lymphoma

 Causes of ascites in cirrhosis of the liver include:

2
 Increased flow of lymph in the hepatic hilar lymphatics

 Increased capillary permeability

 Increased intrahepatic pressure

 Hyperaldosteronism

 Hypoglobinaemia

 Ascites without evidence of oedema in other parts of the body


may

be due to:

 Cirrhosis of the liver

 Chronic renal failure

 Carcinomatosis peritonei

 Congestive cardiac failure

 Abdominal tuberculosis

 Ascites and associated hydrothorax without evidence of


associated

oedema in other parts of the body in a 40-year old woman may


be

due to:

 Malnutrition

 Thecoma of the ovary

 Mesothelioma of the pleura

 Constrictive pericarditis

 Fibroma of the ovary

 In a 20 year old woman with ascites and amenorrhoea for 2


months, anaemia, relative lymphocytosis and a raised ESR

3
support a diagnosis of:

 Ovarian carcinoma

 Meig’s syndrome

 Tuberculous peritonitis

 E coli peritonitis

 Chronic salpingitis

 Fluid from tuberculous ascites:

 Abounds with lymphocytes-over 250/ml

 Abounds with polymorphonuclear leucocytes

 Has protein concentration of less than 35 g/l

 Has a low specific gravity

 Is not straw coloured

 Pseudomyxoma peritonel:

 Is a condition in which the peritoneal cavity is filled with bluish


gelatinous secretion of mucin

 Arises generally from adenoma or adenocarcinoma of the appendix

and occasionally from pseudo-mucinous cystadenoma of the ovary

 May cause intestinal obstruction

 Is usually diagnosed pre-operatively

 Tends not to recur after laparotomy and washing of the peritoneum

4
with saline followed by instillation of mycomycin C

 Acute non-specific mesenteric lymphadenitis:

 Is more common in childhood and rare after puberty

 Is probably caused by arbovirus

 Causes cellular hyperplasia of the mesenteric nodes which are


initially enlarged, pink and soft and later white and firm

 Causes injection of the small intestine, caecum and appendix

 Is treated by appendicectomy and excision of the affected lymph


nodes

 The features of non-specific mesenteric lymphadenitis include:

 Previous upper respiratory infection in most patients

 Centrally located abdominal pain and guarding

 Tenderness in the lower half of the abdomen

 Invariable leucocytosis of 15,000 or over on the first day

 Pyrexia of about 38˚C C usually

A 10-year old girl in Freetown complains of a painless abdominal


lump for 3 months. The lump is found to be in the centre of the
abdomen, about 10 cm in diameter, cystic, smooth-surfaced and
non-tender, and moves upwards and downwards but not
sidewards:

 The most likely clinical diagnosis is:

 Ovarian cyst

5
 Omental cyst

 Pancreatic cyst

 Enterogenous cyst

 Renal cyst

 Investigations to consider include:

 Gastro-intestinal barium series

 Cystoscopy

 Intravenous pyelogram

 Laparoscopy

 Aortograpy

 Complications of omental cyst include:

 Torsion

 Haemorrhage

 Rupture

 Infection

 Calcification
A 50-year old woman complains of a 4-month slightly painful
abdominal mass. The mass is about 5 cm wide and 10 cm long
and located on either side of the midline about the mid-abdomen.
It is hard and immobile and has indefinite edges and nodular
surface:

 It may be:

 Carcinoma of the pancreas

 Carcinoma of the transverse colon

 Retroperitoneal liposarcome

6
 Retroperitoneal lymphoma

 Retroperitoneal neurofibroma

 The diagnosis will be established by:

 G.I. Barium series

 Aortograpy

 Laparotomy and biopsy

 Laparotomy

 I.V.P.

 Torsion of the omentum:

 Is a fairly common emergency

 Usually occurs in non-obese patients between 30 and 50

 Is usually secondary to adhesion of omentum to the peritoneum

or hernia sac

 Is usually mistaken for acute appendicitis

 It treated by untwisting it

 Radical treatment of pseudomyxoma peritonei requires the


following:

 Greater and lesser omentectomy with splenectomy

 Cholecystectomy, antrectomy, right hemicolectomy

 Pelvic and diseased anterior parietal peritonectomy

7
 Left hemicolectomy, stripping of the diaphragm

 Intra-operative, intra peritoneal infusion of mitomycin

CHAPTER 36
PERITONEUM, OMENTUN, MESENTERY AND
RETROPERITONEUM
ANSWERS

 C

 A, B

8
 B

 A, B, D

 E

 A, B, C, D, E

 A, C, D

 A, C, E

 B, D, E

 C

 A

 A, C

 A, C

 B, D

 D

 A, C

 A, B, C, D, E

 C, D, E

 C

 C, D

 A, B, C, E

9
CHAPTER 38
SMALL AND LARGE INTESTINES
(Including Rectum and Anus)

 In the anatomy of the duodenum:

 The duodenum is retroperitoneal and immobile except for the first


part which is

completely covered with peritoneum

 The first part is related posteriorly directly to the gastroduodenal


artery, common

bile duct, portal vein and inferior vena cava

 Because of the relationship with the gallbladder, a large gallstone


may ulcerate

through the wall of the second part and enter the duodenum

 The second part may be damaged during operations on the right


kidney or right

ureter and in right hemicolectomy

 The third part crosses the superior mesenteric vessels

 In the anatomy of the small intestine:

A. The entire small intestine is about 280cm long

B. The lumen widens and the circular folds of the mucosa become
smaller and less

frequent distally

 Aggregation of lymph follicles is marked throughout

1
 The mesenteric vessels to the ileum are shorter but have more
arcades than those

of the jejunum

E. The mesentery to the ileum is thicker than that to the jejunum

 In the anatomy of the small intestine:

A. Brunner’s glands extend deep to the muscular layer and produce


alkaline-rich mucus

B. The absorptive surface of the columnar epithelium is estimated at


300m2

C. The villi are 1-3mm long and there are 20-40/mm2

D. There are in the outer cell membrane enzymes for the digestion of
carbohydrates, polypeptides, nucleic acid and fat

E. The mucosal cells are shed every 5-6 days and originate in the
crypts of Lieberkuhn

 In the anatomy of the large intestine:

A. The entire large intestine is about 150cm long but the taenia coli are
about

25cm shorter

 The vessels perforate the muscular coat at the appendices


epiploica which are

fat-filled projections on the lateral surface

 The caecum and the appendix are completely peritonealized and so


volvulus of

the caecum may occur

 The transverse colon is completely peritonealized and has a long


mesentery but

volvulus does not occur

 There is no free anastomosis between the vessels supplying the

2
various parts

 The rectum:

 Extends from the pelvi-rectal junction at the level of the first sacral
vertebra

to the ano-rectal ring and is 12.5 cm long

 Is supplied by the superior and inferior haemorrhoidal vessels

 Is completely extraperitoneal in its lower third

 In its lower third is related anteriorly to the uterus and vagina in the
female

 In its lower third is related posteriorly to the lower sacral nerves

 The pectinate line:

A. Is in the mid-part of the mucosa of the anus

B. Represents the proctodeal (anal) membrane

C. Is formed by valve-like folds enclosing crypts opening caudad

D. Has above it epithelium of transitional cells

E. Has below it epithelium of squamous cells

 In the anatomy of the anus:

A. Division of the internal sphincter or the anorectal ring leads to anal


incontinence

B. A lesion in any part is painful

3
C. The lymphatic drainage is all to the inguinal lymph nodes

D. The superficial external sphincter is attached to the skin and the


coccyx

E. The blood supply is from the inferior haemorrhoidal vessels

 In the nerve supply of the large bowel:

A. Sensory fibres are found only in the sympathetic fibres

B. The sympathetic nerves originating from T7-T12 and their post-


ganglionic fibres

supply the large bowel up to the splenic flexure

C. The vagus and the nervi erigentes from S2-S2 provide the
parasympathetic supply

D. The sympathetic supply to the rectum is from the hypogastric nerves

E. The sympathetic is secreto-motor

 Circulating peptide hormones include:

A. Secretin

B. Vaso-active intestinal polypeptide

C. Gastric inhibitory polypeptide

D. Enkephalin

E. Glucagon

4
 Succus entericus:

A. Has a Ph of 5-7

B. Contains amylase and enterokinase

C. Contains lipase

D. Contains invertase, lactase and maltase

E. Contains peptidases

 Gastric secretion is inhibited by:

A. Motilin

B. Vaso-active intestinal polypeptide

C. Enkephalin

D. Bombesin

E. G.I.P.

 CCK-PZ:

A. Is found in the duodenum, jejunum and ileum

B. Stimulates marked pancreatic secretion of water, bicarbonate and


enzymes

C. Enhances contraction of the pylorus

D. Stimulates gallbladder contraction

E. Is released by acidification of the duodenum

5
 Vaso-active intestinal polypeptide:

A. Is widespread in the autonomic nervous system

B. Is found in the mucosa of the stomach and small and large intestine

C. Stimulates intestinal secretion of water and electrolytes and


pancreatic

alkaline secretion

D. Causes vasoconstriction of peripheral vessels

E. Is released by the presence of amino acids and carbohydrates in the


duodenum

 Enkephalins:

A. Are found in opiate receptor nerve endings in the C.N.S.

B. Are found in endocrine cells and nerves in the G.I.T.

C. Decrease intestinal muscle tone and enhance transit time

D. Subserve analgesia

E. May be responsible for the analgesia of acupuncture

6
 Absorption of the following is by active transport:

A. Potassium

B. Sodium

C. Water

D. Magnesium

E. Fructose

 In the large bowel there is a net absorption of:

A. Sodium

B. Potassium

C. Water

D. Bicarbonate

E. Chloride

 About iron:

A. 0.5mg is excreted daily in the duodenum and proximal jejunum

B. About 1gm is absorbed daily

C. It is absorbed in the ferrous state

D. It is absorbed by passive transport

E. Its absorption is enhanced by Vit. B6

 Concerning digestion and absorption of fat:

A. Long chain triglycerides are hydrolysed by lipase to diglycerides,


glycerol and

fatty acids

7
B. About 60% of fat is hydrolysed by pancreatic lipase

C. A micelle is formed by bile salt, tri-, di-, and mono-glycerides and


fatty acids

D. A micelle is absorbed by active transport into the cell

E. Normally not more than 4g of fat is passed in the stools daily

 Faeces:

A. Consists of water about 65% and solids 35%

B. Contain bacteria which constitute about 20% of the wet weight

C. Lose 400ml of water daily

D. Contain glucose and fructose

E. Contain deconjugated bile salts, protein, calcium, sodium and


potassium

 In intestinal motility:

A. Non-propagated motility involves only a short segment and chyme


moves

distally only for a short distance

B. Propagated motility involves 10 to 15cm and chyme is moved


distally

C. Contractile activity of the intestine is initiated and regulated only by

8
the

mesenteric plexus

D. The background level of excitability of the intestinal muscle cells


depends on

the release of acetyl-choline by cholinergic fibres of the myenteric


plexus

E. Propagated and non-propagated movements are caused by the


same mechanisms

and differ only in the length of intestine stimulated

 In the small intestine:

 The frequency of short waves or basic electrical rhythm is 12 per


minute and

decreases distally

 Intrinsic cholinergic control of movements by the myenteric plexus


and extrinsic

control by the vagus are high

 CCK and serotonin stimulate movements

 Substance P and secretin inhibit movements

 Motilin and bombesin stimulate movements

 In the colon:

A. The frequency of non-propagated contraction is 3-8 per minute

B. Propagated contraction may involve short or long segments

C. Mass peristalsis occurs once a day

D. Mass peristalsis involves 25cm or more of the transverse or


descending colon

E. Mass peristalsis propels the faeces distally into the sigmoid colon
and rectum

9
 In defaecation:

 The process is under cerebral control with a reflex centre in the


lumbar

region of the spine

B. There is an increase in intra-abdominal pressure from contraction of


the

abdominal muscles and descended diaphragm

C. The pelvic floor rises with straightening of the angle between the
rectum and

anal canal

D. The pelvic colon contracts and shortens

E. The anal sphincters relax and faeces are ejected through the anus

 Dirrhoea is a complication of:

A. Pyloroplasty

B. Gastro-jejunostomy

10
C. Polya partial gastrectomy

D. Truncal or highly selective vagotomy

E. Billroth I partial gastrectomy

 Gastro-jejunal anastomosis may result in:

A. Steatorrhoea

B. Early dumping

C. Late dumping

D. Iron deficiency anaemia

E. Macrocytic anaemia

 Effects of resection of the middle 50% of the small intestine


include:

A. Diarrhoea

B. Dilatation and hypertrophy of the remaining small intestine

C. Epithelial hyperplasia and increased absorption of water,


electrolytes and

glucose per unit area

D. Megaloblastic anaemia

E. Increased bacterial flora

 After total colectomy and ileostomy:

 There is initially a daily loss of 3L of water and 200mmol of sodium


through

the effluent

 The loss is reduced within a month to 600-800ml of water and


60mmol of

sodium daily

11
 Some patients may develop calcium oxalate renal stones unless the
daily urine

output is

maintained at more than one litre

D. Hypokalaemia may result

E. Malnutrition may occur

 Microflora in the:

A. Intestinal tract number about 1014

B. Fasting stomach in health consist of strep. faecalis

C. Jejunum is about 102-104/ml and decreases distally

D. Proximal ileum consist essentially of strep. faecalis and aerobic


lactobacillus

E. Colon consist predominantly of non-sporing gram-negative


anaerobic-bacteroides,

bifido bacterium and eubacterium

 Organisms normally found in the colon include:

A. Esch. coli and other coliforms

B. Entero-virus

C. Sporing anaerobic clostridium species – welchii, tetani,


paraputrificum

D. Staphylococcus, yeast

E. Chlamydia trachomatis, strep. faecalis

 Gut bacteria:

12
A. Produce vitamins especially K and B which are absorbed and
utilized

B. Produce volatile fatty acids which inhibit the growth of exogenous


bacteria

C. May be responsible for autogenous infection e.g. postoperative


tetanus, U.T.I.

D. May in a blind loop from end-side or side-side anastomosis be


responsible for

diarrhoea, steatorrhoea and microcytic anaemia

E. May degrade biliary and other steroids in fat to colonic carcinogens

A 25-year old West African male gives a 2-day history of colicky


abdominal pain

with associated frequent passage of loose, mucoid, blood-stained


stools. There

is no vomiting:

 You will seriously consider:

A. Crohn’s disease

B. Amoebiasis

C. Acute appendicitis

D. Shigellosis

E. Ascariasis

 Important investigations will include:

A. Microscopic stool examination

B. Stool culture

C. Colonoscopy

13
D. Proctoscopy and sigmoidoscopy

E. Barium enema

A 65-year old West African woman has for the past 3 months been
having

intermittent diarrhoea without blood, mucus or associated


abdominal pain:

 You will consider among others:

A. Ulcerative colitis

B. Large bowel tumour

C. Parasitic nematodes

D. Malabsorption

E. Crohn’s disease

 Bran:

A. Is the rough low-fibre fraction of natural whole wheat

B. Is not completely digested and absorbs water in the gut

C. Reduces intracolonic pressure, shortens transit time and increases

stool weight

 Is effective in functional constipation, haemorrhoids, diverticular


disease

and anal fissure

14
E. Is available only in tablet form

 In the management of constipation of non-organic cause:

A. Good sleep should be ensured with a sedative

B. The diet should well-balanced with excess of roughage

C. Regular exercises should be taken

D. Apparients should be taken regularly

E. The bowel should be evacuated at a regular period of the day

 Chronic constipation in a reasonably well-nourished normal 4-


year old boy

may be due to:

A. Aganglionic megacolon

B. Physical inactivity

C. Anal fissure

D. Diverticular disease

E. Idiopathic slow transit

 Moderately severe rectal bleeding without abdominal pain in a 2-


year old boy

may be due to:

A. Intussuceptions

B. Meckel’s diverticulum with a peptic ulcer

C. Entero-colitis

D. Severe ankylostomiasis

E. Juvenile polyp

 The commonest cause of chronic rectal bleeding in people over

15
35 years is:

A. Diverticular disease of the large bowel

B. Vascular ectasia

C. Haemorrhoids

D. Large bowel tumour

E. Ulcerative colitis

 Common causes of massive bleeding in patients over 30 years


include:

A. Peptic ulcer

B. Crohn’s disease

C. Amoebiasis

D. Diverticular disease of the colon

E. Enteric fever

 Investigations usually undertaken to establish the cause of


massive

rectal bleeding include:

16
A. Passage of nasogastric tube and aspiration

B. Proctoscopy and sigmoidoscopy

C. Barium enema

D. Microscopic stool examination and culture

E. Arteriography

 The midgut:

A. Is that part of the bowel supplied by the superior mesenteric artery

B. Extends from the second part of the duodenum at the entrance point
of

the common bile duct to the junction of the middle and distal thirds
of the

transverse colon

C. Herniates through the umbilical opening into the umbilical sac in the
fifth

week of foetal development

D. Begins to return to the abdomen at about the fifteenth week of foetal

development

E. On its final return into the abdomen has rotated through an angle of
270o in

a clockwise direction

 Anomalies of rotation of the midgut include:

A. Infantile umbilical hernia

B. Exomphalos

C. Gastroschisis

D. Volvulus neonatorum

17
E. Situs inversus

 Anomalies of rotation and development of the midgut include:

A. Sub-heptic caecum

B. Mobile caecum with or without the ascending colon

C. Duplication of the gut

D. Ileo-umbilical fistula

E. Duodenal atresia

 Persistence of the whole or part of the vitello-intestinal duct may


lead to:

A. Meckel’s diverticulum

B. Ileo-umbilical fibrous cord

C. Atresia of the terminal ileum

D. Raspberry tumour of the umbilicus

E. Urachal cyst

 Complications of Meckel’s diverticulum include:

A. Carcinoma

18
B. Inflammation

C. Perforation and peritonitis

D. Intussusception

E. Peptic ulceration

 Failure of the uro-rectal septum to fuse with the blind end of the
cloaca is the
cause of the following fistula(e):

A. Recto-vaginal

B. Recto-vesical

C. Recto-urethral (male)

D. Recto-urethral (female)

E. Recto-uterine

 In ano-rectal anomalies:

 The low anomaly is that in which the bowel passes through the
pelvic floor

before it becomes abnormal

B. The low anomaly has small sphincters

C. Continence is always achieved in the low anomaly

D. The high anomaly has hypoplastic external sphincter but no internal


sphincter

E. Continence is never achieved in the high anomaly

 The following are low anomalies:

A. Anorectal agenesis

B. Covered anus

C. Ectopic anus

19
D. Rectal atresia with normal anal canal

E. Anal agenesis

 In persistent anal membrane:

A. The rectum and anus are normally developed

B. There is a complete or incomplete diaphragm at the level of the anal


verge

C. If the membrane is incomplete, the opening is placed near the


periphery

D. A complete membrane bulges down

E. The treatment is incision of the membrane and dilatation of the anus

 Ectopic anus may open in the:

A. Vulva

B. Vestibule

C. Upper vagina

D. Perineum

E. Scrotum

 Anorectal anomalies may be associated with:

20
A. Oesophageal atresia
B. Sacral agenesis
C. Hydronephrosis
D. Septate vagina
E. Malrotation of the gut

 If a female neonate has passed a small amount of meconium in


24h and the

abdomen is distended, she may be suffering from:

A. Anorectal agenesis with a narrow rectovaginal fistula

B. A covered anus

C. Ectopic anus

D. Anorectal stenosis

E. Aganglionic megacolon

A lateral X-ray of a 24h neonate with the head down and the tube
centred on the

greater trochanter shows gas shadows above a line joining the


coccyx and the

pubic symphysis:

 He is probably suffering from:

A. Complete anal membrane


B. Rectal atresia with normal anal canal
C. Anal stenosis
D. Anorectal agenesis
E. Anal agenesis

21
 Treatment will be:

A. Incision and dilatation

B. Anal dilatation

C. Colostomy (transverse or pelvic) followed by abdomino-anal pull


through in

6-12 months

D. Immediate abdomino-anal pull through

E. Colostomy (transverse or pelvic) followed by dilatation of the anus


and closure

of the colostomy later

 In aganglionic megacolon:

A. There is congenital absence of parasympathetic nerve fibres in the


intermural

and submucosal nerve plexus of the large bowel from the anus for a
variable

distance proximally

B. The rectum and lower sigmoid are affected in the majority of cases

C. The affected aganglionic segment is narrow and looks “normal”

D. Diagnosis is established by a barium enema

E. Females are affected more commonly

 Clinical features of Hirschsprung’s disease in a 6-year old boy


include:

22
A. Chronic obstinate constipation often dating from infancy

B. Frequent vomiting

C. Gross abdominal distension

D. Palpable faecal masses in the left flank and absence of peristalsis

E. Loaded rectum on rectal examination

 Complications of aganglionic megacolon include:

A. Acute intestinal obstruction


B. Malnutrition
C. Delayed puberty
D. Respiratory infections
E. Retention of urine

 Necrotizing enterocolitis:

A. Is probably due to immune deficiency

B. Follows stasis of faeces

C. Is characterized by severe vomiting, passage of watery, frothy


stools, abdominal

distension and severe prostration

D. Is more common in babies with Hirschsprung’s disease than in older


children

E. May subside spontaneously

23
 Treatment of necrotizing enterocolitits requires:

A. Rectal tube and rectal washout

B. Nasogastric decompression

C. Intravenous fluids

D. Antibiotics

E. Colostomy

 Definitive operations performed for aganglionic megacolon


include:

A. Anterior resection of the rectum

B. Left hemicolectomy

C. Swenson’s operation – rectosigmoidectomy

D. Duhamel’s operation

E. Soave’s operation

 In aganglionic megacolon:

A. Definitive treatment is done at 6-12 months

B. 80% of untreated babies die by 1 year and very few reach


adolescence

C. Those who are treated do very well

D. Faecal incontinence occasionally follows Swenson’s operation

E. Constipation occasionally follows Duhamel’s operation

 In intestinal tuberculosis:

A. The causative mycobacterium is always the human strain

24
B. The infection is blood-borne
C. The mesenteric lymph nodes are the first to be infected and may
become
calcified if the infection is arrested
D. Progressive infection may produce ulcerative, hypertrophic ulcero-
hypertrophic
lesions
 About 30% of patients show caecal lesions

 In ulcerative ileo-caecal tuberculosis:

A. There is usually an associated pulmonary tuberculosis

B. The terminal ileum shows multiple ulcers whose long axis is


characteristically

disposed longitudinally

C. Stricture from progressive fibrosis and ulcer perforation frequently


occur

D. A main symptom is diarrhoea with passage of foetid stools

E. Right hemicolectomy is the treatment of choice

 In hyperplastic ileocaecal tuberculosis:

25
A. The immunity of the patient to the mycobacterium tuberculosis is low

B. The chronic inflammation produces tissue proliferation which results


in thickening

of the intestinal wall and narrowing of the lumen

C. There are attacks of colicky central abdominal pain and vomiting

D. Barium meal shows lengthening of the ascending colon with


stricture formation

E. Right hemicolectomy is often required

 Characteristics of the ascitic aspiration in ascitic tuberculous


peritonitis

include:

A. High specific gravity (1020)

B. Leucocytosis

C. High protein (3g per 100ml)

D. Pink colour

E. Positive culture for mycobacterium tuberculosis

 Chronic tuberculous peritonitis:

A. Is more common in adult females than in children

B. Is often secondary to tuberculous mesenteric lymph nodes, ileo-


caecal tuberculosis

or tuberculous adnexitis

C. May present as progressive ascitis with associated night sweats,


intermittent

fever and weight loss

D. May result in extensive peritoneal adhesions

26
E. May cause blind loop syndrome

 Complications of tuberculous peritonitis include:

A. Acute mechanical intestinal obstruction

B. Paralytic ileus

C. Pelvic abscess

D. Shock

E. Internal fistulae

 Typhoid perforation:

A. Affects up to 20% of adult patients with enteric fever in many tropical


countries

B. Accounts for about 20% of deaths in typhoid

C. Is responsible for up to 40% of peritonitis in males in some tropical


countries

D. In West Africa usually occurs in the third week of typhoid fever

E. Is more common in males and the highest incidence is in the 31-40


age group

 Typhoid ulcers are:

27
A. Circular in shape

B. Longitudinally directed

C. On the mesenteric border

D. Most abundant in the terminal ileum

E. Found in the caecum also

 Typhoid perforation is:

A. Never more than 1cm wide

B. Multiple in about 20% of patients

C. Found within 45cm of the ileo-caecal junction

D. Usually associated with bleeding

E. Uncommon in patients being treated for typhoid fever

 Clinical features of typhoid perforation include:

A. Headache and joint pains

B. Diarrhoea

C. Subnormal temperature and tachycardia

D. Abdominal pain, tenderness and guarding or rigidity

D. Increased bowel sounds

 Investigations that help in confirming a difficult case of typhoid

perforation include:

A. White cell count and differential

B. Blood culture and Widal

C. Straight X-ray of the abdomen

D. Stool and urine culture

28
E. Four quadrant tap

 These investigations must, whenever possible, be done in every


patient

with typhoid perforation before operation:

A. Serum electrolytes and blood urea

B. Haemoglobin and haematocrit

C. Straight X-ray of the abdomen and chest

D. Four quadrant abdominal tap

E. Sickling test

 In the resuscitation of a patient with typhoid perforation, the


following

is (are) essential:

A. Blood transfusion
B. Normal saline/Ringer’s lactate

29
C. Potassium
D. Magnesium
E. Digoxin

 Drugs that may be used in the management of a patient with


typhoid

perforation include:

A. Ciprofloxacin
B. Tetracycline
C. Gentamicin
D. Penicillin
E. Metronidazole

 Complications of typhoid perforation include:

A. New perforation
B. Entero-cutaneous fistula
C. Burst abdomen
D. Anaemia
E. Psychosis

 Measures taken in the treatment of haemorrhage in typhoid


fever include:

A. Blood transfusion
B. Administration of pitressin
C. Administration of omeprazole
D. Resection of the terminal ileum
E. Sedation

 Complications of enteric fever include:

30
A. Osteomyelitis
B. Cholelithiasis
C. Orchitis
D. Urinary tract infection
E. Myocarditis

 Crohn’s disease:

A. Is most probably a hypersensitivity reaction to a viral infection


B. Is common in tropical countries
C. Is much more common in women
D. Does not occur beyond 50 years
E. May in a few patients present with clinical features suggestive of
acute appendicitis

 Crohn’s disease:

 In half the patients affects only the last 25cm of the terminal ileum
and

stops abruptly at the ileo-caecal valve

B. In a third of the patients the terminal ileum and the right colon are
affected

C. In a few patients affects the proximal small intestine

D. May affect the stomach but not the oesophagus

E. Does not affect the sigmoid colon or rectum

 The pathological features of Crohn’s disease include:

31
A. Oedema, hyperaemia and fine granularity of the serosa of the
affected segment

B. Dense submucosal and subserosal fibrosis

C. Mucosal oedema followed by deep ulceration and nodularity

D. Narrowing of the lumen

E. Thickening and oedema of the mesentery of the involved segment


and

atrophic calcification of its glands

 A patient with Crohn’s disease may present with:

A. Fistula-in-ano
B. Pyrexia of unknown origin
C. Diarrhoea and vomiting
D. Generalized peritonitis
E. Entero-cutaneous fistula

 Complications of Crohn’s disease include:

A. Polyarthritis
B. Glaucoma
C. Entero-vesical fistula
D. Blind loop syndrome
E. Pericarditis

 Drugs of definite benefit in Crohn’s disease include:

A. Sulphasalazine
B. Azothioprine
C. Steroids
D. Metronidazole

32
E. Interferon

 Indications for surgery in Crohn’s disease include:

A. Severe diarrhoea
B. Fistulae
C. Systemic manifestations
D. Downhill course of the patient
E. Persistent abdominal mass

 In the prognosis of Crohn’s disease:

A. It is better in children

B. It is worse when the disease is colonic

C. About 50% of patients have a recurrence after surgery

D. Recurrence after surgery is more severe

E. Recurrence rate after surgery is higher in adults than in children

 Entero-colitis:

A. Always follows previous antibiotic therapy and operation


B. Is caused by streptococcus faecalis
C. Is characterized by abdominal pain and distension and frequent
passage
of watery greenish stools containing shreds of pseudomembrane

33
D. Causes subnormal temperature, tachycardia and circulatory failure
E. Has a high mortality: 50-100%

 The antibiotic(s) to use in enterocolitis is (are):

A. Chloramphenicol
B. Erythromycin
C. Oxytracycline
D. Amikacin
E. Ampicillin

 Amoebiasis:

A. Is invasion of the intestine by Entamoeba hystolytica

B. Occurs only in tropical and subtropical regions

C. Is clinically influenced by the nutritional status of the patient

D. Affects extra bowel foci in 15% of patients

E. Is more common in women and children

 Lesions in amoebiasis include:

A. Superficial punctate ulceration of the mucosa


B. Ulcers with undermined edges and yellowish necrotic floor
C. Exuberant granulations with pseudopolypi
D. Extensive submucosal and subserosal fibrosis
E. Granularity of the serosa

 Features of amoebiasis include:

A. Absence of symptoms
B. Windy looseness of stools preceded by aching in the rectum
C. Nausea and vomiting

34
D. Abdominal colic, constipation, mucus and bright red blood in stools
E. Pyrexia, abdominal distension and tenderness

 The ideal fluid to give to an ill patient with amoebic colitis and a
reasonable
renal output is:

A. Normal saline
B. Ringer’s lactate
C. Blood
D. Fluid 5:4:1
E. Darrow’s solution

 In practical terms the investigation(s) that help(s) to establish a


diagnosis
of amoebic colitis is (are):

A. Microscopy of fresh stools or rectal scrapings


B. Barium enema
C. Stool culture
D. Proctoscopy and sigmoidoscopy

35
E. Gel diffusion precipitation test

 Microscopy of stools confirms amoebic colitis only if it shows:

A. Trophozoites of Entamoeba hystolytica


B. Cysts of Entamoeba hystolytica
C. Haematophagous trophozoites of Entamoeba hystolytica
D. Trophozoides and cysts of Entamoeba hystolytica
E. R.B.C. and W.B.C. and cysts of entamoeba hystolytica

 Drug regimes effective in intestinal amoebiasis include:

A. Metronidazole 800mg tds orally for 7 days


B. Tinidazole (Farsygin) 2g daily for 3 days
C. Diloxanide furoate 0.5g tds for 10 days
D. Oxytetracycline 250mg qds for 10 days
E. Dihydroemetine 90mg IM daily for 10 days

 Amoebic perforation:

A. Is the only cause of peritonitis in amoebic colitis


B. Usually occurs in the caecum or rectosigmoid
C. Always gives rise to marked abdominal pain, tenderness, rebound
tenderness
and rigidity
D. Is best treated conservatively
E. If found at operation is treated by closure

 Amoeboma:

A. Consists of a granulomatous mass with multiple abscesses


B. Commonly occurs in the transverse and sigmoid colons
C. Presents as a palpable mass, intermittent diarrhoea or intestinal

36
obstruction
D. Is best treated by operation
E. Must be differentiated from carcinoma, schistosomiasis and
tuberculosis

 The most certain and acceptable way of distinguishing amoeboma


from

carcinoma is by:

A. Resection and histology


B. Barium enema
C. Sigmoidoscopy and biopsy
D. Finding of amoebae in the stools or rectal scrapings
E. Regression of the mass on antiamoebic treatment
99. Amoebiasis may affect the:
A. Penis
B. Cervix
C. Skin
D. Brain
E. Spleen
100. Ulcerative colitis:
A. Is an idiopathic inflammatory condition of the large bowel affecting
predominantly
the mucosal and submucosal layers
B. In all cases starts in the rectum and then spreads proximally
C. Is rare in the tropics but common in Caucatians
D. Like Crohn’s disease frequently shows skip lesions
E. Is a hypersensitivity disease
 The pathological lesions in ulcerative colitis include:

37
A. Elongation of the colon
B. Oedematous dilated friable colon with numerous areas of necrosis
C. Discrete or confluent mucosal ulcers
D. Normal mucosa intervening the ulcers
E. Abscesses at the mesenteric border and peri-anal region

 Ulcerative colitis may present with:

 Severe diarrhoea, 30-40 motions a day, with mucus, pus and blood
in the
stools and associated fever and abdominal colic
B. Recurrent attacks of diarrhoea
C. Severe rectal bleeding
D. Intestinal obstruction
E. Peritonitis

 Complications of ulcerative colitis include:

A. Conjunctivitis
B. Spondylitis
C. Pyoderma gangrenosum
D. Psychosis
E. Portal hypertension

A 30-year old Caucasian woman, who has been having severe


blood-stained, mucoid diarrhoea with associated colicky
abdominal pain for the past 10 days, now passes only small
amounts of stools. She is febrile and weak and the abdomen is
distended. bowel sounds are absent.

38
 She is probably suffering from:

A. Severe ulcerative colitis


B. Hypokalaemia
C. Peritonitis
D. Toxic megacolon
E. Intestinal obstruction

 Measures taken will include:

A. Infusion of Ringer’s lactate and potassium


B. Possible urgent laparotomy and total procto-colectomy
C. Possible urgent laparotomy and relief of obstruction or drainage of
peritoneum
D. Administration of steroids
E. Administration of cefotaxine and amikacin

 Colorectal carcinoma as a sequel to ulcerative colitis is likely to


occur in:

A. Disease restricted to the left side of the large bowel and rectum
B. Women
C. Patients developing their first attack in childhood or adolescence
D. Patients with at least 5 years history
E. Patients with the fulminating type

 Indications for surgery in ulcerative colitis include:

Massive haemorrhage, intestinal obstruction


Iritis or arthritis
Pseudopolypi
Patients over 50 years

39
Disease exceeding 10 years duration

 Inflammatory reaction of the large bowel epithelium to eggs of


schistosoma

mansoni results in the formation of:

A. Exuberant granulations
B. Tubercles
C. Carcinomata
D. Papillomata
E. Ulceration

 Intestinal schistosomiasis may present with:

A. Dry cough, dyspnoea and urticaria


B. Peritonitis
C. Diarrhoea and palpable colon
D. Liver abscess
E. Splenomegaly, ascites

 Lyphogranuloma venereum:

A. Is caused by Chlamydia trachomatis


B. Is not seen in temperate regions
C. Is more common in females
D. Usually may or may not be sexually transmitted
E. Has an incubation period of 14 to 40 days

40
 The primary ulcer in L.G.V.:

A. Is small
B. Is persistent
C. Is superficial
D. Is painful
E. Starts as a vesicle in the fourchette or anal margin in the female

 Intestinal L.G.V.:

A. Is equally common in the sexes


B. In the male, occurs in homosexuals
C. In the female is associated with fibrosis of the lower posterior vagina
and recto-vaginal septum
D. In the female may cause recto-vaginal fistula
E. In the male may cause recto-vesical fistula

 Inguinal adenitis in L.G.V.:

A. Occurs in the male but not in the female


B. Is painful but is not accompanied by systemic disturbance
C. May suppurate and result in sinuses and fibrosis
D. Does not usually give a positive L.G.V. C.F.T.
E. Can resolve spontaneously as it is self-limiting

 Large bowel stricture in L.G.V.:

A. Is usually tubular
B. Has a constant lower limit at 3 to 5cm from the anal margin
C. May involve the anus, rectum, sigmoid colon and descending colon
D. Depends to a large extent on the duration of the antecedent procto-
colitis

41
E. Involves all the layers of the bowel wall but not the pararectal or
paracolic tissues

 A patient with intestinal L.G.V. may present with:

A. Passage of blood, mucus and pus per rectum


B. Diarrhoea, constipation, alternating constipation and diarrhoea
C. Anal fistulae or perianal abscess
D. Perforation, peritonitis
E. Malabsorption syndrome

 The following in a woman is (are) suggestive of L.G.V.:

A. Vaginal stenosis
B. Perianal warts
C. Perianal granulomata and lymphorrhoids
D. Lymphoedema of the vulva
E. Bridges of skin across the labia

 Intestinal L.G.V. is usually most accurately confirmed by:

A. Sigmoidoscopy and biopsy


B. Barium enema
C. LGVCFT
D. Plasma proteins

42
E. Stool microscopy for the causative organism

 Which of the following may be necessary in the treatment of


intestinal L.G.V.?

A. High fibre diet


B. Total proctocolectomy
C. Steroids
D. Colostomy
E. Digital dilatation

 Effective drugs in L.G.V. include:

A. Sulphadiazine
B. Metronidazole
C. Oxytetracycline
D. Ampicillin
E. Amikacin

 Diverticulosis of the colon:

A. Is caused by high intraluminal pressure and low residue, highly


refined
carbohydrate diet

 Occurs at points through which blood vessels and nerves penetrate


the
bowel musculature
C. Is fairly common in Africans
D. In North America is found in 10% of people at 40 years and in 65%
by 85 years
E. Is associated with unco-ordinated colonic muscular activity

 In diverticular disease:

43
A. The muscle is thickened
B. The mucosa is oedematous and ulcerated
C. The length of the bowel is shortened
D. There may be intramural or extramural abscesses
E. There is paucity of fat in the mesentery and bowel wall

 Clinical features of diverticular disease include:

 Passage of frequent small hard faeces alternating with passage of


watery,
mucoid stools

 Flatulence, dyspepsia and recurring pains in the L.I.F. usually


relieved by
passage of flatus

 Absence of symptoms in some patients

 Periods of exacerbation and remission of symptoms

 Night sweats, anorexia and weight loss

 Complications of diverticular disease include:

A. Massive haemorrhage
B. Carcinoma

44
C. Localized pericolic abscess
D. Anal fistulae
E. Intestinal obstruction

 In the medical treatment of diverticular disease of the sigmoid, the

following may be required:

A. Morphine, codeine
B. High fibre diet
C. Onions cabbage, peas, beans
D. Dioctyl sodium sulphosuccinate, normacol, isogel
E. Oxytetracycline,metronidazole, phthalylsulphathiazole

 Indications for surgery in diverticular disease include:

A. Diverticulosis
B. Massive haemorrhage
C. Fistula formation
D. An attack of diverticulitis
E. Abscesses

 Acquired entero-cutaneous fistula may be due to:

A. Amoebic perforation of the colon


B. Penetrating abdominal injury involving the gut
C. Operation for peritonitis or intestinal obstruction
D. Intestinal tuberculosis, Crohn’s disease
E. Strangulated external hernia

 Entero-cutaneous fistula:

A. Affects the jejunum most commonly


B. Always has a relatively long and narrow tract lined by granulation

45
tissue
C. Has serious effects whether it is small or large intestine
D. If duodenal has a greenish discharge
E. If ileal or colonic gives a frankly faecal discharge

 Complications of entero-cutaneous fistula include:

A. Dehydration, electrolyte deficiencies and metabolic alkalosis


B. Macrocytic anaemia
C. Intra-abdominal abscesses, septicaemia
D. Paralytic ileus
E. Hypoproteinaemia

 Ileo-cutaneous fistula is confirmed by:

A. Chemical examination of the effluent


B. Oral administration of non-absorbable coloured material
C. Straight X-ray of the abdomen
D. Barium meal and follow through
E. Barium enema

 Investigations that MUST be done in a patient with entero-


cutaneous

fistula following typhoid perforation include:

A. Serum electrolytes and blood urea


B. Routine urine examination

46
C. Haemoglobin, white cell count and differential
D. Serum bilirubin, alkaline phosphatase and plasma proteins
E. Plain chest X-ray

 Measures taken in the management of external small bowel fistula


include:

A. Intravenous fluid and electrolyte therapy


B. Oral feeding supplemented by parenteral feeding
C. Administration of antibiotics
D. Insertion of a catheter into the fistula
E. Assumption of Fowler’s position to ease drainage

 Indications for operative intervention in entero-cutaneous fistulae


include:

A. Colonic fistula
B. Continued profuse discharge
C. Wide cutaneous opening and intestinal mucosa continuous with skin
D. Fistula of hernial origin
E. Absence of obstruction distal to the fistula

 Pilonidal sinus:

 Is a sinus from a pocket of sebaceous material which opens in the


midline at the
upper end of the natal cleft
B. Is congenital in most patients
C. Is common in Caucasians, rare in Blacks and unknown in Orientals
D. Is commoner in males and most patients are between 40 and 49
E. May have ramification of secondary sinuses

47
 Fissure-in-ano:

 Is a tear in the long axis of the squamous mucosa in the lower third
of
the anal canal

 Usually occurs in the posterior midline, but in 40% of patients


especially
women, it is in the anterior midline

 Is caused by over-stretching of the anus during the passage of hard


faeces,
but in many a foreign body, e.g. fish bone is the cause
D. Is more common in men
E. Causes paresis of the underlying sphincter which may become
fibrosed
and contracted

 Clinical features of anal fissure include:

 Anal pain coming on during defaecation and lasting for several


hours afterwards
B. Passage of bright red blood which is intimately mixed with the
faeces
C. Anal discharge
D. Pruritus
E. “Sentinel pile”

 Measures that may be taken in the treatment of anal fissure include:

48
A. Purgation
B. Local application of a surface anaesthetic such as lignocaine in a
jelly base
C. Digital stretching of the anal sphincters under anaesthesia
D. Antibiotics
E. Codeine phosphate

 Haemorrhoids:

 Are varices of the vessels of the superior and/or inferior rectal


plexus
of arteries

 Are congregated in three primary positions – right anterior, right


posterior and
left lateral
C. Occur in the anal canal below the dentate line
D. May include secondary and tertiary haemorrhoids between the
primary ones
E. Are second degree if they prolapsed on defaecation but
subsequently retract spontaneously

 Predisposing factors in haemorrhoids include:

A. Heredity
B. Sex
C. Manual work
D. Pregnancy
E. Chronic constipation

49
 The symptoms of haemorrhoids include:

A. Pain on defaecation
B. Passage of bright red blood usually unmixed with faeces
C. Mucoid discharge and perianal discomfort
D. Chronic constipation
E. Recurrent diarrhoea

 Signs in uncomplicated haemorrhoids include:

A. Anal tags at the anal verge


B. Palpable swelling in the anus on rectal examination
C. Protrusion of reddish swellings at the anus on straining
D. Bulging of pinkish swellings into the lumen of the anus as a
proctoscope is withdrawn
E. Excoriation of the anal verge

 Investigation (s) that must be done in a 45-year old patient with

haemorrhoids include:

A. Haemoglobin
B. Sigmoidoscopy
C. Barium enema
D. Colonoscopy

50
E. Microscopy of faeces

 Complications of haemorrhoids include:

A. Thrombosis
B. Neoplasia
C. Microcytic anaemia
D. Pruritus ani
E. Perianal suppuration

 Useful measures taken in the management of first and second


degree

haemorrhoids include:

A. Purgation
B. High fibre diet or bran
C. Rectal ointments or suppositories
D. Sclerotherapy
E. Rubber band ligation

 Bowel preparation before haemorrhoidectomy comprises:

A. Low residue diet for 2 days


B. Purgative for one day
C. Enema or rectal washout night before operation
D. Metronidazole for 3 days
E. Neomycin for 2 days

51
 Post-operative complications after haemorrhoidectomy for
uncomplicated

third degree haemorrhoids include:

A. Retention of urine
B. Pyeliphlebitis
C. Anal stenosis
D. Haemorrhage
E. Anal incontinence

 Organisms responsible for ano-rectal abscesses include:

A. Tubercle bacillus
B. Pneumococcus
C. Esch. coli
D. Bacteroides
E. Actinomycetes

 Ano-rectal abscesses:

A. In most cases follow an intersphincteric abscess caused by infection


of an anal
gland via an anal crypt
B. Are most commonly ischiorectal
C. Are commoner in women
D. Characteristically cause severe throbbing anal pain made worse by
sitting or
even walking
E. Are accompanied by severe constitutional symptoms

 In the treatment of ischio-rectal abscess the most essential

52
measure is:

A. Antibiotic therapy
B. Analgesia
C. Hot fomentation
D. Incision and drainage
E. Bed rest

 Multiple fistulae-in-ano may be associated with:

A. Tuberculosis
B. Lymphogranuloma venereum
C. Amoebiasis
D. Actinomycosis
E. Carcinoma of the anus or rectum

53
 Fistula-in-ano:

A. In most cases originates in the anal crypts


B. May result from a badly-drained ano-rectal abscess
C. May in some cases be sinus
D. In most cases is characterized by the intermittent passage of faeces
and
flatus through the external orifice in the perineum
E. If the external opening is behind a transverse line drawn across the
mid-point
of the anus, runs directly in a straight line with the internal opening

 Essential investigations in fistula-in-ano include:

A. E.S.R.
B. X-ray of the chest
C. Barium enema
D. Sigmoidoscopy and proctoscopy
E. Swab of the pus for microscopy, culture and sensitivity of isolated
organisms

 Operative procedures for fistula-in-ano include:

A. Incision and drainage


B. Fistulotomy
C. Fistulectomy and primary closure of the perineal wound
D. Internal sphincterotomy and drainage
E. Anoplasty

 Pruritus ani may be due to:

A. Perianal discharge

54
B. Ingestion of certain food items
C. No known cause
D. Parasitic infestation
E. Mycotic disease of the anal skin

 Investigations that must be done in pruritus ani include:

A. Urinalysis
B. Microscopy of stools
C. Vaginal smear
D. Urethral smear in the male
E. Microscopic examination of perianal scrapings

 In the management of pruritus ani in which no cause has been


found, useful

measures include:

 Washing of the anal region with warm water and soap after every
defaecation
and three times daily
B. Local application of steroids cream, or lotio mag. carbol or lotio
calamino
C. Phenobarbitone or diazepam three or four times daily
D. Mild laxatives
E. Analgesics

55
 The commonest cause of acquired anal incontinence is:

A. Senility
B. Spinal tumour or injury
C. Tabes dorsalis and general paralysis of the insane
D. Trauma to the sphincters
E. Faecal impaction

 The following may be associated with anal incontinence:

A. Prolapse of the rectum


B. Ectopic anus
C. Fistula-in-ano
D. 3o haemorrhoids
E. Leprosy

 In anal incontinence:

A. Loss of sensation in the anal wall may be the cause


B. Division of the internal anal sphincter may be the cause
C. High residue diet and anticholinergics may be helpful
D. Thiersch operation is useful when the anus is patulous as in senility
and mental deficiency
E. There may be continence for flatus

 Mucosal (incomplete) prolapse of the rectum:

A. Occurs only in young children, usually under 4 years


B. In most cases has an obvious underlying cause
C. Projects at the anus during defaecation and always has to be
replaced digitally
D. May be a form of intussusception

56
E. On palpation is found to consist of only two layers of mucosa

 Most patients with incomplete rectal prolapsed are treated by:

A. Temporary Thiersch operation using catgut


B. Submucous injection of sclerosing agents
C. Non-operative measures
D. Linear cauterization of the anal mucosa
E. Ligature excision of the redundant mucosa

 Complete rectal prolapse:

A. Occurs at a younger age in males than in females


B. Has ill-understood aetiology
C. Is in every patient associated with diminished or absent rectal
sensation and
atony of the anal sphincters
D. In most patients is preceded by constipation or diarrhoea
Is associated with a shallow recto-vesical pouch

 Complications of complete rectal prolapsed include:

57
A. Intussusception
B. Irreducibility
C. Gangrene
D. Rupture
E. Ischaemic colitis

 Ripstein operation is:

A. Wrapping of polyvinyl alcohol sponge around the mobilized rectum


B. Anterior resection of the redundant rectum and sigmoid colon
C. Approximation of the levator ani to strengthen the pelvic floor
D. Amputation of the prolapsed rectum
E. Fixation of the rectum to the hollow of the sacrum with a sling of
polypropylene
or Teflon mesh

 The commonest cause of ano-rectal stricture in West Africa is:

A. Lymphogranuloma venereum
B. Irritant enema and suppositories
C. Congenital anomalies of the anus
D. Impaction of foetal head in the pelvis
E. Post-haemorrhoidectomy

 Benign tumours of the small intestine usually present with:

A. Diarrhoea
B. Attcks of flushing, bronchospasm and abdominal colic
C. Acute intestinal obstruction
D. Intestinal bleeding
E. Perforation

 Melanin pigmentation of the lips, oral mucosa, face and hands and

58
recurrent

attacks of abdominal colic is suggestive of the following


condition(s) of the

small bowel:

A. Leiomyoma
B. Neurofibroma
C. Adenoma
D. Intestinal polyposis
E. Fibrosarcoma

 Malignant tumours of the small intestine may present as:

A. Hepatomegaly
B. Abdominal mass
C. Perforation
D. Macrocytic anaemia
E. Malabsorption syndrome

 Argentaffinoma (Carcinoid tumour):

A. Is an interesting but rare tumour


B. Is least common in the ileum

59
C. If in the appendix rarely metastasizes
D. Produces prostaglandins, kinins, histamine, indoles as well as 5HT
E. May cause pulmonary stenosis

 Adenoma of the large bowel is:

A. Less common than papilloma


B. More common in males especially over 45
C. Always single and occurs most commonly in the sigmoid colon
D. Precancerous and the main symptom is bleeding
E. Treated by colonoscopy and polypectomy

 Papilloma of the large bowel:

A. Occurs most frequently in the sigmoid colon


B. Is usually pedunculated but may encircle the bowel lumen
C. Has a much greater tendency to malignancy than adenoma
D. Causes profuse mucous diarrhoea
E. In the rectum is treated by anterior resection of the rectum if it is
above
10cm from the anal verge

 Juvenile adenoma:

A. Occurs predominantly in the rectum and is usually multiple


B. Presents as rectal bleeding
C. Is a true tumour
D. Is precancerous
E. Is treated by excision

 Familial adenomatous polyposis:

 Is a hereditary disease in which the large bowel is thickly covered

60
with adenomata
B. Is transmitted by females and the gene is dominant
C. Usually presents with lower abdominal pain and diarrhoea around
25 years
D. If untreated, becomes malignant about 25 years after the onset of
symptoms
E. Is treated by hemicolectomy of the more predominantly affected half
of the
large bowel

 Gardner’s syndrome comprises:

A. Desmoid tumours
B. Osteomata of the skull and mandible
C. Multiple lipomata
D. Multiple sebaceous and desmoids cysts
E. Familial polyposis

 Carcinoma of the colon and rectum:

A. Is relatively less common in Black Africa than in Western Europe


B. Is seen most frequently in Scotland
C. Has about the same relative age specific rates in developed and
developing communities
D. Occurs at an earlier age in men
E. Has the highest incidence in the age group 60-79

 In the Astler-Coller modification of Duke’s staging of carcinoma of


the

61
rectum and colon:

A. Stage B is when the growth has spread to the regional lymph nodes
B. The stage of a carcinoma depends on the duration of symptoms and
the
histological and biological characteristics and rate of growth of the
tumour
C. In the rectum 15% are stage A and B1, 35% stage B2 and 50%
stage C1, C2 and D
D. Stage A tumours contain the highest proportion of slow growing
tumours
E. Stage C tumours contain the highest proportion of cases of long
duration
of symptoms

 The common symptoms in carcinoma of the large bowel is (are):

A. Change bowel in habits and abdominal pain


B. Constipation
C. Diarrhoea
D. Alternating constipation and diarrhoea
E. Abdominal pain, distension and passage of blood or mucus in the
faeces

 Clinical features of carcinoma of the rectum include:

A. Rectal bleeding – bright red blood mixed with faeces and mucus or
passed alone
B. Spurious diarrhoea
C. Haemorroids
D. Pain on defaecation
E. Enlarged inguinal lymph nodes

 Carcinoma of the caecum or ascending colon may present with:

62
A. A lump
B. P.U.O.
C. Anaemia
D. Vague dyspepsia
E. Alternating constipation and diarrhoea

 In complications of large bowel carcinoma:

A. Intestinal obstruction is more common on the right side of the colon


B. Occasionally bleeding may be acute and endanger life
C. “Appendix abscess” in a patient over 40 could be a malignant
paracolic abscess
D. Perforation occurs in 17% of patients and is best treated by one-
stage resection
E. Recto-vesical and recto-vaginal fistulae are fairly common

 Cleansing of the large bowel before resection can be achieved by

adopting alone:

A. Whole gut irrigation on the morning of operation

63
B. Saline purges for 3 days before operation
C. Enema 2 days before operation
D. Rectal washouts night before operation
E. Oral mannitol 24hrs before operation

 Current methods of preventing post-operative infection after large


bowel

surgery include:

A. Metronidazole and Cefuroxime I.V. one hour before anaesthetic


induction

and repeated 8 and 16h post-operatively

B. Oral neomycin and metronidazole for 5 days before operation

C. Oral phthalylsulphathiazole, neomycin and metronidazole for 5 days


pre-operatively

D. Oxytetracycline and metronidazole orally for 5 days before operation

E. Erythromycin, neomycin and metronidazole orally for 3 days before


operation

 Dangers of pre-operative bowel preparation with oral antibiotics


include:

A. Nephrotoxicity
B. Agranulocytosis
C. Enterocolitis
D. Stomatitis
E. Fungal infection of the gut

 The treatment of carcinoma of the:

A. Caecum is right hemicolectomy


B. Hepatic flexure is extended right hemicolectomy

64
C. Transverse colon is left hemicolectomy
D. Descending colon is left hemicolectomy
E. Sigmoid colon is sigmoid colectomy with or without left
hemicolectomy

 In the treatment of carcinoma of the rectum:

 Tumour more than 10cm from the anal verge is treated by anterior
resection
of the rectum
B. A tumour less than 10cm from the anal verge and poorly
differentiated is best
treated by abdomino-perineal resection of the rectum
C. A tumour between 5 and 10cm from the anal verge and either well-
or moderately
well-differentiated is best treated by low anterior resection
D. Adjuvant radiotherapy 4000-5000Gy reduces the incidence of local
recurrence
E. Pre-operative radiotherapy also improves the 5-year survival

 In carcinoma of the large bowel:

65
A. Curative resection is possible in 50%
B. Palliative resection is possible in 20%
C. Patients with obstruction have an operative mortality of 25% and a
25%
5-year survival rate.
D. The crude survival of all patients at 5 years is 20%
E. The crude survival of all patients at 10 years is 14%

 Indications for a permanent colostomy include:

 Relief of obstruction of the left side of the colon due to carcinoma or


diverticulitis
B. Excision of the rectum and anus for carcinoma or stricture
C. Resting of a severely inflamed sigmoid colon and rectum, e.g.
L.G.V.
D. Diversion of faeces in recto-vesical or recto-vaginal fistula
E. Anorectal atresia

 Complications of colostomy include:

A. Prolapsed
B. Metabolic acidosis
C. Retraction and stenosis
D. Strangulation of small bowel
E. Dirrhoea

 Malignant tumours of the anus include:

 Adenocarcinoma

 Squamous cell carcinoma

 Fibrosarcoma

66
 Basal cell carcinoma

 Malignant melanoma

 Squamous cell carcinoma of the anus:

A. Is the commonest malignant tumour of the anus


B. May present as a localized ulcer with rolled edges and indurated
base
C. Bleeds but does not usually cause pain on defaecation
D. Is treated by abdomino-perineal resection of the rectum and anus in
all patients
E. Usually spreads to the inguinal nodes and bilateral block dissection
of the groins
should be done whether or not the nodes are enlarged

 Sacro-coccygeal teratoma is:

A. Common
B. Commoner in females
C. Always firmly attached to the coccyx and sacrum
D. Malignant when it presents after 2 months
E. Treated by excision, supplemented with radiotherapy and cytotoxic
therapy if
it is malignant

67
 Concerning typhoid ileal perforation:
A. Has an incidence of up to about 20 percent of adult typhoid cases
B. Accounts for 40-50 percent deaths in typhoid fever
C. Is responsible for 20% cases of peritonitis excluding salpingitis
D. It occurs most frequently in the proximal ileum
E. Occurs more frequently in multiples
192. The necrosis in the Payer’s patches of the terminal ileum that
precedes ulceration

and perforation or haemorrhage in typhoid enteritis is the result


of which of these

pathological processes:

A. Embolisation of the ileal arterioles


B. Venous thrombosis attending the typhoid inflammatory reaction.
C. Direct effect of the endotoxin tumour necrosis factor on the
mucosa
D. A cellular hypersensitivity response akin to the Schwartzman
reaction
E. A Raynaud like phenomenon affecting the terminal ileo-caecal
artery

193. A 20 years old West African presented at the Medical Surgical


Emergency Centre with a week’s history of high fever,
headaches, nausea, weakness and generalized malaise.
Abdominal pain centred on the right iliac fossa and diarrhoea
set in two days before presentation. On examination he had a
tinge of jaundice with scattered crepitations in both lung
fields, a distended abdomen with diffuse tenderness but
devoid of guarding and bowel sounds were heard but sparse.
Rectal examination was unremarkable:

The most likely diagnosis is:

 Severe episode of falciparum malaria attach

68
 Peritonitis from appendicitis
 Amoebic colitis
 Typhoid Perforation
 Acute intestinal obstruction with strangulation
194. The most useful diagnostic investigation is:

A. Plain x-rays of the abdomen-erect


B. Abdominal ultrasound
C. Repeated examination of fresh sample of stool
D. A full blood count
E. Liver function tests

195. The treatment regime of choice for patient in Question 193 is:
A. Immediate emergency surgical intervention
B. Conservative regime – expectant therapy, I/V, s antibiotics
C. Period of resuscitation and observation, intervening only if there is
no improvement
D. Resuscitation to get the patient fit for expeditions laparotomy
E. Resuscitation peritoneal paracentesis, with peritoneal lavage
196. In the resuscitation of the patient in Question 193 the most
important factor
determining the outcome of management is:

A. The pre-operative Haemogloblin/Haematochrit


B. The pre-operative blood PH level
C. The pre-operative blood urea a level
D. The pre-operative PCO2 level
E. The pre-operative serum potassium level

69
F. The pre-operative bicarbonate level
G. The pre-operative serum albumin level
H. The pre-operative serum sodium level
197. The three most commonly encountered complications following
the operative
treatment of the patient in Question 193 are:

A. Atelectasis/bronchopneumonia, anaemia, surgical site infection

B. Anaemia, surgical site infection, wound dehiscence

C. Anaemia, residual intra-abdominal abscesses, enterocutaneous


fistula

D. Residual intra peritoneal abscesses, anaemia acute renal failure

E. Anaemia, intra vascular haemolysis, surgical site infection

198. Which of these factors influence the prognosis in the patient


with typhoid
perforation:
A. Duration of illness before admission
B. Patient who perforates on protracted medical care
C. Pre operative serum potassium level
D. Combination of haemorrhage and perforation
E. Presentation with associated jaundice

199. A young man of 28 is seen in the Medical Surgical Emergency


with a weeks

history of fever, malaise, unproductive cough and abdominal


discomfort. He

is in the clinic because of passage of large amounts of altered


blood and

70
melaena. On examination he is very pale, his pulse is 110/min,
BP 10O/60 and

his abdomen is, soft, non tender unremarkable. P.R. shows


melaena stool on

the finger.

The most likely diagnosis:

A. Rupture oesophageal varice after an acute respiratory tract


infection
B. Amoebic colitis
C. Tuberculous enteritis
D. Enteric mucosal ulcerations
E. Ischaemic colitis
200. Which of these tests would be the most promising means of
determining the

cause for the gastrointestinal bleeding in Question 199.

A. Upper GI Endoscopy
B. Urgent Colonoscopy
C. Stool microscopy
D. Blood culture
E. Stool culture
201. The treatment of choice for the patient in question 199 is:
A. Injection Sclerotherapy at Upper GI Endoscopy
B. Endoscopic rubber band ligation of varices
C. Urgent resuscitation and preparation for laparotomy
D. Resuscitation conservative management with exhibition of
Ciprofloxacin
and metronidazole
E. Resuscitation exhibition of Ciprofloxacin and metronidazole with
immediate
laparotomy
202. A twenty eight year old woman presents with a week’s history

71
of high fever,
headaches joint pains and pain in the right upper abdomen;
jaundice becomes
evident at the end of the week. Examination depicts a gravely
ill patient,
dehydrated jaundiced with tenderness and guarding in the
right upper quadrant
the abdomen; Murphy’s sign is positive.

The most likely diagnosis is:

A. Acute calculous cholecystitis


B.Biliary colic with cholangitis
C. Acute salmonella cholecystitis
D. Acute pancreatitis
E. Fulminating viral hepatitis

203. How can the diagnosis of the patient in Question 202 be most
readily clinched?
A. Abdominal ultrasound
B. Plain x-ray of the abdomen
C. Blood culture
D. Urgent laparoscopic examination
E. Diagnostic abdominal tap
204. The treatment of choice in the patient in question 202 is:

A. Laparoscopic cholecystectomy
B. Laparotomy with view to open cholecystectomy
C. Resuscitation conservative management with exhibition of
ciprofloxacin/metronidazole.
D. Laparotomy with a view to exploration of the common bite duct.
E. Resuscitation and peritoneal lavage
205. The aetiological basis of Crohn’s Disease is yet to be
established. Which of

these factors finds support from the weight of investigative

72
evidence?

A. A transmissible agent akin to mycobacterial organisms is


responsible
B. A non caseating, granuloma producing agent
C. Associated severe lymphangiestasis suggests lymphatic
obstruction from
lymphangitis
D. Hypersensitivity to some dietary content – milk
E. Pathogenic of propensity of excessive amounts of the cytokine
TNFa
206. Pathologically the hall marks of Crohn’s Disease are
characterized by which
combination of features:

A. (i) Pronounced bowel and mesentery thickening, (ii) progressive


luminal
narrowing (iii) mucosal ulceration fissuring and fistulae

 (i) Pronounced bowel wall thickening, (ii) luminal dilatation (iii)


mucosal

ulceration and fistulae

 (i) Bowel wall thickening, (ii) caseating mucosal ulcers (iii)


fistulae
 (i) Bowel and mesentery thickening (ii) progressive luminal
dilatation
(iii) mucosal ulcers and fistulae

 (i) Progressive bowel dilatation, (ii) thickening of the mesentery


(iii) mucosal ulcer modularity and fissures

73
207. The thickening of the bowel wall that characterizes Crohn’s
Disease is
maximally seen in the:

 Mucosa
 Muscolaris mucosae
 Submucosa
 The smooth muscle coat
E. Serosa
208. The distribution of Crohn’s Disease (Morphologically) is most
characteristically
in the:

 Terminal ileum, stopping abruptly at the ileo-caecal (50% of


cases)
 Involvement of the right colon as well as ileum
 Proximal small bowel – jejuno-ileitis
 Predominantly colonic, with little involvement of the ileum
 Involving upper GI (mouth oesophagus stomach as well as
intestine
209. In Crohn’s Disease, microscopically the earliest change in the
bowel is:
A. Gross oedema most strikingly in the submucosa
B. Gross aedema of the mucosa
C. Ulceration of the mucosa
D. Fine granularity of t he serosal surface
E. Marked vascular/lymphatic dilatation
210. In Crohn’s Disease the late stages are marked mostly by:
A. Mucosal ulceration with profuse bleeding
B. Dense submucosal and serosal fibrosis
C. Acute local abscesses
D. Gross bowel dilatation – toxic mega colon
E. Segmental bowel infarcts from embolisation

74
211. A man of 35 visiting West Africa from Europe, complains of acute
onset of lower
abdominal pain, anorexia nausea, and vomiting accompanied by
diarrhoea of
two days duration. On examination he is dehydrated with low
grade pyrexia
(38.4°C) and there is tenderness with guarding localized to the
right iliac fossa.

The most likely diagnosis is:


A. Acute appendicitis
 Leaking duodenal ulcer
 Amoebic colitis
 Acute presentation of Crohn’s Disease
 Paracaecal abscess from leaking carcinoma of the caecum
212. Which investigative procedure is most likely to lead to
confirmation of the
diagnosis:

A. Full blood count


 Plain x-ray of the abdomen
 Examination of hot specimen of stool
 Abdominal ultrasound
 Laparoscopy
213. The care of the ileostomy in a patient with ulcerative colitis
entails:
A. (i) Daily stoma examination, (ii) preventing of skin excoriation
(iii) exclusion of dietary elements that make it run
B. (i) Daily stoma examination, (ii) periodic irrigation, (iii) application

75
of
zinc oxide ointments to the skin
C. (i) Daily stoma examination, (ii) exclusion of dietary element that
make it run (iii) regular dosing with buscopan
D. (i) Regular dosing with codeine phosphate (ii) daily stoma
examination
(iii) periodic zinc oxide application
E. (i) Daily soma examination (ii) periodic irrigation (iii) regular
dosing with buscopan

214. A woman of 38 presented with a two month history of


deterioration general
health anorexia, weight loss, low grade fever, lower abdominal
pains with
alternating diarrhoea and constipation, tenesmus anal pain. She
had passed
pus, mucus and blood in the stool and had purulent vaginal
discharge. On
examination she was anaemic and had a tender palpable mass in
the left iliac
fossa. PR showed granular mucosal thickening with tenderness.

The most likely diagnosis is:


A. Amoebic colitis with proctitis
B. Carcinoma of the rectum
C. Tuberculosis of the recto sigmoid
D. Ulcerative colitis
E. Lympho-granuloma venereum – procto-colitis

76
215. The diagnosis most commonly likely to be established is:
A. Examination of fresh stool specimen
B. Sigmoidocopy and Proctoscopy + biopsy
C. Mantoux test
D. Serological Tests including CFT (complement fixation test)
E. Microscopy and culture of vaginal fluids
216. The treatment of choice for the patient in question 214 is:

A. A course of metronidazole lasting at least 10days

B. Preparation for the procedure of abdomino-perineal resection of


the rectum

C. Anti tuberculous chemotherapy – D.O.T

D. A 21 Day course of Oxytetracycline 2G/a day

E. De functioning iliac colostomy

217. A woman of 40 complains of lower abdominal pain, loss of


weight constipation

with periodic diarrhoea and tenesmus of two months


duration. Over the

past two weeks she has noticed rectal pain and vaginal
discharge.

Examination reveals an emaciated patient, anaemic with


abdominal distension;

the bowel sounds are normal. Granulomatus lesions with


fistulae are evident

around the anus and anal stenosis is evident on digital


examination:

The most likely diagnosis is:

 Anal carcinomatous stricture


 Melanoma of the anal canal
 Ano-rectal stricture from lymphogranuloma verereum
 Amoeboma of the rectum

77
 Ano-rectal tuberculous stricture

218. The diagnosis can be most readily established by:

A. Examination of fresh specimen of stool

B. Sigmoidoscopy and proctoscopy + biopsy

C. Stool culture

D. Barium enema examination

E. Serological tests including compliment fixation tests

219. Diverticular disease of the colon occurs as a result of:


A. Out-pouchings of mucosa through the muscularis propria-pulsion
diverticula
B. Out-pouchings of mucosa and submucosa through the muscularis
propria-pulsion diverticula
C. Out-pouchings of mucosa submucosa through the muscularis
propriae along the blood vessels.
D. Out-pouchings of mucosa – traction diverticula
E. Out-pouchings of mucosa and submucosa - traction diverticula
220. The aetiological factors associated with diverticular disease of
the colon

includes which of the following combinations:

 Advancing age, increasing dietary fibre, increasing transit time and


increasing
colonic pressures
 Advancing age, decreasing dietary fibre, increasing transit time
and
increasing colonic pressures
 Increasing colonic pressures, increasing dietary fibre, increasing
transit
time, increasing content of spasmogens

78
 Increasing obesity, decreasing dietary fibre, increasing age,
decreasing
bowel content of spasmogens
 Advancing age, increasing colonic pressures, increasing dietary
fibre,
decreasing transit time

221. The macroscopic pathological changes in the colon in


diverticulosis include
which of these combinations:

 Muscular coat thickening, luminal narrowing, bowel shortening


and
accumulation of mesenteric and colonic fat

 Muscular coat thickening, bowel shortening, luminal dilatation,


oedema
of mesentery
 Attenuation of mucosa, luminal narrowing, fat accumulation in
mesentery,
muscular thickening
 Fat accumulation, oedema, luminal narrowing, muscle coat
attenuation
 Muscle coast thickening, distal luminal dilatation, bowel
shortening, fat
accumulation
222. A 75 year old man presents in the Emergency Room with

79
Intermittent attacks

of left sided abdominal pains, passage of hard stools and


mucus. There is a

mild systemic upset and on examination the left colon was


palpable and tender.

Rectal examination was unremarkable.

The most likely diagnosis is:

 Ischaemic colitis
 Irritable bowel syndrome
 Diverticulitis
 Paracolic abscess
 Lymphogranuloma venereum
223. The diagnosis is most readily established by:

A. Sigmoidoscopy
B. Colonoscopy
C. Barium meal and follow through
D. Barium enema
E. Serological Tests including complement fixation test

80
CHAPTER 38
SMALL AND LARGE INTESTINES
ANSWERS

 D 39. A, C, D, E 77. A. E
115. A, B, C

 A, D, E 40. A, B, C, D 78. A, C, E
116. A, C, D, E

 B, E 41. A, B, C 79. E
117. C

 D 42. B, D, C 80. A, B, C
118. D, E

 C, E 43. A, B, D 81. A, B, C, D
119. A, C

 A, B, E 44. A, B, D 82. A, E
120. A, D, E

 D 45. B, C, D, E 83. A, C, D
121. A, C, D

 C, D 46. A, C 84. A, C
122. B, C, D

 A, C, E 47. A, B, C 85. B, D, E
123. A, C, E

 B 48. B, C, E 86. A, C
124. B, D

 B, E 49. A, D, E 87. C, E
125. C, E

 C, D 50. A, B, D 88. B
126. A, B, C, D, E

 A, B, C 51. A, B, C, D, E 89. C
127. D

 A, B, D, E 52. A, B, C, D, E 90. A, B, C

81
128. C, E

 B, D, E 53. B, D 91. A, B, E
129. B, D

 A, C, E 54. C 92. E
130. A, C

 A, C 55. B, C 93. A, D
131. A, C, D

 A, C 56. A, C 94. C
132. B, C, D

 A, B, E 57. A, B, C, D 95. A, B, C, E
133. C, E

 B, D, E 58. B, C, D 96. B, D
134. A, D

 A, C, E 59. A, B, C, D, E 97. A, C, E
135. A, C, D, E

 A, B, D, E 60. C, D, E 98. E
136. B, C

 B, D, E 61. A, C, D, E 99. A, B, C, D
137. B, E

 A, B, C, E 62. D 100. A, C
138. A, B, D, E

 A, B, C, D 63. A, D 101. B, C, E
139. B, C

 A, B, C 64. B, C, E 102. A, B, C, D, E
140. C, D

 B 65. A, C 103. B, C
141. A, B

 A, D, E 66. B, C, D, E 104. B, D
142. A, C, D

 A, C, D 67. A, C, E 105. A, B, D, E
143. B, D, E

 B, C, E 68. A 106. C
144. A, B, C

 B, D 69. B, D 107. A, C, E

82
145. A, C, D

 A, B, D 70. B, C 108. A, B, D, E
146

 B, C 71. A, D 109. A, C, E
147. A, D, E

 C 72. C, E 110. A
148. D

 B, C, E 73. A, B, E 111. A, C, E
149.A, B, C, D, E

 A, C, E 74. B, C 112. B, C, D
150. A, B, C

 B, D, E 75. A, C, E 113. C, E
151. B, D

 C 76. A, B, C, D, E 114. A, B
152. B, C, D

153. A, B, C, D, E
154. A, B, C, E
155. A, B, C
156. D
157. A, B, D
158. D
159. E
160. C
161. A, B, D
162. B, C, D
163. E
164. A
165. C, D
166. D
167. A, B, C
168. A, C, D, E
169. B, D, E
170. C, D, E
171. B, E

83
172. A, C
173. A, B, D, E
174. A, B, C, E
175. C, D
176. A, B, C, D
177. A, B, C
178. A, C, D
179. B, D
180. A, E
181. A
182. C, E
183. A, B, D, E
184. A, B, C, D, E
185. A, B, C, D, E
186. B
187. A, C, D
188. A, B, D, E
189. A
190. B
191. A, B, C
192. D
193. D
194. A
195. D
196. E
197. A
198. A, B, C, D, E
199. D
200. D
201. D
202. C
203. C
204. C
205. E
206. A
207. C
208. A
209. A
210. B
211. D
212. E
213. A
214. E
215. D
216. D
217. C
218. E
219. C
220. B

84
221. A
222. C
223. D

85
CHAPTER 40

PANCREAS

1. The most intimate posterior relation(s) of the neck of the pancreas is (are)
A. The aorta
B. The superior mesenteric vein and artery and portal vein
C. Splenic vein, the inferior vena cava and 2nd lumbar vertebra
D. The right crus of the diaphragm and the fibres of right psoas muscle
E. Splenic and inferior mesenteric veins
2. The “normal” pattern of the exocrine pancreatic ductal system – main
(Wirsung) duct joined by the minor (Santorini) duct which opens 2 cm
proximal to the main ductal entry into the duodenum-occurs in which percentage
of dissected bodies?
A. 80
B. 70
C. 60
D. 50
E. 40
3. Quantitatively and qualitatively the most important influence on pancreatic
exocrine secretion is the:
A. Intestinal phase
B. Vagal phase
C. Gastrin secretion
D. Cholecystokinin secretion
E. Output of 5 hydroxytryptamine
4. The following conditions follow exclusion of pancreatic juices from the
duodenum:
A. Profound gastric hypersecretion
B. Excessive losses of fat in the stool
C. Impaired absorption of vitamin B12
D. Nutritional hepatic damage
E. Chronic constipation
5. It is generally recognized that the diagnosis of pancreatic disease is
particularly difficult; which of these statements might account for this:
A. The organ is deep seated and completely retroperitoneal
B. It has a tremendous functional reserve even with 90-95% deficiency
C. It has a precarious blood supply
D. Close proximity to large vascular channels
E. Combination of exocrine and endocrine functions
F. Close proximity to the caeliac plexus.
6. In Pancreas Divisum:
A. The duct of Wirsung is diminutive, while the duct of Santorini assumes the major
channel of drainage
B. There is poorer drainage of t he organ through the minor papilla
C. There is high degree of association with recurrent pancreatitis
D. Recurrent abdominal pain is attributed to the anomaly
E. Levels of serum amylase and lipase are in the pathological range
7. In Ectopic pancreas:
A. The aberrant pancreatic tissue is in the stomach duodenum and jejunum
B. In 75% of cases the tissue is seen in the submucosa
C. In 75% of cases the lesions are symptomatic
D. Occurring in a Meckel’s diverticulum it produces the peptic ulcer syndrome
E. Sited in the small intestine, it may be the lead point of an intussusception
8. The following are considered regular pathological features of acute
pancreatitis:
A. Varying degrees of pancreatic ductal dilatation
B. Focal necrosis
C. Focal suppuration
D. Haemorrhagic areas
E. Gastric erosions
9. In normal individuals duodenal reflux into the pancreatic duct is
prevented by:
A. Oblique course of the pancreatic duct on its way to the duodenum
B. The tone of the duodenum
C. The activity of the sphincter of Oddi
D. Mucosal valves guarding the duct entrance
E. The valvulae coniventes of the mucosa
10. The principle(s) that activate(s) proteolytic enzymes in the pathogenesis
of most cases of acute pancreatitis is (are):
A. Increased serum activity of calcium ions
B. Infected bile
C. (Enteropeotidase)
D. Urogastrone
E. Phosphorylase A
11. The effect of activated pancreatic enzymes on the pancreas is the
production of:
A. Diffuse capillary, arteriolar and venous thrombosis
B. Diffuse small vessel haemorrhages
C. Interstitial oedema and acinar necrosis
D. Increased endothelial permeability
E. Capillary vasoconstriction
12. The effects of activated pancreatic enzymes on the pancreas include:
A. Increased tendency to platelet thrombus formation
B. Release of activated phosphorylase A attacking cellular membranes
C. Initiation of a reaction leading to release of kallidins and other
vasoactive amines
D. Increased cellular permeability
E. Necrosis

13. In most tropical countries the most frequently encountered aetiological


factor associated with acute pancreatitis is:
A. Cholelithiasis
B. Alcoholism
C. Idiopathic
D. Malnutrition
E. Diabetes
14. Fat necrosis in acute pancreatitis:
A. Is most numerous immediately around the pancreas
B. May also occur subcutaneously
C. Determines in its extent the degree of serum calcium depression
D. May occur subpleurally
E. May occur sub-pericardially
15. The serum amylase level is significantly elevated after:
A. Intravenous cholangiogram
B. Oral cholecystography
C. Percutaneous transhepatic cholangiography PTC with the Chiba needle
D. Endoscopic retrograde cholangiopancreatography ERCP
E. Superior mesenteric arteriography
16. Theoretically normal values of the serum amylase (Somogyi 100-150,
Wolgemouth 12-20) may be obtained in acute pancreatitis:
A. If the attack has destroyed the acinar cells
B. If a previous attack has resulted in massive glandular destruction
C. If determinations are carried out after three days
D. If there is an associated pancreatic tumour
E. If the patient is under 10
17. In patients with acute pancreatitis, hypocalcaemia and hypomagnesaemia
may occur as a result of:
A. Extensively distributed fat necroses
B. The use of glucagon in the therapy
C. Prolonged parenteral nutrition
D. The use of dextrose rather than saline infusions
E. Surgical intervention
18. A man of 35 is acutely ill with abdominal pain, vomiting and fever and
weakness. He has signs of peritoneal irritation and laboratory investigation
shows leucocytosis, some degree of hyperglycaemia and
glycosuria. The diagnosis may be:
A. Acute pancreatitis
B. Diabetic ketosis
C. Acute staphylococcal pneumonia with septicaemia
D. Perforation of a duodenal ulcer
E. Typhoid fever
19. Reduction of exocrine pancreatic secretion is highly desirable in the
treatment of acute pancreatitis and this can be achieved by:
A. Nasogastric decompression by means of a nasogastric tube
B. Administration of non-absorbable antacids, e.g. aluminium hydroxide
C. Regular doses of propantheline 30 mg I/M 8 hourly
D. Glucagon
E. Trasylol (aprotinin)
20. Exocrine pancreatic secretion may be reduced by:
A. Glucagon i/v
B. Cimetidine 200mg i/v 6 hourly
C. Oral acetazolamide 250 mg bd
D. Administration of mist magnesium trisilicate BPC 15 ml tds
E. Naloxone

21. The indications for operation in acute pancreatitis include:


A. Cases with doubtful diagnosis
B. Patients with readily correctable underlying pathology, e.g. impacted
common duct stone
C. Patients in whom conservative measures are showing no improvement
D. Hypocalcaemia
E. Severe shock
22. The commonest complication of acute pancreatitis of all forms is:
A. Pseudocyst formation
B. Pancreatic abscess formation
C. Acute renal failure
D. Hypocalcaemia
E. Gastric erosions

23. The end result of all forms of pancreatitis is:


A. Diabetes
B. Pancreatic ductal dilatation
C. Radiologically evident pancreatic calcification
D. Fibrosis with pancreatic insufficiency
E. Fatty degeneration of the pancreas
24. The current consensus of opinion on the mechanism of genesis of acute
pancreatitis holds that most cases can be explained on the basis of:
A. Obstruction of t he main pancreatic ductal outlet
B. Biliary reflux to or from the pancreatic duct
C. Reflux of duodenal contents into the pancreatic duct
D. Direct effect of toxic agents (e.g. alcohol) on the exocrine pancreatic cells
E. Systemically induced micro-infarcts in the pancreas

25. Which of these operations is most likely to be complicated by an attack of


acute pancreatitis:
A. Polya gastrectomy
B. Long arm T-tube drainage
C. Right Hemicolectomy
D. Thyroidectomy
E. Caesarian section
26. Which of these endocrine conditions has a causative relationship with
acute pancreatitis:
A. Cortisone and ACTH excess
B. Hyper thyroidism
C. Hyper parathyroidism
D. Insulinoma
E. Gastrinoma
27. The Pathological changes in acute Pancreatitis:
A. The fundamental pathological lesion in the pancreatitis as a diffuse vascular
occlusion and focal haemorrhages
B. Mild attacks of pancreatitis show as oedema
C. The hall mark of severe acute pancreatitis is glandular necrosis
D. Fulminating attacks are indicated by haemorrhages which may extend to the
retroperitoneal tissues
E. Suppuration is a regular feature of all stages of the pathological process
28. In most African countries acute pancreatitis is usually on aetiological
basic of:
A. Gall stone disease
B. Abnormal calcium metabolism e.g. hyperparathyroidism
C. An idiopathic process
D. Hyperlipidemia
E. None of the above

29. A man of 45 presents in the emergency room after a heavy meal with mild but
persistent, unrelenting epigastric pains radiating to the back, vomiting,
hicupping and constipation. The patient is fit, stout, anicteric and has persistent
epigastric tenderness with minimal guarding. There is no organomegaly and
patient has no past experience of dyspepsia.
The most likely diagnosis is:
A. Acute cholecystitis
B. Acute exarcerbation of duodenal ulcer
C. Slow leak of a peptic ulcer
D. Acute Pancreatitis (mild)
E. Early amoebic liver abscess
30. Which of these will most readily provide a lead to the diagnosis?
A. Full blood count
B. Abdominal ultrasound
C. Serum lipase estimation
D. Serum amylase estimation
E. Plain x-ray of the chest
31. A man of 40 presents with sudden onset of severe epigastric pains, repeated
vomiting, and persistent reteching. The attack, followed within half an hour of a
meal in which alcohol was not consumed. On examination his temperature is
39°C, pulse 96/min, BP110/70. He is tender in the epigastrium, with guarding and
rebound tenderness. The lung bases are dull to percussion and a pleural
effusion is detectable on the left side.
The most likely diagnosis is:
A. Perforated duodenal ulcer
B. Strangulating intestinal obstruction
C. Severe acute cholecystitis with gall bladder gangrene
D. Severe acute pancreatitis
E. Ruptured amoebic liver abscess
32. The diagnosis can most likely be confirmed by which of these investigations.
A. Upper GI endoscopy
B. Abdominal ultrasound
C. Abdominal CT scan
D. Estimation of serum C-reactive protein level
E. Serum calcium level
33. The treatment of choice in this patient is:
A. Preparation for early emergency laparotomy
B. Exhibition of bacteriocidal broad spectrum antibiotics
C. Resuscitation, antibiotic cover, nasogastric decompression
D. Resuscitation, antibiotic cover, bowel decompression, monitoring for need for
surgical intervention.
E. Resuscitation, bacteriocadal antibiotics, + trasylol
34. A 50 year old man, a known diabetic for 10 years, presents with severe unrelenting
abdominal pains of two days duration, vomiting, constipation. On examination he
is shocky, pulse 180/min BP 90/60, temperature 36°0°C. His abdomen is diffusely
tender with guarding and there is dark discoloration of the skin in the left flank.
The diagnosis is:
A. Dissecting aneurysm of the aorta
B. Intestinal obstruction with gangrenous bowel
C. Diabetic ketoocidosis
D. Fulminating acute pancreatitis
E. Severe acute cholecystitis with septic shock (SIRS)
35. The indicated treatment here is:
A. Resuscitation with IV fluids and implementation of the Alberti regime
B. Resuscitation with blood transfusion and antibiotics
C. Nasogastric decompression and monitoring for abdominal signs
D. Resuscitation, with IV fluids, bacteriocidal antibiotics preparation for bowel
resection
E. Resuscitation with IV fluids, bacteriocidal antibiotics, laparotomy for pancreatic
necrosectomy
36. In the confirmation of the diagnosis of acute pancreatitis urine estimations of
enzymes remain elevated for much longer; which is the most dependable urinary
test currently:
A. 24 hour total urinary amylase out put
B. 24 hour total urinary lipase out put
C. Urinary amylase excretion rate
D. Amylase/creatinine clearance ratio
E. Urinary amylase strip test (Rapignost-amylase)
37. The most useful diagnostic imaging procedure in acute pancreatitis is:
A. Plain x-rays of the abdomen
B. Ultrasonography
C. T99 Scentigram
D. CT Scan
E. Contrast Enhanced Computer Tomogram (CECT)
38. Which of these imaging techniques permits ready detection of pancreatic
necrosis?
A. Plain x-rays of the abdomen
B. Ultrasonography
C. T99 Scentigraphy
D. CT Scan
E. Contrast enhanced CT

39. By the leading systems of grading acute pancreatitis (Glasgow and APACHE II)
which of the following statements on proportions of mild pancreatitis is true:
A. 80-90% of cases follow a mild course
B. 70-80% of cases follow a mild course
C. 60-70% of cases follow a mild course
D. 50-60% of cases follow a mild course
E. 40-50% of cases follow a mild course
40. By the leading systems of grading acute pancreatitis (Glasgow and APACHE II)
which of the following statements in proportion of severe cases of pancreatitis is
true:
A. 10-20 % present as severe attack with mortality of 20-40%
B. 25-35% present as severe attack with mortality of 20-40%
C. 36-40% follow a severe attack with morality of 20-40%
D. 40-50% follow a severe attack with mortality of 20-40%
E. 50-60% follow a severe attack with mortality of 20-40%
41. By the Glasgow modification of the Ransom Criteria for Grading acute
Pancreatitis which of the following parameters reliably predict severe acute
pancreatitis:
A. Pa O2 < 60minHg
B. Serum urea > 16mmol/L
C. White cell count > 15 x 109/L
D. Glucose (FBS) > 10mmol/L
E. Calcium (serum) < 2mmol/L
F. Serum Albumin < 32 G/L
G. Serum AST (GOT) > 200U/L
H. Serum Lactic dehydragenase > 60U/L

42. The management of acute pancreatitis:


A. Takes place in an intensive care or high dependency setting
B. Involves initial resuscitation
C. Requires concurrent periodic review of clinical and biomedical data
D. Entails a repair phase for the cumulative damage to the pancreas and the rest of
the body
E. Is incomplete without rehabilitation of the gland and patients life style
43. Evidence based justification has been established for which of these procedures
usually carried out as part of the treatment of the patient with mild acute
pancreatitis:
A. Energetic fluid and Electrolyte Replacement
B. Suppression of Pancreatic Secretions
C. Suppression of acid output with (H2RA), anticholinergic agents, glucagone, and
octreotide
D. Use of antibiotics to prevent onset of sepsis and abscesses
E. Routine use of trasylol in all cases
44. Total Parenteral Nutrition (TPN) in acute pancreatitis:
A. Avoids pancreatic stimulation while ensuring adequate nutrition
B. Should be deployed for all grades of acute pancreatitis
C. Should be reserved for severe and fulminating cases
D. Should be used in mild cases only
E. Should as much as possible be avoided because of attendant complications
45. Trasylol (Aprotinin):
A. Is a specific trypsin, kalikrein, and bradykinin inhibitor
B. Produces improvement in clinical signs in acute pancreatitis given in adequate
doses
C. Has not been demonstrated to show survival benefits in acute pancreatitis
D. Is more effective given orally c.f. parenterally
E. Should be avoided on account of associated thrombogenic potential
46. Peritoneal lavage in acute pancreatitis:
A. There is empirical evidence of benefit to all patients
B. Is known to benefit only patients with the complication of acute renal failure
C. Benefit particularly patients with jaundice and cyanosis
D. Is of proven benefit in severe acute pancreatitis
E. Should be avoided on account of unacceptable complication rate
47. Early (within a week) surgical intervention is indicated in acute pancreatitis:
A. Where the diagnosis is uncertain
B. When an epigastric mass is palpable
C. When jaundice becomes evident
D. When acute renal failure supervenes
E. When ultrasonography demonstrates gallstones in the gall bladder
48. Pancreatic resection in acute pancreatitis is considered absolutely indicated
when:
A. There is gross oedema of the gland
B. There is extensive “fat necrosis” demonstrable throughout the gland and the
peritoneum on MRI
C. CT evidence of pancreatic necrosis is demonstrable
D. The CT demonstrated necrosis is sampled and seen to be infected
E. There is persistent non- tender abdominal distention.
49. A man of 40 gives a history of severe persistent epigastric pains of two weeks
duration accompanied by intermittent fever repeated vomiting and constipation.
On examination he is icteric with patchy crepitations and dullness in the base of
the left lung. There is a firm non-tender epigastric mass which does not move
with respiration.
The most likely diagnosis is:
A. Pancreatic abscess
B. Free pancreatic effusion
C. Pseudocyst of the pancreas
D. Amoebic liver abscess
E. Non Hodgkin’s lymphoma
50. The treatment of choice is:
A. Immediate drainage of the mass by aspiration under CT control
B. Conservative management with monitoring of the mass until it regresses completely
C. Immediate drainage under ultrasonic control
D. Cysto-gastrostomy
E. External drainage of the mass
51. A man of 40, an SS genotype sickle cell patient, presents with an acute episode
of persistent epigastric pain and vomiting following a meal; he does not admit to
alcohol consumption. The pain settles after a week of withholding feeds but
recrudesces with resumption of feeding with spiking temperature. He is very ill
on examination with a temperature of 39°C. A tender mass is palpable in the
epigastrium extending into the Lt. hypochondrium
The most likely diagnosis is:
A. Slow leak from a gastric ulcer
B. Splenic abscess
C. Acute pancreatitis with abscess formation
D. Acute pancreatitis with pseudocyst
E. Strangulated epigastric hernia
52. The treatment of choice is:
A. Resuscitation, exhibition of antibiotics, expectant treatment
B. Urgent laparotomy
C. Resuscitation, exhibition of antibiotics, CT controlled exploration of the mass
D. Resuscitation, exhibition of antibiotics, laparotomy to explore the mass
E. Conservative measures with full Triple therapy

53. A 45 year old alcoholic presents with severe epigastric pains, much retching and
vomiting of 48 hours duration. He is seen to be very pale with distending,
doughy, mildly tender abdomen with fluctuating signs. There is evidence of
shifting dullness and ultrasonography indicates presence of fluid in the
abdomen but the outlines of the liver, spleen and kidneys could not be
determined.
The most likely diagnosis is:
A. Bleeding Duodenal ulcer
B. Bleeding oesophageal varices
C. Acute Pancreatitis with rupture of rupture of an abscess
D. Acute Pancreatitis with spontaneous splenic rapture
E. Spontaneous haemoperitoneum from primary liver cell carcinoma
54. The prognosis in acute pancreatitis:
A. Has worsened in the past two decades
B. Is better with underlying gall bladder disease than with alcoholism
C. Is indifferent to the presence of met-haemoglobinaemia
D. Is much worse with operative intervention not-withstanding the preparation or
extent
E. Is not affected by mean levels of glucose attained in treatment

55. Hyperparathyroidism associated acute pancreatitis:


A. Is produced by activation of trypsinogen by calcium ions
B. May occur without radiologically evident pancreatic calcification
C. Should be suspected in severe attacks of pancreatitis with normal serum
calcium levels
D. Serum parathormone assays may prove helpful in diagnosis
E. Seldom present as a recurrent problem

56. Fat necroses in acute pancreatitis:


A. Represent areas of saponification of fatty acids released by lipase
activity
B. Are opaque, pale yellow lesions
C. Are most numerous immediately around the pancreas
D. May occur by metastatic spread in the subpleural, subpericardial or
subsynovial fat
E. Are related by their extent to the degree of serum calcium depression
57. The establishment of the diagnosis of acute pancreatitis in practice
depends on:
A. A careful analysis of the patient’s history and physical signs
B. A high index of suspicion on the part of the clinician
C. Radiological demonstration of pancreatic calcification
D. Appearance of carpopedal spasm
E. The finding of abnormal liver function tests
58. In fulminating pancreatitis:
A. Retroperitoneal haemorrhage frequently occurs
B. Diabetic ketosis may supervene
C. Granular and hyaline casts may appear in the urine
D. Jaundice is a regular feature
E. Cyanosis is particularly common in the young
59. In acute pancreatitis:
A. Elevation of serum amylase and lipase appropriately studied forms
the sheet anchor of the laboratory diagnosis
B. Persistent elevation of the serum amylase level suggests pseudocyst
Development
C. The height of the serum amylase level bears no relation to the severity
of the disease
D. The amylase level bears no relation to renal status of the patient
E. The amylase level remains elevated in the peritoneal exudates longer
60. In acute pancreatitis:
A. The amylase level rises above normal within a few hours of onset
B. Persistent elevation of the serum amylase level suggests continued
ductal obstruction
C. The serum lipase level is a more reliable diagnostic feature than the
amylase
D. The rapidity of return of the serum amylase level to normal is of little
prognostic significance
E. The urinary amylase level is elevated for several days
61. In the acute abdomen the serum amylase may be significantly raised:
A. By complicating acute renal failure
B. When acute exacerbation of a peptic ulcer is the precipitating factor
C. When a dose of pethidine had been administered by the family physician
D. When a dose of atropine is given as part of the premedication
E. When the patient is presumed to have associated diabetic ketosis
62. In the acute abdomen the serum amylase may be significantly raised:
A. When there is associated stress gastric and duodenal ulceration
B. When acute cholecystitis is the cause
C. When there is complicating subphrenic abscess
D. In the presence of acute upper intestinal obstruction
E. When there is a perforated peptic ulcer
63. The urinary amylase level in acute pancreatitis:
A. May remain elevated for about a week after the attack
B. Usually more helpful in confirmation of diagnosis than the serum amylase
C. Is independent of the urinary creatinine clearance
D. Is reliably estimated by studies of the hourly amylase excretion rate
E. Is elevated in patients with associated diabetic ketosis
64. In patients with acute pancreatitis:
A. The serum calcium level may occasionally be of diagnostic value
B. The serum calcium level may be of prognostic value
C. The evidence of hypocalcaemia is noted by the third day of the illness
D. High serum calcium levels may be a pointer to the aetiological basis
E. Serum calcium and potassium levels are closely correlated
65. In patients with acute pancreatitis:
A. Calcium may appear in the urine
B. Hypocalcaemia is often potentiated by associated hypomagnesaemia
C. Levels of serum calcium and amylase are closely correlated
D. High calcium levels should suggest associated parathyroid disease
E. The calcium levels are influenced by exhibition of glucagon
66. In acute pancreatitis:
A. There is usually a profound leucocytosis
B. Anaemia is a regular feature
C. The liver function tests may be normal
D. Methaemoglobinaemia is an index of severity of the illness
E. There is transient hyperglycaemia and glycosuria
67. In acute pancreatitis:
A. The average leucocyte response is about 10x109/dl
B. Anaemia is rare
C. Hyperbilirubinaemia frequently occurs
D. Methaemoglobinaemia distinguishes oedematous from haemorrhagic
form of the disease
E. The hyperamylasaemia induces hepatic glycogenolysis
68. In acute pancreatitis:
A. Radiologically evident pancreatic calcification may be seen in 20%
of patients with relapsing illness
B. Peritoneal fluid is indicated by generalized fuzzy appearance of the
plain X-ray film
C. Diagnostic paracentesis is contraindicated
D. ECG changes of ST segment depression and T-wave inversion and
Q-T interval prolongation can be expected
E. Where indicated peritoneoscopy is more preferable to laparotomy
69. In acute pancreatitis:
A. Plain X-rays of the abdomen are generally non-contributory
B. Abdominal paracentesis may be of diagnostic and therapeutic value
C. ECG changes of myocardial ischaemia can be expected
D. There are also frequently ECG changes of hyperkalaemia
E. Where indicated laparotomy is more preferably to peritoneoscopy
70. The principles of management of acute pancreatitis include:
A. Ensuring physiological rest of the inflamed organ
B. Selective use of bactericidal broad spectrum antibiotics to prevent
complications
C. Adequate fluid and electrolyte replacement
D. Early administration of appropriate proportions of crystalloids and
colloids to guard against shock
E. Routine use of total parenteral nutrition in patients not settling within
a week to ten days

71. The principles of management of acute pancreatitis include:


A. Maintaining pancreatic secretions by means of secretin and
pancreazymin
B. Early return to oral alimentation as the patient desires
C. Judicious use of steroids in large doses in severely ill patients
D. Watchfulness for early detection of complications
E. Resort to laparotomy in patient presenting diagnostic difficulties
72. Urgent surgical intervention is needed in which complications of acute
pancreatitis?
A. Pseudocyst formation
B. Pancreatic abscess formation
C. Intestinal obstruction
D. Splenic rupture
E. Gastro-intestinal bleeding from gastric erosions
73. The pseudocyst that develops as a complication of acute pancreatitis:
A. Most frequently becomes evident about 3 weeks after the onset of
the pancreatitis
B. Usually occurs on basis of persistent pancreatitic ductal obstruction
C. Shows marked predilection for gall stone pancreatitis
D. Has a tendency to regress spontaneously
E. May be surgically treated in the appropriate cases best by
cystogastrostomy
74. The pseudocyst that develops as a complication of acute pancreatitis:
A. Becomes manifest within ten days of onset of the illness
B. Shows a marked predilection for alcoholics
C. Should not be tackled surgically until the foramen of Winslow is
suitably closed off by adhesions
D. May be suspected from continued elevation of the serum
amylase level
E. May be surgically treated best by external drainage
75. The prognosis in acute pancreatitis is:
A. Better with underlying gall bladder disease than alcoholism
B. Poorer in patients with methaemeglobinaemia
C. Not adversely affected in patients adequately prepared and operated
on by generally accepted principles
D. Determined by the degree of correction of the fluid and electrolytes
disturbance
E. Adversely affected by the onset of renal failure
76. The two principal features of chronic pancreatitis are:
A. Abdominal pain and vomiting
B. Abdominal pain and jaundice
C. Abdominal pain and pancreatic insufficiency
D. Abdominal pain and diabetes
E. Abdominal pain and diarrhoea
77. In patients with chronic pancreatitis estimation of duodenal secretions
shows:
A. Bicarbonate below 90 mmol/1 and amylase below 6 1u/kg body
wt. 80 mins after secretin and pancreozyprim
B. Bicarbonate below 80 mmol/1 and amylase below 5 1u/kg body
wt. 80 mins after secretin and pancreozyprim
C. Bicarbonate below 70 mmol/1 and amylase below 4 1u/kg body
wt. 80 mins after secretin and pancreozyprim
D. Bicarbonate below 60 mmol/1 and amylase below 3 1u/kg body
wt. after secretin and pancreozyprim
E. Bicarbonate below 50 mmol/1 and amylase below 2 1u/kg body
wt. 80 mins after secretin and pancreozyprim
78. Operations in chronic pancreatitis are rather disappointing but they are
indicated in patients with:
A. Recognizable causally related pathology
B. Intractable pain
C. Progressive weight loss unresponsive to general corrective measures
D. Jaundice
E. Severe steatorrhoea
79. Pseudocyst of the pancreas may be complicated by:
A. Secondary infection
B. Erosion of adjacent vessels leading to localized or free peritoneal
haemorrhage
C. Rupture
D. Displacement of adjacent organs-stomach, transverse colon
E. Obstructive jaundice
80. Most cases of chronic pancreatitis are preceded by an acute episode(s)
of pancreatitis. Epidemiologically the most important factor that drives the
process is:
A. Nutritional
B. Metabolic anomalies, (gall stones, hyper calcaemias, etc)
C. Hereditary influences
D. Idiopathic
E. None of the above
81. The principal change in chronic pancreatitis is progressive exocrine
destruction and replacement by fibrous tissue. Which of the following features
can be traced to this:
A. Ductal dilatation
B. Segmental portal hypertension
C. Elevated pressures within the ductal system
D. Duodenal obstruction
E. Strictures in the transverse colon
F. Pancreatic calcification
G. Islet cell atrophy
82. The confirmation of the diagnosis of pancreatitis is a continuing challenge to the
clinician; currently which procedure provides the most useful information:
A. Plain x-rays of the abdomen PA and lateral
B. Ultrsonography
C. Examination of Duodenal Secretions
D. Contrast enhanced helical CT scan of the pancreas
E. ERCP
83. The principal challenge in the management of chronic pancreatitis is intractable
pain. What should be the surgeons first choice in control of this pain:
A. Functional neural block (Coeliac ganglion) with phenol
B. Pylorus preserving proximal pancreato-duodenectomy (PPPP)
C. Duodenum preserving resection of head of pancreas (DPRHP)
D. Longitudinal pancreato-jejunostomy
E. Total pancreatectomy or pancreato-duodenectomy
84. The jaundice which presents as part of the syndrome in chronic pancreatitis is
best managed by:
A. Cholecysto-jejunostomy with entero-entero anastomosis
B. Insertion of self-expanding metal stents into the common bile duct
C. Pancreatico-duodenectomy
D. Total pancreatectomy and choledocho-jejunostomy
E. Pancreatic transplantation
85. A 35 year old alcoholic is admitted with a severe attack of upper abdominal pain
and vomiting, initially productive of coffee grounds fluid, but in 24h changing to
frank haematemesis. He is feverish, jaundiced and gravely ill. On examination
a mass is felt to extend across the epigastrium to the Lt. hypochondrium; it is
tender tense, felt to move with respiration, and apparently pulsatile.
The most likely diagnosis is:
A. Leaking aortic aneurysm
B. Primary liver cell carcinoma
C. Alcoholic hepatitis with liver decompensation
D. Pseudocyst following an attack of acute pancreastitis
E. Amoebic liver abscess
86. The most useful investigation here is:
A. Liver function tests
B. Upper GI endoscopy
C. Barium meal examination
D. Ultrasonic scanning as well as CAT scan
E. Serum and urinary amylase estimation
87. The management of choice is:
A. Percutanous exploration of mass under CAT scan guidance
B. Resuscitation and expectant management
C. Laparotomy after resuscitation
D. Chemotherapy using adriamycin
E. Resuscitation, antibiotic therapy, rest to the bowel, monitoring size of mass and
eventual laparotomy
88. In periampullary carcinoma the jaundice:
A. Occurs early and is progressive
B. Occurs early, is often fluctuating and may be associated with significant
anaemia
C. Is late and progressive
D. Is usually transient and mild
E. Is associated with high fever, rigors and colic and persistent
89. In periampullary carcinoma:
A. The jaundice is intermittent in 25% of cases
B. Significant anaemia may be a feature
C. The stools may have a silvery grey appearance
D. Pain may be absent
E. Urinary urobilinogen is absent
90. The most reliable index of complete biliary obstruction is:
A. Absence of urobilinogen in the urine
B. Serum alkaline phosphatase levels of 40 King Armstrong Units
and above
C. Relative proportions of serum direct and indirect reacting bilirubin
D. Normal SGOT and SGPT levels in patients with direct hyperbilirubinaemia
E. Three days facal fat exceeding 18G on standard diet
91. Which of the following tests would be your first line investigation in
patients with suspected cholestatic jaundice?
A. Barium meal
B. Percutaneous transhepatic cholangiography (PTC)
C. Ultrasonography
D. Selective celiac angiography
E. Computerized axial tomography (CAT Scan)
92. The resectability rate (%) in cancers of head of pancreas is:
A. 10
B. 20
C. 30
D. 40
E. 50
93. The resectability rate (%) in periampullary cancers is:
A. 40
B. 50
C. 60
D. 70
E. 80
94. The most important factor that determines resectability of
pancreatic carcinoma (head) is:
A. Involvement of the subpyloric lymph nodes
B. Invasion of the pyloric region of the stomach
C. Duodenal invasion
D. Involvement of the portal vein
E. Involvement of the splenic and superior mesenteric veins
95. The insulin requirement after total pancreatectomy is surprisingly low
and is of the order of:
A. 10 units a day maximum
B. 30 units a day maximum
C. 50 units a day maximum
D. 70 units a day maximum
E. 90 units a day maximum
96. B-cell tumours producing hyperinsulinism:
A. Are in 75% of cases benign adenomas
B. Are in 12-18% of cases multiple
C. May rarely take the form of diffuse adenomatosis
D. May be ectopic in 2% of cases
E. Are often associated with lipomata
97. The most useful diagnostic investigation in suspected insulinoma
cases is:
A. Glucose tolerance test
B. Provocative fasting test
C. Multiple radio immuno assays of insulin in the blood during symptoms
D. Tolbutamide provocative test
E. L-leucine provocative test
98. Which of the following tests may be used to confirm the presence of
insulinoma?
A. Selective celiac angiography
B. Contrast pancreatography
C. Radioactive selenium photoscanning
D. Barium meal
E. Infra-red photoscanning
99. B-cell tumours of the pancreas may be conservatively managed by:
A. ACTH and hydrocortisone administration
B. Use of streptozotocin
C. Administration of alloxan and diazoxide
D. Administration of mitomycin
E. Administration of 5FU
100. The main pancreatic exocrine ductal system (Duct of Wirsung):
A. Receives tributaries at right angles from tail to head of gland
B. Is joined by the minor duct as it turns gently inferiorly
C. Is replaced by the minor duct (Santo rini) in 5-10% of people
D. Rather infrequently forms a common channel with the common
bile duct
E. Has an effective antireflux mechanism at its entry into the duodenum.
101. The exocrine secretion of the pancreas:
A. Measures between 1.5 and 2.51 a day
B. Has a pH of 7.0-7.5
C. Has similar sodium and potassium concentrations as plasma
D. Has amylolytic, lipolytic and proteolytic properties
E. Is in terms of quantity and quality of juice primarily under the control
of vagal stimulation
102. The endocrine cells of the pancreas:
A. Account for 1.5 per cent of the weight of the organ
B. Are irregularly scattered throughout the organ
C. Usually function as a homogenous unit
D. Are responsible for secretion of a wide spectrum of physiologically
active polypeptides
E. Are associated with specific clinical syndromes which remit with their
removal
103. The beta cells of the islets of Langerhans:
A. Comprise 70% of cells of each islet
B. Elaborate, store and secrete insulin
C. Undergo atrophy with pancreatic duct ligation
D. Are stimulated by elevation of blood sugar level
E. Release insulin under the influence of oral hypoglycaemic
agents-tolbutamide and sulphonyl ureas
104. Pathological features of cancers of the pancreas:
A. The cancers are predominantly adenocarcinomas (90%), arising from the ductal
system
B. There is a marked fibrous reaction
C. The lesion is enveloped in a shell of chronic pancreatitis
D. 80% of pancreatic cancers occur in the head region, with remaining
20% evenly distributed
E. Biologically cancers of the ampulla and lower biliary tract are less aggressive than
pancreatic cell cancers.
F. Local infiltration of adjacent organs and vessels is common
G. Intraductal papillary, mucinous neoplasms (IPMNS) benign and malignant are on
the increase.
H. Intraductal papillary, mucinous neoplasms (IPMNS) are more closely
associated with diabetes
105. In the diagnosis of carcinoma of the pancreas the first and most constant
symptom is:
A. Pain epigastric pain
B. Anorexia with weight loss
C. Jaundice
D. Steatorrhoea
E. Nausea and retching
106. In the diagnosis of carcinoma of the pancreas the relationship between
these symptoms is true:
A. The pain usually precedes the jaundice by several weeks
B. Painless jaundice is rare in pancreatic cancer
C. Painless jaundice is more often seen in periampullary carcinoma
D. Anorexia and weight loss precede the pain and jaundice
E. 80% of patients with jaundice of pancreatic origin have pain
107. The recommended sequence of diagnostic investigations in a patient
suspected to have pancreatic cancer is:
A. ERCP, FNA, CT, MRI
B. Abdominal Ultrasound with FNA, CT, MRI
C. CT, MRI, ERCP
D. Barium meal, Liver Function Tests, CT
E. Endoluminal ultrosonography, MR Cholangiopancreatography
108. In the patient with pancreatic cancer presenting with jaundice, pre-
operative biliary drainage using an endosopically placed stent has the
effect of:
A. Lightening the jaundice
B. Improving hepatic function
C. Increasing rate of sepsis in the post operative period
D. Improving overall survival rate post operatively
E. Interferring with technical procedures in the definitive surgery
109. Ten days after pancratico-duodenectomy for carcinoma of the head of the
pancreas a 50 year old man complains of epigastric pain that in a few hours
spreads to the rest of the abdomen. There is no nausea or vomiting. On
examination he is pale and temperature is 38C, the pulse is 110/min reg. BP
100/70. The abdomen is diffusely tender with variable guarding and there is no
palpable mass. The healed peritoneal drainage site is intact. Rectal examination
is unremarkable.
The most likely development is:
A. Anastomatic leakage – biliary, pancreatic, gastric
B. Intra abdominal abscess
C. Pancreatic fistula development
D. Secondary haemorrhage
E. Paralytic ileus
110. What is the most appropriate course of action?
A. Immediate surgical intervention – re-operative exploration
B. Resuscitation with IV infusions and further investigations
C. Resuscitation with IVs. further Vit. K injections, re-operation once patient is stable
D. Nasogastric intubation, bacteriocidal antibiotics, monitoring
E. Resuscitation with IVs. esomeperazole 40mg od for 2/52, monitoring
111. What are the options in the management of a 60 year old man with
carcinoma of the body of the pancreas which was found to be infiltrating the
portal vein at exploration?
A. Proceed with excisional surgery with reconstruction of the portal vein
B. The patient is allowed to recover with post operative exhibition of
Gemcitabine/Capecitabine
C. The abdomen is dosed, with administration of xeloda (5FU) and radiotherapy
D. Post operative course of radiotherapy
E. Symptomatic treatment
F. Palliative non operative regime
112. The 5 year survival of a 55 year old man with a non-metastatic carcinoma
of the head of the pancreas that undergoes a successful pancreatitico
duodenectomy is:
A. Less than 1 percent
B. 1-5 percent
C. 6-20 percent
D. 25-30 percent
E. 30-35 percent
113. The 5 year survival of 50 year old man with a non-metastatic carcinoma of
the ampulla of Vater that undergoes a curative pancreaticoduodenectomy:
A. Less than 5 percent
B. 5-10 percent
C. 11-15 percent
D. 16-25 percent
E. 30-35 percent
114. The prognosis for a locally advanced non-metastatic carcinoma of the
head of the pancreas treated by tipple bypass surgery is of the order of:
A. 2-4 months
B. 5-6 months
C. 7-8 months
D. 9-10 months
E. 11-12 months
115. Concerning β cell Tumours of the Pancreas:
A. Most are benign adenomas (75%)
B. 12-18% are multiple
C. About 2% of the adenomas are ectopic- in omentum, mesentery, stomach and
intestinal wall
D. About 12% are malignant
E. Further 12% show features of vascular invasion, and haemorrhages
116. Dependable symptoms of islet tumours include:
A. Adrenaline-like response-tachycardia, sweating, tremors
B. Cerebral symptoms-headaches, loss of consciousness, convulsions
C. Attacks are triggered by fasting
D. The attacks are relieved promptly by sugar
E. Progressive weight gain is a constant feature
117. Whipple’s Triad which provides the basis for confirmation of β cell islet
tumours of the pancreas is made up of:
A. (i) Precipitation by exercise, (ii) blood sugar <60mg/100mls in attack,
(iii) gradual relief of symptoms with glucose
B. (i) Precipitation by exercise, (ii) blood sugar < 50mg/100mls in attack,
(iii) prompt relief of symptoms with glucose
C. (i) Precipitation by fasting (ii) blood sugar <40mg/100mls with attack
(iii) indifference to ingestion of sugar
D. Precipitation by exercise, (ii) elevation of catecholamine during attack
(iii) prompt relief with sugar
E. Precipitation with fasting (ii) Elevation of insulin level in attack,
(iii) indifference to sugar administration
118. A 40 year old woman presents with periodic early morning pre-prandial
attacks of sweating, palpitations, tremors, headaches dizziness and blurring of
vision. There are no attacks in the course of the day. On examination she is a
fit, well built woman with normal haematological
findings and normal routine blood chemistry.
The most likely diagnosis is:
A. Hepatogenic hypoglycaemia – primary liver cell cancer
B. Β cell endocrine tumour of the pancreas
C. Functional hypoglycaemia
D. Sub pleural mesothelioma
E. Retroperitoneal sarcoma
119. The diagnosis can best be clinched by:
A. Performing glucose Tolerance test
B. CT. Scan of the chest
C. CT Scan of the abdomen
D. Verifying if Whipple’s Triad holds
E. Provocative test using I/V Sodium Tolbutamide
120. The diagnosis can best be clinched by:
A. Measuring plasma insulin levels by radio-immunoassay
B. Provocative Test using I/V tolbutamide
C. Provocative Test by administering I/V l - lencine
D. Verifying if Whipples’ Triad holds
E. Contrast enhanced helical CT scan.
121. The treatment of choice for β cell islet tumours of the pancreas:
A. Conservative management with stomatostatin, (octreotide)
B. Distal subtotal pancreatectomy
C. Surgical exploration, identification and excision of tumour
D. External beam radiotherapy
E. Combination therapy with Stretozotocin, doxorubicin and 5 FU
122. Concerning the features of a gastrinoma-Zollinger-Ellison Syndrome:
A. Produced by non-β islet cell tumour of the pancreas
B. There is slight male preponderance
C. May occur at any age above 7, but usually in between 3rd and 5th
decades
D. 60% are malignant, 35 are benign and 5% islet cell hyperplasia
E. Most of the associated duodenal ulcers are located in the1st part of the duodenum
but a few are post bulbar or jejunal
123. A man of 35 presents with periodic intractable epigastric pains some 2-3 hours
after meals over a period of a year. The attacks are associated at times with
incapacitating diarrhoea and weakness. On examination he is dehydrated, the
abdomen is mildly distended with epigastric tenderness and guarding and bowel
sounds are absent. P.R. shows watery stools only.
The most likely diagnosis is:
A. Amoebic colitis
B. Irritable bowel syndrome
C. Ulcerative colitis
D. Crohn’s Disease
E. Zollinger-Ellison Syndrome
124. Which of these tests will clinch the diagnosis
A. Upper GI Endoscopy
B. Gastric Secretory Tests
C. Serum gastrin estimations with gastrin stimulation (secretin)
D. Serum calcium and parathyroid hormone levels
E. Abdominal CT Scan
125. Treatment in this patient should be directed at:
A. At surgical procedures on the target organs here-stomach and
duodenum
B. Location and excision of the causative tumour
C. Exhibition of protracted course of H2 receptor antagonists (H2 RA)
D. Course of proton pump inhibitor-Escomeperazole
E. Streptozotozin and 5 FU therapy
126. Pancreatic cancer:
A. Is important in cancer epidemiology in many tropical countries
B. Is distributed evenly throughout the gland
C. When it affects the head, is usually difficult to differentiate from
ampullary and periampullary lesions
D. May on occasion induce portal hypertension with oesophageal
varices
E. May in 10 per cent of cases be associated with overt diabetes
127. Pancreatic cancer:
A. Accounts for 3.0% of all malignant tumours in West Africa
B. Is predominantly an adenocarcinoma – over 90% of cases, with
fibrous reaction
C. Has characteristically spread locally, by lymphatic and blood stream
on presentation
D. May in approximately 20% of cases be associated with hyperglycaemia
and glycosuria
F. Occurs four times as frequently in diabetics as in the general population
128. Pancreatic cancer:
A. Is distributed predominantly in the head (70 per cent) with the
30 per cent in rest of body
B. Is always an adenocarcinoma
C. May be associated with migrating thrombophlebitis
D. May in approximately 40 per cent of cases be associated with
hyperglycaemia and glycosuria
E. Should be suspected in severe refractory diabetes beyond the sixth
decade requiring insulin for maintenance

129. In the diagnosis of pancreatic cancer:


A. A high index of suspicion is a distinct advantage
B. It is useful to note its peak incidence in the 40-50 age group
C. The nature of the pain as a continuous epigastric dull ache radiating
to the back is important
D. Jaundice is a regular feature of head lesions and follows after several
weeks
E. Cholangitis with fever and rigors is a marked feature of the jaundice
130. In the diagnosis of pancreatic cancer:
A. Its predilection to elderly makes (60-70) is a characteristic feature
B. Pain is a prominent feature in 80 per cent of cases
C. Anorexia and weight loss are the first and most constant symptoms
D. Jaundice occurs in 75% of head lesions
E. The jaundice occurring in the pancreatic body tumour is more
frequently from hepatic hilar lymph node compression
131. In carcinomas of head of pancreas:
A. The liver is palpable in 60% of patients at presentation
B. The gall bladder is palpable in 50-60% of jaundiced patients
C. Presentation is earlier than body and tail lesions
D. The Charcot triad of symptoms is seldom encountered
E. Ascites is nearly always a late phenomenon
132. The pro-operative preparation of the jaundiced patient includes:
A. Adequate rehydration and correction of electrolytes disturbance
B. Intensive hyperalimentation to replenish glycogen stores
C. Correction of hypoprothrombinaemia by parenteral vit. K therapy
D. Maintenance of adequate urine flow by means of osmotic diuretic, e.g.
mannitol
E. Pre-operative parenteral dose of cefoxitin in patients with calculous
obstruction

133. The pre-operative preparation of the jaundiced patient includes:


A. Massive saline purge to rid the bowel of ammoniacal products
Of bacterial digestion
B. High protein high fat diet to provide adequate energy
C. Correction of hypoprothrombinaemia by oral vit. K therapy
D. Pre-operative exhibition of antibiotics in all cases
E. Routine transfusion of whole blood and blood products in all cases
134. B-cell tumours of the pancreas producing hyperinsulinism:
A. Are in 75 per cent of cases benign adenomas
B. May in 12 to 18 per cent of cases be multiple
C. May very rarely take the form of diffuse adenomatosis
D. May in 2 per cent of cases be ectopic
E. If malignant may be soft, haemorrhagic and with tendency to vessel
invasion
135. The diagnosis of B-cell pancreatic tumour hyperinsulinism depends on:
A. Symptoms being precipitated by fasting or strenuous exercise
B. Fall of blood sugar below 2.7mmol/l
C. Symptoms being relieved without delay by administration of glucose
D. Symptoms of hypoglycaemia on adrenaline stimulation
E. A fasting test lasting up to 72h but seldom carried beyond 30h
136. Spontaneous hypoglycaemia may:
A. Result from functional or organic hyperinsulinism
B. Be attributable to abnormalities of the pituitary, the adrenal and the
central nervous system
C. Be diagnosed when the conditions enunciated in Whipples triad are
Satisfied
D. Be complicated by excessive weight gain
E. Logically be treated only by surgical excision of the lesion

137. Spontaneous hypoglycaemia may:


A. Result from hepatogenic hypoglycaemia
B. Be attributable to wide-spread septicaemia
C. Be diagnosed in patients with flat glucose tolerance tests
D. Be complicated by irreversible brain damage
E. Be treated by radiotherapy delivered to the causative islet cell tumour
CHAPTER 40
PANCREAS
ANSWERS

1. B 38. E 75. A, B, C, D, E 112. C


2. B 39. B 76. C 113. E
3. A 40. B 77. A 114. E
4. A, B, C, D 41. A, B, C, E, F, G, H 78. A, B, C, D 115. A, B, C, D, E
5. A, B 42. A, B, C, D, E 79. A, B, C, D, E 116. A, B, C, D, E
6. A, B, C, D 43. A 80. E 117. B
7. A, B, D, E 44. A 81. A, B, C, D, E, F, G 118. B
8. A, B, C, D 45. A, B, C 82. D 119. D
9. A, B, C, D 46. D 83. A 120. D
10. C 47. A 84. B 121. C
11. A, B, C, D 48. D 85. D 122. A, B, C, D, E
12. A, B, C, D, E 49. C 86. D 123. E
13. B 50. B 87. E 124. C
14. A, B, C, D, E 51. C 88. B 125. B
15. D 52. D 89. A, B, C, D 126. A, C, D, E
16. A, B, C 53. D 90. A 127. A, B, C, D
17. A, B, C, D 54. B 91. C 128. A, C, E
18. A, B, C 55. A, B, C, D 92. A 129. A, C, D
19. A, B, C, D, E 56. A, B, C, D, E 93. D 130. A, B, C, D, E
20. A, B, C, D 57. A, B 94. D 131. A, B, C, D, E
21. A, B, C 58. A, B, C 95. B 132. A, B, C, D, E
22. A 59. A, B, C, E 96. A, B, C, D 133. C
23. D 60. A, B, E 97. C 134. A, B, C, D, E
24. C 61. A, B, C 98. A, B, C 135. A, B, C, D, E
25. A 62. A, B, D, E 99. A, B, C 136. A, D, E
26. A, B, C 63. A, B, D 100. A, B, C, D, E 137. A, B, D
27. A, B, C, D 64. A, B, C, D 101. A, D
28. E 65. B, D, E 102. A, B, C, D, E
29. D, 66. B, C, D, E 103. A, B, E
30. C 67. A, C, D, E 104. A, B, C, D, E, F, H
31. D 68. A, B, D 105. B
32. C 69. B, C, E 106. A, B, C, D, E
33. D 70. A, B, C, D, E 107. B
34. D 71. C, D, E 108. A, B, C
35. E 72. B, D 109. D
36. D 73. A, B, D, E 110. C
37. E 74. B, C, D 111. C, F
CHAPTER 41

SPLEEN AND PORTAL HYPERTENSION

1. The long axis of the spleen is in the line of the:


A. Splenic artery
B. Gastro-splenic omentum
C. Tenth rib
D. Eleventh rib
E. Coeliac axis

2. The spleen is held in place by:


A. The peritoneal reflections from the greater curve of the stomach
B. The peritoneal reflections of the diaphragm
C. The lienorenal ligament
D. Left triangular ligament
E. The lesser omentum

3. Which of the following structural changes accompany (ies) splenic


enlargement?
 The gastro-splenic omentum is taken up
 The splenocolic ligament is taken up
 Contact with the anterior abdominal wall is never lost
 The long axis is obliquely toward the umbilicus
 The lesser omentum is taken up

4. The Normal spleen has the following morphological features:


A. Anatomically lies between the 9th and 11th ribs (L)
 Anatomically lies between the 7th and 10th ribs (L)
 Weights 200G
 Weights 150G
 Measures 2.5cm by 7.5cms
 Measures 3.5cm by 9.5cm
 Has a hilum which is closely related to the tail of the pancreas
 Is held in position by the peritoneal reflections – gastrosplenic and lieno-
renal ligaments
 Is held in position by the lesser omentum and greater omentum

5. Which of these change(s) occur with enlargement of the spleen?


A. The gastrosplenic omentum is taken up
 The lesser omentum is taken up
 The greater omentum is fore shortened
 The spleen projects beyond the left hypochondrium
 The direction of enlargement is towards the left iliac fossa
 The enlarging spleen retains its contact with anterior abdominal wall

1
 The enlargement allows inter position of bowel between the organ and
the abdominal wall
 The enlarging spleen loses its contact with the splenic colic flexure

6. Accessory spleens:
A. Are found in at least 10% of subjects
B. Are more common in adult life
C. Undergo spontaneous atrophy in early childhood
D. Should be looked for and excised in all patients coming to splenectomy
for Trauma
E. Are most commonly found in the hilum of the spleen, region of the
pancreas, along the greater curvature of the stomach
F. Are most commonly found around the left testis
G. Often occur in the left broad ligament and pouch of Douglas

7. Which of the following blood elements are removed by the spleen?


A. Mishapen erythrocytes or spherocytes
B. Erythrocytes with nuclear content, e.g. Howell Joly bodies
C. Erythrocytes exceeding average age of 120 days
D. Leucocytes after a maximum age of 12 h
E. Sickled red cells

8. The following are readily extracted from the circulation by the spleen:
A. Leucocytes after a maximum age of 1 h
B. Platelets in general
C. Circulating lipoid elements
D. Erythrocytes of average age of 30 days
E. Plasma proteins

9. The main extramedullary site of haemopoiesis between the 5th and 8th
month of foetal life is:
 The liver
 The kidney
 The spleen
 Haemic nodes
 Suprarenals

10. Extra medullary haemopoiesis physiologically occurs in the spleen


during:
A. 1st - 2nd month of foetal life
 3rd - 4th month of foetal life
 5th - 8th months of foetal life
 9th month of foetal life
 1st month of foetal life only

2
11. Which of these changes after splenectomy (is) are true:
A. Decrease in T and B lymphocytes
 Stable but low serum 1g M levels
 Temporary alteration in 1gG and 1g A
 Decrease in opsonin activity
 Decrease in complement levels esp. C3
 Deficiency in phagocytosis

12. The risk of over whelming post splenectomy sepsis is highest in patients
requiring
splenectomy for:
A. Trauma
 Thrombocytopenic purpura
C. Hereditary` spherocytosis
D. Thalassaemia
E. Non Hodgkin’s lymphoma

13. Two weeks after splenectomy for abdominal trauma a 38 year old man
returns
for follow up complaining of fever and painful swelling of the left leg of
3 days duration. Three days before this development he had noticed
headaches and joint pains and a dry cough.
The most likely diagnosis is:
A. Attack of malaria as post operative complication
 Cellulitis of the left leg
 Congestive cardiac failure
 DVT
 Thrombophlebitis of the left leg

14. The treatment of choice is:


 A full course of Artesunate - Amodiaquine
 Crystalline Penicillin 2mega units 6 hourly I/M
 Review of patients cardiopulmonary function and digitalization
 Broad spectrum antibiotics and bed rest
 Anticoagulation with low molecular weight heparin – warfarin
compression stockings

15. Splenectomy is attended by:


A. Yellow marrow replacement in six months
B. Attacks of transient rheumatic pains

3
C. Early fall in haemoglobin level
D. Increased resistance to malaria
E. Some impaired immunity to bacterial infection

16. The most useful index (hall mark) of splenic overactivity is:
A. Plasma bilirubin level
B. Plasma haptoglobin level
C. Reticulocyte count
D. Red blood count
E. Leucocyte count

17. The normal half-life (in days) of red cells as measured by the radio-
active
chromium plating technique is:
 30
 60
 90
 120
 150

18. The indication(s) for staging laparotomy in patients with Hodgkin’s


disease is (are):
 Systemic symptoms in patient with disease on both sides of the body
 Hepatosplenomegaly
 Mixed cellular type of histology
 Abdominal mass
 Axillary lymphadenopathy

19. The indication(s) for staging laparotomy in patients with Hodgkin’s


disease is (are):
 Patiens with high cervical lesions
 Systemic symptoms in patients with bilateral neck disease
 Lymphocyte predominant lesions
 A radiologically clear mediastinum
 Axillary lymphadenopathy

20. Early changes in blood attendant on splenectomy are:


A. Leucocytosis, erythrocytosis and thrombocytosis
B. Lymphocytosis and erythrocytosis and thrombocytosis
C. Eosinophilia, plasmacytosis and thrombocytopenia
D. Neutrophilia, eosinophilia, erythrocytosis
E. Thrombocyhtosis, eosinophilia, erythropenia

21. Late changes in blood attendant on splenectomy are:


A. Leucocytosis, thrombocytosis erythropaenia
B. Moderate eosinophilia and plasmacytosis
C. Lymphocytosis, erythrocytosis and neutrophilia

4
D. Thrombocytopenia, erythropenia, neutrophilia
E. Neutropenia, thrombocytopaenia, erythropaenia

22. The following are considered absolute indications for splenectomy


Except:
 Rupture
 Torsion
 Splenic vein thrombosis
 Congenital hereditary spherocytosis
 Secondary thrombocytopenic purpura

23. Tropical Splenomegaly syndrome (Big Spleen Disease) is:


 An immunological disorder related in some way to schistosomiasis
 Caused by visceral leishmaniasis
 An abnormal immune response to the virus that causes infectious
mononucleosis
 An immunological disorder related to attacks of malaria
 One of the connective tissue lipoid storage disorders

24. The major criteria which establish the diagnosis of the tropical
splenomegaly syndrome include:
 Spleen that is well below the costal margin
 Spleen at least 2000G in weight
 Residence in a malaria endemic area
 Ig M level raised more than 25 times above normal
 Good response to anti malarials
 All the above
25. The minor criteria which establish the diagnosis of the tropical
spenomegaly syndrome include:
 Liver biopsy showing hepatic sinusoidal lymphocytosis
 Normal phyto haemoguttinin response
 Evidence of hypersplenism
 Lymphocyte proliferation
 Familial occurrence

26. The essential features of primary hypersplenism are:


A. There is significant splenomegaly
B. Anaemia is significant and persistent
C. Cytopenia is evident on examination of the blood film
D. There is compensatory hyper plastic bone marrow
E. No other actiological factor is discovered
F. Symptoms and signs are promptly improved by splenectomy

27. The treatment of choice in a patient proven to have primary


hypersplenism is eventually:
 A course of baceriocidal antibiotics

5
 Multiple cyclical chemotherapy
 Short courses of steroids
 Segmental splenectomy
 Splenectomy

28. In patients with secondary hypersplenism splenectomy alone is contra


indicated except:
 There is a level of anaemia controllable with haematinics
 The white cell count hovers around 4000/dl without infections
 The platelet count falls below 40,000/dl with recurrent haemorrhages
 There is recurrent sore throat but the WBC is persistently above
6000/dl
 The Hb level hovers around 12.G/dl but with persistent tiredness and
palpitations

29. A girl of 9 is seen in the Emergency Room with an attack of colicky


abdominal pain of 24h duration, followed by passage PR of mucus and
a few hours later mucus mixed with blood. On examination she was
not anaemic or dehydrated; petechiae were noticed in the oral mucosa.
The abdomen was soft non-tender; the spleen was one finger below
LCM but the liver and kidneys were not palpable, and there were no
other masses palpable. Rectally there was no tenderness. There was
evidence of blood and mucus on the finger. The entire episode settled
after 24h.
The most likely diagnosis is:
 Ileo colic intussuception
 Henoch-schonleen peerpura
 Familial adenomatous polyposis
 Ulcerative colitis
 Idiopathic thrombocytopenia

30. The investigation most likely to clinch the diagnosis is:


 Erect plain x-rays of the abdomen
 Colonoscopy
 Upper GI Endoscopy
 Full haematological profile of the child
 An estimation of the INR
31. The indicated management for this child is:
 Diagnostic laparoscopy and splenectomy
 Exploratory laparotomy at which splenectomy is performed
 Exploratory laparotomy to exclude Intussusception
 Transfusion of fresh blood
 Careful follow up of the patient with laboratory tests

32. A man of 40 has come to the surgical OPD with recurrent attacks of
abdominal discomfort, intermittent diarrhea with passage of mucus

6
and fresh blood over the past 4 years. On examination he was
apyrexial, not pale, but had signs of petichiae in the oral-mucosa and
bloctches on the skin. The abdomen was flat, non-tender, and there
were no masses. The liver, spleen and kidneys were not felt. P.R. –
the only finding was blood and mucus on the examining finger.

The most likely diagnosis is:


 Carcinoma of the rectum
 Carcinoma of the caecum
 Amoebic colitis with formation of a granuloma
 Ileo-caecal tuberculosis
 Idiopathic thrombocytopenia

33. The investigation most likely to clinch the diagnosis is:


 Colonoscopy
 Estimation of INR and prothrombintin
 Plain x-rays of the abdomen
 Repeated microscopic examination of rectal scrapings
 Full haematological profile of the patient

34. The treatment of choice is:


 Diagnostic laparoscopy
 Exploratory laparotomy at which splenectomy is done
 Initial exhibition of steroids failing which splenectomy is done
 Initial exhibition of vincristine, failing which splenectomy is done
 Transfusions of fresh blood as necessary to maintain physiological
levels of red cell and other formed elements

35. A Ghanaian woman of 35 who has suffered for many years with
seasonal (cold season) joint pains consults at the surgical out patients
complaining of epigastric pains radiating into the left hypochondrium,
fever and anorexia with vomiting of a week’s duration. On
examination she is icteric with mild anaemia, a pulse rate of 100/min
and BP 110/70; there is tenderness with guarding across the
eipgastrium, as well as a tender fixed mass in the left hypochondrium.
The Hb is 8.5G/dl, WBC 16,000/dl and a plain abdominal x-ray shows
a mass in the left hypochnodrium with a fluid level.

The most likely diagnosis is:


 Acute pancreatitis with a pseudocyst
 Spleen infarction
 Subphrenic abscess
 Splenic abscess
 Perigastric abscess complicating a gastric carcinoma

36. The treatment of choice is:

7
 Resuscitation and exhibition of broad spectrum bacteriocidal
antibiotics
 Percutaneous exploration of the mass under ultrasonic guidance
 Open drainage of the localising abscess
 Resuscitation splenectomy with drainage
 Expectant management with mentoring of the mass

37. Select the true statement:


 The portal vein carries 75% of total hepatic blood volume and supplies
50% of 02 needs
 The portal vein carries 65% of total hepatic blood volume and supplies
40% of 02 needs
 The portal vein carries 55% of total hepatic blood volume and supplies
30% of 02 needs
 The portal vein carries 45% of total hepatic blood volume and supplies
20% of 02 needs
 The portal vein carries 35% of total hepatic blood volume and supplies
50% of 02 needs

38. Select the true statement:


 The hepatic artery carries 25% of total hepatic blood volume and
supplies 50% of 02 needs
 The hepatic artery carries 20% of total hepatic blood volume and
supplies 40% of 02 needs
 The hepatic artery carries 15% of total hepatic blood volume and
supplies 35% of 02 needs
 The hepatic artery carries 15% of total hepatic blood volume and
supplies 30% of 02 needs
 The hepatic artery carries 10% of total hepatic blood volume and
supplies 25% of 02 needs

39. Determined by Percutaneous, Transplenic, Puncture, patients with


oesophageal varices are seldom found to have portal pressures below:
A. 25cm of Saline
B. 30cm of Saline
 35cm of Saline
 40cm of Saline
 45cm of Saline

40. The mechanism for the genesis of portal hypertension is most often:
 The result of increased portal inflow secondary to a hyperdynamic
systemic circulation
 The result of increased portal inflow secondary to splanchnic
hyperaemia
 Increased intra-hepatic post sinusoidal resistance within the portal
system

8
 Increased extra-hepatic pre-sinusoidal resistance within the portal
system
 Increased intra-hepatic pre-sinusoidal resistance within the portal
system

41. A West African youngster of 18 is seen with explosive recurrent


episodes
of haematemesis since childhood. As a rule these have settled briskly
spontaneously. On examination he is well nourished, pale, and
without lymphadnopathy; his abdomen is flat, non-tender, the liver is
not palpable and there are no stigmata of liver dysfunction. There is
moderate splenomegaly.
The most likely diagnosis is:
 Acute peptic ulcer
 NSAID abuse
 Oesophageal varices from portal cirrhosis
 Oesophago-gastric varices from extra-hepatic pre-sinusoidal portal
obstruction
 Reflux oesophogitis

42. An 18 year old West African presents in his first episode of


haematemesis, having vomited two cupfuls of blood followed two
hours later by melaena. The year before he had had laparotomy for
an acute abdomen when a gangrenous appendix had been removed.
On examination he is pale, an icteric and the only findings in his
abdomen are a broad infra umbilical midline scar and a tipped spleen

The most likely diagnosis is:


 Gastritis from NSAID abuse
 Acute DU complicating the laparotomy
 Port hypertension from hepatic cirrhosis
 Oesophageal varices from thrombophlebitis of the radicals of the portal
vein
 Post bulbar ulcer from the Zollinger-Ellison syndrome

43 The diagnostic investigation of choice is:


 Abdominal ultrasound employing modern Doppler techniques
 Barium swallow
 Upper GI Endoscopy + injection sclerotherapy where indicated
 Splenic venography
 Barium meal and follow through

44. The treatment of choice for this patient (Question 42) is:
 Endoscopic ligation/eradication of the varices by sclerotherapy

9
 Bed rest and rest to the GI Tract
 Protracted exhibition of propranolol
 Tripple therapy
 Conservative treatment using glypressin

45. The MAIN surgical challenge in portal hypertension is:


A. Control of haemorrhage from oesophageal varices
B. Control of the problems of hypersplenism
C. Control of gross ascites
D. Control of liver failure
E. Control of porto-systemic encephalopathy

46. The varices that bleed from the oesophagus in portal hypertension are:
 Mucosal
 Submucosal
 Intermuscular
 Intramucosal
 Adventitial

47. Portal hypertension is most frequently due to:


A. Increased flow in the portal venous system
B. Augmented flow from the hepatic artery through presinusoidal arterio-
portal shunts
 Mechanical blockage of flow in the pre-and post-sinusoidal circulation
 Increased resistance to flow in the hepatic veins
 Intrinsic narrowing of the portal veins

48. In the experience of most tropical surgeons the mechanical obstruction


to portal flow in portal hypertension most frequently arises from:
 Pre-hepatic or infrahepatic sector
 Intrahepatic sector
 Intrahepatic, post-sinusoidal sector
 A combination of A, B and C
 Post-hepatic sector

49. The most important aetiological factor in portal hypertension in most


tropical countries is:
 Thrombophlebitis of the umbilical vein
 Cryptogenic hepatic cirrhosis
 Hepatic fibrosis from schistosomiasis
 Hepatic vein thrombosis from venoocclusive disease
 Primary liver cell cancer

50. The most useful investigative procedure for the diagnosis of bleeding
oesophageal varices is:
 Barium swallow and meal

10
 Splenic venography
 Selective coelliac angiography
 Oesophagogastroscopy
 Radio-active scintigraphy

51. Needle liver biopsy may prove of value in the assessment of the patient
with portal hypertension to:
 Exclude cirrhosis in alcoholics with apparent hepatomegaly
 Distinguish intra-from extra-hepatic block
 Identify hepatomegaly ostensibly produced by disease unlikely to cause
portal hypertension
 Determine the state of liver function

52. Operative risk in patients with portal hypertension is dependent on


serum levels of:
 Bilirubin, albumin, prothrombin and attendant ascites
 Electrolytes, albumin and attendant ascites
 Electrolytes, urea, bilirubin and attendant encephalopathy
 Blood ammonia, bilirubin and attendant encephalopathy
 Albumin, ammonia, urea and attendant ascites

53. In patients facing surgical intervention for liver disease the most
dependable measure of estimating the functional reserve of the liver is:
 The Gardner Child scoring system
 The Pugh-Child scoring system
 Serum albumin level
 Serum bilirubin level
 Prothrombin level (seconds prolonged)
 Presence of Ascites
 Signs of Porto-systemic encephalopathy
 Galactose Elimination capacity
 Aminopyrine breath test
 Hepatic Clearance of Aminoacids

54. In patients with oesophageal varices the predictability of variceal


haemorrhage depends on:
 The level of portal pressure only - > 25cm saline
 The variceal size and portal pressure
 The portal pressure, variceal size, and cherry-red spots and wall
markings
 Variceal size, cherry-red spots and wall markings
 None of the above

55. Recent randomized controlled trials have shown that the most effective
agent for control of variceal bleed is:
 I/V Infusin of Octreotide - 25-50µgm per hour for 48hr

11
 I/V Terlipressin 2mg 6 hourly
 Nadolol 60mg bd
 I/V Glypressin + glyceryl trinitrite
 Esomeperazole 40mg bd for 2/52

56. Recent randomized controlled clinical trials in endoscopic


management of bleeding oesophageal varices have shown that (one
correct answer):
 Band ligation + Octreotide yield the best results - > 90% success
 Sclerotherapy + Octreotide yield the best results
 Band ligation + Terlipressin yield the best results
 Sclerotherapy + Terlipressin yield the best results
 Band ligation alone yields the best results
 Sclerotherapy alone yields the best results

57. The indications for Transjugular Intra hepatic porto-systemic stent


shunt
TIPSS include (one answer):
 Prophylaxis against bleeding in newly diagnosed portal hypertension
 Patient with portal hypertension who do not respond to use of
Octreotide
 Patients with portal hypertension with uncontrolled variceal bleeding
refractory to band ligation
 Patients with portal hypertension with uncontrolled variceal bleeding
refractory to sclerotherapy
 Patient s with portal hypetension refractory to use of balloon
tamponade

58. Complications of the Transjugular intra hepatic porto-systemic stent


shunt (TIPSS) procedure include:
 Variceal bleeding in 10-20% of cases
 Mild controllable encephalopathy – 30%
 Transient worsening of liver function – 30%
 Stent insufficiency – stent stenosis, thrombosis
 Haemolysis in 10% of patients before endothelization
 Sepsis is rare

59. Porto-systemic encephalopathy may be precipitated by:


A. Liver abscess
B. Porto-systemic shunts
C. Recent gastro-intestinal bleed
D. Uncontrolled protein intake in patients with liver decompensation
E. Hepato-splenomegaly

12
60. By current assessment, absolute contra-indications to shunt surgery in
portal hypertension include:
 Established jaundice
 Persistent ascites
 Hypo-albuminaemia
 Uncontrollable alcoholism
 Hypo-gammaglobinaomia

61. The first line measures in the arrest of bleeding from oesophaeal
varices include:
 Blood replacement, sedation with barbiturate, oral neomycin and
saline purge
 Oral neomycin, volume replacement, and immediate endoscopy
 Volume replacement, saline purge and use of the
SengstakenBlakemore tube
 Sedation with morphine, oral neomycin and immediate endoscopy
 Arrest of bleeding with the Sengstaken Blakemore tube and
barbiturate sedation

62. The therapeutic dose of vasopression used in patients with continued


bleeding in oesophageal varices is:
 10 units in 200 ml of saline in 20 minutes IV
 20 units in 200 ml of saline in 20 minutes IV
 30 units in 200 ml of saline in 20 minutes IV
 40 units in 200 ml of saline in 20 minutes IV
 50 units in 200 ml of saline in 20 minutes IV

63. The therapeutic effect of vasopression is mainly through:


A. Constriction of the oesophageal mucosal vessels
B. Constriction of the radicles of the portal circulation
C. Constriction of the splanchnic arterioles
D. Selective constriction of the portosystemic collaterals
E. Constriction of the hepatic artery and portal veins

64. The indications for the deployment of the Sengstaken Blakemore tube
include:
 Routine use in patients with proven bleeding varices
B. Patients with gastric varices
C. As a temporary holding measure in patients with uncontrollable variceal
haemorrhage
D. Routinely as a means of confirming diagnosis of haemorrhagic varices
E. In all patients who undergo injection sclerotherapy
65. The complications associated with use of the Sengstaken Blakemore
tube include:
A. Aspiration pneumonia
 Pressure necrosis of the lower end of the oesophagus

13
C. Suffocation from displacement of the tube
D. Re-bleeding after deflation of the oesophageal balloon
E. Reflux oesohagitis

66. Direct Tamponade using the Sengstaken-Blake more tube is seldom


employed currently because of effectiveness of endoscopic treatment
and pharmacotherapy. Its real value now is:
 As first line measure for torrential variceal haemorrhage
 In management of patients with Pugh-Child C grade liver function
 In patient with portal hypertension from primary liver cell carcinoma
 To tide over a patient in preparation for definitive surgery
 To stabilize a patient for transfer to a more suitable facility for
management
 To distinguish between patients with gastric or oesophageal varices

67. The complications of Balloon Tamponade in the management of


bleeding oesophageal varices include:
 Oesophageal mucosal ulceration/necrosis
 Airway obstruction from the dislodged gastric balloon
 Oesophageal ruptures from inadvertent filling of gastric balloon within
the oesophagus
 Recurrent bleeding on deflation of the balloons
 Aspiration pneumonia

68. The simplest surgical intervention for patients with persistent bleeding
from oesophageal varices is:
 Transgastric variceal ligation
 Emergency transsection of the oesophagus
 Gastric trans-section with para-oesophageal gastric devascularisation
 Distal spleno renal shunt
 Splenectomy
 Splenectomy and spleno renal shunt

69. Which of these features (is)are true of the distal spleno-renal shunt
(Warren) used in the management of bleeding oesophageal varices:
 Maintains a near normal portal blood flow
 Does not decompress the viscera of the left upper abdomen
 Provides complete freedom from Porto systemic encephalopathy
 Has a lower rebleeding rate than porto-caval shunt
 Maintains a lower portal pressure than the porto-caval shunt

14
 Is the shunt of first choice for elective portal decompression
 As the right upper abdomen is untouched leaves the option open for
later liver transplantation

70. Concerning the prophylactic treatment of patients with portal


hypertension who have never bled:
 Randomised controlled trials have demonstrated survival advantage of
shunting procedures
 Randomised controlled trials have demonstrated survival advantage of
endoscopic treatment of varices
 Randomised controlled trials have demonstrated the usefulness of
prophylactic use of glypressin
 Randomised controlled trials of beta blockade have demonstrated
reduced incidence of initial variceal bleeding
 Radomised controlled trials of beta blockade have consistently
demonstrated advantage in prolongation of survival in the treated
group

71. In the prevention of recurrent oesophaeal variceal bleeds:


A. Shunt operations effectively control further bleeds
B. Shunt operations do not prolong the life of the patient
C. Deterioration of liver function is the principal cause of death in shunted
patients
D. Prophylactic shunts (no previous bleeds) are more effective than
therapeutic shunts (at least one previous bleed)
E. Injection sclerotherapy is a useful short term measure

72. Portosystemic encephalopathy:


A. Is precipitated by reduction in arterial supply to the liver
B. Is unrelated to the portal flow into the liver
C. Can be expected in about 30% of shunted individuals
D. Is not affected in its frequency by the type of shunt established
E. Is effectively controlled by the combination of laevulose and neomycin

73. The most urgent therapeutic needs of the patient actively bleeding
from oesophageal varices include:
 Immediate oesophagogastroscopy
 Transfusion of fresh blood, barbiturate sedation
 Oral neomycin and saline purge
 Early deployment of the Sengstaken Blakemore tube
 Emergency portocaval shunt within 8 h of presentation

74. In patients with continued bleeding from oesophageal varices

15
vasopressin:
 Produces constriction of the splanchnic arterioles
 Contracts the oesophageal musculature and this aids control
 May be given in doses of 20 units 200 ml of saline over 20 minutes
 In the effective dose may cause bowel evacuation and this may be
beneficial
 May safely be exhibited in all individuals so afflicted

75. The Sengstaken Blakemore tube:


A. Achieves direct tamponade of bleeding varices
B. Is usually deployed by passing it through the mouth
C. In its modern form has four lumens
D. On the average achieves arrest of haemorrhage in 30 per cent of cases
it is used on
E. Requires careful monitoring of its various channels once applied

76. The Sengstaken Blakemore tube:


A. Has no diagnostic functions in patients with haematemesis
B. On the average achieves arrest of haemorrhage in 60 per cent of cases
C. Passes readily down the nose and is well tolerated by most patients
D. Serves only as a temporary control measure in the work up of the
patient with bleeding varices
E. May occasionally produce jejunal rupture

77. The peculiarities of portal hypertension in most tropical countries


include:
 A relatively higher incidence of cases due to extrahepatic blockage
of the portal vein
 A relatively higher incidence of cases due to intrahepatic
presinusoidal blockage
 Intrahepatic post-sinusoidal and post-hepatic obstruction may be a
feature
 Infrequent porto-systemic encephalopathy
 Shows a tendency to occur at a younger age group

78. In the diagnosis of oesophageal varices:


A. Barium swallow is superior to oesophagoscopy
B. Evidence of liver failure is an essential pre-requisite
C. Percutaneous transplenic venography is mandatory
D. Selective celiac angiography may provide useful information
E. Scintigraphic determination of the portal circulation time is the most
accurate procedure

16
79. The prognosis in portal hypertension is dependent on:
A. Serum bilirubin levels above 2 mg/100ml
B. Serum albumin levels of less than 3g/100 ml
C. Prolonged prothrombin time over 2 seconds above normal
D. Presence of ascites
E. Elevated level of alkaline phosphatase

80. The prognosis in portal hypertension is dependent on:


A. Presence of ascites
B. Presence of encephalopathy
C. Presence of jaundice
D. Presence of hepatosplenomegaly
E. The level of portal pressure

81. The portal vein:


A. Has an oxygen saturation of 85 per cent
B. Provides 50 per cent of the liver’s oxygen needs
C. Carries 75 per cent of the total hepatic blood volume
D. Is notable in being completely devoid of valves
E. Has profuse arterio venous connections

82. The portal vein:


A. Has an average hydrostatic pressure of 15 cm of saline as
determined by cannulation at laparotomy
B. Has no presinusoidal communication with the hepatic artery
C. At the foramen of Winslow lies medial to the common bile duct and
the hepatic artery
D. Receives venous blood from entire sub-diaphragmatic gut
except the lower part of the anal canal
E. Has several retroperitoneal connections with the systemic
circulation

83. In portal hypertension:


A. The transplenic pressure is usually above 25 cm of saline
B. The hepatic arterial circulation is apparently unchanged
C. The Wedge hepatic pressure may, in some cases, be normal
D. The Vasalva manoeuvre produces no change in the level of pressure
E. The percutaneous transhepatic pressure is always higher than the
transplenic pressure

84. In portal hypertension:


A. There is no relation between level of pressure and probability of

17
bleeding
B. The tension in the portal venous system is sustained
C. The varices are submucosal
D. Melaena is a more frequent presenting symptom than haematemesis
E. The bleeding is more frequently occult than obvious

85. In the management of hypersplenism, splenectomy should be


considered if:
 Attendant anaemia is causing symptoms
 Leucopenia is below 1 x 109/dl
 There is thrombocytopenia below 40 x 109/dl
 There is a reticulocyte count of 1.5%
 Response to conservative treatment is slow though progressive

86. In sickle cell disease splenectomy:


A. May occasionally relieve symptoms of acute crisis
B. Is recommended in most patients
C. May usefully lengthen red cell survival
D. Often produces portal hypertension
E. Significantly reduces the frequency of crisis

87. The post-operative complications particularly associated with


splenectomy include:
 Malarial infection in children in the tropics
 Typhoid perforations
 Pneumococcal infections in adults
 Wound infections
 Disseminated intravascular clotting syndrome

88. The portal vein:


A. Receives blood from the entire subdiaphragmatic gastro-intestinal
tract
B. Has the superior and inferior mesenteric, splenic and left gastric
veins as its main tributaries
C. Is the only portal system in the body
D. Carries 75 per cent of the total hepatic blood volume
E. Normally enters the liver through three branches corresponding
to the hepatic vein tributaries

89. The spleen:


A. Normally weighs 150 g in the adult
B. Decreases in weight with age
C. Lies high in the left upper quadrant subjacent to the diaphragm
D. Is the most vascularized organ in the body
E. Develops in the dorsal mesogastrium

90. The splenic artery:

18
A. Carries about 20% of the cardiac output
B. Is a branch of the celiac axis
C. Has a wary course following the upper border of the pancreas
D. Only occasionally gives rise to the short gastric arteries
E. May occasionally be aneurismal in female patients

91. The spleen:


A. On the average filters 350 litres of blood per day
B. Stores in its meshes significant reserves of blood
C. Reflexly releases stored blood from stimulus of hypoxia
D. Has no haemopoietic function in life
E. Is capable of extracting nuclear remnants from circulating
erythrocytes

92. The spleen:


A. Probably elaborates a hormone that inhibits haemopoiesis
B. Is an important centre for antibody formation against pyogenic
organisms
C. Is directly involved in the absorption of iron from the
gastrointestinal tract
D. Is important in the pathogenesis of auto-immune disease
E. Should be removed in patients with Hodgkin’s disease as a
therapeutic procedure

93. In the patient with actively bleeding oesophageal varices the most
suitable first line
of arresting the haemorrhage after resuscitation is:
 I/V Octreatide-therapy
 Use of direct tamponade-Sengstake-Blakemore tube
 Endoscopic banding
 Endoscopic sclerotherapy
 Trans-jugular, intra hepatic portosystemic stent shunt (TIPSS)
 Transgastric ligation of the varices + splenectomy
 Distal spleno-renal (Warren) shunt

19
CHAPTER 41

SPLEEN AND PORTAL HYPERTENSION


ANSWERS

1. C 38. A 75. A, C, E
2. A, B, C 39. A 76. B, D, E

3. A, B, C, D 40. C 77. A, B, E

4. A, C, E, G, H 41. D 78. D
5. A, D, F 42. D 79. A, B, C, D,

6. A, E 43. C 80. A, B, C

7. A, B, C, D, E 44. A 81. A, B, C, D
8. B, C 45. A 82. D, E

9. C 46. B 83. A, C

10. C 47. C 84. A, C

11. A, B, C, D, E, F 48. C 85. A, B, C

12. D 49. B 86. A, C

13. D 50. D 87. A, C

14. E 51. A, B, C, D 88. B, D

15. A, B, E 52. A 89. A, B, C, D, E

16. C 53. B 90. A, B, C, E

17. A 54. C 91. A, B, C, E

18. A, B, C, D 55. A 92. A, B, D, E

19. B 56. A 93. C

20. A 57. C
21. B 58. A, B, C, D, E, F

20
22. E 59. B, C, D
23. D 60. A, B, C, D
24. A, B, C, D. E. F 61. A
25. A, B, C, D, E 62. B
26. A, B, C, D, E, F 63. C
27. E 64. C
28. C 65. A. B, C, E
29. E 66. D, E
30. D 67. A, B, C, D, E
31. E 68. A
32. E 69. A, F, G
33. E 70. D
34. C 71. A, B, C, E
35. D 72. A, C
36. D 73. B, C
37. A 74. A, B, C, D

21
CHAPTER 42
THE ADRENAL GLANDS

 Concerning the anatomy of the adrenal glands:

 Each gland weights 3-4 g

 They are both triangular in shape

 They cap the kidney at the level of the eleventh dorsal vertebra

 They are encapsulated, invested in renal fascia and embedded in

peri-renal fat

 They consist of an outer smaller cortex and an inner bigger


medulla

 In the anatomy of the adrenal gland:

 The medulla is derived from mesoderm and the cortex from the
neural crest

 Histologically the cortex is sub-divided into an outer zona


fasciculate, a middle zona glomerulosa and an inner zona
reticularis

 The medulla is composed of groups of chromaffin cells

 The same arteries supply the cortex and medulla

 The venous drainage leaves through the periphery

 Hormones produced by the adrenal cortex in the female include:

 Cortisol

 17-ketosteroids

1
 Oestrogens and progesterone

 17-hydroxycorticosteroid

 Testosterone

 Hormones produced by the adrenal medulla include:

 Nor-adrenaline

 Angiotensinogen

 Histamine

 Aldosterone

 Dopamine

 ACTH controls the adrenal SECRETION of

A. Aldosterone
B. Cortisol
C. Adrenaline
D. Oestrogens
E. Androgens

6. Cortisol:
A. Increases glycogen deposition in the liver
B. Increases re-absorption of sodium and excretion of potassium by
cells
across cell membranes
C. Promotes gluconeogenesis
D. Inhibits peripheral utilization of glucose
E. Potentiates the action of insulin
7. Quantitatively, the most important steroid secreted by the
adrenal cortex is:

 Testosterone
 Cortisone
 Aldosterone
 Deoxycorticosterone
 Cortisol
8. If in the ACTH Stimulation Test there is no rise in the urinary 17-

2
hydro-
xycorticosteroids and 17-oxosteroid from their initial value, the
most likely diagnosis is:

 Hypopituitarism
 Addison’s disease (primary adrenal insufficiency)
 Cushing’s syndrome
 Hyperplasia of the adrenal cortex
 Carcinoma of the adrenals
9. If a patient is given 2 mg of dexamethazone in divided doses
daily and
the urinary 17-hydroxy-corticosteroid does not fall to less than 3
mg per
day by the second day, you will suspect:

A. Addison disease
B. Hypopituitarism
C. Cushing’s syndrome
D. Adreno-genital syndrome
E. Pituitary chromophobe adenoma

10. The commonest cause of Addisonian crisis (acute adrenal


insufficiency) is:
 Tuberculosis of the adrenals
 Idiopathic atrophy of the adrenals
 Sheehan’s syndrome
 Sudden steroid withdrawal
 Adrenal metastases
11. Clinical features of hypo-adrenocortism include:
A. Hyperpigmentation of the palms and soles of the feet
B. Increase in weight
C. Normal blood pressure or hypertension
D. Muscular weakness
E. Drowsiness
12. In hypo-adrenocortism:

A. The serum potassium and sodium are low


B. The fasting blood sugar is low
C. The eosinophil count is depressed following ACTH stimulation
D. The glucose tolerance test is usually flat
E. The plasma cortisol level is low but generally rises considerably in
response to ACTH stimulation

3
13. Clinical features of Addisonian crisis (acute adrenal insufficiency)
include:

A. Severe abdominal pain with rigidity


B. Vomiting
C. Hypotension
D. Prostration
E. Tachypnoea
14. Drugs used in the treatment of hypo-adrenocortism include:

A. Nor-adrenaline
B. Hydrocortisone
C. Fludrohydrocorisone
D. Aminoglutethimide
E. Metyrapone
15. Cushing’s Disease is caused by:

A. Adrenal adenoma
B. Adrenal carcinoma
C. Adrenal hyperplasia
D. Pituitary basophilic adenoma
E. Pituitary chromophobe adenoma
16. Cushing’s syndrome may be caused by:

 Bronchogenic carcinoma
 Treatment with exogenous cortisol
 Carcinoma of the thymus
 Hyperplasia of the adrenal cortices
 Adrenal carcinoma
17. Cushing’s syndrome:

A. Is fairly common
B. Does not occur in infants or the elderly
C. Is 3-4 times commoner in women
D. May be associated with adrenogenital syndrome
E. Is most commonly caused by idiopathic hyperplasia of the
adrenal cortices

18. The first and most characteristic feature of Cushing’s


syndrome is:

A. Increase in weight
B. Hypertension

4
C. Rounding of the face (“moon” face)
D. Easy bruising of the skin and multiple ecchymoses
E. Insulin-resistant diabetes
19. Causes of hypertension in Cushing’s syndrome include

 Renal ischaemia
 Premature arteriosclerosis
 Excessive production of angiotensin
 Retention of sodium
 Retention of water
20. Causes of increase in weight in Cushing’s syndrome include:

A. Increased deposition of fat


B. Hypertrophy of muscles
C. Retention of sodium
D. Excessive deposition of glycogen
E. Retention of water
21. In Cushing’s syndrome, there:

A. Is obesity of the limbs


B. Is obesity of the trunk
C. Are fat pads in the cervical region and supraclavicular areas
D. Is accumulation of fat and fluid in the cheeks
E. Is thinning of the peri-orbital areas
22. In Cushing’s syndrome, there may be:

A. Impotence and softening of the testes


B. Amenorrhoea or menorrhagia
C. Increased libido in the female
D. Scanty growth of hair on the upper lip and chin in the female
E. Hypertrophy of the breasts in the female
23. In Cushing’s syndrome, there may be:

A. Osteosclerosis of the vertebrae


B. Pathological fractures of the ribs
C. Renal calculi
D. Nephrocalcinosis
E. Vertebral compression

24. In Cushing’s syndrome the:

A. Blood cortisol level is characteristically high


B. Blood cortisol is higher at 8a.m. than at 12 mid night

5
C. Urine 17-hydroxycorticosteroid is markedly elevated
D. The urine free cortisol may be low
E. There is no response to cortisol suppression with small doses
of dexamethasone
A patient with a clinical diagnosis of Cushing’s syndrome has a
high
plasma cortisol level. On ACTH stimulation, there is no rise in the
plasma cortisol level:

25. The diagnosis may be:


A. Pituitary basophilic adenoma
B. Adrenal adenoma
C. Adrenal carcinoma
D. Pituitary chromophobe adenoma
E. Adrenal hyperplasia
26. Investigations that will help determine the site of origin of the
disease include:
 MRI
 Skeletal X-ray
 Ultrasonography of the abdomen
 CT Scan
 Skull X-ray
27. The Methods of treating Cushing’s disease include:

A. Pituitary irradiation
B. Subtotal adrenalectomy
C. Bilateral adrenalectomy
D. Selective pituitary resection
E. External adrenal irradiation
28. Pre-operative measures in Cushing’s syndrome include:

A. Weight reduction by restriction of food intake


B. Metyrapone administration for some weeks
C. Guanethidine administration
D. Digitalis
E. Dexamethasone
29. Post-operative complications after bilateral adrenalectomy
include:

 Acute adrenal insufficiency


 Acute hypercortism
 Infections especially of the wound
 Wound dehiscence

6
 Increased pigmentation

30. In the prognosis of Cushing’s syndrome:

 Complete cure is certain in adrenal adenoma


 Complete cure is certain in about 75% of patients after bilateral total
adrenalectomy for pituitary-dependant cases
 Nelson’s syndrome carries a grave outlook
 Replacement therapy with cortisol is required for life in all patient
 Patients with overt pituitary tumours do better than those with
undetected tumours
31. Syndromes of excessive production of androgens may be caused
by:

 Bilateral adrenal hyperplasia


 Adrenal carcinoma
 Arrhennoblastoma
 Stein-Leventhal syndrome (large polycystic ovaries)
 Racial factors
32. The most likely cause of adrenogenital syndrome in childhood is:

A. Bilateral adrenal hyperplasia


B. Enzymatic defect in the production of cortisol
C. Adrenal adenoma
D. Arrhenoblastoma
E. Adrenal carcinoma
33. Clinical features of the adrenogenital syndrome in premenopausal
females include:

 Premature baldness
 Polymenorrhoea
 Diminution in breast size
 Atrophy of the clitoris and vagina
 Deepening of the voice
34. Clinical features of the adrenogenital syndrome in girls at puberty
include:

 Primary amenorrhoea
 Enlargement of the clitoris
 Acne
 Atrophy of the muscles
 Enlarged larynx

7
35. Clinical features of feminism include:

A. Increased libido
B. Rapid growth in prepubertal boys
C. Normal penis
D. Premature epiphysial maturation in prepubertal boys
E. Palpable adrenal tumour in about half of the patients

36. The most characteristic laboratory finding in adrenogenital


syndrome is elevated:

 Urinary pregnanetriol
 Plasma cortisol
 Plasma testosterone
 Urinary ketosteroids
 Urinary cortisol
37. Congenital adrenal hyperplasia:

 Is a familial disorder of recessive inheritance


 Is a rare cause of virilism or feminism in children
 Is caused by enzymatic defect in the production of cortisol
 Causes elevated urinary pregnanetriol
 Is treated with oestrogens and hydrocortisone
38. In hyperaldosteronism (Conn’s syndrome):

A. Adrenal carcinoma is the commonest cause


B. The causative tumour is bilateral in about 10% of patients
C. The majority of tumours are on the right side
D. Most tumours are large
E. Atrophy of the contralateral adrenal is unusual
39. The clinical features of hyperaldosteronism include:

A. Nocturia
B. Headaches
C. Polydipsia
D. Anxiety and tremors
E. Tetany

40. In hyperaldosteronism:

8
A. The plasma rennin activity is high
B. The plasma aldosterone is elevated but the urinary
aldosterone is reduced
C. The urine has a high osmolality
D. The serum sodium is high
E. The Na/K ratio of the saliva is reduced
41. In primary hyperaldosteronism:

A. Hypertension is due to sodium retention and potassium loss


B. Weakness and flaccid paralysis are due to hypokalaemia
C. Polyuria is due to chronic pyelonephritis
D. Tetany is due to hypocalcaemia
E. Fludrocortisone is given to control the disease before operation

42. Phaeochromocytoma:

A. Is a fairly common cause of hypertension


B. Becomes pale when the cut surface is exposed to air
C. Is malignant in about 10% of patients
D. Is bilateral in about 30% of patients
E. May be associated with medullary carcinoma of the thyroid
43. Symptoms of phaeochromocytoma:

 Excessive sweating
 Paraesthesiae
 Tremors and anxiety
 Chest or abdominal pains
 Weight loss and increased appetite
44. In phaeochromocytoma:

 Hypertension is paroxysmal and not persistent


 There may be transient or persistent glycosuria
 Symptoms are often precipitated by emotion
 Females are more commonly affected
 A family history may be obtained in a few
45. Phaeochromocytoma may be found in the:

A. Bladder
B. Posterior mediastinum
C. Areas around the abdominal aorta
D. Spleen
E. In the liver

9
46. The most reliable diagnostic test in phaeochromocytoma is:

A. Phentolamine (Rogitine) test


B. Glucagon test
C. Estimation of urinary vanyl mandelic acid
D. Estimation of catecholamines in the blood and urine
E. Estimations of 24h excretion of ketosteroids
47. The safest and most reliable localizing test in phaeochromocytoma
is:

A. I.V.P
B. Blood samples taken at intervals along the inferior vena cava and
analysed for their CA content
C. CAT Scanning
D. Ultrasonography
E. Adrenal scintigraphy

48. Hypertensive crisis during removal of phaeochromocytoma is


controlled
with intravenous:

 Phentolamine
 Propranolol
 Proctolol
 Phenoxybenzamine
 Methyldopa
49. Dangerous hypotension following removal of phaeochromocytoma
is controlled with:

 Nor-adrenaline
 Blood
 Large volumes of electrolyte solution
 Methedrine
 Propranolol
50. Possible causes of hypertension and hypokalaemia include:

A. Chronic pyelonephritis
B. Carbenoxolone
C. Liquorice
D. Oral contraceptives

10
E. Renal artery stenosis
A 40-year old man with headache, polydipsia and recurrent
attacks of weakness and paralysis of the limbs is found to have
a blood pressure of 180/110 mmHg:

51. The most likely clinical diagnosis is:

A. Idiopathic hypertension with cerebro-vascular spasm


B. Phaeochromocytoma
C. Chronic nephritis
D. Cushing’s syndrome
E. Primary hyperaldosteronism
52. Useful SCREENING tests in a small hospital will be:

 Serum electrolytes and urea


 Urinalysis and urinary electrolytes
 Straight X-ray of the abdomen
 Glucose tolerance test
 Full blood count

53. Neuroblastoma:
 Is a highly malignant tumour of the adrenal cortex and medulla
 50% of cases occur before 2 years and 90% below 10 years
 Spreads by the blood stream but not by the lymphatics
 Has a knobbly contour
 May secrete dopamine
54. Neuroblastoma:

 Is usually painless
 Often causes haematuria
 May on straight X-ray of the abdomen show calcification
 Is much more common in females
 Is best treated by removal and irradiation of the area
55. In treating Addisonian crisis, one needs:

11
 I.V. Normal saline
 Potassium chloride
 I.V. glucose
 ACTH
 Antibiotics
56. Adrenal incidentaloma:

 Is a tumour arising from the cells of the neural crest in the


adrenal gland
 Is in most cases silent, symptomless and does not affect the life
of the patient
 Does not require adrenal hormonal screening
 Does not require adrenal removal
 Shows evidence of extensive calcification

CHAPTER 42
THE ADRENAL GLANDS
ANSWERS

 A, C, D 32. E

 C 33. A, C, E

12
 A, C, D, E 34. A, B, C

 A 35. B, D, E

 B, D, E 36. D

 A, C, D 37. A, C, D

 E 38. B, E

 B 39. A, B, C, E

 C 40. D, E

 D 41. B

 A, D, E 42. C, E

 B, C, D 43. A, B, C, D, E

 A, B, C, D 44. B, C, E

 B, C 45. A, B, C, D

 D 46. D

 A, B, C, D, E 47. C

 C, D 48. A

 A 49. B, C

 D, E 50. B, C, D

 A, C, E 51. E

 B, C, D 52. A, B, C

 A, D 53. B, D

 B, C, D, E 54. A, C, E

 A, C, E 55. A, C, E

 B, C 56. B

 A, C, D

13
 A, C

 B, E

 A, C, D, E

 A, B

 A, B, C, D

14
CHAPTER 50
VASCULAR AND LYMPHATIC SYSTEMS

1. The intima of an artery is:

        A. Thick
B. Relatively acellular
C. Non-wettable
D. Nourished via the vasa vasorum
E. Supplied by nerves
2. The media of an artery is:
 Composed of fibro-elastic tissue and smooth muscle
 Relatively thin but responsible for the strength of the arterial wall
 In big arteries composed predominantly of muscle fibres
 Nourished by the vasa vasorum
 Separated from the adventitia by the internal elastic lamina
3. The veins:
 Have the same capacity as the arteries
 Have a wall consisting of muscle and fibro-elastic tissue
 With the exception of the abdominal ones have valves which allow the
blood to flow only towards the heart
 In the body cavities allow blood to flow as a result of the negative
pressure in the chest during respiration
 Have sympathetic nerve supply
4. Where facilities exist, the investigations that MUST be done in every
patient
with peripheral arterial disease include:

 X-ray of chest and limb


 Arteriography
 E.C.G
 Ultrasonography
 Ankle systolic pressure and ankle brachial pressure index
5. The commonest cause of peripheral arterial disease is:
 Thrombo-angiitis obliterans
 Raynaud’s disease
 Atherosclerosis
 Aneurysm

1
 Embolism

6. In the ankle systolic pressure and ankle brachial pressure index:


 The systolic pressure of the posterior tibial artery and brachial artery
are measured
 A special electronic sphygmomanometer and the pencil probe
Doppler
velocimeter are used
 The ankle systolic pressure is normally 20-30 mmHg higher than the
brachial
systolic pressure
 The ankle BP/arm BP is greater than one
 The result is reproducible
7. In the ankle systolic pressure and ankle brachial pressure index
test:
 The ankle BP is elevated in peripheral arterial disease
 The ankle BP/arm BP is elevated in peripheral arterial disease
 Peripheral arterial disease is indicated if the ankle BP fails to rise after
exercise for one minute on a bicycle ergometer at 500 Kpm/min
 The tests are not useful in assessing progress to treatment
8. Complications of translumbar aortic arteriography include:
 Allergy
 Injury to the spinal cord
 Cerebrovascular accident
 Damage to the kidney
 Spasm of the superior mesenteric artery
9. Atherosclerosis:
 Affects the media primarily and the intima also
 Is equally common in the sexes
 In the lower limb occurs mainly from the infra-renal aorta to the
division
of the popliteal
 In the upper limb affects the subclavian, axillary, brachial and radial
 Is commoner in medium-sized arteries than ones at sites of origin of
branches

2
10. In atherosclerosis:
 Lipids are deposited in the deeper layers of the intima
to form plaques
 Plaques may become calcified or fibrosed
 There is ulceration of the endothelium
 The internal elastic lamina is thickened
 The muscle of the media is replaced by fibrous tissue

11. Atherosclerosis of a part of an artery may result in:


 Distension
 Aneurysm
 Thrombosis
 Narrowing of collaterals and branches above and below the
affected part
 Tortuosity
12. Atherosclerosis is related aetiologically to:
 High intake of unsaturated fat
 Cigarette-smoking
 Hypercholesterolaemia
 Alcoholism
 Hypertension
13. In diabetics, atherosclerosis:
 Develops at an earlier age
 Is commoner in men
 Affects the popliteal, tibial and associated small arteries commonly
 Has the same incidence of gangrene as is non-diabetics
 Leads to ulceration more commonly than in non-diabetics
14. Thrombo-angiitis obliterans:
 Is common
 Affects the large and medium arteries of the limbs
 Is often associated with migratory phlebitis of the superficial or deep
veins
 Occurs almost exclusively in middle-aged cigarette-smoking males
between
40-and 55

3
 Is probably an allergic reaction to nico-tine and regresses if smoking
is stopped
15. In thrombo-angiitis obliterans:
A. There is initial dense round cell infiltration of the adventitia
and media
B. There is endothelial proliferation of the intima
C. The lumen is occluded by a thrombhus which is later replaced by
firm granulation tissue
D. The lumen is never canalized
E. The artery or vein becomes enclosed in fibrous tissue

16. Arterial occlusion in the lower limb:


 May be multiple and is bilateral in about 70% of patients
 If in the popliteal causes a more severe ischaemia than in the
superficial
femoral
 If in the aorta or iliac causes more reduction in blood flow than in the
femoral
 Is more often below the level of the knee joint
 Is seen most often in patients between 50 and 65
17. Intermittent claudication is:
 Deep-seated cramplike pain of the calf muscles brought on by
walking
and relieved by rest
 Caused by hypoxia of pain fibres
 Felt in the thigh and buttock as well if the superficial femoral artery is
occluded
 Not felt in the peroneal, extensor or short muscles of the foot even
when the popliteal is affected
 The main symptom of lower limb arterial occlusion
18. Rest pain is continuing burning pain in the foot when the:
 Patient stops walking
 Foot is in the horizontal position
 Foot is elevated
 Foot is in the dependent position

4
 Patient is asleep
19. Signs of incipient gangrene of the foot include:
 Coldness and numbness
 Ulceration
 Brittle nails and thin, shiny, hairless skin
 Cyanosis of the toes
 Absence of popliteal or dorsalis pedis pulse

20. In the examination of a patient with suspected peripheral arterial


occlusion:

A. The lower the level at which pallor of the sole appears when the leg is
elevated, the more severe the occlusion
B. Red discoloration of the sole when the leg is hanging suggests that
the ischaemia is not severe
C. A bruit at a particular site is due to dilatation of the artery above
the site of obstruction
 Absence of the femoral pulse in both limbs is always indicative of
occlusion of the terminal aorta
 Muscles of the affected limb are not wasted
21 In patients with chronic arterial occlusion of the lower limb:
 About 40% improve spontaneously
 About 55% remain unchanged
 Only 5% require operation
 Amputation is required in about 90% of those requiring operation
 About half die within 5 years usually from C.V.A. or coronary
Thrombosis
22. In the medical management of chronic arterial occlusion of the
lower limb:

 Vasodilator drugs have been found effective

5
 Smoking should be reduced
 Fat intake should be low and the fat can be polysaturated
 Leg exercises may increase the claudication distance by 8% and
improve
the physical and psychological state
 Walking should be slow
23. Indications for operative treatment in chronic arterial occlusion of
the lower limb include:

 Intermittent claudication
 Rest pain
 Inability to walk more than 150-200 m
 Pregangrene
 Absent popliteal pulse

24. Substances currently used for by-pass grafting include:

A. Dacron
B. Autogenous artery
C. Autogenous saphenous vein
D. Glutaraldehyde stabilized umbilical vein
E. Nylon
25. Synthetic graft:

A.Is used for large vessels such as aorta


B.Is normally not used at or below the groin
C.Has blood clotting in its interstices and around it
D.Has fibrous tissue replacing the blood clot and the formation
of a pseudo-endothelium in the lumen
E. Has a patency rate at 5 years of about 90%
26 Ischaemia is much less common in the upper limb than in
the lower because:

 Arterial disease is uncommon in the arteries of the upper limb


 Of extensive collateral circulation in the arm and forearm
 The arteries are relatively wider
 The palmer arch of the hand provides vascular communication
between the radial and ulnar arteries
 Blocked forearm artery dilates to allow the flow of some blood

6
27. A cervical rib:

A. May articulate with the first rib on manubrium


B. Compresses the first and second dorsal roots of the brachial plexus
. and subclavian artery
C. Is often symptomless
D. May cause pain in the forearm precipitated by work and relieved by
rest,
ulceration of the fingers and absent or feeble radial pulse
E. Should always be excised
28. In Raynaud’s phenomenon:

A. Exposure to severe cold causes blanching or cyanosis


of the fingers
B. The blanching or cyanosis is due to sympathetic over-stimulation
C. Middle-aged women are more commonly affected
D. Gangrene may occur
E. Sympathectomy is required in most patients

29. Origins of systemic embolism include:

A. Right atrium
B. Infarcted myocardium
C. Aneurysm
D. Deep venous thrombosis
E. Subacute bacterial endocarditis
30. Common sites where a systemic embolus may lodge include:

A. Posterior cerebral artery


B. Aortic bifurcation
C. Bifurcation of the common iliac
D. Adductor canal
E. Hepatic artery
31. Clinical features of an embolus at the bifurcation of the popliteal
artery include:

 Sudden severe pain in the leg with associated paralysis


 Numbness and coldness of the leg
 Pallor of the leg
 Loss of sensation of the toes and forefoot
 Absent popliteal, posterior tibial and dorsalis pulses
32. In dry gangrene the affected part:

7
A. Is swollen
B. Is infected
C. Is black
D. Has shriveled skin
E. Develops a red line of demarcation between the living and
dead tissue in a few days
33. In moist gangrene the affected part:

A. Is macerated
B. Always develops a red line of demarcation between the living and
and dead parts in a few days
C. Is green or black
D. Is not usually infected
E. Is not swollen

34. Causes of dry gangrene include:

A. Injury to the main artery


B. Thrombo-angiitis obliterans
C. Extensive thrombosis of the veins of a part
D. Obstruction of both artery and vein
E. Sudden occlusion of the main artery
35. An aneurysm is:

A. A blood-filled sac which communicates with a vessel


B. True, if it is a localized dilatation of an artery
C. False, if the dilatation affects only a part of the wall
D. Dissecting if the dilatation is caused by the formation of a false
channel between the media and adventitia
E. Most commonly caused by syphilis
36. The commonest site for an aneurysm is the:

A. Aortic arch
B. Popliteal
C. Thoracic aorta
D. Femoral
E. Terminal abdominal aorta
37. Clinical features of an aneurysm include:

A. A fusiform or saccular swelling in the line of an artery

8
B. Marked pulsation
C. A thrill and systolic bruit over it
D. Reduction in the size and pulsation of the swelling when the artery
is compressed distal to the swelling
E. A smaller pulse distal to the swelling than on the contralateral side
38. In aneurysm of the abdominal aorta:

A. The vessel is at least 6 cm in diameter


B. Up to 80% of patients may die in the first year of diagnosis if
treatment
is not given
C. The swelling starts 3-5 cm above the renal arteries
D. The swelling extends to the common iliacs in about 50% of patients
E. The swelling extends anteriorly and posteriorly as it enlarges

39. An aneurysm of the abdominal aorta presents most commonly


with:
A. Abdominal and back pain
B. Pulsatile abdominal swelling
C. Impotence
D. Rupture heralded by abdominal or back pain
E. Intermittent claudication
40. The most useful of the following investigations for abdominal
aorta is:

 Plain abdominal X-rays


 Ultrasonography
 Aortograpy
 Intravenous pyelogram
 CAT-scanning
41. Conditions that may be associated with an abdominal aortic
aneurysm include:

A. Hypertension
B. Peptic ulcer
C. Mitral incompetence
D. Hydronephrosis
E. Ischaemic heart disease
42. In the treatment of abdominal aortic aneurysm:

9
A. Excision of the aneurysm and replacement with a dacron graft is
preferred to an inlay tube graft
B. Aggressive fluid and electrolyte therapy including infusion of
mannitol is essential
C. Ischaemia of one or both limbs may occur post-operatively
D. The mortality in elective cases is only about 2%
E. About 50% of patients surviving operation are alive at 5 years
43. Renal artery stenosis:

A. Is most commonly caused by fibromuscular dysplasia


B. Should always be suspected in a child or young adult with severe
hypertension without renal disease
C. Should be suspected in an atherosclerotic patient with hypertension
that is difficult to control
D. Is in every case associated with an epigastric bruit and accounts for
1-2% of hypertension
E. Is the most curable cause of hypertension

44. In the investigation of renal artery stenosis:

A. I.V.P. shows delayed excretion but more concentration of the


affected side because of reduced G.F.R.
B. Renal scan shows reduced vascularity and normal shape on the
affected side
C. Aortograpgy demonstrates the degree and extent of renal stenosis
D. Split renal function test shows depression of volume and concentration
and osmolality on the affected side
E. Differential rennin estimation of the blood is not helpful
45. In the treatment of renal artery stenosis:

A. Endarterectomy, bypass grafting or reimplantation of the renal


artery into the aorta should always be attempted even when
aortography demonstrates damage to the kidney
B. Nephrectomy should only be done if endarterectomy, bypass
grafting or reimplantation of the renal artery fails
C. Normotension is restored in 50% of patients after endarterectomy,
bypass grafting or re-implanatation of the renal artery
D. Fifty per cent do not show any improvement after endarterectomy, etc
E. Nephrectomy of the diseased affected kidney cures about 70%
46. Adequate venous circulation in the lower limb depends on the:

A. Competence of the valves of the veins especially those in the

10
superficial veins
B. Integrity of the osseo-fibrous compartment of the limb
C. Contraction of the veins
D. Contraction and power of the muscle bulk
E. Negative pressure in the abdominal cavity
47 The long saphenous vein:

A. Is formed by union of veins on the media aspect of the foot with


the dorsal venous arch
B. Passes behind the medial malleolus
C. At the medial aspect of the knee in just in front of the femoral condyle
D. At the saphenous opening passes backwards to join the femoral vein
E. Has no communication with the short saphenous

48. Varcose veins of the leg are:

A. Dilatation, elongation and tortuosity of the superficial and deep veins


of the leg
B. Rare in developing countries
C. More common in women
D. Caused primarily by ageing and obesity
E. Not always associated with reversal of blood flow from the deep
to the superficial veins
49. Symptoms of varicose veins of the leg include:

A. Increased fatigue on standing


B. Aching or burning sensation or leg pain on standing for brief periods
C. Severe aching pain or cramp in the leg brought on by walking and
relieved by rest
D. Marked persistent oedema of the ankle not relieved by elevating the
legs
E. Ache around the knee which is worse on waking up in the morning
50. A palpable cough thrill impulse at a point 3.7 cm below and lateral
to the pubic
tubercle in a patient with varicose veins suggests an:

 Associated femoral hernia


 Arterio-venous fistula
 Associated femoral aneurysm
 Associated saphena varix

11
 Incompetent sapheno-femoral valve
51. Skin changes in varicose veins include:

A. Scaling and weeping eczematous dermatitis


B. Atrophy
C. Pigmentation
D. Fibrosis
E. Deep fissures of the skin including the sole
52. The following may be found in a woman with varicose veins of a leg:

A. Varicose vulval veins


B. Dilated tortunous veins around the umbilicus
C. Fibroid uterus
D. Dilated superficial inferior epigastric vein
E. Hepatomegaly

53. A 40-year old woman has varicose veins of the right leg. When the
leg
is elevated with her in the recumbent position, the veins do not
collapse.
The cause of the varicosity may be:
 Competent of the valves of the saphenous veins
 Deep venous thrombosis
 Incompetence of the sapheno-femoral valve
 A pelvic tumour
 Obstruction of the femoral or external iliac vein
54. If in the Brodie-Trendelenberg test, the varicose veins fill up from
below
immediately the patient stands up and before the tourniquet is
removed,
then the:

 Sapheno-femoral valve is incompetent


 Valves of the communicating perforators are incompetent
 Valves of the saphenous system are necessarily incompetent
 Valves of the deep veins are necessarily incompetent

12
 There must be obstruction in the deep veins
55. If in the Brodie-Trendelenberg test, the varicose veins do not fill up
when the
patient stands up from above when the tourniquet is released then
the:

 Sapheno-femoral valve is incompetent


 Valves of the saphenous system are necessarily incompetent
 Valves of the communicating perforators are incompetent
 There may be obstruction of the external iliac or femoral vein
 Valves of the deep veins are incompetent
56. Which of these statements is true regarding the circulation of
blood in the
venous system of the lower limb?

 An upward movement in all the superficial veins


 A downward movement in the superficial veins
 A downward movement in the deep veins
 An upward movement in the deep veins
 None of the above
57. The dynamic system that maintains the venous circulation of the
lower
limb is dependent on:

 The integrity of the smooth muscle coat of the deep veins


 The union between the superficial and deep fascia
 Gravitational forces generated within the limb
 Integrity of the osseo-fibrous compartment of the limb
 Systolic pressure in the arterial input into the limb

58. The factors that maintain adequate venous circulation in the lower
limb
include:

 Muscle power in the limb


 Integrity of the osseo-fibrous compartment of the limb
 Competence of the valves guarding the communications between
deep
and superficial veins
 The smooth muscle coat of the deep veins

13
 Intra thoracic negative pressure maintenance
59. Varicose veins represent dilatation, elongation and tortuosity of
which
component of the lower limb venous system principally?

 The main trunks of the saphenous systems


 The main trunks of the deep femoral system
 The commicating system of veins
 The immediate tributaries of the superficial venous system
 The radicals of the long and short saphenous systems
60. The factors currently accepted as principally predisposing to the
development of varicose veins include:

 Vascular injuries from Road Traffic Accidents


 Repeated intimal inflammation – thrombophlebitis
 Hyperdynamic venous circulation from strenuous exercise
 Development of the compartment syndrome
 Familial defective, structural strength of the venous wall
 Increased hydrostatic pressure that follows prolonged standing
 Organisation of minute moral platelet thrombi in proxity to the valves
of veins
 Increasing age and obesity
61. A woman of 50 complains of aching pains and burning sensation
and
heaviness of the lower limbs especially towards the end of the day.
These pains are relieved when she puts her legs up at the end of
the day.
Examination shows no defects in the main pulses (femoral,
popliteal,
posterior tibial and dorsalis pedis).

The most likely diagnosis is:

A. Early peripheral arterial disease - atherosclerosis


B. Thromboangiitis obliterans (Buerger’s Disease)
C. Osteoarthritis of the hip and knee

14
D. Uncomplicated (Early) varicose veins
E. Peripheral neuritis
F. Disc lesion – L4/5
G. Peripheral arterial embolism
62. How should this lady’s symptoms be best investigated:

A. Do the Brodie-Trendelenberg Test


B. Ankle/Brachial Pressure Index estimation
C. Doppler ultrasound examination of the legs
D. Venography of the legs
E. Plethysmography of the leg
63. A man of 55 complains of intermittent swelling of the leg up to the
knee
darkening and scaling of the skin around the ankles over the past
three
years. He does not experience intermittent claudication and he has
no
pain in the feet at rest. Examination reveals evidence of varicose
veins
with incompetent perforators on the medial aspect of the knee.
The
peripheral pulses are all present and symmetrical.

The most like diagnosis is:


A. Uncomplicated varicose veins of the (Rt.) leg)
B. Lymphoedema of the leg
C. Deep venous thrombosis of the (Rt.) leg
D. Eczematous dermatitis from varicose veins
E. Recurrent subactute cellulitis of the leg
64. The most useful investigation for the management of this patient
is:
A. Ankle Brachial Pressure Index (ABPI)
B. Perthes Test
C. Duplex Ultrasound scan of the leg
D. Venogram of the leg
E. Retrograde arteriogram of the leg

15
65. A 40 year old diabetic man presents with recurrent itchy,
eczematous lesion
of his Rt. leg in the garter area, of nine months duration. He has
recently
experienced paraesthsia with recurrent breakdown of the skin. On
examination there is an ulcer just above the medial molleolus, with
shallow
sloping edges, firm, fibrotic base, depicting exuberant
granulations. There
are no varicose veins evident but there is a positive sapheno
femoral reflux
with a saphena varix.

The most likely diagnosis is:


A. Ischaemic leg ulcer
B. Diabetic ulcer
C. Venous ulcer
D. Epithelioma of the ankle area
E. Sickle cell ulcer
66. The most useful investigation for this ulcer is:

 Fasting blood sugar


 Ankle Brachial Presssure Index (ABPI)
 Duplex ultrasound examination of the limb
 Hb electrophoresis
 Wound swab for culture
67. The first line treatment for this patient is:

 Bed rest and careful local attention to heal the ulcer


 Excision and grafting of the ulcer
 Injection sclerotherapy for the superficial varicose veins
 Operative treatment by stripping of the long saphenous system
 Operative management-Cocket’s procedure
68. The typical varicose ulcer:

A. Is sited over or below the medial malleolus


B. Has a firm fibrotic base
C. Is very deep with sloping edges
D. Has exuberant granulation tissue in the floor
E. Does not become malignant

16
69. Bleeding from a varicose vein is:

A. Usually spontaneous but may be due to minor trauma


B. Usually associated with pregnancy
C. Often slight
D. Readily controlled by lowering the leg and applying local pressure
On the bleeding point
E. An indication for operative treatment of the varicose veins
70. Thrombophlebitis of varicose veins:

A. Is liable to occur in pregnancy


B. May be associated with oral contraceptive therapy
C. Often spreads to the deep veins
D. May lead to acute ulceration
E. Is usually treated by ligation to prevent spread
71. Indications for sclerotherapy in varicose veins include:

A. Pregnancy
B. Thrombo-phlebitis
C. Small varicose veins with competent valves of the communicating
perforators
D. Small varices following definitive surgery
E. Varicose veins with associated ulcer
72. The sclerosants currently in use for sclerotherapy of varicose vein
include:

A. Sodium morrhuate
B. Hydrocortisone in almond oil
C. Phenol in almond oil
D. Myodil
E. Sodium tetradecyl sulphate
73. The operative treatment of varicose veins:

A. Is ligation and transection of the long saphenous vein at the


sapheno-femoral junction and its removal with or without ligation
of the 6 or more tributaries joining its femoral part
B. Is undertaken even in the presence of complications such

17
as ulceration
C. Should in most cases be combined with Cockett’s operation
D. Has a mortality of about 1%
E. Has an average recurrence rate of less than 10%

74. What is the immediate treatment for the patient with varicose veins
complicated by acute bleeding from the region of the ankle?

A. Injection sclerotheraping for the varicose veins


B. Trendelenberg procedure to extract the veins
C. Local compression dressing to arrest the bleeding, investigation
and injection of the varices
D. Ligation of the bleeding varices
E. Bed rest with elevation of the limb
75. The contra indications to sclerotherapy for early varicose veins
include:
A. Atherosclorosis
B. Thrombophlebitis
C. Local infections
D. Local Allergies
E. Pregnancy
76. Currently the sclerosant medium of choice for sclerotherapy for
varicose
veins is:

Phenol in arachis oil


Phenol in almond oil
Sodium tetradecyl sulphate
Ethanolamine
Hypertonic saline
77. The principle underlying the modern technique of injection
sclerotherapy
for varicose veins of the lower limbs aims at:

 Inducing thrombosis of the varices


 Inducing intimal adhesions of the varices with minimal thrombosis
 Encouraging both initimal adhesions and thrombosis
 Perivascular fibrosis
 Dissolution of the varices

18
78. The indications for operative management of varicose veins include:

 Major varicosities
 Incompetent perforators
 First bleeding episode per se
 Saphenofemoral incompetence per se
 First episode of ulceration
 Complication with superficial thrombophlebitis

79. The attendant complication(s) of varicose veins that must first be


dealt with
prior to surgical intervention include:

 Eczematous dermatitis
 Ulcers – including grafting
 Thrombophlebitis
 Bleeding from varices
 Chronic lymphoedema
80. The Cockets procedure (subfascial ligation of incompetent
perforators) is
indicated in all patients with:

A. Incompetent perforators
B. Recurrent varicose veins
C. Recurrent ankle ulceration and incompetent deep perforators at the
ankle
D. Incompetent perforators at the ankle
E. Varicose veins and chronic oedema
81. What instructions should be given to patients who undergo the
Trendelenberg operation for stripping of varicose veins.

A. Should be confined to bed for a week to avoid bleeding


B. Should be made to walk within a few hours to a void DVT
C. Should have leg elevated for the first 24h then made to walk as much
as possible
D. Should avoid walking for a week
E. Should prefer standing to walking
82. The operation of endovenous lasser ablation (EVLA) for varicose

19
veins
has been shown by recent Rendomised controlled Trials:

 To be as effective as the standard Trendelenberg operation


 To give better results than the standard surgical interventions
 To be less effective than the standard Trendelenberg operation
 Not to ensure early return to work
 To have a higher recanalization and neovascularization than the
Trendelenberg operation

83. The lymphatic:


A. System drains 2-4 L of excess tissue fluid containing 75-200 g of
protein from all the organs back to the circulation daily
B. Capillaries form intercommunicating endothelial channels which
are freely permeable to macromolecules such as proteins and
bacteria
C. Vessels all have smooth muscle in their walls and valves which
allow lymph to flow centrally towards the central veins
D. Nodes are involved in immune responses and produce lymphocytes,
plasma cells and polymorphonuclear leucocytes
E. Terminal vessels, the right and left thoracic ducts, drain into the right
and left subclavian veins near their junction with the jugular veins
84. Inguinal lymphadenitis may be caused by:

A. N. gonococcus
B. Wuchereria bancrofti
C. Chlamydia trachomatis
D. Haemophilus ducreyi
E. Herpes genitalis
85. Acute pre-or post-auricular lymphadenitis may be due to infection
of the:

A. External or middle ear


B. Tonsils
C. Naso-oro-pharynx
D. Scalp
E. Mouth

20
86. External iliac adenitis or abscess:

A. Is in the iliac fossa above but continuous with the inguinal


ligament
B. Is often associated with fever and malaise
C. May follow infection of the glans penis, clitoris or lower limb
D. Is usually caused by Esch. Coli
E. Should be differentiated from appendix mass and septic arthritis
of the hip

87. In the differential diagnosis of lymphadenopathy:

A. Tuberculosis is the most likely cause if the age is under 10, and
and malignancy the most likely in those over 50
B. A short history is suggestive of acute infection and a long history
of chronic infection or malignancy
C. In the groin, pain is suggestive of acute inflammation, L.G.V. or
tuberculosis and absence of pain of malignancy or syphilis
D. Fever, malaise and myalgia are suggestive of acute or chronic
infection and not malignancy
E. Enlarged modes in the posterior triangle with a transient rash should
In endemic areas raise a suspicion of trypanosomiasis
88. In Lymphadenopathy:

A. Localized, firm, discrete or matted nodes with areas of cold


suppuration
or sinuses suggest tuberculosis
B. Localized, hard, discrete or mated nodes in a 50-year old man
suggest
filariasis
C. Enlarged epitrochlear nodes in the presence of generalized
lymphadenopathy
suggest syphilis
D. Maculo-papular rash with generalized lymphadenopathy, especially
if the spleen is enlarged, supports infectious mononucleosis,
toxoplasmosis
or Hodgkin’s lymphoma
E. Petechiae with generalized lymphadenopathy and hepato-
splenomegaly

21
maybe due to acute leukaemia
89. Investigations that will be useful in a 45-year old man with a 3-
month
history of enlarged lymph nodes in the right anterior triangle of the
neck
and a negative full clinical examination include:

A. V.D.R.L
B. Plain chest X-ray
C. Full blood count and E.S.R.
D. Blood film for microfilariae and aspiration of an enlarged node
E. Biopsy of an enlarged node

90. Lymphoedema:
A. Is accumulation of excessive amounts of interstitial fluid due to
inadequate
drainage as a result of obstruction of the lymph vessels
B. Affects the skin primarily
C. May also affect the muscles
D. Affects dependent parts of the body
E. Is primary if it is due to congenital mal-development of the lymph
vessels
91. Below-knee lymphoedema in a woman of 30 years in West Africa is
likely
to be due to:

A. Recurrent sepsis of the foot


B. Tuberculosis of the inguinal nodes
C. Wuchereria bancrofti
D. Lymphoedema tarda
E. Metastases in the inguinal lymph nosed
92. Lymphoedema of the whole leg and scrotum in a 40-year old male
in
West Africa is likely to be due to:

22
A. Lymphoedema praecox
B. Trauma to the thigh
C. Tuberculosis of the inguinal nodes
D. Brugia Malayi
E. Metastases in the inguinal lymph nodes
93. Lymphoedema of the vulva alone in a 40-year old woman in West
Africa
is likely to be due to:
A. Lymphogranuloma venereum
B. Tuberculosis of the inguinal nodes
C. Chronic pyogenic sepsis of the vulva
D. Lymphoedema tarda
E. Wuchereria bancrofti
94. Lymphoedema of a breast in a 40-year old woman in West Africa
is likely to be due to:

A. Brugia malayi
B. Tuberculosis of the axillary lymph nodes
C. Lymphodema congenital
D. Axillary node metastases
E. Recurrent infection

95 Complications of lymphoedema include:

A. Recurrent cellulitis and lymphangitis


B. Thrombophelitis
C. Lymphorrhoea
D. Ulceration
E. Lymphangio-sarcoma
96. If in lymphangiography of lymphoedematous leg of a 20-year old
girl,
the vessels are:

 Narrow and the lowest vertical nodes are not outlined while the
vessels
from the thigh and their corresponding nodes are normal, then the
cause is chronic sepsis
 Dilated and tortuous and the nodes are not visualized, then
tuberculosis
Is the likely cause

23
 Dilated and tortuous and the nodes are blocked or dilated, then the
likely
cause is filariasis
 Not visualized, then aplasis is the most likely cause
 Increased in number, caliber and tortuosity, then congenital varicose
dilation is the cause
97. Lymphoedema of the scrotum:
A. Is usually seen between 20 and 60
B. Is invariably accompanied by hydrocele
C. Does not usually involve the penis
D. May be associated with tiny vesicles of the skin of the scrotum
E. May be secondary to urinary fistulae
98. Unilateral lymphoedema of the lower limb in a 45 year old woman
should
be differentiated from:

A. Deep venous thrombosis


B. Congestive cardiac failure
C. Congenital arterio-venous fistula
D. Congenital neurofibromatosis
E. Obstruction of the inferior vena cava

99. Measures taken in the treatment of lower limb lymphoedema which


is soft
in a 35-year old woman due to chronic infection include:

A. Sleeping with the foot of the bed elevated


B. Application of well-fitting elastic stocking on the leg as soon as she
gets out of bed
C. Charles’s operation
D. Long-term antibiotic therapy
E. Course of frusemide
100. In the treatment of lymphoedema of the:
A. Lower leg, buried flap operation (Thompson’s operation) gives the

24
best cosmetic result
B. Breast, amputation should be done
C. Vulva, antibiotics and diuretics suffice
D. Scrotum, excision should be done
E. Upper limb, amputation should be done
101. The histological types of Hodgkin’s lymphoma include:
A. Lymphocytic predomdinant
B. Nodular sclerosis
C. Histio-cytic predominant
D. Mixed cellularity
E. Lymphocyte depleted
102. Hodgkin’s disease:
A. Is pathologically characterized by the presence of giant cells
B. Is distributed throughout the world
C. Occurs in the third decade and over
D. Is “curable”
E. Is granulomatous rather than neoplastic
103. Hodgkin’s disease may present:
A. Most commonly as cervical or supraclavicular lymphadenopathy
B. As persistent pyrexia of unknown origin or cyclic fever
C. Often with paraplegia
D. With weight loss, lassitude or pruritus
E. With an abdominal “mass”

104. Enlarged lymph nodes in Hodgkin’s disease are:


A. Single or confluent
B. Hard
C. Generally attached to skin
D. Usually movable
E. Painful and tender
105. Essential ROUTINE investigations in every patient with suspected
or
proven Hodgkin’s disease in West Africa include:
A. Bone marrow aspiration or biospy
B. X-ray of the chest

25
C. Lower extremity lymphangiogram
D. Lymph node biopsy
E. Staging laparotomy with splenectomy
106. In the Ann Arbor clinical staging of Hodgkin’s disease:
A. Stage I is limited disease of a single lymph node region or a single
extra-lymphatic organ or site
B. Stage II is involvement of 2 or more lymph node regions on the same
side of the diaphragm (II) or localized involvement of extra-lymphatic
organ or site and of one or more lymph node regions on both sides
of the diaphragm (IIE)
C. Stage III is involvement of lymph node regions on both sides of the
diaphragm (III) which may be accompanied by localized involvement
of the spleen (IIIS) or by solitary involvement of an extra-lymphatic
organ (IIIE) or both (IIISE)
D. Stage IV is diffuse or disseminated involvement of one or more
extra-lymphatic
organs or tissues with or without associated lymph node involvement
E. Lymphangiogram must be done in all cases
107. In the treatment of Hodgkin’s disease:
A. Stage I is treated by radiotherapy
B. Stages II is treated by radiotherapy and chemotherapy
C. Stages III and IV are treated by chemotherapy
D. The chemotherapy has been 6, 2-weekly cycles utilizing nitrogen
mustard,
vincristine, pro-carbazine and prednisone followed by a 2-week rest
period
E. Long-term disease-free remissions have been obtained in over 90%
of previously untreated patients

108. Lymphocytic and histiocytic lymphomas:


A. Occur throughout life with a peak incidence at 40-70
B. Present most commonly with extra-nodal involvement e.g. terminal
ileum mass
C. May also present with painless enlarged lymph nodes in the neck,
axillae
or groin
D. May also present with hepto-splenomegally, ascites, pleural effusion
lymphoedema
E. Do not require staging laparotomy with splenectomy as in Hodgkin’s
109. In the treatment of lymhocytic and histiocytic lymphomas:

26
A. Block dissection is done for Stage I
B. Radiotherapy is used for Stage I disease
C. For Stages III and IV, the trend is to use radiotherapy and cyclical
chemotherapy
D. Currently, cyclophosphamide, vincristine and prednisone are given
every
21 days for 6 courses
E. Prognosis is best in the well-differentiated histiocytic lymphoma
110. Burkitt’s tumour:
A. Is the commonest malignant neoplasm in children in Africa
B. Does not occur outside of Africa
C. Occurs in epidemic form in parts of Africa
D. Has a high incidence across Africa within a belt 10-15˚ north and south
of the equator below 1500 m in altitude
E. Has the highest age incidence in both sexes between 4-12 years with
a
median age at 7-8
111. The histology of Burkitt’s lymphoma shows:
A. Lymphoctes-poorly differentiated
B. Undifferentiated lymphoreticular stem cells
C. Eosinophils
D. Histiocytes-well differentiated
E. Macrophages with abundant clear cystoplasm
112. Burkitt’s tumour presents most commonly as:
A. Cervical lymphadenopathy
B. Ascites
C. Painless progressive enlargement of one or more jaw quadrants
especially of the maxilla
D. An Abdominal mass
E. Flaccid paraplegia

113. In females a characteristic feature of Burkitt’s tumour is:


A. Firm circumscribed swellings in both breasts
B. Ovarian tumours usually bilateral
C. Uterine tumour
D. Hepato-splenomegaly
E. Ascites
114. Burkitt’s tumour may involve the:
A. C.N.S. and cranial nerves
B. Testes

27
C. Lungs, salivary glands
D. Subcutaneous tissues, long bones
E. Thyroid
115. Essential investigations in every confirmed case of Burkitt’s
lymphoma
include:

 Liver biopsy
 Intravenous pyelogram
 Lumbar puncture for CSF cytology
 Staging laparotomy and splenectomy
 Bone marrow aspiration for cytology
116. Currently, the treatment of choice of Burkitt’s lymphoma without
CNS
involvement is:
A. Radiotherapy
B. Surgery for localized tumour
C. Cyclophosphamide administered as a single large dose of 40 mg/kg
I.V. and repeated for 4 courses at 2-3 weekly intervals
D. Methotrexate administered as a large dose I.V. followed by citrovorum
factor 24 h later
E. Cyclical combination chemotherapy with cyclohosphamide, vincristine
and methotrexate at 2-3 weekly intervals
117. It has been advised that following successful induction therapy in
Burkitt’s
lymphoma, there should be:

A. Maintenance therapy with cyclophoshamide


B. Maintenance therapy with methotrexate
C. Maintenance therapy with cytosine arabinose
D. No further treatment
E. Intrathecal administration of methotrexate or cytosine

118. Measures that must be taken during treatment of Burkitt’s


lymphoma
include:

A. Blood transfusion
B. Monitoring of serum electrolytes and blood urea
C. Monitoring of blood uric acid
D. Administration of antibiotics

28
E. I.V. fluid therapy
119. In the prognosis of Burkitt’s lymphoma:
A. Long-term survival varies from 40-65%
B. About 50% of those showing remission have a relapse
C. Over 80% of those showing relapse have C.N.S. involvement
D. Chemotherapy has not been effective in relapse
E. Serum from cured patients has given favourable response in replapse
120. Sabfasical endoscopic perforator surgery has:
A. Proven a veritable minimally invasive alternative to open surgery
 Lower recurrence rate to conventional open surgery
 A higher neovascularization rate than conventional open surgery
 A lower rate of failure to ligate long saphenous vein tributaries
 A high success rate in locating and ligating significant perforators
121. Infective agents that have been implicated in the etiology of
lymphomas
include:

A. Helicobacter pylori
B. Human herpes virus-8
C. Hymenolepis nana
D. Hepatitis A
E. HTLV type I
122. The following are true of Hodgkin’s lymphoma:

 Majority of cases occur in the elderly


 The characteristic cell is the Reed-Sternberg cell
 It is rare in Jews
 Presenceof unexplained fever affects prognosis
 It is usually disseminated at the onset

123. In a patient with stage IIA Hodgkin’s Lymphoma you would expect:

A. Drenching night sweats


B. No signifiant weight loss
C. Bulky disease

29
D. Involvement of two or more lymph node regions on the same side
of the diaphragm
E. Unexplained fever
124. Gastric MALT lymphoma:

A. Is assoicated with H. pylori infection


B. Has origins from a mature B cell
C. Is associated with an increased incidence of perforations
D. Develops following EBV infection
E. Is associated with a history of long standing peptic ulcer
125. In endemic Burkitt:

A. Children are most often affected


B. Marrow involvement occurs early
C. The malignant cell is highly sensitive to Cyclophosphamide
D. EBV antibodies are usually absent
E. Extranodal sites are usually involved

CHAPTER 50
VASCULAR AND LYMPHATIC SYSTEMS

ANSWERS

1. B, C 38. B, E 75. A, B, C, D, E
112. C, D, E
2. A 39. D 76. C

30
113. A, B
3. D, E, 40. E 77. B
114. A, B, D, E
4. A, C, D, E 41. A, B, D, E 78. A, B
115. B, C, E
5. C 42. B, C, E 79. A, B, C, D, E
116. C
6 A, D, E 43. B, C, E 80. C
117. E
7. C 44. C, D 81. C
118. A, B, C, E
8. A, B, D 45. C 82. A
119. A, B
9. C 46. B, D 83. A, E
120. C
10. A, C, E 47. A, D 84. A,B, C, D, E
121. A, B, E
11. A, B, C, D 48. C, E 85. A, D
122. B, D
12. B, C, E 49. A, B 86. A, B, C, E
123. B, D
13. A, C, E 50. E 87. B, E
124. A, B
14. C 51. A, B, C, D 88. A, C, E
125. A, C, E
15. A, B, C, E 52. A, C, D 89. B, C, E
16. A, B 53. B, D, E 90. A, D, E

17. A, E 54. B 91. A, D


18. B, C 55. A 92. C, E
19. A, B, D 56. D 93. A, C
20. A 57. D 94. B, D
21. A, E 58. A, B, C, E 95. A, C, D, E
22. D, E 59. D 96. A, B, C, D, E
23. B, D 60. E, F, G, H 97. B, D, E
24. A, C, D 61. D 98. A, C, D
25. A, B, C, D 62. C 99. A, B, D, E
26. B, D 63. D 100. A, B, D,
27. A, C, D 64. C 101. A, B, D, E
28. A, D 65. C 102. B, D
29. B, C, E 66. C 103. A, B, D, E
30. B, C, D 67. A 104. A, D
31. A, B, C, D 68. B, D 105. B, D, E
32. B, C, D, E 69. A 106. A, C, D
33. A, C 70. A, B 107. A, C, D
34. B 71. C, D 108. A, C, D
35. B, D 72. E 109. B, D
36. E 73. A, E 110. A, D, E
37. A, C, E 74. C 111. B, E

31
CHAPTER 51

SURGICAL ASPECTS OF THE


HAEMOGLOBINOPATHIES

 In West Africa the incidence of the sickling trait varies:


 From 10% in Northern Ghana to 30% in the Northern Nigeria
 From 5% in Northern Ghana to 10% in the Northern Nigeria
 From 20% in Northern Ghana to 15% in the Northern Nigeria
 From 15% in Northern Ghana to 20% in the Northern Nigeria
 From 25% in Northern Ghana to 25% in the Northern Nigeria
 Noted features of sickle cell crisis include:
 The hand and foot syndrome
 Acute tender hepatomegaly
 Tachycardia and tachypnoea
 Sequestration syndrome
 Recurrent jaundice
 An Africa boy of 12 presents with sudden onset of fever,
nausea, vomiting and diarrhoea of 24 h duration. He admitted
to two previous episodes of similar illness which required
short periods of detention at the local health centre. On
examination he had tachycardia, tachypnoea, a tingue of
icterus with a palpable tender liver. Rectal findings were non-
contributory.

The most likely diagnosis is:


 Amoebic liver disease
 Severe attack of shigellosis
 Subacute hepatitis
 Sickle cell crisis
 Viral hepatitis
 In sickle cell anaemia (Hb SS genotype) the average steady
state Hb level is about (g/dl):
 7
 8

1
 9
 10
 11

 Hepatomegaly in sickle cell disease is primarily due to:


 Congestive hepatic crisis
 Cholestasis
 Viral hepatitis
 Amyloidosis
 Multiple liver infarcts with abscess formation
 Generalized lymphadenopathy is seen in which of the following
categories of sickle cell patients?
 In 5% of SC patients and over 90% of SS patient
 In 10% of SC patients and over 90% of SS patient
 In 15% of SC patients and over 70% of SS patient
 In 20% of SC patients and over 60% of SS patient
 In 25% of SC patients and over 50% of SS patient
 A 15-year old Ghanaian boy presented with a recurrent ulcer
just above the medial malleolus of his leg of four years
standing. There was no history of trauma but he admitted to
recurrent episodes of fever diffuse, bodily pains and weakness.
The ulcer itself had flat gently sloping edges with pale
granulations in the floor.

The most likely diagnosis is:


 Varicose ulcer
 Gravitational ulcer
 Chronic myeloid leukaemia
 Sickle cell leg ulcer
 Non-specific ulcer of infective origin

2
 Which investigation will confirm the diagnosis?
 Blood film
 X-ray of the tibia
 Hb electrophoresis
 White cell count
 Venogram
 You will advise:
 Ambulatory treatment with bandaging
 Application of bland antiseptics and bed rest
 Surgical debridement and grafting
 Application of viscopaste to the leg
 Dressing with Solcoseryl
 A girl of 18 with Hb SC disease presented with acute
respiratory distress, cough and frothy sputum and soon after
admission became drowsy. On examination, the temperature
was 37˚C and there were petechial rashes on both arms; there
were no pulmonary signs.

The most likely diagnosis is:


 Lobar pneumonia
 Acute right heart failure
 Pulmonary embolism
 Fat embolism
 Left ventricular failure
 The most useful investigation is:

3
 Bronchoscopy
 Pleural tap
 Radioactive Xenon scan of the chest
 Examination of the urine for fat globules
 ECG

 You would advise


 Penicillin G I M 1mega unit 6 hourly
 Intercostal tube drain
 Anticoagulation with heparin
 Intermittent oxygen, IV 5% Dextrose with 5% alcohol and
heparinisation
 Digitalization
 The bone changes consequent on sickle cell disease may take
the
form of:
 Erythroid hyperplasia
 Osteoporosis affecting the inner aspects of the cortex
 Avascular necrosis of the cortical bone
 Sclerosis of involved bone
 March fractures of the metatarsals
 The bone changes of sickle cell disease are particularly
encountered in the:
 Pelvis
 Skull
 Vertebral bodies
 Metacarpals
 Radius
 Which of the following lesions may be found in the sickle cell

4
patient?
 Osteochondritis dissecans
 Subchondral and central sclerosis
 Coxa vara
 Flatening and irregularity of the femoral head
 Milkman’s fracture

 The initial treatment of choice for the management of the bone


lesion of sickle cell disease affecting the hip is:
 Active physiotherapy
 Prolonged bed rest with the limb in traction
 Containment upper femoral osteotomy
 Prophylactic chloramphenicol
 Increasing calcium and vit. D supplements
 Sickle cell crisis:
 May be precipitated by any cause of diminution in the tissue
oxygen tension
 Occurs most commonly in children and young adults
 May present predominantly as the “hand and foot syndrome”
 In its abdominal form is often clinically indistinguishable from
acute intestinal obstruction
 May present as a rapidly developing hepatomegaly or
splenomegaly
 Sickle cell crisis:
 Is primarily related more to the pH than the oxygen tension in the
tissues
 Shows a marked tendency to diminish in frequency over the age

5
of 30
 Is frequently marked by muscle tenderness
 In its abdominal form may be indistinguishable from pancreatitis
 Reticulocytosis ranging from 10-15% may be associated with it
 In sickle cell disease:
 Not all patients are found to be anaemic
 Symptoms only begin to manifest after 3 months of life
 The average (steady state) Hb level for SC patients is 8g/dl
 The most severe anaemia occurs with SB thalassaemia
 Growth disturbance may produce a stunted figure, much below
average weight

 In sickle cell disease:


 Complete deoxygenation causes the red cell to become ovoid,
shortened and crescentic
 The PCV is above normal
 HbC has dilutional effect on the sickling process more than HbA
 Growth disturbance may result in tall asthenic body build
 Low levels of gonadotrophin are encountered
 Two weeks after successful uneventful delivery of her second
baby a West African woman of 30 is brought to the Emergency
Room with a sudden attack of abdominal pain, nausea and
vomiting with headaches joint pains and fever. There was a
similar episode after her first pregnancy which settled
spontaneously. On examination she is moderately pale and
dehydrated; the pulse is 90min regular and BP 90/70. The chest
is clear and the abdomen is diffusely tender without guarding,
but a vague tender mass is palpable in the left hypochondrium.

The most likely diagnosis is:

6
 An attack of falciparum malaria
 Typhoid fever
 Sequestration syndrome of sickle cell disease
 Splenic infarct
 Viral hepatitis A
 The most useful diagnostic investigation is:
 Plain x-ray of the chest-erect
 Plain x-ray of the abdomen-erect
 Abdominal ultrasound
 Liver function tests
 A full haematological profile

 The indicated treatment is:


 Resuscitation with I/V fluids and broad spectrum antibiotics
 Bed rest with attention to nutrition
 Resuscitation with I/V fluids, broad spectrum antibiotics,
antimalarial therapy, close monitoring P02, PC02, PH
 Resuscitation and early surgical intervention
 Resuscitation + Tripple Therapy for peptic ulcer
 A child of ten noted for his complaints of cold season aches
and pains is seen in the Emergency Room in his third attack of
fever headaches excruciating pain in the back and limbs. On
examination he was not pale nor icteric but his tongue was
furred and coated. His legs felt warm and there was tenderness
especially proximal to the left medial malleolus. X-rays of the
limbs were grossly normal.

7
The most likely diagnosis is:
 Recurrent attacks of malaria
 Recurrent attacks of typhoid
 Osteomylitis of the left tibia
 Hand foot syndrome of sickle cell disease
 Attacks of Dengue fever
 The most useful diagnostic test is:
 Blood film for malarial parasites
 Blood culture
 A full haematological profile
 Chest x-rays
 Liver function tests

 The indicated therapy is:


 A course of Artesunate – Amodiaquine for 3 days
 A course of Ciprofloxacin and Metronidazole
 Course of Cloxacillin 500mg qds x 7 days
 Resuscitation + broad spectrum antibiotics antimalarials and
monitoring P02, PCO2 PH
 Bed rest, adequate nutrition and splinting the affected joints
 A young lady of 15 comes to the Casualty Department with
sudden onset of abdominal pain initially periumbilical, but later
settling in the (Rt.) lower quadrant of the abdomen. The patient
also complains of fever headaches, vomiting and dysuria. He
remembers two previous similar attacks that settled
spontaneously. On examination she is pale with a temperature
of 39°C; the chest is clear, but the abdomen is diffusely tender
without guarding and rectal examination is unremarkable. The

8
patients blood count is reported as “Total WBC 40,000/cub
mm, including normoblasts with reticulocystosis – 12%.

The most likely diagnosis is:


 Acute appendicitis
 Mesenteric adenitis
 Salmonellosis
 Cholecystitis
 Sickle cell crisis
 Which test is most likely to yield further useful information
 Full haematological profile
 Abdominal ultrasound
 Plain x-ray of the abdomen-erect
 Blood culture
 Abdominal paracentesis

 The indicated therapy for this patient is:


 Resuscitation followed immediately by appendectomy
 Exhibition of bactriocidal antibiotics
 Resuscitation with I/Vs + Ciprofloxacin and Metronidazole
 Resuscitation with I/Vs bacteriocidal antibiotics and monitoring of
abdominal signs, PCO2, PO2 and PH
 Resuscitation + Tripple therapy
 The average steady state Hb maintained by patients with
Sickle Cell disease (Genotype SS) in the typical socio-
economic circumstances in West Africa is:
 8gm/dl
 9gm/dl
 10gm/dl

9
 11gm/dl
 12gm/dl
 The average steady state Hb maintained by patients with Sickle
Cell disease (Genotype SC) in the typical socio-economic
circumstances in West Africa is:
 8-9gm/dl
 10-11gm/dl
 12-13gm/dl
 14-15gm/dl
 16gm/dl

 A girl of 12 known to have Sickle Cell disease (Genotype SS)


has had episodes of colicky abdominal pain and dyspepsia,
fever, and fluctuating jaundice over the past 3 years. During
these attacks the urine has been dark but the stools have
retained a normal colour. On examination she was jaundiced
and mildly pale. The abdomen was full, the liver and spleen
were palpable and non tender; there were no other masses.
Normal stool was encountered on P.R. Liver function tests
indicated both direct and indirect hyperbilirubinaemia and a
threefold increase in the alkaline phosphatase level.
Abdominal ultrasound was essentially normal.
The most likely diagnosis is:
 Cholestatic jaundice from gallstone obstruction of the CBD
 Cholestatic jaundice from intra canalicular cholestasis

10
 Haemolytic jaundice attendant on repeated episodic sickle cell
crises
 Hepatocellular Jaundice – Hepatitis A
 Hepatocellular jaundice – Hepatitis B
 The test most likely to clinch the diagnosis is:
 Percutaneous Transhepatic Cholangiography (PTC)
 Endoscopic retrograde Cholangiopancreatogrophy (ERCP)
 Magnetic Resonance Cholangiopancreatogrophy (MRCP)
 Hepatic Scintigraphy
 Helical CT Scan of the abdomen
 The treatment for the patient in Question 32 is:
 Resuscitation + adequate nutritional supplements
 Resuscitation + check on G6PD status and review of charted
drugs given
 Bed rest and attention to urgent nutrition needs
D. Resuscitation + monitoring for signs of progression and need for
surgical
intervention
E. Resuscitation, I/V administration of 10% fructose, Vitamin. K and
prophylactic antibiotics

 It is evident that the patient in Question 32 would need surgery;


which of these procedure(s) is mandatory at this operation:
 Drainage of the peritoneal cavity
 Drainage of the subphrenic spaces on the right
 Administration of Vit. K during the surgery
 Operative cholangiography + T tube drainage of biliary free
 Establishment of a choledocho-duodenostomy
 Which of these features confirms a state of significant hyper-

11
splenism in a patient with splenomegaly in Sickle Cell Disease,
and therefore requiring splenectomy?
 The spleen is > 4cm below the Lt. costal margin
 The HB falls < 6.5g/dl
 The reticulocyte count exceeds 15%
 The platelet count falls below 200 x 109/l
 None of the above
 A man of 25 presents in the Emergency Room with a sudden
onset of severe pain in the left hypochondrium, much
worsened on inspiration. He is a pyrexial, and anicteric; his
chest is clear and has a tender mass that moves with
respiration in the left hypochondrium. There have been no
previous attacks:

The most likely diagnosis here is:


 Pancreatic pseudocyst
 Subphrenic abscess
 Splenic infarct
 Left diaphragmatic pleurisy
 Amoebic liver abscess affecting the left lobe

 Which one diagnostic test would you choose to clinch the


cause for this presentation:
 Chest x-ray
 Abdominal ultrasound
 A full haematological profile
 CAT Scan of the abdomen
 Examination of fresh specimen of stool
 Hepatomegaly in sickle cell disease:

12
 May be due to viral hepatitis
 Is attended by the morbid anatomical change of sinusoidal
engorgement with fibrin clots
 Is marked by central lobular parenchymal necrosis
 Is very frequently attended by jaundice of obstructive
intrahepatic type
 May be accompanied by fever, leucocytosis and abdominal pain
 Hepatomegaly in sickle cell disease:
 Is frequently associated with extrahepatic ductal obstruction by a
stone
 Is attended by sinusoidal haemosiderosis
 May be associated with biliary canalicular thrombi
 May on occasion be an indication for laparotomy
 Is often seen with anasarca
 Jaundice in sickle cell disease:
 Is most frequently acholuric arising from excessive red cell
destruction
 May produce an elevation of both direct and indirect reacting
bilirubin
 May be difficult to distinguish from extrahepatic ductal
obstruction
 May on occasion be an indication for laparotomy
 Should initially be treated conservatively with adequate fluids
and nutritional supplements

 The spleen in sickle cell disease:


 Undergoes progressive fibrosis from repeated infarcts
 Is one of the organs involved in the sequestration syndrome

13
 Is characteristically reduced to a small nubbin of fibrous tissue in
adolescent SS patients
 Is seldom reduced in size in patients with SC disease
 May occasionally require removal as a measure to reduce
frequency of haemolytic episodes
 The kidney in sickle cell disease:
 May show defective tubular concentration in the form of
hyposthenuria
 Has a tendency to excessive concentration of urine
 Is often grossly enlarged (symmetrically)
 Need not be investigated for other causes of haematuria in
patients presenting with haematuria
 If the source of unilateral presistent bleeding may require
surgical excision
 Haematuria in sickle cell disease patients:
 Occurs in 2-4% of Hb SS and Hb SC patients
 Is more likely to arise from causes other than the sickling lesion
in the kidney
 May be prolonged and recurrent and usually painless
 Is treated with blood transfusion, alkaline infusions and
occasionally aminocaproic acid
 May warrant a nephrectomy if unilateral and persistent

 Haematuria in sickle cell disease patients:

14
 Arises usually from ulcerations of the renal papillae, the result of
papillary infarct
 Is likely to be persistent because of constant contact of the
ulcerating lesion with urokinase
 Is usually total and recurrent
 May be treated by blood transfusions, alkaline infusions and
diuretics
 Is never life threatening enough to warrant nephrectomy
 The recurrent haematuria which occurs in some 2-4% of patient
with Sickle Cell disease (Hb SS and SC) arises from:
 Ulceration of the tips of the renal papillae
 Infarction in the medulla of the kidney
 Sickling phenomena in the cortex of the kidney
 Ulceration in the trigone of the bladder
 Infacts of the mucosa of the renal pelvis
 The bleeding in Sickle Cell haematuria becomes repetitive and
persistent because:
 Of repeated episodes of sickling crisis
 The hyposthenuria that accompanies Sickle Cell disease
 Febrinolytic enzymes (urokinase) being continuously in contact
with
the urine
 High pH of the urine in Sickle Cell disease
 Attendant urinary tract infection
 The first line management in Sickle Cell patients with
haematuria is:
 Intravenous infusion of alkalinizing agents (NaHCO3)
 Blood transfusions
 Administration of diuretic agents
 Infusions of Amino caproic acid

15
 Infusion of ringers lactate

 Priapism in sickle cell disease:


 Occurs in 60% of cases in boys under 12 years of age
 Is episodic and tends to occur during sleep
 Is liable to produce pain, distress and exhaustion
 Usually eventually settles on conservative measures
 Is liable to lead to penile fibrosis if allowed to persist for several
weeks
 Priapism in sickle cell disease:
 Occurs in 80% of cases in boys over 12 years of age
 Is more likely to occur during the day than at night
 Seldom distresses the patient
 May be treated by sapheno-cavernosal anastomosis if persistent
 May in recurrent cases be complicated by impotence
 The priapsm that occurs in patients with Sickle Cell disease:
 Most frequently occurs in adolescents > 18 yrs
 Starts with initial sickling in the corpus spongiosum
 Is recognized as severe if it lasts about 3h
 Should be recognized as an emergency
 Should be treated prophylactically by caverno-spongiosal shunts
 Leg ulcers in sickle cell disease:
 Are seen in 10% Hb SS patients in West Africa
 Occur in 90% of cases on the lower third of the leg above the
medial malleolus
 Are bilateral in a third of cases
 Arise from a combination of factors, trauma, infection and local
circulatory deficiencies

16
 Should be treated by bed rest and wound dressing with mild
antiseptics

 Leg ulcers in sickle cell disease:


 Are seen in 2% of Hb SC patients in West Africa
 Occur in 50% of cases on the lower third of the leg above the
medial malleolus
 Are bilateral in 20 per cent of cases
 Are seldom complicated by malignant change
 May be treated by surgical debridement and grafting
 Regarding leg ulcers in Sickle Cell disease:
 The incidence in the West Indies is 60% of Hb SS patients
 The incidence in the Ghana is 10% of Hb SS patients
C. The incidence in the USA (Blacks) is 10% of Hb SS patients
D. The incidence in the West Africa is 2% of Hb SC patients
E. The peak incidence in the West Indies occurs in 10-14 year
group
F. The peak incidence in the West Africa occurs in 15-19 year
55. The site of election for leg ulcers in Sickle Cells disease
patients is:
A. Lower third of the leg above the medial malleolus
B. Front of the tibia
C. Dorsum of the foot
D. Sole of the foot
E. Over the lateral malleolus
56. The leg ulceration that occurs in Sickle Cell disease (Hb SS):
A. Is bilateral in one-third to half the cases

17
B. Is more often than not associated with varicose veins with
incompetent
perforatiors
 Is usually deep seated with exposure of muscle, tendon and
periosterum
 Shows no stratification of incidence in socio economic groups
 Is frequently complicated by malignant degeneration

57. In the management of the chronic ulcers that complicate


Sickle-Cell
Disease thrust of therapy should be:
 Rapid elevation of the Hb level by multiple transfusions
 Early ulcer excision and pinch grafting
 Bed rest with adequate nutrition and local dressings
 Emphasis on supply of trace elements – (zinc sulphate: 200mg
tds)
 Routine grafting of ulcers with healthy granulation tissue
58. Lobar pneumonia in sickle cell disease:
A. May precipitate or follow a crises
B. Occurs approximately a hundred times more frequently than in
the
general population
C. Has a tendency to recurrence (70%) and multiple lobe
involvement (40%)
 Has a predilection for the upper lobes
 May be accompanied by pulmonary hypertension and right
ventricular hypertrophy
59. Lobar pneumonia in sickle cell disease:

18
A. Is seldom a cause for a crisis
B. Occurs as frequently as in the general population
C. Characteristically affects the lower lobes
D. Is common because of pulmonary congestion, thromboses and
impairment of immunity on account of autosplenectomy
E. Is often complicated by cardiac arrhythmias and right heart failure

60. Pneumonia is known to occur approximately a hundred times


more
frequently in patients with Sickle Cell disease than in the
general
population. The reason(s):
 Impairment of polymornuclear phagocytic action by pulmonary
congestion
 Thrombotic episodes in the pulmonary circulation
 Deficiency in heat labile opsonizing activity of the serum against
the pneumococcus
 Frequency of functional auto-splenectomy in the patient with SS
disease
 High level of lung tissue hypoxia
61. Ocular changes in sickle cell disease may take the form of:
A. Sluggish flow with minimal arteriolar occlusion especially at the
periphery of the fundus
B. Vascular occlusion with demonstrable interruption of blood flow in

19
capillaries of periphery of fundus
C. Retinal, choroidal and anterior uveal infarcts
D. Micro-aneurysms, dilated capillaries and veins
E. Proliferative neovascularization of the vitreous from which vitreous
haemorrhages may occur
62. Ocular changes in sickle cell disease may take the form of:
A. Heightened flow in the arterioles of the fundus with silver wiring
appearance
B. Ischaemic areas showing whitening of the retina
C. Haemorrhages of varying severity which may be retinal, peri-
retinal
D. Neovascularization of the retina, budding capillaries, a
compensatory
reaction to retinal ischaemia
E. Tears and detachment of the retina

63. The earliest change in development of ocular changes in Sickle


Cell
disease is:
A. Vascular Occlusion of arterioles and capillaries at the periphery
of the
retina
B. Dilatation and tortuosity of vessels at the posterior pole of the
fundus
C. Retinal Neovascularistion – budding capillaries attempting to
bridge
adjacent arterioles and venules
D. Vitreous haemorrhages
E. Retinal detachment

20
64. In the management of patients with sickle cell disease:
A. Blood transfusion is more useful than alkaline infusions in
decreasing
the sickling tendency
B. Oxygen therapy has no immediate effect on the sickling process
C. Prophylactic antimalarial therapy is of proven benefit
D. Prophylactic Vit. B12 has been found beneficial
E. Efforts should be made to achieve pre-operative Hb levels of 12
g/dl
in adults
65. In the management of patients with sickle cell disease:
A. Expanding the plasma volume using alkaline and dextrose
infusions
decreases the sickling tendency
B. Oxygen therapy relieves the pain and other distressing
symptoms
accompanying crises
C. Search for and prompt treatment of acute infections may abort a
crisis
D. Anticoagulants such as Arvin may prove useful in aborting or
arresting
an attack
E. Efforts should be made to achieve pre-operative Hb levels of 10
g/dl
in children

66. The changes in the vertebral column in sickle cell disease:


A. Affect particularly the lumbar vertebral bodies
B. Lead to reduction in height of the vertebral bodies
C. Result in bulging of the intervertebral discs into the bodies

21
D. Take the form of the classic cod-fish appearance
E. May take the form of collapse of the vertebral bodies
67. The pathological changes of sickle cell disease affecting the
hip:
A. Are common and indistinguishable from Perthes’ disease
B. Are more common in Hb SS patients than SC or other varieties
C. Are usually related to some sepsis in the upper end of the
femur
or hip joint
D. May produce septic dislocation of the hip
E. May result in total sequestration of the femoral head
68. The pathological changes of sickle cell disease affecting the
hip:
A. Are rare and distinct from all childhood hip affections
B. Usually results in bony ankylosis
C. Are nearly always the result of early experience of trauma to the
hip
D. Are frequently arrested by prolonged bed rest with the affected
limb
in traction
E. Are seldom bilateral
69. The head of the humerus in sicklers:
A. Is notably subject to infarction though not as frequently as the
femur
B. Does not deform as badly as the femoral head
C. Is more liable to develop early osteoarthritis than in the normal
population
D. Usually escapes early osteoarthritis
E. Showing avascular necrosis is best treated by short wave
diathermy
and mobilization exercises
70. Osteomyelitis in sickle cell disease:

22
A. Affects nearly 90% of Hb SS patient in one or more bones
before
the age of 10
B. May start anywhere along the shaft of the long bone
C. Usually starts in the metaphysis and spreads down the
diaphysis
D. Has a less stormy course than in the Hb AA patient
E. Should be treated by parenteral ampicillin and cloxacillin
71. Osteomyelitis in sickle cell disease:
A. Affects nearly 50% of Hb SS patients in one or more bones
before
the age of 10
B. Not uncommonly produces segmental lesions along the shaft
of the same bone
C. Is most frequently produced by staph. pyogenes
D Is frequently complicated by septicaemia
E. Should be treated by oxygen therapy, antibiotics, limb
immobilization
and timely evacuation of intramedullary pus
72. For children with Sickle Cell disease the principal cause of
post
operative morbidity and mortality is:
A. Acute chest syndrome and splenic sequestration
B. Post operative malaria
C. Dehydration
D. Urinary tract infection
E. Low Hb in the post operative period Hb < 8.0g/dl
73. In patients with Sickle Cell disease undergoing operation it
is generally
recommended that to obtain the normal P02 tension in the
system the
oxygen ventilation during surgery should be at least:

23
A. 20%
B. 25%
C. 30%
D. 35%
E. 40%
CHAPTER 51

SURGICAL ASPECTS OF THE ABNORMAL


HAEMOGLOBINOPATHIES

ANSWERS

1. A 38. C
2. A, B, C, D, E 39. A, B, C, E

3. D 40. B, C, D

4. B 41. A, B, C, D, E

5. A 42. A, B, C, D, E

6. A 43. A, C, E

7. D 44. A, B, C, D, E
8. C 45. A, B, C, D

9. C 46. A

10. D 47. C

11. D 48. A

12. D 49. A, B, C, D, E

13. A, B, C, D 50. D, E

14. A, B, C, D 51. D
15. A, B, C, D 52. A, B, C, D, E

16. B 53. A, D, E

17. A, B, C, D, E 54. A, B, C, D, E, F

18. B, C, D, E 55. A
19. A, B, (D) E 56. A

24
20. C, D, E 57. C
21. C 58. A, B, C, E

22. E 59. C, D, E
23. C 60. A, B, C, D, E

24. D 61. A, B, C, D, E,
25. C 62. B, C, D, E
26. D 63. B
27. E 64. C
28. A 65. A. B, C, E
29. D 66. A, B, C, D, E
30. A 67. A, C, D, E
31. B 68. D
32. C 69. A, B, C, E
33. C 70. A, B, E
34. D 71. B, C
35. D 72. A
36. A, B, C, D 73. C
37. C

25
CHAPTER 26
THE HEART

1. A wide pulse pressure is indicative of:


A. Aortic insufficiency
B. Aortic stenosis
C. Patent ductus arteriosus
D. Aortopulmonary window
E. Mitral stenosis
2. Structures forming the left heart border on plain chest x-ray include
A. Ascending aorta
B. Aortic knuckle
C. Left pulmonary artery
D. Left atrial vestibule
E. Left ventricle
3. Pulmonary plethora with enlarged pulmonary arteries on x-ray is
consistent
with:
A. Pulmonary atresia
B. Atrial septal defect
C. Tetralogy of Fallot
D. Patent ductus arteriosus
E. Chronic constrictive pericarditis
4. In the plain P- A chest film the right heart border is formed by the:

A. Superior vena cava


B. Right atrial appendage
C. Right atrium
D. Right ventricle
E. Inferior vena cava
5. The pulmonary wedge pressure is assumed to be the pressure in the:
A. Infundibulum of the right ventricle
B. Main right ventricular chamber
C. Right atrium
D. Left atrium
E. Left ventricle
6. Right heart catheterization is done through:
A. A femoral artery
B. A femoral vein
C. The right brachial artery

1
D. An antecubital vein
E. A radial artery

7. The normal pressure (mmHg) within the:


A. Right atrium is 0-40
B. Right ventricle is 0-60
C. Left atrium is 0-12
D. Left ventricle is 10-120
E. Main pulmonary artery is 50-100
8. The commonest congenital heart defect is:
A. Co-arctation of the aorta
B. Patent ductus arteriosus
C. Atrial septal defect
D. Ventricular septal defect
E. Pulmonary stenosis
9. Acyanotic congenital heart defects include
A. Patent ductus arteriosus
B. Truncus arteriosus
C. Atrial septal defects
D. Pulmonary valvular stenosis with atrial septal defect
E. Congenital aortic stenosis
10. Cyanotic congenital heart defects include
A. Co-arctation of the aorta
B. Tricuspid atresia
C. Tetralogy of Fallot
D. Ventricular septal defects
E. Transposition of the great vessels
11. Clinical features of congenital heart disease in infants include:
A. Difficulty with feeding
B. Central cyanosis
C. Recurrent respiratory infections
D. Upper extremity hypertension
E. Poor weight gain
12. In atrial septal defect:
A. Blood flow is usually from left to right
B. Pulmonary blood flow is decreased
C. Heart failure in infancy is common
D. Recurrent pulmonary infection is uncommon
E. Asymptomatic survival to adulthood is not uncommon

2
13. Atrial septal defect:
A. Represents a right to left shunt
B. May close spontaneously
C. Is asymptomatic in the majority of infants and children
D. Shortens life expectancy although survival to adulthood is the rule
E. Is characterized by a soft mid-systolic murmur at the pulmonary area
and
physiological splitting of the second heart sound.
14. Ventricular septal defect:
A. Most commonly occurs in the perimembranous area
B. The conduction tissue is closely related to its margin
C. May close spontaneously
D. Causes a right to left shunt
E. Increases pulmonary blood flow
15. Ventricular septal defect:
A. Closes spontaneously in about 90% of patients usually by 6 months of
age
B. Causes a pansystolic murmur
C. If large, may lead to hypertensive pulmonary vascular disease
D. Is often complicated recurrent pulmonary infections
E. Eisenmenger’s syndrome is an indication for urgent surgical closure
16. Features of ventricular septal defect include:
A. Bi-ventricular enlargement
B. A thrill in the 3rd or 4th right intercostal space
C. A loud holodiastolic murmur in the lower left sternal border
D. Oligemic lung fields on P-A chest films
E. Boot-shaped heart on P-A chest films
17. Contra-indications to surgery in ventricular septal defect include:
A. Age less than 10 years
B. Reversible pulmonary hypertension
C. Zero net shunt across the defect
D. A large defect with retarded growth and recurrent pulmonary infections
E. Eisenmenger’s syndrome
18. Congenital pulmonary valve stenosis
A. Is preferably treated by surgery in the current era
B. Causes reverse splitting of the second heart sound

3
C. Is preferably treated by catheter intervention techniques currently
D. May coexist with annular and infundibular hypoplasia
E. Commonly presents in adulthood.

19. Cyanotic congenital heart disease with increased pulmonary blood


flow
include the following:
A. Truncus arteriosus
B. Transposition of the great arteries
C. Cor triatriatum sinistrum
D. Tricuspid atresia
E. Total anomalous pulmonary venous connection
20. In adults with congenital heart disease:
A. Atrial septal defect is the commonest lesion.
B. As much as 20-30% may be found incidentally.
C. Repair of a aortic coarctation is always curative.
D. Closure of an ASD in the 6th decade is always curative.
E. Primary repair of Fallot’s tetralogy is contraindicated after 30 years of
age
21. Lesions that may be treated using a percutaneous catheter-delivered
intervention include:
A. All atrial septal defects.
B. Perimembranous ventricular septal defects.
C. Patent ductus arteriosus.
D. Tetralogy of Fallot.
E. Atrioventricular (AV canal) defects.
22. Pulmonary stenosis:
A. Imposes a pressure load on the right ventricle
B. Primarily causes right ventricular dilatation
C. Increases pulmonary blood flow
D. May increase right ventricular pressure to suprasystemic levels
E. Is preferably treated with pulmonary valve replacement
23. Congenital aortic stenosis:
A. May be subvalvular, valvular or supra valvular (the commonest)
B. If valvular, results from congenital malformation of the valve leaflets
C. If subvalvular may be in the form of a fibrous shelf
D. If supravalvular occurs at the sino-tubular junction
E. If supravalvular, is associated with William’s syndrome

4
24. In congenital aortic stenosis:
A. Left ventricular ejection time is prolonged
B. Diastolic time is prolonged
C. Coronary arterial filling time is prolonged
D. Neonatal intervention may be required
E. Myocardial ischemia is often evoked

25. Notching on the posterior aspects of the 3rd to 8th ribs on both
sides in a 10-
year old boy is typical of:
A. Congenital aortic stenosis
B. Congenital pulmonary stenosis
C. Arterial hypertension
D. Patent ductus arteriosus
E. Coarctation of the aorta
26. Complications after correction of congenital aortic stenosis include:
A. Residual obstruction
B. Re-stenosis
C. Aortic incompetence
D. Mitral incompetence
E. All of the above
27. Coarctation of the aorta:
A. Is a localized deformity of the aortic adventitia
B. Is more commonly juxta-ductal
C. Ductal closure is contributory to the etiopathogenesis
D. Occurs in the region of the aortic isthmus
E. The left subclavian artery may be used for patch aortoplasty
28. Coarctation of the aorta:
A. Causes a significant pressure gradient in the aortic isthmus
B. May be in the thoracic or abdominal aorta
C. May be associated with aneurysm of the aorta
D. Collaterals cause rib notching
E. Is commonly associated with bicuspid aortic valves and intracranial
aneurysms
29. Symptoms of coarctation of the aorta include:
A. Headaches
B. Epistaxis
C. Upper extremity hypertension

5
D. Abdominal pain after meals
E. Feeble lower limb pulses
30. Useful investigations in suspected coarctation of the aorta include:
A. Cardiac catheterization
B. Aortography
C. E.C.G.
D. Plain chest X-ray
E. Echocardiography

31. Complications of coarctation of the aorta include:


A. Recurrent pulmonary infections
B. Rupture of intracranial aneurysms
C. Rupture of the aorta
D. Aortic regurgitation
E. Acute renal failure
32. The components of Tetralogy of Fallot include:
A. Obstruction to right ventricular outflow
B. Ventricular septal defect
C. Atrial septal defect
D. Overriding aorta
E. Hypertrophy of the right ventricle
33. In Tetralogy of Fallot:
A. The peripheral oxygen saturation level may be as low as 40 %
B. The haematocrit is less than 40%
C. There may be cerebrovascular accident in infancy
D. There may be episodes of loss of consciousness
E. Patients who are cyanotic at birth live to only 2 years
34. The most significant clinical feature of Tetralogy of Fallot is:
A. Dyspnoea on sucking
B. Cyanosis worsening with effort
C. Retarded physical development
D. Haemoptysis
E. Worsening of dyspnoea by squatting
35. In Tetralogy of Fallot:
A. A systolic murmur is heard along the right parasternal border
B. A systolic murmur is loudest over the 2nd and 3rd left intercostal
spaces
C. A diastolic murmur is heard at the apex

6
D. A systolic murmur is heard in the neck
E. The second heart sound is normal
36. The most essential investigation in suspected Tetralogy of Fallot is:
A. Plain chest X-ray
B. E.C.G.
C. Echocardiography
D. Cardiac catheterization and angiocardiography
E. Blood gas studies

37. The Blalock-Taussig shunt involves anastomosis of:


A. Aorta to left pulmonary artery
B. Aorta to right pulmonary artery
C. Right subclavian artery to right pulmonary artery
D. Left subclavian artery to right pulmonary artery
E. Innominate artery to left pulmonary artery
38. Management of hypercyanotic spells in tetralogy of Fallot include:
A. Placement in knee chest position
B. 21% oxygen administration
C. Morphine administration by mouth
D. Intravenous phenylephrine to reduce systemic blood pressure
E. NaHCO3 to treat acidosis
39. In total anomalous pulmonary venous connection:
A. All the pulmonary venous return may be connected to the coronary
sinus
B. Requires urgent correction early in life
C. Presents with pulmonary oedema when there is associated obstruction
D. The cardiac silhouette is enlarged on the P- A chest film
E. The lung fields are plethoric on the P- A chest film
40. Transposition of the great arteries:
A. Is the commonest cyanotic congenital heart lesion
B. Requires a ventricular septal defect, atrial septal defect, or patent
ductus arteriosus to be compatible with postnatal survival
C. Gives rise to parallel systemic and pulmonary circulations
D. Causes central cyanosis despite increased pulmonary blood flow
E. Spontaneous correction without intervention has been reported
41. Procedures used in the treatment of transposition of the great
arteries

7
include:
A. Balloon atrial septostomy
B. Anastomosis between the right pulmonary artery and the ascending
aorta
(Waterston shunt)
C. Atrial septectomy (Blalock-Hanlon operation)
D. Interventricular septectomy
E. Atrial switch operation (Mustard procedure)

42. In tricuspid atresia:


A. The atresia is always complete and in most cases the valve area is
represented by only a depression
B. The right ventricle is hypoplastic
C. Atrial septal defect is necessary for postnatal survival
D. The right atrium is enlarged but the left atrium and left ventricle are
normal in size
E. The physiology is that of a single ventricle
43. In tricuspid atresia:
A. There are exertional anoxic spells consisting of deepening cyanosis,
tachypnoea and loss of consciousness
B. Squatting is not a diagnostic feature
C. Chest X-ray and E.C.G. are distinctive
D. Angiocardiography is the most useful investigation
E. Blalock-Taussig shunt is the definitive operative procedure
44. Rheumatic fever:
A. Causes valve complications in 50% of children who suffer from it
B. Affects the mitral valve most commonly and the tricuspid valve least
C. In West Africa, it is the commonest aetiological factor in acquired
valvular disease
D. May cause stenosis or regurgitation of the valve
E. Is synonymous with rheumatic heart disease
45. Acquired mitral stenosis:
A. Is more common than mitral regurgitation
B. Accounts for about 55% of all cases of rheumatic heart disease
C. Results from scarring and fibrosis of the cusps and fusion along the

8
commissures
D. Is a funnel-shaped orifice which becomes narrower but less deformed
with time
E. Becomes clinically significant about 10 months after the acute valvulitis
46. In acquired mitral stenosis:
A. Symptoms start when the cross-sectional area of the valve is less than
2.5 cm2
B. The left atrium and the right ventricle enlarge
C. There may be functional tricuspid stenosis
D. Paradoxical peripheral embolization may occur
E. Pulmonary hypertension occurs
47. In mitral stenosis the symptoms include:
A. Paroxysmal nocturnal dyspnoea
B. Orthopnoea
C. Dyspnoea on exertion
D. Cough
E. Easy fatiguability

48. Clinical features of acquired mitral stenosis may include:


A. Dysphagia and/or hoarseness of voice
B. Central cyanosis
C. Atrial fibrillation
D. Pleural effusion
E. Apical systolic murmur preceded by an opening snap and increased
first heart sound
49. Post-operative complications of mitral valve replacement include:
A. Bleeding
B. Low cardiac output
C. Cardiac arrhythmias
D. Pulmonary insufficiency
E. Thromboembolic phenomena
50. In the prognosis of acquired mitral stenosis after surgery:
A. Prosthetic valve thrombosis is commoner in the first postoperative year
B. Thromboembolic phenomena always result in symptoms
C. Valve replacement is superior to valve repair in terms of freedom from
reoperation
D. The preoperative left ventricular function is the key determinant of
survival
E. Mechanical valve replacement mandates lifelong anticoagulation
51. In rheumatic mitral regurgitation:
A. The valve is almost always amenable to repair

9
B. Consequent dilatation of the left ventricle worsens the regurgitation
C. Left ventricular function deteriorates with time
D. The pulmonary vascular resistance falls
E. Chronicity tends to induce intractable atrial fibrillation
52. The initial symptoms in rheumatic mitral insufficiency include: []
A. Easy fatigue
B. Orthopnoea
C. Exertional dyspnoea and palpitations
D. Cough
E. Paroxysmal nocturnal dysponea
53. Signs of rheumatic mitral insufficiency include:
A. Pansystolic murmur most prominently heard at the bases and radiating
to the left axilla
B. A diastolic flow murmur with an opening snap at the apex
C. A loud first heart sound
D. A prominent apical thrust of the heart which may be displaced
downwards and
laterally
E. Machinery murmur

54. Symptoms of hyperviscosity in polycythaemic cyanotic patients


include:
A. Headache
B. Faintness and dizziness
C. Blurred vision
D. Amaurosis fugax
E. Increased mentation
55. In the clinical examination of patients:
A. A heart murmur always has pathological significance.
B. Most systolic heart murmurs do not signify cardiac disease.
C. Diastolic murmurs always represent pathological conditions.
D. Most continuous murmurs are of pathological origin.
E. Venous hums are "innocent" continuous murmurs.
56. The mammary soufflé:
A. Is a high-pitched systolic-diastolic murmur.
B. Can be heard over one or both breasts.
C. Is best heard directly over the breast.
D. Has a systolic component that begins after a slight delay following the
first heart sound.
E. Is usually discovered in late pregnancy and during postpartum

10
lactation.
57. The clinical examination reveals features of rheumatic aortic
regurgitation in a
55 year-old man. You would recommend aortic valve replacement
under the
following conditions
A. NYHA Class III or IV symptoms
B. Asymptomatic with ejection fraction at rest of 40%.
C. Asymptomatic with ejection fraction at rest of 65%
D. Intractable symptoms despite adequate medical therapy.
E. The diagnosis alone is sufficient indication for surgery.
58. The following is/are true of arterial blood gases (ABG):
A. ABG reflects lung function.
B. Venous blood gases reflect the adequacy of tissue oxygenation and
tissue carbon dioxide clearance.
C. In a hypoxaemic patient, air contamination of the ABG sample
produces an underestimation of the patients arterial PO2.
D. ABG samples need not be iced if the sample is analyzed within 10
minutes.
E. The metabolic effects on the ABG sample are caused primarily by the
activity of erythrocytes.

59. An obstructed SVC initiates collateral venous return to the heart


from the
upper half of the body through:
A. The azygous venous system
B. The internal mammary venous system
C. The long thoracic venous system with its connections to the femoral
veins.
D. The long thoracic venous system, with its connections to the vertebral
veins.
E. All of the above.
60. Complications of surgical repair of coarctation include:
A. Paraplegia.
B. Rebound paradoxical hypertension in the early post-operative phase

11
C. Recurrent pharyngeal nerve palsy.
D. Phrenic nerve injury with diaphragmatic paralysis.
E. Re-coarctation.
61. Death in untreated aortic coarctation is usually due to:
A. Heart failure.
B. Aortic dissection.
C. Infective endocarditis.
D. Cerebral hemorrhage.
E. All of the above.
62. In Eisenmenger’s syndrome
A. Pregnancy is contraindicated in female patients.
B. During pregnancy, maternal and foetal mortality are similar,
approaching 50%.
C. Flying on commercial airline flights can be safely performed by stable
patients with SPO2 on room air of 85%.
D. Right-sided heart failure and supraventricular arrhythmias are
harbingers of poor prognosis.
E. Intravenous lines carry an increased risk of morbidity or mortality.
63. A "clinically significant" ASD:
A. Causes right heart volume and sometimes pressure overload.
B. May be associated with atrial arrhythmias in those older than 30 years.
C. May permit paradoxical embolism resulting in TIA/stroke.
D. May lead to paroxysmal nocturnal dyspnea.
E. Has a pulmonary-to-systemic flow ratio of 1.

64. Construction of an aorto-pulmonary shunt is preferable to primary


repair
under the following conditions:
A. Neonatal tetralogy of Fallot with unfavorable anatomy.
B. Pulmonary atresia with VSD.
C. Tricuspid atresia with reduced pulmonary blood flow.
D. Single ventricle anatomy with increased pulmonary blood flow.
E. Pulmonary stenosis

12
65. Indications for surgery in mitral regurgitation include:
A. Absence of symptoms
B. Minimal to moderate symptoms
C. Atrial fibrillation
D. Progressive symptoms with left ventricular decompensation
E. Severe left ventricular enlargement and failure
66. Acquired aortic stenosis:
A. Is more common in males and syphilis is probably the commonest
cause
B. Becomes symptomatic when the normal cross-sectional area of the
valve is reduced to 1.0cm2
C. Leads to increase in coronary blood flow to cope with the LV
hypertrophy
D. Has no effect on the right heart or pulmonary vessels
E. Produces early symptoms in most patients
67. Early features of acquired aortic stenosis include:
A. Sudden death
B. Pulmonary oedema
C. Exertional syncope, dizziness or dyspnoea
D. Left ventricular failure
E. Angina pectoris
68. Signs of acquired aortic stenosis include:
A. Arterial hypertension
B. Diastolic thrill in the aortic area
C. Crescendo-decrescendo murmur in the 2nd intercostal space along the
right sternal border
D. Radiation of murmur to the suprasternal notch and the carotids
E. Slow-rising pulse

69. Indications for surgery in acquired aortic stenosis include:


A. Angina pectoris
B. Syncope
C. Congestive cardiac failure
D. Cardiac enlargement

13
E. Dyspnoea of any degree
70. Aortic regurgitation:
A. May result from type A aortic dissection
B. May remain asymptomatic for many years
C. May initially give rise to throbbing in the head and neck
D. Gives a wide pulse pressure due to the very high systolic pressure
E. Is characterized by a blowing aortic systolic murmur
71. A 45-year old man suffering from fatigue and dyspnoea for 3 months
is found
to have distended neck veins which increase on inspiration, pulsus
paradoxus, hepatomegaly, and ascites out of proportion to pedal
swelling.

The most likely clinical diagnosis is:


A. Right ventricular failure
B. Mitral stenosis
C. Aortic insufficiency
D. Mitral insufficiency
E. Chronic constrictive pericarditis
72. A plain X-ray of the chest may show:
A. An enlarged left ventricle
B. An enlarged heart
C. A normal cardiac shadow
D. Calcified pericardium
E. Calcified mitral or aortic valve
73. Treatment will include:
A. Digitalization
B. Bedrest
C. Diuretics
D. Pericardiotomy
E. Mitral or aortic valve replacement

74. A boot-shaped heart on plain chest X-ray in an infant is typical of:

14
A. Congenital aortic stenosis
B. Atrial septal defect
C. Ventricular septal defect
D. Tetralogy of Fallot
E. Patent ductus arteriosus
75. Recurrent Stokes-Adams attack may be associated with:
A. Severe bradycardia
B. Exertional dyspnoea and dizziness
C. Syncope
D. Angina pectoris
E. Convulsions
76. Heart block may be caused by:
A. Coronary artery disease
B. Aortic valve replacement
C. Perimembranous VSD closure in children
D. Digoxin toxicity
E. Conduction system degenerative disease
77. In complete heart block:
A. The stroke volume is increased
B. Physical activity is accompanied by tachycardia
C. The heart rate is 45 or less
D. The E.C.G. shows infrequent QRS waves associated with P waves
E. Atropine is often used for symptomatic relief
78. Concerning myocardial hypertrophy:
A. The pattern of hypertrophy reflects the nature of the stimulus.
B. Pressure-overloaded ventricles develop concentric hypertrophy
C. Volume-overloaded ventricles develop hypertrophy with chamber
dilation
D. Pressure-overload hypertrophy is accomplished predominantly by
augmentation of cell width via parallel addition of sarcomeres
E. Volume overload and/or dilation stimulate augmentation of both cell
width and length via both parallel and series addition of sarcomeres

79. The following pathology may cause coronary artery obstruction:

15
A. Atheroma
B. Fibromuscular dysplasia
C. Dissection
D. Spasm
E. Embolism
80. The physiology of the coronary circulation can be described thus:
A. In the absence of significant pathology, coronary arterial flow provides
adequate myocardial perfusion at rest
B. Compensatory vasodilation provides flow reserve that is more than
sufficient to accommodate the increased metabolic demands during
vigorous exertion.
C. When the luminal cross-sectional area is decreased by 50% or more,
coronary blood flow generally becomes limited with exertion
D. With 60% or greater reduction, coronary flow may be inadequate even
at rest
E. Coronary artherosclerosis is not the commonest cause of coronary
obstruction
81. The following are true concerning bicuspid aortic valves (BAV):
A. BAV is the most frequent congenital cardiovascular malformation in
humans.
B. Men are affected three to four times more frequently than are women.
C. BAV are predisposed to accelerated calcification, with about 85%
becoming stenotic
D. About 15% of the time, they become purely incompetent, complicated
by infective endocarditis, or associated with acute aortic dissection.
E. BAV underlie over two-thirds of aortic stenosis in children and 50% of
adults
82. The pulse generator of an artificial pacing system:
A. Can be programmed to fire at a fixed rate completely uninfluenced by
the heart
B. Can be programmed to function only during periods of complete heart
block
C. May be complicated by loss of capture, infection or lead migration
D. Has a life span of 30-40 years when implanted in adults
E. May be used for temporary or permanent pacing
83. Complications of extra-corporeal circulation include:
A. Haemolysis, anaemia and thrombocytopaenia
B. Excessive post-operative bleeding
C. Platelet dysfunction
D. Air embolism
E. Dissection of the aorta or femoral artery

16
84. In the examination of a 10-year old boy a systolic murmur is heard in
the left
sternal border with maximal intensity in the 3rd interspace; the
pulmonary second sound is widely split.
The most likely diagnosis is:
A. Congenital aortic stenosis
B. Atrial septal defect
C. Ventricular septal defect
D. Patent ductus arteriosus
E. Congenital pulmonary stenosis
85. Myxomas
A. Are the most common primary tumor of the heart in adults.
B. Typically arise in the right atrium along the interatrial septum.
C. Sporadic cases of myxoma are almost always single.
D. Are benign but exhibit malignant behaviour.
E. Medical treatment is superior to surgery in children.
86. Risk factors for acute renal failure after cardiac surgery include:
A. Young age.
B. Baseline renal dysfunction.
C. Left ventricular dysfunction.
D. Peripheral vascular disease.
E. Clinical signs of poor cardiac function
87. In acute aortic dissection:
A. Presentation within the first 2 days is termed acute.
B. The term chronic is reserved for those patients presenting at greater
than 2
days following the initial event.
C. Stanford type A dissection include dissections that involve the
ascending aorta,
arch, and descending thoracic aorta.
D. De Bakey type I involves the ascending aorta.
E. Stanford type B dissection include dissections that originate in the
descending
thoracic and thoracoabdominal aorta

17
.
88. The risk of rupture presented by an enlarging aortic aneurysm may
be
described by the following:
A. Wall tension decreases as the radius of an aneurysm increases.
B. Larger aneurysms have a higher rate of expansion.
C. There is a 40-fold increased risk of rupture or dissection in an
aneurysm 6.0-6.9
cm in diameter compared to an aneurysm 4.0-4.9 cm in diameter.
D. Patients with Marfan syndrome have accelerated aneurysm growth and
tend to
rupture or dissect at smaller sizes.
E. Dissections are associated with an accelerated rate of expansion and
rupture.
89. The aetiology of AV block includes:
A. Ischemic heart disease.
B. Bacterial endocarditis.
C. Cardiomyopathy.
D. Iatrogenic.
E. Systemic lupus erythematosus.
90. In the normal heart, stroke volume is augmented 5-15% by AV
synchrony
versus the asynchronous state. The quantitative importance of AV
synchrony
increases in:
A. Left ventricular hypertrophy
B. Diastolic dysfunction
C. Heart failure
D. Dilated cardiomyopathy
E. Mitral stenosis
91. The mitral valve:

18
A. Has two leaflets, the aortic leaflet and the mural leaflet
B. The anterior leaflet is also known as the aortic leaflet
C. The mural leaflet annulus is intimately related to the circumflex
coronary artery
D. Is the most susceptible to rheumatic valvulitis
E. Is so named because of its resemblance to the bishop’s headdress
92. The aortic arch laterality (sidedness) is determined by:
A. The first branch
B. Which side of the body the aorta descends en route to the abdomen
C. The direction of the arch in the superior mediastinum
D. The bronchus over which it arches
E. The side of the ascending aorta

93. The coronary arteries:


A. Always arise from the aortic sinuses of Valsalva
B. Are perfused in both phases of the cardiac cycle
C. The left coronary is perfused only in diastole
D. The right coronary artery is perfused only in systole
E. Tachycardia increases perfusion of the left coronary artery
94. The following features distinguish the atrioventricular valves:
A. The tricuspid valve has three leaflets in the normal heart
B. The mitral valve in the normal heart has two leaflets supported by
septal and free wall papillary muscles
C. The tricuspid valve guards the entrance to the coarsely trabeculated
ventricle
D. The mitral valve guards the entrance to the finely trabeculated ventricle
E. The two atrioventricular valves are juxtaposed at the same level of the
crux of the
heart
95. The right atrium:
A. Is always right-sided
B. Is identified by its position to the right of the midline
C. Bears an appendage that is tubular in morphology
D. In the normal heart receives three discrete sources of venous drainage
E. Is always connected to the right ventricle
96. Modifiable risk factors for coronary artery disease include:
A. Systemic hypertension
B. Diabetes insipidus

19
C. Hypocholesterolemia
D. Sedentary lifestyle
E. Smoking
97. Causes of aortic regurgitation include:
A. Aortic valve endocarditis
B. Ascending aortic aneurysm
C. Type A aortic dissection
D. William’s syndrome
E. Rheumatic heart disease
98. Aortic stenosis:
A. May result from syphilitic aortitis
B. Can cause sudden death
C. Is characterized by the angina pectoris without coronary artheroma
D. Becomes significant only when the transvalvar gradient reaches
100mmHg
E. Causes syncope

99. Concerning cardiac tumors:


A. Three-quarters of primary tumors are benign
B. 50% of benign tumors in adults are myxomas
C. 75% of the malignant tumors are sarcomas
D. 15% of benign tumors in children are myxomas
E. Approximately 75% of myxomas occur in the left atrium
100. The atrioventricular node:
A. Is located in the floor of the left atrium
B. Is contained entirely in its atrial extent within the triangle of Koch
C. Can be seen through the endocardium of the triangle of Koch at
surgery
D. Koch’s triangle must be avoided to prevent surgical damage to
atrioventricular
conduction
E. The atrioventricular bundle (of His) penetrates directly at the apex of
the
triangle of Koch

20
CHAPTER 26
THE HEART
ANSWER

1. A, C, D 36. C 71. E
2. B, E 37. C 72. D
3. B, D 38. A, E 73. A, B,
C
4. A, C 39. A, B, C, E 74. D
5. D 40. B, C, D 75. A, B, C, D, E
6. B 41. A, C, E 76. A, B, C, D, E
7. C, D 42. A, B, C, D, E 77. A, C
8. D 43. B 78. A, B, C, D,
E
9. A, C, E 44. B, C, D 79. A, B, C, D, E
10. B, C, E 45. C 80. A, B,
11. A, B, C, D, E 46. A, B, E 81. A, B, C, D, E
12. A, E 47. A, B, C, D, E 82. A, B, C, E
13. B, C, D 48. A. C. D 83. A, B, C, D, E

21
14. A, B, C, E 49. A, B, C, D, E 84. B
15. B, C, D 50. A, C, D, E 85. A, C
16. A 51. B, C, E 86. B, C, D, E
17. C, E 52. A, B, C, D, E 87. A, C, D, E
18. B, C, D, E 53. D 88. B, D, E
19. A, B, D, E 54.A, B, C, D 89. A, B, D, E
20. A, B 55. A, B, C, D, E 90. A, B, C, D, E
21. B, C 56. A, B, C, D, E 91. A, B, C, D, E
22. A, D 57. A, B, D 92. D
23. B, C, D. E 58. A, B, D 93. C
24. A, D, E 59. A, B, C, D, E 94. A, C, D
25. E 60. A, B, D, E 95. D
26. A, B, C, D, E 61. A, B, C, D, E 96. A, D, E
27. B, C, D, E 62. A, B, C, D, E 97. A, B, C, E
28. A, B, C, D, E 63. A, B, C, D 98. B, C, E
29. A, B, D 64. A, B, C 99. A, B, C, D, E
30. A, B, C, D, E 65. D, E 100. B, C, D, E
31. B, C, E 66. B
32. A, B, D, E 67. C
33. A, C, D 68. C, D, E
34. B 69. A, B, C, D
35. B 70.A, B, C

22
CHAPTER 50
VASCULAR AND LYMPHATIC SYSTEMS

1. The intima of an artery is:

        A. Thick
B. Relatively acellular
C. Non-wettable
D. Nourished via the vasa vasorum
E. Supplied by nerves
2. The media of an artery is:
 Composed of fibro-elastic tissue and smooth muscle
 Relatively thin but responsible for the strength of the arterial wall
 In big arteries composed predominantly of muscle fibres
 Nourished by the vasa vasorum
 Separated from the adventitia by the internal elastic lamina
3. The veins:
 Have the same capacity as the arteries
 Have a wall consisting of muscle and fibro-elastic tissue
 With the exception of the abdominal ones have valves which allow the
blood to flow only towards the heart
 In the body cavities allow blood to flow as a result of the negative
pressure in the chest during respiration
 Have sympathetic nerve supply
4. Where facilities exist, the investigations that MUST be done in every
patient
with peripheral arterial disease include:

 X-ray of chest and limb


 Arteriography
 E.C.G
 Ultrasonography
 Ankle systolic pressure and ankle brachial pressure index
5. The commonest cause of peripheral arterial disease is:
 Thrombo-angiitis obliterans
 Raynaud’s disease
 Atherosclerosis
 Aneurysm

1
 Embolism

6. In the ankle systolic pressure and ankle brachial pressure index:


 The systolic pressure of the posterior tibial artery and brachial artery
are measured
 A special electronic sphygmomanometer and the pencil probe
Doppler
velocimeter are used
 The ankle systolic pressure is normally 20-30 mmHg higher than the
brachial
systolic pressure
 The ankle BP/arm BP is greater than one
 The result is reproducible
7. In the ankle systolic pressure and ankle brachial pressure index
test:
 The ankle BP is elevated in peripheral arterial disease
 The ankle BP/arm BP is elevated in peripheral arterial disease
 Peripheral arterial disease is indicated if the ankle BP fails to rise after
exercise for one minute on a bicycle ergometer at 500 Kpm/min
 The tests are not useful in assessing progress to treatment
8. Complications of translumbar aortic arteriography include:
 Allergy
 Injury to the spinal cord
 Cerebrovascular accident
 Damage to the kidney
 Spasm of the superior mesenteric artery
9. Atherosclerosis:
 Affects the media primarily and the intima also
 Is equally common in the sexes
 In the lower limb occurs mainly from the infra-renal aorta to the
division
of the popliteal
 In the upper limb affects the subclavian, axillary, brachial and radial
 Is commoner in medium-sized arteries than ones at sites of origin of
branches

2
10. In atherosclerosis:
 Lipids are deposited in the deeper layers of the intima
to form plaques
 Plaques may become calcified or fibrosed
 There is ulceration of the endothelium
 The internal elastic lamina is thickened
 The muscle of the media is replaced by fibrous tissue

11. Atherosclerosis of a part of an artery may result in:


 Distension
 Aneurysm
 Thrombosis
 Narrowing of collaterals and branches above and below the
affected part
 Tortuosity
12. Atherosclerosis is related aetiologically to:
 High intake of unsaturated fat
 Cigarette-smoking
 Hypercholesterolaemia
 Alcoholism
 Hypertension
13. In diabetics, atherosclerosis:
 Develops at an earlier age
 Is commoner in men
 Affects the popliteal, tibial and associated small arteries commonly
 Has the same incidence of gangrene as is non-diabetics
 Leads to ulceration more commonly than in non-diabetics
14. Thrombo-angiitis obliterans:
 Is common
 Affects the large and medium arteries of the limbs
 Is often associated with migratory phlebitis of the superficial or deep
veins
 Occurs almost exclusively in middle-aged cigarette-smoking males
between
40-and 55

3
 Is probably an allergic reaction to nico-tine and regresses if smoking
is stopped
15. In thrombo-angiitis obliterans:
A. There is initial dense round cell infiltration of the adventitia
and media
B. There is endothelial proliferation of the intima
C. The lumen is occluded by a thrombhus which is later replaced by
firm granulation tissue
D. The lumen is never canalized
E. The artery or vein becomes enclosed in fibrous tissue

16. Arterial occlusion in the lower limb:


 May be multiple and is bilateral in about 70% of patients
 If in the popliteal causes a more severe ischaemia than in the
superficial
femoral
 If in the aorta or iliac causes more reduction in blood flow than in the
femoral
 Is more often below the level of the knee joint
 Is seen most often in patients between 50 and 65
17. Intermittent claudication is:
 Deep-seated cramplike pain of the calf muscles brought on by
walking
and relieved by rest
 Caused by hypoxia of pain fibres
 Felt in the thigh and buttock as well if the superficial femoral artery is
occluded
 Not felt in the peroneal, extensor or short muscles of the foot even
when the popliteal is affected
 The main symptom of lower limb arterial occlusion
18. Rest pain is continuing burning pain in the foot when the:
 Patient stops walking
 Foot is in the horizontal position
 Foot is elevated
 Foot is in the dependent position

4
 Patient is asleep
19. Signs of incipient gangrene of the foot include:
 Coldness and numbness
 Ulceration
 Brittle nails and thin, shiny, hairless skin
 Cyanosis of the toes
 Absence of popliteal or dorsalis pedis pulse

20. In the examination of a patient with suspected peripheral arterial


occlusion:

A. The lower the level at which pallor of the sole appears when the leg is
elevated, the more severe the occlusion
B. Red discoloration of the sole when the leg is hanging suggests that
the ischaemia is not severe
C. A bruit at a particular site is due to dilatation of the artery above
the site of obstruction
 Absence of the femoral pulse in both limbs is always indicative of
occlusion of the terminal aorta
 Muscles of the affected limb are not wasted
21 In patients with chronic arterial occlusion of the lower limb:
 About 40% improve spontaneously
 About 55% remain unchanged
 Only 5% require operation
 Amputation is required in about 90% of those requiring operation
 About half die within 5 years usually from C.V.A. or coronary
Thrombosis
22. In the medical management of chronic arterial occlusion of the
lower limb:

 Vasodilator drugs have been found effective

5
 Smoking should be reduced
 Fat intake should be low and the fat can be polysaturated
 Leg exercises may increase the claudication distance by 8% and
improve
the physical and psychological state
 Walking should be slow
23. Indications for operative treatment in chronic arterial occlusion of
the lower limb include:

 Intermittent claudication
 Rest pain
 Inability to walk more than 150-200 m
 Pregangrene
 Absent popliteal pulse

24. Substances currently used for by-pass grafting include:

A. Dacron
B. Autogenous artery
C. Autogenous saphenous vein
D. Glutaraldehyde stabilized umbilical vein
E. Nylon
25. Synthetic graft:

A.Is used for large vessels such as aorta


B.Is normally not used at or below the groin
C.Has blood clotting in its interstices and around it
D.Has fibrous tissue replacing the blood clot and the formation
of a pseudo-endothelium in the lumen
E. Has a patency rate at 5 years of about 90%
26 Ischaemia is much less common in the upper limb than in
the lower because:

 Arterial disease is uncommon in the arteries of the upper limb


 Of extensive collateral circulation in the arm and forearm
 The arteries are relatively wider
 The palmer arch of the hand provides vascular communication
between the radial and ulnar arteries
 Blocked forearm artery dilates to allow the flow of some blood

6
27. A cervical rib:

A. May articulate with the first rib on manubrium


B. Compresses the first and second dorsal roots of the brachial plexus
. and subclavian artery
C. Is often symptomless
D. May cause pain in the forearm precipitated by work and relieved by
rest,
ulceration of the fingers and absent or feeble radial pulse
E. Should always be excised
28. In Raynaud’s phenomenon:

A. Exposure to severe cold causes blanching or cyanosis


of the fingers
B. The blanching or cyanosis is due to sympathetic over-stimulation
C. Middle-aged women are more commonly affected
D. Gangrene may occur
E. Sympathectomy is required in most patients

29. Origins of systemic embolism include:

A. Right atrium
B. Infarcted myocardium
C. Aneurysm
D. Deep venous thrombosis
E. Subacute bacterial endocarditis
30. Common sites where a systemic embolus may lodge include:

A. Posterior cerebral artery


B. Aortic bifurcation
C. Bifurcation of the common iliac
D. Adductor canal
E. Hepatic artery
31. Clinical features of an embolus at the bifurcation of the popliteal
artery include:

 Sudden severe pain in the leg with associated paralysis


 Numbness and coldness of the leg
 Pallor of the leg
 Loss of sensation of the toes and forefoot
 Absent popliteal, posterior tibial and dorsalis pulses
32. In dry gangrene the affected part:

7
A. Is swollen
B. Is infected
C. Is black
D. Has shriveled skin
E. Develops a red line of demarcation between the living and
dead tissue in a few days
33. In moist gangrene the affected part:

A. Is macerated
B. Always develops a red line of demarcation between the living and
and dead parts in a few days
C. Is green or black
D. Is not usually infected
E. Is not swollen

34. Causes of dry gangrene include:

A. Injury to the main artery


B. Thrombo-angiitis obliterans
C. Extensive thrombosis of the veins of a part
D. Obstruction of both artery and vein
E. Sudden occlusion of the main artery
35. An aneurysm is:

A. A blood-filled sac which communicates with a vessel


B. True, if it is a localized dilatation of an artery
C. False, if the dilatation affects only a part of the wall
D. Dissecting if the dilatation is caused by the formation of a false
channel between the media and adventitia
E. Most commonly caused by syphilis
36. The commonest site for an aneurysm is the:

A. Aortic arch
B. Popliteal
C. Thoracic aorta
D. Femoral
E. Terminal abdominal aorta
37. Clinical features of an aneurysm include:

A. A fusiform or saccular swelling in the line of an artery

8
B. Marked pulsation
C. A thrill and systolic bruit over it
D. Reduction in the size and pulsation of the swelling when the artery
is compressed distal to the swelling
E. A smaller pulse distal to the swelling than on the contralateral side
38. In aneurysm of the abdominal aorta:

A. The vessel is at least 6 cm in diameter


B. Up to 80% of patients may die in the first year of diagnosis if
treatment
is not given
C. The swelling starts 3-5 cm above the renal arteries
D. The swelling extends to the common iliacs in about 50% of patients
E. The swelling extends anteriorly and posteriorly as it enlarges

39. An aneurysm of the abdominal aorta presents most commonly


with:
A. Abdominal and back pain
B. Pulsatile abdominal swelling
C. Impotence
D. Rupture heralded by abdominal or back pain
E. Intermittent claudication
40. The most useful of the following investigations for abdominal
aorta is:

 Plain abdominal X-rays


 Ultrasonography
 Aortograpy
 Intravenous pyelogram
 CAT-scanning
41. Conditions that may be associated with an abdominal aortic
aneurysm include:

A. Hypertension
B. Peptic ulcer
C. Mitral incompetence
D. Hydronephrosis
E. Ischaemic heart disease
42. In the treatment of abdominal aortic aneurysm:

9
A. Excision of the aneurysm and replacement with a dacron graft is
preferred to an inlay tube graft
B. Aggressive fluid and electrolyte therapy including infusion of
mannitol is essential
C. Ischaemia of one or both limbs may occur post-operatively
D. The mortality in elective cases is only about 2%
E. About 50% of patients surviving operation are alive at 5 years
43. Renal artery stenosis:

A. Is most commonly caused by fibromuscular dysplasia


B. Should always be suspected in a child or young adult with severe
hypertension without renal disease
C. Should be suspected in an atherosclerotic patient with hypertension
that is difficult to control
D. Is in every case associated with an epigastric bruit and accounts for
1-2% of hypertension
E. Is the most curable cause of hypertension

44. In the investigation of renal artery stenosis:

A. I.V.P. shows delayed excretion but more concentration of the


affected side because of reduced G.F.R.
B. Renal scan shows reduced vascularity and normal shape on the
affected side
C. Aortograpgy demonstrates the degree and extent of renal stenosis
D. Split renal function test shows depression of volume and concentration
and osmolality on the affected side
E. Differential rennin estimation of the blood is not helpful
45. In the treatment of renal artery stenosis:

A. Endarterectomy, bypass grafting or reimplantation of the renal


artery into the aorta should always be attempted even when
aortography demonstrates damage to the kidney
B. Nephrectomy should only be done if endarterectomy, bypass
grafting or reimplantation of the renal artery fails
C. Normotension is restored in 50% of patients after endarterectomy,
bypass grafting or re-implanatation of the renal artery
D. Fifty per cent do not show any improvement after endarterectomy, etc
E. Nephrectomy of the diseased affected kidney cures about 70%
46. Adequate venous circulation in the lower limb depends on the:

A. Competence of the valves of the veins especially those in the

10
superficial veins
B. Integrity of the osseo-fibrous compartment of the limb
C. Contraction of the veins
D. Contraction and power of the muscle bulk
E. Negative pressure in the abdominal cavity
47 The long saphenous vein:

A. Is formed by union of veins on the media aspect of the foot with


the dorsal venous arch
B. Passes behind the medial malleolus
C. At the medial aspect of the knee in just in front of the femoral condyle
D. At the saphenous opening passes backwards to join the femoral vein
E. Has no communication with the short saphenous

48. Varcose veins of the leg are:

A. Dilatation, elongation and tortuosity of the superficial and deep veins


of the leg
B. Rare in developing countries
C. More common in women
D. Caused primarily by ageing and obesity
E. Not always associated with reversal of blood flow from the deep
to the superficial veins
49. Symptoms of varicose veins of the leg include:

A. Increased fatigue on standing


B. Aching or burning sensation or leg pain on standing for brief periods
C. Severe aching pain or cramp in the leg brought on by walking and
relieved by rest
D. Marked persistent oedema of the ankle not relieved by elevating the
legs
E. Ache around the knee which is worse on waking up in the morning
50. A palpable cough thrill impulse at a point 3.7 cm below and lateral
to the pubic
tubercle in a patient with varicose veins suggests an:

 Associated femoral hernia


 Arterio-venous fistula
 Associated femoral aneurysm
 Associated saphena varix

11
 Incompetent sapheno-femoral valve
51. Skin changes in varicose veins include:

A. Scaling and weeping eczematous dermatitis


B. Atrophy
C. Pigmentation
D. Fibrosis
E. Deep fissures of the skin including the sole
52. The following may be found in a woman with varicose veins of a leg:

A. Varicose vulval veins


B. Dilated tortunous veins around the umbilicus
C. Fibroid uterus
D. Dilated superficial inferior epigastric vein
E. Hepatomegaly

53. A 40-year old woman has varicose veins of the right leg. When the
leg
is elevated with her in the recumbent position, the veins do not
collapse.
The cause of the varicosity may be:
 Competent of the valves of the saphenous veins
 Deep venous thrombosis
 Incompetence of the sapheno-femoral valve
 A pelvic tumour
 Obstruction of the femoral or external iliac vein
54. If in the Brodie-Trendelenberg test, the varicose veins fill up from
below
immediately the patient stands up and before the tourniquet is
removed,
then the:

 Sapheno-femoral valve is incompetent


 Valves of the communicating perforators are incompetent
 Valves of the saphenous system are necessarily incompetent
 Valves of the deep veins are necessarily incompetent

12
 There must be obstruction in the deep veins
55. If in the Brodie-Trendelenberg test, the varicose veins do not fill up
when the
patient stands up from above when the tourniquet is released then
the:

 Sapheno-femoral valve is incompetent


 Valves of the saphenous system are necessarily incompetent
 Valves of the communicating perforators are incompetent
 There may be obstruction of the external iliac or femoral vein
 Valves of the deep veins are incompetent
56. Which of these statements is true regarding the circulation of
blood in the
venous system of the lower limb?

 An upward movement in all the superficial veins


 A downward movement in the superficial veins
 A downward movement in the deep veins
 An upward movement in the deep veins
 None of the above
57. The dynamic system that maintains the venous circulation of the
lower
limb is dependent on:

 The integrity of the smooth muscle coat of the deep veins


 The union between the superficial and deep fascia
 Gravitational forces generated within the limb
 Integrity of the osseo-fibrous compartment of the limb
 Systolic pressure in the arterial input into the limb

58. The factors that maintain adequate venous circulation in the lower
limb
include:

 Muscle power in the limb


 Integrity of the osseo-fibrous compartment of the limb
 Competence of the valves guarding the communications between
deep
and superficial veins
 The smooth muscle coat of the deep veins

13
 Intra thoracic negative pressure maintenance
59. Varicose veins represent dilatation, elongation and tortuosity of
which
component of the lower limb venous system principally?

 The main trunks of the saphenous systems


 The main trunks of the deep femoral system
 The commicating system of veins
 The immediate tributaries of the superficial venous system
 The radicals of the long and short saphenous systems
60. The factors currently accepted as principally predisposing to the
development of varicose veins include:

 Vascular injuries from Road Traffic Accidents


 Repeated intimal inflammation – thrombophlebitis
 Hyperdynamic venous circulation from strenuous exercise
 Development of the compartment syndrome
 Familial defective, structural strength of the venous wall
 Increased hydrostatic pressure that follows prolonged standing
 Organisation of minute moral platelet thrombi in proxity to the valves
of veins
 Increasing age and obesity
61. A woman of 50 complains of aching pains and burning sensation
and
heaviness of the lower limbs especially towards the end of the day.
These pains are relieved when she puts her legs up at the end of
the day.
Examination shows no defects in the main pulses (femoral,
popliteal,
posterior tibial and dorsalis pedis).

The most likely diagnosis is:

A. Early peripheral arterial disease - atherosclerosis


B. Thromboangiitis obliterans (Buerger’s Disease)
C. Osteoarthritis of the hip and knee

14
D. Uncomplicated (Early) varicose veins
E. Peripheral neuritis
F. Disc lesion – L4/5
G. Peripheral arterial embolism
62. How should this lady’s symptoms be best investigated:

A. Do the Brodie-Trendelenberg Test


B. Ankle/Brachial Pressure Index estimation
C. Doppler ultrasound examination of the legs
D. Venography of the legs
E. Plethysmography of the leg
63. A man of 55 complains of intermittent swelling of the leg up to the
knee
darkening and scaling of the skin around the ankles over the past
three
years. He does not experience intermittent claudication and he has
no
pain in the feet at rest. Examination reveals evidence of varicose
veins
with incompetent perforators on the medial aspect of the knee.
The
peripheral pulses are all present and symmetrical.

The most like diagnosis is:


A. Uncomplicated varicose veins of the (Rt.) leg)
B. Lymphoedema of the leg
C. Deep venous thrombosis of the (Rt.) leg
D. Eczematous dermatitis from varicose veins
E. Recurrent subactute cellulitis of the leg
64. The most useful investigation for the management of this patient
is:
A. Ankle Brachial Pressure Index (ABPI)
B. Perthes Test
C. Duplex Ultrasound scan of the leg
D. Venogram of the leg
E. Retrograde arteriogram of the leg

15
65. A 40 year old diabetic man presents with recurrent itchy,
eczematous lesion
of his Rt. leg in the garter area, of nine months duration. He has
recently
experienced paraesthsia with recurrent breakdown of the skin. On
examination there is an ulcer just above the medial molleolus, with
shallow
sloping edges, firm, fibrotic base, depicting exuberant
granulations. There
are no varicose veins evident but there is a positive sapheno
femoral reflux
with a saphena varix.

The most likely diagnosis is:


A. Ischaemic leg ulcer
B. Diabetic ulcer
C. Venous ulcer
D. Epithelioma of the ankle area
E. Sickle cell ulcer
66. The most useful investigation for this ulcer is:

 Fasting blood sugar


 Ankle Brachial Presssure Index (ABPI)
 Duplex ultrasound examination of the limb
 Hb electrophoresis
 Wound swab for culture
67. The first line treatment for this patient is:

 Bed rest and careful local attention to heal the ulcer


 Excision and grafting of the ulcer
 Injection sclerotherapy for the superficial varicose veins
 Operative treatment by stripping of the long saphenous system
 Operative management-Cocket’s procedure
68. The typical varicose ulcer:

A. Is sited over or below the medial malleolus


B. Has a firm fibrotic base
C. Is very deep with sloping edges
D. Has exuberant granulation tissue in the floor
E. Does not become malignant

16
69. Bleeding from a varicose vein is:

A. Usually spontaneous but may be due to minor trauma


B. Usually associated with pregnancy
C. Often slight
D. Readily controlled by lowering the leg and applying local pressure
On the bleeding point
E. An indication for operative treatment of the varicose veins
70. Thrombophlebitis of varicose veins:

A. Is liable to occur in pregnancy


B. May be associated with oral contraceptive therapy
C. Often spreads to the deep veins
D. May lead to acute ulceration
E. Is usually treated by ligation to prevent spread
71. Indications for sclerotherapy in varicose veins include:

A. Pregnancy
B. Thrombo-phlebitis
C. Small varicose veins with competent valves of the communicating
perforators
D. Small varices following definitive surgery
E. Varicose veins with associated ulcer
72. The sclerosants currently in use for sclerotherapy of varicose vein
include:

A. Sodium morrhuate
B. Hydrocortisone in almond oil
C. Phenol in almond oil
D. Myodil
E. Sodium tetradecyl sulphate
73. The operative treatment of varicose veins:

A. Is ligation and transection of the long saphenous vein at the


sapheno-femoral junction and its removal with or without ligation
of the 6 or more tributaries joining its femoral part
B. Is undertaken even in the presence of complications such

17
as ulceration
C. Should in most cases be combined with Cockett’s operation
D. Has a mortality of about 1%
E. Has an average recurrence rate of less than 10%

74. What is the immediate treatment for the patient with varicose veins
complicated by acute bleeding from the region of the ankle?

A. Injection sclerotheraping for the varicose veins


B. Trendelenberg procedure to extract the veins
C. Local compression dressing to arrest the bleeding, investigation
and injection of the varices
D. Ligation of the bleeding varices
E. Bed rest with elevation of the limb
75. The contra indications to sclerotherapy for early varicose veins
include:
A. Atherosclorosis
B. Thrombophlebitis
C. Local infections
D. Local Allergies
E. Pregnancy
76. Currently the sclerosant medium of choice for sclerotherapy for
varicose
veins is:

Phenol in arachis oil


Phenol in almond oil
Sodium tetradecyl sulphate
Ethanolamine
Hypertonic saline
77. The principle underlying the modern technique of injection
sclerotherapy
for varicose veins of the lower limbs aims at:

 Inducing thrombosis of the varices


 Inducing intimal adhesions of the varices with minimal thrombosis
 Encouraging both initimal adhesions and thrombosis
 Perivascular fibrosis
 Dissolution of the varices

18
78. The indications for operative management of varicose veins include:

 Major varicosities
 Incompetent perforators
 First bleeding episode per se
 Saphenofemoral incompetence per se
 First episode of ulceration
 Complication with superficial thrombophlebitis

79. The attendant complication(s) of varicose veins that must first be


dealt with
prior to surgical intervention include:

 Eczematous dermatitis
 Ulcers – including grafting
 Thrombophlebitis
 Bleeding from varices
 Chronic lymphoedema
80. The Cockets procedure (subfascial ligation of incompetent
perforators) is
indicated in all patients with:

A. Incompetent perforators
B. Recurrent varicose veins
C. Recurrent ankle ulceration and incompetent deep perforators at the
ankle
D. Incompetent perforators at the ankle
E. Varicose veins and chronic oedema
81. What instructions should be given to patients who undergo the
Trendelenberg operation for stripping of varicose veins.

A. Should be confined to bed for a week to avoid bleeding


B. Should be made to walk within a few hours to a void DVT
C. Should have leg elevated for the first 24h then made to walk as much
as possible
D. Should avoid walking for a week
E. Should prefer standing to walking
82. The operation of endovenous lasser ablation (EVLA) for varicose

19
veins
has been shown by recent Rendomised controlled Trials:

 To be as effective as the standard Trendelenberg operation


 To give better results than the standard surgical interventions
 To be less effective than the standard Trendelenberg operation
 Not to ensure early return to work
 To have a higher recanalization and neovascularization than the
Trendelenberg operation

83. The lymphatic:


A. System drains 2-4 L of excess tissue fluid containing 75-200 g of
protein from all the organs back to the circulation daily
B. Capillaries form intercommunicating endothelial channels which
are freely permeable to macromolecules such as proteins and
bacteria
C. Vessels all have smooth muscle in their walls and valves which
allow lymph to flow centrally towards the central veins
D. Nodes are involved in immune responses and produce lymphocytes,
plasma cells and polymorphonuclear leucocytes
E. Terminal vessels, the right and left thoracic ducts, drain into the right
and left subclavian veins near their junction with the jugular veins
84. Inguinal lymphadenitis may be caused by:

A. N. gonococcus
B. Wuchereria bancrofti
C. Chlamydia trachomatis
D. Haemophilus ducreyi
E. Herpes genitalis
85. Acute pre-or post-auricular lymphadenitis may be due to infection
of the:

A. External or middle ear


B. Tonsils
C. Naso-oro-pharynx
D. Scalp
E. Mouth

20
86. External iliac adenitis or abscess:

A. Is in the iliac fossa above but continuous with the inguinal


ligament
B. Is often associated with fever and malaise
C. May follow infection of the glans penis, clitoris or lower limb
D. Is usually caused by Esch. Coli
E. Should be differentiated from appendix mass and septic arthritis
of the hip

87. In the differential diagnosis of lymphadenopathy:

A. Tuberculosis is the most likely cause if the age is under 10, and
and malignancy the most likely in those over 50
B. A short history is suggestive of acute infection and a long history
of chronic infection or malignancy
C. In the groin, pain is suggestive of acute inflammation, L.G.V. or
tuberculosis and absence of pain of malignancy or syphilis
D. Fever, malaise and myalgia are suggestive of acute or chronic
infection and not malignancy
E. Enlarged modes in the posterior triangle with a transient rash should
In endemic areas raise a suspicion of trypanosomiasis
88. In Lymphadenopathy:

A. Localized, firm, discrete or matted nodes with areas of cold


suppuration
or sinuses suggest tuberculosis
B. Localized, hard, discrete or mated nodes in a 50-year old man
suggest
filariasis
C. Enlarged epitrochlear nodes in the presence of generalized
lymphadenopathy
suggest syphilis
D. Maculo-papular rash with generalized lymphadenopathy, especially
if the spleen is enlarged, supports infectious mononucleosis,
toxoplasmosis
or Hodgkin’s lymphoma
E. Petechiae with generalized lymphadenopathy and hepato-
splenomegaly

21
maybe due to acute leukaemia
89. Investigations that will be useful in a 45-year old man with a 3-
month
history of enlarged lymph nodes in the right anterior triangle of the
neck
and a negative full clinical examination include:

A. V.D.R.L
B. Plain chest X-ray
C. Full blood count and E.S.R.
D. Blood film for microfilariae and aspiration of an enlarged node
E. Biopsy of an enlarged node

90. Lymphoedema:
A. Is accumulation of excessive amounts of interstitial fluid due to
inadequate
drainage as a result of obstruction of the lymph vessels
B. Affects the skin primarily
C. May also affect the muscles
D. Affects dependent parts of the body
E. Is primary if it is due to congenital mal-development of the lymph
vessels
91. Below-knee lymphoedema in a woman of 30 years in West Africa is
likely
to be due to:

A. Recurrent sepsis of the foot


B. Tuberculosis of the inguinal nodes
C. Wuchereria bancrofti
D. Lymphoedema tarda
E. Metastases in the inguinal lymph nosed
92. Lymphoedema of the whole leg and scrotum in a 40-year old male
in
West Africa is likely to be due to:

22
A. Lymphoedema praecox
B. Trauma to the thigh
C. Tuberculosis of the inguinal nodes
D. Brugia Malayi
E. Metastases in the inguinal lymph nodes
93. Lymphoedema of the vulva alone in a 40-year old woman in West
Africa
is likely to be due to:
A. Lymphogranuloma venereum
B. Tuberculosis of the inguinal nodes
C. Chronic pyogenic sepsis of the vulva
D. Lymphoedema tarda
E. Wuchereria bancrofti
94. Lymphoedema of a breast in a 40-year old woman in West Africa
is likely to be due to:

A. Brugia malayi
B. Tuberculosis of the axillary lymph nodes
C. Lymphodema congenital
D. Axillary node metastases
E. Recurrent infection

95 Complications of lymphoedema include:

A. Recurrent cellulitis and lymphangitis


B. Thrombophelitis
C. Lymphorrhoea
D. Ulceration
E. Lymphangio-sarcoma
96. If in lymphangiography of lymphoedematous leg of a 20-year old
girl,
the vessels are:

 Narrow and the lowest vertical nodes are not outlined while the
vessels
from the thigh and their corresponding nodes are normal, then the
cause is chronic sepsis
 Dilated and tortuous and the nodes are not visualized, then
tuberculosis
Is the likely cause

23
 Dilated and tortuous and the nodes are blocked or dilated, then the
likely
cause is filariasis
 Not visualized, then aplasis is the most likely cause
 Increased in number, caliber and tortuosity, then congenital varicose
dilation is the cause
97. Lymphoedema of the scrotum:
A. Is usually seen between 20 and 60
B. Is invariably accompanied by hydrocele
C. Does not usually involve the penis
D. May be associated with tiny vesicles of the skin of the scrotum
E. May be secondary to urinary fistulae
98. Unilateral lymphoedema of the lower limb in a 45 year old woman
should
be differentiated from:

A. Deep venous thrombosis


B. Congestive cardiac failure
C. Congenital arterio-venous fistula
D. Congenital neurofibromatosis
E. Obstruction of the inferior vena cava

99. Measures taken in the treatment of lower limb lymphoedema which


is soft
in a 35-year old woman due to chronic infection include:

A. Sleeping with the foot of the bed elevated


B. Application of well-fitting elastic stocking on the leg as soon as she
gets out of bed
C. Charles’s operation
D. Long-term antibiotic therapy
E. Course of frusemide
100. In the treatment of lymphoedema of the:
A. Lower leg, buried flap operation (Thompson’s operation) gives the

24
best cosmetic result
B. Breast, amputation should be done
C. Vulva, antibiotics and diuretics suffice
D. Scrotum, excision should be done
E. Upper limb, amputation should be done
101. The histological types of Hodgkin’s lymphoma include:
A. Lymphocytic predomdinant
B. Nodular sclerosis
C. Histio-cytic predominant
D. Mixed cellularity
E. Lymphocyte depleted
102. Hodgkin’s disease:
A. Is pathologically characterized by the presence of giant cells
B. Is distributed throughout the world
C. Occurs in the third decade and over
D. Is “curable”
E. Is granulomatous rather than neoplastic
103. Hodgkin’s disease may present:
A. Most commonly as cervical or supraclavicular lymphadenopathy
B. As persistent pyrexia of unknown origin or cyclic fever
C. Often with paraplegia
D. With weight loss, lassitude or pruritus
E. With an abdominal “mass”

104. Enlarged lymph nodes in Hodgkin’s disease are:


A. Single or confluent
B. Hard
C. Generally attached to skin
D. Usually movable
E. Painful and tender
105. Essential ROUTINE investigations in every patient with suspected
or
proven Hodgkin’s disease in West Africa include:
A. Bone marrow aspiration or biospy
B. X-ray of the chest

25
C. Lower extremity lymphangiogram
D. Lymph node biopsy
E. Staging laparotomy with splenectomy
106. In the Ann Arbor clinical staging of Hodgkin’s disease:
A. Stage I is limited disease of a single lymph node region or a single
extra-lymphatic organ or site
B. Stage II is involvement of 2 or more lymph node regions on the same
side of the diaphragm (II) or localized involvement of extra-lymphatic
organ or site and of one or more lymph node regions on both sides
of the diaphragm (IIE)
C. Stage III is involvement of lymph node regions on both sides of the
diaphragm (III) which may be accompanied by localized involvement
of the spleen (IIIS) or by solitary involvement of an extra-lymphatic
organ (IIIE) or both (IIISE)
D. Stage IV is diffuse or disseminated involvement of one or more
extra-lymphatic
organs or tissues with or without associated lymph node involvement
E. Lymphangiogram must be done in all cases
107. In the treatment of Hodgkin’s disease:
A. Stage I is treated by radiotherapy
B. Stages II is treated by radiotherapy and chemotherapy
C. Stages III and IV are treated by chemotherapy
D. The chemotherapy has been 6, 2-weekly cycles utilizing nitrogen
mustard,
vincristine, pro-carbazine and prednisone followed by a 2-week rest
period
E. Long-term disease-free remissions have been obtained in over 90%
of previously untreated patients

108. Lymphocytic and histiocytic lymphomas:


A. Occur throughout life with a peak incidence at 40-70
B. Present most commonly with extra-nodal involvement e.g. terminal
ileum mass
C. May also present with painless enlarged lymph nodes in the neck,
axillae
or groin
D. May also present with hepto-splenomegally, ascites, pleural effusion
lymphoedema
E. Do not require staging laparotomy with splenectomy as in Hodgkin’s
109. In the treatment of lymhocytic and histiocytic lymphomas:

26
A. Block dissection is done for Stage I
B. Radiotherapy is used for Stage I disease
C. For Stages III and IV, the trend is to use radiotherapy and cyclical
chemotherapy
D. Currently, cyclophosphamide, vincristine and prednisone are given
every
21 days for 6 courses
E. Prognosis is best in the well-differentiated histiocytic lymphoma
110. Burkitt’s tumour:
A. Is the commonest malignant neoplasm in children in Africa
B. Does not occur outside of Africa
C. Occurs in epidemic form in parts of Africa
D. Has a high incidence across Africa within a belt 10-15˚ north and south
of the equator below 1500 m in altitude
E. Has the highest age incidence in both sexes between 4-12 years with
a
median age at 7-8
111. The histology of Burkitt’s lymphoma shows:
A. Lymphoctes-poorly differentiated
B. Undifferentiated lymphoreticular stem cells
C. Eosinophils
D. Histiocytes-well differentiated
E. Macrophages with abundant clear cystoplasm
112. Burkitt’s tumour presents most commonly as:
A. Cervical lymphadenopathy
B. Ascites
C. Painless progressive enlargement of one or more jaw quadrants
especially of the maxilla
D. An Abdominal mass
E. Flaccid paraplegia

113. In females a characteristic feature of Burkitt’s tumour is:


A. Firm circumscribed swellings in both breasts
B. Ovarian tumours usually bilateral
C. Uterine tumour
D. Hepato-splenomegaly
E. Ascites
114. Burkitt’s tumour may involve the:
A. C.N.S. and cranial nerves
B. Testes

27
C. Lungs, salivary glands
D. Subcutaneous tissues, long bones
E. Thyroid
115. Essential investigations in every confirmed case of Burkitt’s
lymphoma
include:

 Liver biopsy
 Intravenous pyelogram
 Lumbar puncture for CSF cytology
 Staging laparotomy and splenectomy
 Bone marrow aspiration for cytology
116. Currently, the treatment of choice of Burkitt’s lymphoma without
CNS
involvement is:
A. Radiotherapy
B. Surgery for localized tumour
C. Cyclophosphamide administered as a single large dose of 40 mg/kg
I.V. and repeated for 4 courses at 2-3 weekly intervals
D. Methotrexate administered as a large dose I.V. followed by citrovorum
factor 24 h later
E. Cyclical combination chemotherapy with cyclohosphamide, vincristine
and methotrexate at 2-3 weekly intervals
117. It has been advised that following successful induction therapy in
Burkitt’s
lymphoma, there should be:

A. Maintenance therapy with cyclophoshamide


B. Maintenance therapy with methotrexate
C. Maintenance therapy with cytosine arabinose
D. No further treatment
E. Intrathecal administration of methotrexate or cytosine

118. Measures that must be taken during treatment of Burkitt’s


lymphoma
include:

A. Blood transfusion
B. Monitoring of serum electrolytes and blood urea
C. Monitoring of blood uric acid
D. Administration of antibiotics

28
E. I.V. fluid therapy
119. In the prognosis of Burkitt’s lymphoma:
A. Long-term survival varies from 40-65%
B. About 50% of those showing remission have a relapse
C. Over 80% of those showing relapse have C.N.S. involvement
D. Chemotherapy has not been effective in relapse
E. Serum from cured patients has given favourable response in replapse
120. Sabfasical endoscopic perforator surgery has:
A. Proven a veritable minimally invasive alternative to open surgery
 Lower recurrence rate to conventional open surgery
 A higher neovascularization rate than conventional open surgery
 A lower rate of failure to ligate long saphenous vein tributaries
 A high success rate in locating and ligating significant perforators
121. Infective agents that have been implicated in the etiology of
lymphomas
include:

A. Helicobacter pylori
B. Human herpes virus-8
C. Hymenolepis nana
D. Hepatitis A
E. HTLV type I
122. The following are true of Hodgkin’s lymphoma:

 Majority of cases occur in the elderly


 The characteristic cell is the Reed-Sternberg cell
 It is rare in Jews
 Presenceof unexplained fever affects prognosis
 It is usually disseminated at the onset

123. In a patient with stage IIA Hodgkin’s Lymphoma you would expect:

A. Drenching night sweats


B. No signifiant weight loss
C. Bulky disease

29
D. Involvement of two or more lymph node regions on the same side
of the diaphragm
E. Unexplained fever
124. Gastric MALT lymphoma:

A. Is assoicated with H. pylori infection


B. Has origins from a mature B cell
C. Is associated with an increased incidence of perforations
D. Develops following EBV infection
E. Is associated with a history of long standing peptic ulcer
125. In endemic Burkitt:

A. Children are most often affected


B. Marrow involvement occurs early
C. The malignant cell is highly sensitive to Cyclophosphamide
D. EBV antibodies are usually absent
E. Extranodal sites are usually involved

CHAPTER 50
VASCULAR AND LYMPHATIC SYSTEMS

ANSWERS

1. B, C 38. B, E 75. A, B, C, D, E
112. C, D, E
2. A 39. D 76. C

30
113. A, B
3. D, E, 40. E 77. B
114. A, B, D, E
4. A, C, D, E 41. A, B, D, E 78. A, B
115. B, C, E
5. C 42. B, C, E 79. A, B, C, D, E
116. C
6 A, D, E 43. B, C, E 80. C
117. E
7. C 44. C, D 81. C
118. A, B, C, E
8. A, B, D 45. C 82. A
119. A, B
9. C 46. B, D 83. A, E
120. C
10. A, C, E 47. A, D 84. A,B, C, D, E
121. A, B, E
11. A, B, C, D 48. C, E 85. A, D
122. B, D
12. B, C, E 49. A, B 86. A, B, C, E
123. B, D
13. A, C, E 50. E 87. B, E
124. A, B
14. C 51. A, B, C, D 88. A, C, E
125. A, C, E
15. A, B, C, E 52. A, C, D 89. B, C, E
16. A, B 53. B, D, E 90. A, D, E

17. A, E 54. B 91. A, D


18. B, C 55. A 92. C, E
19. A, B, D 56. D 93. A, C
20. A 57. D 94. B, D
21. A, E 58. A, B, C, E 95. A, C, D, E
22. D, E 59. D 96. A, B, C, D, E
23. B, D 60. E, F, G, H 97. B, D, E
24. A, C, D 61. D 98. A, C, D
25. A, B, C, D 62. C 99. A, B, D, E
26. B, D 63. D 100. A, B, D,
27. A, C, D 64. C 101. A, B, D, E
28. A, D 65. C 102. B, D
29. B, C, E 66. C 103. A, B, D, E
30. B, C, D 67. A 104. A, D
31. A, B, C, D 68. B, D 105. B, D, E
32. B, C, D, E 69. A 106. A, C, D
33. A, C 70. A, B 107. A, C, D
34. B 71. C, D 108. A, C, D
35. B, D 72. E 109. B, D
36. E 73. A, E 110. A, D, E
37. A, C, E 74. C 111. B, E

31
CHAPTER 51

SURGICAL ASPECTS OF THE


HAEMOGLOBINOPATHIES

 In West Africa the incidence of the sickling trait varies:


 From 10% in Northern Ghana to 30% in the Northern Nigeria
 From 5% in Northern Ghana to 10% in the Northern Nigeria
 From 20% in Northern Ghana to 15% in the Northern Nigeria
 From 15% in Northern Ghana to 20% in the Northern Nigeria
 From 25% in Northern Ghana to 25% in the Northern Nigeria
 Noted features of sickle cell crisis include:
 The hand and foot syndrome
 Acute tender hepatomegaly
 Tachycardia and tachypnoea
 Sequestration syndrome
 Recurrent jaundice
 An Africa boy of 12 presents with sudden onset of fever,
nausea, vomiting and diarrhoea of 24 h duration. He admitted
to two previous episodes of similar illness which required
short periods of detention at the local health centre. On
examination he had tachycardia, tachypnoea, a tingue of
icterus with a palpable tender liver. Rectal findings were non-
contributory.

The most likely diagnosis is:


 Amoebic liver disease
 Severe attack of shigellosis
 Subacute hepatitis
 Sickle cell crisis
 Viral hepatitis
 In sickle cell anaemia (Hb SS genotype) the average steady
state Hb level is about (g/dl):
 7
 8

1
 9
 10
 11

 Hepatomegaly in sickle cell disease is primarily due to:


 Congestive hepatic crisis
 Cholestasis
 Viral hepatitis
 Amyloidosis
 Multiple liver infarcts with abscess formation
 Generalized lymphadenopathy is seen in which of the following
categories of sickle cell patients?
 In 5% of SC patients and over 90% of SS patient
 In 10% of SC patients and over 90% of SS patient
 In 15% of SC patients and over 70% of SS patient
 In 20% of SC patients and over 60% of SS patient
 In 25% of SC patients and over 50% of SS patient
 A 15-year old Ghanaian boy presented with a recurrent ulcer
just above the medial malleolus of his leg of four years
standing. There was no history of trauma but he admitted to
recurrent episodes of fever diffuse, bodily pains and weakness.
The ulcer itself had flat gently sloping edges with pale
granulations in the floor.

The most likely diagnosis is:


 Varicose ulcer
 Gravitational ulcer
 Chronic myeloid leukaemia
 Sickle cell leg ulcer
 Non-specific ulcer of infective origin

2
 Which investigation will confirm the diagnosis?
 Blood film
 X-ray of the tibia
 Hb electrophoresis
 White cell count
 Venogram
 You will advise:
 Ambulatory treatment with bandaging
 Application of bland antiseptics and bed rest
 Surgical debridement and grafting
 Application of viscopaste to the leg
 Dressing with Solcoseryl
 A girl of 18 with Hb SC disease presented with acute
respiratory distress, cough and frothy sputum and soon after
admission became drowsy. On examination, the temperature
was 37˚C and there were petechial rashes on both arms; there
were no pulmonary signs.

The most likely diagnosis is:


 Lobar pneumonia
 Acute right heart failure
 Pulmonary embolism
 Fat embolism
 Left ventricular failure
 The most useful investigation is:

3
 Bronchoscopy
 Pleural tap
 Radioactive Xenon scan of the chest
 Examination of the urine for fat globules
 ECG

 You would advise


 Penicillin G I M 1mega unit 6 hourly
 Intercostal tube drain
 Anticoagulation with heparin
 Intermittent oxygen, IV 5% Dextrose with 5% alcohol and
heparinisation
 Digitalization
 The bone changes consequent on sickle cell disease may take
the
form of:
 Erythroid hyperplasia
 Osteoporosis affecting the inner aspects of the cortex
 Avascular necrosis of the cortical bone
 Sclerosis of involved bone
 March fractures of the metatarsals
 The bone changes of sickle cell disease are particularly
encountered in the:
 Pelvis
 Skull
 Vertebral bodies
 Metacarpals
 Radius
 Which of the following lesions may be found in the sickle cell

4
patient?
 Osteochondritis dissecans
 Subchondral and central sclerosis
 Coxa vara
 Flatening and irregularity of the femoral head
 Milkman’s fracture

 The initial treatment of choice for the management of the bone


lesion of sickle cell disease affecting the hip is:
 Active physiotherapy
 Prolonged bed rest with the limb in traction
 Containment upper femoral osteotomy
 Prophylactic chloramphenicol
 Increasing calcium and vit. D supplements
 Sickle cell crisis:
 May be precipitated by any cause of diminution in the tissue
oxygen tension
 Occurs most commonly in children and young adults
 May present predominantly as the “hand and foot syndrome”
 In its abdominal form is often clinically indistinguishable from
acute intestinal obstruction
 May present as a rapidly developing hepatomegaly or
splenomegaly
 Sickle cell crisis:
 Is primarily related more to the pH than the oxygen tension in the
tissues
 Shows a marked tendency to diminish in frequency over the age

5
of 30
 Is frequently marked by muscle tenderness
 In its abdominal form may be indistinguishable from pancreatitis
 Reticulocytosis ranging from 10-15% may be associated with it
 In sickle cell disease:
 Not all patients are found to be anaemic
 Symptoms only begin to manifest after 3 months of life
 The average (steady state) Hb level for SC patients is 8g/dl
 The most severe anaemia occurs with SB thalassaemia
 Growth disturbance may produce a stunted figure, much below
average weight

 In sickle cell disease:


 Complete deoxygenation causes the red cell to become ovoid,
shortened and crescentic
 The PCV is above normal
 HbC has dilutional effect on the sickling process more than HbA
 Growth disturbance may result in tall asthenic body build
 Low levels of gonadotrophin are encountered
 Two weeks after successful uneventful delivery of her second
baby a West African woman of 30 is brought to the Emergency
Room with a sudden attack of abdominal pain, nausea and
vomiting with headaches joint pains and fever. There was a
similar episode after her first pregnancy which settled
spontaneously. On examination she is moderately pale and
dehydrated; the pulse is 90min regular and BP 90/70. The chest
is clear and the abdomen is diffusely tender without guarding,
but a vague tender mass is palpable in the left hypochondrium.

The most likely diagnosis is:

6
 An attack of falciparum malaria
 Typhoid fever
 Sequestration syndrome of sickle cell disease
 Splenic infarct
 Viral hepatitis A
 The most useful diagnostic investigation is:
 Plain x-ray of the chest-erect
 Plain x-ray of the abdomen-erect
 Abdominal ultrasound
 Liver function tests
 A full haematological profile

 The indicated treatment is:


 Resuscitation with I/V fluids and broad spectrum antibiotics
 Bed rest with attention to nutrition
 Resuscitation with I/V fluids, broad spectrum antibiotics,
antimalarial therapy, close monitoring P02, PC02, PH
 Resuscitation and early surgical intervention
 Resuscitation + Tripple Therapy for peptic ulcer
 A child of ten noted for his complaints of cold season aches
and pains is seen in the Emergency Room in his third attack of
fever headaches excruciating pain in the back and limbs. On
examination he was not pale nor icteric but his tongue was
furred and coated. His legs felt warm and there was tenderness
especially proximal to the left medial malleolus. X-rays of the
limbs were grossly normal.

7
The most likely diagnosis is:
 Recurrent attacks of malaria
 Recurrent attacks of typhoid
 Osteomylitis of the left tibia
 Hand foot syndrome of sickle cell disease
 Attacks of Dengue fever
 The most useful diagnostic test is:
 Blood film for malarial parasites
 Blood culture
 A full haematological profile
 Chest x-rays
 Liver function tests

 The indicated therapy is:


 A course of Artesunate – Amodiaquine for 3 days
 A course of Ciprofloxacin and Metronidazole
 Course of Cloxacillin 500mg qds x 7 days
 Resuscitation + broad spectrum antibiotics antimalarials and
monitoring P02, PCO2 PH
 Bed rest, adequate nutrition and splinting the affected joints
 A young lady of 15 comes to the Casualty Department with
sudden onset of abdominal pain initially periumbilical, but later
settling in the (Rt.) lower quadrant of the abdomen. The patient
also complains of fever headaches, vomiting and dysuria. He
remembers two previous similar attacks that settled
spontaneously. On examination she is pale with a temperature
of 39°C; the chest is clear, but the abdomen is diffusely tender
without guarding and rectal examination is unremarkable. The

8
patients blood count is reported as “Total WBC 40,000/cub
mm, including normoblasts with reticulocystosis – 12%.

The most likely diagnosis is:


 Acute appendicitis
 Mesenteric adenitis
 Salmonellosis
 Cholecystitis
 Sickle cell crisis
 Which test is most likely to yield further useful information
 Full haematological profile
 Abdominal ultrasound
 Plain x-ray of the abdomen-erect
 Blood culture
 Abdominal paracentesis

 The indicated therapy for this patient is:


 Resuscitation followed immediately by appendectomy
 Exhibition of bactriocidal antibiotics
 Resuscitation with I/Vs + Ciprofloxacin and Metronidazole
 Resuscitation with I/Vs bacteriocidal antibiotics and monitoring of
abdominal signs, PCO2, PO2 and PH
 Resuscitation + Tripple therapy
 The average steady state Hb maintained by patients with
Sickle Cell disease (Genotype SS) in the typical socio-
economic circumstances in West Africa is:
 8gm/dl
 9gm/dl
 10gm/dl

9
 11gm/dl
 12gm/dl
 The average steady state Hb maintained by patients with Sickle
Cell disease (Genotype SC) in the typical socio-economic
circumstances in West Africa is:
 8-9gm/dl
 10-11gm/dl
 12-13gm/dl
 14-15gm/dl
 16gm/dl

 A girl of 12 known to have Sickle Cell disease (Genotype SS)


has had episodes of colicky abdominal pain and dyspepsia,
fever, and fluctuating jaundice over the past 3 years. During
these attacks the urine has been dark but the stools have
retained a normal colour. On examination she was jaundiced
and mildly pale. The abdomen was full, the liver and spleen
were palpable and non tender; there were no other masses.
Normal stool was encountered on P.R. Liver function tests
indicated both direct and indirect hyperbilirubinaemia and a
threefold increase in the alkaline phosphatase level.
Abdominal ultrasound was essentially normal.
The most likely diagnosis is:
 Cholestatic jaundice from gallstone obstruction of the CBD
 Cholestatic jaundice from intra canalicular cholestasis

10
 Haemolytic jaundice attendant on repeated episodic sickle cell
crises
 Hepatocellular Jaundice – Hepatitis A
 Hepatocellular jaundice – Hepatitis B
 The test most likely to clinch the diagnosis is:
 Percutaneous Transhepatic Cholangiography (PTC)
 Endoscopic retrograde Cholangiopancreatogrophy (ERCP)
 Magnetic Resonance Cholangiopancreatogrophy (MRCP)
 Hepatic Scintigraphy
 Helical CT Scan of the abdomen
 The treatment for the patient in Question 32 is:
 Resuscitation + adequate nutritional supplements
 Resuscitation + check on G6PD status and review of charted
drugs given
 Bed rest and attention to urgent nutrition needs
D. Resuscitation + monitoring for signs of progression and need for
surgical
intervention
E. Resuscitation, I/V administration of 10% fructose, Vitamin. K and
prophylactic antibiotics

 It is evident that the patient in Question 32 would need surgery;


which of these procedure(s) is mandatory at this operation:
 Drainage of the peritoneal cavity
 Drainage of the subphrenic spaces on the right
 Administration of Vit. K during the surgery
 Operative cholangiography + T tube drainage of biliary free
 Establishment of a choledocho-duodenostomy
 Which of these features confirms a state of significant hyper-

11
splenism in a patient with splenomegaly in Sickle Cell Disease,
and therefore requiring splenectomy?
 The spleen is > 4cm below the Lt. costal margin
 The HB falls < 6.5g/dl
 The reticulocyte count exceeds 15%
 The platelet count falls below 200 x 109/l
 None of the above
 A man of 25 presents in the Emergency Room with a sudden
onset of severe pain in the left hypochondrium, much
worsened on inspiration. He is a pyrexial, and anicteric; his
chest is clear and has a tender mass that moves with
respiration in the left hypochondrium. There have been no
previous attacks:

The most likely diagnosis here is:


 Pancreatic pseudocyst
 Subphrenic abscess
 Splenic infarct
 Left diaphragmatic pleurisy
 Amoebic liver abscess affecting the left lobe

 Which one diagnostic test would you choose to clinch the


cause for this presentation:
 Chest x-ray
 Abdominal ultrasound
 A full haematological profile
 CAT Scan of the abdomen
 Examination of fresh specimen of stool
 Hepatomegaly in sickle cell disease:

12
 May be due to viral hepatitis
 Is attended by the morbid anatomical change of sinusoidal
engorgement with fibrin clots
 Is marked by central lobular parenchymal necrosis
 Is very frequently attended by jaundice of obstructive
intrahepatic type
 May be accompanied by fever, leucocytosis and abdominal pain
 Hepatomegaly in sickle cell disease:
 Is frequently associated with extrahepatic ductal obstruction by a
stone
 Is attended by sinusoidal haemosiderosis
 May be associated with biliary canalicular thrombi
 May on occasion be an indication for laparotomy
 Is often seen with anasarca
 Jaundice in sickle cell disease:
 Is most frequently acholuric arising from excessive red cell
destruction
 May produce an elevation of both direct and indirect reacting
bilirubin
 May be difficult to distinguish from extrahepatic ductal
obstruction
 May on occasion be an indication for laparotomy
 Should initially be treated conservatively with adequate fluids
and nutritional supplements

 The spleen in sickle cell disease:


 Undergoes progressive fibrosis from repeated infarcts
 Is one of the organs involved in the sequestration syndrome

13
 Is characteristically reduced to a small nubbin of fibrous tissue in
adolescent SS patients
 Is seldom reduced in size in patients with SC disease
 May occasionally require removal as a measure to reduce
frequency of haemolytic episodes
 The kidney in sickle cell disease:
 May show defective tubular concentration in the form of
hyposthenuria
 Has a tendency to excessive concentration of urine
 Is often grossly enlarged (symmetrically)
 Need not be investigated for other causes of haematuria in
patients presenting with haematuria
 If the source of unilateral presistent bleeding may require
surgical excision
 Haematuria in sickle cell disease patients:
 Occurs in 2-4% of Hb SS and Hb SC patients
 Is more likely to arise from causes other than the sickling lesion
in the kidney
 May be prolonged and recurrent and usually painless
 Is treated with blood transfusion, alkaline infusions and
occasionally aminocaproic acid
 May warrant a nephrectomy if unilateral and persistent

 Haematuria in sickle cell disease patients:

14
 Arises usually from ulcerations of the renal papillae, the result of
papillary infarct
 Is likely to be persistent because of constant contact of the
ulcerating lesion with urokinase
 Is usually total and recurrent
 May be treated by blood transfusions, alkaline infusions and
diuretics
 Is never life threatening enough to warrant nephrectomy
 The recurrent haematuria which occurs in some 2-4% of patient
with Sickle Cell disease (Hb SS and SC) arises from:
 Ulceration of the tips of the renal papillae
 Infarction in the medulla of the kidney
 Sickling phenomena in the cortex of the kidney
 Ulceration in the trigone of the bladder
 Infacts of the mucosa of the renal pelvis
 The bleeding in Sickle Cell haematuria becomes repetitive and
persistent because:
 Of repeated episodes of sickling crisis
 The hyposthenuria that accompanies Sickle Cell disease
 Febrinolytic enzymes (urokinase) being continuously in contact
with
the urine
 High pH of the urine in Sickle Cell disease
 Attendant urinary tract infection
 The first line management in Sickle Cell patients with
haematuria is:
 Intravenous infusion of alkalinizing agents (NaHCO3)
 Blood transfusions
 Administration of diuretic agents
 Infusions of Amino caproic acid

15
 Infusion of ringers lactate

 Priapism in sickle cell disease:


 Occurs in 60% of cases in boys under 12 years of age
 Is episodic and tends to occur during sleep
 Is liable to produce pain, distress and exhaustion
 Usually eventually settles on conservative measures
 Is liable to lead to penile fibrosis if allowed to persist for several
weeks
 Priapism in sickle cell disease:
 Occurs in 80% of cases in boys over 12 years of age
 Is more likely to occur during the day than at night
 Seldom distresses the patient
 May be treated by sapheno-cavernosal anastomosis if persistent
 May in recurrent cases be complicated by impotence
 The priapsm that occurs in patients with Sickle Cell disease:
 Most frequently occurs in adolescents > 18 yrs
 Starts with initial sickling in the corpus spongiosum
 Is recognized as severe if it lasts about 3h
 Should be recognized as an emergency
 Should be treated prophylactically by caverno-spongiosal shunts
 Leg ulcers in sickle cell disease:
 Are seen in 10% Hb SS patients in West Africa
 Occur in 90% of cases on the lower third of the leg above the
medial malleolus
 Are bilateral in a third of cases
 Arise from a combination of factors, trauma, infection and local
circulatory deficiencies

16
 Should be treated by bed rest and wound dressing with mild
antiseptics

 Leg ulcers in sickle cell disease:


 Are seen in 2% of Hb SC patients in West Africa
 Occur in 50% of cases on the lower third of the leg above the
medial malleolus
 Are bilateral in 20 per cent of cases
 Are seldom complicated by malignant change
 May be treated by surgical debridement and grafting
 Regarding leg ulcers in Sickle Cell disease:
 The incidence in the West Indies is 60% of Hb SS patients
 The incidence in the Ghana is 10% of Hb SS patients
C. The incidence in the USA (Blacks) is 10% of Hb SS patients
D. The incidence in the West Africa is 2% of Hb SC patients
E. The peak incidence in the West Indies occurs in 10-14 year
group
F. The peak incidence in the West Africa occurs in 15-19 year
55. The site of election for leg ulcers in Sickle Cells disease
patients is:
A. Lower third of the leg above the medial malleolus
B. Front of the tibia
C. Dorsum of the foot
D. Sole of the foot
E. Over the lateral malleolus
56. The leg ulceration that occurs in Sickle Cell disease (Hb SS):
A. Is bilateral in one-third to half the cases

17
B. Is more often than not associated with varicose veins with
incompetent
perforatiors
 Is usually deep seated with exposure of muscle, tendon and
periosterum
 Shows no stratification of incidence in socio economic groups
 Is frequently complicated by malignant degeneration

57. In the management of the chronic ulcers that complicate


Sickle-Cell
Disease thrust of therapy should be:
 Rapid elevation of the Hb level by multiple transfusions
 Early ulcer excision and pinch grafting
 Bed rest with adequate nutrition and local dressings
 Emphasis on supply of trace elements – (zinc sulphate: 200mg
tds)
 Routine grafting of ulcers with healthy granulation tissue
58. Lobar pneumonia in sickle cell disease:
A. May precipitate or follow a crises
B. Occurs approximately a hundred times more frequently than in
the
general population
C. Has a tendency to recurrence (70%) and multiple lobe
involvement (40%)
 Has a predilection for the upper lobes
 May be accompanied by pulmonary hypertension and right
ventricular hypertrophy
59. Lobar pneumonia in sickle cell disease:

18
A. Is seldom a cause for a crisis
B. Occurs as frequently as in the general population
C. Characteristically affects the lower lobes
D. Is common because of pulmonary congestion, thromboses and
impairment of immunity on account of autosplenectomy
E. Is often complicated by cardiac arrhythmias and right heart failure

60. Pneumonia is known to occur approximately a hundred times


more
frequently in patients with Sickle Cell disease than in the
general
population. The reason(s):
 Impairment of polymornuclear phagocytic action by pulmonary
congestion
 Thrombotic episodes in the pulmonary circulation
 Deficiency in heat labile opsonizing activity of the serum against
the pneumococcus
 Frequency of functional auto-splenectomy in the patient with SS
disease
 High level of lung tissue hypoxia
61. Ocular changes in sickle cell disease may take the form of:
A. Sluggish flow with minimal arteriolar occlusion especially at the
periphery of the fundus
B. Vascular occlusion with demonstrable interruption of blood flow in

19
capillaries of periphery of fundus
C. Retinal, choroidal and anterior uveal infarcts
D. Micro-aneurysms, dilated capillaries and veins
E. Proliferative neovascularization of the vitreous from which vitreous
haemorrhages may occur
62. Ocular changes in sickle cell disease may take the form of:
A. Heightened flow in the arterioles of the fundus with silver wiring
appearance
B. Ischaemic areas showing whitening of the retina
C. Haemorrhages of varying severity which may be retinal, peri-
retinal
D. Neovascularization of the retina, budding capillaries, a
compensatory
reaction to retinal ischaemia
E. Tears and detachment of the retina

63. The earliest change in development of ocular changes in Sickle


Cell
disease is:
A. Vascular Occlusion of arterioles and capillaries at the periphery
of the
retina
B. Dilatation and tortuosity of vessels at the posterior pole of the
fundus
C. Retinal Neovascularistion – budding capillaries attempting to
bridge
adjacent arterioles and venules
D. Vitreous haemorrhages
E. Retinal detachment

20
64. In the management of patients with sickle cell disease:
A. Blood transfusion is more useful than alkaline infusions in
decreasing
the sickling tendency
B. Oxygen therapy has no immediate effect on the sickling process
C. Prophylactic antimalarial therapy is of proven benefit
D. Prophylactic Vit. B12 has been found beneficial
E. Efforts should be made to achieve pre-operative Hb levels of 12
g/dl
in adults
65. In the management of patients with sickle cell disease:
A. Expanding the plasma volume using alkaline and dextrose
infusions
decreases the sickling tendency
B. Oxygen therapy relieves the pain and other distressing
symptoms
accompanying crises
C. Search for and prompt treatment of acute infections may abort a
crisis
D. Anticoagulants such as Arvin may prove useful in aborting or
arresting
an attack
E. Efforts should be made to achieve pre-operative Hb levels of 10
g/dl
in children

66. The changes in the vertebral column in sickle cell disease:


A. Affect particularly the lumbar vertebral bodies
B. Lead to reduction in height of the vertebral bodies
C. Result in bulging of the intervertebral discs into the bodies

21
D. Take the form of the classic cod-fish appearance
E. May take the form of collapse of the vertebral bodies
67. The pathological changes of sickle cell disease affecting the
hip:
A. Are common and indistinguishable from Perthes’ disease
B. Are more common in Hb SS patients than SC or other varieties
C. Are usually related to some sepsis in the upper end of the
femur
or hip joint
D. May produce septic dislocation of the hip
E. May result in total sequestration of the femoral head
68. The pathological changes of sickle cell disease affecting the
hip:
A. Are rare and distinct from all childhood hip affections
B. Usually results in bony ankylosis
C. Are nearly always the result of early experience of trauma to the
hip
D. Are frequently arrested by prolonged bed rest with the affected
limb
in traction
E. Are seldom bilateral
69. The head of the humerus in sicklers:
A. Is notably subject to infarction though not as frequently as the
femur
B. Does not deform as badly as the femoral head
C. Is more liable to develop early osteoarthritis than in the normal
population
D. Usually escapes early osteoarthritis
E. Showing avascular necrosis is best treated by short wave
diathermy
and mobilization exercises
70. Osteomyelitis in sickle cell disease:

22
A. Affects nearly 90% of Hb SS patient in one or more bones
before
the age of 10
B. May start anywhere along the shaft of the long bone
C. Usually starts in the metaphysis and spreads down the
diaphysis
D. Has a less stormy course than in the Hb AA patient
E. Should be treated by parenteral ampicillin and cloxacillin
71. Osteomyelitis in sickle cell disease:
A. Affects nearly 50% of Hb SS patients in one or more bones
before
the age of 10
B. Not uncommonly produces segmental lesions along the shaft
of the same bone
C. Is most frequently produced by staph. pyogenes
D Is frequently complicated by septicaemia
E. Should be treated by oxygen therapy, antibiotics, limb
immobilization
and timely evacuation of intramedullary pus
72. For children with Sickle Cell disease the principal cause of
post
operative morbidity and mortality is:
A. Acute chest syndrome and splenic sequestration
B. Post operative malaria
C. Dehydration
D. Urinary tract infection
E. Low Hb in the post operative period Hb < 8.0g/dl
73. In patients with Sickle Cell disease undergoing operation it
is generally
recommended that to obtain the normal P02 tension in the
system the
oxygen ventilation during surgery should be at least:

23
A. 20%
B. 25%
C. 30%
D. 35%
E. 40%
CHAPTER 51

SURGICAL ASPECTS OF THE ABNORMAL


HAEMOGLOBINOPATHIES

ANSWERS

1. A 38. C
2. A, B, C, D, E 39. A, B, C, E

3. D 40. B, C, D

4. B 41. A, B, C, D, E

5. A 42. A, B, C, D, E

6. A 43. A, C, E

7. D 44. A, B, C, D, E
8. C 45. A, B, C, D

9. C 46. A

10. D 47. C

11. D 48. A

12. D 49. A, B, C, D, E

13. A, B, C, D 50. D, E

14. A, B, C, D 51. D
15. A, B, C, D 52. A, B, C, D, E

16. B 53. A, D, E

17. A, B, C, D, E 54. A, B, C, D, E, F

18. B, C, D, E 55. A
19. A, B, (D) E 56. A

24
20. C, D, E 57. C
21. C 58. A, B, C, E

22. E 59. C, D, E
23. C 60. A, B, C, D, E

24. D 61. A, B, C, D, E,
25. C 62. B, C, D, E
26. D 63. B
27. E 64. C
28. A 65. A. B, C, E
29. D 66. A, B, C, D, E
30. A 67. A, C, D, E
31. B 68. D
32. C 69. A, B, C, E
33. C 70. A, B, E
34. D 71. B, C
35. D 72. A
36. A, B, C, D 73. C
37. C

25
CHAPTER 43
SYMPTOMS, EXAMINATION, INVESTIGATION AND
INSTRUMENTS IN UROLOGY

 Renal pain is located in the:

 Costo-vertebral angle

 Area over the upper lumbar vertebrae

 Loin

 Upper lateral quadrant of the abdomen

 Centre of the abdomen

 Renal pain is usually caused by:

 Inflammation of the nephrons

 Stretching of the renal capsule

 Excretion of irritating substances

 Irritation of the renal pelvis

 Inflammation of the renal pelvis

 Ureteric pain:

 Is persistent with spasms of colic

 Radiates from the tip of the 9th costal cartilage to the suprapublic

area, scrotum or labium

 May be caused by distension of the ureter

 May be caused by spasmodic contraction of the ureteric muscle

1
 Usually causes referred pain in the skin of the lumbar region

 Features of strangury include:

 Difficulty of micturition

 Agonizing pain

 Haematuria

 Passage of urine in drops

 Causation by chronic obstruction to outflow of urine

 Prostatic pain:

 Is caused by inflammation or malignant growth of the prostate

 Is deep in the rectum

 Is referred to the perineum or suprapubic region

 May be referred to the iliac fossae or scrotum

 Is constant but may be colicky

 Urethral pain:

 Is scalding, i.e. burning

 Occurs in the suprapubic region and along the urethra

 Occurs during or without micturition

 Is caused by acute obstruction to urine outflow

 Is caused by inflammation of the urethra

 In the symptomatology of genito-urinary disease:

2
 Difficulty is inability to void urine

 Oliguria is due to inability of a normal or diseased kidney to

elaborate urine

 Dysuria is difficulty or pain in micturition

 Enuresis is the involuntary passage of urine at night and not by


day

 Anuria is when little or no urine is excreted

 Nocturia may be due to:

 Excessive concentration of urine by the kidneys

 Contracture of the bladder

 Inflammation or irritation of the bladder

 Hypertrophy of the bladder

 Blood in the bladder

 In the radiography of the urinary system:

 Excretion urography is contra-indicated if there is a history of

allergy, e.g. asthma

 Excretion urography is contra-indicated if the blood urea is over

5 mmol/l

 Retrograde pyelography is done to confirm the finding of excretion

urography

 Renal angiography helps in the diagnosis of renal artery stenosis


or aneurysm and renal tumours or cyst

 Renal function can be assessed by measuring the uptake and

3
excretion

of radioactive iodohippurate

 Parenchymal infection of the urinary tract is suggested by the


presence

in the urine of:

 Red cell casts

 Sugar

 Crystals

 Leucocyte casts

 Protein

 In the investigation of urological diseases plain X-ray of the


chest, spine, pelvis and skull is indicated in:

 Calculi

 Carcinoma of the kidney

 Chronic renal failure

4
 Benign prostatic hypertrophy

 Carcinoma of the prostate

 Diagnostic tests in the investigation of suspected urinary


schistosomiasis include:

 Urine examination and culture

 Cystoscopy

 Urethral catheterization

 Stool examination

 Full blood count

 Clinically, G.F.R. is best measured with:

 Inulin

 Vit. B12

 Creatinine

 Urea

 Glucose

 The Foley catheter:

 Is self-retaining

 May be two-, three-, or four-way

 Is usually used as a suprapubic catheter but can also be used as a


urethral catheter

 Can only be inserted into the bladder with a stilette

 Can be sterilized by boiling

5
 Gibbon’s catheter:

 Is a self retaining catheter

 Has two flanges for fixing it to the penis or groin

 Is suitable for use in women

 Has a stiff tip

 Cannot be used in patients with urethral stricture or prostatic


enlargement

 Depezzer catheter:

 Is not self-retaining

 Is a suprapubic catheter

 Is commonly made of red rubber

 Can also be used as a urethral catheter in the female

 Can be sterilized by boiling

 Filiform catheters:

 Are made of red rubber or woven nylon

 Can be autoclaved

 Are used for dilating tight urethra strictures

 May be left in formalin cabinet or soaked in oxycyanide of mercury

 May be used to dilate the female urethra

 Sounds are:

6
 Straight bougies

 Used to dilate the female urethra

 Used to dilate any part of the male urethra

 Not autoclaved

 Used to calibrate the urethra during urethroplasty

 The cystoscope:

 Is used for visualizing the bladder and the urethra

 Can be used therapeutically

 Is used to remove lesions of the urethra

 Is used to remove some tumours of the bladder

 With the fibre light gives a larger viewing area

 Lister’s bougies:

 Are used for dilating urethral strictures

 Are made of woven nylon

 Are autoclaved

 Should not be used with force

 Can be used to dilate for female urethra

 A smooth filling defect at the base of the bladder on an I.V.P is

7
suggestive of:

 Prostatic carcinoma

 Vesical carcinoma

 Benign prostatic hypertrophy

 Vesical calculus

 Bladder diverticulum

 The value of renal biopsy includes:

 Diagnosis of renal disease process with unclear clinical picture

 Diagnosis of acute reversible or self limiting disease

 Prognostication of unclear acute renal diseases

 Determination of associated hypertension

 Accurate determination of associated hyperparathyroid state

8
CHAPTER 43
SYMPTOMS, EXAMINATION, INVESTIGATION AND
INSTRUMENTS IN UROLOGY
ANSWERS

 A, C, D

 B, D, E

 C, D

 A, B, D

 B, C

 A, B, E

 C, E

 B, C

 A, D, E

 A, D, E

 B, E

 A, B, D

 C

 A

 A, B, D

 B, C

 C

 A, B, E

9
 B, D, E

 A, C, D

 C, D

 A, B, C

10
CHAPTER 44
ACUTE RENAL FAILURE, HAEMATURIA, HAEMATOSPERMIA
 Acute renal failure:

 In oliguria 24hr urine output is <400mls

 Extra – renal uraemia is due to irreversible renal hypoperfusion

 Improperly treated pre-renal uraemia may lead to acute tubular


necrosis

 Acute obstructive renal failure ultrasonography shows dilated


ureters and calyces

 The end products of metabolism that may accumulate include


phosphates and creatinine

 Extra-renal failure:

 Reduction in GFR is due to decrease in renal blood flow

 In cases where the fluid and electrolyte imbalance worsens further


decrease in renal blood flow results from vasoconstriction

 Rehydration and frusemide induce diuresis.

 There is decrease of sodium concentration in urine

 Urine urea concentration is markedly reduced

 Intrinsic renal damage:

 The causes may be due to aminoglycosides use

 The nephrotoxins tend to cause patchy tubular necrosis

 In situations where acute tubular necrosis is likely to occur ,


administration of mannitol or frusemide may be protective

 Herbal preparations may cause it

 There is high concentration of urine creatinine

1
 Acute obstructive uropathy:

 Alternating periods of oliguria and polyuria is suggestive

 Anuria following a major pelvic operation is frequently due to


ureteric injury or ligation

 Urine urea concentration is usually low

 Vesical schistosomiasis may present as anuria

 Pelvic tumours are common cause

 In the aetiology of acute renal failure, pre-renal failure may be


caused by:

 Entero-cutaneous fistula

 Ruptured ectopic pregnancy

 Myocardial infarction

 Cirrhosis of the liver

 Radiation

 Non-obstructive causes of anuria include:

 Cortical necrosis

 Severe acute tubular necrosis

 Ischaemia of transplanted kidney

 Pelvic neoplasia

 Bilateral renal artery occlusion

 In the investigation of a patient with acute renal failure:

 24hr urine output is usually less than 400mls

 Glycosuria, proteinuria and haematuria may point to ATN

 In renal hypoperfusion, the urine/plasma ratios of urea and


osmolarity are decreased

2
 Radiographic contrast media may cause renal damage

 Urine sodium over 30mmol/l may point to pre-renal uraemia

 In the management of pre-renal failure:

 Acute blood loss is best replaced with normal saline

 Adequacy of volume replacement is best determined using CVP


measurements

 In the use of frusemide to induce diuresis, the state of hydration is


not important

 The maximum dose of frusemide is 80mg in a 70kg adult

 Mannitol may precipitate pulmonary oedema

 In oliguric phase of ATN:

 There is metabolic alkalosis

 There is a rise in serum bicarbonate and potassium levels

 There are falls in plasma free calcium and blood platelets

 Dietary sodium intake is encouraged due to a fall in plasma sodium


levels

 The changes in body fluids can be determined by daily weighing of


the patient

 In the emergency treatment of hyperkalaemia

 Under ECG monitoring, 40mls of 10% calcium gluconate can be


given slowly

 50% dextrose (100mls) and 20 units of insulin as bolus or


continuous infusion is temporary measure

 Calcium resonium resin is used in patients with hypertension

 Serum potassium of 6mmol is an indication for dialysis

 ECG changes seen in hyperkaelaemia include tenting of T waves


and flattening of the P waves.

3
 Acidosis in renal failure:

 It ameliorates the effects of hyperkalaemia

 It is best treated using hypertonic sodium bicarbonate

 Treatment by dialysis is effective

 If treated with hypertonic sodium bicarbonate may be complicated


by pulmonary oedema

 It is a form of respiratory acidosis

 The indications for dialysis includes:

 Blood urea >30mmol/l despite conservative measures or daily rise


of urea above 5.5mmol/l

 Rapid rise of potassium to levels > 6mmol

 Fall of serum bicarbonate about 2mmol daily or below 10mmol

 Severe uraemia with gastro intestinal bleeding and dehydration

 Pulmonary oedema

 In peritoneal dialysis

 The peritoneum acts as a semipermeable membrane

 Crystalloids diffuse from the circulation to the dialysis fluid in the


peritoneal cavity if they are absent in the fluid

 Compared to haemodialysis, peritoneal dialysis requires less


equipment

 Open insertion of peritoneal catheter is preferred in suspected


adhesions from previous surgery

 The dialysis fluid may be hypertonic 6% glucose and 1000mls is


adequate

 Contra-indication to peritoneal dialysis include:

 Wound dehiscence

 Extensive infections of the anterior abdominal wall

4
 Polycystic kidneys

 Recent laparotomy

 Extensive burns of anterior abdominal wall

 In peritoneal dialysis:

 There may be protein loss into the dialysate

 Bladder may perforate leading to peritonitis

 Closed method of catheter insertion is preferred in an unconscious


patient

 The peritoneal catheter need not be secured as it needs to be


changed frequently

 Radio-opaque peritoneal catheter line is essential for radiological


demonstration

 Haemodialysis:

 Venous canulation can be inserted into the internal jugular vein

 Arteriovenous shunt may involve the use of the radial artery

 Heparin use is contraindicated in all situations as it may cause


excessive bleeding

 The patient’s blood is passed on the other side of cellophane


membrane in the same directions

 In emergency, haemofiltration using venous canulation can be used

 Disequilibrium syndrome:

 It follows rapid dialysis

 It is due to lagging of urea and bicarbonate in the brain behind


those in the blood

 Clinical presentation includes headaches, vomiting, fits and


unconsciousness

 It is treated by the administration of barbiturates orally

5
 Prevention is by slow but short periods of dialysis

 In the diuretic phase of acute renal failure:

 The urine output increases to over 2000mls/24hrs

 There is loss of sodium and gain in potassium

 May be observed as the first presentation by a patient

 Vigorous rehydration is essential

 Frequent measurement of body weight, blood urea, and creatinine


levels are needed for management

 The survival results after acute renal failure from tubular necrosis
is worse in:

 Surgical patients

 Non-surgical patients

 ICU-patients

 Patients less than 60yrs

 Cases due to vascular surgery

 Acute obstructive uropathy:

 The definition of the cause may require ureteropyelography

 Percutaneous nephrostomy is required in prostatic obstruction

 Emergency corrective surgery is mandatory

 In the presence of inoperable malignant lesions, ureterostomy is


contra-indicated

 Diuretic phase is not a feature

 Anaesthesia in renal failure:

 Acute renal failure can be prevented by maintenance of adequate


circulating blood volume

 Mannitol and frusemide to induce diuresis is required in short

6
procedures

 In established acute renal failure, digoxin, pethidine and morphine


are best avoided

 Sudden cardiac arrest is due to hypokalaemia and acidosis

 Fresh blood is required for transfusion ,if large quantities are


required

 Chronic renal failure:

 Over 90% of renal tissue is usually lost for nitrogenous waste


products to be retained

 Progress of renal failure may be accelerated by recurrent urinary


tract infection and hypertension

 Condition may be symptomless in early stages

 It can lead to epileptic attacks

 A low protein diet of 0.5g/kg body weight may be required.

 Haematuria:

 It is the abnormal presence of >3 RBCs in spun urine

 The presence of proteinuria as well suggests glomerular disease

 Red blood cells are absent in the centrifuged urine in chemical


haematuria

 It is usually due schistosomiasis in tropical countries

 Haematuria in sickle cell disease is commonly as a result of


papillary necrosis

 Terminal haematuria of recent onset in a 56 year old man is most


likely

due to:

 Vesical schistosomiasis

 Bladder calculi

7
 Renal cell Carcimona

 Prostatic calculi

 Urinary tract infection

 In the investigations of haematuria:

 Sterile pyuria may suggest genito-urinary tuberculosis

 Diverticulitis of bladder diverticulum may cause painful haematuria

 Haematuria from BPH is due to prostatic varices

 Haematuria from bladder diverticulum can be due to neoplasia

 Cystoscopy is of no value in a suspected upper urinary tract lesion

 The causes of haematospermia include:

 Prostatitis

 Schistosomiasis

 TRUS biopsy of prostate gland

 Congested urethra from sexual excess

 Urethritis

 In the management of haematospermia:

 Transrectal ultrasound helps detect lesion of the seminal vesicles

 VDRL is be indicated

 Urethrocystoscopy is required

 A five day course of antibiotics is usually adequate in the presence


of urethritis

 Urethral dilatation is contraindicated if the cause is a urethral


stricture

8
CHAPTER 44
ACUTE RENAL FAILURE, HAEMATURIA, HAEMATOSPERMIA
ANSWERS

9
 A,C,D,E 25. A, B, C, D

 A,B,C,D 26. A, B, C, D, E

 A,B,C,D 27. A, B, C

 A,B,D,E

 A,B,C,D

 A,B,C,E

 A,B,D

 B,D,E

 C,E

 A,B,C,D,E

 C,D

 A,B,C,D,E

 A,B,C,D,E

 A,B,D,E

 A,B,E

 A,B,E

 A,B,C,E

 C,E

 A,C,E

 A

10
 A,C,E

 B,C,D,E

 A,B,C,E

 B,E

11
CHAPTER 46

BLADDER, URETHRA AND PENIS

 The bladder
  When distended is separated from the lower anterior abdominal wall 
by peritoneum
 Has a well‐defined internal sphincter at its orifice
 Orifice is closed by active contraction of the internal sphincter during 
filling
 Epithelium is transitional cell
 Sensory nerves run chiefly through the sympathetic and end in T9 to 
L2

 The sphincter urethrae (external sphincter):
  Surrounds the membranous urethra
 Lies above the upper layer of the urogenital diaphragm
 Has smooth muscle fibres
 Is under voluntary control for interrupting or withholding  urine flow 
for short periods
 Is innervated by the pelvic nerves which end in S2, 3, 4

 When the volume of urine in the bladder is:
  10ml the internal pressure is 5‐10cm of water
 300ml  bladder  fullness  is  appreciated  and  there  is  a  desire  to 
micturate
 Over  450ml  bladder  contractions  become  regular  and  powerful  with 
an urgent desire to void
 500 ml the contractions are continuous and very strong and the desire 
to void continues
 700‐800 ml the contraction is painful

 In disorders of micturition:
  The  inhibited  bladder  has  a  small  capacity  and  throughout  filling 
there are brief irregular contractions
 The  commonest  cause  of  bed‐wetting  in  children  is  the  uninhibited 
bladder
 Spinal cord transection causes permanent loss of voiding reflex
 Cauda  equine  tumours  cause  permanent  bladder  flaccidiy  and 
distension.
 All  reflex  contactions  of  the  bladder  are  abolished  and  distension  is 
not appreciated.

 In extrophy of the bladder

1
  The rami of the symphysis pubis are close together
 Coitus is not possible in both sexes because of the associated genital 
anomalies
 Closure of the bladder is the operation of choice
 The incidence of carcinoma is higher than in the normal bladder
 Most of the untreated patients die of pyelonephritis before 10years

 Urachus may give rise to:
  Umbilical sinus
 Bladder diverticulum
 A cyst
 Vesico‐umbilical fistula
 Bladder neck obstruction

A  45‐year  old  woman  complains  of  frequency,  dysuria,  urgency,  mild 


haematuria, suprapubic discomfort and fever for 2 days.

 The most likely clinical diagnosis is:
  Carcinoma of the bladder
 Pyelonephritis
 Cystitis
 Salpingitis
 Urethritis

 The MOST useful diagnostic investigation is:
  Urinalysis
 Culture of midstream urine and sensitivity test
 Urethral smear for microscopy and culture
 Cervical or high vaginal swab for microscopy
 Cystoscopy

 Bacterial cystitis
  Is commoner in women because of their short urethra
 Is always associated with a predisposing factor
 Is most commonly caused by staph. Aureus
 Most commonly affects the trigone and bladder base
 Is initially best treated by oexytetracycline while sensitivity  tests are 
awaited

 Interstitial cystitis:
  Is  a  common  chronic  inflammatory  condition  seen  in  middle‐aged 
women subjected to nervous strain
 Causes  deposition  of  fibrous  tissue  in  the  muscle  bundles, 

2
degeneration of muscle bundles and diminished bladder capacity
 Presents with progressive severe frequency, nocturia and suprapubic 
relieved by micturition
 Usually yields Esch. Coli or other gram negative organisms on culture 
of the urine
 Responds well to antibiotics and steroids.

 Parasitic diseases of the bladder include:
  Entamoeba histolytica
 Schistosoma japonicum
 Trichomonas vaginalis
 Onchocerca volvulus
 Taenia echinococcus

 The  following  may  be  seen  at  cystoscopy  of  a  patient  with  urinary 
schistosomiasis:
  Tubercles
 Carcinoma
 Ulcers
 Papilloma
 Diverticula

 Common complications of schistosomiasis of the bladder include: 
  Atonic distended bladder from chronic outlet obstruction
 Vesical calculus
 Pyelonephritis
 Recurrent acute epididymo‐orchitis
 Infertility

 The commonest type of bladder diverticulum:
  Consists of mucosa surrounded by muscle fibres
 Originates from urachus
 Empties  easily into the bladder
 May become larger than the bladder
 May like other types of diverticula rarely become malignant

 Bladder cancer:
  Is the commonest G.U. neoplasm
 Is more common in blacks than in Caucasians
 In Africa has a high incidence in Southern Malawi and Coastal Kenya
 Has a M/F ratio of 4‐5/1
 Has the highest incidence in the  41‐ 50 age group

3
 Bladder cancer is associated with:
  Smoking
 Asbestos industry
 Dye, textile or mining industry
 Urinary schistosomiasis
 No obvious cause in most patients

 Most bladder cancers:
  Are sessile
 Are in the bladder base
 Are squamous cell in type
 Are multiple
 Have a tendency to recur

 On  bimanual  examination  under  G.A.  of  a  60‐year  old  man  with 
suspected carcinoma of the bladder, a movable 2 cm mass with rubbery 
consistency is found.  The lateral ligaments are free
A  .    The  tumour  is  likely  to  involve  only  the  submucosa  or  superficial 
muscular layer
B. The clinical stage is T2
C. The U.I.C.C. stage is T2a
D.  Lymph node involvement is not likely to have taken place
E. The prognosis should be good

 In the treatment of transitional carcinoma of the bladder:
  AT1  tumour  of  the  base  is  best  treated  by  transurethral  endoscopic 
diathermy resection, but the recurrence rate is high
 For    severe  widespread  carcinoma  in  situ  of  the  urothelium, 
intravesical chemotherapy with thiotepa every 3 months is advocated 
 The U.I.C.C. stage is T2a well‐differentiated low grade tumour at the 
base, radical cystectomy is indicated
 For  a  T3  tumour,  a  combination  of  pre‐operative  radiotherapy  and 
radical cystectomy gives a survival of 40% in 5 years
 For  a  T4  tumour,  systemic  chemotherapy  gives  better  results  than 
radiotherapy

 A  patient  who  has  been  treated  for  cancer  of  the  bladder  should  be 
followed up for:
  6 months
 1 year
 5 years
 10 years
 The rest of his or her life

4
 In the prognosis of carcinoma of the bladder the 5‐year survival:
  Untreated patients is  5%
 AT1 tumour with local diathermy resection is about 75%
 AT2a tumour with partial cystectomy is 50%
 A T2b tumour with radical cystectomy is 40%
 T4 tumour with radiotherapy is nil

 The fistula is
  High vesico‐vaginal
 Low vesico‐vaginal
 Urtero‐vaginal
 Urethro‐vaginal
 Vesico‐uterine

 The commonest cause of retention of urine in adult males in West Africa 
is:
  Urethral stricture
 Bening prostatic hypertrophy
 Prostatic carcinoma
 Acute urthritis
 Paraphimosis

 Retention of urine:
  If painless is likely to be chronic
 If acute is associated with previous dribbling
 May be caused at all ages in both sexes by bladder neck obstruction
 If associated with haematuria in an old man is most likely caused by 
bladder cancer
 If sudden during micturition and associated with pain at the tip of the 
penis or perineum is most probably caused by urethral stricture

 In the anatomy of the male urethra the:
  Membranous urethra is the most fixed and narrowest part
 External urethral sphincter surrounds the bulbar urethra
 Least dilatable part is the external meatus 
 Epithelium  is  stratified  or  pseudo‐stratified  in  the  glans  and 
transitional proximal to the glans
 Lymphatics drain into the inguinal nodes

 Congential stenosis of the external urethral meatus:
  Is rare
 Occurs only in males
 May cause bloodyspotting and crusting of the meatus

5
 May cause renal failure
 Is treated by regular urethral dilatation

 Epispadias:
  Is a congenital deformity in which the urethra opens on the dorsum 
of the penis proximal to the glands
 Is caused by failure of union of the anterior and posterior walls of the 
urethra
 Does not occur in females
 Is associated with ventral curvature of the penis which is often short
 Makes coitus impossible in most patients

 Hypospadias:
  Is not associated with urinary incontinence
 Is commoner in females
 If perineal, the male external genitalia resembles that of the female
 If  penoscrotal  or  perineal  is  associated  with  a  miniature  penis  with 
marked dorsal curvature
 If glandular does not require any treatment

A 2‐year old boy has been having difficulty of micturition and dribbling for 
over  a  year.    On  examination,  he  is  cachectic  and  a  distended  bladder  is 
palpable.  A urethral catheter is passed without difficulty.

 The most likely clinical diagnosis is:
  Stenosis of the meatus
 Congenital non‐meatal urethral stricture 
 Phimosis
 Posterior urethral valves
 Bladder neck obstruction

 The investigations that will establish the diagnosis include:
  Excretory urography
 Micturating cystourethrogram
 Urethrogram
 Urethrocystoscopy
 Blood urea and serum electrolytes

 Possible complications include:
  Urinary fistulae
 Bladder diverticula
 Urinary tract infection
 Hypertension

6
 Unilateral hydronephrosis

 Treatment will include:
  Suprapubic cystostomy
 Urethral dilatation
 Urethroplasty
 Loop ureterostomy
 Meatotomy

A 25‐year old man in Singapore develops a thick yellow urethral discharge 
and  burning  on  micturition  5  days  after  sexual  intercourse  with  a  casual 
companion

 The organism(s) likely to be responsible is (are):
  Chlamydia trachomatis
 N. gonorrhoeae
 Candida albicans 
 T. vaginalis
 Esch. Coli

 The most useful investigation is:
  Culture of urethral discharge
 Urine culture
 Immunofluorescent antibody test
 Microscopic  examination  of  a  urethral  smeal  (gram  and  methylene 
blue stains)
 VDRL

 The treatment of first choice is
  Ceftriazone 250 mg IM
 Procaine penicillin G 3.8m
 Azithromycin 500 mg daily for 3 days
 Doxycycline 100mg bd for 7 dyas
 Oxytetracycline 500 mg qds for 4 days

  Symptoms  persist  in  spite  of  treatment.    Drugs  to  be  recommended 
include:
  Ciprofloxacin 500 mg b.d. for 7 days
 Chloramphenicol 2 g daily for 5 days
 Azithromycin 500 mg daily for 3 days
 Doxycycline 100 mg bd for 7 days
 Erythromycin 500 mg qds for 10 days

7
 Symptoms subside after 48h.  But 24 days after the intercourse, a thin 
watery  discharge  most  marked  first  thing  in  the  morning  recurs.    The 
patient claims he has not had any sexual contact since treatment.  The 
discharge is due to:
  Re‐infection
 Chroniciy of the original infection
 Prostatorrhoea or spermatorrhoea
 Primary non‐specific urethritis
 Secondary non‐specific urethritis

 Useful investigations include:
  Two glass urine test
 Microscopy of urethral smear for polymorphs
 Microscopy of gram‐stained urethral smear
 VDRL
 Urinalysis

 The most likely organism is:
  Gonorrhoea
 Pencillinase producing N. gonorrhoeae
 Esch. Coli
 Cylamydia trachomatis
 T‐strain mycoplasma

 The drug of first choice is:
  Doxycycline 100mg bd for 14 days
 Tetracycline 250mg qds for 14‐21days
 Procaine penicillin G 4.8m Im
 Erythromycin  500mg qds for 7 days
 Sulphasoxazole 2 g daily for 5 days

 The most effective drug for pharyngeal gonococcal infection is:
  Ampicillin
 Amoxillin
 Pivampicillin
 Ceftriazone
 Spectinomycin

 Complications of gonococcal urethritis include:
  Tenosynovitis
 Meningitis
 Encephalitis
 Arthritis

8
 Pyelonephritis

 Causes of secondary non‐gonococcal urethritis include:
  Urethral stricture
 Urethral tumour
 Urethral catheterization
 Cystitis
 Sexual intercourse

 Urethral stricture may be caused by:
  Primary non‐specific urethritis
 Urethral instrumentation
 Urethral calculus
 Vaginal douching
 Prostatitis

 Urethral stricture
  Following urethritis is formed within a year but takes on an average 
20 years to become symptomatic
 Following trauma becomes symptomatic in two months
 Following  urethritis is commonest in the penile urethra
 May lead to extravasation of urine or be associated with carcinoma of 
the urethra
 Clinical cases are all of acquired origin

 The most reliable investigation for the diagnosis of urethral stricture is:
  Urethrogram and voiding cystourthrogram
 Excretory urography
 Passage of urethral metal bougie (22 Ch)
 Urethroscopy
 Cystoscopy

 The  I.V.P.  of  a  55  year  old  West  African  male  shows  bilateral 
hydronephrosis and a high residual urine.  Diagnosis may be:
  Tuberculosis of the bladder
 Benign prostatic hypertrophy
 Carcinoma of the urethra
 Urinary schistosomiasis
 Carcinoma on the lateral wall o the bladder

 In the management of urethral stricture, dilatation of the urethra:
  Is curative
 Is performed for passable incomplete strictures

9
 For  tight  strictures  is  performed  with  small  size  metal  bougies  or 
filiforms and flollowers
 Is repeated at increasing intervals and stopped after 2 years
 May cause septic shock

 Internal urethrotomy:
  Is splitting of a stricture under direct vision
 Is suitable for long uncomplicated impassable strictures
 Requires  subsequent  splinting  of  the  urethra  with  an  indwelling 
catheter for 1 week
 Gives permanent cure
 Does not require subsequent dilatation

 Urethroplasty:
  Is indicated in complete strictures
 Is curative in most patients
 Is always done in 2 stages
 May be complicated by fistulae … restenosis
 May result in the formation of a hair‐ball in the urethra

 In the initial management of the complications of urethral stricture:
  Retention of urine may be relieved by suprapubic tap using a needle.
 Extravasation of urine is managed by continuous bladder drainage via 
a urethra catheter.
 Fistulae require urgent urethroplasty
 Uraemia necessitates suprapublic cystostomy and continuous bladder 
drainage 
 Dilatation of the urethra is never done.

 Primary carcinoma of the urethra:
  Is most commonly transitional cell
 Is best diagnosed by urethrogram
 If  confined  to  the  distal  half  of  the  penis  is  treated  by  partial 
amputation of the penis
 In women is treated, if early, by localized interstitial radiotherapy
 Has a poor prognosis even in early cases

A  30‐year  old  woman  complains  of  dyspareunia,  dribbling  of  urine  after 
micturition and a swelling in the anterior vaginal wall.

 The most likely clinical diagnosis is:
  Urethral caruncle
 Urethro‐vaginal fistula

10
 Carcinoma of the urethra
 Diverticulum of the urethra
 Prolapse of the urethra

 Senile urethritis:
  Is caused by Esch. Coli
 Is associated with a dry pale vagina
 Causes dyspareunia, frequency, vaginal and vulval irritation
 Is diagnosed by urine culture and urinalysis
 Is treated with sulphonamides or appropriate antibiotic

 In the anatomy of the penis
  The  lymphatics  of  the  penile  skin  and  prepuce  drain  into  the 
superficial inguinal nodes
 The  lymphatics  of  the  glans  and  corpora  drain  into  the  internal  iliac 
nodes
 The nerve supply is from the genitor‐femoral nerve
 The  corpora  consist  of  spongy  saccules  into  which  arterioles  open 
directly
 Venous  return  is  to  the  prostatic  venous  plexus,  internal  pudendal 
and saphenous vein

 Paraphimosis:
  Is constriction of the prepuce
 May be precipitated by sexual intercourse
 Presents as pain and oedematous swelling of the glans and  prepuce
 May lead to gangrene of the prepuce and glans
 Is treated by circumcision

A  35‐year  old  man  who  denies  sexual  intercourse  in  the  past  6  weeks 
presents  with  a  punched  out  painless  ulcer  about  1cm  in  diameter  on  the 
glans.  The base of the ulcer is hard.  The inguinal nodes on both sides are 
palpable, discrete, non‐tender and hard.

 The most likely clinical diagnosis is:
  Lymphogranuloma venereum
 Chancroid (soft chancre)
 Syphilitic chancre
 Epithelioma
 Non‐specific ulcer

 The most useful investigation is:
  VDRL  test

11
 Biopsy
 Microscopy: Fluorescent antibody dark‐field examination
 Frei’s test
 Treponema immobilization test

 Treatment is:
  Excision of the ulcer
 Partial amputation of the penis and radiotherapy of the lymph nodes
 Procaine penicillin 1 mega unit daily for 10‐14days
 Sulphadiazine 1g qds for 10‐14days
 Daily (eusol dressing)

  Chancroid (soft chancre) is:
  Caused by Haemophilus ducreyi
 Always solitary and starts as a painful pustule
 Reliably diagnosed by serological test
 Common in the temperate and tropical regions
 Treated with a sulphonamide for 5  days

 Granuloma inguinale:
  Is  a  chronic  infection  of  the  skin  and  subcutaneous  tissues  of  the 
genitalia, perineum or inguinal regions
 Is caused by a virus
 Starts as a pustule or papule which ulcerates
 May lead to lymphoedema and urethral stricture
 Is best treated with procaine penicillin 1 mega daily for 14‐21 days

 Erosive and gangrenous balanitis:
  Is caused by Borelia vincenti
 Presents  as  erosions  or  painless  irregularly  shaped  usually  multiple 
ulcers
 Occurs in the circumcised and uncircumcised
 Always spreads to the inguinal nodes
 Is treated with penicillin or oxytetracycline for 7 days.

 Herpes genitalis:
  Is caused by herpes simplex virus type II
 Starts  as  vesicles  which  rupture  to  form  small  round  painful  tender 
erosions or ulcers
 Is associated with painful enlarged inguinal lymph nodes
 Does not recur
 Is treated with oxytetracycline, chloramphenicol or ampicillin

12
 Plastic induration of the  penis (Peyronie’s disease):
  Caused by calcification of the corpora cavernosa
 Usually causes dorsal chordee
 Is felt as well‐demarcated raised indurated plaque in the shaft
 Is confirmed by biopsy
 Is treated satisfactorily with vitamin E or low dosage radiotherapy

 Priapism:
  Is prolonged erection of the penis
 Is most commonly caused in West Africa by SS disease
 Requires a full blood count
 May result in impotence
 Is best treated  by a venous shunting operation.

 Carcinoma of the penis is common in:
  Ethiopia
 Uganda
 Nigeria
 Kenya
 China

 The incidence of carcinoma of the penis is:
  Nil in communities where circumcision is done before the age of 12 
years.
 Is high among the uncircumcised of Zambia
 Is high among uncircumcised Hindus
 Is low among the uncircumcised in Mozambique
 Is highest in the fifth decade

 Carcinoma of the penis:
  Occurs most commonly on the coronal sulcus
 Is histologically transitional cell
 Spreads early to the inguinal lymph nodes
 Spreads rarely via the blood stream
 Is of low grade malignancy in most patients

 Carcinoma of the penis presents as:
  Purulent discharge from the prepuce
 Scalding
 An ulcer with punched out edges
 A lump under the prepuce
 Urethral fistula

13
 In the treatment of carcinoma of the penis:
  A small stage 1 exophytic tumour on the coronal sulcus glans in a 40‐
year old man is treated by partial amputation of the penis
 A small lesion localized to the prepuce in a 60‐year old man is treated 
by circumcision
 A  stage  II  tumour  involving  the  distal  shaft  in  a  50‐year  old  man  is 
treated by total amputation
 Enlarged  mobile  inguinal  lymph  nodes  persisting  3  weeks  after 
treatment  of  the  primary  are  preferably  treated  by  bilateral  ilio‐
inguinal block dissection
 Bleomycin and methotrexate are effective in about 40 % of patients

 Indications for suprapubic cystostomy include:
  Acute retention of urine due to urethral stricture
 Periurethal abscesses
 Rupture of the urethra
 Extravasation of  urine
 Contracted bladder due to tuberculosis

 Indications for uretero‐ileal‐cutaneous diversion of urine (ileal conduit) 
include:
  Vesico‐vaginal‐rectal fistula
 Contracted bladder due to schistosomiasis
 Ectopia vesicae
 Urethro‐cutaneous fistulae
 Uncomplicated spina bifida

 Complications of uretero‐sigmoidostomy include:
  Pyelonephritis
 Metabolic alkalosis
 Hypokalaemia
 Hyponatraema
 Renal lithiasis

 A plain X‐ray of the pelvis shows calcification in the bladder wall.  The 
cause may be:
  Tuberculosis
 Carcinoma
 Hyperparathyroidism
 Schistosomiasis
 Atheroscierosis of the vesical vessels

 Drugs effective against PPNG and non‐PPNG include:

14
  Erythromycin
 Ciprofloxacin
 Ceftriazone
 Ampicillin
 Ofloxacin

 Drugs  effective  against  gonococcal  and  non‐gonococcal  urethritis 


include:
  Trimethropim‐sulphamethoxazole
 Cefoxitin
 Tetracycline
 Pivampicillin
 Erythromycin

77. The following are associated with extrophy of the bladder:
 Waddling gait

 Epigastric hernia

 Urinary incontinence

 Male to female ratio is 4 : 1

 Bladder malignancy

        78. The best time after birth to close extrophy of the bladder is:
 Within 72 hours

 Within one month

 Before one year of age

 At the time of puberty

 After puberty

79. The following tumour may be associated with bladder extrophy:
 Squamous cell carcinoma

 Transitional cell carcinoma

 Rhabdomyosarcoma

 Adenocarcinoma

15
 Leimyosarcoma

     80. The  incidence of bladder extrophy is up to:
 1 in 150,000

 1 in 100,000

 1 in 70,000

 1 in 50,000

 1 in 5,000

   81.  The treatment for a patent urachus is:
 Excision of the patent urachus and adjoining cuff of bladder

 Excision of a cuff of bladder

 Fulguration of the urachal tract

 Partial cystectomy

 Fulguration of the urachal tract and dome of the bladder

82. The commonest infecting organism in acute bacterial cystitis is:
 P. mirabilis

 Staphylococcus

 E. coli

 Pseudomonas

 Klebsiella

83. The following are features of interstitial cystitis:
 Commoner in females than males

 Occurs mostly in middle aged women

 It is accompanied by urinary infection

 Characterized by irritable bladder symptoms and pain 

 Symptoms are short‐lived

16
84. The best method for diagnosing interstitial cystitisis:
 IVU

 Urethral dilatation

 Cystoscopy and biopsy

 Micturating cystourethrogram

 CT scan 

85. The best treatment for pain relief in interstitial cystitis:
 Cystolysis

 Ileo‐cystoplasty

 Urinary diversion

 Cystodiathermy of Hunner’s ulcer

 Sympathectomy

86. Characteristics of  vesical schistosomiasis include: 
 Large bladder capacity 

 Haemorrhgic cystitis

 Initial haematuria

 Bilharzial tubercles

 Squamous metaplasia

87. The ideal urine sample for demonstrating eggs of S. haematobium is:
 24 hr urine

 12 hr urine

 Early morning urine

 Mid‐day urine

 Voided urine any time of day

88. The following is most commonly associated with urinary Schistosomiasis:

17
 Transitional cell carcinoma

 Rhabdomyosarcoma

 Squamous cell carcinoma

 Spindle cell sarcoma

 Adenocarcinoma

89. Complications of Bladder diverticulum include:
 UTI

 Calculus

 Tumour

 Bladder outlet obstruction

 Ureterocele

  90. The following are risk factors for developing bladder cancer:
 Exposure to aniline dyes

 Cigarette smoking

 Alcohol drinking

 Vesical schistosomiasis

 High dietary fat

91. The following is associated with increased incidence of bladder cancer:
 Bleomycin

 Cisplatinum

 Cyclophosphamide

 Vincristine 

 Actinomycin D

92. The following are risk factors for development of bladder cancer:
 Alcohol/Drinking 

18
 Cigarette smoking 

 Schistosomiasis

 Irradiation

 Aniline dyes

93. Adenocarcinoma of the bladder may result from the following:
 Cystitis glandularis

 Bladder extrophy

 Urachal remnant

 Misplaced prostatic glands

 Squamous metaplasia

94.  The  risk  of  transitional  cell  carcinoma  (TCC)  of  the  bladder  after  earlier      
development of TCC of the renal pelvis:
 < 5%

 5‐ 10%

 10 – 20%

 20 – 30%

 > 40%

95. Most common cancer of the urinary bladder is:
 Adenocarcinoma

 Rhabdomyosarcoma

 Transitional cell carcinoma

 Leimyosarcoma

 Squamous cell carcinoma

96. Presenting symptoms of bladder cancer:
 Weight loss

19
 Necroturia

 Strangury

 Frequency

 Initial Haematuria

97.  Most essential diagnostic investigation for bladder cancer:
 CT scan

 Patient symptoms

 Intravenous urogram

 Cystoscopy and biopsy

 Urine cytology

98. The following are useful for the staging of bladder cancer:
 Cystoscopy

 Urine cytology 

 CT Scan

 Bimanual examination

  Abdominopelvic  Ultrasonography

99.  As  a  guide  to  prognosis  and  therapy,  the  following  is  the  most  important 
distinction between levels of invasiveness of urothelial tumour:
 Between epithelium and lamina propria

 Between superficial and deep lamina propria

 Between lamina propria and muscle layer

 Between superficial and deep muscle layers

 Between muscle layer and perivesical fat

20
100. Bladder cancer Invasion into superficial muscle corresponds to this stage:
 T1

 T2a

 T2b

 T3a

 T3b

101. Diagnostic finding on an IVU, suggestive of bladder cancer is:
 Non‐visualization of the kidney

 Filling defect in bladder

  Hydro‐uretero‐nephrosis

 Significant residual urine

 Bladder diverticulum

102. Carcinoma in situ of the bladder has the following characteristics:
 Cystoscopic presence of tumour

 Positive urine cytology

 Irritative bladder symptoms

 Best diagnosed by multiple bladder biopsies

 Cystectomy is always the treatment of choice

103. Treatment for Ta and T1 bladder cancer includes:
 Radiotherapy

 TUR

 Intravesical chemotherapy

 Cystodiathermy

 Cystectomy

104. Indications for intravesical chemotherapy for bladder cancer include:

21
 Non‐muscle invasive transitional cell carcinoma

 Single/large tumour

 Multiple tumours

 High grade tumours

 Rapid tumour recurrence

105. Treatment of choice for a well differentiated carcinoma in situ of the bladder
Is:
 Partial cystectomy

 Total cystectomy

 Transurethral resection

 Intravesical BCG therapy

 Radiotherapy 

106. Leading causes of vesicovaginal fistula in developing countries include:
 Irradiation

 Obstructed labour

  Gynaecological surgery

 Malignancy

 Trauma

107. Investigations for the evaluation of vesicovaginal fistula:
 Micturating cystourethrogram

 EUA

 Dye test

 Urethrocystostocopy

 IVU

22
108. Characteristics of chronic urinary retention include:
 Painless bladder distension

 Dribbling micturation

 Overflow incontinence

 Commonly results from a neuropathy

 Bladder volume is similar to that of acute retention

109. The length of the adult male urethra is:
 10.5 cm

 15.5 cm

 20.5 cm

 25.5 cm

 30.5 cm

110. The incidence of male epispadias is:
 1 in 55,000

 1 in 75,000

 1 in 95,000

 1 in 115.000

 1 in 135,000

111. Urinary incontinence from epispadias has the following features:
 Occurs in the penile type

 Occurs in the complete type

 It is stress incontinence

 It is total incontinence

 Treatment requires bladder neck reconstruction

112. The following type of hypospadias may present with ambiguous genitalia:

23
 Penoscrotal

 Perineal

 Penile

 Subcoronal 

 Glandular

113. The estimated incidence of hypospadias is:
 1 in 100

 1 in 200

 1 in 300

 1 in 400

 1 in 500

114. The following anomalies may be associated with hypospadias:
 Imperforate anus

 Undescended testis  

 Myelomeningocele

 Inguinal hernia

 Ambiguous genitalia

115.  Technique for the repair of glandular hypospadias:
 Parameatal based flap

 Transverse preputial skin flap

 Tranverse distal penile skin flap

 Inner preputial skin flap

 Meatal advancement and glanuloplasty

116. Diagnostic investigsations for posterior urethral valves:

24
 Urethroscopy

 Retrograde urethrogram

 Ultrasonography

 CT scan

 Micturating cystourethrogram

117.   Treatment of choice for posterior urethral valves:
 Continuous bladder catheterization

 Suprapubic cystostomy

 Vesicostomy

 Endoscopic valve ablation

 Open excision of valves

118. Average incubation period for gonococcal infections:
 1 – 2 days

 3 – 4 days

 5 – 6 days

 7 ‐ 8 days

  9 – 10 days

119.  Features of gonococcal urethritis in the male include:
 Profuse purulent discharge

 Scanty watery discharge

 Dysuria

 Thickened and tender urethra

 Normal external urethral meatus

120. Complications of gonococcal urithritis include:

25
 Reiter’s syndrome 

 Urethral stricture

  Prostatitis

 Urethral cancer

 Pelvic inflammatory disease

121. Characteristics of genital ulcers of lymphogranuloma venereum include:
 Small and round

 Painful

 Usually multiple

 Sloping edges

 Yellowish floor 

122.  Extravasated  urine  from  anterior  urethral  injury  is  bounded  initially  by  the 
following structures:
 Dartos fascia

 Camper’s fascia

 Colles’ fascia

 Bucks fascia

 Scarpa’s fascia

123. Traumatic rupture of the bulbous urethra is associated with the following:
 May result from a straddle injury

 Produces superficial extravasation of urine

 Produces deep extravasation of urine

 Produces urethral bleeding

 Digital rectal examination shows a high riding prostate

26
124. Investigations for diagnosing urethral stricture include:
 Uroflowmetry

 Ultrasonography 

 Retrograde urethrogram 

 Urethroscopy

 Micturating cystourethrogram

125. Common techniques for treating urethral strictures include:
 Internal Urethrotomy

 Dilatation

 Urethral stent implantation

 One‐stage urethroplasty 

 Two‐stage urethroplasty

126.  Aetiological factors for cancer of the penis include:
 Leukoplakia

 Granuloma inguinale

 Phimosis

 Chancroid

  Balanitis xerotica obliterans

127. Penile cancer occurs most frequently at the following sites:
 Glans

 Meatus

 Prepuce

 Coronal sulcus

 Shaft

27
128.  T2 penile cancer has the following level of invation:
 Invasion of subepithelial connective tissue

 Invasion of urethra or prostate

 Invasion of the corpora

 Invasion of the scrotum

 Invasion of the testis

129.  Commonest site for metastasis from carcinoma of the penis is:
 Lung

 Lymph node

 Liver

 Bone

 Brain

130.  The following are prognostic factors for cancer of the penis:
 Site of the tumour

  Grade of the tumour

 Presence of inguinal metastasis

 Age of the patient

 Stage of the tumour

131. Common aetiological factors of priapism:
 Leukaemia

 Thrombocythaemia

 Pharmacologically induced

 Trauma

 Sickle cell disease 

132. Success rate (retained potency) after treatment of major priapism:

28
 < 10%

 10 – 20%

 20 – 30%

 30 ‐40%

 40 ‐50%

133. Treatment for a small T1 preputial cancer with no lymphadenopathy in a
young man is: 
 Radiotherapy

 Circumcision 

 Electrocautery

 Partial penectomy

 Total penectomy

134.  The origin of penile inervation include:
 Genito‐femoral nerve

 T10 – L2 spinal segments

 The dorsal nerve

 The cavernous nerve

 S2 – S4 spinal segments 

135. The action of nitric oxide within the penile smooth muscle cell involves:
 Activation of adenyl cyclase and elevation of cyclic AMP level

 Activation of phosphodiesterase type 4

 Opening of calcium channels

 Activation of guanylyl cyclase and elevation of cyclic GMP level

 Closure of potassium channels

29
136. Venous flow reduction during normal penile erection is due to:
 Active constriction of the superficial and deep dorsal veins

 Compression of the subtunical venules and emissary veins 

 Active contraction of emissary veins

 Opening of penile arteriovenous shunts

 Relaxation of the ischiocavernous muscle 

137. Normal penile erection involves the following processes:
 Arterial dilatation and venous constriction

 Relaxation of the ischiocavernous muscle

 Arterial dilatation, venous compression and sinusoidal relaxation

 Contraction of the cavernous smooth muscle

 Filling and expansion of the sinusoidal spaces

138. Best treatment for low‐flow priapism that presents within 12hours of onset 
is:
 Aspiration of the corpora

 Intravenous fluids plus analgesics

 Aspiration  of  corpora  and  intracavernosal  injection  of  alpha  adrenergic 


agonist

 Application of cold packs

 Surgical shunt

139. Post‐radical prostatectomy Impotence is often due to injury to:
 Dorsal nerve of the penis

 Cavernous nerve

 Genitofemoral nerve

 Sympathetic ganglion

30
 Ilioinguinal nerve

140. Fluid and electrolyte imbalance in ureterosigmoidostomy is characterized by:
 Hyperkalaemia with hyperchloraemia

 Hypokalaemia with hyperchloraemia

 Hypomagnesaemia with acidodis

 Hyperkalaemia with hypochloraemia

 Hyperkalaemia with acidosis

141. Cancer occurring in urinary intestinal diversion is most likely to occur in:
 Ileal conduit

 Colonic conduit

 Augmentation cystoplasty

 Ureterosigmoidostomy

 Orthotopic neo‐bladder

Answers

 D
 A, D
 A, C, E
 A, B
 D
 A, B, C, D
 C
  B
 A, D
 B, C
 A, C, E
 A, B, C, D
 B, C
 D, E
 C, D
 A, C, D, E
 B, E

31
 B, D
 A, D
 E
 B, C, E
 C
 B
 A, C
 C, D
 C, D
 A, E
 A, C, E
 D
 B, D
 B, C
 A, D
 B
 D
 A
 A, C, D, E
 D
 A, B, C
 D
 A
 D
 A, B, D
 A, B, C, D
 A, B, C, D
 A, B, D
 D
 B, C
 B, E
 A, C
 A, B, D, E
 A, D
 C, D
 D
 B, C
 A, D, E
 B, C, D
 C, 
 C
 C
 A
 A, C, D
 D, E
 A, B
 B, C

32
 C, D, E
 B, E
 C, D
 D, E
 A, D, E
 B, D
 A, B, C, D
 A, B, C
 A, C, E
 A, D
 B, C, E
 C, E

77. A, C, E 102. B, C, D


78. A 103. B, C, D
79. D 104. A, C, D, E
80. D 105. D
81. A 106. B, C
82. C 107. B, C, D, E
83. A, B, D 108. A, B, C
84. C 109. C
85. E 110. C
86. B, D, E 111. B, D, E
87. D 112. B
88. E 113. C
89. A, B, C, D 114. B, D, E
90. A, B, D 115. E
91. C 116. A, E
92. B, C, D, E 117. D
93. A, B, C, D 118. C
94. E 119. A, C, D
95. C 120. B, C, E
96. A, B, D 121. A, D, E
97. D 122. A, C, E
98. C, D, E 123. A,B,D
99. C 124. B, C, D
100. B 125. A, B, D, E
101. B 126. A, C, E

127. A, C, D, E 137. C


128. C 138. C
129. B 139. B

33
130. B, C, E 140. B, C
131. C, E 141. D
132. E
133. B
134. B, C, D, E
135. D
136. B

                                                                                                                                                                 

34
CHAPTER 48

SCROTUM, TESTIS AND EPIDIDYMIS

 Extravasated urine from ruptured urethral bulb gravitates into the:
  Scrotum
 Penis
 Femoral triangle
 Perineum
 Lower anterior abdominal wall

 In the anatomy of the testis and epididymis the:
  Lymphatics drain into the inguinal and para‐oartic lymph nodes
 Testicular veins drain into the inferior vena cava
 Nerve supply is from the sympathetic with a connection at T12
 Epididymis and testis may be completely invested with tunica vaginalis
 Epididymis may be anterior to the testis

 Essential for sperm metabolism and motility is:
  Glucose
 Cholesterol
 Acid phosphatase
 Fructose
 Ascorbic acid

 Prostatic secretion is rich in:
  Lactate dehydrogenase
 Alkaline Phosphatase
 Fibrinolysin
 Folic acid
 Calcium

 In the physioilogy of the male generative organs:
  The seminal vesicles secrete an acid viscous fluid
 A large part of the semen is formed by the epididymis
 The interstitial cells secrete androgens principally testosterone
 The cells of sertoli probably secrete androgens
 The testes are under the influence of pituitary FSH and interstitial cell 
stimulating hormone

 The incidence of imperfect descent of the testis is:
  About 30% in premature infants
 About 5% in full‐term infants

1
 In about 47% of patients on the left side
 In about 30% of patients on the right side
 In 20% of patients bilateral

 In undescended testis:
  The testis is normal at birth
 Retardation of its development begins at 5 years 
 Retardation of its development is probably due to an inherent defect and 
not to the temperature of the inguinal canal
 The interstitial cells atrophy if retardation of development occurs
 The cells of Sertoli are never affected 

 An ectopic testis:
  Is normal but will not develop unless it is brought into the scrotum long 
before puberty
 Has come out of the superficial inguinal ring
 Is  deflected  from  the  normal  path  of  descent  by  stenosis  at  the  neck  of 
the scrotum
 Lies most commonly in the femoral region
 Is commoner than undescended testis

 A retractile testis:
  Is  a  testis  that  is  held  up  at  the  superficial  inguinal    ring  and  has  not 
descended into the  scrotum
 Is retracted by contraction of an active cremaster muscle
 Enters the scrotum spontaneously
 Clinically can be ‘milked’ into the scrotum
 Is associated with an abnormal scrotum

 In unilateral undescended testis:
  The scrotum looks almost like a triangle with the apex pointed upwards
 The patient is usually obese
 The penis is small for the age
 A solid swelling is usually  palpable in the inguinal canal
 Attacks of pain in the groin may be the presenting symptom

 Inundescended testis:
  If  it  is  bilateral,  about  63%  of  those  who  have  prepubertal  orchdopexy 
become sterile
 Abou t 30% are sterile or subfertile if it is unilateral
 The quantity of semen is of normal volume if it is unilateral
 The sperm count is about half the normal average in most patients with 
unilateral descent

2
 If bilateral, 10% of patients are fertile

 The complications of maldescended testis include:
  Inguinal hernia
 Hydrocele of the cord
 Torsion
 Recurrent infection
 Malignancy

 In the operative treatment of  undescended testis:
  Orchidopexy  is  performed  ideally  between  8    and  10  years  to  achieve 
fertility
 Pre‐operative androgen therapy is beneficial
 If  the  undescent  is  bilateral,  one  side  is  done  first  and  the  other  six 
months later 
 Herniotomy is nearly always required
 Post‐operative complications include atrophy

 Indications  for  chorionic  gonadotrophin  therapy  in  maldescended    testis 


include:
  Small penis
 Obesity
 Ectopic testis
 Bilateral undescent
 Associated gynaecomastia at birth

 In maldescent presenting after puberty:
  Orchidectomy is performed in unilateral undescent
 Orchidopexy  is  done  on  one  side  and  orchidectomy  on  the  other  in 
bilateral undescent
 Orchidectomy is done in unilateral ectopic testis
 Chorionic gonadotropin therapy is beneficial
 Follow‐up throughout life must always be done irrespective of the type

 Organisms  associated  with  idiopathic  gangrene  of  the  scrotum  (Fournier’s 


gangrene) include:
  Micro‐aerophilic strep. Haemolyticus
 Staph. Aureus  and B. proteus
 Pseudomonas
 Fuso bacterium and Esch coli
 Cl. Welchii

 In idiopathic gangrene of the scrotum:

3
  The whole thickness of the scrotum, nearly always the anterior surface, 
sloughs off
 The testes are exposed and may at times be affected by the gangrenous 
process
 There is usually an associated difficulty with micturition
 The patient is usually quite  ill
 Suprapubic cystostomy leads to rapid recovery

 Acute epididymo‐orchitis may follows:
  Prostatectomy
 Urethral instrumentation
 Urinary tract infection
 Reflux of urine
 Excessive sexual intercourse 

 Organisms causing acute epidid ymo‐orchitis include:
  Esch. Coli
 Strep. Viridians
 S. Haematobium
 B. proteus
 Bacteroides

 In acute epididymo‐orchitis:
  There is always a history of urethritis
 The groin may be painful
 The epididymis is swollen and very tender
 Elevating the scrotum worsens the pain
 There is always an associated hydrocele

 The antibiotic to us in  acute epididymo‐orchitis before the sensitivity of the 
causative organism is known as:
  Ampicillin
 Chloramphenicol
 Erythromycin
 Amikacin
 Ciprofloxacin

 Recurrent  attacks  of  funiculitis  and  epididymo‐orchitis  which  subside 


spontaneously are suggestive of:
  Gonorrhoea
 Urinary tract infection
 Trichomonad infection
 Filariasis

4
 Prostatitis

 Tuberculous epididymo‐orchitis:
  Is usually blood‐borne
 Affects the body of the epididymis before the tail
 May be associated with frequency of micturition
 May present as anterior scrotal sinus
 Is often associated with a palpable indurated seminal vesicle

 Investigations in s uspected tuberculous epididymo‐orchitis include:
  Culture of early morning specimen of urine
 Exretion urography
 Cystoscopy
 Sputum culture
 Biopsy

 Torsion of the spermatic cord:
  Is caused by trauma to the testis
 Is usually in a clockwise direction in the right  testis and anti‐clockwise in 
the left 
 May have five to ten turns
 If unrelieved in 6 h may lead to gangrene and atrophy of the testis but not 
usually of the interstitial cells
 Is always intra‐vaginal

 Torsion of the spermatic cord:
  Is seen  from the neonatal period to about 40 years
 Occurs  mainly  between  the  neonatal  period  and  one  year  and  10‐=15  y 
ears
 Is  treated  by  orchidectomy  if  at  operation  the  viability  of  the  testis  is 
doubtful 
 Is  treated  by  fixation  of  the    testis  to  the  dartos  muscle  if  the  testis  at 
operation is viable
 Does not require exploration of the other  testis at operation.

 The clinical features of torsion of the spermatic cord include:
  Sudden onset of agonizing pain in the scrotum and groin which may be 
referred to the iliac fossa
 Vomiting in half the patients
 Frequency and dysuria
 Swollen, firm, very  tender and retracted test is
 Lessening of pain when the   testis is elevated

5
 In the prognosis of torsion of the spermatic cord:
  If exploration is done within 4 h the salvage rate is 100%
  If exploration is done at 10 h the salvage rate is about 50%
 If exploration is done after 24 h the salvage rate is almost nil
 About 70% of apparently viable  testes atrophy
 Only about 5% of apparently viable testes have a normal sperm count.

 A congenital hydrocele:
  Is collection of fluid in a processus vaginalis
 Is seen in all age groups
 Is smaller or empty in the morning
 Is distended by the end of the day
 May be associated with a direct inguinal hernia

 An infantile hydrocele:
  Is  collection  of  fluid  in  a  processus  vaginalis  a  segment  of  which  is 
obliterated 
 Causes a cystic swelling in the inguinal canal
 Is usually seen in infants and occasionally in adults
 May be associated with an indirect inguinal hernia
 May disappear spontaneously

 Primary vaginal hydrocele:
  Occurs in all age groups especially those between 10 and 20 years
 Results  from  deficient  drainage  of  fluid  in  the  tunica  vaginalis  by  the 
lymphatic vessels which may be blocked or hypoplastic
 Always  has a thin sac
 Is not always unilocular
 May contain calcareous deposits on its wall

 The fluid in primary vaginal hydrocele:
  Has a specific gravity of 1.022 to 1.025 
 Is brown in colour
 Contains cholesterol and albumin
 Contains sodium chloride, carbonates and fibrinogen
 Collects rapidly

 The fluid in secondary vaginal hydrocele:
  Collects slowly
 Is always serous or blood‐stained
 Is commonly an exadate from inflamed tunica vaginalis
 May be considerable, up to 500ml 
 Contains uric acid

6
 Secondary vaginal hydrocele may be associated with:
  Acute or tuberculous epididymo‐orchitis
 Filariasis
 Torsion of the spermatic cord
 Testicular tumours
 Lymphoedema of the scrotum

 Complications of primary vaginal hydrocele include:
  Infection
 Atrophy of  the testis
 Calculus formation
 Impotence
 Haematocele

 The  methods  of  managing  secondary  vaginl  hydrocele(not  due  to 


lymphoedema of the scrotum) include:
  Jaboulay’s operation
 No treatment
 Lord’s Operation
 Tapping
 Aspiration and injection of sodium tetradecyl sulphate

 A primary vaginal hydrocele in a 30‐year old man may be treated by:
  Hydrocelectomy
 Tapping
 Lord’s operation
 Hydrocele‐orchidectomy
 Jaboulay’s operation

 A cyst of the epididymis is:
  A retention cyst of the seminiferous tubules
 Lobular and may be up to 10cm in diameter
 Above and behind the testis
 Usually multi‐locular and the surface is bosselated
 Left  alone if it is small and aspirated if it is large

 Varicocele is:
  Dilatation, elongation and tortuosity of the spermatic vein
 On the right side in about 90% of patients
 Seen commonly in young unmarried men
 Best felt with the patient lying down 
 Treated by ligation and division of the spermatic vein at the internal ring 

7
if there is persistent pain or subfertility

 In a patient with a varicocele the:
  Sperm count is depressed
 Percentage of motile sperms is depressed 
 Volume of the semen is depressed
 Percentage of immature sperms is depressed 
 Fructose concentration of the semen is depressed

 Testicular tumours
  Account for about 1% of tumours in Causasian males
 Are more commonly encountered in Africans
 In about 10% arise in undescended testes
 Are all radio sensitive
 Are most commonly seminomas

 In the pathology of testicular tumours:
  Seminoma is fast‐growing and the cell resembles a spermatocyte
 Seminoma  is  firm  and  grayish  or  creamy  in  colour  and    usually 
homogenous
 Teratoma  may  be  slow,  or  fast‐growing  and  originates  in  a  primitive 
pluripotent cell
 Teratoma has a honeycombed cystic appearance  and is usually yellow in 
colour
 Teratoma is seen between 40 and 50

 Of the teratomas of the testis:
  Malignant teratoma undifferentiated (MTU) is the most malignant
 Teratoma differentiated (TD) behaves like a benign tumour
 Malignant  teratoma  intermediate  (MTI)  has  a  combination  of 
differentiated and undifferentiated teratoma
 Malignant  teratoma  trophoblastic  (MTT)  and  malignant  teratoma 
anaplastic (MTA) produce chorionic gonadotrophins
 Malignant teratoma trophoblastic spreads rapidly by the blood stream to 
the lungs and liver when the tumour is small

 Testicular tumours usually metastasize to the following lymph nodes:
  Inguinal
 External iliac
 Para‐aortic
 Mediastinal
 Supraclavicular

8
 Teticular tumours present most commonly as :
  Secondary vaginal hydrocele
 Enlarged painless or heavy testes
 Acute pain simulating acute epididymo‐orchitis
 Hepatomegaly
 Respiratory problem

 Gynaecomastia, loss of libido and aspermia may be due to:
  Malignant teratoma intermediate
 Seminoma
 Lymphoma
 Interstitial cell tumour
 Sertoli cell tumour

 Sexual precocity and herculean muscular development in a boy of 10years 
may be due to:
  Interstitial cell tumour
 Sertoli cell tumour
 Seminoma
 Malignant teratoma trophoblastic
 Teratoma differentiated

 Essential  investigations  in  a  patient  with  suspected  testicular  tumour 


include:
  Scrotal ultrasound
 X‐ray of the chest
 Serum human chorionic gonadotrophin
 Alfa‐fetoprotein
 Serum acid phosphatase

 Measures taken in the treatment of testicular tumours include:
  Orchidectomy through the scrotum
 Pre‐operative radiotherapy of the testis
 Radiotherapy of the para‐aortic and ipsilateral pelvic nodes
 Post‐operative cystotoxic therapy whether or not there are metastases
 Biopsy,  cytotoxic  chemotherapy,  and  surgery  for  residual  masses  in 
advanced malignant teratoma

 Cytotoxic drugs used in the treatment of malignant teratoma include:
  Vinblastine and bleomycin 
 Bleomycin and Etoposide
 Cis‐Platinum
 Etoposide

9
  Markers used in the management of testicular tumours include:
  Serum alkaline phosphatase
 Serum human chorionic gonadotrophin
 Carcino‐embryonic antigen
 Serum uric acid
 Alfa‐fetoprotein

A 3‐month old baby has a soft, fluctuant swelling extending from the right 
hemiscrotum to the internal ring.  According to the mother, it is small in the 
morning and get bigger by the end of the day.

 The most likely clinical diagnosis is:
  Complete, right, indirect, reducible inguinal hernia
 Right vaginal hydrocele plus right, indirect, incomplete reducible inguinal 
hernia
 Irreducible complete right indirect inguinal hernia
 Congenital hydrocele
 Infantile hydrocele

 A  firm  non‐tender,  smooth‐surfaced,  scrotal  swelling  with  impalpable 


epididymis in a 40‐year old man of 3 months duration is most likely to be:
  Chronic epididymo‐orchitis
 Gumma of the testis
 Seminoma of the testis
 Teratoma of the testis
 Haematocele

 A    tense,  fluctuant,  bosselated  swelling  about  5  cm  in  diameter  situated 


above and behind the testis in a 30‐year old man is likely to be:
  Chronic epididymitis
 Acquired cyst of the epididymis
 Encysted hydrocele of the cord
 Tumour of the epididymis
 Congenital cyst of the epididymis

 A  non‐tender,  tense,  fluctuant,  ovoid,  smooth‐surfaced  scrotal  swelling 


with  an  impalpable  testis  in  a  35‐year  old  man  of  1‐year  duration  is  most 
likely a:
  Teticular tumour
 Sprmatocele
 Haematocele
 Complete irreducible inguinal hernia

10
 Vaginal hydrocele

 A  non‐tender,  soft,  fluctuant,  ovoid,  smooth‐surfaced  scrotal  swelling  in  a 


20‐year old man which is not attached to the testis or epididymis but moves 
downward when traction is applied to the  testis is most probably:
  An encysted hydrocele of the cord
 A complete inguinal hernia
 An infantile hydrocele
 A cyst of the hydatid of Morgani
 A lipoma of the cord

57. Incidence of undescended testis in full‐term babies examined at birth is: 
 1.4%

 2.4%

 3.4%

 4.4%

 5.4%

         58.  Most common location of the ectopic testis is:
 Perineum

 Femoral

 Suprapubic 

 Contralateral scrotum

 Superficial inguinal pouch

59.  The incidence of cryptorchidism at one year of age is:
 3.8%

 2.8%

 1%

 0.8%

 O.1% 

60. Factors that may influence testicular descent include: 

11
 Scrotum

 Gubernaculum

 Temperture

 Obstructing fibrous bands

 Hormonal imbalance

61.  Best time after birth for children with undescended testis to undergo 
surgery is:
 Within 12 months

 12 – 18 months

 18 – 24 months

 24 – 30 moths

 30 – 36 months

62.  The following may predispose to Fournier’s gangrene:
 Perianal sepsis

 Urethral stricture

 Periurethral abscess

 Testicular torsion

 Diabetes mellitus  

63.  The following are true of Fournier’s gangrene:
 It is a form of necrotizing fasciitis

 May occur in previously healthy individuals

 The disease affects the testes

 It can affect both adults and children

 It is caused by opportunistic and synergistic organisms

           64. The following are implicated in Fournier’s gangrene:

12
 Enterococci

 Microaerophilic Streptococcus

 Bacteroides

 Fusobacterium

 Clostridium

65. Complications resulting from acute epididymo‐orchitis include:
 Abscess formation

 Testicular atrophy

 Development of chronic pain

 Infertility

 Chronic epididymo‐orchitis

66. Torsion of the testis is generally common in the following age group:  
 5 – 10 years

 10 – 15 years

 15 – 20 years

 20 –255 years

 25 – 30years

67.  The following may predispose to testicular torsion:
 Horizontal inversion

 Hydatid of Morgagni

 Long narrow mesorchium

 Chronic epididymitis

 High investment of tunica

68.  The following are true of testicular torsion:
 It is most common around puberty

13
 It does not occur in neonates

 It must be differentiated from epididymo‐orchitis

 Watchful waiting is the preferred treatment

 On the right the direction of torsion is clockwise

69.  Irreversible ischaemic injury of the testis becomes pronounced after
the following duration of torsion:  
 1 hr

 2 hrs

 4 hrs

 6 hrs

 8 hrs

70.  The following swelling may fluctuate in the course of the day:
 Infantile hydrocele

 Congenital hydrocele

 Vaginal hydrocele

 Encysted hydrocele of the cord

 Epididymal cyst

71.  The most likely cause of testicular injury from varicocele is:
 Elevated scrotal temperature

 Testicular hypoxia

 Effect of adrenal metabolites

 Altered levels of spermatic vein testosterone

 Testicular hypertension

72.  Germ cell tumours constitute the following percentage of all testicular
Malignancies:
 < 80%

14
 80 – 85%

 85 – 90%

 90 – 95%

 > 95%

73.  Testicular tumours constitute the following percentage of male
cancers 
 1%

 2%

 3%

 4%

 5%

74. The undescended testis accounts for the following percentage of 
testicular tumours:

 4%

 6%

 8%

 10%

 12%

75.  The commonest testicular tumour of infancy and childhood is:
 Seminoma 

 Yolk sac tumour

 Adenomatoid tumour

 Choriocarcinoma

 Lymphoma

15
76. The following are germ cell tumours:
 Yolk sac

 Seminoma

 Choriocarcinoma

 Leydic cell tumour

 Teratocarcinoma 

77.  Commonest presentation of a germ cell tumour of the testis in a young
adult man is:
 Testicular pain

 Testicular swelling

 Reactive hydrocele

 Bilateral gynaecomastia

 Chronic cough with haemoptysis

78.  Concerning tumour markers used in testicular cancer management: 
 Serum alpha‐ fetoprotein (AFP) elevation do not occur in pure 
seminoma

 Syncytiotrophoblastic cells are responsible for the  production 
of beta‐human chorionic gonadotrophin (HCG)

 AFP originates in cells from the yolk sac

 Approximately 5 – 10% of seminoma patients have raised HCG

 Lactate  dehydrogenase  is  not  a  tumour  marker  for  germ  cell 


tumours

79. Alpha‐fetoprotein is produced by the following testicular tumours:
 Yolk sac

 Seminoma

 Teratocarcinoma

16
 Embryonal carcinoma

 Choriocarcinoma

80.   The following are routine investigations in a case of testicular
Cancer:
 Tumour markers

 Lymphangiogram

 Histopathology of affected testis

 Bone scan

 Chest X’ray

           81.   Testicular tumour markers are indicated for the following:
 Staging testicular tumours

 Determine malignant potential of the tumour

 Determine best treatment option

 Monitor response to treatment 

 Detect relapse after treatment

82.  Alpha fetoprotein is raised in the following:
 Teratoma

 Choriocarcinoma

 Pregnancy

 Pure seminoma

 Primary hepatoma

83.  The most radiosensitive testicular tumour is:
 Choriocarcinoma

 Seminoma

 Embryonal cell carcinoma

17
 Yolk sac tumour

 Teratocarcinoma

84. Preferred treatment option for Stage 1 seminoma after orchidectomy
is: 
 Surveillance

 Chemotherapy

 Retroperitoneal lymph node dissection

 Combination of chemotherapy and radiotherapy

 Radiotherapy to para‐aortic and ipsilateral pelvic nodes

85.  Preferred treatment for Stage 1 teratoma after orchidectomy is:
 Surveillance

 Chemotherapy

 Radiotherapy

 Retroperitoneal lymph node dissection

 Combination of chemotherapy and radiotherapy

86.   Treatment of choice for metastatic testicular teratoma is: 
 Actinomycin D , Methotrexate and Chlorambucil

 Cisplatinum, Methotrexate and Bleomycin

 Bleomycin, Etoposice and Cisplatinum

 Vinblastin, Bleomycin and 5‐Fluorouracil

 Cisplatinum, vinblastin and 5‐Fluorouracil

87.  Modified retroperitoneal lymph node dissection preserves fertility in    
  most patients by sparing the following structure:

 Genitofemoral nerve

 Internal iliac artery

18
 Seminal vesicles

 Postganglionic sympathetic nerve fibers

 Pelvic parasympathetic plexus

88.  Optimum treatment for patients with advanced seminoma after 
orchidectomy is:
 Radiation

 Immunotherapy

 Retroperitoneal lymph node dissection

 Radiation plus chemotherapy

 Chemotherapy

89.  The 5 – year survival rate for Stage I seminoma is:
 70%

 75%

 80%

 85%

 90%

90.  In non‐seminomatous germ cell tumours, the following factors 
determine the risk of metastatic disease:

 Presence of teratoma

 T‐stage

 Presence of embryonal cell carcinoma elements

 Vascular invasion

 Lymphatic invation

Answers 
 A, B, C, D, E

19
 D, E
 D
 A, C, E
 C, E
 A, E
 A, E
 B
 B, C, D
 E
 A, D
 A, C, E
 C, D E
 B, D
 A
 A, D, E
 A, D
 A, B, C, D
 A, D
 A, B, C, D
 A, B, D
 B, C
 E
 D
 C, E
 A, B, C
 B, D
 B, D
 A, C, D
 C, D, E
 B, D, E
 A, C, D
 C
 A, B, D, E
 A, B, C, E
 B, D
 A, C
 C
 C
 A, B
 A, C, E
 B, C, D
 B, C, E
 C, D, E

20
 B
 E
 A
 A, B, C, D
 C, E
 A, B, C, D
 B, E
 D
 C
 B
 E
 A

57. C 83. B
58. E 84. E
59. D 85. A
60. B, D, E 86 C
61. A 87. D
62. A, B, C, D 88. E
63. A, B, D, E 89.  E
64. B, C, D, E 90. B, C, D, E
65. A, B, C, D, E
66. B
67. A, C, E
68. A, C, E
69. C
70. B
71. A
72. D
73. A
74. D
75. B
76. D
77. B
78. A, B, C, D
79. A, C, D
80. A, C, E
81. A, D, E
82. A, C, E

21
CHAPTER 6
CUTANEOUS ULCERS, SINUSES, FISTULAE

 Concerning an ulcer, the :


 Edge is where the healthy skin begins
 Floor is what is palpated
 Base is what is seen around the ulcer
 Size is helpful in diagnosis
 The margin is synonymous with the edge
 A specific ulcer may be caused by:
 Mycobacterium ulcerans
 Deep venous thrombosis
 Treponaema pertenue
 Esch. Coli
 Sickle cell disease
 A tropical ulcer:
 Occurs only in the tropics
 Is caused by Borrelia vincenti
 Is predisposed to by malnutrition and walking bare-footed
 May be initiated in a minor skin wound
 Starts as a painless ulcer
 A tropical ulcer:
 Starts as a painful septic blister
 Has a ragged, foul-smelling black, or grey slough of the skin when
the blister ruptures
 In the acute stage, has an unhealthy floor with offensive, yellowish
discharge
 In the acute stage, has a sloping edge
 70% of cases found in lower limbs
 A tropical ulcer of 8 weeks duration:
Becomes a chronic ulcer
Has a sloping edge
Has a grey or pink granulation in the floor
Has a soft base
Contains the causative organisms which are identifiable by culture
of a swab
 Complications of a tropical ulcer include:
 Pyaemia

1
 Chronic osteomyelitis of the underlying bone
 Tetanus
 Lymphoedema of the foot
 Malignant change in the ulcer

 The usual treatment of a tropical ulcer in the acute stage include:


 A 5-day course of streptomycin or sulphonamide
 Bed rest
 Daily eusol dressing
 Excision of the ulcer
 Application of a Plaster-of-Paris
 A tuberculous ulcer:
 Has a regular outline
 Has thin, blue, undermined edges
 Has a floor covered with pink granulation and thin watery discharge
 Has an indurated base
 May be found just above the medial malleolus
 Buruli ulcer:
 May start as a painless subcutaneous nodule on the trunk
 May heal proximally while spreading distally
 Progresses slowly and has sloping edges
 May have bridges of skin across it
 Is usually successfully treated by drugs alone.
 Which of the following drugs can be used in the treatment of
Buruli ulcer:

 Rifampicin
 Clofazimine
 Streptomycin
 Co-trimoxazole
 INAH
 An ulcer of yaws:
 May start as a granulomatous papule
 Has sloping edges
 Has dirty yellowish sloughing floor
 May heal spontaneously, the skin over it becoming depigmented

2
 Does not cause enlargement of regional lymph nodes
 An ulcer:

 With a nodular floor is likely to be non-specific


 With a serpiginous outline is likely to be tuberculous
 In the sole is most probably neuropathic or malignant melanoma
 With a punched out edge is most probably syphilis or yaws
 With an indurated base is malignant

 A non-specific ulcer:
 In the acute phase is painful and has a sloughing floor covered with
purulent discharge and a sharp edge with oedematous tender
surrounding skin
 In the transition phase has a sloping edge and grayish-yellow floor.
 In the reparative or healing phase is painless and pink granulation
fills the floor
 In the chronic phase has grayish floor with a thin odourless
discharge
 The fifth phase is malignant change
 In an indolent or callous ulcer, the:
 Base is indurated
 Floor has pink granulation tissue
 Edges are rigid and hard
 Surrounding skin is warm and oedematous
 Epthelium at the edge grows inwards
 Complications of non-specific ulcers of the lower leg include:
 Chronic osteomyelitis
 Amyloidosis
 Below knee lymphoedema
 Deformities of the foot or ankle
 Venous thrombosis
 A painful circular ulcer with sloping edges on the dorsum of the
foot near the big toe in a 60-year old man could be due to:

 Leprosy
 Varicose veins

3
 Arterial disease
 Malignant melanoma
 An abscess
 The prime cause of a venous ulcer following deep venous
thrombosis is:
 Varicosity of the long saphenous vein
 Venous stasis
 Back pressure in the veins
 Destruction of the valves of the perforating and deep veins
 High venous pressure in the legs
 A venous ulcer usually:
 Is situated just above the medial malleolus
 Is irregularly circular
 Has a deep floor with pink granulation
 Has punched out edges
 Has indurated base

 Essentials in the treatment of a venous ulcer include:


 Application of an antibiotic dressing to the ulcer daily
 Below knee supportive occlusive elastic, elastoplasts or viscopaste
bandage from the toes to the tibial tubercle
 Administration of antibiotics
 Early stripping of varicose veins
 Physiotherapy
 Decubitus ulcer:

Results from inadequate nursing care


Shows first as a darkened area of the skin
May expose muscles and tendons but not bone
Can occur in any part of the body
Can be prevented by 3-hourly turning of the patient and adequate
skin care
 Neurotropic(Trophic) ulcer:
 May be caused by leprosy or diatetes mellitus
 Is usually on the dorsum of the foot
 Is usually shallow
 Heals quickly
 May be painful

4
 Haemoglobinopathic ulcer:
 Is most often found in the lower limb close to the lateral malleolus
 Is associated with atrophic skin and hyperpigmentation
 Heals quickly
 Often breaks down again after healing
 Bed nest worsens it
 Haemoglobinopathic ulcer is difficult to treat because of:
 Lowered oxygen tension
 Recurrent haemolytic crisis
 Fibrosis around the ulcer
 Low haemoglobin level
 Lack of appropriate personnel
 A 50-year old man has a 0.5cm ulcer on the nose which has a
raised and rolled up edge. It is of 2/12 duration.

The most likely clinical diagnosis is:

 Squamous cell carcinoma


 Malignant melanoma
 Syphilitic ulcer
 Non-specific ulcer
 Basal cell carcinoma

 A sinus:
 Connects two epithelial surfaces
 May be congenital
 Has a discharge which is always present
 Is treated with antibiotics
 Unresolved infection may cause acquired sinus.
 Causes of a sinus include:
 Chronic ulcer
 Achnonycosis
 Amyloidosis
 Buruli ulcer
 Foreign body e.g. Sutures
 In the treatment of a sinus:
 Excision of sinus is indicated in congenital type
 Dressing alone adequate for acquired type

5
 Tract of the sinus may be laid open and dressed
 In tuberculous, sinus treatment of TB lesion leads to healing
 Penicillin exhibition contributes to healing
 Causes of acquired fistula include:
 Abscess that ruptures into a cavity and skin
 Burns
 Granulomatous lesions
 Benign tumours
 Schistosoma haematobium
 The following are examples of neoplastic ulcers:
 Squamous cell carcinoma
 Renal cell carcinoma
 Kaposi sarcoma
 Malignant melanoma
 Dermato fibrosarcoma Protuberans
 Diabetic Ulcer:
 Ulcers in a patient with Diabetes Mellitus
 May be caused by infection
 May be caused by trauma
 May be caused by pressure necrosis
 Basis in hypoaesthesia from blood loss

 The following are examples of specific ulcers:


 Pyogenic ulcers
 Decubitus ulcers
 Syringomyelia
 Tropical ulcers
 Syphilitic ulcers
 Tuberculous ulcer:
 Seen usually in the groin and on the neck
 May be associated with normal sized lymph nodes
 Culture of discharge may show AFBS
 Could be associated with night sweats

6
 May be associated with tuberculous bone infection
 In Buruli ulcer differential diagnosis of the pre ulcerative form
include:
 Onchocercoma nodule
 Lipoma
 Sebacous cyst
 Naevus
 Phyconycosis
 Complications of Buruli ulcer include:
 Anaemia
 Contracture
 Proximal lymphoedena
 Marjolin’s ulcer
 Osteitis
 In the treatment of Buruli ulcers the following practitioners are
involved:
 Physiotherapist
 General Surgeon
 Orthopedic Surgeon
 Opthalmologist
 Maxilloficial Surgeon

CHAPTER 6
CUTANEOUS ULCERS, SINUSES, FISTULAE
ANSWER

 A
 A, C

7
 C, D
 A, B, C
 A, B, C
 B, C, D, E
 B
 B
 A, B, D
 A, C
 C, D
 C, D
 A, C
 A, C
 C, D
 C, E
 D
 A, B
 B
 A, B, E
 A
 B, D
 A, B, C, D
 E
 B
 B, E
 A, C, D
 A, C
 A, B, C, D
 A, B, C, D
 D, E
 A, C, E
 A, C, E
 A, B, D
 A, C

8
CHAPTER 14
BURNS

 A Burn:
 Is coagulative destruction of skin which may be caused by hot water
 Occurs most commonly in children
 Is pathologically different from scald
 Injury depends on the temperature of the heat source but not on the
duration of exposure of the tissues to such temperature
 In the adult may be due to RTA
 In deep partial thickness burns destruction of the following occur:
 Lymph vessels
 The hair follicles and sebaceous glands
 Sweat glands
 Red cells
 Capillaries
 Mediators released after burns include:
 Histamine
 Cyclo-oxygenase pathway products
 Serotonin
 Prostaglandins
 Thyroxine
 Cold water immersion after thermal injury:
 Increases the risk of infection
 Reduces pain after partial thickness burns
 Can result in hypothermia if applied to large burns
 Has been reported to stabilize mast cells
 May decrease oedema if applied immediately
 The protein content of oedema fluid in burns may be as much as
(g/1):
 60
 20
 30
 40
 50
 The electrolyte composition of oedema fluid in burns:
 Has a higher sodium concentration than plasma
 Has a lower potassium concentration than plasma
 Has a higher bicarbonate concentration than plasma

1
 Has a lower chloride concentration than plasma
 Is the same as that of plasma

 In burns:
 The maximum oedema fluid obtainable is about 10% of body
weight or 50% of total E.C.F
 There is initial vasodilatation followed by vasoconstriction
 The rate of fluid exudation from the capillaries is highest in the first
6h. and falls in 48h.
 Increased capillary permeability may occur even in unburnt areas
 Immediate charring of the skin lessens external and subcutaneous
fluid loss
 Anaemia in burns may be caused by:
 Thrombosis
 Increased red cell fragility
 Changes of dressings
 Bone marrow depression
 Skin grafting
 In burns:
 There is depression of cellular and immunological mechanisms
 Leucocytes cannot phagocytose bacteria normally
 There is depression of lysosomes
 IgG is not affected
 Transfer of leucocytes and antibodies to sites of bacterial infection
is rapid and not depressed
 In burns:
 Renal failure is commonly caused by toxic substances produced in
the burnt skin
 The urine may occasionally be port-wine
 Uraemia is always associated with oliguria
 There may be necrosis of the distal convoluted tubules
 The renal vessels are dilated
 Organisms that may infect a burn in the first 24h include:
 Proteus
 Streptococcus
 Pseudomonas

2
 Esch. Coli
 Staphylococcus
 Predominant organisms in a scald wound by seven days include:
 Staphylococcus
 Pseudomonas
 Pneumococcus
 Clostridium tetani
 Streptococcus

 In the exposure method of treating burn wound:


 Antibiotic spray, povidone or mercurochrome may be used
 The eschar inhibits bacterial growth
 Any environment is suitable
 It is not necessary for attendants or visitors to be gowned or
masked
 The method should end as soon as the integrity of the eschar is
broken
 Silver sulphadiazine cream:
 Is highly antibacterial and prevents pseudomonas and other
infections
 May cause severe hyponatraemia and hypokalaemia
 May cause argyria
 Is non-toxic but painful
 Is applied daily or on alternate days with or without dressing
 Silver nitrate dressing:
 Is effective in the prevention or treatment of pseudomonas infection
 Is applied every 24h and is time consuming
 Stains beddings and the floor
 Leaches sodium, potassium and chloride from the burn wound
 Is widely used.
 Lypphilized heterograft of pig skin:
 Limits fluid and electrolyte loss from the burn wound
 Is vascularized
 Does not harbour infection
 Should be changed weekly until an autograft is available
 Is a xenograft
 Exposure method is suitable for burns of the:

3
 Face
 Neck
 Entire trunk
 Perineum
 Axilla
 Curling’s ulcer:
 Occurs only in the stomach and duodenum
 Occurs only in severe burns
 Is often complicated by bleeding
 May be prevented by the administration of antacids or cimetidine to
severely burnt patients in the first three weeks
 Is best treated by partial gastrectomy or vagotomy and drainage

 The commonest cause of death in burns is:


 Shock
 Renal failure
 Burn wound sepsis
 Septicaemia
 Pneumonia
 After severe burns:
 There is positive nitrogen balance
 The resting metabolic expenditure may be 150-200% of normal
 High calorie high protein diet is essential
 Enteral nutrition can always provide all the energy and protein
 The protein requirement can be estimated from the urinary nitrogen
 Prognosis in burns depends on:
 Age of patient
 Extent of burn
 Depth of burn
 Respiratory tract injury
 Pre-existing disease
A 20-year old West African male in Lagos weighing 60kg sustained

4
burns
of the face, right upper limb and anterior part of the trunk
following a fire
in is bedroom at 12 midnight. He is brought to the ‘Casuality’ at
2.00 a.m.
 Your first concern will be to:
 Check his pulse and blood pressure and administer morphine.
 Remove his clothes and determine the extent and depth of his burn
 Check the rate of his breathing
 Find out if the airway is clear
 See if there is singeing of his nasal hair or soot in the nostril
 Examination of the burnt area shows that there are blisters, areas
of erythema and other areas of skin which are all sensitive to pin
prick.
The burn is:
 Superficial partial thickness
 Deep partial thickness
 Full thickness and superficial partial thickness
 Partial thickness (superficial and deep)
 Full thickness

 The burn is estimated at 30% of his body surface. The total fluid
requirement (in litres) in the first 24h is about:
 5.5
 6
 6.5
 7.2
 7.5
 The fluids will consist of:
 2L 5 per cent dextrose, IL blood and the rest as Ringer’s lactate
 Ringer’s lactate

5
 2L blood and the rest as ringer’s lactate
 2L 5 per cent dextrose and the rest as Ringer’s lactate
 IL 5 per cent dextrose; 2L dextran 70 and the rest as Ringer’s
lactate
 Half the fluids should be given by:
 6a.m
 8a.m
 10a.m
 12noon
 2p.m
 Initial investigations will include:
 Haematocrit
 Blood group
 Serum electrolytes and blood urea
 Culture of the burnt skin
 White cell count and differential
 The antibiotic to administer at this stage is:
 Chloramphenicol
 Cloxacillin or penicillin
 Gentamicin
 Oxytetracycline
 Cephaloridine
 The most important parameter to monitor is:
 Quarter-hourly pulse and blood pressure
 Central venous pressure
 Filling of peripheral veins
 Hourly urinary output
 Skin temperature and orientation

 After cleaning the burn wounds with one per cent centrimide you
will apply:
 Terracortril spray daily
 0.5% silver nitrate dressing and add more silver nitrate every 3h

6
 Silver sulphadiazine cream on alternate days without dressing
 Mafenide creame twice daily
 Povidone-iodine daily
 The fluid requirement (in litres) in the second day is
approximately:
 4
 4.5
 5
 5.5
 6
 On the second day the urine is found to be port-wine in colour.
You will after examining the urine:
 Restrict fluid intake
 Order an urgent I.V.P.
 Give blood
 Give 10% mannitol
 Give sodium bicarbonate
 On the 8th day he complains of some pain in the wound, anorexia
and generally being unwell. His temperature is 38.50C and the skin
around the discharging wound looks reddened.
The clinical diagnosis is:
 Cellulitis
 Lymphangitis
 Septicaemia
 Burn wound sepsis
 Necrosis of the burn
 Effects of electrical injuries include:
 Cardiac or respiratory arrest
 Ventricular fibrillation
 Auricular fibrillation
 Rupture of blood vessels
 Thrombosis of vessels
 Late complications of electrical burn injuries include:
 Impotence
 Cataract
 Epilepsy
 Intractable headache
 Abnormal gait

7
 In electrical injuries:

 Direct current is more dangerous than alternating current


 The action potentials of muscles and nerves are altered
 The contact burns are small and deep whether the tension is high
or low
 Fluid loss is massive and cannot be estimated from the body
surface area burnt
 Amputation may be necessary
 A patient with flash burns of the left side of the head, neck, left
arm and lower extremity (leg) has suffered what percentage body
surface burn?
 9
 18
 5
 30
 27
 The advocates of the open treatment of burns consider it superior
to the closed method because:
 It tends to reduce the incidence of burn contractures
 It tends to reduce the oedema in the burned area.
 It is more economical of nursing care
 It keeps the skin surface cool and dry and thereby deprives micro-
organisms of an environment favoruable for multiplication
 It saves on antibiotics
 Impairment of immunity in trauma or burn patients is associated
with:
 Abnormalities of neutrophil function
 Immunosuppressive factors in the serum
 Polycythaemia
 Decreased lymphocyte mitogenic response
 Circulating suppressor leucocytes
 Smoke inhalation is in the early post-burn phase associated with:
 Hypoxaemia
 Increased platelet adhesiveness
 Hypercapnia
 Abnormal chest X-ray
 Elevated carboxyhaemoglobin
 Concentration of organisms per gram of tissue indicative of ‘burn
wound sepsis’ is:

8
 102
 103
 104
 105
 106

 In the Burn tissue:


 It is invaded by Gram-ve organisms within 24hrs
 It is invaded by Gram+ve organism with 7 days
 Bacteria invade the eschar and also by direct spread
 Sepsis in full thickness is poorly controlled because of reduced
blood supply
 Local invasive infection present as septicemia
 In the burn patient the following complications may occur:
 Cushing ulcers
 Splenic eulargement
 Gastric dilatation
 Pyelonephritis
 Flash backs may occur
 In the burn patient with cardiopulmonary complication:
 Pulmonary oedema may follow inhalation injury
 Inflammation not important here
 Aspiration of gastric contents plays a role
 Tracheitis of intubation may occur
 Heat from inhaled gases plays a part
 For prevention of post burns contracture:
 Physiotherapy should start from day 1
 Splinting is very important
 Night splints may be used up to 6 months
 Pressure garments are mandatory
 Early surgery is useful
 In burn prevention:
 Education should be limited to the home environment
 Education on cooking methods is very important with regards to
scalds in children
 Appropriate design of materials for household appliances should be
considered

9
 Inflammable liquids should not be stored in the home
 Epileptics should be banned from getting near fires
 In electrical burn:
 Extent of injury may be difficult to assess
 In the presence of myoglobinuria IV lasix in indicated
 Infusion of IV fluids must be at the rate to increase urine output to
above
50 mls/hr
 Excision of necrotic muscles is indicated 48hrs after diagnosis
 Silver sulphadiazine is contraindicated in this type of burn.

 Chemical burns may be caused by:


 Cement
 Strong acids
 Strong alkali
 Petroleum products
 Concentrated brine
 Principles of management in burns:
 Restore
 Reconstruct
 Repair
 Revive
 Rehabilitate
 In burns the following investigations on blood taken from the
patient is important:
 Cortisol levels
 Adrenaline levels
 Haematocrit
 Blood area
 Creatinine

10
CHAPTER 14
BURNS
ANSWER

 B, E 35. A, B, C, D, E
 A, B 36. B, D, E
 A, B, C, D 37. B
 B, C, D, E 38. D
 E 39. A, B, D, E
 E 40. A, C, E
 A, D 41. D
 A, B, C, D, E 42. C, D
 A, C 43. C, D, E
 B, D 44. A, C, D, E
 B, E 45. B, C, E
 B 46. B, C, E
 A, B, E 47. A, C
 A, E 48. A, B, C, D
 A, C 49. A, C, D, E
 A, E 50. C, D

11
 A, B, D
 C, D
 C
 B, C, E
 A, B, C, D, E
 D
 D
 C
 D
 B
 A, B, C
 B
 D
 C
 E
 D, E
 D
 A, B, D, E

12
CHAPTER 16

SUPERFICIAL SOFT TISSUE SWELLINGS

 Keloid:

 Is characterized by deposition of an unduly large amount of


collagen which is raised above the skin level but does not
extend beyond the margin of the wound.

 Occurs in the darkly pigmented but not white races

 Results from the undue prolongation of the phase of collagen


deposition and maturation

 Is associated with a predisposition which is inherited as a


dominant character

 Appears after four months of the wound and continues to


grow beyond a year.

 Keloid:

 Is found at all ages

 May occur in any part of the body

 If quiescent, may start growing again during pregnancy

 May become malignant

 Is the same as hypertrophic scar

 SATISFACTORY methods of treating keloids include:

 Excision

 Excision followed by radiotherapy

 Injection of triamcinolone acetonide into them

 Excision followed by injection of a solution made of nitrogen


mustard, hyalase and triamcinolone into the wound

1
 None of the above

 A sebaceous cyst:

 Contains desquamated keratin, sebum, fat and cholesterol


crystals

 Can occur in any part of the body although the commonest


sites are the scalp and face

 May rarely become malignant

 Is spherical, firm, attached to skin and has a definite edge

 Is lined by columnar epithelium

 Dermoid cyst:

 Results from transplantation of a piece of epithelium into the


underlying soft tissue during an injury.

 Often contains hair

 If external angular in position may have an intracranial


extension

 May be the cause of a pre-auricular sinus and a sinus of the


suprasternal space

 Is often seen on the fingers of tailors

 A ganglion:

 Is mucinous degeneration of connective tissue arising from a


tendon sheath or the synovial membrane or capsule of a joint

 Does not occur in the palm

 Is usually unilocular

 Is tense or soft, fluctuant, circumscribed and attached to skin

2
 Is best treated by aspiration of its contents and injection of
hydrocortisone sclerosant

 Varruca vulgaris:

 Is usually seen in the elderly

 Is often multiple and occurs commonly in the hands and


knees

 Is an epidermal overgrowth caused by a virus

 Is spherical and usually about 0.5cm or more in diameter

 Disappears with the surface application of salicylic acid

A 25-year old man has two swellings over the right greater trochanter.
The swellings are firm, mobile, circumscribed, non-tender,
subcutaneous and attached to the deep fascia.

 The most likely clinical diagnosis is:

 Fibromata

 Lipomata

 Implanatation dermoids

 Calcified guinea worms

 Onchocercomata

 Naevi(moles)

 Are present in every human being

 Get more numerous from birth till adulthood

 Are always flat and hairless

 Are collections of melanocytes always in the basal layer of


the epidermis

3
 Should be excised as they may become malignant.

 Signs of malignant change in a naevus include:

 Bleeding

 Ulceration

 Hairiness

 Increase in size

 Increase in pigmentation

 Desmoid tumour:

 Is a rare encapsulated fibrosarcoma

 Arises from the muscular aponeurosis of the anterior


abdominal wall usually above the umbilicus

 May originate at the site of operation scar or injury

 Occurs mostly in parous women in the third or fourth decade

 Is treated by wide excision followed by irradiation

 Haemangiomas:

 Are congenital tumours involving blood vessels

 If capillary are intradermal and not raised above the skin


surface

 If cavernous are elevated above the skin surface

 If capillary may involve muscle, nervous tissue or bone

 If cavernous may be associated with dyschondroplasia

4
 The treatment of cavernous haemangioma at the age of five is:

 Leave alone

 Excision

 Injectionof a sclerosant

 Application of carbon dioxide snow

 Cryosurgical using liquid nitrogen

 Neurofibroma:

 Arises from the nerve cell

 May be associated with scoliosis

 Is inherited as a Mendelian dominant

 May become malignant

 Is excised in most patient s

 Lipoma:

 Is a swelling of abnormal fat enclosed in a capsule of


connective tissue

 Does not occur beyond t he subcutaneous region

 Is not found in the palm or sole

 Is soft but not fluctuant

 May be multiple, small and painful

 Squamous cell carcinoma

 Is commoner in Africans than in Caucasains

 Is commonest in the lower limbs in Africans and in the head

5
and neck region in Caucasians

 Spreads early to the lymphatics but vascular spread is


unusual

 Is microscopically well-differentiated in the majority

 May arise from sinuses following chronic would infection.

 In the treatment of squamous cell carcinoma:

 If the underlying bone is not involved wide excision or radio


therapy is effective.

 If the underlying bone is involved, excision followed by the


radio therapy is the treatment of choice

 Block dissection should be done routinely whether or not the


lymph nodes are palpable.

 Recurrence after irradiation responds well to further irradiation

 Cytotoxic chemotherapy has been found effective in about


50% of patients.

 Cytotoxic drugs currently useful in squamous cell carcinoma


especially of the head and neck include:

 5-Fluouracil

 Methotrexate

 Cyclophosphamide

 Cisplatin

 Bleomycin

 Basal cell carcinoma:

 Is as common in Caucasians as it is in Africans

6
 Occurs in the face in 90% of patients

 Has a raised everted edge

 Spreads by direct infiltration and lymphatics but not by the


blood s stream

 Is best treated by local excision followed by adjuvant


chemotherapy.

 Malignant melanoma:

 Is twice as common in women as in men

 In Caucasians is found most commonly in the lower leg

 Of the nodular type is the commonest variety

 Of the lentigo maligna type is very slow-growing, occurs


mainly in older age group

 Of the eye is equally common in Africans and causasians

 In the treatment of malignant melanoma:

 A subungual lesion is treated by amputation of the affected


finger

 The lesion is widely excised and followed by radiotherapy

 Block dissection is usually done only if or when the regional


nodes are enlarged and mobile

 Fixed regional nodes are irradiated

 Lesions may rarely disappear spontaneously

 Cytotoxic drugs of benefit in malignant melanoma include:

 Vincristine

7
 Melphalan

 Adriamycin

 Actionmycin D

 Dacarbazine

 In the prognosis of malignant melanoma:

 Lesions on the extremities have a better prognosis than those


on the trunk

 Lesions confined to the primary site have a 5-year survival of


about 50% and those with regional lymph nodes involvement
15%.

 Nodular lesions have the best prognosis

 Men have a better prognosis than women

 The 5-year survival is better than for most other forms of


cancer if it is diagnosed early and treated adequately.

 Dermatofirosarcoma:

 Is thrice as common in females

 Occurs mostly in the abdominal wall

 Is usually nodular and may resemble a neurofibroma

 Is of low-grade malignancy, locally invasive and tends to


recur repeatedly after excision

 Is treated by wide excision or radiotherapy

 Kaposi’s sarcoma:

 Is a multifocal sarcoma of blood vessels

8
 Is most common in Jews, Italians and Africans and the highest
incidence in Africa is in Uganda

 Has recently been observed in a highly malignant form


among Americans, especially homosexuals, suffering from
acquired immune-deficiency disease.

 Occurs in all age groups and is twice as common in males in


adults and children.

 Histological is of two types – monomorphic and mixed.

 Nodules of Kaposi’s sarcoma are found in:

 Bones

 Intestines

 Brain

 Lungs

 Liver

 The skin nodules in Kaposi’s sarcoma:

 In the white-skinned are brown

 Do not normally ulcerate

 Are often preceded by oedema which in the lower limb


affects the whole limb

 May spontaneously become flattened and hypopigmented

 May follow a chronic, indolent benign course

 Lymphadenopathic Kaposi’s sarcoma:

 Is usually seen after 15 years

9
 May be localized or generalized

 Can easily be differentiated from Hodgkin’s or tuberculosis

 Has a good prognosis

 X-ray of bones in Kaposi’s sarcoma may show:

 Pathological fractures

 Sclerosis

 Osteoporosis

 Bone destruction but with thin cortical margin

 Cystic changes

 Kaposi’s sarcoma can be treated effectively by:

 Surgery

 Surgery and radiotherapy

 Radiotherapy

 Radiotherapy and chemotherapy

 Chemotherapy

 Cytotoxic drugs that may be used in the treatment of Kaposi’s


sarcoma include:

 Paclitaxel

 Chlorambucil

 Doxorubicin

 Vincristine

 Etoposide

10
 Fibrosarcoma:

 Is seen frequently in young adults

 May arise in a scar

 Is circumscribed, firm or hard and often warm

 Spreads by he lymphatics and blood stream

 Is best treated by wide excision and adjuvant chemotherapy

 Rhabdomyosarcoma:

 Is a rare malignant tumour of non-striated muscle

 Occurs in all age groups

 Spreads by lymphatics and blood stream

 Of the embryonal type is seen in childhood and in the trunk


and periphery

 Has a poor prognosis as early recurrence is the rule

 A hypertrophic scar

 During growth is raised above the skin and is itchy

 After growth is flat and wide

 Continues to grow after one year

 Pathologically consists of collagen nodules

 Is treated by excision

 A keloid differs from healthy skin by

 A rich vasculature

11
 Low mesenchymal density

 Thickened epidermal cell layer

 Presence of broad bundles of collagen

 Predominance of sweat glands

 Malignant melanoma

 Accounts for 4% of skin cancers

 Accounts for 70% of skin cancer related deaths

 Is increasing worldwide at a faster rate than any other cancer


– 10% yearly

 Has highest incidence in Auckland – 56.2 per 100,000

 Worldwide, the incidence has doubled over the last 10 years

 Risk factors in developing malignant melanoma include

 Pre-existing benign naevus

 Freckles

 High naevus count

 Increased CFC in the atmosphere

 Sunburn from age 20years

 Concerning melanoma

 Worldwide, the commonest is superficial spreading


melanoma

 Superficial spreading melanoma arises from a pre-existing


mole

 Nodular melanoma is seen most commonly on the legs and

12
trunk and grows slowly

 Acral lentiginous melanoma is the commonest form in blacks


and occurs in the soles, palms and nail beds

 Lentigo maligna melanoma is usually seen in the elderly and is


fast-growing

 WHO recommends that the excision margin should be 2cm if the


thickness of the melanoma is

 <1mm

 1-1.5mm

 1.5-2mm

 2.1-4mm

 >4mm

 Prognosis in malignant melanoma is better

 In those over 60 years

 If it arises in a pre-existing naevus

 In non-whites

 Genital sites

 In stable diabetics

 Rhabdomyosarcoma

 May be found in child aged 6-10yrs

 Is very fleshy and lobulated

 May be found in the bladder and vagina

13
 Hepatic metastases occur very early

 Wide excision of the lesion together with involved muscle is


the treatment of choice

 Rhabdomyosarcoma

 Is unlikely to respond to irradiation treatment

 Cyclophosphamide can be used in the treatment of this


tumour cyclically.

 Infants as young children are worse off progrostically


compared to ulcer patients

 Intracavitary rhabdomyoscra has the resemblance of bunch


of grapes

 Vinblashire can be used in the treatment within tumour

 Liposcraoma

 May simulate a benign lipoma

 May be seen commonly on the flanks

 Is usually very large in size

 Is treated by wide excision

 May be found in the abdominal cavity

 In Aids-related kaposi’s Sarcoma

 This form of the disease in slow growing

 Lesions here are centrifugal

 May occur in the mouth and throat of the patient

 Prognosis is poor

14
 May present a jigger

 In AIDS – related Kaposi’s Sarcoma

 Stool occult blood test is positive in the patient with skin


lesions

 FNAC may be required for diagnosis

 Lesions may be found in the stomach at oesophagus

 Sheletal survey may show bone lesions

 A differential diagnosis of this condition is pyogenic


granuloma

 The following are forms of Kaposi sacroma

 Nodular

 Fungating

 Fleshy

 Plaques

 Visceral

 Soft tissue sarcomas

 Arise from mesodermal tissue

 Account for 5% of tumours

 Arise in the lower limb in 80% of cases

 Have incidence of 2:100,000

 Are commoner in 5th and 6th decades

15
 In soft tissue sarcomas

 Macroscopically this tumour may consist of firm homogenous


or mucoid tissue

 Exposure to tanners can lead to this condition

 The tumour usually have a well defined capsule

 Histological type of this tumour is anglosarcoma

 Spread through lymphatics is rare

 The following are varieties of melanomas

 Nodular

 Acral lentiginous melanoma

 Subcutaneous

 Lentigo maligma melanoma

 Superficial necrotizing melanoma

 In subungual melanoma

 10% of cases occur in blacks

 The lesion may appear as a longitudinal pigmentation under


the nail

 Most are nodular melanomas

 The thumb may be involved

 One sees the commonest cause of in transit metastasis

16
 The following are important in the assessment of melanoma

 Tumour thickness

 Presence of lymph nodes

 Presence of ulceration

 Host response

 Lymphatic vessels invasion

 In investigations for melanoma

 Excisional biopsy is indicated

 Chest Xray

 Abdomal ultrasound of abdomen and pelvis

 Serum LDH

 Uric acid levels

 Bowan’s Disease

 Is extra-epithetial squamous carcinoma of low malignancy

 Is found only in sun exposed areas

 Occurs in both sun exposed and non sun exposed areas

 Is slightly palpable and crusted

 Response to topical 5% 5 FU cream application.

 Squancus cell carcinoma

17
 Is 2-3 times more common in woman

 It spreads by infiltrating surrounding and subjacent tissues and


bone

 Usually shows early lymphatic spread

 Seldom shows vascular metastases.

 May have a nodular floor.

 Marjolins ulcer is a squanous cell carcinoma caused by chronic


irritation with repeated breakdown and repair due to

 Burno

 Varicose vein

 Pressure sore

 Lonizing radiation

 Chronic ulcers

 Neurofibromatosis

 Is an autosomal recessive disorder

 Is composed of Schwann cells, fibroblasts, mast cells and


vascular components

 May be only composed of skin

 Exhibits a plexiform type known as type 2 neurofibromatosis

 May become malignant in 10% of deep seated lesions

 Lymphangioma

 It is a congenital malformation of the lymphatic channels

 In its superficial form consisting of lymph cyst is called cystic

18
hygroma

 May have associated elements of haemangioma

 May produce macroglossia

 Presenting as Cystic hygroma is usually compressible

Answers

 C

 C, D

 E

 A, C

 B, C, D

 A

 B, C

 E

 A, B

 A, B, D, E

 C, D

 B, C, E

 A

 B, D

 E

 B, D

 A, E

 B, D, E

19
 B

 A, D

 C, E

 B, E

 A, B, E

 B, C,D

 A, C

 A, B, C, D, E

 B, E

 B

 C, D, E

 D, E

 A, C, D, E

 A, B

 C, E

 A, B, D

 A, C, D

 A, B, D, E

 A, B, C, D

 A, B, D

 C, D, E

 B

 B, C, E

 A, B, D

20
 A, C, D

 C, D, E

 B, C, D

 A, D, E

 A, D, E

 A, C, D, E

 A, B, D

 B, C, D

 A, C, D, E

 A, B, C, D

 C, D, E

 B, D, E

 A. B, E

 B, E

 A, C, D

21
CHAPTER 18
PLASTIC SURGERY

 The free transfer of tissues or organs from an individual to


another individual of the same species is:

 Autograft

 Isograft

 Heterograft

 Xenograft

 Homograft

 Rejection of graft is the rule in:

 Isograft

 Allograft

 Homograft

 Heterograft

 Split (partial thickness) skin graft:

 Is mde up only of epidermis

 Is usually dry because it lacks sebaeceous

glands

 Grows hair

 Shrinks considerably

 Can withstand much trauma

1
 Full thickness graft:

 Can withstand much trauma

 Shrinks considerably

 Contains sweat glands

 Contains sebaceous glands

 Takes very readily

 Grafts:

 Quickly acquire the same sensation as the recipient site

 Initially become hyper-pigmented

 Acquire the normal skin colour in a short period of time

 If partial thickness have more normal skin colour than if full


thickness

 Full thickness has more requirement for take

 A graft will die unless the blood supply from the recipient site to
the graft is developed within:

 1 day

 1-2 days

 2-3 days

 4 days

 5 days

 Survival of skin graft is undermined if:

2
 There is close approximation of the graft to the recipient site

 Plaster cast is used to immobilize the graft

 Diathermy is used for haemostasis during the operation

 There is no dressing on the graft

 There is infection by strep. haemolyticus or pseudomonas

 Methods that can be used to cover a recipient site when the graft
is inadequate include:

 Mesh graft

 Postage stamp graft

 Sheet graft

 Pinch graft

 Alternating strips of autograft and allograft

 Storage of skin graft that is not used immediately:

 Is usually carried out at 4˚-6˚C

 At the usual storage temperature keeps the graft viable for 4 weeks

 Is best done in saline

 At the usual storage temperature may cause crystallization of water


and salts in the tissues

 Requires the addition of antibiotics to the storage medium

 In bone grafting:

 Cortical bone is preferred to cancellous bone because its hard


framework gives good mechanical support

3
 Banked bone is as good as living bone

 Too much hammering and chiseling may kill off the osteoblasts and
undifferentiated cells of the bone marrow

 If the osteocytes and osteoblasts survive, resorption of the ground


substance, hydroxyaparite, does not occur

 The donor site should be drained for 48 h

 Donor areas used for the bone grafting include the:

 Ribs

 Femur

 Tibia

 Iliac crest

 Skull

 Cartilae graft:

 Does not survive easily as it requires vascularization

 Is obtained from the fibula

 Is used in the restoration of defective external ear, nasal or facial


contours

 May warp whether it is living or killed

 Does not grow in children

 Buried dermis graft:

 Is readily vascularized and tough but shrinks easily

4
 Forms epidermoid cysts

 Loses the sweat glands

 Loses the arrectores pilorum muscle

 May be used to form pseudoarthrosis after excision of the


mandibular head in ankylosis of the temporo-mandibular joint

 Indication for fascial grafts include:

 Repair of hernia defect

 Surgery of facial palsy

 Repair of dural defects

 Surgery of complete rectal prolapsed

 Correction of stress incontinence

 A flap:

 Is never without a functional blood supply

 Consists of whole skin

 Is called a local flap when it is transferred direct to where it is to be


used

 Must have the end attached to a new position left for one week to
pick up new blood supply before the other end is detached

 May be carried on the wrist

 Autotransplantation is currently possible with the:

 Ear

5
 Heart

 Kidney

 Limb

 Liver

 In the primary homograft reaction:

 The graft is vascularized by the 2nd day

 The graft vesels become more dilated and the graft oedematous
from

the 6th day

 Polymorphonuclear leucocytes and small lymphocytes collect below

the graft bed

 Disruption of the vessel wall with haemorrhage into the graft occurs

 Thrombosis of the vessels occurs from the14th day

 The lymphocytes involved in homograft rejection reaction:

 Are developmentally derived from lymphatic nodes

 Are conditioned by the thymus

 Migrate to the lymph nodes from where they continuously circulate

in the body

 Leave the blood stream through the capillaries to enter the


lymphatic

system

 On antigenic stimulation re-enter lymph nodes and undergo


increase

in size

6
 The anomaly of cleft palate and cleft lip:

 Is found in 1:2000 live births in Africans

 Occurs in 5: 1000 live births in Caucasians

 Is commoner in Japanese than in Africans or Caucasians

 Is commoner in females

 In 40% is associated with congenital abnormalities of the head,


heart

Hand, feet or genitalia

 The following may be aetiologically associated with cleft palate


and

cleft lip:

 Vit. C deficiency

 Rubella

 Toxoplasmosis

 Syphilis

 Nitrogen mustard

 The commonest cause of cleft palate and cleft lip is:

 Heredity

 Vit. A deficiency

 Vit. B deficiency

 Irradiation

7
 Unknown

 Cleft palate and cleft lip occur in:

 The first trimester

 The second trimester

 The third trimester

 Any period during pregnancy

 Only post mature foetus

 The face is developed from:

 3 processes

 4 processes

 5 processes

 6 processes

 7 processes

 The fronto-nasal process forms the:

 Nose

 Nostrils

 Nasal septum

 Upper lip

 Premaxilla

 The maxillary and the fronto-nasal processes fuse to form the:

 Cheeks

8
 Philtrum

 Upper jaw

 Nasal septum

 Palate (except the premaxilla)

 In the development of the face:

 Fusion of the mandibular processes forms only the lower lip

 The mouth is formed by the fusion of the maxillary and mandibular


processes

 Excessive fusion of the mandibular and maxillary processes results


in microstoma

 Fusion of the premaxilla and maxillary process starts inferiorly and


so abnormalities are more common in the upper part of the upper
lip

 Fusion of the maxillary palatal shelves, nasal septum and


premaxillary

palatal shelves begins rostrally (anteriorly)

 Cleft of the upper lip:

 Results from failure or breakdown of the fusion of the whole

fronto-nasal process with the maxillary process

 If unilateral, is on one side of the philtrum

9
 If it extends into the nostril splitting the alveolus, then is complete

 If it is incomplete, is only a minor lip notch and does not involve the

whole lip

 Is always solitary and not associated with cleft palate

 Cleft palate (secondary palate):

 Is complete if the fissure affects the entire length of the palate

 Is complete and bilateral if the V-shaped premaxilla is completely

separated from the cleft palate

 If incomplete, the fissure affect only the uvula or the uvula and soft
palate

 If occult, there is dehiscence of palatal musculature but an intact


mucosa

 Cleft anterior to incisive foramen

 Methods of feeding a child with cleft palate include:

 Bottle with a long teat with a hole larger than normal

 Breast-feeding

 A plastic bottle that can be squeezed

 Cup-feeding

 Spoon-feeding

10
 The main aim of surgery in cleft palate (secondary palate) and

cleft lip is:

 Cosmetic

 To ensure good speech

 To provide a safe airway

 Prevention of infection of the internal auditory meatus and middle


ear

 Facilitate feeding

 Surgery of cleft palate( secondary palate) is done at (in months):

 1-6

 6-12

 12-18

 18-24

 24-30

 Most surgeons usually repair cleft lip at (in weeks):

 4-8

 8-12

 12-16

 16-20

 20-24

 In basic principles of plastic surgery:

 Skin incisions are planned to optimize the resultant wound

11
 Langer’s lines are skin lines of maximal tension that run parallel to
skin creases

 Langer’s lines are seen more easily on the faces of older persons

 Incisions across wrinkle lines resulted in puckered scars

 Skin colour match is very important when providing skin over

 The following are examples of grafts:

 Cartilage

 Bone

 Fat

 Nerve

 Muscle

 In the use of flaps:

 They can be classified according to movement with which they are


swung into position

 It is called random flap if there is a named vessel in it

 It is called axial pattern flap if there is no named vessel

12
 In free tissue transfer there is successful anastomosis of vessels in
the flap to vessel at the recipient site

 The donor site of a flap is always skin grafted or closed primarily

 The following are examples of flaps:

 Groin flap

 Axillary flap

 Fasciocutaneous flap

 Bone flap

 Muscle flap

 In tissue expansion:

 Used to increase the volume of an otherwise small potential

dome flap

 An inflatable silicone balloon is used

 It is inflated within a week with saline

 Could be complicated by infection

 The silicone is then removed at the end of inflation and skin utilized

as a flap

 The following are problems associated with cleft lip:

 Feeding

 Cosmetic

 Hearing

 Airway

13
 Speech

 In cleft palate repair:

 If velopharyngeal incompetence persists then pharyngoplasty is


indicated

 Speech therapy is indicated before repair of the cleft

 Can be done from 6 month up to 18 month of age

 This can be combined with the removal of the adenoids

 The aim is to produce a long mobile palate that will close the
oronasal sphincter

 In breast reduction:

 Procedure is carried out for patients with macromastia

 Obesity and hormonal imbalances may cause macromastia

 A method for this procedure is amputation with pedided flaps

 Lactation and nipple sensation maybe affected

 Complications include infection and dermal necrosis

 Pressure sores:

 Unrelieved pressure over bony prominences may lead to tissue


necrosis

 May affect heel or scapular area

14
 Ischial areas and occipital may be affected

 Can be prevented by the use of 2nd generation cephalosporins

 Management includes excision of the ulcer

 Processes involved in the ‘take” of skin graft include:

 Fibrin seal

 Plasmic imbibitions

 Inosculation

 Revascularization

 Clot fixation

 Flaps can be classified as follows:

 Movement

 Innervation

 Lymphatic drainage

 Blood supply

 Tissue content

 Following are included in the scope of plastic surgery:

 Trauma

 Vascular malformations

 Cranio facial surgery

15
 Abnormal scarring

 Skin cancers

 Main problems with the child born with cleft palate deformity only:

 Appearance

 Feeding

 Failure to thrive

 Bipedal oedema

 Speech deformity

 In cleft care recent developments include:

 Bone graft

 Use of prosthesis

 Secondary surgery

 Counselling

 Speech therapy

 Aesthetic procedures in plastic surgery include:

 Reduction mammoplasty

 Angrecitation mammoplasty

 Reconstruction of breasts

 Mytidectomy

 Genioplasty

 With regard to speech therapy in cleft palate:

16
 Repair to palate carried out before normal speech develops

 Carried out by the orthodontists

 Velo-laryngeal incompetence will affect speech

 Defects in speech due to problems with pronunciation of vowels

 Intra oral pressure buildup is needed to pronounce fricatives

 Counselling of parents of cleft lip and palate patients:

 If both parents are healthy but with 2 children affected probability

of next child having CLP is 9%

 Healthy parent one child with disease probability with next is 4%

 Parent with the disease probability of next child 1%

 One parent and or child affected the probability of next child is 15%

 Both parents and a child affected probability of the next child is up


to 60%

 Ear reconstruction:

 In microtia reconstruction in indicated

 In protuberant ear (bat ears) reconstruction is indicated

 Reconstruction is done when the child is a teenager on account of


maturity

 The rib is used for the reconstruction

17
 Elevation of framenoration and skin graft done:

 In the lower extremity procedures:

 Extensive soft tissue defects normally follow trauma

 Extensive soft tissue defects normally follow burns

 Extensive soft tissue defects normally follow excision of tumours

 Exposed tibia can be covered by a local facio-cutaneous flap

 Deep vein thrombosis and pulmonary embolism should always be


considered in fasciocutaneous flap

CHAPTER 18
PLASTIC SURGERY
ANSWERS

18
 E 30. B

 B, C, D 31. B

 B, D 32. B

 A, C, D 33. C, D, E

 B, E 34. A, B, C, D

 C 35. A, D, E

 E 36. A, C, E

 A, B, D, E 37. B, D, E

 A 38. B

 C, D 39. A, C, E

 C, D 40. A, B, D

 C 41. A, B, C, E

 B, E 42.B, C, D

 A, B, C, E 43. A, D, E

 A, E 44. A, B, C, D, E

 A, C, D 45. B, C, E

 B, D 46. A, B, C, D, E

 B, C, E 47. A, B, D, E

 A, C 48. A

 B, C, E 49. A, B, D, E

 E 50. A, E

 A 51. A, C, D

 C

19
 A, B, C, E

 A, C, E

 C, E

 B, C

 B, D

 A, C, E

20
CHAPTER 19
MONTH, TONGUE AND SALIVARY GLANDS

 Conditions which predispose to cancrum oris include:


 Malnutrition
 Poor oral hygiene
 Stomatitis
 Debilitating disease
 Late weaning

 The commonest debilitating disease associated with cancrum oris is:


 Gastroenteritis
 Typhoid
 Measles
 Tuberculosis
 Whooping cough

 The organism(s) found consistently in the oedematous stage of cancrum


oris is/are:
 Borrelia vincenti
 Fusiformis fusiformis
 Staph aureus
 Strep. Viridians
 Bacteroides melaninogenicus

 Clinical features of cancrum oris include:


 Painful swollen red gum around a deciduous tooth
 An alveolar ulcer which spreads to the adjacent cheek
 Multiple ulcers of the mouth
 Tense, swollen, dark red cheek which becomes black
 Pyrexia and toxaemia

 Part or the whole of the following structures may be lost in cancrum oris:
 Mandible or maxilla
 Lips
 Palate
 Nose
 Eye

 Complications of cancrum oris include:


 Ankyloglossia
 Trismus
 Salivary fistula
 Squamous cell carcinoma
 Dental cyst

1
 Measures taken in the treatment of the EARLY stage of cancrum oris
include:
 Intravenous fluids
 High protein high calorie diet and vitamins
 Sequestrectomy
 Administration of penicillin
 Nasogastric decompression

 Causes of stomatitis include:


 Tobacco
 Spirits
 Deficiency of Vit. B
 Allergy to dentures
 Drug reaction

 A ranula:
 Occurs in the floor of the mouth usually in the midline
 Is a transparent white or bluish cyst
 Contains clear serous fluid
 Is commonly seen in young adults
 Is excised or marsupialized

 Papillomas of the mouth:


 May be caused by chronic irritation
 If pedunculated, are often hard and impair the function of the mouth
 Are small
 May become malignant
 Should always be excised

 Carcinoma of the mouth:


 Is commonly associated with tobacco chewing
 Affects females more commonly
 Spreads late to the cervical lymph nodes
 Is treated by radiotherapy
 Is advanced improves on methotrexate and cisplatin

 Causes of macroglossia include:


 Lymphanigioma
 Congenital arterio-venous fistula
 Neurofibromatosis
 Carcinoma
 Acute inflammation

2
 Leukoplakia is probably caused by:
 Syphilis
 Tobacco chewing
 Herpes febrilis
 Spirits
 Ill-fitting dentures

 Leukoplakia of the buccal cavity:


 Is a form of chronic superficial inflammation of the buccal mucosa
 Affects males and females equally
 Is premalignant
 Microscopically shows infiltration of the dermis with lymphocytes and
hyperplasia and hyperkeratinisation or atrophy of the epidermis
 Is initially white

 Clinical features of leukoplakia of the tongue include:


 Hypertrophy of the tongue
 Soreness
 Thickened white patches
 Fissures
 Warty area

 A dental ulcer of the tongue:


 Is caused by a ragged tooth or ill-fitting denture
 Is painless
 Is shallow
 Is longitudinal is shape
 Has undermined edges

 Apthous ulcers of the tongue


 Are probably caused by strep. Viridians and fusiformis
 Start as vesicles which rupture
 Are small and deep
 Have red margins
 Usually heal spontaneously in two weeks

 A recurrent aphthous ulcer:


 Is more common in women
 Is painful and shallow
 Commonly occurs at the base of the tongue
 Is usually multiple
 Heals spontaneously

 Gummatous ulcer of the tongue:


 Is rare

3
 Is typically sited in the mid-posterior tongue
 Has punched out edges
 Has a regular outline
 Often heals spontaneously

 Carcinoma of the tongue:


 Is as common in females as in males
 Has a very high incidence in Africa
 In India is estimated to account for 20 per cent of all malignancies
 Has the highest incidence between 50 and 70
 Is in most cases a well-differentiated squamous carcinoma in the anterior
two-thirds and a transitional cell carcinoma in the posterior third

 Predisposing conditions in carcinoma of the tongue include:


 Sepsis
 Dental Ulcer
 Alcohol drinking and smoking
 Chewing of betelnut or tobacco
 Geographical tongue

 The commonest site for carcinoma of the tongue is the:


 Posterior third
 Tip
 Dorsum
 Under surface
 Lateral margin

 Carcinoma of the tongue presents as:


 Pain in the ear
 Warty outgrowth or fissure
 Cough or dyspnoea
 Excessive salivation
 Alteration in voice

 Where facilities exist, the treatment of carcinoma of the tongue without


overt secondaries may be:
 Surgery
 Surgery followed by radiotherapy
 Radiotherapy
 Surgery followed by cytotoxic therapy
 Pre-operative radiotherapy and surgery

 The operative procedure(s) for carcinoma of the tongue without overt


secondaries is/are:

4
 Wide local excision
 Partial glossectomy
 Subtotal glossectomy
 Partial glossectomy plus block dissection of the ipsilateral cervical lymph
nodes
 Subtotal glossectomy plus block dissection of the ipsilateral lymph nodes.

 Complications of radiotherapy for carcinoma of the tongue


 Stomatitis
 Permanent dry mouth
 Maxillary readio-necrosis
 Increased caries
 Deformity of the tongue.

 Structures removed in block dissection of cervical nodes include:


 Lymph nodes and submandibular salivary gland
 Internal jugular vein
 Sternomastoid
 Posterior and anterior bellies of the digastrics
 Deep fascia

 In carcinoma of the tongue the 5-year survival is:


 25% for all patients
 Nearly 50% if there are no metastases
 15% if there are nodal deposits
 60% if the tumour is less than 2cm
 30% if the tumour is greater than 2cm and there are no metastases

 Carcinoma of the lip:


 Is ten times more common in the lower lip
 Affects females in 98% of patients
 Is aetiologically related to exposure to sunlight
 Is in some cases a transitional cell carcinoma
 Usually spreads directly to the deep cervical lymph nodes

 Carcinoma of the lip:


 Is commonly seen between 40 and 50
 Presents as an infiltrating or nodular lesion or as a painful ulcer
 May be treated by surgery or radio-therapy
 Requires block dissection of the ipsilateral lymph nodes whether or not the
lymph nodes are palpable.
 Has an overall 5-year survival rate of about 70%

 In the anatomy of the parotid gland:


 The facial nerve and its subdivisions are sandwiched between the

5
superficial and deep lobes
 The entry of the facial nerve is about 2.5cm below the external auditory
meatus in the direction of the tip of the nose.
 The duct opens into the mouth opposite the site of the upper second
premolar tooth
 Tumours and inflammation cause a uniform enlargement of the gland
 The gland extends from the zygomatic arch to the angle of the mandible.

 In the anatomy of the submandibular gland.


 The lingual and hypoglossal nerves lie between the deep and superficial
parts of the gland
 The superficial part is crossed by the cervical branch of the facial nerve
and the anterior facial vein
 The facial artery is related to the posterior and superior parts
 The submandibular duct arises from the deep part and opens by a visible
orifice about 1cm from the frenulum.
 Lymph nodes are embedded in it and so the gland is removed in block
dissection of the nodes for malignant lymph nodes.

 The sublingual gland:


 Lies in front of the deep part of the submandibular gland
 Is on the under surface of the tongue
 Is about 6cm long
 Has about 12 small ducts which open on papillae along the sublingual fold
 Secretes serous saliva and it is rare for calculi to form in it.

 Saliva:
 Contains ptyalin which hydrolyses starch to glucose
 Aids speech
 Prevents buccal infection
 Secreted by the parotid gland is mucous
 Secreted by the submandibular gland is serous and mucous

 Salivary secretion:
 Is about 2L/day
 By the submandibular gland is about 1L/day
 Has a pH of about 7.0
 At the normal pH is saturated with calcium
 Is effected by sympathetic stimulation of the salivary glands.

 Salivary calculus
 Is ten times commoner in the submandibular gland than in the parotid
although the parotid section contains a higher concentration of calcium.
 Consists essentially of phosphates of calcium and magnesium
 May be oval or round in shape

6
 Is usually single
 Is radio-opaque in all patients

 The most typical clinical feature of salivary calculus is:


 Persistent pain in the gland
 Persistent painful swelling of the gland
 Painful swelling of the gland during eating which subsides gradually after
the meal
 Enlarged tender gland
 Palpable calculus in the duct

 In Mickulicz disease:
 There is usually symmetrical enlargement of the salivary and lacrimal
glands
 The month is dry
 Auto-immunity is probably the cause
 The salivary tissue is replaced by fibrous tissue
 Radiotherapy has not been found beneficial

 Pleomorphic adenoma of the parotid gland:


 Is benign and has a true capsule
 Is often multicentric in origin
 May recur locally by implanation
 Histologically may show mucoid, myxoid and chondroid degeneration of
the connective tissue
 May become malignant in 10%

 Characteristics of malignant change in a pleomorphic adenoma of the


parotid gland include:
 Pain
 Lobulated surface
 Increased rate of growth
 Facial weakness
 Hard consistency although the tumour is mobile

 The treatment of a pleomorphic adenoma in the superficial part of the


parotid gland is:
 Excision
 Superficial parotidectomy followed by radiotherapy
 Excision with a margin of healthy surrounding tissue.
 Total parotidectomy
 Superficial parotidectomy

 Adenolymphoma:
 Occurs in all races

7
 Is often multiple and may be bilateral
 Occurs predominantly in females over 40
 Usually occurs at the angle of the mandible or below it
 Is about 2-3cm in diameter, firm with cystic areas or entirely soft and
fluctuant

 The treatment of primary carcinoma of the parotid gland without lymph


node enlargement is:
 Total parotidectomy plus block dissection of the ipsilateral lymph nodes
 Total parotidectomy plus block dissection of the ipsilateral lymph nodes
plus radiotherapy plus cytotoxic therapy
 Total parotidectomy plus postoperative radiotherapy
 Total parotidectomy plus radiotherapy plus cytotomic therapy
 Pre-operative radiotherapy plus total parotidectomy

 Complications of superficial parotidectomy include:


 Facial weakness or palsy
 Dryness of the mouth
 Calculus formation
 Salivary fistula
 Auriculo-temporal syndrome

 In the auriculo-temporal syndrome:


 There is pain in the skin over the parotid region during meals
 There is sweating in the skin over the parotid region during meals
 There is coldness over the skin over the parotid region during meals
 There is pallor over the skin over the parotid region during meals
 The cause is probably growth of the divided parasympathetic into the
auriculo-temporal nerve.

A man of 60 presents with pain in the right ear of six months duration;
two months earlier he had noticed a painless induration on the right
lateral aspect of the tongue. On examination he is a fit individual with
clean oral hygiene and a flat non ulcerating thickening of the epithelium
on the right edge of the tongue with much salivation. There are no
palpable lymph nodes.

 The most likely diagnosis is:


 Chronic superficial glossitis
 Carcinoma of the tongue
 Leukoplakia
 Dental ulcer of the tongue
 Syphilitic tongue

 The most appropriate diagnostic step is


 Full haematological profile

8
 Ultrasound scan of the tongue and oral carity
 Retro screen of the patient
 Cytology of oral washings
 Wedge biopsy of the lesion.

 Your preferred treatment for this patient is:


 Chemotherapy using cisplastin
 Partial glossectomy with block dissection of the ipsilateral cervical lymph
nodes
 Subtotal glossectomy with block dissection of the neck
 Radiotherapy
 Partial glossectomy with past operative radiotherapy

CHAPTER 19

 A, B, D
 C
 E
 A, B, D, E
 A, B, C, D, E
 A,B , C
 A, B, D
 A, B, C, D, E
 B, E
 A, D, E
 A, D, E
 A, B, C, E
 A, B, D, E
 A, C, D
 B, C, D, E
 A, C, D
 A, D, E
 A, , E
 A, B, C
 C, E
 A, C, D
 E
 A, B, D, E
 A, C

9
 A. N. C
 B, D
 A, B, C, E
 A, B, C, D, E
 A, C
 C, E
 A, B, E
 B, C, D, E
 A, D, E
 A, B, E
 C, D
 A, B, C
 C
 A, B, C
 B, C, D
 A, C, D
 E
 B, D, E
 C
 A, B, D, E
 B
 B
 E
 B

10
CHAPTER 20

OTORHINOLARYNGOLOGY

 In the posterior compartment of the nose :

 Are three conchae or turbinates all of which are normally seen on routine examination
of the nose.

 The naso-lacrimal duct opens in the superior meatus .

 The middle meatus contains the ostia of the posterior ethmoidal cells and the
spenoidal sinuses

 The inferior meatus contains the ostia of the maxillary and frontal sinuses and the
anterior ethmoidal cells.

 The lining mucosa,especially over the inferior turbinate, undergoes physiological


swelling and contraction in response to environmental changes in temperature and
humidity.

 The maxillary sinuses :

 Are present at birth but small and highly placed below the orbit.

 Are fully developed by 15 years

 Are related in its floor to the teeth from the incisor to the third molar

 Are related medially to the fronto-nasal and naso- lacrimal ducts

 Have the floor above that of the nose at birth, level at 7 years and below that level
at full development.

 The frontal sinuses :

 Are normally present at birth

 Develop from anterior ethmoidal cells

 Are related to the cavernous sinuses and nasopharynx and the sphenoids

 Are related to the anterior cranial fossa, orbit and olfactory nerves.

 Are nearly always symmetrical

 Unilateral nasal discharge and occasional epistaxis in a5 – year old boy are most

1
suggestive of:

 Suppurative rhinitis

 Allergic rhinitis

 Foreign body in the nose

 Sinusitis

 Rhinitis sicca

 In epistaxis :

 The bleeding in most patients is from a plexus of the capillaries in the antero-
inferior portion of the nasal septum

 The commonest primary (local) cause is nasal infection

 Hypertension ,infectious mononucleosis, measles or renal failure may be the


cause

 The blood in the nose is removed and the nose pinched for 5-10 minutes

 A foley catheter may be useful

 A boil in the vestibule of the nose should never be squeezed because it :

 Is very painful

 May cause cellulitis of the vestibule

 May cause septicaemia

 May cause cavernous sinus thrombosis

 May cause pyaemia

 Sneezing with itching of the nose ,eyes and throat ,watering of the eyes and clear
nasal discharge is most suggestive of :

 Acute rhinitis

2
 Allergic rhinitis

 Maxillary sinusitis

 Frontal sinusitis

 Acute pharyngitis

 Chronic nasal discharge with headaches occurs in

 Chronic maxillary sinusitis

 Chronic rhinitis

 Septal haematoma

 Nasal polyp

 Deviated nasal septum

 In the treatment of acute rhinitis, the following is/are useful:

 Antibiotics

 Vitamin C

 Topical nasal decongestant

 Steroids

 Systemic antihistaminics

 In chronic maxillary sinusitis:

 A radiography shows thickening of mucosa ,marked thickening of bone, polyp or


air fluid level

 Antral lavage is done weekly if pus is present

 Caldwell luc is done if successive weekly washings remain purulent

 Caldwell luc is indicated if the mucosa is polypoid

 MRI imaging always confirms the diagnosis

3
 A painless granuloma that originates in the inferior turbinate and spreads in the
sub mucosa and through the bony structures to the dorsum of the nose and the
central part of the face is most probably:

 Carcinoma of the nose

 Fibroangioma of the nose

 Rhino sporidosis

 Nasal entomophthorosis

 Aspergilloma

 Potassium iodide or septrin is used for:

 Aspergillosis

 Mucormycosis

 Nasal entomophthorosis

 African histoplasmosis

 Rhinosporidosis

A 50-year old man complains of a 3- month history of nasal obstruction which was at
first unilateral and is now associated with offensive discharge.

 You will suspect most strongly :

 Chronic Sinusitis

 Nasal polyp

 Septal deviation

 Foreign body

 Carcinoma of the nose or sinus

 Which of the following elevate (s) the pharynx during deglutition?

 Superior constrictor

 Stylopharyngeus

4
 Cricopharyngeus

 Middle constrictor

 Palatopharyngeus

A 10 – year old boy suffers from mouth breathing, nasal obstruction ,nasal discharge
and recurrent right earache.

 The most likely clinical diagnosis is :

 Maxillary sinusitis

 Otitis media

 Rhinitis

 Infection of the adenoids

 Recurrent tonsillitis

 Which of the following organism(s) may cause tonsillitis?

 Influenza virus

 Staphylococcus

 Streptococcus

 C – diphtheria

 Pneumococcus

 Complications of tonsillitis include :

 Rheumatic fever

 Otitis media

 Vincent’s angina

 Acute glomerulo- nephritis

 Glandular fever

A 20-year old woman has a 3-day history of severe pain on the right side of the
throat which radiates to the ear. Dysphagia is severe, trismus is present and pyrexia

5
is marked.

 The most likely clinical diagnosis is :

 Right otitis media

 Right tonsillitis

 Right peritonsillar abscess

 Pharyngitis

 Vincent’s angina

 The treatment of peritonsillar abcess may consist of :

 Administration of antibiotics

 Administration of antibiotics , incision and drainage

 Administration of antibiotics ,incision ,and drainage and tonsillectomy later

 Administration of antibiotics and tonsillectomy

 Administration of antibiotics and gargles

 Indication(s) for tonsillectomy is/are:

 Acute tonsillitis

 Recurrent tonsillitis

 B - haemolytic streptococcus carrier

 Recurrent otitis media

 Recurrent sorethroat and anlarged tonsils

 Secondary haemorrhage after tonsillectomy

 Is caused by separation of slough due to infection

 Occurs 3-4 days after operation

 Nearly always requires ligation of the bleeding vessels under G.A

6
 May cause atelectasis

 Occurs in 25 per cent of cases in children (up to 14)

A 20- year old man complains of sorethroat ,headache and fever for one week,
examination reveals redness of the throat, enlarged cervical lymph nodes ,splenomegaly
and a skin rash.

 The most likely clinical diagnosis is :

 Scarlet fever

 Agranulocytic angina

 Diphtheria

 Infectious mononucleosis

 Vincent’s angina

 In agranulocytic angina:

 Phenothiazine may be the cause

 There may be ulceration of the vagina and rectum generalized lymphadenopathy

 The red blood cell count is markedly depressed

 Broad-spectrum antibiotics , IM pentnucleatide, Vit. B12 and banked blood


should be given

 Sucking of nystain lozenges is beneficial

 Retropharyngeal abscess:

 If acute may occur at any age

 If acute, shows on examination as a reddish bulge to one side of the midline of


the posterior pharyngeal wall

 If chronic, usually occurs in adolescents and adults

 If chronic, originates from tuberculosis of the cervical spine and presents as a


unilateral pink bulge of the posterior pharyngeal wall

7
 If acute, is treated by incision under G. A.

 A pharyngeal pouch:

 Is a diverticulum of the mucosa and sub-mucosa posteriorly through the middle


and inferior constrictor muscles

 Is more common in females

 Causes gurgling noises in the neck after swallowing

 Requires oesophagoscopy

 May present with choking and violent coughing

 Malignant pharyngeal tumours:

 Are commonly sarcomatous

 Are often silent and present as cervical lymphadenopathy

 May present as epistaxis or earache

 Are more common in females

 Are best treated by cytotoxic drugs or radiotherapy

 Stridor is noisy respiration which:

 May be caused by limitation of adduction of the vocal cords

 May occur during the inspiratory phase only or during the inspiratory and
expiratory phases

 Does occur during the expiratory phase only

 May be due to obstruction in the bronchial tree

 May be due to glosso-palatopharyngeal obstruction

 In the innervation of the larynx the

 Recurrent laryngeal nerve supplies all the muscles

8
 Recurrent laryngeal is sensory to the areas below the vocal cords

 Superior laryngeal nerve is sensory to the areas above the cords

 Superior laryngeal nerve does not supply any muscle

 Recurrent laryngeal nerve contains no sensory fibres

A 4-year old girl has severe inspiratory stridor, prostration, fever and some
cyanosis for one day.

 The most likely clinical diagnosis is:

 Inhalation of a foreign body

 Bronchopneumonia

 Acute laryngo-tracheo-bronchitis

 Acute bronchitis

 Pharyngeal diphtheria

 Endoscopy is likely to show:

 Inflamed pharynx with yellowish grey membranes

 Inflamed oedematous larynx with pseudomembranes exudate or crust

 Red oedematous larynx with small superficial ulcers

 Foreign body in the larynx

 Nothing in the pharynx or larynx

 Organism(s) likely to be found on bacteriological examination may be:

 C. Diphtheria

 Pneumococcus

 Influenza virus

 Staphylococcus

 Haemolytic streptococcus

9
 Treatment may include:

 Naso-tracheal intubation

 Administration of diphtheria antitoxin

 Tracheostomy

 Removal of foreign body

 Administration of oxygen, antibiotics and hydrocortisone

 The vocal nodule is:

 An adenoma

 Common in singers

 Found at the junction of the posterior and middle thirds of each cord

 Treated initially by excision

 Is a veritable premalignant lesion

 A 50-year old man has persistent hoarseness for 4 weeks without any pain.
You will suspect most strongly:

 Chronic laryngitis

 Carcinoma of the larynx

 Papilloma of the larynx

 Carcinoma of the vocal cord

 Chronic pharyngitis

 Carcinoma of the larynx:

 Is squamous in 50 per cent of patients

 Is glottic in position in 10 per cent of patients

10
 Has a male/female ratio of 10:1

 Metastasizes to lymph nodes late if it is supraglottic in position

 Spreads commonly via the blood-stream

 A 30-year old woman is unable to phonate for 3 weeks. Laryngoscopy reveals


adductor paralysis of both cords.

This is most likely due to:

 Unilateral recurrent laryngeal nerve paralysis

 Unilateral superior laryngeal nerve paralysis

 Haemorrhage in the medulla

 Functional origin

 Carcinoma of the thyroid

A newlyborn neonate has dyspnoea with indrawing of the suprasternal notch


and subcostal region during inspiration.

 You will suspect:

 Laryngomalacia

 Laryngo-tracheo-bronchitis

 Bifid epiglottis

 Angioneurotic oedema

 A cyst of the larynx

 In dyspnoea associated with laryngeal obstruction, there is/are


characteristically:

 Expiratory stridor

 Inspiratory stridor

 Inspiratory and expiratory stridor

 Hoarseness of voice

11
 Cyanosis

 In the after-care of a tracheostomy patient:

 A bell should be by the bedside

 It is not necessary to humidify the air entering the trachea

 Atropine injection is useful as it allows easier breathing

 Antibiotics must be administered routinely

 The patient must sleep propped up in bed to avoid respiratory obstruction

 Which of the following is/are true of the external auditory meatus?

 The outer two-thirds is cartilaginous and the inner one-third bony

 The skin lining is loosely bound to the wall

 The lining of the cartilaginous portion contains hair follicles and sebaceous and
ceruminous glands

 The direction in the cartilaginous part is slightly downward and backward and in
the bony part upward and forward

 There is no bony portion in the infant and the tympanic membrane is almost
horizontal

 Reception and nervous transmission of sound is by the

 Vestibule

 Cochlea

 Semi-circular canals

 Utricle

 Tympanic membrane

 To inspect the adult eardrum the pinna must be pulled:

12
 Outwards

 Forwards

 Backwards

 Downwards

 Upwards

 To inspect the infant’s eardrum the pinna must be pulled:

 Outwards

 Forwards

 Backwards

 Downwards

 Upwards

 Symptoms of perforation of the tympanic membrane include:

 Deafness

 Dizziness

 Vestibular nystagmus

 Tinnitus

 Headache

 Perforation of the tympanic membrane:

 May be caused by a hard slap

 May cause deafness, dizziness and vestibular nystagmus

 May cause blood-stained otorrhoea and tinnitus

 Is always treated expectantly

 None of the above

13
 Keratosis obturans:

 Is accumaulation of the normal secretion of the meatus

 May cause ulceration of the meatal lining and pressure erosion of the bone

 Commonly causes dizziness

 Commonly causes increasing impairment of hearing

 May be syringed out

 Black powdery deposit in the external meatus is suggestive of inflammation by:

 B. Pyocyneus

 Candida

 Aspergillus

 Monilia

 Allergen

A 10-year old boy who has been having severe pulsating right earache, high
fever and headache for a week now complains of severe pain behind the right
ear.

 The most likely clinical diagnosis is:

 Acute suppurative otitis media

 Acute mastoiditis

 Secretory otitis media

 Acute zygomatic mastoiditis

 Boil in the external meatus

 Examinations will reveal

 Post- aural oedema and tenderness

 Swelling in the external meatus

14
 Pus in the external meatus which collects rapidly after it has been cleaned

 Oedema over the zygomatic arch and behind the ear

 Bulging of the tympanic membrane

 Treatment will include:

 Administration of nasal decongestants

 Cortical mastoidectomy

 Myringotomy

 Administration of penicillin

 Spirit ear drops

 Complications of suppurative otitis media incude :

 Lower motor neurone paralysis of the facial nerve

 Acute suppurative labyrinthitis

 Cerebral abscess

 Meningitis

 Cavernous sinus thrombosis

 Acute non suppurative or viral labyrinthitis:

 May follow respiratory infection , meningitis or encephalitis

 Is manifested by vertigo,staggering gait,tinnitus and nystagmus

 May damage the labyrinth irreversibly

 Is treated by mastoidectomy and drainage of the labyrinth

 In upper motor neurone facial palsy:

 The forehead cannot be wrinkled

15
 The lower half of the face is paralysed

 The lower lid sags

 Emotional movement of the face is completely paralysed and returns more slowly
than voluntary movement

 The angle of the mouth is pulled towards the unaffected side.

 In bell’s palsy:

 The forehead is paralysed

 Emotional movements of the face are not paralysed

 Whistling cannot be done

 There is loss of taste of the anterior two – thirds of the ipsilateral side of the
tongue

 Prednisolone is efficacious

A 45- year old man has an hour, of severe rotational vertigo associated with unilateral
deafness, tinnitus, headache and nausea.

 The most likely clinical diagnosis is :

 Hypertension

 Hypotension

 Intracranial tumor

 Labyrinthitis

 Menier’s disease

56. Drugs administered will include:

 Nicotinic acid

 Vit. B complex

 Vit. C

 Stemetil

16
 Penicillin

 The most common causes of conductive deafness include :

 Advancing age

 Otitis media

 Meniere’s syndrome

 Otosclerosis

 Perforation of the drum.

 Malignant otitis externa/ear


 Is a malignancy of external ear
 Is caused by pseudomonas
 Is an epithelioma
 It is common in young adults
 Is managed by ciproxin therapy
59. Acoustic neuroma
A. Usually presents with unilateral tinnitus
B. If bilateral is associated with neurofibromatosis
C. Is a schwanomma of vestibular nerve
D. Is managed by radiotherapy
E. Is managed by surgical excision
60. Complication of chronic suppurative otitis media
A. Excludes facial nerve palsy
B. May be extracranial
C. Includes labrynthitis
D. May present with seizures
E. Is managed by 3rd generation cephalosporines

61. Nasopharyngeal cancer


A. Is endemic in South Africa
B. Is endemic in south East Asia
C. Has genetic and environmental factors as aetiology
D. Is diagnosed by excision of neck node
E. Is managed by chemoradiation

62. Nasopharyngeal juvenile angiofibroma

17
A. Is common in boys
B. Is common in girls
C. Usually presents with recurrent torrential epistaxis
D. Is usually diagnosed by biopsy of the lesion from nasal cavity
E. Diagnosis is usually confirmed by angiography

18
CHAPTER 20

KEY

19
 E

 A, D, E

 B, D

 C

 A, C, D, E

 D

 B

 A, B, C, D, E

 B, C, E

 B, C, D

 D

 C

 E

 B

 D

 A, B, C, E

 A, B, D

 C

 B, C, D

 B, C, E

 A, D

 D

 A, B, E

 B, C

 C, E

20
 B, C

 B

 B, C

 C

 B

 C, E

 A, C, E

 B

 D

 C

 D

 A, C, E

 B

 A

 C, E

 B

 A, C, E

 C, D

 A, D

 A, C

 B, D

 C

 B

 A, C

 B, D

21
 A, B, C, D, E

 B, C

 B, C, E

 A, C E

 E

 A, B, D

 B, D

 B, E

 A, B, C, E

 B, D, E

 B, C, E

 A, C, E

22
CHAPTER 21

JAW INJURIES AND SWELLINGS

 The main danger of jaw injuries is:

 Bleeding

 Respiratory obstruction

 Shock

 Infection

 Aspiration of vomitus

 The position of an unconscious patient with jaw injuries should be:

 Lateral

 Head down and flexed

 Head down and extended

 Elbow fully extended

 Thigh extended

 Immediate actions to be taken in the casualty in an unconscious patient


with facial or jaw injuries suffering from respiratory obstruction include:

 Bronchoscopy

 Suctioning of the oro-pharynx

 Pushing the tongue to one side

 Passage of an oral airway or endotracheal tube

 Tracheostomy

 In the management of soft tissue injuries of the face:

1
 General anaesthesia is preferable to local analgesia

 Doubtful tissue should only be excised if it is dead

 Subcutaneous tissue and skin should always be accurately approximated

 Any type of suture material can be used

 Sutures are removed in 7-10 days

 Fractures of the mandible:

 Are more often unilateral than bilateral

 Occur most commonly in the angle and body in the canine/incisor regions

 Result from direct and not indirect trauma

 In the tooth-bearing segment are invariably compound

 If bilateral frequently involve the condyle and the opposite


canine/premolar

 In the management of fracture of mandible:

 The mandible should be immobilized to the maxilla by wiring the teeth of


both jaw in occlusion

 Splinting of the teeth on either side of the fracture is simple and effective

 A sling of bandage passing under the chin and pinned firmly to a head
bandage is adequate

 Immobilization of condylar fractures is maintained for 6-8 weeks

 Antibiotics must always be given for about ten days

 Clinical feature of fracture of the zygoma include:

 Circumorbital and sub-conjunctival ecchymosis

2
 Protrusion and swelling of the cheek

 Loss of sensation of the cheek, nose, upper and lower lips

 Epistaxis

 Elevation of the floor of the orbit

 Dislocation of the temporo-mandibular joint:

 Is commonly posterior and bilateral

 In the majority of cases both condyle and disc are displace posteriorly on
the articular eminence

 Is usually caused by opening the mouth too wide as in yawning

 Does not result from direct trauma on the chin

 May occur during dental extraction

 In the clinical features of anterior dislocation of the temporo-mandibular


joint:

 The mouth is held open and cannot be closed

 The upper teeth are anterior to the lower teeth

 The area in front of the ear is hollow

 The teeth can be passively occluded

 There is bleeding from the ear

 In the treatment of anterior dislocation of the tempero-mandibular joint

 Anaesthesia is always necessary

 The thumb should always be protected with gauze

 Direct firm upward traction of the mandible is exerted

3
 There is no need for immobilization if reduction is done within 48h of
injury

 Antibiotic cover is advisable

 In chronic gingivitis, the gum is:

 Swollen

 Quite painful

 Red

 Tender

 Subject to bleeding on the least provocation

 Aetiological factors in gingivitis include:

 Pregnancy

 Irritation of tartar on the teeth

 Administration of oral penicillins

 Gum recession

 Malnutrition

 In Vincent`s gingivitis (acute necrotizing ulcerative gingivitis):

 The causative organisms are Borrerlia vincenti and strep viridians

 There are ulcers which are covered by greyish pseudomembrane

 There are small vesicles which may coalesce to produce large ulcers

 Pus exudes from around the teeth

 Cancrum oris may be a sequel

4
 Periapical (alveolar) abscess may follow:

 Caries leading to dead tooth

 Gingivitis

 Trauma to a tooth

 Stomatitis

 Root canal therapy

 Organisms usually found in periapical abscess include:

 Fusiformis fusiformis

 Strep viridians or haemolyticus

 Borrelia vincenti

 Staph.pyogenes

 Esch coli

 Periapical abscess may lead to:

 Ludwig`s angina

 Acute necrotizing ulcerative gingivitis

 Osteomelitis of the jaw bone

 Orbital cellulitis

 Median mental sinus

 Clinical features of cervico-facial actinomycosis include:

 A rapidly progressive painful indurated swelling of the check, ramus,


parotid regions of the neck.

 Abscess formation and discharge of thick profuse pus through multiple

5
sinuses

 Trismus in some patients

 Recurrent healing and break down of sinuses

 Presence of yellow granules in the pus

 In the treatment of cervico-facial actinomycosis, the following have been


found effective:

 Penicillin

 Chloramphenicol

 Clindamycin

 Teracycline

 Amikacin

 Toothache with associated tender swelling of the surrounding cheek and


fever in a 20-year old woman is most likely due to:

 Acute osteomyelitis of a jaw bone

 Gingivitis

 Impacted tooth

 Cellulitis of the cheek

 Periapical (alveolar)abscess

 Osteomyelitis of the jaw usually results from :

 Sickle cell disease

 Dental extraction

 Infection of the antrum

6
 Compound or simple fracture of the jaw

 Radiation of jaw tumour

 Acute osteomyelitis of the mandible:

 Is not as common as that of the maxilla

 Is usually caused by staph pyogenes, streph-haemolyticus or bacteriodes

 May cause trismus

 After a week show a rarefaction of the bone

 Is best treat by a 14-day course of penicillin,or ampicillin or cloxacillin

 A fibrous epulis:

 Is a true gingival tumour

 Microscopically contains fibroblasts, lymphocytes, plasma cells and a


covering of stratified squamous epithelium

 Usually follow prolonged irritation by the margin of a carious cavity,


calculus on a tooth or of an ill fitting denture

 Is usually sessile but may be pedunculated

 May recur unless it is excised with thorough curettage of the base

 Giant cell epulis:

 Is an inflammatory hyperplasia of fibroblasts with numerous multi-


nucleated giant cells found on the gingiva

 Of the vascular variant has a stroma traversed by sinusoidal spaces filled


with red blood cells

 Is common in adults

 Of the vascular type is soft, well-pedunculated and purple to deep red in

7
colour

 Grows rapidly

 Granuloma gravidarum:

 Occurs in the gum of pregnant women with gingivitis

 Is hard

 Is red

 Is lobulated

 Should be removed as soon as it is diagnosed during pregnancy as it


continues to grow

 Odontomes include:

 Cyst of eruption

 Ameloblastoma

 Cementoma

 Dentinoma

 Dental cyst

 A 25-year old man has a swelling of the ramus of the right mandible. The
surface is lobulated and egg shell crackling is elicited.

It may be:

 Periodontal cyst

 Ossifying fibroma

 Ameloblastoma

 Osteoclastoma

8
 Odontogenic myxoma

 Periodontal (dental) cyst:

 Is caused by proliferation of the odontogenic epithelial remnants following


stimulation by chronic periapical inflammation

 Is the least common cyst of the jaw

 Affects the maxillary second incisor most frequently

 Has a missing tooth

 Is treated by excision of the affected jaw

 Dentigerous cyst:

 Develops around the crown of an un-erupted tooth

 Occurs in people over 25 years

 Presents as a smooth swelling with a thick outer wall usually on the labial
side of the alveolar part of the jaw

 On X-ray shows a radiolucent swelling without a tooth

 Is treated by removing the cyst lining and tooth and filling the cavity with
bone chips

 Ameloblastoma:

 Arises from the epithelium of the enamel and is always fast-growing and
solid

 Occurs commonly in those between 20 and 50years

 In most patient is in the molar or ramus region of the mandible

 Has a lobulated surface and the overlying teeth are usually not affected

 May metastasize through the blood stream to the lungs

9
 Microscopically ossifiying fibroma shows:

 Giant cells

 Irregularly arranged interlacing collagen fibres

 Osteoblasts

 Proliferating fibroblasts

 Area of calcification

 Ossifying fibroma of the jaw may present with:

 Pathological fracture

 Cervical lymphadenopathy

 Malocclusion of the teeth

 Dysphagia

 Painful facial swelling

 X-ray of ossifying fibroma of the jaw may show:

 Expansion of the cortex of the jaw

 Radiolucent swelling

 Area of new bone formation in the jaw

 Area of bone destruction in the jaw

 Patchy radio-opacity in the radiolucent swelling

 The ideal treatment of ossifying fibroma of the mandible is:

 Excision of the affected part of the mandible

10
 Hemimandibulectomy

 Radiotherapy

 Complete excision of the tumour

 Complete excision of the tumour followed by radiotherapy

 Osteoclastoma of the jaw:

 Is more common in the maxilla

 Is usually seen between 10-20

 Is rather soft and multicystic

 Does not affect the adjoining teeth

 Never becomes malignant

 The main presenting symptom of fibrous dysplasia of the jaw is:

 Pathological fracture

 Malocclusion of the teeth

 Pain

 Swelling

 Unerupted teeth

 A tumour of the mandible of a 12-year old boy may be due to:

 Osteosarcoma

 Burkitt`s lymphoma

 Fibrous dysplasia

 Periodontal cyst

11
 Dentigerous cyst

CHAPTER 21

 B

 A, C

 B, D

 B, C

 D, E

 A, E

 A, D

 C, E

 A, C

 B, D

 A, C, D, E

 A, B, D, E

 B, E

 A, C, E

 B, D

12
 A, C, D, E

 C, E

 A, C, D

 E

 A, B, D, E

 C, D

 B, C, E

 A, B, E

 A, C, D

 C, D

 C, E

 A

 A, E

 B, C

 B, D, E

 A, C

 A, B, E

 D

 C

 D

 A, B, C, E

13
CHAPTER 22
THE THYROID AND PARATHYROID GLAND

1. In the anatomy of the thyroid gland the:

         A.  Isthmus lies in front of the first, second and third tracheal rings
B. Weight of the gland is 7-25 g in the African
C. “Strap muscles” consist of the sternothyroid, sterno-hyoid and the
inferior belly of the omo-hyoid
D. Medial relations of the lateral lobes are the hyoid, thyroid and cricoid
cartilages
upper six tracheal rings, oesophagus and recurrent laryngeal nerve
E. Superior and middle thyroid veins drain into the internal jugular vein
and the
Inferior thyroid veins into the innominate vein

2. In the physiology of the thyroid:

 T3 is four times more active than T4 by weight


 The iodine in the blood is concentrated 10-25 times by the gland
 T3 and T4 are bound in the blood only to thyroxine binding globulin
 Only about 0.05 per cent of T4 and 0.5 per cent T3 in the blood are
unbound, free and physiologically active
 T.S.H. stimulates the trapping of iodine and synthesis of thyroid
hormones
but not the storage and release of the hormones

3. Probably the most reliable current laboratory test in suspected


hyperthyroidism is:

 Serum T3 concentration
 T3 resin uptake
 Serum T4
 Serum free thyroxine
 Free T4 index

4. The most sensitive test of hypothyroidism is:

 Free T4 index
 Serum thyroid stimulating hormone
 Test of hypothalamic-pituitary axis

1
 Protein bound iodine
 Serum T3 concentration

5. In thyroid scan:

 If radioactive technetium is used, the scan is done at 30 minutes


 If radioactive iodine is used the scan is done at 12 h
 A cold (non-functioning) nodule may be an adenoma, a cyst or a
malignant tumour
 A nodule with most of the isotope concentrated in it is toxic or
autonomous
 A nodular goiter has the isotope concentrated in the nodules

6. Thyroglossal cyst:

 Is always in the midline


 Characteristically rises on protrusion of the tongue but does not move
on swallowing
 Is soft and fluctuant and attached to the skin
 May become infected
 May require multiple skin incisions for its removal and that of its tract

7. Goitre:

 Is an enlarged thyroid due to any cause except malignancy


 Is only palpable when the gland is at least 50 g
 May be euthyroid or hyperthyroid but not hypothyroid
 Afflicts at least 200 million people in the world according to WHO
estimates
 Occurs invariably in mountainous terrain

8. A simple goitre may be caused by:

 Low calcium content of drinking water


 Phenylbutazone
 Tolbutamide
 Some proprietory cough or asthma mixtures
 Paraminosalicylic acid

9. Endemic simple goitre:

 Exists when more than 10 per cent of any community have goitre
 Does not occur in West Africa

2
 Is caused principally by low iodine content of the local soil
 Is caused by cassava containing cyanogenic glucosides in some parts
of Africa
 Affects adolescent boys and girls in roughly the same ratio

10. Sporadic goitre:

 Is pathologically different from endemic goitre


 May be caused by pubertial growth, pregnancy, lactation or drugs
 Is not caused by iodine deficiency in the food
 Is clinically indistinguishable from endemic goitre
 May be caused by deficiency of enzymes necessary for oxidizing
iodine

11. In the pathology of simple goitre:

 There are initially localized areas of hyperplasia and hypertrophy


of the thyroid cells
 During the resting phase the follicles are distended with colloid
 Spurs of epithelium may project into the follicles
 Hyperplasia is followed by atrophy
 Nodules result from cycles of hyperplasia and atrophy

12. A patient with simple nodular goitre may complain of:

 Hoarseness of voice
 Snoring during sleep
 Difficulty in swallowing
 Oedema of face and conjunctivae
 Engorged manubrial subcutaneous veins

13. Simple nodular goitre:

 May occur at any age from the neonate to the elderly but the peak
incidence is from 21 to 30
 Affects women five times more than men
 Is symptomless in the vast majority of patients(apart from the
swelling)
 Is multinodular involving both lobes in about 50 per cent of patients
 Feels soft or firm and has indefinite edges

14. Diffuse hyperplastic goitre:

3
A. Is seen more commonly in endemic areas
B. May affect children
C. Is seen most commonly in girls from around puberty to 20
D. Is a uniformly enlarged and soft thyroid gland
E. Does not become big enough to compress the trachea

15. Complications of nodular goitre include:

 Tetany
 Infection
 Sudden haemorrhage
 Malignancy
 Myxoedema

16. Investigation(s) essential in simple nodular goitre is/are:

 X-ray of neck
 Free thyroxine index
 Serum T3 concentration
 Direct laryngoscopy
 Isotope uptake test

17. A nodular goitre apparently confined to one lobe is treated by:

A. Administration of thyroxine
B. Excision
C. Lobectomy of the affected lobe
D. Administration of Lugol’s iodine
E. Subtotal thyroidectomy

18. Diffuse hyperplastic goitre is usually treated by:

 Administration of thyroxine
 Subtotal thyroidectomy
 Administration of Lugol’s iodine
 Lobectomy
 Radio-iodine

19. Thyrotoxicosis:

 Is a clinical condition resulting always from the stimulating effects of


thyroxine

4
 If primary, is associated with a pre-existing diffuse hyperplastic
goiter
 If secondary, may be associated with malignant thyroid
 May be precipitated by ingestion of large doses of iodine in a patient
with long-standing goitre
 Is uncommon in Africans

20. In thyrotoxicosis:

 There is peripheral polymorphonuclear leucocytosis


 The thyroid and retro-orbital tissues are infiltrated by lymphocytes
and
follicles of lymphocytes may be seen in the thyroid
 Thyroid stimulating antibodies are found in the sera of all patients
 Immunoglogulins and complement are deposited in the stroma of the
thyroid
 There is peripheral lymphocytosis

21. In primary thyrotoxicosis:

 The acini are always small in size


 The acini may be increased in number
 The epithelial cells are always enlarged
 There may be infolding of the tall columnar epithelium into the lumen
of the acini
 The colloid of the acini is scanty

22. The following is/are found in the orbit and extraocular muscles in
exophthalmos

 Erythrocytes
 Plasma cells
 Giant cells
 Polymorphs
 Reticulocytes

23. Symptoms of thyrotoxicosis include:

 Constipation
 Insomnia

5
 Gynaecomastia
 Oligomenorrhoea
 Tiredness

24. SIGNIFICANT symptoms in thryotoxicosis include:

 Preference for cold weather


 Dyspnoea on effort
 Excessive sweating
 Occasional irritability
 Increased appetite and normal weight

25. The IMPORTANT signs in thyrotoxicosis include:

 Moist hands
 Auricular fibrillation
 Lid lag
 Palpable thyroid
 Hyperkinetic movements

26. Exophthalmos:

 Is protrusion or prominence of the eyeball


 Is always bilateral
 Occurs in most patients with primary thyrotoxicosis
 If malignant maybe associated with a euthyroid state
 Is usually painful

27. Feature(s) of exophthalmos is/are:

 Conjunctival oedema
 Diplopia
 Corneal ulceration
 Diminishing visual acuity
 Directly related to the severity of hyperthyroidism

28. Pertibial myxoedema:

 Involves only the pretibial skin


 May not be bilateral
 Is characterized by irregular thickening of the skin with coarse hair
 Is caused by infiltration of the skin with mucin-like substance
 Is associated with myxoedema

6
29. The thyroid gland in thyrotoxicosis:

 Is enlarged in all patients


 If primary is uniformly enlarged and rather soft or firm
 If secondary, is firm and multinodular in all patients
 In over 95 per cent has a thrill and bruit over it
 Shrinks to normal size during treatment with antithyroid drugs

30. In a district hospital in West Africa, the investigations(s) needed to


aid the
diagnosis of thyrotoxicosis in most patients is/are:

 Serum T4, T2 resin uptake, FT41


 Wayne’s clinical diagnostic index
 Isotope uptake test
 Sleeping pulse chart
 Achilles tendon reflex duration

31. Primary thyrotoxicosis and anxiety neurosis can be differentiated


by:

 Weight loss
 Blood pressure
 Sleeping pulse rate
 Restlessness
 Appetite

32. The differential diagnoses of unilateral exophthalmos include:

 Primary thyrotoxicosis
 Meningioma
 Carotid-cavernous fistula
 Optic nerve glioma
 Orbital tumour

33. In the drug treatment of thyrotoxicosis:

 Propranolol reduces anxiety


 Methly thiouracil prevents trapping of iodine and binding of iodine

7
to tyrosine
 Potassium perchlorate prevents trapping of iodine and coupling of
MIT and DIT to T2 and T4
 Carbimazole prevents binding of iodine to tyrosine and coupling of
MIT and DIT to T2 and T4
 Phenobarbitone relieves palpitation and excessive sweating

34. In the drug treatment of thyrotoxicosis:

 The patient usually becomes euthyroid in about 4 weeks


 Treatment is continued at maintenance dose for 6 months after the
patient becomes euthyroid
 50% of patients relapse in 1-6 months after cessation of treatment
 Digoxin may be indicated
 It is inadvisable to give thyroxine as it delays remission

35. In the treatment of thyrotoxicosis the adult dose of:

 Carbimazole is 10-15 mg 8 hourly


 Sodium perchlorate is 10-20 mg 8 hourly
 Propyl-thiouracil is 200-400 mg 8 hourly
 Carbimazole at maintenance level is 1-2 mg 8 hourly
 Methylthiouracil is 50-100 mg 8 hourly

36. Toxic reactions of anti-thyroid drugs include:

 Diarrhoea
 Arthralgia
 Skin rashes
 Depression
 Anaemia

37. Drug therapy in thyrotoxicosis is the treatment of choice in:

 Patients with large goitres


 Men
 Adolescents
 Patients with exophthalmos
 Secondary thyrotoxicosis

8
38. Radio-active iodine:

 Depresses the activity of most of the thyroid cells but does not
destroy them
 Is fully effective in 4 weeks
 May induce carcinomaof thethyroid in the elderly
 Is only given to patients over 55 years
 Leads to hypothyroidism in 40-50% of the patients in 5 years

39. Indications for subtotal thyroidectomy in thyrotoxicosis include:

 Relapse after previous drug therapy


 Recurrence after previous operation
 Drug goitre
 Thyrocardiac
 Multinodular goitre

40. Subtotal thyroidectomy in thyrotoxicosis:

 Gives rapid and permanent cure


 Involves removal of three-quarters of the thyroid
 Is the elective treatment in all patients in the developing countries
because patients may not take their drugs
 Is contra-indicated if the gland is impalpable
 Should be preceded by laryngoscopy as about 15% of patients may
have cord paralysis without clinical manifestation

41. Propranolol:

 Is an alpha-blocking agent
 Controls the symptoms and signs of thyrotoxicosis because it can
depress the production of thyroxine
 Is effective within 24 h of administration and the dose is80 mg orally
daily
 Is usually given for 4-7 days before and 10-14 days after subtotal
thyroidectomy
 Is currently preferred to the usual anti-thyroid drugs for making toxic
patients euthyroid

42. Soon after the removal of the endotracheal tube after subtotal

9
thyroidectomy
for secondary thyrotoxicosis, a patient is found to be having
severe difficulty
in breathing.

The most likely cause is:

 Collapse of the trachea


 Haemorrhage
 Oedema of the larynx
 Damage to both recurrent laryngeal nerves
 Massive collapse of a lung

43. You will do the following:

 Pass a bronchoscope and aspirate any mucus plugs


 Open the wound immediately
 Reinsert the endotracheal tube
 Give oxygen
 Do a tracheostomy

44. Respiratory obstruction due to post-operative haemorrhage after


subtotal tyroidectomy:

 Usually occurs within 24 h after operation


 Is due to collection of blood in the subcutaneous space
 Is prevented by a well-placed corrugated drain
 Occurs only if there is considerable collection of blood in the wound
 Is best treated by the team that performed the operation

45. Recurrent laryngeal nerve paralysis:

 Is permanent
 If unilateral causes hoarseness of voice and non-productive cough
 If bilateral causes severe hoarseness of voice and difficulty in
breathing
 If bilateral requires early repair of the nerves
 If bilateral and there is no recovery after one year, requires
arytenoidectomy with lateral fixation of one cord

10
46. Hypoparathyroidism after subtotal thyroidectomy:

 Usually occurs 10-14 days post-operatively


 Causes paraesthesia and numbness of the distal parts of the limbs
 Causes weakness of the limbs
 Is controlled with vitamin D
 Is usually temporary as all the parathyroids are removed only rarely

47. Thyrotoxic crisis after subtotal thyroidectomy for thyrotoxicosis:

 Is fairly common
 Is caused by excessive release of TSH in the post-operative period
 Is characterized by excitation, profuse sweating, hyperpyrexia and
and tachycardia
 Is treated with propranolol, Lugol’s iodine, carbimazole, diazepam and
intravenous fluids
 May require oxygen administration and cold sponging

48. In the late complications of subtotal thyroidectomy for


thyrotoxicosis:

 Hypothyroidism occurs in about 10% of patients in 2-5 years


 Recurrence of thryrotoxicosis occurs in about 2% of patients
 All the recurrence occur in the first year post-operatively
 Hypothyroidism is best treated with thyroid extract
 Tracheal compression from fibrosis around it may occur

49. In the treatment of severe, progressive exophthalmos:

 Ophthalmic solutions of guanethidine or bethanidine in methylcellulose


is instilled locally for adrenergic blockade
 Steroids are given in large doses to reduce oedema and for
Immunosupression
 Metronidazole is given for its anti-microbial effect
 Lateral tarsorrhhaphy may become necessary to protect the eye
 Transcranial orbital decompression must be done if the eyelids
cannot be apposed

50. Thyrotoxicosis in pregnancy may cause:

11
 Abortion
 Postmature labour
 Stillbirth
 Eclampsia
 Foetal goitre and hypothyroidism

51. In the treatment of thyrotoxicosis in pregnancy:

 Drug treatment is effective


 The drugs should be discontinued in the last 2 months of pregnancy
because of their possible effects on the foetus
 The baby’s T.S.H. and FT4 should be done after birth
 Subtotal thyroidectomy is effective
 Subtotal thyroidectomy should be done in the first trimester and
post-operative thyroxine given to prevent hypothyroidism in the foetus

52. Thyrotoxicosis in children:

 Is best treated by antithyroid drugs for 3 years


 May relapse in most of the patients after
 drug therapy, further treatment
being required
 After drug treatment, permanent remission occurs in 50-75% of
patients
 May present as chorea or emotional disturbance
 May be treated by operation if there is relapse

53. Subtotal thyroidectomy for thyrotoxicosis in children has a:

 Low incidence of hypothyroidism


 Low incidence of recurrence
 High incidence of tetany-about 17%
 High incidence of permanent hypoparathyroidism-about 10%
 High incidence of damage to the recurrent laryngeal nerves

54. Hypothyroidism:

 Is fairly common in Africa


 May be due to excessive ingestion of P.A.S., phenylbutazone

12
or aspirin
 If congenital is always associated with a goitre
 In the adult is not associated with a goitre
 If congenital causes mental and physical underdevelopment
sometimes
with deaf-mutism

55. Clinical features of hypothyroidism in the adult include:

 Paraesthesia with muscle pain


 Diminished sweating but tolerance to cold
 Hoarse voice
 Puffiness of supraclavicular fossa, wrist and periorbital area
and fine hair
 Pulse lower than 80/minute and normal ankle jerks

56. Suppurative thyroiditis:

 Usually occurs in goitrous nodules often after previous


haemorrhage
 Is most commonly caused by staphylococcus
 Presents with acute pain in a previously painless goitre with dysphagia
and often earache
 On examination gives a very tender warm goitre which does not move
on swallowing
 Is treated by antibiotics and needle aspiration of any abscess

57. In auto-immune thyroiditis (Hashimoto’s disease):

 Auto-antibodies to thyroxine-binding globulin are the cause


 The parenchyma of the thyroid is destroyed in some parts and there
are no areas of hyperplasia
 The interstitial tissue is infiltrated with plasma cells and lymphocytes
with active germinal centres
 There may be areas of fibrosis between the lobules
 The gland in all patients is atrophic and replaced by fibrous tissue and
lymphocytes

58. Auto-immune thyroiditis may present with:

13
A. Pain or discomfort in the thyroid area of the neck
B. Severe hyperthyroidism
C. Hypothyroidism
D. Nodular goitre
E. Solitary nodule

59. In the management of auto-immune thyroiditis:

A. The diagnosis can only be established by needle biopsy


B. Prednisone is given for about 10 days
C. Thyroxine is given for 1-3 months
D. Subtotal thyroidectomy may become necessary
E. A course of antibiotics may be benefical

60. Adenoma of the thyroid:

 Is more common in men


 Is firm, encapsulated and movable within the thyroid
 May be functional and “hot” or non-functional and “cold”
 Does not cause thyrotoxicosis
 Is treated by excision

61. Malignant neoplasm of the thyroid:

 Is fairly common
 Is seen from childhood to old age with a peak incidence in Africa
In the 4th decade
 Is 3 times more common in men
 May arise in a nodular goitre
 May present with hypothyroidism

62. In the aetiology of malignant neoplasms of the thyroid:

A. Children exposed to irradiation of the head and neck may develop


carcinoma 20 years later
B. About 38% of those who were exposed to the atom bomb in Hiroshima
and Nagasaki developed carcinoma of the thyroid
C. Sustained T.S.H. stimulation in endemic areas induces predominantly
papillary carcinoma
D. Sustained T.S.H. stimulation in non-endemic areas induces

14
predominantly
mixed papillary and follicular carcinoma
E. Heredity is a factor in some cases of medullary carcinoma

63. Papillary adenocarcinoma:

A. Is the commonest carcinoma of the thyroid in the world


B. Accounts for most cases in children
C Has the highest incidence in the 2nd decade
D. Is slow-growing and remains localized for a long time
E. Metastasizes usually to the cervical and upper mediastinal lymph
nodes and the blood stream

64. Follicular adenocarcinoma:

A. Is the commonest carcinoma of the thyroid in Nigeria


B. Has the highest incidence in the 6th decade
C. Is slow-growing and usually osteosclerotic when it metastasizes
to bone
D. Contains colloid and its metastases can concentrate radio-iodine
E. May present with a pathological fractures

65. Medullary carcinoma:

 May be associated with “café au lait” spots and neurofibromata


 Is often inherited as a recessive genetic trait
 May secrete histamine, prostaglandins, somatostatin, serotonin
and calcitonin
 Has the highest incidence in the 6th and 7th decades
 Spreads by the lymphatics and blood vessels

66. Anaplastic adenocarcinoma:

 May present with respiratory obstruction or dysphagia


 Consists entirely of spindle-shaped small cells
 Spreads early by direct infiltration and only later via the lymphatics
and blood vessels
 Occurs in all age groups especially the young
 Is radio-sensitive

15
67. A 35-year old woman presents with a solitary thyroid nodule which
has been
present for about 4-years, but has recently been enlarging. It is firm,
rather
circumscribed and fairly mobile. There are no enlarged cervical
lymph nodes.

You will consider:

A. A cyst
B. An adenoma
C. Nodular goitre
D. Papillary or follicular adenocarcinoma
E. Anaplastic carcinoma

68. If the scan shows it to be “hot” with some uptake in the rest of the
thyroid,
you will consider:

A. A cyst
B. An adenoma
C A papillary or follicular adenocarcinoma
D. Nodular goitre
E. An autonomous nodule
69. If the scan shows it to be “cold”, you will consider:

 A cyst
 An adenoma
 A papillary or follicular carcinoma
 Nodular goitre
 An autonomous nodule

70. Carcinoma of the thyroid may present as:

 Horner’s syndrome
 Cervical lymphadenopathy without thyroid swelling
 A pulsatile skull swelling
 Hyperthyroidism
 Diarrhoea

16
71. Measures taken in the treatment of operable papillary or follicular
carcinoma
of the thyroid include:

 Near-total thyroidectomy
 Irradiation
 Radio-active iodine
 Cytotoxic therapy with adriamycin or bleomycin
 Administration of thyroxine

72. Treatment measure in anaplastic carcinoma of the thyroid include:

 Near-total thyroidectomy
 Irradiation
 Radio-iodine
 Cytotoxic therapy with bleomycin and adriamycin
 Administration of thyroxine

73, In the prognosis of carcinoma of the thyroid:

 84% and 60% of those with papillary carcinoma are alive at 5 and 30
years
 60% and 40% of those with follicular carcinoma are alive at 5 and 30
years
 No patient with anaplastic carcinoma is alive at 3 years
 Those over 40 have a better prognosis than those under 40
 Women have a better prognosis than men

74. The most frequently encountered symptom (Hall mark) of


thyrotoxicosis is:

 Palpitations
 Profuse sweating
 Progressive weight loss inspire of polyphagia
 Consistent heat intolerance
 Diarrhoea

75. In clinical diagnosis the most dependable sign (Hall mark) of


thyrotoxicosis
is:

17
 Moist hands
 Atrial fibrillation
 A sleeping Pulse Rate > 120/min
 A sleeping Pulse Rate > 90/min
 Pretibital mxyoedema

76. In the Exophthalmos seen in the Hyperthyroidism the:

 Upper eyelid is tangential to the pupil


 Upper eyelid is at least tangential to the upper limbus
 Upper eyelid exposes the sclera in all cases
 Lower eyelid is tangential to the limbus
 Lower eyelid retracts to expose the sclera

77. In a cervical swelling which moves with swallowing, which


combination of
clinical signs supports a diagnosis of thyroid cancer (follicular
carcinoma):

A. Firm, Tender, ill defined


B. Soft non tender, clear edges
C. Firm non tender, clear edges
D. Soft tender well defined
E. Rubbery, non-tender well defined

78. Which of these investigations would you order:

 X-rays of the neck for the thoracic inlet


 Ultrasound scan of the neck swelling
 Radioiodine (T132) uptake of entire gland
 Open tissue biopsy
 Estimate serum level of calcitonin

79. A man of 45 presents with a painless right sided anterior neck


swelling which
moves with deglutition. On examination the lesion is a non tender,
well defined solitary nodule; there were no signs of toxicity, but
patient has hot flushes and diarrhoea

The most likely diagnosis is:

18
 Benign solitary nodule
 Hashinoto’s thyroiditis
 Lymphadenoid goitre
 Papillary carcinoma of the thyroid
 Medullary carcinoma of the thyroid

80. Your investigation of choice is:

 Plain x-ray of neck to show thoracic inlet


 Ultrasound scan of the gland
 FNAC under ultrasound guidance
 Radioactive lodine uptake test
 MRI study of the thyroid gland

81. For treatment you would recommend:

 Daily dosing with L. Thyroxine 0.1 mg od. for 3 months


 Radioiodine therapy
 External Radiation with linear accelerator
 Total Thyroidectomy
 Subtotal Thyroidectomy

82. In the anatomy of the parathyroid glands:

A. The colour is purple


B. They are situated on the medial borders of the thyroid lobes
C. They may be found in the superior, anterior or posterior mediastinum
D. They are usually on the surface of the thyroid but may be embedded in
it
E. There are four but the number may vary

83. Parathyroid hormone:


 Increases resorption of calcium from bone
 Decreases renal tubular reabsorption of calcium
 Increases proximal tubular reabsorption of phosphate
 Promotes absorption of calcium from the gut
 Secretion is controlled by the level of the plasma concentration
of ionic calcium

84. Clinical features of hypoparathyroidism include:

19
 Convulsions
 Stridor and dyspnoea
 Excessive sweating
 Pyrexia
 Restlessness
85. Late complications of hypoparathyroidism include:

 Chronic duodenal ulceration


 Cataract
 Dementia
 Chronic renal failure
 Mental sluggishness

86. Diagnostic laboratory findings in hypoparathyroidism include:

 Low serum phosphate


 High urine phosphate
 Low serum calcium
 Low or absent urine calcium
 High alkaline phosphatase

87. Hyperparathyroidism may be caused by:

 A simple adenoma of the parathyroid in a few patients


 Multiple adenomata in the majority of patients
 Hyperplasia of all the glands in some patients
 Carcinoma of the parathyroid in about 10% of patients
 Chronic renal failure, rickets, Vit. D into-xication or steatorrhoea
in a few patients

88. Hyperparathyroidism:

 Is an uncommon disease
 Is more common in men
 May be seen at any age
 May be symptomless
 May precipitate acute pancreatitis

89. Symptoms of hyperparathyroidism include:

20
 Anorexia
 Nausea and vomiting
 Muscle cramps
 Numbness around the lips, polyuria and polydipsia
 Restlessness

90. Bony features of hyperparathyroidism include:

 Thickening of the mandible or maxilla


 Shortening of the height
 Pathological fractures
 Bone pain and tenderness especially of the spine and shoulder
 Cystic bony swellings

91. Radiological features of hyperparathyroidism include:

 Sclerotic skull
 Subperiosteal resorption of bone seen particularly in the
middle phalanges of the index and middle fingers, upper part
of the tibia and neck of the femur
 Nephrocalcinosis
 Periosteal thickening of the radius and ulna
 Calcification of the spleen

92. In hyperparathyroidism:

 The plasma calcium is over 2.55 mmol/L and may be as high as


5mmol/L
 The plasma phosphate is low, less than 0.8 mmol/L in about 90% of
patients
 The urinary phosphates and calcium are elevated
 The plasma calcium remains elevated when cortisone 150mg is given
daily if primary hyperparathyroidism
 The serum chloride is less than 95 mmol/L and the serum bicarbonate
more
than 24 mmol/L

93. Hypercalcaemia may be caused by:

21
 Excessive intake of antacids
 Multiple myelomatosis
 Sarcoidosis
 Osteitis deformans
 Multiple bone metastases

94. Symptoms of hypercalcaemic crisis include:

 Severe weakness
 Abdominal cramps
 Copious vomiting
 Irritability
 Drowsiness

95. In the management of hypercalcaemia due to acute


hyperparathyroidism:

 3 L of normal saline is given in 9 h to restore extracellular fluid volume,


establish diuresis and form a complex with calcium which is excreted
by
the kidney
 I.V. phosphate 1 g or oral phosphate 3 g daily is given
 Prednisone 30-100 mg daily helps to reduce the serum calcium by
reducing mobilization of calcium from bone
 Haemodialysis may be required
 Exploration of the parathyroids is undertaken as soon as the patient’s
general condition improves

96. In the post-operative management after excision of an adenoma of


the
parathyroid:

 Chvostek’s and Trousseau’s signs are checked 4 hourly


 The serum calcium is checked daily
 The serum phosphate is checked daily
 The urinary excretion of calcium is estimated daily
 If the serum calcium becomes low calcium gluconate, Vit. D2 and
EDTA
are administered simultaneously.

22
CHAPTER 22
THE THYROID AND PARATHYROID GLAND

ANSWERS

 A, B, E

 A, B, D

 D

 B

 C

 D, E

 D, E

 B, C, D, E

 A, C, D, E

 B, D, E

 A, B, C, D, E

 A, B, C, D, E

 C

 A, B, C, D

 C, D

23
 A

 C

 A, B, C

 C, D

 B, E

 B, D, E

 B

 B, C, D, E

 A, C

 A, B, D, E

 A, D

 A, B, C, D

 B, C, D

 B

 B, D

 C, E

 A, B, C, D, E

 D

 A, C, D

 A

24
 B, C, E

 C

 E

 A, E

 A, C, D

 C

 D

 C, D, E

 A

 B, E

 B, E

 C, E

 A

 A, B, D

 A, C

 A, D

 A, C

 C, D

 B, E

 A, C, D

25
 A, C

 A, D

 A, C, D

 B, D

 B, C

 B, D

 A, E

 A, B, D

 A, D, E

 A, C, E

 A

 B

 B, E

 A, B, C, D

 A, B, D, E

 A, C

 B, D

 A, B, E

 D

 D

26
 B, E

 A

 B

 E

 C

 D

 C, D, E

 A, D, E

 A, B

 B, C, E

 C, D

 C, E

 A, C, D, E

 A, B, D, E

 E

 B, C

 A, C, D

 B, C, E

 A, C, D, E

 C, D, E

27
 B

28
SURGERY

CHAPTER 23

THE NECK

 In a cut throat, if the wound is above the level of the hyoid


cartilage the following structures may severed:

A. Epiglotis
B. Common carotid artery
C. External jugular vein
D. Internal jugular vein
E. Brachial plexus

 In the management of a cut throat, the most essential measure is


to:

A. Stop any bleeding


B. Treat for shock
C. Prevent infection
D. Ensure a clear airway
C. Prevent air embolism

 Complications of cut throat include:

A. Horner’s syndrome

 Aerial fistula
C. Dysphonia

1
D. Mediastinitis
E. Surgical emphysema

 Ludwig’s angina:

A. Is severe inflammation of the mouth


B. Is caused by staph aureus

 May cause respiratory obstruction

 Should always be drained

 Is often dental in origin

 A branchial cyst:

 Is persistence of the space formed by fusion of the second with the


fifth
branchial arch

 Contains a clear thin fluid

 Is usually lined by columnar epithelium

 Usually presents in childhood

 Is more common in females

 A branchial fistula:

A. Is usually acquired from rupture of a branchial cyst


B. Is in most cases a true fistula with the internal opening in the tonsilar

2
pit
C. Usually presents around puberty
D. Opens at the junction of the middle and lower thirds of the anterior
border of
the sternomastoid

 Passes between the internal and external carotids

 A cystic hygroma:

 Is a congenital carvenous lymphangioma consisting of a unilocular


cyst
containing clear fluid

 Occurs in the posterior triangle of the lower third of the neck, axilla
or cheek

 May cause respiratory obstruction

 Has indefinite edge

 May regress spontaneously if it becomes infected

 Chemodectoma:

A. Usually benign but 20% are malignant


B. A tumour of the internal carotid artery
C. Soft and transmits the carotid pulsation
D. Located at the level of the hyoid bone deep to the sternomastoid
E. Mobile in all directions

 A firm immobile swelling in the mid-lateral part of the neck of a


neonate is most likely a:

A. Lymph node
B. Lipoma
C. Sternomastoid tumour
D. Aberrant thyroid

3
E. Neurofibroma

 A soft fluctuant, circumscribed, midline swelling under the chin of


a one-year old boy is most probably a:

A. Thyroglossal cyst
B. Dermoid cyst
C. Brancial cyst
D. Sebaceous cyst
E. Ranula

A 25 year old man has a 3 month old, painless swelling below the
angle of the mandible. It is tense, fluctuant mobile, and most
deep to the sternomastoid

 The most likely diagnosis is:

A. Chronic abscess
B. Pharyngeal diverticulum
C. Lipoma
D. Branchial cyst
E. Laryngocele

 Painless, multiple, matted hard swellings with skin attachment in


the mid-lateral part of the neck in a 45-year old man may be due
to:

A. Metastases
B. Hodgkin’s lymphoma
C. Tuberculosis
D. Sarcoidosis

4
E. Multiple neurofibromatosis

 Suspected malignant right supra clavicular lymphadenopathy in


50 years
old man may be due to primary cancer of:

A. Larynx
B. Right lung
C. Stomach
D. Pancreas
E. Right breast

 A firm mobile swelling behind the ramus of the mandible in a 40-


year old man may be:

A. Chemodectoma
B. Lymph node
C. Parotid tumour
D. Submandibular salivary gland tumour
E. Lipoma

 Injuries in the posterior triangle of the neck:

A. Unless directed inferiorly seldom involve vital structures


B. Very frequently involves vital structures
C. Must be explored in all cases
D. Is often associated with dysphonia
E. CT scan is the investigation of choice

 In Ludwig’s angina:

A. Strept viridians and E. coli are the usual organisms


B. Advanced carcinoma of the floor of the mouth may be a causative
factor

5
C. Excessive salivation and trismus are presenting symptoms
D. Broad spectrum antibiotics such as cefuroxime and metronidazole
should be
Administered
E. The tongue is oedematous and is displaced upwards and forwards

 In tuberculous cervical lymphadenitis:

A. The upper deep cervical lymph nodes may be invaded by the


tubercle via the
tonsils in adults and the elderly
B. The lower cervical nodes may be secondarily infected from
pulmonary
tuberculosis via the mediastinal nodes in children and young adults
C. Occasionally all the cervical nodes are affected probably as a result
of blood-
borne invasion
D. HIV/AIDS may be a predisposing factor
E. Lymph nodes in the supraclavicular region or posterior triangle are
affected in
the elderly

 In the investigation of tuberculous lymphadenitis:

A. Lymphopaenia will be found if there is associated HIV

6
B. The ESR may be normal
C. Screening for HIV is not essential
D. The Mantoux test, unlike in pulmonary tuberculosis, is usually
unremarkable
E. Excision biopsy may be necessary to establish the diagnosis

 Branchial cyst:

A. Has abundant lymphoid tissue in the wall


B. Occurs mostly in patients aged 20-25 years
C. Is fluctuant and transilluminates light
D. Protrudes from the anterior border of the lower third of the
sternomastoid
muscle
E. Has to be differentiated from a cyst of the space of Burns

 Brancial fistula:

A. Arises from failure of the second branchial arch to fuse with the fifth
arch
B. Arises from failure of the first branchial arch to fuse with the fourth
arch
C. Has abundant lymphoid tissue in the wall
D. Should be excised through 2 or 3 small transverse incisions over the
course
of the tract
E. Is lined by columnar and ciliated epithelium in its inner portion and
stratified
squamous in its outer portion

 Cystic hygroma:

A. May be present at birth and may cause obstructed labour


B. Is brilliantly translucent

7
C. Is best treated by sclerotherapy as this has been found to be very
effective
D. Is completely compressible
E. Has a 90% chance of recurrence following excision

 Carotid body tumour:

A. Is a rare but rapidly growing tumour of the carotid body


B. Occurs in patients aged 30-50 years
C. Diagnosis is confirmed by Duplex Doppler Imaging
D. Cannot be excised because of its location
E. Is best treated by radiotherapy

8
9
CHAPTER 23
THE NECK
ANSWERS

 A, C, D

 E

 B, C, D, E

 C, E

 A, E

 C, D, E

 B, C, E

 A, D

 C

 B

 D

 A

 A, B, C

 B, C

 A

 A, B, C, D

 C, D, E

10
 A, E

 A, B, C

 A, C, D, E

 A, B

 B, C

11
CHAPTER 59

OPHTHALMOLOGY

Best option out of 5

1. Regarding the anatomy of the walls of the orbit


 The medial wall bones are rarely fractured
 Infraborbital nerve is usually spared in fractures of the floor
 The orbital surface of the zygomatic bone forms posterior part of the
lateral wall
 The orbital plate of the frontal bone together with the body of sphenoid
complete the roof
 The orbital surface of the zygomatic bone contributes to the floor

2. The Optic foramen in addition to the optic nerve transmits


A. Central retinal artery
 Central retinal vein
 Ophthalmic artery
 Ophthalmic vein
 Ophthalmic branch of the 5th cranial nerve

3. An axial CT scan of the orbit at the level of the horizontal muscles is


likely to show all the following
 The crystalline lens
 The optic nerve
 The ciliary ganglion
 The lacrimal gland
 Lesser wing of sphenoid

4. The following may suggest the diagnosis of orbital cellulitis in a


patient with swollen eyelids
 Restricted range of extraocular eye movements
 Raccoon sign
 Purulent nasal discharge
 Bruit on listening over the eye
 Subconjunctival haemorrhage

5. In the management of orbital cellulitis


 Failure of response to antibiotics always indicates resistant organisms
 Systemic steroids are helpful in controlling eyelid swelling
 Oral antibiotics should be started in high doses initially
 Patients who are diabetic need to be switched from oral hypoglycaemic
agents to insulin
 Orbital abscess settles with the appropriate antibiotics

6. Visual disturbance in a patient with proptosis

1
 Optic nerve compression may be indicated by red colour desaturation
 Diplopia is common in axial proptosis
 Photopsia is common in cases caused by primary optic nerve tumours
 Ocular surface lubricants have no effect on visual disturbance
 Is always reversible

7. The main lacrimal glands


 Open into the eye lid margins
 Secretions have acidic pH
 Are similar histologically to the salivary gland
 Are different histologically from the accessory lacrimal glands
 May be excised without any adverse effect on the eye

8. Watering of the eyes


 Is epiphora when it is due to excessive secretion of tears
 Is Lacrimation when it is due to obstruction to outflow of tears
 May be caused by chronic dacryocystitis
 May be caused by emotion or exposure to darkness
 Rarely occurs in Bell’s Palsy

9. The following may indicate inadequate tears in a patient


 Frequent foreign body sensation in the eyes
 Difficulty opening the eyes in the mornings
 Non staining of the cornea with fluorescein drops
 High tear meniscus
 Presence of blepharitis

10. Meibomian glands


 Are found in the tarsal plates of the eyelids
 Are more numerous in the lower than the upper eyelids
 Secretions prevent excessive drying of tears
 May be excised in excessive tearing
 Open into the follicles of the eye lashes

11. Glands found in the eyelids


 Meibomian glands open directly into the tarsal conjunctiva
 Glands of Zeiss are accessory tear glands
 Glands of Moll secrete mucin
 Goblet cells discharge into the lid margins
 May become cystic

12. Fast movement of the eyelids during a blink reflex is enabled by


 The tarsal plate which has a high concentration of mitochondria
 Presence of a loose areolar subcutatneous tissue

2
 Levator palpabral superioris muscle
 Reflex stimulation of the zygomatic branch of the Facial Nerve
 The presence of the Muller muscle which is enervated by the
sympathetic nerve

13. The eyelid margin


 Is sharp where it is opposed to the eyeball
 Has more rows of lashes in the upper than the lower lid
 Contain the openings of the Meibomian glands
 Forms the canthi which are more acute laterally than medially
 Contain the grey line which lies between the anterior edge and the
Meibomian gland orifices
14. Ptosis
 May be congenital
 May be caused by paralysis of the cervical sympathetic nerve
 In Myasthenia gravis is always partial
 May be present without weakness of inervation
 With absent lid crease is always congenital

15. Blepharitis
 Is always an infective condition
 Is often associated with dry eyes
 Treatment include shampooing of the lid margin
 When chronic may lead to loss of eyelashes
 May require prolonged antibiotic treatment

16. A stye
 Is inflammation of the of the Meibomian glands
 Is often caused by Staphylococcal infection
 Is usually painless
 Is best treated by local application of steroid eye ointment
 May be drained by epilation

17. Chalazion
 Is a granuloma of the Meibomian glands
 Is rarely multiple in the same eyelid
 Is a swelling that is fixed to skin
 The underlying tarsal conjunctiva is usually normal
 Is treated by incision and curettage

18. In trichiasis
 The eyelid margin is turned inwards
 Causes include keratitis
 Symptoms of foreign body sensation in the eye are surprisingly rare
 There may be an extra row of eyelashes more posterior than usual
 Epilation often provides a cure

3
19. In ectropion
 The lid margin loses its normal opposition to the globe
 The cause may be lid laxity
 The normal wetting of the cornea is affected
 Watering eyes is often a frequent complain
 Treatment is not necessary as it is self limiting

20. Lagophthalmos
 Is inability to open the eye completely
 May be caused by injury to the Facial nerve
 May lead to glaucoma
 Is often seen in proptosis
 Is treated by padding the eye with sterile gauze.
21. The cornea
 Is a major refractive component of the eye
 Has stratified squamous epithelium
 Is transparent mainly because of the arrangement of the collagen fibres
in its stroma
 Is vascularised in disease processes
 Requires healthy precorneal tear film for its function

22. Corneal oedema


 Is rare because it is not normally vascularised
 Is prevented by regeneration of endothelial cells when damaged
 When diffuse may be caused by raised intraocular pressure
 May be limited to the epithelial cell layer only
 May be responsible for gross thickening of the cornea

23. Clinical features of corneal ulcer include


 Pain
 Watering
 Photophobia
 Dilated pupil
 Circumcorneal redness

24. Treatment of acute suppurative corneal ulcer


 Frequent topical broad spectrum antibiotics
 Pilocarpine eye drops to relieve photophobia
 Initial topical steroids to reduce inflammation
 Padding the eye with antibiotic ointment
 Conjunctival swab for culture and sensitivity testing

25. Herpetic corneal ulcer


 Is often dendritic in the acute stage
 May sometimes be painless

4
 Is rarely super infected by bacteria
 May sometimes require weak steroid eye drops for treatment
 Are effectively treated by Acyclovir ointment

26. Arcus senilis


 Is usually found at the corneal periphery in the elderly
 Is usually a lipid deposition
 Is usually unilateral
 Causes blurring of vision
 Should be treated with Statins

27. Exposure keratitis


 Causes may include ptosis
 May be prevented in the comatose patient by padding with sterile
gauze
 May require tarsorrhaphy in advanced cases
 Is treated with frequent artificial tears eye drops
 May require surgical correction of severe ectropion
28. Ophthalmia Neonatorum
 Often bilateral purulent conjunctivitis occurring in the neonate within 28
days of birth
 Is always caused by gonococcus
 Always occurs during birth
 Eye lids are often swollen and closed
 Does not require systemic antibiotics

29. Complications of Ophthalmia Neonatorum


 Corneal ulceration
 Corneal perforation
 Panophthalmitis
 Anterior staphyloma
 Blindness

30. Follicular conjunctivitis


 Is associated with vernal keratoconjunctivitis
 Is associated with conjunctival papillary hypertrophy
 Follicles are made of locallised aggregation of eosinophils
 Follicles have a core of capillaries
 is found in acute trachoma

31. Acute haemorrhagic (Apollo XI) conjunctivitis


 Was first described in Ghana in 1969
 Is caused by enterovirus
 Has incubation period of 5-7 days
 Is transmitted by finger to eye contact

5
 Topical antibiotics are essential for treatment

32. Features of acute haemorrhagic conjunctivitis


 Acute onset of pain, redness and photophobia
 Scanty lacrimation
 Nearly always bilateral
 Conjuntival follicular reation
 Lid swelling and preauricular lymphadenitis

33. Trachoma
 Is a keratoconjunctivitis
 Rarely causes blindness
 May be eradicated without antibiotics
 Is caused by a chlamydial organism
 Blindness is due to corneal scarring

34. Signs of trachoma include


 Conjuntical papillary and follicular reaction
 Vascularisation of the cornea
 Corneal ulceration
 Irregular small pupil
 Optic atrophy
35. Brownish discoloration of the conjunctiva in a child is suggestive of

 Trachoma
 Vernal conjunctivitis
 Siderosis Bulbi
 Ochronosis
 Onchocerciasis

36. Pinguecula
 Is found in the interpolpebral conjunctiva on either side of the cornea
 Occurs in the young
 Is a degenerative lesion
 May affect visual acuity
 Is treated by excision

37. Pteyigium
 Is an avascular thickening of the conjunctiva encroaching on the
cornea
 Is most common in the tropics and sub tropical regions of the world
 Is most common after 60 years
 Does not affect vision
 Needs excision with conjunctival autografting

38. Sub-conjunctival haemorrhage

6
 May present as a painless sector redness of the eye
 Usually disappears within 2 weeks
 May complicate whooping cough
 Needs to be drained sometimes
 May be found in someone with a fracture of the base of the skull

39. The following may complicate scleritis


 Uveitis
 Retinal detachment
 Glaucoma
 Keratitis
 Cataract

40. Glaucomatous changes in the optic disc include


 Focal thinning of the neuro-retinal rim
 Deepening of the cup
 Splinter haemorrhage on the disc
 Widening of the cup
 Pallor of the neuro-retinal rim

41. Primary open angle glaucoma


 Is the commonest type of glaucoma among people of African descent
 May be genetically determined
 Usually present with severe symptoms
 Causes loss of central vision in the early stages
 May exist in spite of normal intraocular pressure
42. Features of congenital glaucoma include
 Photophobia
 Enlarged cornea and globe
 Corneal haziness
 Marked rise in intraocular pressure
 Breaks in Descemet’s membrane

43. In Primary angle closure glaucoma


 Onset of pain and redness is usually gradual
 The conjunctiva is often dry
 Vision is often severely blurred
 Pain is caused by ischaemia of the iris
 Vomiting is rarely associated

44. The following are known causes of secondary open angle glaucoma
 Conjunctivitis
 Corneal ulcer
 Steroids
 Pigment dispersion in the anterior chamber

7
 Trauma

45. The following may be used in the medical treatment of glaucoma


 Pilocarpine
 Atropine
 Timolol
 Xalatan
 Phenylephrine

46. Treatment of acute angle closure glaucoma include


 Peripheral iridectomy
 Pilocarpine
 Acetazolamide
 Cyclopentolate
 Hypertonic saline

47. Treatment of congenital glaucoma include


 Pilocarpine
 Goniotomy
 Combined trabeculotomy and trabeculectomy
 Peripheral iridectomy
 Glaucoma drainage device implantation

48. Relative afferent pupillary defect


 Is detected by the swinging light test
 Is present in an eye with dense cataract
 May indicate macula scar
 Is useful in the diagnosis of retinal detachment
 May sometimes be found in glaucoma patients
49. Pharmacologic manipulation of the pupil
 Pilocarpine stimulates the sphincter pupilae
 Tropicarmide blocks the dilator pupillae
 Phenylephrine can be used to dilate the pupil
 Atropine effect lasts up to 2 weeks
 Cyclopentolate paralyses the dilator papillae

50. Features of anterior uveitis include


 Pain worsened by light
 Keratic precipitates on the corneal endothelium
 Circumcorneal redness of the conjunctiva
 Small irregular pupil
 Cells in the anterior chamber

51. Treatment of iridocyclitis


 Topical Atropine
 Topical Phynylephrine

8
 Topical steroids
 Padding of the eye
 Topical antibiotics

52. Symptoms suggestive of cataract include


A. Ability to read without ones reading glasses
B. Itching of the eyes
C. Vision becoming more blurred in when out in the sun than indoors
D. Car headlights become more troublesome than before when driving at
night
E. Inability to tell the difference between day and night.

53. Cataract
 Occurs more frequently in old age than from trauma
 May occur more frequently some families than others
 May sometimes be associated with narrowing of the anterior chamber
 Is always associated with poor vision
 Can be prevented sometimes by the appropriate immunization

54. Conditions that may be associated with cataract include


 Diabetes
 Hyperparathyroidism
 Hyperthyroidism
 Systemic steroid use
 Topical steroid use

55. In the surgical management of age related cataract


 The pupil should be constricted before surgery to reduce the
intraocular pressure
 The anterior capsule is not always removed
 The intracapsular extraction gives better results than extracapsular
extraction
 Primary posterior capsulotomy should always be done
 The post-operative refraction can be determined pre-operatively
56. After cataract extraction
 Objects are unduly bright
 Colours appear more vivid than before surgery
 Things appear bluish because of the intraocular lens implanted
 Accommodation is possible with some types of intraocular lens
 Spectacle correction of aphakia makes things larger than they are.

57. Aphakia

9
 May result from couching
 May result in distance vision becoming poorer than near vision
 May be associated with deep anterior chamber
 Causes the pupil to become jet black
 is best treated with intraocular lens implantation

58. Pupillary light reaction


 Direct response is an all or none phenomenon
 Constricts only in response to light
 Can be used to test the integrity of the afferent visual pathway
 The swinging light test can be used to test the consensual response
indirectly
 Is absent in an eye with dense cataract.

59. Pharmacologic manipulation of the pupil


 Effect of Atropine lasts more than 1 week
 Tropicamide paralyses the dilator muscle to cause mydriasis
 Pilocarpine stimulates the constrictor muscle to cause miosis
 Phenylephrine blocks the dilator pupillae
 Cyclopentolate may precipitate angle closure in its action

60. The vitreous humour


 Is made up predominantly of water
 Gradually loses its jelly-like consistency with age
 Separates from the retina in an age related degenerative process
 May not be removed from the eye without adverse effect on the eye
 Is responsible for the symptom complex of flashes and floaters

61. Vision loss caused by vitreous haemorrhage


 Rarely occurs suddenly
 Is associated with defective red reflex
 Is common among people with haemoglobin SC genotype
 Does not resolve spontaneously
 May become irreversible

62. The normal retina


 Is transparent
 Is supplied by the Central Retinal Artery only
 Has nerve fibres which are non myelinated
 Is most sensitive to light at the macula
 Has an inseparable junction between the pigment epithelial and the
 neurosensory layers.
63. Retinal lesions found in acute close globe ocular injury
 Retinal haemorhage
 Retinal oedema

10
 Retinal tears
 Retinal exudates
 Retinal venous occlusion

64. Concerning retinal haemorrhages


a. Most arise from capillaries and post capillary venules
b. Retinal arterioles rarely bleed
c. Haemorrhages in the retinal nerve fibre layer appear flame shaped.
d. They produce lasting damage after they have been absorbed.
e. Roth spots are white centred haemorrhages

65. Cotton wool spots


 Result from ischaemia of the retinal nerve fibre layer
 May be found in hypertensive retinopathy
 Leave marks on the retina on resolution
 May be found in HIV/AIDS patients
 Do not occur in collagen vascular diseases

66. Ophthalmoscopic findings in central retinal artery occlusion include


 Arteriolar narrowing
 Dilatation of retinal venules
 Numerous retinal haemorrhages
 Milky white retina
 Oedematous optic disc

67. Retinal changes associated with hypertension include


 General arteriolar narrowing
 Compression of veins at the arterio-venous crossings
 Flame shaped haemorrhages
 Optic atrophy
 Retinal arteriolar emboli

68. The following retinal lesions may be found in Diabetes


 Microaneurisms
 Narrowing of veins
 Neovascularisation
 Vitreous haemorrhage
 Retinal detachment

69. Vision loss in Diabetes


 Is most commonly due to macular oedema
 Is irreversible
 Is more likely the longer the duration of the diabetes
 May be prevented with tight glycaemic control
 Is more likely if the individual has uncontrolled hypertension
70. Prevention of blindness due to diabetes

11
 Ask the patient to see the Ophthalmologist as soon as soon as they
notice any visual loss
 Laser treatment for any exudates in the retina
 Regular eye examination by an ophthalmologist
 Topical treatment o retinopathy using steroids
 Pan retinal laser photocoagulation for new vessels on the disc

71. Sickle cell retinopathy


 Is more common in patients with SS than SC disease
 Occurs more frequently in the posterior pole of the retina
 May cause blindness which is irreversible
 Often cause vitreous haemorrhage
 Often is an exudative retinopathy

72. Management of vision loss in sickle cell retinopathy


 Conservative management till vitreous haemorrhage clears
 Vitrectomy for non clearing vitreous haemorrhage
 Adequate hydration and prevention of acidosis
 Cryotherapy of peripheral retinal neovascularisation
 Prescription of spectacle lenses

73. Retinal detachment


 Results from separation of retinal pigment epithelial layer from
neurosensory retina
 Is a cause of sudden painless loss of vision
 Is seen on ophthalmoscopy as transparent wavy membrane
 May lead to irreversible blindness
 Is often associated with relative afferent papillary defect

74. Retinoblastoma
 Is the commonest tumour of the retina in childhood
 Occurs most commonly after age 5 years
 Is always unilateral
 In the early form presents with pain and redness of the eye
 Is almost invariably fatal

75. Concerning the innervations of eye muscles


 The superior oblique is supplied by the trochlear nerve
 The lateral rectus is supplied by the abducent nerve
 The inferior oblique is supplied by IV cranial nerve
 The supply to the ciliary muscles are by the III cranial nerve of the
opposite side
 The upper eyelid muscles are supplied by 4 nerves

76. Causes of Esotropia


 Head tilting

12
 Excessive use of accommodation in high hypermetropia
 Congenital myopia
 Loss of vision in the eye before age 4 years
 Enophthalmos
77. Detection of various forms of strabismus
 Alternate cover test detects alternating strabismus
 Latent strabismus is detected by eliminating simultaneous perception
 Intermittent strabismus is diagnosed from the history
 In esotropia, the corneal light reflex is displaced nasally
 In pseudo strabismus, the corneal light reflex is central

78. In alternating strabismus


 The deviating eye alternates between eso and exo deviations
 Either eye can take up fixation
 The vision in each eye is normal
 The image of the deviating eye is usually suppressed
 Diplopia is usually present

79. The following may indicate the presence of optic nerve disease
 Red colour desaturation
 Reduced light brightness sensitivity
 The presence of relative afferent papillary defect
 Near vision more affected than distance vision
 New vessels on the optic disc

80. A patient found to have papilloedema


 May also have prolonged history of headache
 May have no intracranial space occupying lesion on imaging
 May also have strabismus
 Always has reduced vision
 May have orbital optic nerve compression

13
Chapter 59 Answers

 A 43. C,D

 C 44. C,D,E

 A, B, C, E 45. A,C,D

 A, C 46. A,B,C

 D 47. B,C,E

 A 48. A,D,E

 C, D, E 49. A,C,D

 C 50. A,B,C,D,E

 A, B, E 51. A,C

 A, C 52. A,C,D

 B, E 53. A,B,C,E

 B,D 54. A,B,D,E

 A,B,C,D,E 55. E

 A,B,D 56. A,B,C,D,E

 B,C,D,E 57. A,C.D.E

 B,E 58. C,D

 A, E 59. A,C,E

 D 60. A,B,C,E

 A,B,C,D 61. B,C,E

14
 B 62. A, C,D

 A,C,D,E 63. A,B,C

 C,D,E 64. A,B,C,E

 A,B,C,E 65. A, B,D

 A 66. A,D

 A,B,D,E 67. A,B,C,D

 A,B 68. A,B,C,D,E

 C,D,E 69. A,C,D,E

 A,D 70. C,E

 A,B,C,D,E 71. C,D

 E 72. A,B,D

 A,B,C,D 73. A,B,D,E

 A,C,D,E 74. A

 A,C,D,E 75. A,B

 A,B,C 76. B,D

 B 77. B,C,E

 A,C 78. B,C,D

 B,E 79. A,B,C

 A,B,C,E 80. A,B,C

 A,B,C,D,E

 A,B,C,D

 B,E

 A,B,C,D,E

15
CHAPTER 53

INJURIES TO BONES AND JOINTS

1. Which of the following is/are TRUE of fractures


 A multi-fragmented fracture is one in which there are more than one
fracture at different sites in the same bone.
 An oblique fracture results from compression forces acting on the
cortex
 An impacted fracture is one in which the fractured ends are interlocked
and is caused by a compression forces
 A greenstick fracture is one in which both cortices bend
 A stress fracture is caused by repeated applications of a load below the
ultimate strength of the bone
2. In the healing of a fracture
 Bleeding from the fracture site and soft tissues forms a haematoma
and fibrin clot that is a source of haemopoietic cells which can secrete
growth factors GF
 Non-steroidal anti-inflammatory drugs (NSAIDs) enhances the healing
of a fracture
 The reparative response occurs between 2days and 2 weeks
 Ultrasound can decrease the time to clinical healing and radiological
union
 Remodeling is completed in a year
3. The time taken for a fracture to unite:
 In an adult upper limb is about 6 weeks
 In an adult lower limb is about 18 weeks
 In a lower limb in a child is about 12 weeks
 In an upper limb in a child is about 3 weeks
 Is shorter in a transverse fracture than in an oblique fracture
4. In the healing of a fracture
 Callus becomes radiologically visible after 3 weeks
 External bridging callus ossifies by endochondral ossification to form
woven bone
 Infection of the fracture site leads to low oxygen tension which will
cause excessive formation of cartilage and fibrous tissue
 Callus formed will remain a permanent part of the bone
 Cancellous bone heals poorly compared to dense cortical bone
5. Open reduction and internal fixation of a fracture:
 Should be done ideally 4-7 days after injury

1
 Allows early mobilization of muscles and joints thereby preventing
wasing of muscles and stiffness of joints
 Reducses the incidence of deep venous thrombosis
 Promotes early healing of the fracture
 Should be delayed until other life threatening injuries have been treated

6. The following are causes of delayed union of a fracture:


 Poor blood supply to the fracture as a result of excessive soft tissue
stripping
 Excessive mobility at the fracture site
 Early mobilization of joints and muscle at the fracture site
 Distraction of the fracture ends by excessive traction
 Large doses of Vit. A
7. In non-union of a fracture:
 Clinically presents as painful movement at the fracture site
 There is a gap between the fracture fragments, which may be
considerable and is pathologically filled with cartilage.
 Bone growth may be stimulated with autogenous bone graft at the
fracture site.
 The cause maybe inadequate blood supply to the fracture.
 A spiral fracture has a higher risk of non-union than a transverse
fracture
8. The most serious complication of an open fracture is:
 Haemorrhage
 Infection
 Delayed union
 Non-union
 Ischaemic contracture of muscles
9. The following are indications for open reduction and internal fixation
of fractures
 Pathological fractures
 Multiple fractures
 Open fractures with severe contamination
D. Fractures of the patella
E. Fractures complicated by compartment syndrome
10. Three days after sustaining a fracture of the shaft of the femur, a 50-

2
year old man
becomes confused, dyspnoeic and has a low-grade fever. Some
petechial rashes are observed. You will suspect most strongly:

A. Lobar pneumonia
 Septicaemia
 Pulmonary embolism
 Fat embolism
 Cerebrovascular accident

11. What the patient needs most is:

A. Antibiotics
 Oxygen
 Heparin
 Sedation
 To be left alone

12. X-ray appearance of a sclerotic, smooth and well-defined fracture


margins is
evidence of:

 Callus
 Delayed union
 Union
 Non-union
 Avascular necrosis
13. Fracture of the clavicle:
 Occurs most commonly in the middle third and usually as a result of
direct violence
 If in the middle third, the medical fragment is pulled upward and
backward and the lateral fragment downward and forward
 Causes the patient to tilt his head to the contralateral side
 Is best treated with a “figure-of-eight” bandage which is re-applied
every other day and retained for six weeks
 Is commonly complicated by non-unions and mal-unions.
14. Fractures of the scapula:
 Are caused by direct violence
 Are usually associated with fractures of the underlying ribs

3
 Seldom cause severe displacement of the fragments
 Are best treated by the use of a sling for 3 weeks followed by active
mobilization exercises.
 The best treatment is open reduction and internal fixation with plates
and screws
15. Acromio-clavicular joint dislocations:
 Is commonly caused by a fall or blow on the tip of the shoulder
 Is common among horse riders and car drivers
 Causes downward displacement of the clavicle and upward
displacement of the acromion
 X-rays should be taken with the patient standing
 Is best treated by the use of a sling even when displacement is
considerable.
16. The shoulder joint is:
 Held together by a strong but thin capsular ligament which is lax
superiorly
 Strengthened anteriorly by the subscapularis tendon
 Strengthened inferiorly by the infraspinatus tendon
 Strengthened superiorly by the supraspinatus tendon
 Weakest in the superior aspect

17. The Bankart lesion is:


 Fracture dislocation of the shoulder joint
 A tear in the capsule of the shoulder
 Detachment of the capsule of the shoulder joint from the scapula
usually with the glenoid labrum
 A tear in the capsule of the shoulder joint and rotator cuff muscle
tendons
 Detachment of the capsule of the shoulder joint from the humerus
A 40-year old man after falling on the outstretched hand has severe
pain and tenderness in the right shoulder and supports the right
elbow with the left hand. The right arm is found to be abducted and
the tip of the shoulder flat.

4
18. The most likely clinical diagnosis is:
 Acromio-clavicular dislocation
 Posterior dislocation of the shoulder
 Fracture of the surgical neck of the humerus
 Anterior dislocation of the shoulder
 Fracture of posterior half of the head of the humerus
19. The lesion which is commonly associated with this injury is damage
to the:
 Median nerve
 Ulnar nerve
 Radial nerve
 Axillary nerve
 Brachial artery
20. Complications of anterior dislocation of the head of the humerus
include:
 Injury of the axillary artery
 Injury to the supraspinatus and other rotator cuff muscle
 Fractures of the greater tuberosity
 Recurrent dislocations of the shoulder
 Injury to the pectoralis major
21. The pathological findings in recurrent dislocations of the shoulder
include
 Tear of the supraspinatus
 Postero-lateral notch in the humeral head
 Detachment of the glenoid labrum and capsule from the scapular neck
 Anterior pouch of the capsule under the coracoid process
 Laxity of the infraspinatus muscle

22. In Kocher’s manoeuver for reducing anterior dislocation of the


shoulder:
 The patient is placed supine on the couch
 The elbow is extended and a gentle traction applied on the arm while it
is externally rotated fully

5
 The arm is then adducted sweeping the elbow across the body to the
midline
 The arm is then internally rotated so that the hand falls near the
opposite shoulder
 A collar and cuff sling is then applied
23. The Bankart’s operation is used for:
 Luxatio erecta
 Posterior dislocation of the shoulder
 Fracture dislocation of the shoulder
 Fracture of the tuberosity of the humerus
 Recurrent dislocation of the shoulder
24. In a tear of the supraspinatus tendon, the typical feature is:
 Tenderness over the shoulder
 Stiffness of the shoulder
 Inability to initiate abduction, but when abduction is passively initiated
the patient is able to continue
 Inability to adduct the shoulder
 Palpable defect in the supraspinatus tendon
25. In the treatment of fractures of the surgical neck of the humerus
 If the fracture is impacted, the arm is put in a sling and daily active
assisted movements started
 If the fracture is impacted and the displacement is minimal, the arm is
put in a plaster-of- Paris U-slab and strapped to the body for 3 weeks
after which active assisted movements are started
 If displacement is severe, manipulation is done and the arm held in
plaster-of-Paris shoulder Spica for 6 weeks.
 If there is associated dislocation of the head, open reduction is often
necessary
 If the axillary nerve is injured, exploration and open reduction are
urgent
26. If a patient suspected of having fracture of the shaft of the right
humerus is
unable to extend the wrist and fingers, you will:

 Suspect an associated fracture of the shafts of the radius and ulna


 Suspect associated injuries of the radial and ulnar nerves
 Suspect associated injury of the radial nerve
 Explore any nerve suspected to be injured and perform an open
reduction of the fracture
 Treat the fracture by closed methods and splint the wrist and fingers in
a position
of function

6
27. A child of 6 after falling on the outstretched hands has a painful
tender swelling
of the right elbow. You will:

 Suspect MOST strongly separation of the medial epicondyle or


dislocation of
the elbow
 Check the radial pulse
 Examine for radial nerve injury
 Check the patient’s ability to move the fingers
 Test for anaesthesia of the median and ulnar nerve distribution
A girl of 6 who fell on the outstretched right hand presents with
painful swelling of the elbow. She is unable to move the joint. The
medial and lateral epicondyles and the point of the olecranon are
normally related to each other.

28. The most likely clinical diagnosis is:

 Dislocation of the elbow


 Supracondylar fracture
 Fracture of the neck of the radius
 Fracture of the olecranon process
 Separation of the lateral condyle
29. Your next step will be to:

 Take an x-ray to confirm your diagnosis


 Examine the forearm for any neurological disorder
 Check the radial pulse
 Examine the shoulder for any lesion
 Check the brachial pulse
30. After correcting the injury and immobilizing the arm, your next most
important
step will be to:

 Do a check x-ray
 Examine the hand for swelling
 Check the radial pulse
 Check the brachial pulse
 Check for any neurological lesion
31. In a supracondylar fracture:

7
 The lower fragment in 90% of patients is displaced backwards and
upwards and may be rotated medially
 The brachial artery may uncommonly be injured and requires treatment
within 6 hours of the injury
 Stiffness of the elbow is common and is treated by forced extension,
passive movements and active exercises
 Mal-union is common and usually requires open reduction
 The median nerve is commonly injured

32. In Volkmann’s ischaemic contracture of the forearm:


 The most common cause is supracondylar fractures of the humerus,
but
fractures of the shafts of the radius and ulna may also be a cause.
 The forearm muscles are fibrosed and contracted
 The fingers are stiff, contracted and clawed
 There is no loss of skin sensation
 Operative treatment is not necessary in most patients
33. Myositis ossificans of the elbow:
 Is a common complication of supracondylar fractures
 Presents as increasing immobility of the elbow a few weeks afer injury
 Gives rise to a painful and very stiff elbow
 Is treated by stretching of the elbow followed by passive and active
movements
 Is best diagnosed by ultrasound imaging
34. Fractures of the lateral condyle of the humerus:
 Is common in adults and occasionally seen in children
 Is caused by direct violence on the elbow
 May lead to cubitus valgus and ulnar neuritis
 Usually requires open reduction for accurate reduction which is
important
 Is often complicated by ulnar nerve injury
A 20-year old man falls on the outstretched right arm. He complains
of pain around the elbow and it is found that the olecranon is
unusually prominent. The line from the medial epicondyle through
the tip of the olecranon to the lateral epicondyle is angulated.

35. The most likely clinical diagnosis is:


 Supracondylar fracture

8
 Fracture of the olecranon process
 Dislocation of the elbow
 Monteggia fracture dislocation
 Fracture of the head of the radius
36. In dislocation of the elbow:
 The cause is usually direct violence to the elbow
 The radius and ulna are displaced forward in relation to the condyles of
the humerus
 The line from the medial epicondyle through the tip of the olecranon to
the lateral epicondyle is straight
 A back slab is applied for 6 weeks after reduction
 Recurrence may be due to associated fracture of the coronoid process

37. In fracture of the head of radius:


 There is severe pain on the lateral aspect of the elbow
 Rotation-supination movements are particularly painful
 The radial artery may be damaged
 Treatment in all patients is surgical excision of the head
 Joint stiffness and osteoarthritis are common complications.
38. In fractures of the shafts of the radius and ulna:
 An isolated fracture of the ulna or radius is unusual
 The entire shafts including the wrist and elbow joints should always be
x-rayed
 The proximal fragments are pronated if the fractures are in the upper
third
 If the fractures are below the mid-shaft, the proximal and distal
fragments
should be held in mid-pronation to get them in line
 Union usually occurs in the adult in 8-12 weeks
39. Features of Colles’ fracture include:
 Fracture of the distal 6cm of the radius
 Dorso-lateral displacement of the distal fragment
 Dorsal rotation of the distal fragment
 Fracture of the ulna styloid
 Falling on an outstretched hand
40. Pain and swelling over the wrist following an injury may be due to:

9
 Madelung’s deformity
 Fracture of the radial styloid
 Fracture of the scaphoid
 Smith’s fracture
 Monteggia fracture
41. In a Colles’ fracture:
 The wrist is immobilized in palmar flexion and ulnar deviation after
reduction
 An above elbow plaster is usually applied especially if the fracture is
comminuted
 Immobilization is usually for 3-4 weeks
 Complications include reflex sympathetic dystrophy, shoulder-hand
syndrome, rupture of the extensor pollicis longus tendon and
Volkmann’s ischaemic contracture of the fore-arm
 Is most common in middle-aged men
42. In Smith’s fracture:
 There is a fracture of the distal end of the radius
 The distal fragment is displaced posteriorly
 There is anterior rotation of the distal fragment
 The cause is a pronation injury produced on falling on the flexed wrist
 After reduction, an above elbow plaster is applied with the wrist in full
supination and slight dorsiflexion

43. A Monteggia fracture is:


 Transverse fracture of the upper third of the ulna
 Transverse fractures of the upper third of the ulna and lower third of
the radius
 Transverse fracture of the lower third of the ulna with dislocation of the
proximal radio-ulnar joint
 Transverse fracture of the upper third of the ulna with dislocation of the
proximal radio-ulnar joint
 Transverse fracture of the upper third of the radius with dislocation of
the inferior radio-ulnar joint.
44. A Monteggia fracture is:
 Caused by a fall on the outstretched hand
 Produced by a blow on the forearm
 Markedly angulated either anteriorly or posteriorly

10
 Usually treated by closed manipulation and open reduction being
necessary
only occasionally
 If accompanied by fracture of the head of the radius in a child is treated
by excision of the head of the radius and intramedullary nailing of the
ulna
45. A Galleazzi fracture is:
 Fracture of the radius with dislocation of the proximal radio-ulnar joint
 Produced by a blow on the forearm
 Treated by open reduction and plating
 Treated by closed manipulation
 Mobilized by a below-elbow plaster
46. Fractures of the scaphoid:
 Is produced by a direct blow on the carpus
 Typically causes pain and tenderness in the anatomical snuff box
 Is not always evident on x-ray
 May be complicated by avascular necrosis and osteoarthritis
 Is immobilized in a below-elbow plaster with the thumb in full internal
rotation
and the plaster extended to the metacarpo-phalangeal joint
47. Kienbock’s disease:
 Is an insidious avascular necrosis of the scaphoid
 May be associated with trauma
 Causes paraesthesia of the hand
 Is usually seen in young adults
 Is usually seen in children

48. Bennett’s fracture:


 Is a fracture-dislocation of the first metacarpal through the
carpometacarpal joint
 Has the metacarpal displaced anteriorly and proximally on the carpal
bone
 Is stable because the collateral ligament is not ruptured
 Is easy to reduce by manipulation but the reduction is difficult to

11
maintain
 May necessitate arthrodesis
49. A rolled bandage held in the palm of the hand and held by a crepe
bandage is
used to immobilize:

 Fractures of the phalanges


 Fractures of the metacarpals
 Fracture of the lunate
 Dislocation of the carpo-metacarpal joint
 Dislocation of the metacarpo-phalangeal joint
50. An x-ray shows a fracture of the shaft of the ulna, you will:

 Reduce the fracture and apply a plaster of Paris cast


 Perform an open reduction and internal fixation
 Request an x-ray of the elbow to show the proximal radio-ulnar joint
 Request an x-ray of the wrist
 Request an x-ray of the whole shaft including the elbow and wrist joints
51. Which of the following is true of fractures of the phalanges?
 The distal phalanx is fractured more commonly than the proximal and
middle phalanges
 The deformity in the shaft fractures of the proximal phalanx is flexion of
the proximal fragment from the pull of the interossei and the lumbricals
 Fractures of the middle phalanx are usually not displaced
 Undisplaced fractures of the proximal phalanx are immobilized in a
plaster of Paris from the wrist to the tip of the finger
 Displaced fractures of the middle or proximal phalanx are reduced by
traction and flexion of the finger over a roll of cotton gauze and held
down by cotton gauze

12
52. In mallet finger:
 The terminal phalanx is held in flexion deformity
 The terminal phalanx cannot be flexed
 The flexor tendon avulses a portion of the terminal phalanx at its
insertion
 The finger is immobilized in a splint with the proximal interphalangeal
joint
flexed and the distal interphalangeal joint hyperextended for 3 weeks
 If a large portion is avulsed then operative excision is necessary
53. A patient is suspected of having a fracture of the scaphoid. X-ray is
negative.
You will:

 Leave the patient alone


 Apply strapping of the wrist for 3 weeks
 Apply a scaphoid plaster for 2 weeks and repeat the x-ray
 Apply a splint to the wrist and repeat the x-ray after 2 weeks
 Apply a scaphoid plaster for 8 weeks
54. Falling on the outstretched hand may result in:
 Fracture of the scaphoid
 Dislocation of the lunate
 Fracture of the radial head
 Smith’s fracture
 Monteggia fracture
55. Falling on the outstretched hand may result in fracture of the:
 Clavicle
 Shaft of the humerus
 Surgical neck of the humerus
 Olecranon
 Shafts of the radius and ulna
A man of 25 standing with his face to a wall is squashed from behind
by a reversing car. He complains of severe pain around the pelvis
and is unable to stand. His pulse is 100bpm, regular and of moderate
volume and the blood pressure is 100/60mmHg.

56. The most likely clinical diagnosis is:


 A vertical fracture of the pubic ramus and ischium on one side
 Hinge-separation of the symphysis pubis
 A vertical fracture of the pubis and ischium and a vertical fracture of the
ilium all on both sides
 A vertical fracture of the pubic ramus and ischium on both sides
 A vertical fracture of the pubic ramus and ischium and hinge separation

13
of the symphysis

57. Which of the following complications may occur?


 Sciatic nerve palsy
 Rupture of the bladder
 Perforation of the stomach
 Paralytic ileus
 Rupture of the urethra
58. After confirming the diagnosis by x-ray you will:
 Apply a short hip spica
 Apply a full hip spica
 Leave the patient free in bed and encourage active moments of the
spine and hips
 Put the pelvis in a Bryant pelvic sling and suspend it from an overhead
beam
 Apply skeletal traction through the tibia
59. Relaxation of which of the following muscles causes shortening and
displacement in intra-capsular fractures of the hip?
 Gluteus maximus
 Gluteus medius
 Piriformis
 Superior gamelli
 Obturator internus

A 30-year old woman in the front seat of a car injures her right hip
following an accident involving the car. The right leg is shortened,
internally rotated, flexed and adducted.

60. The most likely clinical diagnosis is:


 Basal fracture of the femur
 Subcapital fracture of the femur
 Posterior dislocation of the hip
 Anterior dislocation of the hip
 Central dislocation of the hip
61. Which of the following complications may occur?
 Sciatic nerve palsy
 Osteoarthritis of the hip

14
 Rupture of the femoral artery
 Myositis ossificans
 Fracture of the anterior lip of the acetabulum

62. In anterior dislocation of the hip:


 The hip is slightly extended
 The hip is abducted
 The hip is externally rotated
 The limb appears lengthened because the head of the femur lies at a
slightly lower level than the acetabulum
E. The limb appears shortened and internally rotated
63. Fracture of the neck of femur:
 Is most common in elderly men especially over 70
 Is aetiologically most often associated with osteoporosis
 Is usually caused by severe violence in the young and elderly
 Results in pulling of the lesser trochanter upwards and forwards by the
psoas muscle
E. Usually produces no deformity
64. A 75-year old lady stumbles and falls down. She experiences pain in
the right
groin and is unable to raise the leg. The leg is found on examination
to be shortened and externally rotated. The clinical diagnosis may
be:

 Subcapital fracture of the femur


 Transcervical fracture of the femur
 Basal fracture of the femur
 Intertrochanteric fracture of the femur
 Anterior dislocation of the hip
65. Avascular necrosis and/ or non-union is a problem in:
 Basal fracture of the femur
 Sub trochanteric fracture of the femur
 Transcervical fracture of the femur

15
 Intertrochanteric fracture
 Anterior dislocation of the hip
66. Non-union of fracture of the neck of the femur in an elderly woman is
BEST
treated by:
 Full hip spica
 Hemi-arthroplasty
 Dynamic hip screw
 MacMurray’s osteotomy
 Insertion of a bone graft

67. In the treatment of fractures of the neck of the femur:


 Transcervical fracture in patients under 70 is treated with cancellous
hip screws
 Displaced Transcervical fractures in patients over 70 is best treated by
a hemi-arthroplasty or by a total hip replacement
 A basal fracture is best treated by internal fixation with a dynamic hip
screw
 Intertrochanteric fracture in the young is immobilized with a full hip
spica
 Subcapital fractures in the young is treated by total hip replacement
68. Avascular necrosis of the head of the femur:
 Is uncommon after the Subcapital fracture of the neck
 Follows badly treated fracture of the neck of femur
 Is influenced by weight bearing forces
 Causes severe groin pain when lying down
 Radiologically has a glossy ground-glass appearance in the early
stages
69. Avascular necrosis of the head of the femur may be treated by:
 Analgesics and rest
 Physiotherapy
 Replacement of the head by a prosthesis
 Total hip replacement

16
 Arthrodesis of the hip
A car knocks down a 30-year old man. He complains of severe pain in
the left thigh and is unable to raise the leg. Examination reveals
deformity and swelling of the thigh with flexion, abduction and
external rotation of the proximal part of the thigh.
70. The fracture of the femur is at the:
 Greater tuberosity
 Upper third
 Middle third
 Lower third
 Supracondyles
71. Which of the following may occur?
 Injury to the popliteal artery
 Injury to the femoral artery
 Injury to the popliteal nerves
 Injury to the sciatic nerve
 Shock

72. Indications for internal fixation of a fracture of the shaft of the femur
include:
 Old or frail patients
 Patients between 5 and 12
 Patients with multiple injuries to the lower limb
 Pathological fractures from metastasis
 Angulation of the fractured fragments
73. A Gallow’s splint is used in children for treating fracture of the:
 Tibia and fibula
 Supracondylar region of the femur
 Shaft of the femur
 Neck of femur
 Anterior dislocation of the hip
74. A 25-year old man is suspected of having a supracondylar fracture of
the right

17
femur. You will:

 Check the popliteal pulse


 Examine the lower leg for any lesion of the sciatic nerve
 Check the dorsalis pedis pulse
 Check the femoral pulses
 Check movement of the bones
75. Supracondylar fracture of the femur is treated by:
 Manipulation and application of above knee plaster
 Manipulation and application of a full hip spica
 Open reduction and fixation with a blade plate
 Open reduction and intramedullary nailing
 Weight traction on the leg by skin or skeletal traction
76. In the treatment of fracture of the patella:
 A comminuted fracture is best treated in a patient under 40 by
immobilization
of the knee in a plaster cylinder for 6 weeks and in a patient over 40yrs
by patellectomy
 A transverse fracture in a patient under 40 is best treated by excision
of the patella
 A transverse fracture in a patient over 40 is treated by operative
reduction
and fixation of the fragments with screws
 A crack fracture in a patient under 40 is best treated by aspiration of
haemarthrosis and immobilization of the knee in a plaster cylinder
for 6 weeks
 A crack fracture in a patient over 40 is best treated by bed rest and
early mobilization

77. Which of the following disabilities of the knee may occur after
excision of the
patella?

A. Slight reduction in extension


 Some reduction in flexion
 Slight disability in climbing stairs
 Slight disability in descending stairs
 Locking of the knee

18
78. Which of the following muscles maintain the knee in the extended
position
during walking?

 Popliteus
 Rectus femoris
 Vastus medialis
 Gracilis
 Vastus lateralis
79. Which of the following is/are true of the ligaments of the knee?
 The lateral collateral ligament is more commonly ruptured than the
medial collateral ligament
 In rupture of the medial collateral ligament the knee is unstable
on adduction
 Medial collateral ligament rupture is usually treated by applying a knee
brace for 6 weeks
 The anterior cruciate ligament is ruptured by an extension force driving
the tibia forwards relative to the femur
 Anterior cruciate ligament rupture is treated by surgical reconstruction
80. Which of the following is/are true of tears of the menisci?
 As the medial meniscus is attached to the medial collateral ligament, it
is less commonly torn than the lateral meniscus
 They occur if the semi-flexed knee is twisted while weight-bearing
 In the bucket handle tear, the commonest, the tear extends into both
horns
and the fragment is displaced inwards
 They cause haemarthrosis
 They may heal spontaneously
81. Clinical features of acute tears of the medial meniscus include:
 Inability of the patient to straighten the knee fully after injury to the knee
 Swelling of the knee
 Wasting of the quadriceps muscle
 Attempted passive extension of the knee causes pain in the whole
knee
 Appley’s grinding test is positive

82. Which of the following is/are true of fracture of the lateral condyle of

19
the tibia?
 It is less common than that of the medial condyle
 Oblique shearing fractures should always be treated conservatively
 A comminuted compression fracture is treated by Hamilton Russell skin
traction and knee exercises
 Depressed plateau fracture is treated by open reduction in the young
 It may lead to genu varum
83. Which of the following is/are true of fractures of the tibia and fibula?
 Fracture of the tibia is always accompanied by the fracture of the fibula
 Oblique fractures are unstable and tend to shorten and displace in
plaster
even when the initial reduction is good
 Shortening may be overcome by skeletal traction using a Steinman’s
pin
through the calcaneum
 The full-length leg plaster extends from the mid-thigh to the metatarsal
heads
with the knee slightly flexed at about 10o and the ankle at 90o
 It is not essential to examine the leg for circulatory impairment when
the patient
is first seen as pressure on blood vessels and nerves is caused by
tight
dressing or plaster cast
84. Indications for open reduction and internal fixation in fractures of the
tibia and
fibula include:

 Fractures of the lower third of the tibia


 Overlap causing mal-union
 Mal-union from angulation
 Oblique fracture
 Delayed union
85. Which of the following injuries of the ankle is/are stable?
 Complete rupture of the lateral ligament
 Spiral fracture of the lateral malleolus
 Spiral fracture of the lateral malleolus with avulsion fracture of the
medial
malleolus (bimalleolar fracture)
 Avulsion fracture of the medial malleolus
 Spiral fracture of the lateral malleolus with avulsion fracture of the
posterior
part of the lower end of the tibia (tri-malleolar fracture)
86. Abduction-lateral rotation injuries of the ankle include:
 Avulsion fracture of the tip of the lateral malleolus

20
 Anterior marginal fracture of the lower end of the tibia with anterior shift
of the talus
 Avulsion fracture of the medial malleolus with spiral fracture of lateral
malleolus
 Isolated shearing fracture of the medial malleolus
 Shearing fracture of the medial malleolus with transverse fracture of
the lateral malleolus

87. A middle-aged man while playing tennis experiences a sudden pain


behind the
right ankle. He is unable to plantar flex the foot or support the body
on the toes.

The most likely injury is:

 Fracture of the lower end of the tibia


 Tear of the posterior ligament of the ankle
 Rupture of the Achilles tendon
 Tear of the gastrocnemius muscle
 Tear of the tibialis posterior muscle
88. A patient falls from a height and is found to have a fracture of the
calcaneus.
Which of the following may also be fractured?

 Lower tibia
 Pelvis
 Lower thoracic spine
 Lumbar spine
 Basi-occiput
89. A 30-year old farmer is carrying a heavy load on her trips and the
load falls
backwards from the head. She may sustain:

 Fracture of a spinous process of a cervical vertebra


 Fracture of the lamina of a cervical vertebra
 Fracture dislocation of the cervical spine with injury to the cord
 Wedge compression fracture of the body of a cervical vertebra
 Avulsion fracture of the anterior-inferior border of a cervical vertebra
(tear drop)
90. In injuries of the spine:

21
 Cord injury without radiological evidence of vertebral injury is indicative
of extension-rotation injury
 Wedge compression fracture occurs in pure extension injury and is
stable
 Wedge compression fracture is commonest in the thoracic region
 Contusion on the forehead suggests an extension injury
 Fracture dislocation at T12-L1 is stable
91. In paraplegia following spinal injury:
 Normal bladder function usually returns after 3 months
 Diarrhoea often occurs
 Violent flexor and adductor spasm of the paralyzed limbs may occur
 Spirit applied daily to the skin prevents pressure sores
 There should be daily passive movements of all paralyzed joints and
the unaffected upper limb muscles

A 7year old boy is rushed into the emergency room with a swollen,
painful
and deformed left elbow. He had fallen off from his school desk
about 3 hours
ago.

92. Concerning the management of this kid


 A pulsating hematoma in the cubital fossa is a hard sign of a brachial
artery injury
 A supracondylar fracture is the most likely working diagnosis
 Compartment syndrome is not a possible complication of this injury
 An on-table angiogram is indicated if the pulses are found to be
diminished
with a sluggish capillary refill
 Definite treatment will involve closed manipulation and application of a
completed above elbow plaster of Paris cast.
93. Long term complications of this injury includes:
A. Cubitus varus
B. Volkmann’s ischemic contractures
C. Heterotopic ossification
D. Stiffness at the elbow joint
E. Cubitus valgus

22
A twenty-year-old male who is involved in a motorcycle accident as a
pillion rider is brought to the casualty with suspected multiple
injuries. Clinically he has a Glasgow coma scale of 12, a fractured
femur and also has blood at the tip of his penis.

94. A series of mandatory x-rays called the ‘trauma series’ should be


done to assess
his injuries. These include

A. Skull x-rays
B. Chest x-rays
C. Cervical spine x-rays
D. Lumbar spine x-rays
E. Pelvic x-rays
95. Pelvic x-rays done show grossly displaced fractures of the pelvis.
 You will pass a 24g Foley’s catheter if he goes into acute urinary
obstruction
 An intravenous urogram is indicated to make a diagnosis of urethral
rapture
 A paralytic ileus is a complication that arises from a retroperitoneal
hematoma
which is best treated by surgical evacuation
 A pelvic external fixator when applied helps to stabilize the pelvis and
reduce hemorrhage
 The mortality rate from pelvic fractures is about 20%

96. In the management of open fractures of the tibia


 Primary suturing of the soft tissue is recommended to prevent further
contamination of the fracture

 External fixation of the fracture is not recommended if the wound is


less than a centimeter in diameter

 Fracture healing will be delayed if there is extensive stripping of soft


tissue like muscle and periosteum

 Wound swabs at the time of debridement is taken to determine any


bacterial contamination

23
 Penicillin should be used as prophylaxis against osteomyetitis

97. The following are complications that can occur following the
treatment of a

Colles’ fracture except:

 Reflex Sympathetic Dystrophy

 Frozen shoulder

 Carpal tunnel syndrome

 Stiff wrist joint

 Mal-union
98. A 75year old lady presents to the casualty with an intracapsular
fracture of the

right hip. She is being prepared for a hemi-arthroplasty as definite


treatment:

 Her risk of deep vein thrombosis (DVT) is 70% if no prophylaxis is


given against it.
 Aspirin at a high dose of 300mg daily is recommended for prevention
of DVT
 The advantage of warfarin is that it has rapid onset of action and does
not require monitoring
 A prior history of DVT does not increase her risk of a new clot
 General anesthesia is superior to regional anesthesia for the on table
prevention of DVT during hip fracture surgery

99. A 19yr old university student presents to the casualty with an


anterior dislocation
of her right shoulder joint during a basketball game:

 Damage to the axillary nerve will present as decreased sensation at


the
point of the shoulder

 Open reduction of the shoulder is the definitive management

24
 The risk of recurrent dislocation of the shoulder joint is higher in
patients
below the age of 20yrs

 Bankart’s lesion is a tear in the anterior capsule of the shoulder joint

 The mechanism of injury is usually by a posteriorly directed force to


the
shoulder in abduction and external rotation

100. In the management of supracondylar fractures of the humerus in


children

 Limb shortening due to premature physeal closure is a common


complication

 An angiogram is indicated where there is suspicion of a compartment


syndrome

 Closed reduction of the fracture with percutaneous pinning with


kirschner wires is recommended for displaced fractures

 Injury to the ulnar nerve is a common complication

 Cubitus varus also referred to as a ‘gunstock’ deformity is a late


complication

25
CHAPTER 53
INJURIES TO BONE AND JOINTS
ANSWERS

 B, E 38. A, B, D, E 75. C
 A, C, D 39. B, C, D, E 76. D
 A, D 40. B, C, D 77. A, B,
C, D
 A, B, C 41. A 78. B, C,
E
 A, B, C, E 42. A, C, D, E 79. C, D,
E
 A, B, D 43. D 80. B, C
 B, C, D 44. B, C 81. A, B,
E
 B 45. B, C 82. D
 A, B, D 46. B, C, D 83. B, C
 D 47. B, D 84. A. B,

26
C
 B 48. A, D, E 85.B, D
 D 49. B 86. C
 B 50. E 87. C
 A, B, C, D 51. B, C, E 88. A, B,
C, D, E
 A, D 52. A, D 89. A, B,
C, E
 B, D 53. C 90. A, C,
D
 C 54. A, C 91. C
 D 55. A, C, E 92. A, B,
D
 D 56. B 93. A, B,
C, D, E
 A, B, C, D 57. B, D, E 94. B, C,
E
 B, C, D 58. C 95. D, E
 A, C, D 59. B 96. C
 E 60. C 97. C
 C 61. A, B, D 98. A
 A, D 62. B, C, D 99. A, C,
E
 C 63. B, D 100. C, E
 B. D, E 64. A, B, C, D
 B 65. C, E
 C 66. B
 C 67. A, B, C
 A, B 68. E
 A, B, C 69. B, C, D
 A, B, C 70. B
 C, D 71. E
 C 72. A, C, D
 E 73. C
 A, B, E 74. C

27
CHAPTER 54
ORTHOPAEDIC SURGERY

1. In the anatomy of bone:

 The metaphysis is the part of the diaphysis adjoining the epiphyseal


plate
 The actively growing segment of the bone is the epiphysis
 The periosteum ensheaths the bone and is made up of fibrous tissue
capable of laying down bone
 Bone is made up of bone cells in a matrix of mineralized osteoid
 The diaphysis is the site of origin of most diseases of the bone
2. In bone:
 There is no lymphatic drainage
 The nerve supply is rich in the periosteum and scanty in the rest of the
bone
 The circulus vasculosus of Hunter supplies the metaphysis
 The Harvesian system (concentric cylindrical tubes centered on blood
vessels)
is found in compact and cancellous bone
 The cancellous bone is the initial site of osteomyelitis or myeloma
3. The rate of formation and breakdown of bone is influenced by:
 Vitamin B
 Exercise
 Mineralo-corticoids
 Level of serum calcium
 Calcitonin
4. Bone:
 Contains about 90% of the calcium in the body
 Contains about 85% of the phosphate in the body
 Resorption is enhanced by calcitonin
 Contains magnesium, carbonate and chloride but no fluoride
 Formation stops in old age
5. Calcium:
 Mobilization from bone is enhanced by parathormone
 Excretion by the renal tubules is stimulated by dihydroxy-chole-
calciferol
 Excretion by the renal tubules is stimulated by parathormone
 Absorption from the intestine is stimulated by calciferol

1
 Absorption from the intestine is stimulated by calcitonin
6. Synovial joints include:
 Syndesmosis
 Ellipsoid joint
 Pivot joint
 Gomphosis
 Amphiarthrosis

7. Concerning joints:
 The co-efficient of friction is very low and equals that of ice
 The articular cartilage is avascular and depends on synovia and sub-
chondral
osseous blood supply for nutrition
 A detached articular cartilage does not grow
 Synovia makes the articular cartilage the slipperiest surface known
 Joint space seen on x-ray is the space between the articular cartilages
8. Which of the following is/are transmitted by dominant genes:
 Sickle cell disease
 Achondroplasia
 Haemophilia
 Congenital dislocation of the hip
 Talipes equino-varus
9. In the grading of muscle power according to the Medical Research
Council
classification grade:

 3 is movement with gravity eliminated


 1 is no flicker
 5 is normal power
 2 is movement against gravity
 4 is movement against resistance
10. In the examination of the hip joint:
 True shortening is found by measuring the distance between the
anterior
superior iliac spine and the medial malleolus with the pelvis level
 Apparent shortening is found by measuring the distance between the
symphysis

2
pubis and the medial malleolus with the pelvis level
 If apparent shortening is equal to true shortening then there is no
abduction deformity
 Flexion deformity is measured as the angle at the beginning of
movement
 It is always essential to measure Bryant’s triangle to estimate a raised
greater trochanter
11. In the examination of the hip and lower limb:
 A positive Trendelenburg’s test indicates a stable hip
 A raised greater trochanter is seen in a dislocated hip
 Nelaton’s line is undisturbed in coxa vara
 Abduction to 90o is possible in a normal adult hip
 The level of the pelvis is not of any significance

12. The following lines/signs assist in the assessment of the hip on x-


rays:
 Perkin’s lines
 Pauwell’s lines
 McMurrays’s sign
 Trethowan’s sign
 Shenton’s line
A 10-year old boy who claims he fell down 10 days previously
presents with severe
pain on the medial upper aspect of the right lower leg, general
malaise and vomiting
for 2 days. He is ill and the temperature is 40oC. A point on the
medial upper aspect
of the lower leg just below the knee is very tender. There is a slight
swelling over
the spot. Active movement of the lower leg is markedly limited but
passive
movement of the knee is present.

3
13. The most likely clinical diagnosis is:
 Fracture of the upper end of the tibia
 Cellulitis
 Septic arthritis of the knee
 Osteomyelitis of the upper end of the tibia
 Haematoma
14. The diagnosis can be confirmed by:
 X-ray of the lower leg and knee
 Haemoblobin, white blood cell and differential
 Erythrocyte sedimentation rate (ESR)
 Repeated blood cultures
 Aspiration
15. An x-ray of the leg and knee will show:
 New bone formation and elevated periosteum
 Loss of joint space
 Osteolysis
 Fracture
 No bony abnormality
16. Treatment will include:
 Full leg plaster of Paris and analgesics
 Bed rest, splintage of the leg in a Thomas’ splint and analgesics
 Cloxacillin and ampicillin intravenously for 5 days followed by oral
administration for up to 6 weeks
 Aspiration of the knee
 Intravenous fluids and/or blood transfusion

17. Acute haematogenous osteomyelitis:


 Is most commonly caused by staph pyogenes
 Is associated with trauma in a few patients and overt septic skin
lesion in the majority
 In children commonly starts in the diaphysis
 In Africa has a higher incidence among sicklers
 Always leads to the formation of a sequestrum
18. Complications of acute haematogenous osteomyelitis include:

4
 Osteosarcoma
 Brain abscess
 Pathological fracture
 Bone deformity
 Ankylosis of the adjoining joint
19. Acute septic arthritis:
 May be caused by N. gonorrhea
 Gives rise to joint swelling, restricted active movement but presence of
passive movement
 Is best diagnosed by aspiration of the joint
 Maybe associated with hyperuricaemia
 On x-ray shows Osteolysis of the bones in the joint
A 16-year old boy complains of intermittent pain at the medial lower
end of the
right lower leg for about 2 months. The affected area is slightly warm
and tender.
X-ray shows a cystic area surrounded by sclerotic bone at the lower
end of the tibia.

20. The most likely diagnosis is:


 Osteosarcoma
 Non-ossifying fibroma
 Simple bone cyst
 Brodie’s abscess
 Osteoid osteoma
21. The treatment is:
 A course of penicillin for 4-6 weeks
 Amputation followed by cytotoxic chemotherapy
 Opening of the swelling and curettage
 Excision of the swelling
 Opening of the swelling and filling it with bone chips

22. Chronic osteomyelitis:

5
 Is caused by E. coli
 Presents with recurrent sinuses preceded by painful tender swelling
 On x-ray characteristically shows periosteal new bone formation
 May lead to squamous cell carcinoma
 Is treated by laying the sinus open
23. Tuberculosis of bones and joints:
 Is most common in the hip
 Is most commonly caused by the bovine type of M. tuberculosis
 Is blood-borne usually from the intestinal tract.
 Starts in a focus at the metaphysis
 May cause meningitis or military tuberculosis
24. Clinical features of tuberculous arthritis include:
 Pain in the joint which improves at night when the muscles are relaxed
 Limping and restriction of active but not passive movements
 Swelling of the joint due to synovial effusion and degeneration
 Tenderness of the joint
 Pyrexia at night
25. In the investigation of suspected tuberculous arthritis in a 16-year
old boy,
the following MUST be done:

 IVP
 Culture of early morning gastric washout
 Biopsy (needle or open) of synovial membrane
 Lumbar puncture
 Cystoscopy
26. The general principles in the treatment of joint tuberculosis include:
 Immobilization of the joint in the position of function until the symptoms
subside and ESR is normal
 Administration of streptomycin, PAS and INAH for at least 18months
 Provision of protein-rich diet and vitamin D
 Arthrodesis of the affected joint when the disease becomes quiescent
 Examination of all contacts
27. In tuberculosis of the spine:
 The commonest sites are the lower cervical, upper thoracic and lower
lumbar vertebrae
 An abscess from the lumbar spine may present in the iliac fossa, groin,
triangle of Petit or gluteal region
 The first radiological sign is destruction of the body of the vertebra
 There may be concomitant lung involvement in about 50% of patients
 The Mantoux and Heaf’s tests are weakly positive in most patients

6
28. Tuberculous (Pott’s) paraplegia:
 Is not likely to arise during treatment of tuberculosis of the spine
 Is always spastic in type
 May spare the sphincters and sensation
 Requires operation only if bed rest and chemotherapy do not produce
some recovery within 3 months
 Has a poor prognosis; only about 40% of patients are able to walk
A woman of 25 complains of a painless swelling on the flexor aspect
of the wrist
of 2 months duration. The swelling is found on examination to
extend into the
hand and to be soft, non-tender and with a crepitus on pressure. The
fingers are
held in flexion.

29. The most likely clinical diagnosis is:


 Rheumatoid arthritis of the wrist
 Tuberculous arthritis of the wrist
 Tuberculous tenosynovitis
 Sprain of the wrist
 Gout
30. The most useful investigation is:
A. X-ray of the wrist
 Mantoux test
 White cell counts and differential
 Biopsy
 Serum uric acid
31. In acute infections of the hand:
 The commonest organisms are Esch. coli and Strep. haemolyticus
 The antibiotics of first choice are penicillin, ampicillin or ox tetracycline
 Suppuration is inevitable
 Pus should be drained as soon as it is formed, the incision being
placed over
the point of maximum tenderness
 Acute osteomyelitis may be a complication
32. Chronic paronychia:
 Is an insidious low grade infection of the nail fold

7
 Is caused by Staph. aureus
 Is treated with tetracyclines or penicillin for 3-4 weeks
 If unresponsive to conservative treatment will require the removal of
the nail
 May cause osteomyelitis of the phalanx

A carpenter complains of throbbing pain in the palm of the right


hand. A tender
swelling in the palm with associated swelling of the dorsum is found.
The fingers
are held in flexion but extension of the metacarpo-phalangeal joints
and fingers is
not painful.

33. The most likely diagnosis is:


 Cellulitis of the hand
 Septic blister of the palm
 Palmar space infection
 Infection of the ulnar bursa
 Infection of the radial bursa
34. In osteogenesis imperfecta:
 Collagen formation is normal but osteoid formation is defective
 The sclera of the eye is blue
 Otosclerosis occurs early in adulthood
 The serum calcium and alkaline phosphatase are normal
 Fluoride is of proven value
35. Achondroplasia:
 Is a deformation
 Is caused by defective growth of the diaphysis
 Is associated with subnormal intelligence
 Gives rise to three divergent mid-fingers of about the same length
 Gives rise to genu varus
36. Features of dyschondroplasia ( Ollier’s disease)
 Ecchondromata

8
 Enchondromata
 Limb shortening
 Malignant degeneration
 Sexual precocity
37. Diaphyseal aclasis:
 Is familial with a male preponderance
 Affects the growing ends of long and short bones
 Results from defective moulding of the metaphysis leaving islands of
bone on the diaphysis
 Continues to grow even after cessation of growth of the skeleton
 May rarely become malignant

38.In cleido-cranial dysostosis:


 There is premature fusion of the fontanelles and membrane bones
 The vault of the skull is high and pointed
 The clavicles are either missing or defective in the center or at either
end and
the shoulders can be made to touch in front
 There may be an extra long index finger and syndactyly
 Operative correction of the defects gives satisfactory results
39. In fibrous dysplasia:
 The affected bones are replaced by fibrous tissue and bend easily
 The metaphysis is the part affected
 Radiolucent areas looking like cysts abound
 If there is associated skin pigmentation with sexual precocity in the
male, it
is known as Albright’s syndrome
 Calcium and calcitonin have been found useful
40. A defect of the upper limb:
 Is usually a malformation
 Is usually due to damage of the limb bud which appears in the 4th
intra-uterine week

9
 If a congenital amputation, then development of the limb stops at any
level
from the elbow to the wrist
 If the short bones are absent then it is phocomelia
 Is radio-ulnar synostosis if the radius and ulna are joined distally by a
bridge
of bone which prevents pronation and supination
41. Congenital dislocation of the hip:

 Is fairly common in Black Africa


 Presents as adducted and flexed hip
 Gives a click when it is reduced
 Can be cured if discovered at birth and retained in a Barlow splint for
3 months
 If resistant to conservative measures should be treated by open
reduction
at 4 years.
42. Talipes equino-varus:
 Is a defect of the foot in which the foot is plantar-flexed, inverted
and adducted
 Is primarily a subluxation of the mid-tarsal joint
 May be a malformation or a deformation
 In a child that is walking, only the outer side of the sole touches the
ground
 Is more common in females

43. In Klippel-Feil syndrome, there:


 Is a high hair line
 Is a very short neck
 Is unilateral Sprengel’s shoulder
 May be fusion of vertebrae
 May be kyphosis
44. Congenital infantile torticollis:
 Is a deformation from intra-uterine pressure
 Leads to fibrosis of some fibres of the sterno-mastoid muscle
 Causes tilting of the head to the affected side

10
 Causes rotation of the face to the affected side
 Is managed in the early cases by division of the sterno-mastoid muscle
45. In peripheral nerve lesion:
 Axonotmesis is anatomical division of a nerve
 Neurotmesis is a lesion in which the Schwann sheath is intact but the
medulla is damaged and undergoes Wallerian degeneration.
 Neuropraxia is a lesion in which there is cessation of transmission of
nerve
impulses but no detectable anatomical damage
 The rate of nerve growth in neurotmesis is 1mm a day
 Recovery is possible in Axonotmesis only if there is surgical
approximation
of the divided ends
46. In peripheral nerve injuries:
 Extended elbow with pronated forearm and hand is suggestive of injury
of
the lower trunk of the brachial plexus
 Clawing of all fingers of the hand is suggestive of median nerve injury
 Foot drop is suggestive of medial popliteal nerve injury
 Loss of sensation of the sole of the foot is suggestive of posterior tibial
nerve injury
 Analgesia of the dorsum of the thumb, index and middle fingers just
beyond
the distal interphalangeal joint is suggestive of median nerve injury
47. Erb’s palsy:
 Results most commonly from difficult breech delivery
 Affects the biceps and brachialis
 Causes supination of the forearm and extension of the elbow
 Untreated, results in a stiff internally rotated shoulder
 Has a poor prognosis

48. Klumpke’s palsy:


 Results from breech delivery
 Affects C8 and T1

11
 Causes paralysis only of the hand
 Recovers rapidly
 Is treated by passive movements
49. Anterior poliomyelitis:
 Is an acute inflammation of the cells of the anterior, posterior and
lateral
horns of the spinal cord and motor cells of the brain stem
 Is caused by an entero-virus of which there are anti-genetically four
 Is endemic with occasional epidemics
 Is now rare
 Affects mostly people over 5 years
50. In anterior poliomyelitis:
 The virus enters the oro-pharynx only by inhalation of droplets
 Paralytic poliomyelitis probably accounts for about 20% of cases in
an epidemic
 Paralysis is related to injury or vigorous activity during the incubation
period but not to recovery from a debilitating illness
 Males are more commonly affected
 The incubation is usually 1-2 weeks but may last 5 weeks
51. Symptoms of anterior poliomyelitis include:
 Diarrhoea
 Photophobia and stiffness of the neck
 Paraesthesia, sore throat and headache
 Muscle pain and tenderness
 Fever, cough and dyspnoea
52. Paralysis in anterior poliomyelitis
 Comes on gradually
 Reaches a peak in 2-3 weeks and most recovery is within 6 months
of attack
 Is flaccid in type
 Involves in order of increasing intensity, the lower limbs, trunk and
upper limb
 Causes incontinence

12
53. Measures adopted in the treatment of residual paralysis in anterior
poliomyelitis include:

 No treatment
 Tenotomy
 Tendon transplantation
 Arthrodesis
 Osteotomy
54. In anterior poliomyelitis:
 The mortality is about 10%
 About 50% have residual paralysis
 Elimination from a community can be achieved by the immunization of
most
of the infants
 The Salk vaccine is more effective than the Sabin vaccine in prevention
 The Sabin vaccine is given orally
55. Cerebral palsy may be caused by:
 Rhesus incompatibility
 Sickle cell disease
 Antepartum haemorrhage
 Asphyxia neonatorum
 Encephalitis.
56. In spastic paralysis or paresis:
 The lesion is in the basal ganglia
 The infant has no difficulty in sucking but may have difficulty in holding
up the head later
 The hands go involuntarily into flexion at the wrist with adduction of the
thumb
 There is ataxia of gait
 There may be choreiform movements and hyperkinesia
57. Osteochondritis juvenilis:
 Is associated with disturbance of the vascular supply to a segment of
bone and its adjoining cartilage
 Affects the metaphysis primarily
 Is caused by hormonal imbalance
 Is best treated by repeated injection of hydrocortisone with xylocaine
into
the affected joint
 Usually takes 12-18 months to settle

13
58. Examples of traction epiphyseal or apophyseal type of
Osteochondritis
juvenilis include:

 Kienboch’s disease of the lunate


 Freiberg’s disease of the head of the 2nd or 3rd metatarsal
 Osgood-Schlatter’s disease of the tibial tuberosity
 Osteochondritis dissecans of the medial femoral condyle
 Perthes’ disease
59. A 7-year old boy presents with a painful right hip and limp of 4 weeks
duration. There
is no associated fever, weight loss, or anorexia. Examination reveals
limitation of most hip movements. The most likely clinical diagnosis
is:

 Synovitis
 Tuberculous arthritis of the hip
 Septic arthritis of the hip
 Osteochondritis of the femoral capital epiphysis
 Irritable hip
60. In Perthes’ disease:
 The male/female ratio is ¼
 Late cases have limitation of abduction and internal rotation
 X-ray may show a small or fragmented opaque capital epiphysis
 Most patients under 6 years do well with or without treatment
 Treatment averts deformity in most patients over 9 years
61. Blount’s disease (osteochondrosis deformans tibiae)
 Is seen all over the world
 Results from disturbance of growth at the lateral aspect of the upper
tibial metaphysis
 Results in genu valgum
 If bilateral, the child may be so deformed as to walk with the pelvis a
few
inches from the ground
 Is treated by tibial osteotomy
62. Idiopathic slipped upper femoral epiphysis:

14
 Is a gradual downward and anterior displacement of the femoral capital
epiphysis in adolescents.
 Causes limitation of external rotation and adduction
 May be associated with hypogonadism
 Is bilateral in about 40% of patients
 Is best treated by reduction

63. Osteoarthritis:
 Is an atrophic degenerative joint disease
 Causes limitation of external rotation and adduction
 Maybe be associated with hypogonadism
 Is bilateral in about 40% of patients
 Is best treated by reduction
64. Osteoarthritis may be caused by:
 Genu valgum
 Septic arthritis
 Intra-articular cortisone therapy
 Delayed union of a fracture
 AS disease
65. In osteoarthritis:
 There is fragmentation of the articular cartilage
 The synovial membrane is thin
 Young blood vessels invade the peripheral articular cartilage
 There is friction on the subchondral bone
 The adjoining bone undergoes osteolysis
66. The characteristic clinical symptoms of osteoarthritis is/are:
 Increasing stiffness
 Pain
 Crepitus
 Deformity
 Swelling
67. The pain in osteoarthritis:

15
 Is initially intermittent and later becomes constant
 Occurs on movement
 Is not relieved by rest
 Arises from spasm of local muscle and not from the joint
 Is not referred to another joint
68. In osteoarthritis of the hip, the joint is:
 Abducted
 Internally rotated
 Flexed
 Ankylosed
 Externally rotated

69. X-ray of a joint in osteoarthritis shows:


 Broadening of joint space
 Subchondral osteoporosis
 Osteophytes
 Lipping of marginal bone
 Articular cyst formation
70. Oral drugs that may be used in osteoarthritis include:
 Acetylsalicylic acid
 Cortisone
 Ketobrufen
 Piroxicam
 Morphine
71. The following may be adopted in the treatment of osteoarthritis:
 Hot fermentation
 Arthroplasty
 Amputation
 Arthrodesis
 Osteotomy
72. Growth hormone:

16
 Is produced by the basophil cells of the anterior pituitary
 Stimulates protein synthesis and growth
 Enhances glucose uptake by the cells
 If produced excessively before epiphyseal fusion leads to acromegaly
 If deficient in childhood leads to a dwarf with small trunk, small limbs,
big
head and normal intelligence
73. Features of acromegaly include:
 Increased height
 Prominent mandible
 Thickening of face
 Prominent supra-orbital ridge
 Long limbs
74. Bone is affected by:
 Thyroxine
 Testosterone
 Oestrogen
 Cortisol
 Glucagon

75. In osteoporosis:
 There is a reduction in bone mass per unit volume
 Osteoid formation is normal
 Calcification of osteoid is deficient
 Post-menopausal women are more commonly affected
 The long bones are more affected than the vertebrae and pelvis
76. Aetiological factors in osteoporosis probably include:
 Increased muscular activity
 Thyrotoxicosis
 Diminished ACTH production
 Inadequate intake of protein and Vit. C
 Diminished progesterone output
77. In osteoporosis:

17
 Most patients have severe bone pain
 The patient may present with fracture of the spine, neck of femur,
neck of humerus or lower end of the radius
 There is progressive loss of height
 The serum alkaline phosphatase is elevated
 The serum calcium is depressed
78. SPECIFIC treatment for osteoporosis includes:
 Calcium 3g/day
 Large doses of Vit. C
 Thyroxine
 Bisphosphonates
 Oestrogens
79. Paget’s disease (osteitis deformans):
 Is generalized affection of the skeleton in middle aged and elderly men
but not women
 Is characterized by hardening of the bones
 Is caused by deficiency of testosterone
 May cause pathological fractures
 Becomes sarcomatous in about 25% of patients
80. Paget’s disease:
 Is asymptomatic in most patients
 Leads to kyphosis and stooping
 Leads to genu valgum
 May result in cardiac failure because of frequently associated
valvular affections
 May shorten the height

81. In Paget’s disease:


 The serum alkaline phosphatase is markedly raised
 The serum phosphate is usually low
 The serum calcium is usually normal
 The affected bones on x-ray are dense, often deformed, narrow and
contains cyst-like areas
 Calcitonin 50-100 i.u. intravenously daily for several months may
arrest the disease
82. Clinical features in scurvy include:

18
 Subperiosteal haematoma
 Fever
 Bleeding gums
 Sabre tibia
 Separation of the epiphyses
83. Rickets:
 Is seen in children and adolescents
 Is in all patients caused by inadequate Vit. D intake
 Results in failure of calcification of osteoid to osseous bone
 Pathologically gives rise to a wide, thick cartilage zone of the
epiphyseal growth plate
 Is now rare in the world because of routine administration of Vit. D to
children
84. Clinical features in rickets include:
 Diarrhoea and vomiting
 Tetany
 Knock knees but not bow legs
 Thickening and widening of the wrists
 Bossing of the frontal, parietal, costal and phalangeal bones
85. In rickets:
 The serum alkaline phosphatase is normal
 The serum phosphorous is usually low
 The blood urea may be elevated
 The urine phosphate may be elevated
 The faecal fat may be more than 6g daily
86. In the treatment of rickets:
 Vit.D 1,000 i.u. given daily gives dramatic results
 The serum calcium, phosphate and alkaline phosphatase are
checked monthly
 Hypocalcaemia or hypercalcaemia may occur
 Oral calcium 1g should be given daily
 Bony deformities may recover spontaneously

87. Osteomalacia:
 Is uncommon and is generally seen in elderly women and rarely in
pregnant or lactating women

19
 May deform the pelvis
 May complicate Billroth I partial gastrectomy
 Is confirmed by biopsy of the iliac crest
 Is treated with Vit. D 2,000 i.u. and calcium 1g daily
A 1-year old male child has swollen limbs, which he cannot move. He
is restless.
The gums bleed easily.

88. The most likely clinical diagnosis is:


 Sickle cell disease
 Acute poliomyelitis
 Scurvy
 Acute osteomyelitis
 Anaemia
89. Investigations that will be useful in establishing the diagnosis
include:
 Full blood count and sickling
 X-ray of the affected limbs
 Serum ascorbic acid
 Urinary excretion of ascorbic acid after a loading dose
 Blood culture
90. Lumbar spinal epidural abscess:
 Is fairly common
 Is a metastatic infection caused by staph. aureus
 Causes localized pain often with radicular radiation
 Causes rapidly developing spastic paraplegia and sphincteric
incompetence
 Is evacuated through laminectomy under antibiotic cover
91.The correct posture when sitting is:
 Feet flat on the ground
 Chest lifted
 Head low
 Shoulder relaxed
 Arm fully pronated
92. Bone changes in sickle cell disease include:
 Enlarged costo-chondral junction
 Expansion of the diploe
 Platyspondyly
 Coxa vara
 Dactylitis

20
93. Osteomyelitis in sickle cell disease is characterized by:
 Asymmetrical involvement of the bones
 Diaphyseal involvement
 Refraction to medical treatment in the early stages
 Multifocal distribution
 Severe septicaemia
94. In prolapse of lumbar intervertebral disc:
 The annulus fibrosus is degenerated
 The anterior longitudinal ligament is frayed
 The nucleus pulposus herniates into the neural canal following an
extension injury
 Sneezing may be a precipitation trauma
 The commonest site is L3-4
A 40-year old man is suspected of having a prolapsed lumbar
intervertebral disc
has pain in the right lumbar area which radiates over the right
buttock behind the
thigh and calf into the foot.

95. The most likely disc is:


 L1-2
 L2-3
 L3-4
 L4-5
 L5-S1
96. In prolapsed lumbar intervertebral disc:
 Paraesthesia, hyper and hyposensitivity in a dermatome area may
occur
 Impairment of tendon reflex is unusual
 Sphincteric disturbance is common
 Muscle atrophy may occur
 Lasegue’s sign is negative
97. In spondylosis:
 The nucleus pulposus shrinks as a result of progressive decrease in
its water content

21
 There is degeneration of the cartilaginous plates over the vertebral
bodies
 The annulus fibrosis remains intact
 Pain is centered in the spine and associated with stiffness and
limitation
of spinal movements
 X-ray shows narrowing of the intervertebral joints and new bone
formation
at the periphery

98. Ankylosing spondylitis:


 Is uncommon in West Africans and in Caucasians is more common in
young adult females
 Is characterized by calcification and subsequent ossifications of the
ligaments
and capsule of the joints
 Starts in the thoracic intervertebral joints and spreads downwards to
the
lumbosacral and sacro-iliac joints
 Gives rise to pain and stiffness worse in the morning and after a period
of rest
 Leads to extension deformity of the spine and the ESR is markedly
raised
99. Severe low back pain in a 40-year old woman, which is not relieved
by rest in
bed, may be due to:

 Faulty posture on standing and sitting


 Complete rupture of an interspinous ligament
 Cervicitis
 Anxiety neurosis
 Vertebral metastatic carcinoma from the breast
100. Investigations that must be done in every patient with low back pain
are:
A. Plain x-ray of thoracic, lumbar and sacral spine
 Tomogram
 Myelogram

22
 ESR
 Serum alkaline phosphatase
101. Measures taken in the management of muscular or ligamentous
strain of
the spine include:
A. Rest in bed
 Short wave diathermy and massage
 Wearing of surgical corset
 Operative exploration
 Muscle strengthening exercises
102. Cancellous osteoma:
A. Is a true tumour
 Is found commonly at the end of long bones
 Continues to grow throughout life
 May undergo malignant change
 Is usually symptomless

103. Compact (ivory) osteoma:

A. Arises in any bone


 Is sessile and slow-growing
 Is painful
 Is covered by a cap of cartilage
 May cause epilepsy if it involves the inner table of the skull
104. Osteochondroma:
A. Is mushroom in shape, pedunculated or sessile
 Has a growing cap of cartilage
 Is most probably malignant if it continues to grow after skeletal
maturity
 Changes to osteo-chondrosarcoma in about 25% of patients
 Should be left alone if it is near the trunk, e.g. scapula or pelvis, as it
is not
likely to become malignant
105. Osteoclastoma:

23
 Arises at the junction of the epiphysis and the metaphysis and
spreads
to the epiphysis
 Normally penetrates into the joint
 Is a spindle-cell tumour with giant cells formed by coalescence of
some of
the spindle cells
 Is benign in 50% and frankly malignant in 15% of cases, and the rest
though
benign recur after removal
 Affects the 10-20 age group
106. The common sites of osteoclastoma include the:
A. Scapula
 Lower end of the femur
 Lower end of the tibia
 Pelvis
 Lower end of the radius
107. Treatment of osteoclastoma may be by:
A. Amputation
 Excision
 Curettage and bone graft
 Radiotherapy
 Cytotoxic therapy

108. Enchondroma:
 Is a true tumour arising usually in the metaphysis or shaft of the short
long
bones of the hand or feet
 Is usually seen in children
 May produce a pathological fracture
 Shows on x-ray as a sclerotic zone within the bone with cortical

24
ballooning
 Is treated by excision
109. Benign chondroblastoma:
A. Is seen in adolescents under 20yrs
 Is more common in females
 Histologically consists of young cartilage cells with areas of giant cell
transformation, myxoid and collagenous cells
 Starts in the epiphysis
 May rarely recur after excision or curettage and show malignant
characteristics
110. Bone ‘swellings’ originating from osteoblast include:
A. Fibrous osteoma
 Ossifying fibroma
 Osteochondroma
 Aneurysmal bone cyst
 Osteoclastoma
111. A simple bone cyst:
A. Is common in children but rare in adults
B. May involve any bone
 Arises in the epiphysis
 Is usually symptomless
 Is treated by excision
112. A solitary radiolucent lesion in the metaphysis of the femur in a boy
15
may be:
A. Non-ossifying fibroma
 Chondroma
 Osteoclastoma
 Solitary bone cyst
 Benign chondroblastoma
113. Osteosarcoma:
A. Is the commonest bone tumour
 Is usually metaphyseal in origin
 Contains elements of bone and cartilage but not of fibrous tissue
 Arises from primitive spindle cells
 Does not arise from extra-osseous tissue

25
114. Osteosarcoma may arise from:
A. Cancellous osteoma
 Enchondroma
 Osteochondroma
 Osteitis deformans
 Ossifying fibroma
115. Osteosarcoma spreads mainly to the:
A. Lymph nodes
 Liver
 Lungs
 Brain
 Other bones
116. Osteosarcoma:
A. Has a peak incidence in the first decade
 Occurs around the knee in 50% of patients
 Has a male/female ratio of 1.5/1
 Does not present with pathological fracture
 Presents with a painful, tender, warm swelling
117. Radiological features of osteosarcoma include:
A. Diaphyseal bone destruction
 Soft tissue mass or bone formation
 Radiate bone spicules of the cortex in the affected area
 Dense sclerosis of the rest of the bone
 Triangular condensation of bone at the site of periosteal elevation
118. The most essential investigation after establishing a diagnosis of
osteosarcoma is:

A. Skeletal survey
 X-ray of the lungs
 Blood profile
 E.S.R
 Serum alkaline phosphate
119. The present acceptable forms of treatment of osteosarcoma
include:
A. Amputation followed by cyclical combination cytotoxic therapy
 Radiotherapy followed by amputation
 Cytotoxic therapy followed by local resection
 Amputation
 Cyclical combination cytotoxic chemotherapy

26
120. Cytotoxic drugs currently used in the treatment of osteosarcoma
include:
A. Methrotrexate
 Actinomycin D
 Vincristine
 Cyclophosphamide
 Doxorubicin
121. An x-ray shows a disappearing bone. The conditions to consider
include:
A. Osteoclastoma
 Aneurysmal bone cyst
 Angiosarcoma
 Neurofibromatosis
 Histoplasmosis duboisii
A 50-year old man complains of recurrent pain in the ribs. X-rays of
the ribs, skull
and pelvis shows numerous areas of punched out rarefactions
without expansion
of the cortex. The lungs are clear.

122. You will consider:


A. Metastases from carcinoma of the prostate
 Metastases from the thyroid
 Multiple myeloma
 Bronchogenic carcinoma
 Ewing’s tumour
123. Thorough clinical examination is unrewarding. The investigation
that will be
helpful include:

A. Full blood count


 Biopsy
 Serum acid phosphatase
 Bence-Jones proteinuria
 Thyroid scan
124. An x-ray of the shaft of the femur in a 10-year old boy showing an

27
“onion peel”
appearance is indicative of:

A. Multiple myeloma
 Osteoclastoma
 Fibrosarcoma
 Syphilitic periostitis
 Ewing’s tumour

125. Multiple myeloma:


A. Originates from plasma cell
 Is twice as common in women
 May present as paraplegia, severe anaemia or pathological fracture
 Is confirmed by x-ray of the bones
 Is radiosensitive
126. Ewing’s tumour:
A. Occurs mostly in the age group 20-30 years
 Is twice as common in women
 May present as paraplegia, severe anaemia or pathologic fracture
 Is confirmed by x-ray of the bones
 Is radiosensitive
A 30-year old woman has a painful tender swelling at the upper end
of the right
fibula of 3 months duration. It feels like an eggshell. It is radiolucent
with
thinned cortices on x-ray.

127. The most likely diagnosis is:


A. Solitary bone cyst
 Ewing’s tumour
 Enchondroma
 Haemangioma
 Osteoclastoma

28
128. The treatment of choice is:
A. Radiotherapy
 Curettage and bone grafting
 Above-knee amputation
 Excision of the upper end of the fibula
 Leave alone
129. In DeQuervain’s disease the following is/are involved:
A. Abductor pollicis brevis
 Extensor pollicis brevis
 Radial tubercle
 Flexor carpi radialis
 Abductor pollicis longus

130. In trigger finger:


A. The sheath of the long flexor is affected
 There is nodular thickening of the sheath
 The nodular thickening is opposite the middle part of the shaft of
the metacarpus
 There is a sudden snap as the affected finger is flexed
 The treatment of choice is to slit the sheath
131. In carpal tunnel syndrome:
A. The male/female ratio is 1/10
 The pain occurs mainly at night and is also aggravated by washing,
sewing or writing
 The cause is compression of the ulnar nerve beneath the flexor
retinaculum
 The thenar muscles may be wasted
 Injection of hydrocortisone in xylocaine into the space has not been
found beneficial
132. Tennis elbow:
A. Is pain on the outer side of the elbow
 The pain occurs mainly at night and is also aggravated by washing,

29
sewing
or writing
 The cause is compression of the ulnar nerve beneath the flexor
retinaculum
 The thenar muscles may be wasted
 Injection of hydrocortisone in xylocaine into the space has not been
found beneficial
133. Mallet finger:
A. Is partial flexion deformity of the distal interphalangeal joint
 Is caused by rupture of the middle slip of the extensor tendon
expansion
of the finger
 Is painful
 If seen within a week should be treated by immobilization of the
finger with
the tendon relaxed
 If old should be ignored

A 55-year old man experiences a sudden pain in the right shoulder.


He is
unable to abduct the shoulder even after local injection of
xylocaine into the
painful spot in the shoulder. However, if the shoulder is passively
abducted
to about 90o, he is able to continue active abduction.

134. The clinical diagnosis is:


A. Pericapsulitis of the shoulder
 Osteoarthritis of the shoulder
 Complete tear of the supraspinatus tendon
 Fracture of the lateral end of the clavicle

30
 Fracture of the greater tuberosity of the humerus
135. Acute tendinitis of the shoulder:
A. Occurs at all ages
B. Is associated with a calcareous deposit above the greater tuberosity
of the humerus
 Causes sudden intense pain in the shoulder
 May be associated with fever
 Is treated by local injection of hydrocortisone in xylocaine or oral
phenylbutazone and a course of antibiotics
136. Pericapsulitis or frozen shoulder syndrome:

 Is a condition in which there is spontaneous onset of shoulder pain


accompanied
by increasing severe limitation of shoulder movements
 Occurs in the 40-60 age group
 Is associated with a friable joint capsule which shows fibrosis,
increase in
heparin and chondroitin and decrease in hyaluronic acid
 Runs a short course of about a fortnight
 Is treated by local injection of hydrocortisone in xylocaine
137. Ruptured tendo-Achilles:
A. Is caused by degeneration in the tendon
 Is usually seen in the elderly
 Gives rise to swollen heel
 Does not affect walking
 Is best sutured
138. Dupuytren’s contracture:
 Is fibrous replacement and contracture of the palmar aponeurosis
with
flexion deformity of the distal palm and fingers
 Is caused by trauma and is often familial
 Occurs most frequently in the aponeurosis proximal to the index and
middle fingers
 Is frequently bilateral
 Is equally common in the sexes

139. Ainhum:

31
A. Occurs in all races
 Is a progressive deepening constricting groove which encircles the
base of a toe
 Affects the second toe most commonly
 Is caused by infection
 May result in auto-amputation
A 50yr old woman presents to the orthopedic clinic with increasing
pain in the
right thigh of 3 months duration. The pain does not respond to
Paracetamol and
anti-inflammatory drugs. She had had a mastectomy 4 years earlier
for breast
cancer. Preliminary x-rays show an osteolytic lesion in the proximal
third of the
femur.

140. Appropriate investigations for the management of this patient


includes:
A. Computer Tomography (CT) scans of the chest
B. Magnetic Resonance Image (MRI) of the femur
C. Bone scintigraphy
D. Serum Calcium levels
E. Open Biopsy of the lesion
141. Treatment of this patient will involve:
A. Cyclical chemotherapy
 External beam radiotherapy to the femur
 Intravenous zolendronic acid
 Wide local excision of the tumour
 Prophylactic stabilization of the femur if there is a risk of a
pathological fracture
An eight-year-old sickle cell anemia patient presents to the
emergency room with
pain around the left shoulder of 3 days duration. Pain is severe,
constant and worse
with attempts at moving the shoulder. She looks dehydrated and
has a high-grade
fever of 39.5 degrees Celsius. She has no pain in any other parts of
her body.

142. The differential diagnosis of the patient includes:


A. Septic arthritis of the shoulder
B. Acute osteomyelitis of the proximal humerus

32
C. Sickle cell bone pain crises
D. Non accidental fracture of the proximal humerus
E. Malaria

143. Comment on the following diagnostic workup:


A. A normal x-ray of the shoulder excludes the diagnosis of osteomyelitis
 An ultrasound of the shoulder joint will differentiate fluid collection in
the
shoulder joint from septic arthritis of the joint
 A normal Erythrocyte Sedimentation Rate (ESR) completely rules out
an
infectious process
 The most likely organism to be cultured from the blood is
staphylococcus spp
 Blood cultures will be positive in about 60% of such cases
144. The further management of this patient after a diagnosis of septic
arthritis
of the shoulder will involve:

 Needle aspiration of the shoulder joint and a two week course of


intravenous antibiotics
 Arthrotomy and washout of the joint and a total of six weeks of
antibiotics
 Application of a shoulder Spica to rest the joint for six weeks
 Instillation of antibiotics into the joint has been found to be very useful
in
eradicating the infection
 The C-reactive protein (CRP) is used to monitor the long-term
response
to your treatment.
145. Complications of the above condition includes:
A. Shoulder stiffness
 Avascular necrosis of the humeral head
 Osteoarthritis of the shoulder joint
 Ankylosis of the shoulder joint

33
 Myositis ossificans of the shoulder
146. A 12yr old boy presents with rapidly growing painful lump in the
proximal tibia
of three months duration. X-rays suggest an osteosarcoma of the
tibia. Your management will include:

A. CT scan of the chest


 MRI of the tibia
 Serum calcium levels
 Radioisotope bone scan
 Biopsy of the lesion
147. Achondroplasia:
A. Is the most common form of disproportionate dwarfism
 Autosomal dominance inheritance
 Lumbar stenosis is the most likely cause of disability
 Defect in hypertrophic zone of physis
 Trident hand is a characteristic feature

148. Rickets:
A. It’s a defect in the mineralization of physis
 Low calcium is a characteristic feature
 Vitamin-D resistant rickets is X-linked dominant
 Physeal cupping is characteristic in x-rays
 Vit-D treatment resolves most deformities
149. Club foot deformities have the following components:
A. Forefoot pronation
 Forefoot supination
 Ankle equinus
 Hind-foot varus
 Mid-foot recurvatum
150. Hallux valgus:
A. Is common amongst elderly men
 Has a genetic predisposition
 Can occur in adolescent girls
 Is bilateral in 50% of cases
 Is always treated by surgery

34
CHAPTER 54
ORTHOPAEDIC SURGERY
ANSWERS

 A, D 42. A, B, C 83. C. D
124. E
 B, E 43. B, D 84. B, D
125. A, C
 B, E 44. B, C 85. B, C, D, E
126. B, C
 B 45. C 86. E
127. E
 A 46. D, E 87. A, B, D
128. D
 B, C 47. B, D, E 88. C
129. B, E
 B, D 48. A, B 89. B, D
130. B, E

35
 B 49. C 90. B, C
131. B, D
 C, E 50. E 91. A, B
132. A
 A, D 51. A, B, D 92. B, C, E
133. A
 B 52. C 93. B
134. C
 A, B, D, E 53. A, B, C, D, E 94. A, D
135. B, C
 D 54. A, E 95. E
136. A, B, E
 D 55. A, C, D, E 96. A
137. C, D, E
 E 56. C 97. A, B, D, E
138. A. D
 B, C, E 57. A, E 98. B, D
139. B, E
 A, D 58. C 99. C, D, E
140. A, C, D
 B, C, D 59. D 100. A, D, E
141. B, C, D
 A, C 60. B, C, D 101. B, C, E
142. A, B, D
 D 61. D, E 102. B, E
143. B, D
 C 62. C 103. B, E
144. B
 B, D 63. E 104. A, B, C
145. A, C, D
 E 64. A, B, C 105. A, C, D
146. A, B, D, E
 C, D, E 65. A, C, D 106. B
147. A, B, C, E
 C 66. A, B, C, D, E 107. A, B, C, D
148. A, C, D, E
 A, C, E 67. A, B 108. C
149. B, C, D
 B, D 68. C 109. A, C, D, E
150. B, C, D
 C 69. C, D 110. A, C, E
 C 70. A, C, D 111. A, B, D
 D 71. B, D, E 112. A, B, D, E
 D, E 72. B 113. B, D
 A, D 73. B, C, D 114. C, D
 C 74. A, B, C, D 115. C

36
 B, C 75. A, D 116. C, E
 D, E 76. B, D 117. B, C, E
 B, C 77. B, C 118. B
 A, C, E 78. D 119. A, C
 C 79. D 120. A, C, D, E
 A, C 80. A, B, E 121. B, C, D, E
 A 81. A, C, E 122. B, C
 C, D 82. A, C 123. A, B, D

37
CHAPTER 52
NEUROSURGERY
1. Congenital abnormalities of the C.N.S.:

        A. Have the same incidence as abnormalities in other systems


B. In most cases have a cause
C. May be associated with maternal infections such as rubella,
toxoplasmosis
syphilis near term
D. May be associated with hydramnios, toxaemia of pregnancy or
malpresentation
of the foetus
E. Are always overt
2. Cerebrospinal fluid:
 Is all formed by the choroid plexus of the 3rd ventricle
 Is absorbed by the arachnoid villi
 Formed daily is roughly 600 ml
 That can be contained in the ventricular system and subarachnoid
Spaces in normal individuals is 100-150 ml
 In the ventricles increases with resultant ventricular dilatation and
hydrocephalus
only because of obstruction in its pathway
3. Hydrocephalus:
A. Is always associated with increased intracranial tension
B. In infants is always congenital
C. In children may be caused by meningitis or cerebral tumours
D. In adults results from obliteration of the subarachnoid pathways at the
ten+torial
hiatus and base of the brain
E. Is not always progressive
4. Clinical features of hydrocephalus in infants include:

1
 Irritability and somnolence
 Poor suckling
 Frequent epileptic fits
 Reduced vision or blindness
 Absent tendon jerks

5. Clinical features of hydrocephalus in children over 5 years include:


A. Rapid increase in head circumference
B. Headaches usually in the morning, and vomiting
C. Tension and bulging at the anterior fontanelle
D. Ataxia
E. Papilloedema
6. Clinical features of hydrocephalus in adults include:
A. Disturbance of gait
B. Headache
C. Defective cerebration
D. Vomiting
E. Urinary incontinence
7. The differential diagnosis of hydrocephalus in an infant include:
A. Dementia
B. Subdural haematoma
C. Cerebellar tumour
D. Subgaleal fluid
E. Scalp oedema
8. Which of the following is the most useful investigation in
hydrocephalus
in an adult?
 Plain skull x-rays
 Angiography
 Pneumoencephalography
 E.E.G

2
 CT scan
9. The Spitz-Holter or Pudenz valve is used for:
A. Ventricular tap
B. Ventricular-peritoneal shunt
C. Ventriculo-atrial shunt
D. Ventriculo-umbilical shunt
E. Ventricula-peritoneal shunt

10. Craniosynostosis:
A. Is closure of one or more cranial sutures
B. Affects most commonly the sagittal suture
C. May lead to blindness
D. Does not cause mental retardation
E. Should be corrected before the baby is 4 months and preferably in
the
neonatal period
11. Spina bifida:
A. Is a congenital deformity of the spine characterized by incomplete
closure of the vertebral canal
B. Is a midline fusional defect which occurs in the first 4 months of
embryonal
life
C. Occurs most frequently in the thoracolumbar junction
D. Has an incidence of 10 per 1,000 births
E. May be associated with a similar deformity of the spinal cord
12. Associated factors in spina bifida include:
A. Heredity
B. Drugs
C. Conception before 17 or over 30 years
D. Geographical location – England, Nigeria, U.S.A
E. Hydramnios

3
13. In spina bifida occulta:
A. The spinal defect is not obvious
B. There are hair patches and portwine staining at the site
C. There may be limping when the child begins to walk and the tendon
jerks
are brisk
D. Nocturnal enuresis and delay in bladder control may occur
E. Hemivertebrae or fusion of 2 or more vertebrae may be present

14. Congenital dermal sinus:


A. Connects a skin opening with the intra-spinal subarachnoid space
B. Is lined by columnar epithelium
C. May be associated with an intraspinal ependymoma
D. Causes recurrent meningitis
E. Has a tuft of hair around the external opening at the centre of a
small
skin dimple
15. Meningocele:
A. Is herniation of the meninges through a spinal bifida
B. Has a wall consisting of skin and dura
C. Commonly leaks C.S.F.
D. Is pedunculated, cystic and most commonly sited in the lumbar
region
E. Is usually accompanied by neurological deficits
16. Meningocele:
A. Is the commonest variety of spina bifida
B. Is not associated with hydrocephalus
C. May rarely contain lipomatous tissue

4
D. Should be excised and the dura repaired
E. Has an excellent prognosis
17. Myelomeningocele:
A. Is an uncommon variety of spinal bifida
B. Is usually in the thoracic or cervical region
C. Is sessile and covered by atrophic skin
D. Is always associated with hydrocephalus
E. Should always be treated

18. Clinical features of myelomeningocele include:


A. Paraparesis
B. Dribbling of urine
C. Anal incontinence
D. Equinovarus
E. Genu valgum
19. A neonate has an occipital, midline sessile, round swelling about
10 cm in
diameter. It is soft and fluctuant superficially but feels solid in the
deeper
part.

The most likely clinical diagnosis is:


A. Meningocele
B. Myelomeningocle
C. Encephalocele
D. Subgaleal cyst
E. Dermoid cyst
20. A plain X-rays of the skull shows a defect 2 cm wide in the occipital

5
bone.
You will advise:
A. Deferred treatment for a few months
B. Excision of the swelling
C. No treatment at all
D. Aspiration
21. Intracerebral abscess:
A. Is generally solitary and unilocular but may be multiple or multilocular
B. Is usually primary but may be secondary to an adjacent or distant
primary
source
C. Is most commonly caused by pneumococcus or streptococcus
D. Is formed in the grey matter
E. Occurs most frequently in the occipital and parietal lobes

22. Intracerebral abscess may follow:


A. Paranasal sinusitis
B. Chronic otitis media
C. Bronchiectasis
D. Bacterial endocarditis
E. Furunculosis
23. Clinical features of a frontal intracerebral abscess include:
A. Headache and vomiting
B. Drowsiness, stupor or coma
C. Ipsilateral spastic hemiparesis
D. Epileptic fits
E. Contralateral homonymous hemianopia
24. The single most useful investigation in intracranial abscess in West

6
Africa
now is:
A. Plain skull x-rays
B. E.E.G.
C. Angiography
D. Ultrasonography
E. CT scan
25. Complication(s) of intracerebral abscess include:
A. Dementia
B. Epilepsy
C. Neurological deficits
D. Recurrent headaches
E. Meningitis

26. Idiopathic trigeminal neuralgia:


A. Is a sudden, sharp, transient, paroxysmal pain limited to the
distribution of one or more branches of the trigeminal nerve
B. Is more common in middle-aged men
C. May be triggered by smiling, talking, eating or light touching of
the face
D. Causes diminished corneal reflex
E. Causes wasting and weakness of the masticatory muscles
27. Drugs used in the management of idiopathic trigeminal neuralgia
include:
A. Phenylbutazone
B. Carbamazepine

7
C. Chlorpromazine
D. Phenytoin sodjum
E. Mephenesin carbamate
28. Methods used in the management of idiopathic trigeminal neuralgia
include:
A. Xylocaine block
B. Alcohol block
C. Radio-frequency coagulation
D. Infra-orbital nerve avulsion
E. Section of the trigeminal nerve
29. The commonest intracranial tumour is:
A. Meningioma
B. Metastases
C. Glioma
D. Neurilemmoma
E. Pituitary adenoma

30. The usual childhood C.N.S. tumours include:


A. Medulloblastoma
B. Meningioma
C. Ependymoma
D. Pituitary adenoma
E. Choroid plexus papilloma
31. Gliomas include:
A. Ependymoma
B. Astrocytoma
C. Haemangioblastoma
D. Pinealoma

8
E. Medulloblastoma
32. Astrocytoma:
A. Arises from glial cells
B. Is the commonest C.N.S. primary tumour
C. In adults is not as frequent in the cerebral hemispheres as other
tumours
D. Of the cerebellum occurs more commonly in children
E. Of the cerebellum is usually of grade 3 or 4
33. Medulloblastoma:
A. Is more prevalent in the elderly
B. Usually involves the vermis of the cerebellum
C. Is slow-growing
D. May spread through the subarachnoid space
E. Is highly cellular with small cells forming incomplete or pseudo-
rosettes
34. Meningioma:
A. Arises from the endothelial cells of the arachnoid villi
B. Is always solitary
C. Is not well-encapsultated
D. May develop as a carpet-like tumour over the inner surface of the
dura
E. Can invade the brain, destroy the bone, and metastasize to distant
organs

35. Pituitary tumours:


A. Are common in West Africa
B. May arise from t he anterior or posterior lobe
C. May result from stimulation by hypothalamic secretions
D. Are equally common in all age groups
E. Are more commonly acidophilic or basophilic

9
36. Basophilic adenoma:
A. Nearly always secretes ACTH to cause Cushing’s syndrome
B. Is more common in females
C. Is more common in young adults
D. Enlarges the sella turcica
E. Affects the optic chiasma
37. Tumours of development origin include:
A. Glomus jugulare
B. Craniopharyngioma
C. Chordoma
D. Chromophobe adenoma
E. Paraphyseal cyst
38. Intracranial dermoid cyst:
A. Is usually found in the anterior cranial fossa
B. Is spherical and well-encapsulated
C. Contains sebaceous material and hair
D. May communicate with a dermal sinus
E. May be the cause of granulomatous meningitis

39. Headache in intracranial tumours:


A. May be localized or generalized and intermittent or constant
B. If occipital or subocccipital is suggestive of a supratentorial tumour
C. May be “bursting” or “throbbing” and decreases considerably with
coughing or sneezing
D. Is often worse on rising in the morning due to a rise in arterial carbon
dioxide level during sleep

10
E. Is probably due to displacement of the cranial nerves and meninges
40. In intracranial tumours:
A. There may be vomiting which is worse in the evening
B. Vomiting is usually associated with abdominal discomfort
C. Vomiting is more common in posterior fossa neoplasms
D. A seizure may be the initial symptoms
E. Insomnia may be an early symptom
41. Clinical features of intracranial tumours include:
A. Impairment of judgement, lack of insight
B. Enlarging head in children under 5 years
C. Papilloedema
D. Tachycardia and elevation of the blood pressure
E Alertness
42. Localizing signs in frontal lobe tumours include:
A. Progressive intellectual impairment accompanied by loss of critical
judgement and memory
B. Loss of two point discrimination
C. Urgency of micturition followed by incontinence
D. Contralateral homonymous hemianopia
E. Bilateral spastic paraparesis

43. Clinical features of parietal lobe tumours include:


A. Pathological disinhibition characterized by frivolousness and
superficial
joking
B. Astereognosis (inability to identify size, shape and texture of common
objects)

11
C. Acalculia (inability to do simple arithmetic)
D. Progressive deafness and tinnitus
E. Inability to find one’s way in familiar surroundings
44. Clinical features of occipital lobe tumours include:
A. Inability to recognize colours
B. Ipsilateral homonymous hemianopia
C. Inability to recall the appearance of familiar objects
D. Dysphasia
E. Auditory hallucinations
45. Clinical features of temporal lobe tumours include:
A. Contralateral homonymous hemianopia
B. Dysphasia
C. Hemiparesis
D. Hallucination of unpleasant taste or smell followed by cloudiness
of consciousness
E. Narcolepsy

46. A 9-years old boy has been having paroxysmal headache and
intermittent
“swelling” of his head. He also has polyuria without glycosuria
and
narcolepsy. He has noticed enlargement of his penis, hair over the

12
chest,
pubis and face.

The most likely clinical diagnosis is a tumour in the:


A. Fourth ventricle
B. Pituitary
C. Cerebellum
D. Third ventricle
E. Paracentral lobule of the frontal lobe
47. He is probably suffering from a tumour involving the:
A. Pituitary
B. Midbrain
C. Optic chiasma
D. Medulla
E. Occiput
48. Symptoms of a tumour in the cerebello-pontine angle include:
A. Progressive deafness and tinnitus
B. Unsteadiness of gait
C. Dysarthria
D. Weakness of the contralateral limbs
E. Facial numbness or pain

49. Headache, nuchal pain and rigidity, tremor during performance of


voluntary

13
movements and past pointing on reaching an intended target for
two months
suggest a tumor of the:
A. Cerebellar hemisphere
B. Occipital lobe
C. Parietal lobe
D. Temporal lobe
E. The medulla
50. Clinical features of chromophobe adenoma of the pituitary include:
A. Loss of libido
B. Visual loss
C. Optic atrophy
D. Parkinsonism
E. Loss of 2-point discrimination
51. Acromegaly is caused by:
A. Craniopharyngioma
B. Chromophobe adenoma of the pituitary
C. Basophilic adenoma of the pituitary
D. Temporal lobe tumour
E. Acidophilic adenoma of the pituitary

52. Headache, loss of upward gaze, ptosis, and dilatation of the pupils

14
suggest a tumour of the:
A. Mid-brain
B. Pineal gland
C. Pons
D. Fourth ventricle
E. Cerebellar hemisphere
53. New bone formation (hyperostosis) in a plain X-ray of the skull is
suggestive of:
 Oligo-dendroglioma
B. Astrocytoma
C. Meningioma
D. Craniopharyngioma
E. Metastases from carcinoma of the breast
54. A plain X-ray of the skull in raised intracranial pressure may show:
A. Enlargement and ballooning of the sella
B. Sclerosis of the anterior and posterior clinoid processes
C. Splaying of sutures
D. Diminished vascular markings
E. Osteolysis of the inner table
55. Features suggestive of an intracranial tumour in a plain X-ray of
the
skull include:
 Abnormal intracranial calcification
 Abnormal vascular markings
 Displacement of the pineal gland of more than 3mm
 Narrowing of exit foramina of cranial nerves and vessels
 New bone formation

15
56. Very useful investigations in suspected intracranial tumours
include:
A. E.E.G
B. Angiography
C. Cat-scan
D. Ventriculography
E. Plain X-ray of the chest
57. The following may have clinical features like those of an intracranial
tumour:
A. Cerebrovascular accident
B. Chronic subdural haematoma
C. Chronic intracerebral abscess
D. Uraemia
E. Dementia
58. Radiotherapy only is used in the treatment of:
A. Astrocytoma
B. Meningioma
C. Oligodendroglioma
D. Medulloblastoma
E. Ependymoma
59. Radiotherapy is given after surgical extirpation of:
A. Craniopharyngioma
B. Pituitary adenoma
C. Choroid plexus papilloma
D. Pinealoma
E. Neurilemmoma

16
60. Gliomas respond to:
A. CCNU
B. Vincristine
C. BCNU
D. Mithramycin
E. Procarbazine
61. The commonest benign tumour of the skull is:
A. Chondroma
B. Fibroma
C. Osteoma
D. Osteoclastoma
E. Haemangioma
62. A one-year old baby has a spherical swelling over the anterior
fontanelle.
It is soft, fluctuant, non-tender, and does not enlarge when the
baby cries.
It appears to be loosely attached to the overlying aponeurosis and
underlying
pericranium.

The most likely clinical diagnosis is:


A. Meningocele
B. Encephalocele
C. Lipoma
D. Subgaceal cyst
E. Dermoid cyst

17
63. In arterio-venous malformations of the scalp:
A. The arteries and veins are abnormal in caliber, length and number
B. Intervening capillaries are dilated and tortuous
C. The outer table of the skull is always indented
D. There may be an intracranial communication
E. There is no increase in size with time
64. Subgaleal haematoma:
A. Occurs only in the newborn
B. May be caused by vacuum extraction or accident
C. Lies between the pericranium and aponeurosis and causes pain,
tenderness, fever and toxicity
D. Spreads diffusely in the scalp and is not localized
E. Is treated by incision and drainage
65. A vascular, protuberant, non-tender, firm, swelling about 5 cm
wide with
indefinite edge in the occipital region of the scalp in a 45 year old
woman
may be:
A. Turban tumour
B. Malignant angioendothelioma
C. Haemangiopericytoma
D. Epithelioma
E. Fibrosarcoma

18
66. A 55-year old man complains of right sciatic pain of 2 months
duration. The
hamstrings and calf muscles of both sides are wasted and the
knee jerks are increased but the ankle jerks are absent. There is
no fever or any other abnormality.

You will suspect:


 Prolapse of an intervertebral disc
 Tuberculous spondylitis
 Spinal metastases
 Tumour of the cauda equina
 Tumour of the conus medullaris
67. The commonest spinal tumour is:
A. Astrocytoma
B. Meningioma
C. Ependymoma
D. Neurilelmoma
E. Metastasis
68. The location of a spinal tumour can be most accurately
determined by:
A. Clinical examination
B. Plain X-rays of the spine
C. Myelography
D. Angiography
E. MRI
69. Cerebral metastases commonly originate from a primary in the:
A. Bladder

19
B. Kidney
C. Lung
D. Liver
E. Thyroid

70. The commonest primary source of cerebral metastasis in women


in West
Africa is the:
A. Lung
 Ovary
 Stomach
 Thyroid
 Breast
71. Symptoms of spinal cord tumours may include:
A. Reflex penile erection and ejaculation
B. Faecal incontinence or diarrhoea
C. Weakness or stiffness of muscles or ataxia
D. Girdle pain at the level of the tumour
E. Numbness and a sense of walking on cotton wool
72. Intracranial aneurysms:
A. Are all situated on the circle of Willis
B. If saccular, are round or pear-shaped, about 6-15 mm, and without
medial muscle
C. If fusiform are dangerous as they tend to rupture
D. Are commonly mycotic in type
E. Are associated with atherosclerosis and hypertension
73 Complications of intracranial aneurysms include:
A. Epilepsy
B. Dementia

20
C. Subarachnoid haemorrhage
D. Ophthalmoplegia
E. Extradural haemorrage

74. In subarachnoid haemorrhage:


A. There is sudden sharp headache which is initially severe on one
side of the head but becomes generalized
B. Vomiting is rare
C. Photophobia, neck stiffness and positive Kernig’s sign occur
D. Focal neurological signs are unusual and loss of consciousness
is transient
E. The cause is always an intracranial aneurysm
75. Suspected subarachnoid haemorrhage is confirmed by:
A. Angiography
B. Lumbar puncture
C. Cat-scan
D. E.E.G.
E. Burr holes
76. Clinical features of carotico-cavernous fistula include:
A. Pulsating unilateral exophthalmos
B. Chemosis
C. Disfigurement
D. Gradual onset
E. Blindness
77. In the management of spinal tumours:
A. Plain X-rays are of proven value
B. CSF analysis may be diagnostic

21
C. MRI is the investigation of choice
D. Chemotherapy is never indicated
E. High dose corticosteroids are useful

78. A subgaleal cyst of the scalp:


A. Is also called “Adeloye Disease”
B. Represents an inclusion dermoid cyst
C. Has a transmitted cough impulse
D. Occurs over the anterior fontanelle
E. Usually contains CSF like fluid
79. The following are true of bacterial brain abscess:
A. A lumbar puncture is absolutely contra indicated
B. If due to haematogenous spread it is located at the grey/white
matter junction
C. Co-trimoxazole is an effective antibiotic to use
D. Cultures are almost always positive for the offending organism
E. It is essential to irradicate the source if known
80. Concerning radicular lesions:
A. A C5 root lesion may cause pain in the neck
B. C C8 radiculopathy may cause wasting of the thenar and
hypothenar
muscles
C. A C7 root lesion will result in absence of the biceps reflex
D. L2 root lesion will give you a sensory level at the level of the
umbilicus
E. Affection of the L5 nerve root will cause weakness of the extensor
hallucis longus (EHL)
81. Of nerve injuries:

22
A. A neuropraxia refers to “concussion” of the nerve i.e. transient event
B. Neurotmesis refers to transaction of the nerve
C. Neuropraxia usually presents with muscle atrophy
D. In axonometsis both the myelin and axon are lost
E. Recovery is possible in neurotmesis

82. The causes of sciatica include:


A. Lumbar spondylosis
B. Trauma
C. Stroke
D. Herniated intervertebral disc is the commonest cause
E. Diabetic neuropathy
83. The following may be true of brain tumours:
A. Meningiomas may develop several years after head trauma
B. Exposure to radiation can cause gliomas
C. Diet may be may implicated in the causation
D. Most cases are genetic or have a familial history
E. Cellular phones maybe the cause of acoustic neuroma
84. Symptoms and signs of raised intracranial pressure include:
A. Lack of sleep
B. Early morning vomitting
C. Papilloedema
D. Yawning
E. Lateral rectus palsy
85. Which is correct concerning post traumatic epilepsy (PTE)?
A. Late PTE occurs more than 1 week after injury
B. Risk factor is higher with penetrating injuries
C. There is effective intervention to prevent its development
D. Selective damage to vulnerable brain regions such as the

23
hippocampus is important in the pathogenesis
E. CT scan is the study of choice

CHAPTER 52
NEUROSURGERY
                 ANSWERS

1. C 38. B, C, D, E 75. B
2. A, B, D 39. A, D 76. B, C, E

3. C, D, E 40. C, D 77. A, B, C, E

4. A, B, D 41. A, B, C 78. A, B, D, E

5. B, D, E 42. A, C 79. A, B, E

6. A, C, E 43. B, C, E 80. A, B, E

7. B, D, E 44. A, C 81. A, B

8. E 45. A, B, C, D 82. A, B, C

9. B, C, D, E 46. D 83. A, B

10. B, C, E 47. A, C 84. B, C, E

11. A, E 48. A, B, C, E 85. A, B, D

12. A, E 49. A

13. A, B, D, E 50. A, B, C

14. A, D, E 51. E

15. A, D 52. A, B
16. C, D, E 53. C

17. C 54. A, C
18. A, B, C, D 55. A, B, C, E
19. C 56. A, B, C, D, E

24
20. C 57. B, C
21. A 58. D
22. A, B, C, D, E 59. A, B, D
23. A, B, D 60. A, B, C, D, E

24. E 61. C
25. B, C, E 62. D
26. A, C 63. A, D
27. B, D, E 64. B, D
28. B, C, D 65. D, E
29. C 66. C, D
30. A, C, E 67. E
31. A, B, E 68. C, E
32. B, D 69. B, C, E
33. B, D, E 70. E
34. A, D 71. A, C, D, E
35. A, C 72. B, E
36. B 73. A, C, D
37. B, C, E 74. C

25

You might also like